You are on page 1of 383

Downloaded From : www.EasyEngineering.

net

ww
w.E
a syE
ngi
nee
rin
g.n
et

**Note: Other Websites/Blogs Owners Please do not Copy (or) Republish


this Materials, Students & Graduates if You Find the Same Materials with
EasyEngineering.net Watermarks or Logo, Kindly report us to
easyengineeringnet@gmail.com

Downloaded From : www.EasyEngineering.net


Downloaded From : www.EasyEngineering.net

101 SPEED TEST

SBI
ww
w.E
asy
En
gin
BANK CLERK e eri
n g.nExams
with 5 Practice Sets for Preliminary & Main
et

Downloaded From : www.EasyEngineering.net


Downloaded From : www.EasyEngineering.net

• Head Office : B-32, Shivalik Main Road, Malviya Nagar, New Delhi-110017

• Sales Office : B-48, Shivalik Main Road, Malviya Nagar, New Delhi-110017
Tel. : 011-26691021 / 26691713

ww
w.E
Typeset by Disha DTP Team

asy
En
gin
eer
ing
.ne
DISHA PUBLICATION
t
ALL RIGHTS RESERVED

© Copyright Publisher
No part of this publication may be reproduced in any form without prior permission of the publisher. The author and the
publisher do not take any legal responsibility for any errors or misrepresentations that might have crept in. We have tried
and made our best efforts to provide accurate up-to-date information in this book.

For further information about books from DISHA,


Log on to www.dishapublication.com or email to info@dishapublication.co.in

[ ii ]

Downloaded From : www.EasyEngineering.net


Downloaded From : www.EasyEngineering.net

Preface
101 Speed Tests for SBI Bank Clerk Exam
101 Speed Tests for SBI Bank Clerk Exam is revised and updated edition on the basis of Current Exam Patten. It contains all the
IMPORTANT CONCEPTS which are required to crack this exam. The concepts are covered in the form of 101 SPEED TESTS.
No matter where you PREPARE from – a coaching or any textbook/ Guide — 101 SPEED TESTS provides you the right
ASSESSMENT on each topic. Your performance provides you the right cues to IMPROVE your concepts so as to perform better
in the final examination.
It is to be noted here that these are not mere tests but act as a checklist of student’s learning and ability to apply concepts to
different problems.

ww
The book is based on the concept of TRP – Test, Revise and Practice. It aims at improving your SPEED followed by STRIKE RATE
which will eventually lead to improving your SCORE.

• w.E
How is this product different?
1st unique product with 101 speed tests.

asy
Each test is based on small topics which are most important for the SBI Bank Clerk exam. Each test contains around 25-
30 MCQs on the latest pattern of the exam.

En
The whole syllabus has been divided into 5 sections which are further distributed into 96 topics.
1. QUANTITATIVE APTITUDE is distributed into 28 topics.
2.
3.
REASONING is distributed into 24 topics.
ENGLISH LANGUAGE is distributed into 19 topics.
gin
4. COMPUTER KNOWLEDGE/ MARKETING APTITUDE is distributed into 9 topics.
eer

5. GENERAL AWARENESS is distributed into 16 topics.
ing
In the end of each section a Sectional Test is provided so as to sum up the whole section.

.ne
Finally at the end 5 FULL TESTS-3 Tests for Prelim and 2 Tests for Mains are provided so as to give the candidates the real
feel of the final exam.


In all, the book contains 3400+ Quality MCQ’s in the form of 101 tests.
Solutions to each of the 101 tests are provided at the end of the book.
t
• The book provides Separate Tests. The book comes with perforation such that each test can be torn out of the book.
• Separate Time Limit, Maximum Marks, Cut-off, Qualifying Score is provided for each test.
• The book also provides a separate sheet, SCORE TRACKER where you can keep a record of your scores and performance.
• It is advised that the students should take each test very seriously and must attempt only after they have prepared that
topic.
• Once taken a test the candidates must spend time in analysing their performance which will provide you the right cues
to IMPROVE the concepts so as to perform better in the final examination.
It is our strong belief that if an aspirant works hard on the cues provided through each of the tests he/ she can improve
his/ her learning and finally the SCORE by at least 20%.
DISHA PUBLICATION

[ iii ]

Downloaded From : www.EasyEngineering.net


Downloaded From : www.EasyEngineering.net

Syllabus
REASONING QUANTITATIVE APTITUDE
1. ANALOGY 1-2 29. NUMBER SYSTEM - I 59-60

2. CLASSIFICATION 3-4 30. NUMBER SYSTEM - II 61-62


31. LCM, HCF AND FRACTION 63-64
3. SERIES 5-6
32. SQUARE, CUBE, INDICES & SURDS 65-66
4. ALPHABET TEST 7-8
33. SIMPLIFICATION 67-68
5. CODING - DECODING 9-10
34. AVERAGE 69-70
6. BLOOD RELATION 11-12
35. PERCENTAGE 71-72
7.
8.
ww
DIRECTION AND DISTANCE
TIME, SEQUENCE & RANKING TEST
13-14
15-16
36.
37.
RATIO & PROPORTION
ALLIGATION AND MIXTURE
73-74
75-76
9.
10.
w.E
SITTING ARRANGEMENT
ANALYTICAL PUZZLE - I
17-18
19-20
38.
39.
PROFIT, LOSS & DISCOUNT
SIMPLE INTEREST
77-78
79-80
11. ANALYTICAL PUZZLE - II
asy 21-22 40. COMPOUND INTEREST 81-82
12.
13.
VENN DIAGRAMS
SYLLOGISM - I En
23-24
25-26
41.
42.
DISTANCE, SPEED AND TIME
TIME & WORK / PIPE & CISTERN
83-84
85-86

14. SYLLOGISM - II 27-28 43.


gin
PROBLEM BASED ON AGES 87-88

15. SYMBOLS & CODES 29-30


44.
45. eer
PERMUTATION & COMBINATION
PROBABILITY
89-90
91-92
16.
17.
ALPHA NUMERIC SEQUENCE PUZZLE
INPUT-OUTPUT
31-32
33-34
46. AREA AND PERIMETER
ing 93-94

18. MATHEMATICAL OPERATIONS 35-36


47.
48.
VOLUME AND SURFACE AREA
GEOMETRY .ne 95-96
97-98
19.
20.
21.
CLOCK AND CALENDAR
DATA SUFFICIENCY
STATEMENT & CONCLUSION
37-38
39-40
49.
50.
NUMBER SERIES - I
NUMBER SERIES - II
t99-100
101-102
51. DATA INTERPRETATION 103-104
(MATHEMATICAL) 41-42
52. SECTION TEST :
22. STATEMENT & CONCLUSION QUANTITATIVE APTITUDE 105-106
(LOGICAL) 43-44
23. PASSAGE BASED CONCLUSION-1 45-46 ENGLISH LANGUAGE
53. READING COMPREHENSION - I 107-110
24. PASSAGE BASED CONCLUSION-2 47-48
54. READING COMPREHENSION - II 111-114
25. CUBE & DICE 49-50
55. READING COMPREHENSION - III 115-118
26. NON-VERBAL REASONING - I 51-52
56. SYNONYMS 119-120
27. NON-VERBAL REASONING - II 53-54
57. ANTONYMS 121-122
28. SECTION TEST : REASONING 55-58
58. SENTENCE COMPLETION - I 123-124
[ iv ]

Downloaded From : www.EasyEngineering.net


Downloaded From : www.EasyEngineering.net

59. SENTENCE COMPLETION - II 125-126 GENERAL AWARENESS


60. SIMILAR SUBSTITUTION 127-128
81. HISTORY OF BANKING AND
61. CORRECT USAGE OF
ITS DEVELOPMENT 171-172
PREPOSITION 129-130
62. SENTENCE IMPROVEMENT 131-132 82. RBI & ITS MONETARY POLICIES 173-174

63. SPOTTING THE ERRORS - I 133-134 83. BANKING PRODUCT & SERVICES 175-176
64. SPOTTING THE ERRORS - II 135-136 84. BANKING TERM/ TERMINOLOGY 177-178
65. SPELLING TEST 137-138
85. MICRO FINANCE & ECONOMICS 179-180
66. ONE WORD SUBSTITUTION 139-140
86. FOREIGN TRADE 181-182
67. PARA JUMBLES 141-142
68.
69.
ww
IDIOMS AND PHRASES
CLOZE TEST - I
143-144
145-146
87. SOCIO-ECO-POLITICAL
ENVIRONMENT OF INDIA 183-184

70. CLOZE TEST - II w.E 147-148 88. APPOINTMENT/ ELECTION/


RESIGNATION 185-186
71. SECTION TEST :
ENGLISH LANGUAGE
asy 149-152 89. EVENTS/ ORGANISATION/

COMPUTER KNOWLEDGE/ MARKETING APTITUDE


En 90.
SUMMITS

AWARDS AND HONOURS


187-188

189-190
72. COMPUTER FUNDAMENTAL /
BINARY SYSTEM/ OPERATING SYSTEM 153-154
gin
91. BOOKS AND AUTHORS 191-192
73. MS OFFICE/ COMMANDS and
SHORTCUT KEYS
155-156
92.
eer
SPORTS AND GAMES 193-194
74.
75.
SOFTWARES/ PROGRAMMING
INTERNET, NETWORKING AND
157-158
159-160
93.
ing
SCIENCE AND TECHNOLOGY 195-196

76.
COMPUTER ABBREVIATIONS
FUNDAMENTALS OF MARKETING, 161-162
94. CURRENT BANKING
.ne 197-198

77.
PRODUCT AND BRANDING
MARKET SITUATIONS BASED ON PRICE,
DISTRIBUTION, PROMOTION AND
95.

96.
CURRENT AFFAIRS

SECTION TEST :
General & Financial Awareness
t
199-200

201-204
ADVERTISING 163-164
97. PRELIM FULL TEST - 1 205-212
78. MARKET SEGMENTATION,
TARGETING AND POSITIONING 165-166 98. PRELIM FULL TEST - 2 213-220
79. MODERN MARKETING /
MARKETING IN BANKING INDUSTRY 167-168 99. PRELIM FULL TEST - 3 221-228

80. SECTION TEST : 100. Full MAIN TEST - 4 229-242


COMPUTER KNOWLEDGE/
MARKETING APTITUDE 169-170 101. Full MAIN TEST - 5 243-256

101 SPEED TEST - SOLUTIONS 1-116

[v]

Downloaded From : www.EasyEngineering.net


Downloaded From : www.EasyEngineering.net


ww
w.E
asy
En
gin
eer
ing
.ne
t

[ vi ]

Downloaded From : www.EasyEngineering.net


Downloaded From : www.EasyEngineering.net


ww
w.E
asy
En
gin
eer
ing
.ne
t

[ vii ]

Downloaded From : www.EasyEngineering.net


Downloaded From : www.EasyEngineering.net

– –

30 15
15
15
30 15
15
15

ww 15
15

w.E 25 30

25

asy
En
25 gin
eer
ing
.ne
15
15
25
23
20 12 15
20
t
135 150
135 150
135 150
135 150
135 150

[ viii ]

Downloaded From : www.EasyEngineering.net


Downloaded From : www.EasyEngineering.net

Analogy 1
Max. Marks : 30 No. of Qs. 30 Time : 20 min. Date : ........./......../. ..............

1. A 'Square' is related to 'Cube' in the same way as a 'Circle' is 8. NUMBER : UNBMRE : : GHOST : ?
related to (a) HOGST (b) HOGTS (c) HGOST
(a) Sphere (b) Circumference (c) Diameter
(d) HGSOT (e) HGOTS
(d) Area (e) None of these
9. DRIVEN : EIDRVN : : BEGUM : ?
2.
ww
In a certain code BRIGHT is written as JSCSGG. How is
JOINED written in that code?
(a) HNIEFO (b) JPKEFO (c) JPKMDC
(a) EUBGM
(c) BGMEU
(b) MGBEU
(d) UEBGM

3.
(d) KPJCDM
w.E (e) None of these
'34' is related to '12' in the same way as '59' is related to 10.
(e) BGMUE
PRLN : X Z T V : : J L F H : ?

4.
(a) 45
(d) 38
(b) 14
(e) 47
asy
(c) 42

'Mustard' is related to 'Seed' in the same way as 'Carrot' is


(a) RTNP
(d) NTRP
(b) NPRT
(e) RPNT
(c) NRPT

related to
En 11. XWV : UTS : : LKJ : ... ?
(a) IHG (b) JKL (c) STU
(a) Fruit
(d) Root
(b) Stem
(e) None of these
(c) Flower

DIRECTIONS (Q. 5 - 24) : For each of the following questions


gin
12.
(d) MNO
QYGO : SAIQ : : UCKS : ?
(e) KIG

there is some relationship between the two terms to the left of ::


and the same relationship obtains between the two terms to its
(a) WDMV
eer (b) VFNU (c) WDLU

right. In each of these questions the fourth term is missing. This


term is one of the alternatives among the five terms given below.
Find out this term.
13.
(d) VEMU
YAWC : UESG : : QIOK : ?
ing
(e) WEMU

5. ADE : FGJ :: KNO : ?


(a) PQR (b) TPR (c) PQT
(a) MINC
(d) MMKO
(b) MIKE
(e) LIME
.ne (c) KOME

6.
(d) RQP (e) PRS
DGPGJ : MPQPS : : KNENQ : ....?
(a) RUFUX (b) RFUFX (c) RXUXF
14. CFIL : PSVY : : HKNQ : ?
(a) NPSV
(d) MOQT
(b) LPSY
(e) MPSU
(c) LORUt
(d) RFUFX (e) None of these 15. 122 : 170 : : 290 : ?
7. UTS : EDC : : WVU : ? (a) 299 (b) 315 (c) 332
(a) XWV (b) WXY (c) SJM (d) 344 (e) 362
(d) RPO (e) SRP

1. a b c d e 2. a b c d e 3. a b c d e 4. a b c d e 5. a b c d e
RESPONSE 6. a b c d e 7. a b c d e 8. a b c d e 9. a b c d e 10. a b c d e
GRID 11. a b c d e 12. a b c d e 13. a b c d e 14. a b c d e 15. a b c d e

Downloaded From : www.EasyEngineering.net


Downloaded From : www.EasyEngineering.net

2 SPEED TEST 1
16. 42 : 56 : : 110 : ? 25. Which of the following pairs of words have the same
(a) 132 (b) 136 (c) 144
relationship as FAN : HEAT?
(d) 148 (e) 156
17. 12 : 20 : : 30 : ? (a) Water : Drink (b) Light : Night
(a) 15 (b) 32 (c) 35 (c) Teach : Student (d) Air : Breathe
(d) 42 (e) 48
(e) Food : Hunger
18. 3 : 10 : : 08 : ?
(a) 10 (b) 13 (c) 17 26. A disease would always necessarily have
(d) 14 (e) 16 (a) Medicine (b) Bacteria (c) Cause
19. 08 : 28 : 15 : ?
(d) Cure (e) Fever
(a) 63 (b) 126 (c) 65
(d) 124 (e) 26 27. 'Army' is related to 'Land' in the same way as 'Navy' is related
20. 08 : 09 : : 64 : ? to _____.
(a) 16 (b) 25 (c) 125
(a) Ships (b) Battle (c) Water
(d) 32 (e) 20
21. 6 : 24 : : 5 : ? (d) Admiral (e) Defence

22.
(a) 23
(d) 20ww
6 : 35 : : 11 : ?
(b) 22
(e) 19
(c) 26 28. A 'Tumbler' is related to 'Empty' in the same way as a 'Seat' is
related to

23.
(a) 120
(d) 121
3 : 27 : : 4 : ?
w.E (b) 115
(e) 124
(c) 122 (a) Occupied
(d) Sitting
(b) Person
(e) Vacant
(c) Chair

(a) 140
(d) 80
(b) 75
(e) 64 asy
(c) 100
29. Secretly is to openly as silently is to__
(a) scarcely (b) impolitely (c) noisily

24. 3:3
3
8
:: 5:?
En 30.
(d) quietly (e) None of these
Spring is to as coil as ring is to____

(a) 5
5
8
(b) 5
3
8
(c) 5
1
8 gin (a) rope
(d) stretch
(b) loop
(e) None of these
(c) cowl

(d) 2
5
8
(e) 6
1
8 eer
ing
.ne
t

RESPONSE 16. a b c d e 17. a b c d e 18. a b c d e 19. a b c d e 20. a b c d e


21. a b c d e 22. a b c d e 23. a b c d e 24. a b c d e 25. a b c d e
GRID 26. a b c d e 27. a b c d e 28. a b c d e 29. a b c d e 30. a b c d e

Downloaded From : www.EasyEngineering.net


Downloaded From : www.EasyEngineering.net

Classification 2
Max. Marks : 30 No. of Qs. 30 Time : 20 min. Date : ........./......../. ..............
1. Four of the following five are alike in a certain way on the 8. Four of the following five are alike in a certain way and so
basis of their positions in English alphabet and so form a form a group. Which is the one that does not belong to that
group. Which is the one that does not belong to that group. group?
(a) HJG (b) PQN (c) DEB (a) Garlic (b) Ginger (c) Carrot
(d) TUR (e) KLI (d) Radish (e) Brinjal
2.
ww
Four of the following five are alike in a certain way and so
form a group. Which is the one that does not belong to that
group ?
9. Four of the following five are alike in a certain way and so
form a group. Which is the one that does not belong to that
group?

3.
(a) Rose
(d) Marigold w.E (b) Jasmine
(e) Lotus
(c) Hibiscus

Four of the following five pairs of alphas and numerals have


10.
(a) Clutch
(d) Car
(b) Wheel
(e) Gear
(c) Break

Four of the following five are alike in a certain way and so

asy
same relationship between their elements as in the case of
the pair PROBLEM : 2948375 and hence form a group. Which
form a group. Which is the one that does not belong to that
group?
(a) 196 (b) 256 (c) 529
one does not belong to the group?
(a) BORE : 8497 (b) MOEP : 5972
En 11.
(d) 576 (e) 324
Four of the following five are alike in a certain way and so
(c) LBOR : 3849
(e) EROL : 7943
(d) OMEP : 4572
gin form a group. Which is the one that does not belong to that
group?
(a) RPN (b) WSU (c) HDF
4. Four of the following five pairs are alike in a certain way
and hence form a group. Which one does not belong to
that group?
12.
(d) LHJ
eer (e) QMO
Four of the following five are alike in a certain way and so
(a) DONE : NOED
(c) WITH : TIHW
(b) WANT : NATW
(d) JUST : SUTJ
group?
(a) 169 ing
form a group. Which is the one that does not belong to that

(b) 441 (c) 361

5.
(e) HAVE : AVEH
Four of the following five are alike in a certain way and so 13.
(d) 529 (e) 289
.ne
Four of the following five are alike in a certain way and so
form a group. Which is the one that does not belong to that
group?
(a) Hill
(c) River
(b) Valley
(e) Mountain
(c) Dam
form a group. Which is the one that does not belong to that
group?
(a) PM
(d) IF
(b) EB
(e) VY
(c) TQ
t
6. Four of the following five are alike in a certain way and so 14. Four of the following five are alike in a certain way and so
form a group. Which is the one that does not belong to that form a group. Which is the one that does not belong to that
group? group?
(a) 115 (b) 85 (c) 95
(a) 50 (b) 65 (c) 170
(d) 75 (e) 155
(d) 255 (e) 290
15. Four of the following five are alike in a certain way and so
7. Four of the following five are alike in a certain way and so form a group. Which is the one that does not belong to that
form a group. Which is the one that does not belong to that group?
group? (a) 115 (b) 161 (c) 253
(a) 21 (b) 35 (c) 42 (d) 391 (e) 345
(d) 56 (e) 49

1. a b c d e 2. a b c d e 3. a b c d e 4. a b c d e 5. a b c d e
RESPONSE 6. a b c d e 7. a b c d e 8. a b c d e 9. a b c d e 10. a b c d e
GRID 11. a b c d e 12. a b c d e 13. a b c d e 14. a b c d e 15. a b c d e

Downloaded From : www.EasyEngineering.net


Downloaded From : www.EasyEngineering.net

4 SPEED TEST 2
16. Four of the following five are alike in a certain way and so 23. Four of the following five are alike in a certain way and
form a group. Which is the one that does not belong to that hence form a group. Which is the one that does not belong
group? to that group?
(a) OMQ (b) HFJ (c) TPR (a) KP (b) BY (c) DW
(d) TRV (e) VTX (d) HU (e) GT
24. Four of the following five are alike in a certain way and
17. Four of the following five are alike in a certain way based on
hence form a group. Which is the one that does not belong
the English alphabetical series and so form a group. Which
to that group?
is the one that does not belong to that group?
(a) JLNK (b) TVXU (c) ACEB
(a) MLJ (b) WVT (c) OMK (d) PRTQ (e) GJKH
(d) JIG (e) TSQ 25. Four of the following five are alike in a certain way and
18. Four of the following five are alike in a certain way and so hence form a group. Which is the one that does not belong
form a group. Which is the one that does not belong to that to that group?
group? (a) GIJF (b) OQRN (c) KMNL
(a) Diabetes (b) Smallpox (d) UWXT (e) CEFB

ww
(c) Conjunctivitis
(e) Plague
(d) Chickenpox 26. Four of the following five are alike in a certain way and
hence form a group. Which is the one that does not belong
to the group?
19.

w.E
Four of the following five are alike in a certain way and so
form a group. Which is the one that does not belong to that
group?
27.
(a) Break
(d) Divide
(b) Change
(e) Separate
(c) Split

Four of the following five are alike in a certain way and


(a) Mustard
(d) Paddy
(b) Jowar
(e) Bajra asy (c) Wheat hence form a group. Which is the one that does not belong
to the group?
20.

En
Four of the following five are alike in a certain way and so
form a group. Which is the one that does not belong to that
(a) Train
(d) Advocate
(b) Instruct
(e) Coach
(c) Educate

group?
(a) 45 (b) 35 (c) 85
gin
28. Four of the following five are alike in a certain way and
hence form a group. Which is the one that does not belong
to the group?
21.
(d) 25 (e) 75
Four of the following five are alike in a certain way and (a) Extend
(d) Rise eer (b) Higher
(e) Ascend
(c) Upward
hence form a group. Which is the one that does not belong
to that group?
(a) Succeed (b) Victory (c) Triumph
29.
ing
Four of the following five are alike in a certain way and so
form a group. Which is the one that does not belong to that

22.
(d) Compete (e) Win
Four of the following five are alike in a certain way and
group?
(a) Volume (b) Size
.ne (c) Large

hence form a group. Which is the one that does not belong
to that group?
(a) Fair (b) Impartial (c) Indifferent
30.
(d) Shape (e) Weight

t
Four of the following five are alike in a certain way and so
form a group. Which is the one that does not belong to that
group?
(d) Unbiased (e) Just (a) Anxiety (b) Worry (c) Inhibition
(d) Curiosity (e) Weariness

RESPONSE 16. a b c d e 17. a b c d e 18. a b c d e 19. a b c d e 20. a b c d e


21. a b c d e 22. a b c d e 23. a b c d e 24. a b c d e 25. a b c d e
GRID 26. a b c d e 27. a b c d e 28. a b c d e 29. a b c d e 30. a b c d e

Downloaded From : www.EasyEngineering.net


Downloaded From : www.EasyEngineering.net

Series 3
Max. Marks : 30 No. of Qs. 30 Time : 20 min. Date : ........./......../. ..............

DIRECTIONS (Q. 1 - 12) : What should come in place of question 9. 12 18 36 102 360 (?)
mark (?) in the following number series? (a) 1364 (b) 1386 (c) 1384
1. 1050 420 168 67.2 26.88 10.752 ? (d) 1376 (e) None of these
10. 71 78 99 134 183 (?)

2.
ww
(a) 4.3008
(d) 5.6002
0 6 24 60 120 210 ?
(b) 6.5038
(e) None of these
(c) 4.4015
(a) 253
(d) 253
(b) 239
(e) None of these
(c) 246

(a) 343
(d) 295
w.E (b) 280
(e) None of these
(c) 335 11. 342 337.5 328.5 315 297 (?)
(a) 265.5 (b) 274.5 (c) 270
3. 15 19 83 119 631 (?)
(a) 731 (b) 693 asy
(c) 712 12.
(d) 260
161 164 179 242 497 (?)
(e) None of these

4.
(d) 683
19 26 40 68 124 (?)
(e) None of these
En (a) 1540
(d) 1440
(b) 1480
(e) None of these
(c) 1520

(a) 246
(d) 256
(b) 238
(e) None of these
(c) 236
gin
5. 11 10 18 51 200 (?)
eer
DIRECTIONS (Qs. 13 to 17): What will come in place of question
mark (?) in the following number series?
(a) 885
(d) 995
(b) 1025
(e) None of these
(c) 865 13. 12, 30, 56, 90, 132, ?
(a) 178 ing (b) 182 (c) 185
6. 14 24 43 71 108 (?)
(a) 194 ( b) 154 (c) 145 14.
(d) 189
91, 381, 871, 1561, 2451, ?
(e) 196
.ne
7.
(d) 155
144 173 140 169 136 (?)
(a) 157
(e) None of these

( b) 148 (c) 164 15.


(a) 3541
(d) 3716
110, 440, 990, 1760, 2750, ?
(b) 3621
(e) 3772 t
(c) 3681

(d) 132 (e) None of these (a) 3680 (b) 3610 (c) 37820
8. 656 352 200 124 86 (?) (d) 3840 (e) 3960
(a) 67 (b) 59 (c) 62
(d) 57 (e) None of these

1. a b c d e 2. a b c d e 3. a b c d e 4. a b c d e 5. a b c d e
RESPONSE 6. a b c d e 7. a b c d e 8. a b c d e 9. a b c d e 10. a b c d e
GRID 11. a b c d e 12. a b c d e 13. a b c d e 14. a b c d e 15. a b c d e

Downloaded From : www.EasyEngineering.net


Downloaded From : www.EasyEngineering.net

6 SPEED TEST 3
16. 5, 6, 11, 20, 33, 50, ? 24. What will be the next term in ?
(a) 64 (b) 71 (c) 78 DCXW, FEVU, HGTS, ..........
(d) 81 (e) 84 (a) LKPO (b) ABYZ (c) JIRQ
17. 2, 7, 24, 77, 238, 723, ? (d) LMRS (e) None of these
(a) 1948 (b) 1984 (c) 2010 25. ZXVTR....
(d) 2096 (e) 2180 (a) O, K (b) N, M (c) K, S
(d) M, N (e) P, N
DIRECTIONS (Q.18 - 22) :What should come in place of question 26. C, e, G, i, K....
mark (?) in the following number series? (a) O, K (b) m, O (c) k, M
18. 980 484 260 112 50 ? 3.5 (d) M, k (e) O, p
(a) 25 (b) 17 (c) 21 27. m_ _ l m _ l _ m m _ l
(d) 29 (e) None of these (a) mllml (b) mlmll (c) llmlm
19.

ww
1015 508 255 129 66.5 ? 20.875
(a) 34.50
(d) 35.75
(b) 35
(e) None of these
(c) 35.30
(d) mmlml (e) llmll

20. 4.5 18 2.25 ? 1.6875 33.75


(a) 27
w.E
(b) 25.5 (c) 36
28. What should come next in the number series given below ?
1121231234123451234 56123456
(a) 5 (b) 2

21.
(d) 40 (e) None of these
59.76 58.66 56.46 52.06 ? 25.66 asy (c) 8
(e) None of these
(d) 1

(a) 48.08
(d) 43.26
(b) 46.53
(e) None of these
(c) 43.46
En 29. What should come next in the following letter series?
ABC DPQRSA BCDE PQ RSTABCDEFPQ RST
22. 36 157 301 470 ? 891
(a) 646 (b) 695 (c) 639 gin (a)
(c)
A
U
(b) V
(d) W
(d) 669 (e) None of these
(e)
eer
None of these

DIRECTIONS (Qs. 23 to 27) : Which one of the letters when


sequentially placed at the gaps in the given letter series shall
30.
series ?
ing
What will come in place of question mark (?) in the following

complete it?
NDP, QWB, ZFR, ?
(a) SVJ (b) AFS .ne
23. a–ca–bc–bcc–bca
(a) b b a b
(d) b b a a
(b) b a b a
(e) None of these
(c) a a b b
(c) IVS
(e) None of these
(d) SFA
t

16. a b c d e 17. a b c d e 18. a b c d e 19. a b c d e 20. a b c d e


RESPONSE
21. a b c d e 22. a b c d e 23. a b c d e 24. a b c d e 25. a b c d e
GRID 26. a b c d e 27. a b c d e 28. a b c d e 29. a b c d e 30. a b c d e

Downloaded From : www.EasyEngineering.net


Downloaded From : www.EasyEngineering.net

Alphabet Test 4
Max. Marks : 30 No. of Qs. 30 Time : 20 min. Date : ........./......../. ..............
1. How many such pairs of letters are there in the word 9. If the letters of the word ARROGANCE are interchanged,
EXCURSION, each of which has as many letters between first with fifth, second with sixth, third with seventh, fourth
them in the word as they have in the English alphabet? with eighth and the position of the ninth remains unchanged,
(a) None (b) One (c) Two then what will the new arrangement of letters be?
(d) Three (e) More than three (a) GANACRROE (b) GANCRAROE"
2.
ww
How many such pairs of letters are there in the word GOLDEN,
each of which has as many letters between them in the word
as in the English alphabet? 10.
(c) GNACORRAE
(e) None of these
(d) GANCARROE

If all the letters in the word 'PRINCE' are rearranged in

3.
(a) None
(d) Three w.E (b) One
(e) More than three
(c) Two

How many such pairs of letters are there in the word STRIVE
alphabetical order, then how many letter(s) will remain
unchanged?
(a) None (b) One (c) Two
as in the English alphabet ?
(a) One (b) Two
asy
each of which has as many letters between them in the word

(c) Three 11.


(d) Three (e) More than three
How many meaningful English words can be formed with

4.
(d) Four (e) None of these
How many meaningful English words can be formed with the En ESRO using each letter only once in each word?
(a) None (b) One (c) Two
letters 'ATN' using each letter only once in each word?
(l) One
(d) Four
(b) Two
(e) More than four
(c) Three
gin
12.
(d) Three (e) More than three
How many such pairs of letters are there in the word
CONSTABLE, each of which has as many letters between
5. The serial order of how many letters in the word CLIENT will
not differ than their serial order in the arrangement where the (a) None eer
them in the word as in the English alphabet?
(b) One (c) Two
letters of the word are arranged alphabetically?
(a) Four
(d) Two
(b) One
(e) None of these
(c) Three 13.
(d) Three
ing
(e) More than three
How many such pairs of letters are there in the word

6. How many meaningful English words can be made with the


letters ENAL using each letter only once in each word? .ne
PHYSICAL, each of which has as many letters between them
in the word as they have in the English alphabet?
(a) None (b) One (c) Two

7.
(a) One
(d) Four
(b) Two
(e) More than four
(c) Three

How many three - letter meaningful words can be formed


from the word TEAR beginning with 'A' without repeating
14.
(d) Three (e) More than three
How many pairs of letters are there in the word 'SHIFTED' t
each of which has as many letters between its two letters as
any letter within that word? there are between them in the English alphabet?
(a) One (b) Three (c) Five (a) None (c) One (c) Two
(d) Two (e) None of these (d) Three (e) None of these
8. If all the letters in the word ARGUMENT are rearranged in 15. How many meaningful English words can be formed by using
alphabetical order and substituted by the letter immediately any two letters of the word 'GOT'?
following it in the English alphabet, what will be the new (a) Three (b) Two (c) One
arrangement of letters? (d) More than three (e) None of these
(a) BFHNOSUV (b) BFHONSWV
(c) BFHNOUSV (d) BFHNOQUV
(e) None of these

1. a b c d e 2. a b c d e 3. a b c d e 4. a b c d e 5. a b c d e
RESPONSE 6. a b c d e 7. a b c d e 8. a b c d e 9. a b c d e 10. a b c d e
GRID 11. a b c d e 12. a b c d e 13. a b c d e 14. a b c d e 15. a b c d e

Downloaded From : www.EasyEngineering.net


Downloaded From : www.EasyEngineering.net

8 SPEED TEST 4
16. How many three-letter meaningful English words can be 23. If each vowel of the word WEBPAGE is substituted with the
formed from the word NOTE beginning with T and without next letter of the English alphabet, and each consonant is
repeating any letter within that word? substituted with the letter preceding it, which of the following
(a) Three (b) One (c) Two letters will appear thrice?
(d) None (e) None of these (a) G (b) F (c) Q
17. If all the letters in the word MERCIFUL are rearranged in (d) V (e) None of these
alphabetical order and substituted by the alphabet preceding 24. How many meaningful four letter English words can be
them in the English alphabet, what will be the new formed with the letters TPSI using each letter only once in
arrangement of letters? each word?
(a) BDFIEKLQT (b) BDEHKLQT (a) One (b) Two (c) Three
(c) BDEHLKQT (d) BDEJMLQT (d) Four (e) More than four
(e) None of these 25. How many meaningful five-letter words can be formed with
18. If it is possible to make only one meaningful word, from the the letters SLIKL using each letter only once ?
first, the third, the fifth and the eighth letters of word (a) One (b) Two (c) Three
ENTERPRISE using each letter only once, first letter of the (d) More than three (e) None
word is your answer. If more than one such word can be 26. If each vowel in the word HABITUAL is changed to the next

ww
made your answer is `X' and if no such word can be made,
your answer is Y'.
letter in the English alphabet and each consonant is changed
to the previous letter in the English alphabet, which of the
(a) P
(d) X
w.E (b) S
(e) Y
(c) T

19. If in the word DISTANCE all the vowels are replaced by the
following will be fourth from the left ?
(a) A
(d) H
(b) S
(e) None of these
(c) J

asy
next letter and all the consonants are replaced by the previous
letter and then all the letters are arranged alphabetically, which
letter will be third from the right?
27. The positions of how many alphabets will remain unchanged
if each of the alphabets in the word WALKING is arranged in
alphabetical order from left to right ?
(a) M
(d) J
(b) F
(e) None of these
(c) R
En (a) None
(d) Three
(b) One (c) Two
(e) More than three

20. After arranging the letters of the word FOLK in alphabetical


order, if each letter is substituted by the letter immediately gin
28. If the first three letters of the word COMPREHENSION are
reversed, then the last three letters are added and then the
remaining letters are reversed and added, then which letter
preceding to it in the English alphabet, what will be the
resultant form of the word?' (a) H eer
will be exactly in the middle ?
(b) N (c) R
(a) GLMP
(d) EJKN
(b) EJKP
(e) None of these
(c) EKJN

21. If A is denoted by 1, B by 2, C by 3, D by 1, E by 2, F by 3 and


(d) S
ing
(e) None of these
29. If the first and second letters in the word DEPRESSION were
interchanged, also the third and the fourth letters, the fifth
so on, what would be the sum of the digits for the word
MULE ? .ne
and the sixth letters and so on, which of the following would
be the seventh letter from the right ?
(a) 7
(d) 10
(b) 8
(e) None of these
(c) 9

22. How many meaningful English words can be made with the
letters ARTSE using each letter only once in each word?
(a) R
(d) P
(b) O
(e) None of these
(c) S

30. Arrange the given words in alphabetical order. Which one


comes in the middle?
t
(a) None (b) One (c) Two (a) Restrict (b) Rocket (c) Robber
(d) Three (e) More than three (d) Random (e) Restaurant

16. a b c d e 17. a b c d e 18. a b c d e 19. a b c d e 20. a b c d e


RESPONSE
21. a b c d e 22. a b c d e 23. a b c d e 24. a b c d e 25. a b c d e
GRID 26. a b c d e 27. a b c d e 28. a b c d e 29. a b c d e 30. a b c d e

Downloaded From : www.EasyEngineering.net


Downloaded From : www.EasyEngineering.net

Coding - Decoding 5
Max. Marks : 30 No. of Qs. 30 Time : 20 min. Date : ........./......../. ..............

1. In a certain code, PROSE is written as PPOQE. How would 9. If ‘light’ is called ‘morning’, ‘morning’ is called ‘dark’, ‘dark’
LIGHT be written in that code ? is called ‘night’, ‘night’ is called ‘sunshine’ and ‘sunshine’ is
(a) LIGFT (b) LGGHT (c) LGGFT called ‘dusk’, when do we sleep ?
(d) JIEHR (e) None of these (a) Dusk (b) Dark (c) Night
(d) Sunshine (e) None of these
2.

ww
If Z = 52 and ACT is equal to 48, then BAT will be equal to
(a) 39
(d) 46
(b) 41
(e) None of these
(c) 44 10. If A + B = C, D – C = A and E – B = C, then what does D + F
stands for ? Provide your answer in letter terms as well as in
3.
w.E
In a certain code, 15789 is written as EGKPT and 2346 is
written as ALUR. How is 23549 written in that code ?
(a) ALEUT (b) ALGTU (c) ALGUT
number terms.
(a) J & 10
(d) Q & 17
(b) A & 1
(e) None of these
(c) C & 3

4.
(d) ALGRT

asy
(e) None of these
In a certain coding system, RBM STD BRO PUS means ‘the
cat is beautiful’. TNH PUS DIM STD means ‘the dog is
11. In a certain code, ‘247’ means ‘spread red carpet’, ‘256’ means
‘dust one carpet’ and ‘234’ means ‘one red carpet’. Which
digit in that code means ‘dust’?
brown’. PUS DIM BRO PUS CUS means ‘the dog has the
cat’. What is the code for ‘has’ ? En (a) 2
(d) 6
(b) 3
(e) None of these
(c) 5

(a) CUS
(d) STD
(b) BRO
(e) None of these
(c) DIM
gin
12. If table is called chair, chair is called cot, cot is called pot and
pot is called filter, where does a person sit?
(a) pot (b) cot (c) chair
5. If NAXALITE is written in a certain code as LYVYJGRC’ how
will INTEGRATE be written in the same code ?
(a) LGRECYPRC (b) GLRCEPYRC 13.
(d) filter
eer (e) None of these
In a certain code OVER is written as $#%*. and VIST is written
(c) PYWMNOPQR
(e) None of these
(d) BLACKHOLE
(a) ×$*% ing
as #+×–. How is SORE written in that code?
(b) %×$*
6. ENGLAND is written as 1234526 and FRANCE as 785291.
How will GREECE be written in this coding scheme ?
(c) ×*$%
(e) All of these
.ne
(d) None of these

7.
(a) 381191
(d) 562134
(b) 381911
(e) None of these
(c) 394132

In a certain code, CAT is written as SATC and DEAR is written


as SEARD. How would SING be written in that code?
14. In a certain code language `PULSE' is written as `DRKTO'
and 'NEW is written as `VDM'. How will 'PROBES' be written
in that code language?
(a) RDANQO (b) QSPCFT
t
(a) GNISS (b) SINGS (c) SGNIS (c) TFCPSQ (d) OPNADR
(d) BGINS (e) None of these (e) None of these
8. PROMISE is coded as 1234567. What should be the code for 15. If REMIT is written as *£3 7 and CONSUL is written as
MISER in that code language ? = %8 b $5; then OCELOT will be written as
(a) 45672 (b) 43672 (c) 76543 (a) %=3587 (b) %=£5%7
(d) 14572 (e) None of these
(c) %=35% (d) %35%7
(e) None of these

1. a b c d e 2. a b c d e 3. a b c d e 4. a b c d e 5. a b c d e
RESPONSE 6. a b c d e 7. a b c d e 8. a b c d e 9. a b c d e 10. a b c d e
GRID 11. a b c d e 12. a b c d e 13. a b c d e 14. a b c d e 15. a b c d e

Downloaded From : www.EasyEngineering.net


Downloaded From : www.EasyEngineering.net

10 SPEED TEST 5
16. If AMONG is written as NAOGM and SPINE is written as 24. In a certain code,'LOCK' is written as `MPBJ'' and `BLOW' is
NSIEP, then LAMON will be written as written as `CMNV'. How is 'WINE' written in that code?
(a) OALNM (b) MLONA (c) OLMNA (a) VHOF (b) XJMD (c) XJOR
(d) OLNMA (e) None of these (d) VHMD (e) None of these
17. In a certain code SEAL is written as $75@ and DOSE is 25. In a certain code SOLDIER is written as JFSCRNK. How is
written as #8$7. How is SOLD written in that code? GENIOUS written in that code?
(a) $8@# (b) #87$ (c) #8$7 (a) PVTHHFO (b) PVTHFDM
(d) $5@# (e) None of these (c) PVTHMDF (d) TVPHFDM
18. If 'table' is called 'chair'; 'chair' is called `cupboard', 'cupboard' (e) None of these
is called 'chalk', 'chalk' is called 'book', 'book' 26. If blue means green, green means black, black means white,
is called 'duster' and 'duster' is called 'table', what does the white means pink, pink means red and red means orange,
teacher use to write on the black board? then what is the colour of blood?
(a) book (b) cupboard (c) table (a) Red (b) Black (c) White
(d) duster (e) None of these (d) Pink (e) None of these
19. in a certain code `MOTHER' is written as OMHURF. How

ww
will `ANSWER' be written in that code ?
(a) NBWRRF
(c) NBWTRD
(b) MAVSPE
(d) NBXSSE
DIRECTIONS (27 and 28) : Study the following information to
answer the given questions :
In a certain code ‘colours of the sky’ is written as ’ki la fa so’,
(e) None of these
w.E
20. In a certain code 'GROW' is written as '=@%#' and 'WITHIN'
is written as '# ¸ + Ó ¸ D ' How is 'WING' written in that
‘rainbow colours’ is written as ‘ro ki’ and ‘sky high rocket’ is
written as ‘la pe jo’ and ‘the rocket world’ is written as ‘pe so ne’.
code?
(a) # ¸ D = (b) # % D = asy
(c) % ¸ D =
27. Which of the following is the code for ’colours sky high’?
(a) ro jo la
(d) ki jo la
(b) fa la jo
(e) fa ki jo
(c) la ki so

(d) # ¸ Ó = (e) None of these


21. If 'DO' is written as 'FQ' and 'IN' is written as 'KP' then how
En 28. Which of the following will/may represent ‘the’?
(a) Only ’fa’ (b) Either ‘fa’ or ’la’
would 'AT' be written?
(a) CV
(d) DV
(b) BS
(e) None of these
(c) CU
gin (c) Only ’so’
(e) Either ‘so’ or ’fa’
(d) Only ’la’

22. lf 8 is written as B, 1 as R, 6 as K, 9 as O, 4 as M, 7 as W and


3 as T, then how, would WROMBT be Written in the numeric eer
29. In a certain code language DREA is written as BFSE, MING
is written as FMHL and TREA is written as BFSU How will
form?
(a) 714983 (b) 719483 (c) 769483 (a) DTHS ing
TISE be written in that code ?
(b) DSTV (c) DSHS
(d) 719486 (e) None of these
23. In a code language, PINK is written as QHOJ and BOLT is
(d) FUGS

.ne
(e) None of these
30. If table is called chair, chair is called cot, cot is called pot
written as CNMS. How would MUST be written in that code?
(a) NVTS
(d) NITU
(b) NTTS
(e) None of these
(c) NTRS
and pot is called filter, where does a person sit?
(a) pot
(d) filter
(b) cot
(e) None of these
(c) chair
t

16. a b c d e 17. a b c d e 18. a b c d e 19. a b c d e 20. a b c d e


RESPONSE
21. a b c d e 22. a b c d e 23. a b c d e 24. a b c d e 25. a b c d e
GRID 26. a b c d e 27. a b c d e 28. a b c d e 29. a b c d e 30. a b c d e

Downloaded From : www.EasyEngineering.net


Downloaded From : www.EasyEngineering.net

Blood Relation 6
Max. Marks : 30 No. of Qs. 30 Time : 20 min. Date : ........./......../. ..............

1. Pointing to a photograph, Arun said, she is the mother of my 10. Ankit is related to Binny and Chinky, Daizy is Chinky’s
brother’s son’s wife’s daughter. How is Arun related to the lady? mother. Also Daizy is Binny’s sister and Aruna is Binny’s
(a) Cousin (b) Daughter-in-law sister. How is Chinky related to Aruna?
(c) Uncle (d) None of these (a) Niece (b) Sister (c) Cousin

2.
ww
(e) None of these
A’s mother is sister of B and has daughter C. How can A be
related to B from amongst the following ?
(d) Aunt (e) None of these
11. Saroj is mother-in-law of Vani who is sister-in-law of Deepak.
Rajesh is father of Ramesh, the only brother of Deepak. How

3.
(a) Niece
(d) Father
w.E (b) Uncle
(e) None of these
(c) Daughter

Introducing Kamla, Mahesh said : His father is the only son


is Saroj related to Deepak?
(a) Mother-in-law
(d) Mother
(b) Wife
(e) None of these
(c) Aunt

(a) Brother
(d) Son
(b) Father
(e) None of theseasy
of my father. How was Mahesh related to Kamla ?
(c) Uncle
12. A man pointing to a photograph says, “The lady in the
photograph is my nephew’s maternal grandmother.” How is
the lady in the photograph related to the man’s sister who
4.
grandson of my mother. How is Anil related to the girl?
En
Anil, introducing a girl in a party, said, she is the wife of the has only brother and no other sister?
(a) Sister-in-law (b) Cousin
(a) Father
(c) Husband
(e) None of these
(b) Grandfather
(d) Father-in-law
gin(c) Mother
(e) None of these
(d) Mother-in- law

5. A and B are married couple. X and Y are brother. X is the


brother of A. How Y is related to B ? eer
13. A man pointing to a photograph says, “The lady in the
photograph is my nephew’s maternal grandmother and her
son is my sister’s brother-in-law. How is the lady in the
(a) Brother
(c) Son
(e) None of these
(b) Brother-in-law
(d) Son-in-law
(a) Mother ing
photograph related to his sister who has no other sister?
(b) Cousin

6. A man said to a woman, “Your mother’s husband’s sister is


my aunt.” How is the woman related to the man ?
(c) Mother-in-law
(e) None of these
(d) Sister-in-law
.ne
7.
(a) Granddaughter
(d) Aunt
(b) Daughter
(e) None of these
(c) Sister

Showing a lady in the park, Vineet said, ‘She is the daughter


of my grandfather’s only son’. How is Vineet related to that
DIRECTIONS (Qs. 14 - 16) : Read the information given below to
answer the questions that follow.
Rama and Mohan are a married couple having two daughters
named Smita and Devika. Devika is married to a man who is the
t
lady ? son of Madhu and Jeewan. Romila is the daughter of Aman.
(a) Father (b) Son (c) Brother Krishna who is Aman’s sister is married to Sunil and has two sons
(d) Mother (e) None of these Anuj and Ankur. Ankur is the grandson of Madhu and Jeewan.
8. X told Y, “Though I am the son of your father, you are not my 14. How is Krishna related to Devika ?
brother”. How is X related to Y ? (a) Sister-in-law (b) Sister
(a) Sister (b) Son (c) Daughter (c) Aunt (d) None of these
(d) None of these (e) None of these
(e) None of these
9. Rahul’s mother is the only daughter of Monika’s father. How
15. What is the relationship between Anuj and Romila?
is Monika’s husband related to Rahul?
(a) Uncle – Niece (b) Father – Daughter
(a) Uncle (b) Father
(c) Grandfather (d) Brother (c) Husband – Wife (d) Cousins
(e) None of these (e) None of these

1. a b c d e 2. a b c d e 3. a b c d e 4. a b c d e 5. a b c d e
RESPONSE 6. a b c d e 7. a b c d e 8. a b c d e 9. a b c d e 10. a b c d e
GRID 11. a b c d e 12. a b c d e 13. a b c d e 14. a b c d e 15. a b c d e

Downloaded From : www.EasyEngineering.net


Downloaded From : www.EasyEngineering.net

12 SPEED TEST 6
16. Which of the following is true ? 22. How is T related to S in the expression 'T × R ¸ V – S'?
(a) Anuj is the son of Smita. (a) Father (b) Sister (c) Daughter
(b) Romila is the cousin of Krishna. (d) Aunt (e) None of these
(c) Madhu is Sunil’s mother-in-law. 23. How is S related to T in the expression 'T + R – V + S'?
(d) Jeewan is Devika’s maternal uncle. (a) Uncle (b) Nephew
(e) None of these (c) Son (d) Cannot be determined
DIRECTIONS (Qs. 17-20) : Read the information given below to (e) None of these
answer these questions: 24. Which of the following means that 'S is the husband of T'?
(i) In a family of six persons, A, B, C, D, E and F, there are two (a) T × R – V + S (b) T – R ¸ V × S
married couples. (c) T – R + V ¸S (d) T ¸ R × V + S
(ii) D is the grandmother of A and the mother of B. (e) None of these
(iii) C is the wife of B and the mother of F. 25. How is V related to T in the expression 'T ¸ R + V × S'?
(iv) F is the granddaughter of E. (a) Aunt (b) Nephew (c) Niece
17. What is C to A? (d) Uncle (e) None of these
(a) Daughter (b) Grandmother 26. D is brother of B. M is brother of B. K is father of M. T is wife

ww
(c) Mother
(e) None of these
(d) Cannot be determined of K. How is B related to T?
(a) Son (b) Daughter
(c) Son or Daughter (d) Data inadequate
(a) Two
(c) Four w.E
18. How many male members are there in the Family?
(b) Three
(d) Cannot be determined
(e) None of these
DIRECTIONS (Qs. 27 - 30) : Study the following information
(e) None of these
19. Which of the following is true?
(a) A is the brother of F
asy carefully to answer the questions that follow.
Adhir Mishra has three children : Urmila, Raghu and Sumit. Sumit
married Roma, the eldest daughter of Mr. And Mrs. Mohan. The
(b) A is the sister of F
(c) D has two grandsons En Mohans married their youngest daughter to the eldest son of Mr.
and Mrs. Sharma and they had two children Sandeep and Shaifali.

(d) None of these


(e) None of these gin
The Mohans have two more children, Roshan and Bimla, both
elder to Sheila. Sohan and Shivendar are sons of Sumit and Roma.
Leela is the daughter of Sandeep.
20. Who among the following is one of the couples?
(a) C D (b) D E (a) Aunt eer
27. How is Mrs. Mohan related to Sumit ?
(b) Mother-in-law
(c) E B
(e) None of these
(d) Cannot be determined (c) Mother
(e) None of these
ing
(d) Sister-in-law

DIRECTIONS (Q.21-25) : Read the following information carefully


and answer the questions which follow:
(i) 'P ¸ Q' means 'P is son of Q'.
28. What is the surname of Sohan ?
(a) Sharma
(d) Raghu
(b) Mohan
(e) None of these.ne(c) Mishra

(ii) 'P × Q' means 'P is sister of Q'.


(iii) 'P + Q' means 'P is brother of Q'.
(iv) 'P – Q' means 'P is mother of Q'.
29. What is the surname of Leela ?
(a) Sharma
(d) None of these
(b) Mohan
(e) None of these
(c) Mishra t
21. How is T related to S in the expression 'T × R + V ¸ S'? 30. How is Shivendar related to Roma’s father ?
(a) Sister (b) Mother (c) Aunt (a) Son-in-law (b) Cousin (c) Son
(d) Uncle (e) None of these (d) Grandson (e) None of these

16. a b c d e 17. a b c d e 18. a b c d e 19. a b c d e 20. a b c d e


RESPONSE
21. a b c d e 22. a b c d e 23. a b c d e 24. a b c d e 25. a b c d e
GRID 26. a b c d e 27. a b c d e 28. a b c d e 29. a b c d e 30. a b c d e

Downloaded From : www.EasyEngineering.net


Downloaded From : www.EasyEngineering.net

Direction and
Distance
7
Max. Marks : 30 No. of Qs. 30 Time : 20 min. Date : ........./......../. ..............
1. At my house I am facing east, then I turn left and go 10 m, (I) Kunal is 40 metres to the right of Atul.
then turn right and go 5 m, and then I go 5 m towards the (II) Dinesh is 60 metres to the south of Kunal
south and from there 5 m towards the west. In which direction (III) Nitin is 25 metres to the west of Atul.
am I from my house ? (IV) Prashant is 90 metres to the north of Dinesh.
(a) East (b) West (c) North 8. Who is to the north-east of the person who is to the left of
(d) South (e) None of these Kunal?
2.
ww
My friend and I started walking simultaneously towards each
other from two places 100 m apart. After walking 30 m, my
friend turns left and goes 10 m, then he turns right and goes 9.
(a) Dinesh
(d) None of these
(b) Nitin
(e)
(c) Atul

If a boy walks from Nitin, meets Atul followed by Kunal,

w.E
20 m and then turns right again and comes back to the road
on which he had started walking. If we walk with the same
speed, what is the distance between us at this point of time?
(a) 50 m (b) 20 m (c) 30 m
Dinesh and Prashant, then how many metres has he walked
if he has travelled the straight distance all through?
(a) 155 metres (b) 185 metres

3.
(d) 40 m (e) None of these
asy
A watch reads 4:30 O’clock. If minute hand points towards
the East, in which direction does the hour hand point ?
10.
(c) 215 metres
(e) None of these
(d) 245 metres

A square field ABCD of side 90 m is so located that its


(a) North-East
(c) North-West
(b) South-East
(d) North En diagonal AC is from north to south and the corner B is to the
west of D. Rohan and Rahul start walking along the sides
from B and C respectievely in the clockwise and anti-
4.
(e) None of these
A man travels 3 km to the west, turns left and goes 3 km,
turns right and goes 1 km, again turns right and goes 3 km. gin clockwise directions with speeds of 8 km/hr and 10 km/hr.
Where shall they cross each other the second time ?
How far is he from the starting point ?
(a) 7 km
(d) 4 km
(b) 6 km
(e) None of these
(c) 5 km
eer
(a) On AD at a distance of 30 m from A
(b) On BC at a distance of 10 m from B
(c) On AD at a distance of 30 m from D
5. A and B start walking in opposite directions. A covers 3 km and
B covers 4 km. Then A turns right and walks 4 km while B turns 11.
(e) None of these
ing
(d) On BC at a distance of 10 m from C

One morning after sunrise, Reeta and Kavita were talking to


left and walks 3 km. How far is each from the starting point ?
(a) 5 km
(d) 8 km
(b) 4 km
(e) None of these
(c) 10 km
.ne
each other face to face at Tilak square. If Kavita’s shadow
was exactly to the right of Reeta, then which direction was

t
Kavita facing?
6. Ram walks 10 m south from his house, turns left and walks
(a) North (b) South (c) East
25 m, again turns left and walks 40 m, then turns right and
(d) Data inadequate (e) None of these
walks 5 m to reach to the school. In which direction the
12. I am facing west. I turn 45° in the clockwise direction and
school is from his house ?
then 180° in the same direction and then 270° anticlockwise.
(a) South-west (b) North-east (c) East
Which direction am I facing now?
(d) North (e) None of these
(a) South-West (b) South (c) West
7. A river flows west to east and on the way turns left and goes
(d) North-west (e) None of these
in a semicircle round a hillock, and then turns left in a right-
13. Bhavika and Sunaina start simultaneously towards each
angle. In which direction is the river finally flowing ?
other from two places 100m apart. After walking 30 m Bhavika
(a) North (b) South (c) East
turns left and goes 10 m, then she turns right and goes 20 m
(d) West (e) None of these
and then turns right again and comes back to the road on
which she had started walking. If both Bhavika and Sunaina
DIRECTIONS (Qs. 8 & 9) : Study the information given below walk with the same speed, what is the distance between
carefully to answer these questions: them at this point of time?
On a playing ground, Dinesh, Kunal, Nitin, Atul and Prashant are (a) 70 metres (b) 10 metres (c) 40 metres
standing, as described below, facing the North. (d) 20 metres (e) None of these

1. a b c d e 2. a b c d e 3. a b c d e 4. a b c d e 5. a b c d e
RESPONSE 6. a b c d e 7. a b c d e 8. a b c d e 9. a b c d e 10. a b c d e
GRID 11. a b c d e 12. a b c d e 13. a b c d e

Downloaded From : www.EasyEngineering.net


Downloaded From : www.EasyEngineering.net

14 SPEED TEST 7
14. A man starts from a point and moves 3 km north, then turns to (a) North (b) South (c) West
west and goes 2 km. He turns north and walks 1 km and then (d) East (e) North-West
moves 5 km towards east. How far is he from the starting point? 23. Town D is to the West of town M. Town R is to the South of
(a) 11 km (b) 10 km (c) 5 km town D. Town K is to the East of town R. Town K is towards
(d) 8 km (e) None of these which direction of town D?
15. Starting from Point X, Joy walked 15 metres towards West. (a) South (b) East
He turned left and walked 20 metres. He again turned left (c) North-East (d) South-East
and walked 15 metres. After which he turned right and (e) None of these
walked for another 12 metres. How far is Joy from point X if 24. Mohan walked 30 metres towards South, took a left turn
he is facing North at present? and walked 15 metres. He then took a right turn and walked
(a) 27 m (b) 35 m (c) 32 m 20 metres. He again took a right turn and walked 15 metres.
(d) 42 m (e) None of these How far is he from the starting point?
16. Town D is 12 km towards the North of town A. Town C is 15 (a) 95 metres (b) 50 metres
km towards the West of town D. Town B is 15 km towards (c) 70 metres (d) Cannot be determined
the West of town A. How far and in which direction is town (e) None of these
B from town C? 25. W walked 40 metres towards West, took a left turn and
(a) 15 km towards North (b) 12 km towards North walked 30 metres. He then took a right turn and walked 20
(c) 3 km towards South (d) 12 km towards South metres, He again took a right turn and walked 30 metres.

17.
ww
(e) cannot be determined
Rahul started from point A and travelled 8 km towards the
North to point B, he then turned right and travelled 7 km to
How far was he from the starting point?
(a) 70 metres
(c) 90 metres
(b) 60 metres
(d) Cannot be determined

w.E
point C, from point C he took the first right and drove 5 km
to point D, he took another right and travelled 7 km to point
E and finally turned right and travelled for another 3 km to
point F. What is the distance between point F and B?
26.
(e) None of these
Town D is 13 km towards the East of town A. A bus starts
from town A, travels 8 km towards West and takes a right
turn. After taking the right turn, it travels 5 km and reaches

18.
(a) 1 km
(d) 4 km
(b) 2 km
(e) None of these
asy
(c) 3 km

Meghna drives 10 km towards South, takes a right turn and


town B. From town B the bus takes a right turn again, travels
21 km and stops. How far and towards which direction must
the bus travel to reach town D?
drives 6 km. She then takes another right turn, drives 10 km
and stops. How far is she from the starting point?
En (a) 13 km towards South (b) 5 km towards West
(c) 21 km towards South (d) 5 km towards South

19.
(a) 16 km
(d) 12 km
(b) 6 km
(e) None of these
(c) 4 km

Vikas walked 10 metres towards North, took a left turn and gin
27.
(e) None of these
A person travels 12 km due North, then 15 km due East, after
that 15 km due West and then 18 km due South. How far is
walked 15 metres, and again took a left turn and walked 10
metres and stopped walking. Towards which direction was
he facing when he stopped walking? (d) 60 km eer
he from the starting point?
(a) 6 km (b) 12 km
(e) 65 km
(c) 33 km

(a) South
(c) South-East
(e) None of these
(b) South-West
(d) Cannot be determined
28.
ing
In a meeting, the map of a village was placed in such a
manner that south-east becomes north, north-east becomes
west and so on. What will south become?
DIRECTIONS (Q. 20 & 21) : Study the following information
carefully to answer these questions.
(a) North
(d) West (e) South
.ne
(b) North-east (c) North-west

A vehicle starts from point P and runs 10 km towards North. It


takes a right turn and runs 15 km. It now runs 6 km after taking a
left turn. It finally takes a left turn, runs 15 km and stops at point Q.
20. How far is point Q with respect to point P?
29. A school bus driver starts from the school, drives 2 km

t
towards North, takes a left turn and drives for 5 km. He then
takes a left turn and drives for 8 km before taking a left turn
again and driving for further 5 km. The driver finally takes a
left turn and drives 1 km before stopping. How far and
(a) 16 km (b) 25 km (c) 4 km towards which direction should the driver drive to reach the
(d) 0 km (e) None of these school again?
21. Towards which direction was the vehicle moving before it (a) 3 km towards North (b) 7 km towards East
stopped at point Q? (c) 6 km towards South (d) 6 km towards West
(a) North (b) East (c) South (e) 5 km towards North
(d) West (e) North-East 30. Roma walked 25 metre towards south, took a right turn and
22. Raman starts from point P and walks towards South and walked 15 metre. She then took a left turn and walked 25
stops at point Q. He now takes a right turn followed by a left meter. Which direction is she now from her starting point?
turn and stops at point R. He finally takes a left turn and (a) South-east (b) South (c) South-west
stops at point S. If he walks 5 km before taking each turn, (d) North-west (e) None of these
towards which direction will Raman have to walk from point
S to reach point Q?
14. a b c d e 15. a b c d e 16. a b c d e 17. a b c d e 18. a b c d e
RESPONSE 19. a b c d e 20. a b c d e 21. a b c d e 22. a b c d e 23. a b c d e
GRID 24. a b c d e 25. a b c d e 26. a b c d e 27. a b c d e 28. a b c d e
29. a b c d e 30. a b c d e

Downloaded From : www.EasyEngineering.net


Downloaded From : www.EasyEngineering.net

Time, Sequence and


Ranking Test
8
8
Max. Marks : 32 No. of Qs. 32 Time : 20 min.

1. At an enquiry office at a railway station, a passenger was Date : ........./......../. ..............


told that a train for New Delhi has left 15 minutes ago, but and answer the question that follow:
after every 45 minutes a train leaves for New Delhi. The next Six boys A, B, (Qs.
DIRECTIONS C, D, E and
8-11) F are
: Read marching information
the following in a line. They are
carefully
train will leave at 8.30 p.m. At what time was this information arranged according to their height, the tallest being at the back
given to the passanger ? and the shortest in front,

2. ww
(a) 7.45 pm
(d) 8.05 pm
(b) 8.00 pm
(e) None of these
(c) 8.15 pm

A watch is a minute slow at 1 p.m. on Tuesday and 2 minutes


F is between B and A.
E is shorter than D but taller than C who is taller than A.
E and F have two boys between them.
A is not the shortest among them.

w.E
fast at 1 p.m. on Thursday. When did it show the correct time?
(a) 1:00 a.m. on Wednesday
(b) 5:00 a.m. on Wednesday
8. Where is E ?
(a) Between A and B (b) Between C and A
(c) Between D and C (d) In front of C
(c) 1:00 p.m. on Wednesday
(d) 5:00 p.m. on Wednesday
(e) None of these asy 9.
(e) None of these
Who is the tallest ?
(a) B (b) D (c) F
3. Reaching the place of meeting on Tuesday 15 minutes before
8.30 hours, Anuj found himself half an hour earlier than the
En (d) A (e) None of these
10. If we start counting from the shortest, which boy is fourth
one in the line ?
man who was 40 minutes late. What was the scheduled time
of the meeting?
(a) 8.00 hrs (b) 8.05 hrs (c) 8.15 hrs gin (a) E
(d) C
11. Who is the shortest ?
(b) A
(e) None of these
(c) D

4.
(d) 8.45 hrs (e) None of these
A clock gaining 2 min every hour was synchronised at midnight
with a clock losing 1 min every hour. How many minutes behind
(a) C
(d) F
eer (b) D
(e) None of these
(c) B

12. Five newly born babies were weighed by the doctor. In her
will its minute hand be at eleven the following morning ?
(a) 23 (b) 27 (c) 22 ing
report, she stated that child A is lighter than child B, child C
is lighter than D and child B is lighter than child D, but
heavier than child E. Which child is the heaviest?
5.
(d) None of these (e)
Samant remembers that his brother’s birthday is after fifteenth
but before eighteenth of February, whereas his sister
(a) E
(d) A
(b) D
.ne
(e) None of these
(c) C

remembers that her brother’s birthday is after sixteenth but


before nineteenth of February. On which date of February is
Samant’s brother’s birthday?
(a) 18th (b) 16th (c) 19th
t
13. Thirty-six vehicles are parked in a parking in a single row.
After the first car, there is one scooter. After the second car,
there are two scooters. After the third car, there are three
scooters and so on. Work out the number of scooters in the
second half of the row.
(d) 17th (e) None of these (a) 10 (b) 12 (c) 15
6. A is shorter than B but much taller than E. C is the tallest and (d) 17 (e) None of these
D is little shorter than A. Who is the shortest ? 14. In a row at a bus stop, A is 7th from the left and B is 9th from
(a) A (b) E (c) C the right. Both of them interchange their positions and thus
A becomes 11th from the left. How many people are there in
(d) D (e) None of these
that row?
7. In a class of 35 students Kiran is placed 7th from the bottom (a) 18 (b) 19 (c) 20
whereas Sohan is placed 9th from the top. Mohan is placed exactly (d) 21 (e) None of these
in between the two. What is Kiran’s position from Mohan? 15. A, B, C, D and E when arranged in descending order of their
(a) 10 (b) 11 (c) 13 weight from the top, A becomes third, E is between D and A while
(d) 12 (e) None of these C and D are not at the top. Who among them is the second?
(a) C (b) B (c) E
(d) Data inadequate (e) None of these
1. a b c d e 2. a b c d e 3. a b c d e 4. a b c d e 5. a b c d e
RESPONSE 6. a b c d e 7. a b c d e 8. a b c d e 9. a b c d e 10. a b c d e
GRID 11. a b c d e 12. a b c d e 13. a b c d e 14. a b c d e 15. a b c d e

Downloaded From : www.EasyEngineering.net


Downloaded From : www.EasyEngineering.net

16 SPEED TEST 8
DIRECTIONS (Qs. 16 & 17) : Read the relationships given 24. In a row of twenty-five children facing North, W is fifth to
below to answer the questions that follow. the right of R, who is sixteenth from the right end of the row.
Archana is elder than Suman. Arti is elder than Archana but What is W's position from the right end of the row?
younger than Kusum. Kusum is elder than Suman. Suman is (a) Eleventh (b) Tenth (c) Twelfth
younger than Arti and Gita is the eldest. (d) Data inadequate (e) None of these
16. Who is the youngest ? 25. Seema's watch is 6 minutes fast and the train, which should
(a) Archana (b) Suman (c) Arti have arrived at 7 p.m. was 14 minutes late. What time is it by
(d) Kusum (e) None of these Seema's watch when the train arrived?
17. Age wise, who is in the middle ? (a) 7 : 05 pm (b) 7 : 30 pm (c) 7 : 01 pm
(a) Suman (b) Archna (c) Arti (d) 7 : 31 pm (e) None of these
(d) Kusum (e) None of these 26. In a row of children facing North, Ritesh is twelfth from the
18. Five boys took part in a race. Raj finished before Mohit but left end. Sudhir who is twenty-second from the right end is
behind Gaurav. Ashish finished before Sanchit but behind fourth to the right of Ritesh. Total how many children are
Mohit. Who won the race? there in the row?
(a) Raj (b) Gaurav (c) Mohit (a) 35 (b) 36 (c) 37
(d) Ashish (e) None of these (d) 34 (e) None of these

ww
19. Five men A, B, C, D and E read newspaper. The one who
27. Among A, B, C, D and E each reaching school at a different
reads first gives it to C. The one who reads last had taken it
time, C reaches before D and A and only after B. E is not the
from A. E was not the first or the last to read. There were two
readers between B and A. last to reach school. Who among them reached school last?

(a) A
(d) E
w.E
B passed the newspaper to whom?
(b) C
(e) None of these
(c) D
28.
(a) D (b) A
(d) Data inadequate (e) None of these
(c) C

M is older than R. Q is younger than R and N. N is not as old

asy
20. In a row of children, Bhushan is seventh from the left and
Motilal is fourth from the right. When Bhushan and Motilal
exchange positions, Bhushan will be fifteenth from the left.
as M. Who among M, N, R and Q is the oldest?
(a) M (b) R
(d) Data inadequate (e) None of these
(c) M or R

What will be Motilal’s position from the right ?


(a) Eighth (b) Fourth
En
(c) Eleventh
29. Akshay is 16th from the left end in the row of boys and Vijay
is 18th from the right end. Avinash is 11th from Akshay
(d) Twelfth (e) None of these
21. In a queue I am the last person while my friend is seventh
from the front. If the person exactly between me and my gin towards the right end and 3rd from Vijay towards the right
end. How many boys are there in the row?
(a) Data inadequate (b) 42 (c) 40
friend is on the 23rd position from the front, what is my
position in the queue ?
(a) 37 (b) 36 (c) 38
30.
(d) 48
eer (e) None of these
Geeta is senior to Shilpa but not to Deepa. Gayatri is junior
(d) 39 (e) None of these
22. Pratap correctly remembers that his mother's birthday is (a) Shilpa
ing
to Reepa. No one is senior to Fatima. Who is most junior?
(b) Geeta
(d) Data inadequate (e) None of these
(c) Gayatri
before twentythird April but after nineteenth April, whereas
his sister correctly remembers that their mother's birthday is
not on or after twentysecond April. On which day in April is
31.
.ne
Among M, N, P, R and T each one has secured different
marks in an examination. R secured more marks than M and
definitely their mother's birthday?
(a) Twentieth
(b) Twentyfirst
(c) Twentieth or Twentyfirst
T. N secured less marks than P. Who among them secured
third highest marks?
(a) N
(d) T
(b) R
(e) Data inadequate
(c) M
t
(d) Cannot be determined 32. In a column of girls Kamal is 11th from the front. Neela is
(e) None of these 3 places ahead of Sunita who is 22nd from the front.
23. In a row of forty children, R is eleventh from the right end How many girls are there between Kamal and Neela in the
and there are fifteen children between R and M. What is M's column?
position from the left end of the row?
(a) Six (b) Eight
(a) Fourteenth (b) Fifteenth
(c) Seven (d) Cannot be determined
(c) Thirteenth (d) Cannot be determined
(e) None of these (e) None of these

16. a b c d e 17. a b c d e 18. a b c d e 19. a b c d e 20. a b c d e


RESPONSE 21. a b c d e 22. a b c d e 23. a b c d e 24. a b c d e 25. a b c d e
GRID 26. a b c d e 27. a b c d e 28. a b c d e 29. a b c d e 30. a b c d e
31. a b c d e 32. a b c d e

Downloaded From : www.EasyEngineering.net


Downloaded From : www.EasyEngineering.net

Sitting
Arrangement 9
8
9
Max. Marks : 30 No. of Qs. 30 Time : 20 min. Date : ........./......../. ..............

1. A, B, C, D, E, F and G are sitting in a line facing east. C is 7. Who is to the immediate left of H?
immediate right of D. B is at the extreme ends and has E as his (a) W (b) T (c) R
neighbour. G is between E and F. D is sitting third from the (d) Data inadequate (e) None of these
south end. Who are the persons sitting at the extreme ends? (e) None of these
(a) A and E (b) A and B (c) F and B 8. Who is third to the right of H ?
(d) C and D (e) None of these (a) M (b) D (c) Y

ww
DIRECTIONS (Qs. 2-4) : Read the following statements to answer
the questions that follow.
9.
(d) R
(e) None of these
(e) None of these

Who is third to the right of D?

w.E
Nine cricket fans are watching a match in a stadium. Seated in one
row, they are – J, K, L, M, N, O, P, Q and R. L is at the right of M and (a) M (b) R (c) W
at third place to the right of N. K is at one end of the row. Q is (d) M (e) None of these
immediately next to O and P. O is at the third place to the left of K. (e) None of these
J is right next to left of O.
2. Who is sitting in the centre of the row ? asy 10. What is Y's position with respect toT?
(a) Third to the right (b) Fourth to the left
(c) Third to the left (d) Second of the left

En
(a) L (b) O (c) J
(d) Q (e) None of these (e) None of these
3. Who is at the other end of the row ? DIRECTIONS (Q.11-15): Study the following information carefully

4.
(a) R
(d) N
(b) J
(e) None of these
Which of the following statements is true ?
(c) P
gin
to answer these questions :
Eight friends J, K, L, M, N, O, P and Q are sitting around a circle

eer
facing the centre. J is not the neighour of N. L is third to the right
(a) R and P are neighbours. of K. Q is second to the left of N who is next to the right of L. O is
(b) There is one person between L and O. not the neighbour of N or K and is to the immediate left of P.
(c) M is at one extreme end.
(d) N is two seats away from J.
(e) None of these ing
11. Which of the following is the correct position of L?
(a) To the immediate right of N

.ne
(b) To the immediate right of Q
5. A, B, C, D, E and F are sitting around a round table. A is
between E and F, E is opposite D, and C is not in either of the (c) To the immediate left of N
neighbouring seats of E. Who is opposite to B ? (d) To the immediate left of Q

6.
(a) F
(d) None of these
(b) C
(e)
(c) D

Four girls (G1, G2, G3, G4) and three boys (B1, B2, B3) are to
sit for a dinner such that no two boys should sit together
(e) None of these
12. Which of the following pair of persons represent O's
neighbours?
(a) L&N (b) P&K (c) M&P
t
nor two girls. If they are successively sitting, what is the (d) N&P (e) None of these
position of B2 and G3? 13. Which of the following groups has the first person sitting
(a) 5th and 6th (b) 4th and 5th between the other two persons?
(c) 3rd and 4th (d) 2nd and 3rd (a) PKJ (b) JQL (c) QNL
(e) None of these (d) LMN (e) None of these
14. Who is to the immediate right of K?
DIRECTIONS (Q. 7-10) : Study the following information carefully (a) J (b) P
and answer the questions given below: (c) Q (d) Cannot be determined
W, Y, T, M, R, H and D are seven persons, sitting around a circle (e) None of these
facing the centre. T is fourth to the right of M who is second to the 15. Who is to the immediate left of O?
right of R. W is third to the left of R. H is not an immediate neighbour (a) P (b) L (c) Q
of M. D is not an immediate neighbour of W.
(d) M (e) None of these

1. a b c d e 2. a b c d e 3. a b c d e 4. a b c d e 5. a b c d e
RESPONSE 6. a b c d e 7. a b c d e 8. a b c d e 9. a b c d e 10. a b c d e
GRID 11. a b c d e 12. a b c d e 13. a b c d e 14. a b c d e 15. a b c d e

Downloaded From : www.EasyEngineering.net


Downloaded From : www.EasyEngineering.net

18 SPEED TEST 9
DIRECTIONS (Q. 16 - 20): These questions are based on the 24. Who sits between G & D?
basis of following information. Study it carefully and answer the (a) H (b) D (c) F
questions. (d) E (e) None of these
Eight executives J, K, L, M, N, O, P and Q are sitting around a 25. Which of the following is the correct position of B with
circular table for a meeting. J is second to the right of P who is respect to H?
third to the right of K. M is second to the left of O who sits I Second to the right
between P and J. L is not a neighbour of K or N. II Fourth to the right
III Fourth to the left
16. Who is to the immediate left of L?
IV Second to the left
(a) Q (b) O (c) K
(a) Only I (b) Only II (c) Only III
(d) N (e) None of these
(d) Both II & III (e) None of these
17. Who is to the immediate left of K?
(a) N (b) J DIRECTIONS (Q. 26 - 30) : Study the following information
(c) Q (d) Cannot be determined carefully and answer the given questions:
(e) None of these Eight friends P, Q, R, S, T, V, W and Y are sitting around a square
18. Which of the following is the correct position of N? table in such a way that four of them sit at four corners of the

ww
(a) Second to the right of K
(b) To the immediate left of K
square while four sit in the middle of each of the four sides. The
ones who sit at the four corners face the centre while those who
sit in the middle of the sides face outside.

w.E
(c) To the immediate right of M
(d) To the immediate right of K
(e) None of these
P, who faces the centre, sits third to the right of V. T, who faces the
centre, is not an immediate neighbour of V. Only one person sits
19. Who is third to the right of P ?
(a) L
(d) N
(b) J
(e) None of these asy
(c) Q
between V and W. S sits second to right of Q. Q faces the centre.
R is not an immediate neighbour of P.
26. Who sits second to the left of Q?
20. Which of the following groups of persons have the first
person sitting between the other two? En (a) V
(d) Y
(b) P (c) T
(e) Cannot be determined
(a) PJO
(d) MPO
(b) OPJ
(e) None of these
(c) OPM
gin
27. What is the position of T with respect to V?
(a) Fourth to the left (b) Second to the left
DIRECTIONS (21-25) : Study the following information
carefully to answer these questions.
eer
(c) Third to the left (d) Third to the right
(e) Second to the right
28. Four of the following five are alike in a certain way and so
A, B, C, D, E, F, G and H are sitting around a circle facing the
centre. F is third to the right of C and second to the left of H. D is
not an immediate neighbour of C or H. E is to the immediate right
group? ing
form a group. Which is the one that does not belong to that

of A, who is second to the right of G.


21. Who is second to the left of C?
(a) R
(d) S
(b) W
(e) Y
.ne (c) V

22.
(a) A
(d) D
(b) B
(e) None of these
Who is to the immediate right of C?
(c) E
29. Which of the following will come in place of the question
mark based upon the given seating arrangement?
WP TR QW RS ?
(a) Y T (b) VY (c) V Q
t
(a) E (b) B (c) D (d) P Y (e) Q V
(d) B or D (e) None of these 30. Which of the following is true regarding R?
23. Which of the following pairs of persons has first person (a) R is an immediate neighbour of V.
sitting to the right of the second person? (b) R faces the centre.
(a) CB (b) AE (c) FG (c) R sits exactly between T and S.
(d) HA (e) DB (d) Q sits third to left of R
(e) None of these

16. a b c d e 17. a b c d e 18. a b c d e 19. a b c d e 20. a b c d e


RESPONSE
GRID 21. a b c d e 22. a b c d e 23. a b c d e 24. a b c d e 25. a b c d e
26. a b c d e 27. a b c d e 28. a b c d e 29. a b c d e 30. a b c d e

Downloaded From : www.EasyEngineering.net


Downloaded From : www.EasyEngineering.net

Analytical
Puzzle - I
10
8
Max. Marks : 30 No. of Qs. 30 Time : 20 min.

DIRECTIONS (Qs. 1 - 5) : The questions below are based on the Date : ........./......../. ..............
following statements. answer the questions that follow :
Asha and Charu are good in Mathematics and Athletics. Deepa (i) There is a family
DIRECTIONS (Qs. 8 -of12)
5 persons
: Study A,
theB, C, D and E.
information given below to
and Asha are good in Athletics and Studies, Charu and Beena are (ii) They are working as a doctor, a teacher, a trader, a lawyer
good in General Knowledge and Mathematics. Deepa, Beena and and a farmer.

ww
Ela are good in Studies and General Knowledge. Ela and Deepa
are good in Studies and Arts.
(iii) B, an unmarried teacher, is the daughter of A.
(iv) E, a lawyer, is the brother of C.
(v) C is the husband of the only married couple in the family.
1.

2.
(a) Asha
(d) Deepa w.E
Who is good in Studies, General Knowledge, Athletics & Arts?
(b) Beena
(e) None of these
(c) Charu

Who is good in Studies, General Knowledge and Mathematics?


(vi) Daughter-in-law of A is a doctor.
8. Which of the following is a group of female members in the
family ?

3.
(a) Asha
(d) Deepa
(b) Beena
asy
(e) None of these
(c) Charu

Who is good in Studies, Mathematics and Athletics ?


(a) A and D
(d) B and D
(b) D and E
(e) None of these
(c) A, C and E

(a) Asha
(d) Deepa
(b) Beena
(e) None of these
(c) Charu
En 9. Which of the following is the married couple ?
(a) A and B
(d) B and C
(b) C and D
(e) None of these
(c) A and D

4. Who is good in Athletics, General Knowledge and Mathematics?


(a) Asha
(d) Deepa
(b) Beena
(e) None of these
(c) Charu
gin
10. Which of the following is a group of male members in the
family ?

5. Who is good in Studies, General Knowledge and Arts but


not in Athletics ? eer
(a) A, B and C
(d) A, C and D
(b) B and D
(e) None of these
11. Who is the doctor in the family ?
(c) A, C and E

(a) Asha
(d) Ela
(b) Beena
(e) None of these
(c) Charu

DIRECTIONS (Qs. 6 & 7) : Study the information given below


(a) A
(d) D
ing
(b) B
(e) None of these
12. Who is the trader in the family ?
(c) C

carefully and then answer the questions that follow :


Students joining certain university can choose from among seven
(a) A
(d) D
(b) B
.ne
(e) None of these
(c) C

courses : Biology, Astronomy, English, Mathematics, History,


Chemistry and Psychology.
A student cannot take both English and Astronomy.
Chemistry is a prerequisite for Biology and cannot be taken
DIRECTIONS (Qs. 13 - 16) : Study the infomation given below to
answer these questions :
t
There are four friends A, B, C and D. One of them is a Cricketer and
studies Chemistry and Biology. A and B play Football. Both
concurrently with Biology. A student must take at least two of the Football players study Maths. D is a Boxer. One football player
three courses, English, History and Psychology. No course may
also studies physics. The Boxer studies Maths and Accounts. All
be repeated.
the friends study two subjects each and play one game each.
6. What is the maximum number of courses that can a student
13. Who is the Cricketer ?
can take without violating any of the conditions given above?
(a) A (b) B (c) C
(a) 3 (b) 4 (c) 5
(d) D (e) None of these
(d) 6 (e) None of these
14. Who studies Accounts and plays Football ?
7. If a student is taking Astronomy and Mathematics and
(a) A alone (b) B alone (c) D
wishes to take four courses, then in how many different
ways can the other two courses be taken? (d) A or B (e) None of these
(a) One (b) Two (c) Three 15. Who studies Physics?
(d) Four (e) None of these (a) A or B (b) A alone (c) B alone
(d) D (e) None of these
1. a b c d e 2. a b c d e 3. a b c d e 4. a b c d e 5. a b c d e
RESPONSE 6. a b c d e 7. a b c d e 8. a b c d e 9. a b c d e 10. a b c d e
GRID 11. a b c d e 12. a b c d e 13. a b c d e 14. a b c d e 15. a b c d e

Downloaded From : www.EasyEngineering.net


Downloaded From : www.EasyEngineering.net

20 SPEED TEST 10
16. How many games are played and subjects studied by the 22. Who among the teachers was teaching maximum number of
four friends ? subjects?
(a) 1 game and 4 subjects (a) A (b) C (c) B
(b) 2 games and 3 subjects (d) D (e) None of these
(c) 3 games and 4 subjects 23. Which of the following pairs was teaching both Geography
(d) 3 games and 5 subjects and Hindi?
(e) None of these (a) A and B (b) C and A (c) B and C
DIRECTIONS (Qs. 17 - 21) : Study the information given below (d) None of these (e)
to answer these questions : 24. More than two teachers were teaching which subject?
(i) Six plays A, B, C, D, E and F are to be organised from (a) History (b) French (c) Hindi
Monday to Saturday, i.e from 5th to 10th-one play each day. (d) Geography (e) None of these
(ii) There are two plays between C and D and one play between 25. D, B and A were teaching which of the following subjects?
A and C. (a) English only (b) Hindi only
(iii) There is one play between F and E and E is to be organised (c) Hindi and English (d) English and Geography
before F. (e) None of these
(iv) B is to be organised before A, not necessarily immediately. DIRECTIONS (26-30) : Study the following information carefully

ww
(v) The organisation does not start with B
17. The organisation would start from which play ?
(a) A (b) F
and answer the given questions.
Seven flights namely those of Jet Airways, British Airways, Delta,
Quantas, Emirates, Lufthansa and Air India are scheduled to fly to
(c) D
(e) None of these w.E (d) Cannot be determined

18. On which date the play E is to be organised ?


London. There is only one flight to London on each of the seven
days of the week, starting from Monday and ending on Sunday.
Delta flies on Wednesday. Air India flies the day next to British
(a) 5th
(c) 6th
(e) None of these
(b) 7th
asy
(d) Cannot be determined
Airways. British Airways does not fly on Monday or Friday. Two
airlines fly between the days British Airways and Emirates fly.
Emirates does not fly on Sunday. Quantas flies a day before
19. The organisation would end with which of the following
plays? En Lufthansa.
26. On which of the following days does Jet Airways fly ?
(a) A
(c) B
(b) D
(d) Cannot be determined
gin (a) Friday
(d) Thursday
(b) Sunday
(e) None of these
(c) Tuesday

(e) None of these


20. On which day the play B is organised ?
(a) Tuesday (b) Friday (d) Three eer
27. How many flights fly between Lufthansa and Delta ?
(a) None (b) One
(e) Five
(c) Two

(c) Thursday
(e) None of these
(d) None of these

21. Which of the following is the correct sequence of organising


(a) Air India
(d) Lufthansa
ing
28. Which of the following flights flies on Friday ?
(b) Quantas
(e) Jet Airways
(c) Emirates

plays?
(a) AECFBD (b) DFECBA (c) BDEFCA .ne
29. If Delta postpones its flight to Sunday owing to some technical
reasons and all the flights scheduled for Thursday to Sunday
(d) None of these (e)
DIRECTIONS (Qs. 22-25) : Read the following information to
answer these questions :
In a school, there were five teachers. A and B were teaching Hindi
(a) Lufthansa (b) Jet Airways
(c) British Airways (d) Air India
t
are now made to take off a day ahead of the schedule, which of
the following flights would now fly on Friday?

and English. C and B were teaching English and Geography. D (e) Quantas
and A were teaching Mathematics and Hindi. E and B were teaching 30. If Emirates is related to British Airways and Delta is related
History and French. to Quantas in a certain way based upon the given flight
schedule, then Jet Airways will be related to which of the
following based upon the same relationship ?
(a) Lufthansa (b) Quantas (c) Delta
(d) Air India (e) None of these

16. a b c d e 17. a b c d e 18. a b c d e 19. a b c d e 20. a b c d e


RESPONSE
GRID 21. a b c d e 22. a b c d e 23. a b c d e 24. a b c d e 25. a b c d e
26. a b c d e 27. a b c d e 28. a b c d e 29. a b c d e 30. a b c d e

Downloaded From : www.EasyEngineering.net


Downloaded From : www.EasyEngineering.net

Analytical
Puzzle - II
11
Max. Marks : 30 No. of Qs. 30 Time : 20 min. Date : ........./......../. ..............

DIRECTIONS (Q.1- 5) : Read the following passage carefully (a) None (b) One (c) Two
and answer these questions given below it. (d) Three (e) Four
7. Which of the following plays is scheduled on Saturday?
A group of seven friends A, B, C, D, E, F and G, work as Economist, (a) Q (b) W (c) R
Agriculture Officer, IT Officer, Terminal Operator, Clerk, Forex (d) S (e) T
Officer and Research Analyst, for Banks L, M, N, P, Q, R and S, but 8. R is related to S in a certain way. In the same way P is
not necessarily in the same order. C works for Bank N and is related to V based on the given schedule. Which of the

ww
neither Research Analyst nor a Clerk. E is an IT Officer and works
for Bank R.A works as Forex Officer and does not work for Bank L
or Q. The one who is an Agricultural Officer works for Bank M.
The one who works for Bank L works as a Terminal Operator. F
following is W related to the following the same pattern?
(a) P
(d) T
(b) Q (c) R
(e) Cannot be determined

w.E
works for Bank Q. G works for Bank P as a Research Analyst. D is
not an Agricultural Officer.
1. Who amongst the following works as an Agriculture Officer?
9. On which of the following days is Play W scheduled?
(a) Monday
(c) Wednesday
(e) Cannot be determined
(b) Tuesday
(d) Saturday

2.
(a) C
(d) D
(b) B
(e) None of these
What is the profession of C? asy(c) F
10. Which of the following plays is scheduled on Friday?
(a) R
(d) W
(b) T
(e) S
(c) Q
(a) Terminal Operator (b) Agriculture Officer
(c) Economist
(e) None of these
(d) Cannot be determined
En DIRECTIONS (Q.11-Q.15) : Study the following information
3. For which bank does B work?
(a) M
(d) Either M or S
(b) S
(e) None of these
(c) L
gin
carefully and answer the questions given below:
P, Q, R, S, T, V and W are seven friends working in a call centre.
Each of them has different day offs in a week from Monday to
4. What is the profession of the person who works for Bank S?
(a) Clerk (b) Agriculture Officer eer
Sunday not necessarily in the same order. They work in three
different shifts I, II and III with at least two of them in each shift.

5.
(c) Terminal Operator (d) Forex Officer
(e) None of these
Which of the following combinations of person, profession ing
R works in shift II and his day off is not Sunday. P's day off
is Tuesday and he does not work in the same shift with either Q or
W. None of those who work in shift I has day off either on
and bank is correct?
(a) A - Forex Officer - M
(b) D - Clerk - L .ne
Wednesday or on Friday. V works with only T in shift III. S's day
off is Sunday. V's day off is immediate next day of that of R's day
off. T's day off is not on Wednesday. W's day off is not on the
(c) F - Agriculture Officer - Q
(d) B - Agriculture Officer - S
(e) None of these
the same shift with R and his day off is not on Thursday.
11. Which of the following is W's day off?
(a) Tuesday (b) Monday
t
previous day of P's day off. S works in shift I. Q does not work in

(c) Saturday
DIRECTIONS (Q.6-10) : Study the following information to (d) Data inadequate (e) None of these
answer the given questions. 12. Which of the following is R's day off?
Each of seven plays viz. P, Q, R, S, T, V and W are scheduled to be (a) Friday (b) Thursday (c) Tuesday
staged on a different day of a week starting from Monday and (d) Wednesday (e) None of these
ending on Sunday of the same week. Play V is scheduled on 13. Which of the following groups of friends work in shift II?
Thursday. Two plays are scheduled to be held between Play V (a) RP (b) RV (c) QWS
and Play P. Only one play is scheduled between Play T and Play S. (d) Data inadequate (e) None of these
Play T is not scheduled on the day immediately before or 14. Which of the following is Q's day off?
immediately after the day when Play V is scheduled. Play R is (a) Friday (b) Wednesday (c) Thursday
scheduled the day immediately before the day when Play W is (d) Monday (e) None of these
scheduled. Play S is not scheduled after Play Q. 15. Which of the following groups of friends work in shift I?
6. How many plays are scheduled to be staged between Play R (a) RV (b) RP (c) QWS
and Play S? (d) Data inadequate (e) None of these
1. a b c d e 2. a b c d e 3. a b c d e 4. a b c d e 5. a b c d e
RESPONSE 6. a b c d e 7. a b c d e 8. a b c d e 9. a b c d e 10. a b c d e
GRID 11. a b c d e 12. a b c d e 13. a b c d e 14. a b c d e 15. a b c d e

Downloaded From : www.EasyEngineering.net


Downloaded From : www.EasyEngineering.net

22 SPEED TEST 11
DIRECTIONS (Q.16-20) : Study the following information 21. Which of the following groups of employees work in
carefully and answer the questions given below: Administration department?
(a) EGH (b) AF (c) BCD
(i) A, B, C, D, E, F, G and H are eight students, each having a (d) BGD (e) Data inadequate
different height 22. In which department does E work?
(ii) D is shorter than A but taller than G. (a) Personnel (b) Marketing
(iii) E is taller than H but shorter than C. (c) Administration (d) Data inadequate
(iv) B is shorter than D but taller than F. (e) None of these
(v) C is shorter than G. 23. Which of the following combinations of employees
(vi) G is not as tall as F. department-favourite sport is correct?
16. Which of the following is definitely false? (a) E-Administration-Cricket
(a) G is shorter than F (b) C is shorter than F (b) F-Personnel-Lawn Tennis
(c) F is taller than C. (d) B is taller than E. (c) H-Marketing-Lawn Tennis
(e) All are true (d) B-Administration-Table Tennis
17. If another student J, who is taller than E but shorter than G, (e) None of these
is added to the group, which of the following will be definitely 24. What is E's favourite sport?
true? (a) Cricket (b) Badminton (c) Basketball

ww
(a) C and J are of the same height
(b) J is shorter than D.
(c) J is shorter than H.
25.
(d) Lawn Tennis (e) None of these
What is G's favourite sport?
(a) Cricket (b) Badminton (c) Basketball

(e) None of these w.E


(d) J is taller than A.

18. Which of the following will definitely be the third from top
(d) Lawn Tennis (e) None of these
DIRECTIONS (Qs. 26-30): Study the following information
carefully and answer the given questions.

of height?
(a) B (b) F asy
when the eight students are arranged in descending order

(c) G
P, Q, R, S, T, V and W are seven friends, all of a different height
and fatness.
(i) Q is the thinnest and tallest among them.
(d) B or G (e) Cannot be determined
19. How many of them are definitely shorter than F? En (ii) S is not as short as T, but shorter than R.
(iii) W is not as fat as R, but fatter than V.
(a) Three (b) Four
(d) Data inadequate (e) None of these
(c) Five

20. Which of the following is redundant to answer all the above gin
(iv) R and T are taller than W but shorter than R
(v) V is fatter than T and the shorter among them.
questions?
(a) (ii) only 26.
eer
(vi) R is third among them in fatness in descending order.
Who is the fattest among them?
(a) P (b) S
(b) (ii) and (iii) only
(c) (iii) and (iv) only
(d) (ii) and (v) only
(c) Either P or S
(e) None of these ing
(d) Either T or S

(e) All are necessary to answer the above questions. 27.


.ne
Which of the given statements is not required to find out
the thickest among them?
DIRECTIONS (Q.21-Q.25) : Study the following information
carefully and answer the questions given below:
A, B, C, D, E, F, G and H are eight employees of an organization
working in three departments, viz Personnel, Administration and 28.
(a) (i)
(c) (iii) and (iv)
(e) None of these
(b) (vi)
(d) (v)
t
If they are made to stand in ascending order of their heights,
who will come in the middle?
Marketing with not more than three of them in any department.
Each of them has a different choice of sports from football, cricket, (a) R (b) S (c) T
volleyball, badminton, lawn tennis, basketball, hockey and table (d) Data inadequate (e) None of these
tennis, not necessarily in the same order. 29. If they are made to stand in ascending order of their fatness,
D works in Administration and does not like either football or who will be the second from the last?
cricket. F works in Personnel with only A, who likes table tennis. (a) S (b) V (c) W
E and H do not work in the same department as D. C likes hockey (d) Data inadequate (e) None of these
and does not work in Marketing. G does not work in Administration 30. Who obtained the same place in height and fatness among
and does not like either cricket or badminton. One of those who them when arranged in ascending order of their height and
work in Administration likes football. The one who likes volleyball fatness?
works in Personnel. None of those who work in Administration (a) P (b) Q (c) R
likes either badminton or lawn tennis. H does not like cricket. (d) S (e) None of these

16. a b c d e 17. a b c d e 18. a b c d e 19. a b c d e 20. a b c d e


RESPONSE
21. a b c d e 22. a b c d e 23. a b c d e 24. a b c d e 25. a b c d e
GRID 26. a b c d e 27. a b c d e 28. a b c d e 29. a b c d e 30. a b c d e

Downloaded From : www.EasyEngineering.net


Downloaded From : www.EasyEngineering.net

Venn Diagrams
1
122
81
0
Max. Marks : 30 No. of Qs. 30 Time : 20 min. Date : ........./......../. ..............
1. The diagram represent the student who are singers, dancers Rectangle represents males
and poets. Triangle represents educated
Circle represents urban
Singers P Dancers
Q Square represents civil servants
V
3. Who among the following is an educated male who is not an

ww T
U

S
R urban resident?
(a) 4
(d) 9
(b) 5
(e) None of these
(c) 11

w.E Poets
Study the diagram and identify the region which represent
the students who are both poets and singers but not dancer.
4. Who among the following is neither a civil servant nor
educated but is urban and not a male?
(a) 2 (b) 3 (c) 6
(a) P + T + S
(c) T + V + R + S
(e) None of these
(b) T
(d) P + T + U + S
asy 5.
(d) 10 (e) None of these
Who among the following is a female urban resident and

2. School children
En also a civil servant?
(a) 6
(d) 13
(b) 7
(e) None of these
(c) 10
a

d c
b
gin
6. Who among the following is an educated male who hails
from urban, a civil servants?

e
f
Artist (a) 6
(d) 13
eer (b) 7
(e) None of these
(c) 10

Singers
7.
male?
(a) 2 (b) 3 ing
Who among the following is uneducated and also an urban

(c) 11
Above diagram represents school children, artist and singers.
Study the diagram and identify the region. Which represents 8.
(d) 12 (e) None of these
.ne
Who among the following is only a civil servant but neither
those school children who are artist not singers.
(a) a
(c) f
(e) None of these
(b) b
(d) e
9.
a male or urban oriented and uneducated?
(a) 7
(d) 14
(b) 8
(e) None of these
(c) 9

Who among the following is a male urban oriented and also


t
DIRECTIONS (Qs. 3-9) : The following question are based on a civil servant but not educated?
the diagram given below. (a) 13 (b) 12 (c) 6
(d) 10 (e) None of these
3 10 7 10. Which of the following diagram represents the relationship
among sun, moon and stars?
8 12 6 13

4
9 11 14

5
(a) (b) (c) (d) (e)

RESPONSE 1. a b c d e 2. a b c d e 3. a b c d e 4. a b c d e 5. a b c d e

GRID 6. a b c d e 7. a b c d e 8. a b c d e 9. a b c d e 10. a b c d e

Downloaded From : www.EasyEngineering.net


Downloaded From : www.EasyEngineering.net

24 SPEED TEST 12
DIRECTIONS (Qs. 11-14) : Refer to the following Venn DIRECTIONS (Qs. 15-30) : In each of these questions, three
diagram : words are related in some way. The relationship among the words
in question can best represents by one of the five diagram.
HISTORY

16 GEOGRAPHY
(a) (b)
14 13 12
9 18 20 18 9
13 16
15 (c) (d)
MATHEMATICS
19 SCIENCE

11. The number of students who took any three of the above (e)

ww
subjects was
(a) 62 (b) 63 15. Teachers, college, students.

12.
(c) 64
(e) None of these
w.E
(d) 66

The number of students in total, who took History or


16.
17.
18.
19.
Mothers, Homo sapiens, Woman
Cabinet, Home Minister, Minister.
Parrots, Birds, Mice.
Professors, Researchers, Scientists.
Mathematics or Science, was
(a) 183
(c) 424
(b) 190
(d) 430
asy 20.
21.
Men, Rodents, Living beings.
Parents, Mothers, Fathers.

13.
(e) None of these
The number of students who took both History and En 22.
23.
24.
Nitrogen, Ice, Air.
Musicians, Singers, Women
Elephants, Carnivore, Tiger.
Geography among other subjects was
(a) 62 (b) 63 gin
25.
26.
Fish, Herring, Animal living in water
Hospital, Nurse, Patient.
(c) 65
(e) None of these
(d) 66 27.
28.
29. eer
Nose, Hand, Body.
Rings, Ornaments, Diamond Rings.
Furniture, Table, Books.
14. Which subject was taken by the largest number of students?
(a) Mathematics (b) Science
30.
ing
Indoor games, Chess, Table tennis.

(c) Geography
(e) None of these
(d) History
.ne
t

11. a b c d e 12. a b c d e 13. a b c d e 14. a b c d e 15. a b c d e


RESPONSE 16. a b c d e 17. a b c d e 18. a b c d e 19. a b c d e 20. a b c d e

GRID 21. a b c d e 22. a b c d e 23. a b c d e 24. a b c d e 25. a b c d e


26. a b c d e 27. a b c d e 28. a b c d e 29. a b c d e 30. a b c d e

Downloaded From : www.EasyEngineering.net


Downloaded From : www.EasyEngineering.net

Syllogism - I 13
Max. Marks : 30 No. of Qs. 30 Time : 20 min. Date : ........./......../. ..............
DIRECTIONS (Q.1-5) : In each of the questions below are given DIRECTIONS (6 - 10): In each question below arc two statements
three statements followed by two conclusions numbered I and II. followed by two conclusions numbered I and II. You have to take
You have to take the given statements to be true even if they seem the two given statements to be true even if they seem to be at
to be at variance from commonly known facts. Read both of the variance from commonly known facts and then decide which of
conclusions and then decide which of the given conclusions the given conclusions logically follows from the given statements
logically follows from the given statements disregarding commonly disregarding commonly known facts.
known facts.
1.
ww
Statements: Some phones are computers.
All computers are radios.
All radios are televisions.
Give answer (a) if only conclusion 1 follows.
Give answer (b) if only conclusion II follows.
Give answer (c) if either conclusion I or conclusion II follows.

(a) None follows


w.E
Conclusions: I. All televisions are computers.
II. Some radios are phones.
(b) Only I follows
Give answer (d) if neither conclusion I nor conclusion II follows.
Give answer (e) if both conclusions I and II follow.
6. Statements : Some windows arc grills.
All glasses are grills.

2.
(c) Only II follows
(e) None of these
Statements: All rings are fingers. asy
(d) Both I and II follow

7.
Conclusions : I. All grills are windows.
II. At least some grills are glasses.
Statements : Some painters are artists. Some dancers are painters.
Some ears are fingers.
All ears are necklaces.
Conclusions: I. Some necklaces are fingers. En Conclusions : I. All artists are dancers.
II. All painters are dancers.
II. Some necklaces are rings.
(a) None follows (b) Only I follows gin
8. Statements : All cabins are rooms.
All rooms are buildings.
Conclusions: I. All buildings are rooms.

3.
(c) Only II follows
(e) None of these
(d) Both I and II follow

Statements: Some bottles are cups. 9. eer II. All cabins are buildings.
Statements : All rings are necklaces.
Some cups are plates.
No spoon is a plate.
Conclusions: I. Some spoons are bottles. ing
No necklace is a bracelet.
Conclusions: I. No ring is a bracelet.
II. All necklaces are rings.
II. No bottle is a spoon.
(a) None follows
(c) Only II follows
(b) Only I follows
(d) Either I or II follow
10. Statements : All hands are arms.
Some hands are muscles.
Conclusions: I. Some muscles are arms. .ne
4.
(e) None of these
Statements: All pens are erasers.
Some erasers are sharpeners.
Some sharpeners are staples
II. All muscles are arms.
DIRECTIONS (Qs. 11-15): In each question below are two
statements followed by two conclusions numbered I and II. You
t
have to take the two given statements to be true even if they seem
Conclusions: I. Some sharpeners are pens.
to be at variance from commonly known facts and then decide
II. Some staples are erasers.
which of the given conclusions logically follows from the given
(a) None follows (b) Only I follows
statements disregarding commonly known facts. Give answer
(c) Only II follows (d) Both I and II follow
(e) None of these (a) if only conclusion I follows.
5. Statements: All hills are trees. (b) if only conclusion II follows.
All trees are jungles. (c) if either conclusion I or II follows.
All jungles are bushes. (d) if neither conclusion I nor II follows.
Conclusions: I. All trees are bushes. (e) if both conclusions I and II follow.
II. Some jungles are hills. 11. Statements: No holiday is a vacation.
(a) None follows (b) Only I follows Some vacations are trips.
(c) Only II follows (d) Both I and II follow Conclusions: I. No trip is a holiday.
(e) None of these II. Some holidays are definitely not trips.
1. a b c d e 2. a b c d e 3. a b c d e 4. a b c d e 5. a b c d e
RESPONSE 6. a c d e 7. a c d e 8. a c d e 9. a c d e 10. a c d e
b b b b b
GRID 11. a b c d e

Downloaded From : www.EasyEngineering.net


Downloaded From : www.EasyEngineering.net

26 SPEED TEST 13
12. Statements: Some kites are birds. Give answer (c) if either conclusion I or conclusion II follows;
No kite is an aeroplane. Give answer (d) if neither conclusion I nor conclusion II follows;
Conclusions: I. All aeroplanes are birds. Give answer (e) if both conclusion I and conclusion II follow.
II. Some birds are definitely not kites 21. Statements : Some exams are tests. No exam is a question.
13. Statements: All metals are plastics. Conclusions : I. No question is a test.
All plastics are fibres. II. Some tests are definitely not exams.
Conclusions: I. Atleast some fibres are metals. 22. Statements : All forces are energies. All energies are
II. Some metals are not fibres. powers. No power is heat.
14. Statements: All roads are streets. Conclusions : I. Some forces are definitely not powers.
No street is a highway. II. No heat is force.
Conclusions: I. No highway is a road. 23. Statements : All forces are energies. All energies are
II. All streets are roads. powers. No power is heat.
15. Statements: Some animals are plants. Conclusions : I. No energy is heat.
All plants are rocks. II. Some forces being heat is a possibility.
Conclusions: I. All plants are animals. 24. Statements : No note is a coin. Some coins are metals.
II. Atleast some rocks are animals. All plastics are notes.
DIRECTIONS (Q.16-20) : In each questions below are two / three Conclusions : I. No coin is plastic.
statements followed by two conclusions numbered I and II. You II. All plastics being metals is a possibility.

ww
have to take the two / three given statements to be true even if they
seem to be at variance from commonly known facts and then decide
which of the given conclusions logically follows from the given
25. Statements : No note is a coin. Some coins are metals.
All plastics are notes.
Conclusions : I. No metal is plastic.

w.E
statements disregarding commonly known facts. Give answer
(a) if only conclusion I follows.
(b) if only conclusion II follows.
II. All notes are plastics.
26. Statements : Some symbols are figures. All symbols are
graphics.
(c) if either conclusion I or II follows.
(d) if neither conclusion I nor II follows.
(e) if both conclusions I and II follows. asy No graphic is a picture.
Conclusions : I. Some graphics are figures.
II. No symbol is a picture.
16. Statements: No holiday is a vacation.
Some vacations are trips.
Conclusions: I. No trip is a holiday. En 27. Statements : All vacancies are jobs. Some jobs are
occupations.
Conclusions : I. All vacancies are occupations.
II. Some holidays are definitely not trips.
17. Statements: Some kites are birds.
No kite is an aeroplane. gin II. All occupations being vacancies is a
possibility.

Conclusions: I. All aeroplanes are birds.


II. Some birds are definitely not kites. eer
DIRECTIONS (Qs. 28-30) : In each question below are two/three
statements followed by two conclusions numbered I and II. You
have to take the two/three given statements to be true even if
18. Statements: All metals are plastics.
All plastics are fibres.
Conclusions: I. At least some fibres are metals ing
they seem to be at variance from commonly known facts and then
decide which of the given conclusions logically follows from the
given statements disregarding commonly known facts.
II. Some metals are not fibres.
19. Statements: Some animals are plants.
All plants are rocks.
Give answer (a) if only conclusion I follows
Give answer (b) if only conclusion II follows.
.ne
Conclusions: I. All plants are animals.
II. Atleast some rocks are animals.
20. Statements: Some institutes are banks.
All institutes are academies.
Give answer (e) if both conclusion I and conclusion II follow.
(Qs. 28-29) :
Statements : All gliders are parachutes.
t
Give answer (c) if either conclusion I or conclusion II follows.
Give answer (d) if neither conclusion I nor conclusion II follows.

All academies are schools.


Conclusions: I. All banks can never be schools. No parachute is an airplane.
II. Any bank which is an institute in a school. All airplanes are helicopters.
28. Conclusions : I. No glider is an airplane.
DIRECTIONS (Qs. 21-27) : In each group of questions below are II.All gliders being helicopters is a possibility.
two/three statements followed by two conclusions numbered I and 29. Conclusions : I. No helicopter is a glider.
II. You have to take the given statements to be true even if they II. All parachutes being helicopters is a
seem to be at variance from commonly known facts and then decide possibility.
which of the given conclusions logically follows from the two/three 30. Statements : Some mails are chats.
statements disregarding commonly known facts. All updates are chats.
Give answer (a) if only conclusion I follows; Conclusions : I. All mails being updates is a possibility.
Give answer (b) if only conclusion II follows; II. No update is a mail.
12. a b c d e 13. a b c d e 14. a b c d e 15. a b c d e 16. a b c d e
RESPONSE 17. a b c d e 18. a b c d e 19. a b c d e 20. a b c d e 21. a b c d e

GRID 22. a b c d e 23. a b c d e 24. a b c d e 25. a b c d e 26. a b c d e


27. a b c d e 28. a b c d e 29. a b c d e 30. a b c d e

Downloaded From : www.EasyEngineering.net


Downloaded From : www.EasyEngineering.net

Syllogism - II 14
Max. Marks : 25 No. of Qs. 25 Time : 18 min. Date : ........./......../. ..............
DIRECTIONS (Q.1-5) : In each of the questions below are given 4. Statements:
three statements followed by three conclusions numbered I , II All trees are gardens.
and III. You have to take the given statements to be true even if All gardens are stones.
they seem to be at variance from commonly known facts. Read All stones are fences.
all the conclusions and then decide which of the given Conclusions:
conclusions logically follows from the given statements I. Some fences are gardens.

1. ww
disregarding commonly known facts.
Statements:
Some flowers are bins.
II. All gardens are stones.
III. Some stones are trees.
(a) Only I and II follows (b) Only I and III follows

w.E
Some bins are handles
All handles are sticks.
Conclusions: 5.
(c) Only II or III follow (d) All follow
(e) None of these
Statements:
I. Some sticks are bins.
II. Some handles are flowers.
III. Some sticks are flowers.
asy All books are leaves.
Some leaves are jungles.
No jungle is box.
(a) Only II follows (b) Only III follows
(c) Only I and II follow (d) Only I and III follow En Conclusions:
I. Some jungles are books.

2.
(e) None of these
Statements:
Some towers are windows.
gin II. No book is box.
III. Some leaves are boxes.
(a) None follows (b) Only I follows
All windows are houses.
Some houses are temples eer
(c) Only II follows
(e) Only I and II follow
(d) Only III follows

Conclusions:
I. Some towers are temples.
II. Some houses are towers.
ing
DIRECTIONS (Q.6-25) : In each questions below are given
two/three statements followed by two conclusions numbered I
III. Some temples are windows.
(a) Only I follows (b) Only II follows .ne
and II. You have to take the given statements to be true even if
they seem to be at variance with commonly known facts. Read all

3.
(c) Only III follows (d) Only I and II follow
(e) None of these
Statements:
Some walls are doors.
the conclusions and then decide which of the given conclusions
logically follows from the given statements disregarding
commonly known facts. Give answer.
(a) if only conclusion I follows.
t
Some doors are cots. (b) if only conclusion II follows.
Some cots are chairs. (c) if either conclusion I or II follows.
Conclusions: (d) if neither conclusion I nor II follows.
I. Some chairs are doors. (e) if both conclusions I and II follows.
II. Some cots are walls. 6. Statements:
III. No chair is door. Some toys are desks.
(a) Only II follows Some desks are pens.
(b) Only III follows All pens are rods.
(c) Only either I or III follows Conclusions:
(d) Only I follows I. Some rods are toys.
(e) None of these II. Some pens are toys.

RESPONSE 1. a b c d e 2. a b c d e 3. a b c d e 4. a b c d e 5. a b c d e

GRID 6. a b c d e

Downloaded From : www.EasyEngineering.net


Downloaded From : www.EasyEngineering.net

28 SPEED TEST 14
7. Statements: 15. Conclusions:
Some table are huts. I. No card is a paper.
No hut is ring. II. Some papers are cards.
All rings are bangles. 16. Conclusions:
Conclusions: I. All cards being papers is a possibility.
I. Some bangles are tables. II. All boards being papers is a possibility.
II. No bangles is table. 17. Statements: All rings are circles.
8. Statements: All squares are rings.
Some chairs are rooms. No ellipse is a circle.
All rooms are trees. Conclusions: I. Some, rings being ellipses is a possibility.
All trees are poles. : II. At least some circles are squares.
Conclusions: 18. Statements : No house is an apartment.
I. Some poles are chairs. Some bungalows are apartments.
II. Some trees are chairs. Conclusions: I. No house is a bungalow.
9-10. Statements: II. All bungalows are houses.
All buildings are houses.

ww
No house is an apartment.
All apartments are flats.
19. Statements: Some gases are liquids.
All liquids are water.
Conclusions: I. All gases being water is a possibility.
9. Conclusions:

w.E
I. No flat is a house.
II. No building is an apartment.
10. Conclusions:
II. All such gases which are not water can
never be liquids.
20. Statements: All minutes are seconds.

asy
I. All buildings being flats is a possibility.
II. All apartments being building is a possibility.
All seconds are hours.
No second is a day.
Conclusions: I. No day is an hour.
11-12.Statements:
Some oceans are seas.
En II. At least some hours are minutes.
(21-22): Statements: Some teachers are professors.
All oceans are rivers.
No river is a canal.
11. Conclusions: gin Some lecturers are teachers.
21. Conclusions: I. All teachers as well as professors being
I. All rivers can never be oceans.
II. All canals being oceans is a possibility. eer lecturers is a possibility.
II. All those teachers who are lecturers are
12. Conclusions:
I. No ocean is a canal.
II. At least some seas are rivers. ing
also professors.
22. Conclusions: I. No professor is a lecturer.
II. All lecturers being professors is a
13-14. Statements:
No day is night. (23-24):
possibility.
.ne
All nights are noon.
No noon is an evening.
13. Conclusions:
I. No day is noon.
Statements: Some flowers are red.

23. Conclusions:
Some roses are flowers.

I. All those flowers which are roses are red.


t
II. No day is an evening. II. No rose is red.
14. Conclusions: 24. Conclusions :
I. No evening are nights. I. All roses being red is a possibility.
II. All days being noon is a possibility. II. Some flowers can never be roses.
25. Statements: Some hills are mountains.
15-16. Statements:
All mountains are high.
Some papers are boards
Conclusions: I. All hills being high is a possibility.
No board is a card.
II. Some mountains can never be hills.

7. a b c d e 8. a b c d e 9. a b c d e 10. a b c d e 11. a b c d e
RESPONSE 12. a b c d e 13. a b c d e 14. a b c d e 15. a b c d e 16. a b c d e

GRID 17. a b c d e 18. a b c d e 19. a b c d e 20. a b c d e 21. a b c d e


22. a b c d e 23. a b c d e 24. a b c d e 25. a b c d e

Downloaded From : www.EasyEngineering.net


Downloaded From : www.EasyEngineering.net

Symbols & Codes 15


Max. Marks : 25 No. of Qs. 25 Time : 20 min. Date : ........./......../. ..............
DIRECTIONS (Qs.1-5): In each question below is given a group
Number/
of letters followed by four combinations of digits/symbols 9 4 & 5 % 3 # 7 6 @ 8 + 2 $
Symbols
numbered (a), (b), (c) and (d). You have to find out which of the
Letter
combinations correctly represents the group of letters based on X P J H B D K F S T N G R L
Codes
the coding system and the conditions given below and mark the
number of that combination as your answer. If none of the
Conditions:

ww
combinations correctly represents the group of letters, mark (e)
i.e. ‘None of these’ as your answer.
(i) If the first element is a symbol and the last element is a
number, then the codes for both are to be interchanged.
(ii) If both the first and last elements are symbols, then the

w.E
Letters P M A E J K D R W H I U T F
Digits/symbols last element is to be coded as the code for the first
4 $ 1 2 3 # 5 @ © 6 % d 7 9
Conditions element.
(iii) If the group of elements contains only one symbol,
(i)

asy
If the first letter is a consonant and the last letter is a vowel,
the codes of both these are to be interchanged.
(ii) If both the first and the last letters are consonants both these 6. 28%956
then that symbols is to be coded as A.

are to be coded as per the code of the last letter.

En
(iii) If the first letter is vowel and the last letter is a consonant
both these are to be coded as ‘ ’
(a) RNBXHS
(c) RNBXSH
(b) RNAXSH
(d) RNAXHS
Note: All the remaining letters are to be coded as per their original
codes. gin
7.
(e) RNASHX
©62+74
(a) PSRGFT (b) TSRFGP
1. ERWHKA
(a) 2 @ © 6# 1
(d) 2 @ © 6# 2
(b) 1 @ © 6# 2
(e) None of these
(c) 1 @ © 6 # I (c) PSRFGT
(e) TSRGFP eer (d) PRSGFT

2. MPEKDU
(a) $ 42 #5 d (b) $42 #5$ (c) d 4 2# 5 d
8. +5963%
(a) GHXSDG ing (b) GSHXDB

3.
(d)
TMEIUF
d 42 5# $

(a) 7$2% d 9
(e) None of these

(b) 7$2% d 7 (c) 9$2% d 7 9.


(c) GHXDSG
(e) GXHSDG
.ne
(d) GHSXDB

In a certain code MODE is written as #8%6 and DEAF is

4.
(d) 9$2% d 9
JTAERI
(a) % 7 1 2@ 3 (b) 3712@3
(e) None of these

(d) %7 12 @ % (e) None of these


(c) 71 2@
written as %67$. How is FOAM written in that code?
(a) $87#
(d) $87%
(b) $#7%
(e) None of these
(c) #87%

10. In a certain code WEAK is written as 5%9$ and WHEN is


t
5. UKTMIH written as 5*%7. How HANK written in that code?
(a) # 7$ % 6 (b) 6 # 7$ % d (c) # 7 $ % (a) *9$7 (b) 9*$7 (c) $97*
(d) 7 #$ % 6 (e) None of these (d) 9*7$ (e) None of these
DIRECTIONS (Qs.11-15) : In each of these questions a group of
DIRECTIONS (Qs.6-8): In each question below is given a group letters is given followed by four combinations of number/symbol
of numbers/symbols followed by five combinations of letter codes numbered (a), (b), (c) & (d). Letters are to be coded as per the
numbered (a), (b), (c), (d) and (e). You have to find out which of scheme and conditions given below. You have to find out the
the combinations correctly represents the group of numbers/ serial number of the combination, which represents the letter group.
symbols based on the following coding system and the conditions Serial number of that combinations is your answer. If none of the
and mark the number of that combination as your answer. combinations is correct, your answer is (e) i.e. 'None of these'.

RESPONSE 1. a b c d e 2. a b c d e 3. a b c d e 4. a b c d e 5. a b c d e

GRID 6. a b c d e 7. a b c d e 8. a b c d e 9. a b c d e 10. a b c d e

Downloaded From : www.EasyEngineering.net


Downloaded From : www.EasyEngineering.net

30 SPEED TEST 15
19. 592476
Letters D K M B I N P R J A L S E Q G
(a) H COQ$%T (b) Q$% oT
Number /
% 3 7 H 4 @ $ 1 8 5 # 9 2 £ 6 (c) H©Q$%OO (d) CCQ$%o©
Symbol Code
(e) None of these
Conditions 20. 468910
(i) If the first letter is a consonant and the last a vowel, (l) $HJ©KL (b) LHJ@K$
both are to be coded as the code of the vowel. (c) *HJ@K$ (d) $HJ@K*
(ii) If the first letter is a vowel and the last a consonant, the (e) None of these
codes for the first and the last are to be interchanged.
(iii) If no vowel is present in the group of letters, the second DIRECTIONS (Qs. 21-24) : In each of the questions given below,
and the fifth letters are to be coded as ã . a group of digits is given followed by four combinations of letters/
11. KQAPJE symbols numbered (a), (b),(c) and(d). You have to find out which
(a) 3£5$82 (b) 3£58$2 (c) 2£5$82 of the four combinations correctly represents the group of digits
(d) 2£5$83 (e) None of these based on the letter/symbol codes and the conditions given below.
12. EMANRB If none of the four combinations represents the group of digits
(a) *75@12 (b) 275@1* (c) ã 75@2ã correctly, give (e) i.e. `None of these' as the answer.
(d) *75@1* (e) None of these
13. JAQDKP
(a) 85£%38
(d) $5£3%8
ww (b) $5£%3$
(e) None of these
(c) $5£%38
Digit
Code
2
B
8
=
3
T
9
@
4
K
7
$ ©
6 5
P
1
C

14. QDBGRM
(a) £%*617
(d) % £*61ã
w.E (b) $ã *6ã 7
(e) None of these
(c) £%*167
Conditions:
(i) If the first digit is odd and last digit is even, the codes
for the first and the last digits are to be interchanged.
15. IKQLMS
(a) 43£#74
(d) 93£#74
(b) ã 3£#7ã
(e) None of these asy (c) 4£3#74
(ii) If the first as well as the last digit is even, both are to
be coded by the code for last digit.
(iii) If the first as well as the last digit is odd, both are to
DIRECTIONS (Qs. 16-20): In each of these questions, a group
of digits is given followed by four combinations of letters and En be coded as X.
(iv) If the first digit is even and last digit is odd, both are
symbols numbered (a), (b), (c) and (d). The group of digits is to be
coded as per the scheme and conditions given below. The serial
number of the combination which correctly represents the group
gin
21. 2976581
to be coded by the code for the first-digit.

(a) B@$©P = B (b) C@$©P = C


of digits is your answer. If none of the four combinations is correct,
your answer is (e) i.e., 'None of these'.
22.
(c) B@$©P = C
eer
(e) None of these
7269534
(d) C@S©P = B

Digits 5
Code T J
8 4 3 6 2 9
$ # H Q @ L % K
0 7 1 (a) $BC@PTK
(c) $B©@PT$ ing
(b) KB©@PT$
(d) KB©@P=$
Conditions:
(i) If the first as well as the last digit is odd, both are to be
coded as ©.
23.
(e) None of these
8135246
.ne
(ii) If the first as well as the last digit is even, their codes are to
be swapped.
(iii) If '0' is the last digit, it is to be coded as *.
16. 270514 24.
(a) = CTPBK =
(c) ©CTPBK©
(e) None of these
4352718
(b) ©CTP = K©
(d) CTPB$ =
t
(a) ©%LTK© (b) $%LTKQ (a) XTPB$CX (b) KTPB$C=
(c) Q%LTK$ (d) $%*TKQ (c) =TPB$CK (d) KTP$CK
(e) None of these (e) None of these
17. 364279 25. if 1 is coded as$, 5 is coded as %, 9 is coded as , 3 is coded
(a) ©H$Q%© (b) #H$Q%@ as +, 7 is coded as # and 4 is coded as?, what will be the
(c) ©H$Q%# (d) #H$Q%© correct code of the number 435971?
(e) None of these
(a) ? + % # $ (b) ? + % $ #
18. 875306
(a) J%T#Ll1 (b) H%T#LH (c) ? + % # $ (d) $ # % + ?
(c) H%T#LJ (d) J% oT#LJ (e) None of these
(e) None of these
11. a b c d e 12. a b c d e 13. a b c d e 14. a b c d e 15. a b c d e
RESPONSE 16. a b c d e 17. a b c d e 18. a b c d e 19. a b c d e 20. a b c d e
GRID 21. a b c d e 22. a b c d e 23. a b c d e 24. a b c d e 25. a b c d e

Downloaded From : www.EasyEngineering.net


Downloaded From : www.EasyEngineering.net

Alpha Numeric
Sequence Puzzle
16
16
Max. Marks : 30 No. of Qs. 30 Time : 20 min. Date : ........./......../. ..............
DIRECTIONS (Q. 1-5) : Answers the questions given below 8. If all the symbols are dropped from the arrangement, which
referring to the following arrangement: of the following will be the twelfth from the left end
J * R 3 PL2 # I N 7 O C @ K 5 D = M $ 6 B< AQ 4 ? (c) S
1. Four of the following five are alike in a certain way as regards (a) 79
(d) (b) 2None of these
(e)
their position in the above arrangement and so form a group. 9. Four of the following five are alike in a certain way based on
Which is the one that does not belong to that group? their positions in the above arrangement and so form a group.

2.
ww
(a) 2 3 #
(d) # P I
(b) O I C
(e) B $ <
(c) K O 5

What will come in the place of the question mark (?) in the
Which is the one that does not belong to the group?
(a) L$8
(d) 6%G
(b) AKM
(e) JD©
(c) @!F

(a) D K C
w.E
following series based on the above arrangement?
PRJ#L3712@ ON?
(b) 5 @ O (c) D K @
10. What should come in place of the question mark (?) in the
following series based on the above arrangement >
F3U %IT L$ Ù ?

3.
(d) = 5 @ (e) None of these
asy
It the above series is re-arranged in the reverse order,
which will be the eleventh element to the left to the
(a) 927
(d) 2&A
(b) 7&A
(e) 27&
(c) 7AM

sixteenth element from the left end?


(a) J (b) 6 (c) B En DIRECTIONS (Qs. 11-18): Study the following arrangement of
letters/symbols and answer the questions given below:

4.
(d) < (e) None of these
How many such numbers are there in the above arrangement
each of which is immediately preceded by a consonant and
gin D F JT $ # PR ZQ * C MAB@ HK LS + ?
11. How many such symbols are there each of which is
immediately preceded by a symbol and immediately followed
not immediately followed by a symbol?
(a) None (b) Two (c) Four (a) One eer
by a letter?
(b) Two (c) Three

5.
(d) Three (e) None of these
How many such symbols are there in the above arrangement
each of which is immediately preceded by a number and
(d) Four

ing
(e) None of these
12. If the order of the first half of the arrangement is reversed
which of the following letters/symbols will be the fifth to the
immediately followed a consonant?
(a) One (b) Two (c) Three (a) * (b) Q .ne
left of the fifteenth letter/symbol from the left?
(c) T
(d) More than three (e) None

DIRECTIONS (Qs. 6-10) : Study the following arrangement of


consonants, vowels, numbers and symbols carefully and answer
(d) J (e) None of these
13. If all the symbols of the above sequence are denoted by 7
t
and each letter is denoted by 5, then what will be the sum of
all the elements of the sequence?
the questions given below:
(a) 142 (b) 138 (c) 132
H@ F ! 3 U 6 % G I T * PL 8 $ Ù9 S 2 7 &AM K + J © (d) 122 (e) None of these
D4#5&E 14. If all the symbols from the above sequence are dropped,
6. Which of the following is ninth to the right of the twentieth which letter will be seventh to the right of twelfth letter from
from the right end of the above arrangement ? the right?
(a) K (b) M (c) U (a) H (b) B (c) K
(d) A (e) None of these (d) A (e) None of these
7. How many such consonants are there in the above 15. Which of the following is related to ‘FT’ in the same way as
arrangement, each of which is immediately preceded by a ‘DJ’ is related to‘? S’ ?
symbol and also immediately followed by a symbol ? (a) L+ (b) KS (c) HL
(a) None (b) One (c) Two (d) + L (e) None of these
(d) Three (e) More than three
1. a b c d e 2. a b c d e 3. a b c d e 4. a b c d e 5. a b c d e
RESPONSE 6. a b c d e 7. a b c d e 8. a b c d e 9. a b c d e 10. a b c d e
GRID 11. a b c d e 12. a b c d e 13. a b c d e 14. a b c d e 15. a b c d e

Downloaded From : www.EasyEngineering.net


Downloaded From : www.EasyEngineering.net

32 SPEED TEST 16
16. How many such letters are there in the above sequence 23. If the order of the last fifteen elements in the above
each of which occupies the same position from the left in arrangement is reversed, which of the following will be the
the sequence as in the alphabet from left? ninth to the right of the eleventh element from. the left end?
(a) None (b) One (c) Two (a) G (b) % (c) 8
(d) Three (e) None of these (d) 3 (e) None of these
17. Four of the following five are alike on the basis of their 24. How many such consonants are there in the above
position in the above sequence and hence form a group. arrangement, each of which is immediately preceded by a
Which of the following does not belong to that group? symbol but not immediately followed by either a number or
(a) DJ ? (b) T # L (c) FT + a vowel?
(d) PZ @ (e) J # S (a) None (b) One (c) Two
18. Which of the following will be exactly midway between fifth (d) Three (e) More than three
element from the left and eighth element from the right? 25. Four of the following five are alike in a certain way based on
(a) C (b) * (c) Q their position in the above arrangement and so form a group.
(d) M (e) None of these Which is the one that does not belong to that group?
19. Select the combination of numbers so that letters arranged (a) A $ E (b) % V N (c) 2 F V
accordingly will form a meaningful word. (d) 4 K 1 (e) 6 Q ÷

ww
R A C E T
1 2 3 4 5
DIRECTIONS (Qs. 26-30): Study the following arrangement
carefully and answer the questions given below:
(a) 1, 2, 3, 4, 5
(c) 5, 2, 3, 4, 1
(e) None of these w.E (b) 3, 2, 1, 4, 5
(d) 5, 1, 2, 3, 4
M £ 5 T R E 3 $ PJ 1 7 D 1 2 NA4 F H 6 * U 9 # V B @ W
26. If the positions of the first fourteen characters of the above
arrangement are reversed, which of the following will be the
20.

VA R S T E asy
Select the combination of numbers so that the letters
arranged accordingly will form a meaningful word.
twenty-second from the right end?
(a) J
(d) 3
(b) I
(e) None of these
(c) P

(a) 2, 3, 1, 6, 4, 5
(c) 6, 3, 4, 5, 2, 1
(b) 4, 5, 2, 3, 1, 6
(d) 3, 2, 4, 5, 6, 1
En 27. How many such numbers are there in the above arrangement,
each of which is immediately preceded by a vowel and
(e) None of these
DIRECTIONS (Qs. 21-25): Study the following arrangement
gin immediately followed by a consonant?
(a) None (b) One (c) Two
carefully and answer the questions given below:
J 1 #P 4 E K 3 A D $ R U M 9 N 5 1 % T V * H2 ÷
F6G8QW
(d) Three

eer (e) More than three


28. What should come in place of the question mark (?) in the
series given below based on the above arrangement?
21. How many such numbers are there in the above arrangement,
each of which is either immediately preceded by or
R3 £ PIE ?AF I
(a) DNJ ing
(b) D21 (c) IN1
immediately followed by a vowel or both?
(a) None (b) One (c) Two
(d) N4D (e) None of these

.ne
29. How many such consonants are there in the above
(d) Three (e) More than three
22. Which of the following is exactly in the middle between the
tenth from the left and the eighth from the right end in the
above arrangement?
arrangement each of which is immediately preceded by a
symbol but not immediately followed by a number?
(a) None
(d) Three
(b) One
(e) More than three
(c) Two t
(a) M (b) N (c) 1 30. Which of the following is the fifth towards right of the
(d) 5 (e) None of these seventeenth from the right end?
(a) $ (b) 4 (c) 7
(d) A (e) None of these

16. a b c d e 17. a b c d e 18. a b c d e 19. a b c d e 20. a b c d e


RESPONSE 21. a c d e 22. a c d e 23. a c d e 24. a c d e 25. a c d e
b b b b b
GRID
26. a b c d e 27. a b c d e 28. a b c d e 29. a b c d e 30. a b c d e

Downloaded From : www.EasyEngineering.net


Downloaded From : www.EasyEngineering.net

Input-Output 17
17
Max. Marks : 23 No. of Qs. 23 Time : 18 min. Date : ........./......../. ..............
5. Input: paper dry 37 23 height call 62 51
DIRECTIONS (Qs. 1-5): Study the following information carefully
Which of the following steps will be the last but one?
and answer the given questions:
(a) V (b) IV (c) VI
A word and number arrangement machine when given an input
(d) III (e) None of these
line of words and numbers rearranges them following a particular
rule in each step. The following is an illustration of input and DIRECTIONS (Qs.6-10) : A word and number arrangement
rearrangement. (All numbers are two-digit numbers.) machine when given an input line of words and numbers rearranges

ww
Input : good for everything 19 37 26 all 65
Step I : all good for everything 19 37 26 65
Step II : all 65 good for everything 19 37 26
them following a particular rule in each step. The following is an
illustration of an input and rearrangement.
Input : 17 put show on 39 27 85 gold

w.E
Step III : all 65 everything good for 19 37 26
Step IV : all 65 everything 37 good for 19 26
Step V : all 65 everything 37 for good 19 26
Step VI : all 65 everything 37 for 26 good 19
Step I : show 17 put on 39 27 85 gold
Step II : show 85 17 put on 39 27 gold
Step III : show 85 put 17 on 39 27 gold

arrangement is reached. asy


and Step VI is the last step of the rearrangement as the desired
Step IV : show 85 put 39 17 on 27 gold
Step V : show 85 put 39 on 17 27 gold
Step VI : show 85 put 39 on 27 17 gold

En
As per the rules followed in the above steps, find out in each of
the following questions the appropriate step for the given input.
(All numbers are two-digit numbers.)
Step VII : show 85 put 39 on 27 gold 17
and step VII is the last step of the rearrangement of the above
1. Input: won 13 now 25 72 please go 47
How many steps will be required to complete the rearrangement? gin
input.
As per the rules followed in the above steps, find out in each of
the following questions the appropriate step for the given input.
(a) Four
(d) Three
2. Step III of an input is :
(b) Five
(e) None of these
(c) Six
6.
eer
Input : glass full 15 37 water now 85 67
Which of the following will be step VI of the above input?
car 81 desk 15 42 39 tall more
Which of the following will be Step VI?
(a) car 81 desk 42 39 15 tall more ing
(a) 85 now 67 full glass 15 37
(b) water 85 now 67 glass full 15 37
(c) water 85 now 67 glass 37 full 15
(b) car 81 desk 42 15 39 tall more
(c) car 81 desk 42 more 39 15 tall
(d) There will be no such step.
(e) None of these
.ne
(d) There will be no such step.
(e) None of these
3. Step II of an input is:
bell 53 town hall near 27 43 12
How many more steps will be required to complete the
7.

8.
the following steps will be the last but one?
(a) VIII
(d) VI
(b) IX
(e) None of these
(c) VII t
Step II of an input is: ultra 73 12 16 mail sort 39 kite. Which of

Step III of an input is: win 75 voice 15 39 store gap 26. Which
rearrangement? of the following is definitely the input?
(a) Five (b) Four (c) Six (a) voice 15 win 75 39 store gap 26
(d) Three (e) None of these (b) voice win 75 15 39 store gap 26
4. Step II of an input is (c) 15 75 win voice store gap 26
box 93 25 year end 41 32 value (d) Cannot be determined
Which of the following is definitely the input? (e) None of these
(a) 25 year end box 93 41 32 value 9. Step II of an input is: tube 83 49 34 garden flower rat 56. How
(b) 25 year end 93 41 32 value box many steps will be required to complete the rearrangement?
(c) 9325 box year end 41 32 value
(a) Four (b) Five (c) Six
(d) Cannot be determined
(d) Three (e) None of these
(e) None of these

1. a b c d e 2. a b c d e 3. a b c d e 4. a b c d e 5. a b c d e
RESPONSE 6. a b c d e 7. a b c d e 8. a b c d e 9. a b c d e
GRID

Downloaded From : www.EasyEngineering.net


Downloaded From : www.EasyEngineering.net

34 SPEED TEST 17
10. Input : hunt for 94 37 good 29 48 book. Input : 51 pour 32 start now 23 46 house
How many steps will be required to complete the Step I : 23 51 pour 32 start now 46 house
rearrangement? Step II : 23 start 51 pour 32 now 46 house
(a) Four (b) Five (c) Six Step III : 23 start 32 51 pour now 46 house
(d) Seven (e) None of these Step IV : 23 start 32 pour 51 now 46 house
DIRECTIONS (Q.11 to 15) : Study the following information to Step V : 23 start 32 pour 46 51 now house
answer the given questions: Step VI : 23 start 32 pour 46 now 51 house
and step VI is the last step of the rearrangement
A word and number arrangement machine when given an input
As per the rules followed in the above steps, find out in each of
line of words and numbers rearranges them following a particular
the following questions the appropriate steps for the given input.
rule. The following is an illustration of input and rearrangement.
16. Step II of an input is : 18 task bear cold dish 81 63 31
(All the numbers are two-digit numbers.)
How many more steps will be required to complete the
Input : sine 88 71 cos theta 14 56 gamma delta 26
rearrangement?
Step I : cos sine 71 theta 14 56 gamma delta 26 88
(a) Three (b) Four (c) Five
Step II : delta cos sine theta 14 56 gamma 26 88 71
(d) Six (e) None of these
Step III : gamma delta cos sine theta 14 26 88 71 56
17. Input : 72 59 37 go for picnic 24 journey
Step IV : sine gamma delta cos theta 14 88 71 56 26
How many steps will it take to complete the rearrangement?
Step V : theta sine gamma delta cos 88 71 56 26 14

ww
Step V is the last step of the rearrangement.
As per the rules followed in the above steps, find out in each of
the following questions the appropriate steps for the given input.
(a) Three
(d) Six
(b) Four
(e) None of these
18. Input : nice flower 24 12 costly height 41 56
(c) Five

Input for the questions:


w.E
Input : for 52 all 96 25 jam road 15 hut 73 bus stop 38 46 (All the
numbers given in the arrangement are two digit numbers).
Which of the following will be step III ?
(a) 12 nice 34 height flower costly 41 56
(b) 12 nice 34 height 41 flower costly 56
(c) 12 nice 34 flower costly height 41 56
right in step IV ?
(a) 15 (b) road asy
11. Which word/ number would be at 8th position from the

(c) hut
(d) 12 nice flower 34 costly height 41 56
(e) None of these
(d) jam (e) stop
12. Which step number would be the following output? Bus all
En 19. Step II of an input is : 16 victory 19 36 53 store lake town.
Which of the following will be step V ?
(a) 16 victory 19 town store 36 53 lake
for 52 25 jam road 15 hut stop 38 46 96 73.
(a) There will be no such step.
(b) III (c) II (d) V gin (b) 16 victory 19 town 36 store 53 lake
(c) 16 victory 19 town 36 53 store lake
(e) VI
13. Which of the following would be step VII?
(a) stop road jam hut for bus all 15 96 73 5246 38 25 eer
(d) There will be no such step
(e) None of these
20. Step III of an input is : 15 yes 29 ask for soap 42 37
(b) road jam hut for bus all stop 15 25 38 46 52 73 96
(c) stop road jam hut for bus all 96 73 52 46 38 25 15 ing
Which of the following is definitely the input?
(a) ask yes 29 15 for soap 42 37
(b) yes ask 15 29 for soap 42 37
(d) jam hut for bus all 25 road stop 15 96 73 52 46 38
(e) There will be no such step
14. Which word/number would be at 6th position from the left
(c) 29 15 yes ask for soap 42 37
(d) Cannot be determined .ne
in step V?
(a) 25
(d) all
(b) stop
(e) road
15. Which of the following would be step III?
(c) jam
(e) None of these
21. Input : milk pot 18 24 over goal 36 53
Which of the following steps will be the last but one?
(a) VI
(d) VIII
(b) V
(e) None of these
(c) VII
t
(a) hut for bus all 25 jam road 15 stop 38 96 73 52 46
22. Step III of an input is : 36 win 44 95 86 ultra box queen
(b) for bus all 25 jam road 15 hut 38 stop 96 46 73 52
How many more steps will be required to complete the
(c) hut for bus all jam road 15 stop 38 96 73 52 46 25
rearrangement?
(d) for bus all 25 jam road 15 hut stop 38 46 96 73 52
(a) Three (b) Four (c) Five
(e) None of these
(d) Six (e) None of these
DIRECTIONS (Q. 16 to 23) : Study the following information 23. Input : new 22 model 27 pump 38 11 join
carefully and answer the given questions: How many steps will be required to complete the
A word and number arrangement machine when given an input line rearrangement?
of words and numbers rearranges them following a particular rule in (a) Four (b) Five (c) Six
each step. The following is an illustration of input and rearrangement. (d) Seven (e) None of these

10. a b c d e 11. a b c d e 12. a b c d e 13. a b c d e 14. a b c d e


RESPONSE 15. a c d e 16. a c d e 17. a c d e 18. a c d e 19. a c d e
b b b b b
GRID
20. a b c d e 21. a b c d e 22. a b c d e 23. a b c d e

Downloaded From : www.EasyEngineering.net


Downloaded From : www.EasyEngineering.net

Mathematical
Operations
18
18
Max. Marks : 30 No. of Qs. 30 Time : 20 min. Date : ........./......../. ..............
1. If ‘+’ means ‘minus’ ‘–’ means ‘multiplied by’ ‘¸’ means ‘plus’ 8. If L stands for +, M stands for –, N stands for ×, P stands for
and ‘×’ means ‘divided by’, then ¸, then 14 N 10 L 42 P 2 M 8 = ?
10 × 5 ¸ 3 – 2 + 3 = ? (a) 153 (b) 216 (c) 248
53 (d) 251 (e) None of these
(a) 5 (b) 21 (c) 9. It being given that: > denotes +, < denotes –, + denotes ¸, –
3
denotes =, = denotes ‘less than’ and × denotes ‘greater than’.
(d) 18 (e) None of these
2.
ww
In the following question you have to identify the correct
response from the given premises stated according to the
following symbols.
Find which of the following is a correct statement.
(a) 3 + 2 > 4 = 9 + 3 < 2
(b) 3 > 2 > 4 = 18 + 3 < 1

w.E
If ‘+’ means ‘¸’ , ‘–’ means ‘×’, ‘¸’ means ‘+’ and ‘×’ means
‘–’, then 63 × 24 + 8 ¸ 4 + 2 – 3 = ?
(a) 54 (b) 66 (c) 186
(c) 3 > 2 < 4 × 8 + 4 < 2
(d) 3 + 2 < 4 × 9 + 3 < 3
(e) None of these

3.
(d) 48 (e) None of these
Which one of the following is correct?
6 * 4 * 9 * 15 asy 10. If P denotes +, Q denotes –, R denotes × and S denotes ¸
which of the following statements is correct?
(a) 36 R 4 S 8 Q 7 P 4 = 10
(a) ×, = , –
(d) –, ×, =
(b) ×, –, =
(e) None of these
(c) =, ×, –
En (b) 16 R 12 P 49 S 7 Q 9 = 200
(c) 32 S 8 R 9 = 160 Q 12 R 12
4. If > = ¸, Ú = ×, < = +, Ù = –, + = <, × = =, – = >
(a) 6 > 2 > 3 Ù 8 Ú 4 + 13
(b) 6 Ù 2 < 3 > 8 < 4 – 13 gin
11.
(d) 8 R 8 P 8 S 8 Q 8 = 57
(e) None of these
If ‘×’ stands for ‘addition’, ‘<’ for substraction, + for division,
(c) 6 Ú 2 < 3 Ù 8 > 4 × 13
(d) 6 > 2 Ú 3 < 8 Ù 4 + 13
eer
> for multiplication, – for ‘equal to’, ¸ for ‘greater than’ and
‘=’ for ‘less than’, then state which of the following is true?

5.
(e) None of these
Find out the correct answer for the unsolved equation on
the basis of the given equations.
(a) 3 × 4 > 2 – 9 + 3 < 3
(b) 5 × 3 < 7 ¸ 8 + 4 × 1
(c) 5 > 2 + 2 = 10 < 4 × 8 ing
If 6 * 5 = 91, 8 * 7 = 169, 10 * 7 = 211, then11 * 10 = ?
(a) 331
(d) 845
(b) 993
(e) None of these
(c) 678
(d) 3 × 2 < 4 ¸ 16 > 2 + 4
(e) None of these .ne
6. If ‘–’ stands for division, ‘+’ for multiplication ‘¸’for
subtraction and ‘×’ for addition. Which one of the following
equation is correct?
(a) 6 ¸ 20 × 12 + 7 – 1 = 70
DIRECTIONS (Qs. 12 - 16): In an imaginary language, the digits

h, i and j. And 10 is written as ba.


12. (cd + ef) × bc is equal to
t
0, 1, 2, 3, 4, 5, 6, 7, 8 and 9 are substituted by a, b, c, d, e, f, g,

(b) 6 + 20 – 20 ¸ 7 × 1 = 62 (a) 684 (b) 816 (c) 916


(c) 6 – 20 ¸ 12 × 7 + 1 = 57 (d) 1564 (e) None of these
(d) 6 + 20 – 20 ¸ 7 – 1 = 38 13. dc × f – (bf – d) × d is equal to
(e) None of these (a) abb (b) abe (c) bce
7. In an imaginary mathematical operation ‘+’ means (d) bcf (e) None of these
multiplication, ‘×’ means subtraction, ‘¸’means addition and
14. baf + fg – (ca × h/be) is equal to
‘–’ means division. All other rules in mathematical operation
(a) 141 (b) 145 (c) 151
are the same as in the existing system.
Which one of the following gives the result of (d) 161 (e) None of these
175 – 25 ¸ 5 + 20 × 3 + 20 = ? 15. baf ¸ bf × d is equal to
(a) 160 (b) 2370 (c) 7 7 (a) df (b) cb (c) be
(d) 240 (e) None of these (d) d (e) None of these

1. a b c d e 2. a b c d e 3. a b c d e 4. a b c d e 5. a b c d e
RESPONSE 6. a b c d e 7. a b c d e 8. a b c d e 9. a b c d e 10. a b c d e
GRID 11. a b c d e 12. a b c d e 13. a b c d e 14. a b c d e 15. a b c d e

Downloaded From : www.EasyEngineering.net


Downloaded From : www.EasyEngineering.net

36 SPEED TEST 18
16. If ‘+’ means ‘×’, ‘×’ means ‘–’, ‘¸’ means ‘+’ and ‘–’ means 24. If ‘÷’ means ‘+’ ; ‘–’ means ‘×’ ; ‘+’ means ‘÷’ and ‘×’ means
‘¸’, then which of the following gives the result of ‘–’ then 20 ÷ 12 × 4 + 8 – 6 = ?
175 – 25 ¸ 5 + 20 × 3 ¸ 10 = ?
2
(a) 77 (b) 160 (c) 240 (a) 8 (b) 29 (c) 32
3
(d) 2370 (e) None of these
17. If + means ÷ ,– means ×, ¸ means + and × means –, then 36 (d) 26 (e) None of these
× 8 + 4 ÷ 6 + 2 –3 = ? 25. If ‘–’ means ‘×’ ; ‘×’ means ‘+’ ; ‘+’ means ‘÷’ and ‘÷’ means
(a) 2 (b) 18 (c) 43 ‘–’ then 40 × 12 + 3 – 6 ÷ 60= ?
(a) 44 (b) 7.95 (c) 16
1
(d) 6 (e) None of these (d) 28 (e) None of these
2 26. If ‘ + ’ means ‘divided by’ ‘–’ means ‘added to ’ ‘x’ means
18. If the given interchanges namely : signs + and ÷ and ‘subtracted from’ and ÷ means ‘multiplied by’ then what is
numbers 2 and 4 are made in signs and numbers, which one the value of 24 ¸ 12 – 18 + 9 ?
of the following four equations would be correct ? (a) – 25 (b) 0.72 (c) 15.30
(a) 2 + 4 ÷ 3 = 3 (b) 4 + 2 ÷ 6 = 1.5 (d) 290 (e) None of these
(c) 4 ÷ 2 + 3 = 4 (d) 2 + 4 ÷ 6 = 8. 27. If ® stands for ‘addition’ ¬ stands for ‘subtraction’ ­

19. ww
(e) None of these
If L denotes x, M denotes ÷ , P denotes + and Q denotes
stands for ‘division ; ¯ stands for’ multiplication' Z stands
for equal to' then which of the following alternatives is

(a)
13
6
w.E
–, than 8 P 36 M 6 Q 6 M 2 L 3 = ?

(b) –
1
6
(c) 14
1
2
correct?
(a) 7 43 6 1Z4

20.
(d) 5 (e) None of these
asy
If X stands for’ addition’, < for ‘substraction’, + stands for
‘division’, > for ‘multiplication’, –, stands for ‘equal to’,
(b) 3
(c) 5
6
7
2
3
3
2Z5
6Z5

¸ for ‘greater than’ and = stands for ‘less than’, state which
of the following is true ? En (d) 2 5
(e) None of these
6 2Z6

(a) 3 × 2 < 4 ¸ 16 > 2 + 4


(b) 5 > 2 + 2 = 10 < 4 × 2 gin
28. Of ‘x’ Stands for ‘ addition’ ‘z’ for subtraction’ ‘+’ for division'
> for multiplication’ ‘–’ for equal to’ ‘+’ for ‘ greater than’
(c) 3 × 4 > 2 – 9 + 3 < 3
(d) 5 × 3 < 7 ¸ 8 + 4 × 1
(e) None of these eer
and '=' for ' less than' state which of the following is true. ?
(a) 3 x 4 > 2 – 9 + 3 < 3
(b) 5 x 3 < 7 ¸ 8 + 4 x 1
21. If ‘20 – 10’ means 200, ‘8 ÷ 4’ means 12, ‘6 × 2’ means 4 and
‘12 + 3’ means 4, then
(c) 5 > 2 + 2 = 10 < 4 x 8
(d) 3 x 2 < 4 ¸ 16 > 2 + 4 ing
100 – 10 × 1000 ÷ 1000 + 100 × 10 = ?
(a) 1090 (b) 0 (c) 1900 29.
(e) None of these
If ¸ means + – means ÷ x means – and + means x then
.ne
22.
(d) 20

7 – 8 + 40 ÷ 2 = ?
(e) None of these
If × means +, ÷ means – , – means × and + means ÷, then 8 ×

2 3
(36 x 4 ) - 8 x 4
4 + 8 x 2 + 16 + 1
(a) 0 (b) 8
=?

(c) 12
t
(a) 1 (b) 7 (c) 8 (d) 16 (e) None of these
5 5 30. If x means +, – means x , ÷ means + and + means – then
(d) 44 (e) None of these (3 – 15 ÷ 19) x 8 + 6 = ?
23. If ‘+’ means ‘×’ ; ‘–’ means ‘÷’ ; ‘ ×’ means ‘–’ and ‘÷’ means (a) – 1 (b) 2 (c) 4
‘+’ then 9 + 8 ÷ 8 – 4 × 9 = ? (d) 8 (e) None of these
(a) 26 (b) 17 (c) 65
(d) 11 (e) None of these

16. a b c d e 17. a b c d e 18. a b c d e 19. a b c d e 20. a b c d e


RESPONSE 21. a c d e 22. a c d e 23. a c d e 24. a c d e 25. a c d e
b b b b b
GRID
26. a b c d e 27. a b c d e 28. a b c d e 29. a b c d e 30. a b c d e

Downloaded From : www.EasyEngineering.net


Downloaded From : www.EasyEngineering.net

Clock and Calendar 19


19
Max. Marks : 30 No. of Qs. 30 Time : 20 min. Date : ........./......../. ..............
1. The number of times in a day the Hour-hand and the Minute- 8. At what time between 5.30 and 6 will the hands of a clock be
hand of a clock are at right angles, is at right angles?
(a) 44 (b) 48 (c) 24
5 7
(d) 12 (e) None of these (a) 43min. past 5 (b) 43 min. past 5
2. An accurate clock shows the time as 3.00. After hour hand 11 11
has moved 135°, the time would be (c) 40 min. past 5 (d) 45 min. past 5
(e) None of these

3. ww
(a) 7.30
(d) 9.30
(b) 6. 30
(e) None of these
(c) 8.00

An accurate clock shows 8 O' clock in the morning.


9. At what time between 4 and 5 o’clock will the hands of a
watch point in opposite directions?

(a) 150° w.E


Throughout how many degrees will the hour hand rotate,
when the clock shows 2 O' clock in the afternoon?
(b) 144° (c) 168°
(a) 45 min. past 4

(c)
4
50
11
(b) 40 min. past 4
6
min. past 4 (d) 54 min. past 4
11
4.
(d) 180° (e) None of these

asy
March 1, 2008 was Saturday. Which day was it on March 1,
2002? 10.
(e) None of these
How much does a watch lose per day, if its hands coincide
(a) Thursday
(d) Sunday
(b) Friday
(e) None of these
(c) Saturday

En every 64 minutes?
5. How many times are an hour hand and a minute hand of a
clock at right angles during their motion from 1.00 p.m. to
10.00 p.m.? gin (a)
8
32
11
(d) 96 min.
min.
5
(b) 36 min. (c) 90 min.
11
(e) None of these

6.
(a) 9
(d) 20
(b) 10
(e) None of these
(c) 18

At what approximate time between 4 and 5 am will the


11.
(a) Monday eer
The last day of a century cannot be
(b) Wednesday (c) Tuesday
hands of a clock be at right angle?
(a) 4 : 40 am (b) 4 : 38 am (c) 4 : 35 am 12.
(d) Friday
ing
(e) None of these
Which of the following is not a leap year?

7.
(d) 4 : 39 am (e) None of these
At what time between 3 and 4 o’clock, the hands of a clock
coincide?
(a) 700
(d) 2000
(b) 800
(e) None of these
.ne
(c) 1200

(a) 16
4
11
minutes past 3
13.

14.
How many days are there in x weeks x days?
(a) 7 x2
(d) 7
(b) 8 x
(e) None of these
(c) 14 x

It was Sunday on Jan 1, 2006. What was the day of the week
t
5
(b) 15 minutes past 3 on Jan 1, 2010?
61
(a) Sunday (b) Saturday (c) Friday
5 (d) Wednesday (e) None of these
(c) 15 minutes to 2
60 15. On 8th Feb, 2005 it was Tuesday. What was the day of the
4 week on 8th Feb, 2004?
(d) 16 minutes to 4
11 (a) Tuesday (b) Monday (c) Sunday
(e) None of these (d) Wednesday (e) None of these

1. a b c d e 2. a b c d e 3. a b c d e 4. a b c d e 5. a b c d e
RESPONSE 6. a b c d e 7. a b c d e 8. a b c d e 9. a b c d e 10. a b c d e
GRID 11. a b c d e 12. a b c d e 13. a b c d e 14. a b c d e 15. a b c d e

Downloaded From : www.EasyEngineering.net


Downloaded From : www.EasyEngineering.net

38 SPEED TEST 19
16. The calendar for the year 2007 will be the same for the year. 23. Ashish leaves his house at 20 minutes to seven in the
(a) 2014 (b) 2016 (c) 2017 morning reaches Kunal’s house in 25 minutes. They finish
(d) 2018 (e) None of these their breakfast in another 15 minutes and leave for their
17. Today is Monday. After 61 days, it will be office which takes another 35 minutes. At what time do
(a) Wednesday (b) Saturday (c) Tuesday they leave Kunal’s house to reach their office?
(d) Thursday (e) None of these (a) 7.40 a.m. (b) 7.20 a.m. (c) 7.45 a.m.
18. What was the day of the week on 17th June, 1998? (d) 8.15 a.m. (e) None of these
(a) Monday (b) Tuesday 24. Reaching the place of meeting on Tuesday 15 minutes
(c) Wednesday (d) Thursday before 8.30 hours, Anuj found himself half an hour earlier
(e) None of these than the man who was 40 minutes late. What was the
19 If 21st July, 1999 is a wednesday, what would have been scheduled time of the meeting?
the day of the week on 21st July, 1947 ? (a) 8.00 hrs (b) 8.05 hrs (c) 8.15 hrs
(a) Monday (b) Sunday (c) Thursday (d) 8.45 hrs (e) None of these
25. A clock gaining 2 min every hour was synchronised at
20.
ww
(d) Saturday (e) None of these
At an enquiry office at a railway station, a passenger was
told that a train for New Delhi has left 15 minutes ago, but
midnight with a clock losing 1 min every hour. How many
minutes behind will its minute hand be at eleven the

w.E
after every 45 minutes a train leaves for New Delhi. The
next train will leave at 8.30 p.m. At what time was this
information given to the passanger ?
following morning ?
(a) 23
(d) None of these
(b) 27
(e) None of these
(c) 22

(a) 7.45 pm
(d) 8.05 pm
(b) 8.00 pm
(e) None of these asy
(c) 8.15 pm 26. Rama remembers that she met her brother on Saturday,
which was after the 20th day of a particular month. If the
21.
En
A watch is a minute slow at 1 p.m. on Tuesday and 2 minutes
fast at 1 p.m. on Thursday. When did it show the correct
1st day of that month was Tuesday, then on which date did
Rama meet her brother ?
time ?
(a) 1:00 a.m. on Wednesday gin (a) 24th
(d) None of these
(b) 23rd (c) 25th

(b) 5:00 a.m. on Wednesday


(c) 1:00 p.m. on Wednesday
27. In 2
1
2 eer
hours the hour hand of a clock rotates through an

(d) 5:00 p.m. on Wednesday


(e) None of these
angle of
(a) 90° ing
(b) 140° (c) 120°
22. An application was received by inward clerk in the afternoon
of a week day. Next day he forwarded it to the table of the
28.
(d) 75° (e) None of these
.ne
On 27 March, 1995 was a Monday. Then what days of the
senior clerk, Who was on leave that day. The senior clerk
put up the application to the desk officer next day in the
evening. The desk officer studied the application and
29.
week was 1 November, 1994?
(a) Monday
(c) Tuesday
(b) Sunday
(d) Wednesday
16 January 1997 was a Thursday. What day of the week
t
disposed off the matter on the same day i.e., Friday. Which
was 4 January 2000?
day was the application received by the inward clerk ?
(a) Tuesday (b) Wednesday (c) Thursday
(a) Monday
(d) Friday (e) None of these
(b) Wednesday
30. In a year 28th February is Tuesday; if the leap year is
(c) Tuesday
excluded, then 28th March will be a
(d) Previous week’s Saturday
(a) Sunday (b) Tuesday (c) Monday
(e) None of these (d) Saturday (e) None of these

16. a b c d e 17. a b c d e 18. a b c d e 19. a b c d e 20. a b c d e


RESPONSE 21. a c d e 22. a c d e 23. a c d e 24. a c d e 25. a c d e
b b b b b
GRID
26. a b c d e 27. a b c d e 28. a b c d e 29. a b c d e 30. a b c d e

Downloaded From : www.EasyEngineering.net


Downloaded From : www.EasyEngineering.net

Data Sufficiency 20
20
Max. Marks : 25 No. of Qs. 25 Time : 20 min. Date : ........./......../. ..............
7. In a row of girls facing North, what is D’s position from the
DIRECTIONS (Qs. 1 - 20) : Each question below is followed by
left end?
two statements I and II. You are to determine whether the data
I. D is twentieth from the right end.
given in the statement is sufficient to answer the question. You
II. There are ten girls between B and D.
should use the data and your knowledge of Mathematics to choose
8. Town M is towards which direction of Town K?
between the possible answers.
I. Town K is towards North-West of Town D.
Give answer (a) if the statement I alone is sufficient to answer the

ww question, but the statement II alone is not


sufficient.
Give answer (b) if the statement II alone is sufficient to answer
9.
II. Town M is towards South - East of Town D.
How many daughters does P have?
I. K and M are sisters of T.

w.E
the question, but the statement I alone is not
sufficient.
Give answer (c) if both statements I and II together are needed to
10.
II. T’s father is husband of P’s mother.
Towards which direction is Village M from Village T?
I. Village P is to the south of Village M and Village P is to
answer the question.
asy
Give answer (d) if either the statement I alone or the statement II
alone is sufficient to answer the question.
the west of Village T.
II. Village K is to the east of Village M and Village K is to
the north of Village T.

En
Give answer (e) if you cannot get the answer from the statements
I and II together, but need even more data.
11. How is D related to M?
I. K and D are the only sisters of R.
1. What is the age of C, in a group of A, B, C, D and E, whose
average age is 45 years?
I. Average of the ages of A and B is 53 years.
gin
12.
II. M is married to R’s father.
What is R’s position from the left end in a row of children
facing South?
II. Average of the ages of D and E is 47 years
2. Tower ‘P’ is in which direction with respect to tower ‘Q’?
I.
eer
There are forty children in the row.
II. D is tenth to the left of R and fifteenth from the right
end of the row.
I. P is to the West of H, which is to the South of Q.
II. F is to the West of Q and to the North of P.
3. How is K related to N?
13.
ing
Towards which direction was D facing when he started his
journey?
I. N is the brother of M, who is the daughter of K.
II. F is the husband of K
I.
.ne
D walked 20 metres after he started, took a right turn
and walked 30 metres and again took a right turn and
faced West.
4. What is Nidhi’s age?
I. Nidhi is 3 times younger than Rani.
II. Surekha is twice the age of Rani and the sum of their
ages is 72.
14.
walked 30 metres and again took a left turn and faced
West.
How many daughters does A have?
t
II. D walked 20 metres after he started, took a left turn and

5. What is Seema’s age?


I. Seema’s age is half of Reema age I. A has four children.
II. Reema is 5 years younger than her sister. II. B and C are sisters of D who is son of A.
6. What is Deepali’s age? 15. How far is A from the starting point?
I. Deepali is two times younger than Nisha. I. A moves 5 km. towards East, then 2 km. towards left,
II. Supriya is twice the age of Nisha. 10 km, towards right and finally. 2 km, towards right
and stops
II. A moves 2 km. towards East, then 2 km. towards right,
13 km, towards left and finally, 2 km. towards left and
stops.

1. a b c d e 2. a b c d e 3. a b c d e 4. a b c d e 5. a b c d e
RESPONSE 6. a b c d e 7. a b c d e 8. a b c d e 9. a b c d e 10. a b c d e
GRID 11. a b c d e 12. a b c d e 13. a b c d e 14. a b c d e 15. a b c d e

Downloaded From : www.EasyEngineering.net


Downloaded From : www.EasyEngineering.net

40 SPEED TEST 20
16. In a row of 40 students facing North, how many students are (b) if the data in statement II alone are sufficient to answer the
there between R and S? question, while the data in statement I alone are not sufficient
I. S’s position in the row is 15th from the right end. to answer the question.
II. R’s position in the row is 4th from the left end. (c) If the data either in statement I alone or in statement II alone
17. How many children does Suneeta have? are sufficient to answer the question.
I. X is the only daughter of Suneeta. (d) if the data given in both the statements I and II together are
II. Y is brother of X. not sufficient to answer the question.
18. Pole X is in which direction with respect to pole Y? (e) if the data given in both the statements I and II together are
I. Pole H is to the north-east of pole X and to the north of necessary to answer the question.
pole Y. 21. In a row of girls facing North, what is D’s position from the
II. Pole R is to the east of pole X and to the north of pole left end?
Y. I. D is twentieth from the right end.
19. How many children does Seema have? II. There are ten girls between Band D.
I. Seema, the mother of Varsha’s sister has only one son. 22. Town M is towards which direction of Town K?
II. Varsha has only three siblings. I. Town K is towards North-West of Town D

ww
20. How is Anil related to Sanjay?
I. Sanjay’s son is the brother of only sister of Anil.
II. Radhika, the only daughter of Sanjay has only two
II. Town M is towards South-East of Town D
23. How many daughters does P have?
I. K and M are sisters of T.
brothers.
w.E
DIRECTIONS (Qs. 21-25) : Each of the questions below consists
of a question and two statements numbered I and II given below
II. T’s father is husband of P’s mother.
24. On which day of the week from Monday to Sunday did Arun
leave for London?

asy
it. You have to decide whether the data provided in the statements
are sufficient to answer the question. Read both the statements
I. Arun did not leave for London during the weekend.
II. Arun’s brother left for London on Friday two days after
and Give answer
(a) if the data in statement I alone are sufficient to answer theEn Arun left for London.
25. How is ‘new’ written in a code language?
question, while the data in statement II alone are not sufficient
to answer the question.
ginI. ‘new good clothes’ is written as ‘5 3 9’ in that code
language.
II. ‘good clothes are costly’ is written as ‘9673’ in that

eer
code language.

ing
.ne
t

RESPONSE 16. a b c d e 17. a b c d e 18. a b c d e 19. a b c d e 20. a b c d e

GRID 21. a b c d e 22. a b c d e 23. a b c d e 24. a b c d e 25. a b c d e

Downloaded From : www.EasyEngineering.net


Downloaded From : www.EasyEngineering.net

Statement & Conclusion


(Mathematical)
21
Max. Marks : 22 No. of Qs. 22 Time : 20 min. Date : ........./......../. ..............
DIRECTIONS (Qs. 1-4): In these questions, relationships between Give answer
different elements is shown in the statements. These statements (a) if only Conclusion I is true.
are followed by two conclusions. (b) if only Conclusion II is true.
Give answer (a) if only conclusion I follows. (c) if either Conclusion I or II is true.
Give answer (b) if only conclusion II follows.
Give answer (c) if either conclusion I or conclusion II follows. (d) if neither Conclusion I nor II is true.

ww
Give answer (d) if neither conclusion I nor conclusion II
follows.
Give answer (e) if both conclusions I and II follow.
5.
(e) if both Conclusions I and II are true.
Statements: F @ N, N d R, H @ R
Conclusions: I. H d N
1. Statement :
A< L<T< R £ H>K
Conclusions :
w.E 6. Statements:
II. F # R
M # T, T@ K, K $ N
Conclusions: I. M # N

2.
I. H > L
II. K > T
Statement : asy 7. Statements:
II. K d M
T % H, H $ W
P= N > D ³ G< B= J
Conclusions : En Conclusions: I. W # T
II. W % T

3.
I. G < P
II. G < J
Statement :
gin
8. Statements: N d K, K # D, D % M
Conclusions: I. M d K
F£ C³ V=Z<X=U
Conclusions : 9. eer
Statements:
II. D d N
J $ B, B % R, R d F

4.
I. V < U
II. Z < F
Statement :
Conclusions: I. F # B
II. R @ J
ing
Q£ E=I>N ³ R ³ S
Conclusions : .ne
DIRECTIONS (Qs. 10-14): In these questions symbols ©, #, *, $
and @ are used with different meanings as follows:
'A ©B' means ‘A is smaller than B’.
I. E = S
II. S £ N
DIRECTIONS (Qs. 5-9): In the following questions, the symbols
d , %, $, # and @ are used with the following meaning as illustrated
A # B means ‘A is either smaller, than or equal to B’.
`A * B' means ‘A is greater than B’.
A $ B means ‘A is either greater than or equal to B’.
A @ B means ‘A is neither smaller than nor greater than B’.
t
below: In each of the following questions assuming the given
‘P $ Q’ means ‘P is not smaller than Q’. statements to be true, find out which of the two conclusions
‘P @ Q’ means ‘P is not greater than Q’. I and II given below them is/are definitely true.
‘P d Q’ means ‘P is neither smaller than nor equal to Q’. Give answer (a) if only conclusion I is true.
‘P # Q’ means ‘P is neither greater than nor equal to Q’. Give answer (b) if onlyconclusion II is true.
‘P % Q’ means ‘P is neither smaller than nor greater than Q’. Give answer (c) if either conclusion I or conclusion II is true.
Give answer (d) if neither conclusion I nor conclusion II is
Now in each of the following questions assuming the given
true.
statements to be true, find which of the two conclusions I and II Give answer (e) if both conclusions I and II are true.
given below them is/are definitely true? 10. Statements:V # S, S © L, L © J
Conclusions: I. V © L
II. S © J

RESPONSE 1. a b c d e 2. a b c d e 3. a b c d e 4. a b c d e 5. a b c d e

GRID 6. a b c d e 7. a b c d e 8. a b c d e 9. a b c d e 10. a b c d e

Downloaded From : www.EasyEngineering.net


Downloaded From : www.EasyEngineering.net

42 SPEED TEST 21
11. Statements: M # R, R © J, J # H 17. Statements:
Conclusions: I. M # H K # T, T $ B, B @ F
II. R © H Conclusions: I. F $ T
12. Statements:H $ F, F @ G, G « M II. K # B
Conclusions: I. H « M III. T $ F
II. H « G (a) None is true (b) Only I is true
13. Statements:R © J, J « T, T # L (c) Only I and II are true (d) Only II and III are true
Conclusions: I. R @ T (e) All are true
II. J @ L 18. Statements:
14. Statements: W @ T, T $ K, K « F Z # F, R @ F, D c R
Conclusions: I. W $ K Conclusions: I. Z # R
II. W @ K II. F # D
DIRECTIONS (Qs.15-19) : In the following questions, the symbols III. D @ Z
(a) None is true (b) Only I is true
@, #, %, $ and c are used with the following meaning as illustrated
(c) Only III is true (d) Only either I or III is true
below:
(e) All are true
'P # Q' means 'P is neither greater than nor equal to Q'.
19. Statements:

ww
'P c Q' means 'P is neither equal to nor smaller than Q'
'P % Q' means 'P is neither smaller than nor greater than Q.
'P $ Q' means 'P is not smaller than Q'
M c R, R % D, D @ N
Conclusions: I. M c N
II. N $ R

w.E
'P @ Q' means 'P is not greater than Q'.
Now in each of the following questions, assuming the given
statements to be true, find which of the three conclusions I, II and
III given below them is/are definitely true and give your answer
III. M c D
(a) Only I and II are true
(c) Only I and III are true
(b) Only II and III are true
(d) All are true
accordingly.
15. Statements: R @ D, D c W, B $ W asy (e) None of these
DIRECTIONS (Qs.20-22) : In these questions, relationship
between different elements is shown in the statements. These
Conclusions:
I. W # R
(a) None is true
II. B c D III. W $ R
(b) Only I is true En statements are followed by two conclusions.
Mark answers if
(c) Only III is true
(e) All are true
(d) Only either I or III is true
gin
(a) Only conclusion I follows.
(b) Only conclusion II follows.
(c) Either conclusion I or II follows.
16. Statements:
H $ V, V % M, K c M
Conclusions: I. K c V eer
(d) Neither conclusion I nor II follows.
(e) Both conclusion I and II follows.
II. M @ H
III. H c K
20. Statement: P ³Q=R>S>T
Conclusions: I. P ³ T
II. T < Q ing
(a) Only I and III are true (b) Only II and III are true
(c) Only I and II are true (d) All are true
(e) None of these
21. Statement: L £M<N>O³P
Conclusions: I. O < M
II. P £ N
.ne
22. Statement: A > B, B ³ C = D < E
Conclusions: I. C < A
II. D £ B
t

11. a b c d e 12. a b c d e 13. a b c d e 14. a b c d e 15. a b c d e


RESPONSE
16. a b c d e 17. a b c d e 18. a b c d e 19. a b c d e 20. a b c d e
GRID
21. a b c d e 22. a b c d e

Downloaded From : www.EasyEngineering.net


Downloaded From : www.EasyEngineering.net

Statement &
Conclusion (Logical) 22
22
Max. Marks : 20 No. of Qs. 20 Time : 15 min. Date : ........./......../. ..............
DIRECTIONS (Q. 1 to 18) : In each question below is given a 4. Statement : Population increase coupled with depleting
statement followed by two conclusions numbered I and II. You resources is going to be the scenario of many developing
have to assume everything in the statement to be true, then countries in days to come.
consider the two conclusions together and decide which of them Conclusions :
logically follows beyond a reasonable doubt from the information I : The population of developing countries will not
given in the statement. continue to increase in future.

ww
Give answer (a) if only conclusion I follows.
Give answer (b) if only conclusion II follows.
Give answer (c) if either I or II follows.
II : It will be very difficult for the governments of
developing countries to provide its people decent
quality of life.

w.E
Give answer (d) if neither I nor II follows.
Give answer (e) if both I and II follows.
1. Statement : Although we have rating agencies like Crisil.
5. Statement : Mr. X is one of the probable candidates
shortlisted for the post of Director of K. L. M. Institute.
Conclusions :

IT Companies to protect investors.


Conclusions : asy
ICRA, there is demand to have a separate rating agency for I : Mr. X will be selected as Director of K. L. M. Institute.
II : Mr. X will not be selected as Director of K. L. M.
Institute.
I : Assessment of financial worth of IT Companies calls
En
for separate set of skills, insight and competencies.
6. Statement :
‘We follow some of the best and effective teaching learning

2.
II : Now the investors investing in I. T. Companies will get
protection of their investment.
Statement : Company "Y" will improve the manufacturing gin practices used by leading institutes all over the world’. — A
statement of professor of MN Institute.
Conclusions : I. The MN Institute is one of the leading
facilities for the production of shaving kits as a result of
which capacity would increase and cost would be reduced –
world.
eer
institutes of the

II. Whatever is being followed by world’s leading institutes


A spokesperson of the Company "Y".
Conclusions :
I : The products of Company "Y" will complete the market
7. Statement : ing
will definitely be good and useful.

norms in the quality and cost factor.


II : There will be demand of shaving kits of Company "Y"? .ne
In the absence of national health insurance or social security
cover, a person with limited resources has to depend on
government hospitals, which are crowded, overburdened
3. Statement : During 1997-98 the total loss incurred by the
111 Public Sector Units was to the tune of ` 6809 crore,
which was converted into paid capitals by the Government
of its total investment of ` 5129 crore.
and understaffed.
Conclusions : I. National health insurance is meant only
for the affluent
sections of society.
t
Conclusions : II. The government hospitals provide treatment on nominal
I : The Government is left with only one option that is to charges or free.
privatise these units. 8. Statement :
II : The Government did not take care in the matter of We do not need today in India extraordinary specialists but
investments in these public sector units. those trained ordinary doctors who are dedicated to their
profession.
Conclusions : I. We should promote medical profession
with dedicated
ordinary doctors rather than promoting high specialised
medical education.
II. Extraordinary specialists are not dedicated to their
profession.

RESPONSE 1. a b c d e 2. a b c d e 3. a b c d e 4. a b c d e 5. a b c d e

GRID 6. a b c d e 7. a b c d e 8. a b c d e

Downloaded From : www.EasyEngineering.net


Downloaded From : www.EasyEngineering.net

44 SPEED TEST 22
9. Statement : 16. Statements :
“The Government will review the present policy of the diesel A study of planning commission reveals boom in revenues.
price in view of further spurt in the international oil prices” However, this has been of little avail owing to soaring
— A spokesman of the Government. expenditure. In the event, there has been a high dose of
Conclusions : deficit financing, leading to marked rise in prices.
Large financial outlays year after year had little impact on
I. The Government will increase the price of the diesel after
the standard of living.
the imminent spurt in the international oil prices. Conclusions :
II. The Government will not increase the price of the diesel I. A boom in revenues leads to soar in prices.
even after the imminent spurt in the international oil II. Large financial outlays should be avoided.
prices. 17. Statements :
10. Statement : The average number of students per teacher is 50 in the urban
Vegetable prices are soaring in the market. area whereas it is 60 in rural areas. The national average is 55.
Conclusions : Conclusions :
I. Vegetables are becoming a rare commodity. I. The student-teacher ratio in the rural areas is higher than
II. People cannot eat vegetables. in the urban areas.
11. Statement : II. More students study with the same teacher in the rural
Being from a business family, Chandan was apparently areas as compared to those in the urban areas.

ww
convinced by his parents and other family members to join
the family trade.
Conclusions :
18. Statement :
Morning walks are good for health.
Conclusions :

family prospers. w.E


I. People should take up their family profession so that

II. It is necessary to keep in family members happy by


19.
I. All healthy people go for morning walks.
II. Evening walks are harmful.
Statement : The cost of manufacturing cars in state A is 30
per cent less than the cost of manufacturing cars in state B.

12.
choosing family’s business.
Statement :
asy
Global ecological issues have eclipsed local environmental
After transportation fee for the differential distances of states
A and B and the interstate taxes, it is cheaper to manufacture
cars in state B than in state A for selling these cars in State C.
problems which are being faced by the poor societies.
Conclusions :
En Which of he following supports the conclusion draw in the
above statement?
(a) The cost of transportation from state A to state C is
I. Poor societies always have to suffer because of their
poverty.
II. Global ecological issues are not so important. Rich gin more than 30 per cent of the production cost.
(b) The production cost of cars in state B is lower in

eer
comparison to state A.
societies can bear with it. (c) Only entry tax at state C is more for the production
13. Statements : originating in state A.
Of the ten fishermen caught in a storm, nine managed to
return to the shore.
Praveen has not yet returned after four days.
in state B.
ing
(d) Entry tax at state C is more for the products originating

(e) The total of transportation cost of cars from state B to


Conclusions :
I. Praveen got killed in the storm.
II. Praveen has survived the storm. 20. .ne
state C and entry tax of cars at state C is less than 30
per cent of the production cost of cars in state B.
Statement : There was a slow decline in the number of
14. Statements :
Now you don’t need an import licence to own a VCR.
Conclusions :
I. VCRs are now manufactured indigenously.
patients with flu-like symptoms visiting various health
facilities in the city during the last fortnight.
t
Which of the following substantiates the fact mentioned in
the above statement?
(a) Majority of the people suffering from flu visit the health
II. VCRs are now freely permitted to be imported. facilities in the city.
15. Statements : (b) There has been a continuous increase in the sale of
Just about everyone in Germany has been on a diet at one medicines for curing flu in the city limits.
time or the other and millions of them have learned that the (c) People have started visiting the crowded places like
weight they lose is all too easily regained. malls and cinema halls during the last fortnight after a
Still’ despite their frustration, few question the wisdom of cautioned gap of one month.
dieting. (d) There is a sudden increase in the number of deaths
Conclusions : caused by flu-like symptoms followed by respiratory
I. Germany should stop dieting. complications.
II. Germans do not learn from experience. (e) None of these

9. a b c d e 10. a b c d e 11. a b c d e 12. a b c d e 13. a b c d e


RESPONSE
14. a b c d e 15. a b c d e 16. a b c d e 17. a b c d e 18. a b c d e
GRID
19. a b c d e 20. a b c d e

Downloaded From : www.EasyEngineering.net


Downloaded From : www.EasyEngineering.net

Passage Based
Conclusion-1
23
23
Max. Marks : 15 No. of Qs. 15 Time : 15 min. Date : ........./......../. ..............
DIRECTIONS (Qs. 1-3): Study the following information carefully 18.2% rural population moved out of poverty whereas another 22.1%
and answer the given questions. fell into it over this period. This net increase of about four percentage
points was seen to have a considerable variation across states and regions.
The prospects for the Indian economy this year will be influenced by the
4. Which of the following is a conclusion which can be drawn from
behaviour of the monsoon and expansion of commerce and trade. The
the facts slated in the above paragraph ?
Eleventh Plan has envisaged a growth target of 8%. If the agriculture
(a) Accurate estimates of number of people living below poverty
sector does well and the world trade conditions improve then it is possible
line in India is possible to be made.
to achieve a growth of 6-7%. We need to improve our economy and aim

ww
at a higher rate of growth in order to feed our population, maintain the
standard of living and improve the quality of life. It is now more than 10
years since we have adopted reforms. We need to go forward in
(b) Many expert groups in dia are not interested measure poverty
objectively.
(c) Process of poverty measurement needs to take into account

w.E
various factors to tackle its dynamic nature.
liberalisation but we cannot throw open the market for everything. There
(d) People living below poverty line remain in that position for
are sectors like village industries which need protection.
a very long time.
1. Which of the following is an assumption which is implicit in the
(e) None of these 5. Which of the following is an assump-
facts stated in the above paragraph?

asy
(a) India should adopt economic policies of developed countries.
(b) Free market strategy is beneficial for India, but not in all the
sectors.
tion which is implicit in the
facts stated In the above paragraph ?
(a) It may not be possible to have an accurate poverty

(d) A very good monsoon is expected this year.


En
(c) Over the last few years, we have achieved sustained growth.
measurement in India.
(b) Level of poverty in India is static over the years.
(c) Researchers avoid making conclusions on poverty
2.
(e) None of these
Which of the following is an inference which can be drawn from the
facts stated in the paragraph? gin measurement data in India.
(d) Government of India has a mechanism to measure level of
poverty effectively and accurately.
(a) The world trade conditions don't affect Indian economy.
(b) The world trade conditions have a major impact on Indian
economy. eer
(e) None of these 6.
ence which can be made from
Which of the following is an infer-

the facts stated in the above paragraph ?


(c) Indian economy has been downgraded since last decade.
(d) Govt should cut the subsidies in order to obtain sustained growth.
(e) None of these ing
(a) Poverty measurement tools in India arc outdated.
(b) Increase in number of persons falling into poverty varies
considerably across the country over a period of time.
3. Which of the following is a conclusion which can be drawn from
the facts stated in the above paragraph?
(a) India may become a super economic power some day.
studies.
.ne
(c) Government of India has stopped measuring poverty related

(d) People living in rural areas are more susceptible to fall into
(b) The standard of living of people has continuously degraded
in India.
(c) Growth of Indian economy and a good mon soon are
complement of each other.
(d) Indian economy is on the peak of growth.
poverty over the time
(e) None of these
t
DIRECTIONS (Qs. 7 to 9) : Study the given information carefully and
answer the questions that follow:
(e) None of these Indian Navy's now believes that its worst fear has come true, and believes
it's unlikely to find any survivors onboard INS Sindhurakshak. The Navy
DIRECTIONS (Qs. 4-6) : Study the following Information carefully
also came out with a statement, which said that 'the state of these two
and answer the questions given below :
bodies and conditions within the submarine leads to firm conclusion that
Poverty measurement is an unsettled issue, both conceptually and finding any surviving personnel within the submarine is unlikely'.
methodologically. Since poverty is a process as well as an outcome; This is for the first time that the Indian Navy has got first-hand
many come out of it while others may be falling into it. The net effect of confirmation of fatality since two massive explosions rocked INS
these two parallel processes Is a proportion commonly identified as the Sindhurakshak. A Navy release said that the bodies were shifted to INHS
‘head count ratio’, but these ratios hide the fundamental dynamism that Asvini where a DNA identification test would be carried to identify the
characterises poverty in practice. The most recent poverty reestimates crewmember.
by an expert group has also missed the crucial dynamism. In a study The damage and destruction within the submarine around the control
conducted on 13.000 households which represented the entire country In room area indicates that the feasibility of locating bodies of personnel in
1993-94 and again on 2004-05. it was found that in the ten-year period the forward part of the submarine is also very remote as the explosion

RESPONSE 1. a b c d e 2. a b c d e 3. a b c d e 4. a b c d e 5. a b c d e

GRID 6. a b c d e

Downloaded From : www.EasyEngineering.net


Downloaded From : www.EasyEngineering.net

46 SPEED TEST 23
and very high temperatures, which melted steel within, would have 11. Which of the following is an Assumption which is implicit in the
incinerated the bodies too, said Navy officials. But navy officials said facts stated in the above paragraph?
that the divers would continue to search every inch of the submerged (a) Advertising guarantees the manufacturer in exhausting the
submarine till all bodies are either located or it can be stated with finality stocks he has produced.
that no bodies remain to be found. (b) Advertising stimulates consumers to buy more products.
7. Which of the following is the conclusion which can be drawn
(c) Advertising leads to fierce competition.
from the facts stated in the above paragraph?
(d) A manufacturer manufactures a product because there is
(a) No personnel have survived in the INS Sindhurakshak tragedy.
demand in the market.
(b) It is impossible to trace the bodies of all the personnel died in
(e) It is impossible to predict the demand of a product accurately.
the incident.
(c) The Navy is presently concentrating on reaching the interiors 12. Which of the following is an Inference which can be made from
of the submarine to locate and extricate any remaining bodies the facts stated in the above paragraph?
that may still be trapped within. (a) Manufacturers shall not try to predict the demand of a
(d) Navy's rescue and salvage effort is proceeding slowly because product.
it lacks expertise in conducting such an operation. (b) These days there are a lot of tools that helps in predicting the
(e) It was a massive accident leading to the death of all the sailing demand of a product accurately.
crew members. (c) Manufacturers must produce only what they can sell.
8. Which of the following is an assumption which is implicit in the (d) Consumer is the king.

ww
facts stated in the above paragraph?
(a) Some personnel could be found alive.
(b) The submarine and its internal machinery could be reused
thus saving costs for the Navy.
(e) Advertising can minimise the risk taken by manufacturers.
13. Our school district should not spend its money on the new Verbal
Advantage reading program. After all, our students get all the reading

w.E
practice they need by studying history and science.
(c) The temperature inside the submerged submarine would be The argument above depends on which the following assumptions?
very hot. (a) The Verbal Advantage program would not help the students
(d) There is lack of divers who are equally skilled in using gas learn history and science.

hatches.
asy
cutters or other machinery required to break open jammed

(e) Looking at the level of destruction more dead bodies would


(b) Other reading programs are just as effective but less expensive
than the Verbal Advantage program.
(c) The Verbal Advantage program involves only reading practice.
9.
be found.
Which of the following is an inference which can be made from
the facts stated in the above paragraph? En (d) Teaching students history and science is more important
(a) The explosion and very high temperatures, which melted
steel within, would have incinerated the bodies too.
(b) Continuous efforts would lead to extrication of the bodies of
gin than teaching them reading skills.
(e) None of these
14. Efficiency is all right in its place, in the shop, the factory, the store.
all the personnel.
(c) The bodies are expected to be so badly charred that
eer
The trouble with efficiency is that it wants to rule our play as well
as our work; it won’t be content to reign in the shop, it follows us
home.
identification will not be possible.
(d) Submarines are not safe watercraft capable of carrying
independent operation underwater. ing
It can be inferred from the above passage that
(a) Efficiency can become all - pervading
(e) Old vessels and other crafts must be phased out so as to
avoid such accidents in future.
DIRECTIONS (Qs 10 to 12): Study the given information carefully
(b) Efficiency does not always pay

(d) both (b) and (c) (e) .ne


(c) Efficiency can be more of a torture than a blessing
None of these
and answer the questions that follow:
Through advertisting, manufacturing exercises a high degree of control
over consumer's desires. However, the manufacturer assumes enormous
risks in attempting to predict what consumers will want and in producing
in such regions have a large number of seats. t
15. The fare-paying capacity of people who travel on routes connecting
to small towns is very low. Most successful airlines which operate

Which of the following can be inferred from the above information?


goods in quantity and distributing them in advance of final selection by (a) Regional airlines are quite profitable.
the consumers. (b) People from cities are increasingly travelling to small towns.
10. Which of the following is the conclusion which can be drawn (c) Regional airlines have to charge low fares in order to be
from the facts stated in the above paragraph? profitable.
(a) Distribute goods directly to the consumers. (d) The number of people travelling from small towns to cities is
(b) Can eliminate the risk of overproduction by advertising. massive.
(c) Always take moderate and calculated risk. (e) None of these
(d) Can predict with great accuracy the success of any product
they put on the market.
(e) Must depend upon the final consumers for the success of
their undertakings.

RESPONSE 7. a b c d e 8. a b c d e 9. a b c d e 10. a b c d e 11. a b c d e

GRID 12. a b c d e 13. a b c d e 14. a b c d e 15. a b c d e

Downloaded From : www.EasyEngineering.net


Downloaded From : www.EasyEngineering.net

Passage Based
Conclusion-2
24
24
13
Max. Marks : 15 No. of Qs. 15 Time : 15 min. Date : ........./......../. ..............
DIRECTIONS (Qs. 1 to 3): Study the given information carefully and DIRECTIONS (Q. 4 and 5) : In the following questions a paragraph is
answer the questions that follow: given. Read the paragraph carefully and answer the questions which
follow each of these paragraph.
In economics, a recession is a business cycle contraction, a general
slowdown in economic activity. Macroeconomic indicators such as GDP Fashion has become one of the largest fads among the youth. The amount
(Gross Domestic Product), employment, investment spending, capacity of time wastage and expenditure on fashion is very large. What bothers,
utilization, household income, business profits, and inflation fall, while however, is the fact that fashion is here to stay despite countless arguments

ww
bankruptcies and the unemployment rate rise.
Recessions generally occur when there is a widespread drop in spending
(an adverse demand shock). This may be triggered by various events, such
as a financial crisis, an external trade shock, an adverse supply shock or the
against it. What is required, therefore, is that strong efforts should be
made in order to displace the excessive craze of fashion from the minds of
today’s youth.
4. Which of the following statements finds the least support by the

w.E
bursting of an economic bubble. Governments usually respond to recessions
by adopting expansionary macroeconomic policies, such as increasing money
supply, increasing government spending and decreasing taxation.
argument made by the author in the given paragraph?
(a) Youngsters should be motivated to do constructive business
rather than wasting time on fashion.

asy
1. Which of the following is the conclusion which can be drawn from (b) The world of fashion being glamorous and glittery attracts
the facts stated in the above paragraph? people towards itself.
(a) Recession leads to financial crisis. (c) Following the latest fashion increases the self-efficacy of
people, thus increasing their overall mental abilities.

En
(b) Demand and supply play a major role in determining the
fundamentals of an economy. (d) Many universities have implemented a dress code to put a
(c) Cash is the king. So always keep your money safe. check on the increasing fad amongst the youth which was
(d) In a recession, interest rates are reduced so as to enable people
to take loans which bring back the money in the economy
thus increasing spending and economic activity. gin
5.
affecting their grades.
(e) None of these
Which of the following can be inferred from the given paragraph?

eer
(e) Recession is a normal (albeit unpleasant) part of the business (a) The author has made strong efforts to wipe out fashion from
cycle and brings in correction in the market. the minds of youth.
2. Which of the following is an assumption which is implicit in the (b) Steps need to be taken in order to control the growing fad of
facts stated in the above paragraph?
(a) Recessions generally occur when there is a widespread drop
in spending. ing
fashion amongst the youth.
(c) The author is upset with the shift” of fashion from the
traditional ethnic wear to western outfits.

.ne
(d) Fashion world is responsible for lack of creativity among the
(b) When economic activity slows down, firms may decide to
youth.
reduce employment levels and save money rather than invest.
(e) None of these
(c) A recession generally lasts from six to 18 months.

3.
(d) Recession is the result of adverse supply shock or the bursting
of an economic bubble.
(e) A slow growing nation can protect itself from recession.
Which of the following is an inference which can be made from
the facts stated in the above paragraph?
DIRECTIONS (Qs. 6 to 8): Study the given information carefully and
answer the questions that follow:

Due to enormous profits involved in smuggling, hundreds of persons


t
have been attracted towards this anti-national activity. Some of them
(a) Recession leads to decline in consumption, investment, became millionaires overnight. India has a vast coastline both on the
Eastern and Western Coast. It has been a heaven for smugglers who have
government spending, and net export activity.
been carrying on their activities with great impunity. There is no doubt,
(b) In a recession people should generally try to watch their
that from time to time certain seizures were made by the enforcement
spending and not take any undue risks that might put their
authorities, during raids and ambush but even allowing these losses the
future financial goals in jeopardy.
smugglers made huge profits.
(c) Recession affects both the developed and developing nations
6. Which of the following is the conclusion which can be drawn from
of the world.
the facts stated in the above paragraph?
(d) Countries must reduce their imports drastically so as to avoid
(a) smuggling hampers the economic development of a nation.
recession. (b) smuggling ought to the curbed.
(e) Recession is a business cycle contraction which brings a (c) authorities are taking strict measures to curb smuggling.
general slowdown in economic activity. (d) smuggling is fast increasing in our country owing to the quick
profit it entails.
(e) None of these

RESPONSE 1. a b c d e 2. a b c d e 3. a b c d e 4. a b c d e 5. a b c d e

GRID 6. a b c d e

Downloaded From : www.EasyEngineering.net


Downloaded From : www.EasyEngineering.net

48 SPEED TEST 24
7. Which of the following is an assumption which is implicit in the (c) Increase in income of all leads to a better life style of people.
facts stated in the above paragraph? (d) Increase in exports does not guarantee increase in income
(a) Coastlines provide easy escape for the smugglers. levels of all class of people.
(b) People are driven by money not by values. (e) The gap between rich and poor determines national
(c) Smugglers are smart enough to be caught. prosperity.
(d) Government can earn a lot of money from seizing smuggled DIRECTIONS (Qs. 12 to 14): Study the given information carefully
goods. and answer the questions that follow:
(e) Smuggling cannot be stopped as smugglers will find new
The consumption of harmful drugs by the people can be prevented not
ways of doing it.
only by banning their sale in the market but also by instructing users
8. Which of the following is an inference which can be made from about their dangerous effects which they must understand for their safety.
the facts stated in the above paragraph? Also the drug addicts may be provided with proper medical facilities for
(a) The enforcement authorities lack in their efforts to curb their rehabilitation. This will help in scaling down the use of drugs.
smuggling. 12. Which of the following is the conclusion which can be drawn
(b) Authorities are taking bribe from smugglers there by from the facts stated in the above paragraph?
increasing their potential. (a) are on increase in the society
(c) There is no measure taken by authorities. (b) can always be reduced.
(d) Smuggling hampers the GOD of a nation. (c) are due to lack of medical facilities.

ww
(e) None of these
DIRECTIONS (Qs. 9 to 11): Study the given information carefully and
answer the questions that follow:
(d) can be eliminated with the help of banning their sale.
(e) may be channelised through proper system.
13. Which of the following is an assumption which is implicit in the

w.E
Exports and imports, a swelling favourable balance of trade, investments
and bank-balances, are not an index or a balance sheet of national
prosperity. Till the beginning of the Second World War, English exports
were noticeably greater than what they are today. And yet England has
facts stated in the above paragraph?
(a) Such drugs are harmful and can lead to severe depression and
suicidal tendencies.
(b) People using drugs harm the society.

asy
greater national prosperity today than it ever had. Because the income of
average Englishmen, working as field and factory labourers, clerks,
policemen, petty shopkeepers and shop assistants, domestic workers
(c) Scaling down the use of such harmful drugs is beneficial for
the society.

En
(d) Medical facilities needs to be improved in our country.
and other low-paid workers, has gone up. (e) The government is not taking appropriate actions to curb the
9. Which of the following is the conclusion which can be drawn use of harmful drugs.
from the facts stated in the above paragraph?
(a) a country’s economic standard can be best adjudged by per
capita income. gin
14. Which of the following is an inference which can be made from
the facts stated in the above paragraph?
(a) Such harmful drugs should not be available at the first place.
(b) a country’s balance of trade is the main criteria of determining
its economic prosperity.
(c) a nation’s economy strengthens with the increase in exports. eer
(b) Controlling drug addicts can scale down the use of drugs.
(c) People using these drugs must be rehabilitated.

(d) English trade has continually increased since the Second


World War.
their usage.
ing
(d) Awareness about the ill-effects of these drugs can scale down

(e) Bad company leads to spread of use of such drugs.


(e) None of these
10. Which of the following is an assumption which is implicit in the
facts stated in the above paragraph? .ne
15. “If you want a hassle-free holiday package for city M, then join
only our tour. Hurry up; only a few seats available” – An
advertisement of XYZ Tourist Company.
(a) Increasing exports and decreasing imports helps a nation
prosper.
(b) Prosperity of a nation is the outcome of prosperity of its
people.
assumed while making the statement?
t
If the above statement is true then which of the following has been

(a) No seats may be available with other tour operators for city
M.
(c) Greater exports leads to higher GDP which leads to better (b) Nowadays people have a lot of money to spend on their
flow of money in the economy. comforts.
(d) Second World War led to deterioration in the national (c) Travel packages offered by other tour operators are neither
prosperity of England. cheap nor comfortable.
(e) None of these (d) Many people desire convenience and comfort while going
11. Which of the following is an inference which can be made from for a holiday.
the facts stated in the above paragraph? (e) None of these
(a) Countries must focus on increasing income of its people and
not an exports or imports.
(b) Favourable balance of trade will automatically lead to higher
income of people.

RESPONSE 7. a b c d e 8. a b c d e 9. a b c d e 10. a b c d e 11. a b c d e

GRID 12. a b c d e 13. a b c d e 14. a b c d e 15. a b c d e

Downloaded From : www.EasyEngineering.net


Downloaded From : www.EasyEngineering.net

Cube & Dice 25


13
Max. Marks : 20 No. of Qs. 20 Time : 15 min. Date : ........./......../. ..............
1. Each of the six faces of a cube is numbered by one of the K H B
digits from 1 to 6. This cube is shown in its four different
positions in the figure I, II, III, and IV. B K H
A M P
6 1 5 6
What is the letter opposite to A?
(a) H (b) P (c) B
3 2 4 2 6 4 2 4

I
ww II III IV
(d) M (e) None of these
DIRECTIONS (Qs. 5 -8) : A cube is coloured red on all faces. It is
cut into 64 smaller cubes of equal size. Now, answer the following

w.E
Consider the following statements.
1. Figures II and III are sufficient to known as to which
face is opposite to the face numbered 6.
questions based on this statement :
5. How many cubes have no face coloured?
(a) 24
(d) 0
(b) 16
(e) None of these
(c) 8

face is opposite to the face numbered 4. asy


2. figures II and III are sufficient to known as to which
.
6. How many cubes are there which have only one face
coloured?
(a) 4 (b) 8 (c) 16
3. Figures I and IV are sufficient to known as to which
face is opposite to the face numbered 3.
Which of the statements given above are correct? En 7.
(d) 24 (e) None of these
How many cubes have two red opposite faces?
(a) 0 (b) 8 (c) 16
(a) 1 and 3 only
(c) 2 and 3 only
(b) 1 and 2 only
(d) 1, 2 and 3 gin
8.
(d) 24 (e) None of these
How many cubes have three faces coloured?
(a) 24 (b) 16 (c) 8
2.
(e) None of these
Six faces of a cube are numbered from 1 to 6, each face
carrying one different number. Further,
(d) 4
eer (e) None of these

1. The face 2 is opposite to the face 6.


2. The face 1 is opposite to the face 5.
3. The face 3 is between the face 1 and the face 5
9.
ing
4. The face 4 is adjacent to the face 2.
Which one of the following is correct?
(a) The face 2 is adjacent to the face 3
How many dots are there on the dice opposite to the one
dot?
(a) 2 (b) 4 (c) 5 .ne
(b) The face 6 is between the face 2 and the face 4
(c) The face 1 is between the face 5 and the face 6
(d) None of the above
(e) None of these
10.
(d) 6 (e) None of these
Select from alternative, the box that can be formed by folding
the sheet shown.
1
t
3. Six squares are coloured, front and back, red (R), blue (B),
yellow (Y), green (G), white (W) and orange (O) and are 4 3 6
hinged together as shown in the figure given below. If they
are folded to form a cube, what would be the face opposite 2
the white face?
5
R B
G Y O 2 4 3
W (a) 3 5 (b) 6 3 (c) 2 5
(a) R (b) G (c) B
(d) O (e) None of these 4
4. Three views of a cube following a particular motion are given (d) 3 (e) None of these
below: 1

RESPONSE 1. a b c d e 2. a b c d e 3. a b c d e 4. a b c d e 5. a b c d e

GRID 6. a b c d e 7. a b c d e 8. a b c d e 9. a b c d e 10. a b c d e

Downloaded From : www.EasyEngineering.net


Downloaded From : www.EasyEngineering.net

50 SPEED TEST 25
11. Select from alternative, the box that can be formed by folding 14. Which number is on the face opposite to 6 on the dice
the sheet shown. whose four positions as shown below ?

6 6 5 1
2 4 4 4 2
3 2 6
×
(a) 1 (b) 2 (c) 3
= ¸ (d) 4 (e) None of these
15. A cube is painted white on all the sides. It is then cut into 64
+ smaller cubes of equal sizes. How many of these smaller
cubes have no paint on any side?
– (a) 8 (b) 6 (c) 4
(d) 1 (e) None of these
× 16. How many cubes are there in the following figure?
(a) ¸ (b) ¸ (c) ¸
+ = ×

(d)
ww –
¸
× (e) None of these

12. w.E
Select from the alternative, the box that can be formed by
folding the sheet shown in figure (X) :

asy
En (a) 6
(d) 8
(b ) 10
(e) None of these
(c) 12

gin
DIRECTIONS (Qs. 17 - 19) : The questions below are to be
answered on the basis of the three views of a cube given as follows:

(X) eer
4
5
1
6
4
3
6
2
5

17.
(a) 3 (b) 2
ing
Which number is on the face opposite to 1 ?
(c) 6

(A) (B) (C) (D) 18.


(d) 4 (e) None of these
Which number is on the face opposite to 4 ? .ne
13.
(a) A only

(e) None of these


(b) A and C only
(c) A , C and D only (d) A, B, C and D

Three positions of a cube are as shown below :


19.
(a) 2
(d) 1

(a) 3
(b) 3
(e) None of these
(c) 6

Which number is at the bottom face of figure 1?


(b) 2 (c) 6
t
(d) 1 (e) None of these
20. Four positions of a cube are shown below. Which symbol
is opposite to the face having ‘ ’?
?
÷ ÷
÷
× + +
×
The figure on the face opposite the triangle is the :
(a) (b) (c) (d)
(a) pentagon (b) circle
(c) question mark (d) rectangle (a) D (b) – (c) ×
(e) None of these (d) ÷ (e) None of these

RESPONSE 11. a b c d e 12. a b c d e 13. a b c d e 14. a b c d e 15. a b c d e

GRID 16. a b c d e 17. a b c d e 18. a b c d e 19. a b c d e 20. a b c d e

Downloaded From : www.EasyEngineering.net


Downloaded From : www.EasyEngineering.net

Non-verbal
Reasoning - I 26
26
13
Max. Marks : 15 No. of Qs. 15 Time : 15 min. Date : ........./......../. ..............
DIRECTIONS (Qs. 1 to 5): In each of the questions given below which one of the five Answer Figures on the right should come after
the Problem Figures on the left, if the sequence were continued?

Problem Figures Answer Figures


1.
S C O B E B S O L O O L B

C
ww
E

O B K L
E
S
L O B K
B
K L
C E S
K
C S
O
E

L
L O
B K C
S K
C
E
O S
E
L C
L

B K
S

C
E
B K
S
E
C B K
L C
E S O
K

w.E (a) (b) (c) (d) (e)

2.
asy
En
gin (a) (b) (c) (d) (e)

3.
eer
ing
.ne
(a) (b) (c) (d) (e)

4. o o
t
c

c
c

o c c o c o o o
c

c o
o o c
c

(a) (b) (c) (d) (e)

5. E L S S S
L L
E E H H L L
=HOLE H K C C L H L L
O H L C H
K H S S H

(a) (b) (c) (d) (e)

RESPONSE 1. a b c d e 2. a b c d e 3. a b c d e 4. a b c d e 5. a b c d e

GRID

Downloaded From : www.EasyEngineering.net


Downloaded From : www.EasyEngineering.net

52 SPEED TEST 26
DIRECTIONS (Qs. 6 to 10) : In each of the following questions a 12. Problem Figures
series begins with an unnumbered figure on the extreme left. One
and only one of the five lettered figures in the series does not fit
into the series. The two unlabelled figures, one each on the extreme ?
left and the extreme right, fit into the series. You have to take as
Answer Figures
many aspects into account as possible of the figures in the series
and find out the one and only one of the five lettered figures which
does not fit into the series. The letter of that figure is the answer.
6.
(a) (b) (c) (d) (e)
s s s s s s s
13. Problem Figures

= ­ ­= ­=
(a) (b)
=
(c)
­­
(d)
= ­=
(e)
= ­
?
Answer Figures
7.
ww N
× ×
N

(a)
w.E
(b) (c) (d) (e)

14.
(a)
Problem Figures
(b) (c) (d) (e)

8.

(a) (b) (c)


×

(d)
asy (e)
×
?
En Answer Figures
9.
c s
c
s c s
s c
s
c
c s c
s
gin
(a) (b) (c) (d) (e)
(a)
eer (b) (c) (d) (e)

10.
15. Problem Figures

? ing
(a) (b) (c) (d)
DIRECTIONS (Qs. 11-15) : In each of the questions given below
(e)
Answer Figures .ne
which one of the five answer figures on the bottom should come
after the problem figures on the top if the sequence were
continued?
t
11. Problem Figures (a) (b) (c) (d) (e)
z s * o =
o z
o s ? =
* *
Answer Figures

o * o * *
z * o s z s * o s o =
(a) (b) (c) (d) (e)

RESPONSE 6. a b c d e 7. a b c d e 8. a b c d e 9. a b c d e 10. a b c d e

GRID 11. a b c d e 12. a b c d e 13. a b c d e 14. a b c d e 15. a b c d e

Downloaded From : www.EasyEngineering.net


Downloaded From : www.EasyEngineering.net

Non-verbal
Reasoning - II 27
27
13
Max. Marks : 15 No. of Qs. 15 Time : 15 min. Date : ........./......../. ..............

DIRECTIONS (Qs. 1 to 5): In each of the questions, there are two 3. Problem Figures
sets of figures. The figures on upper side are problem figures × ×
marked by letters (1), (2), (3), (4) and (5), and on the bottom side
are answer figures marked by numbers (a), (b), (c), (d) and (e). A ? ×
series is established, if one of the five answer figures is placed in
place of the (?) sign in the problem figures. That figure is your (1) (2) (3) (4) (5)
answer.
1.
ww
Problem Figures
Answer Figures

w.E ?
4.
(a) (b)
Problem Figures
(c) (d) (e)
(1) (2)
Answer Figures
(3) (4)
asy (5)

?
En (1) (2) (3) (4) (5)

(a) (b) (c) (d) (e) gin Answer Figures

2. Problem Figures
· S ·
= S S 0 C S C ? eer
=
? C
0
0 ?
C ? ·
=
? =
·
0
5.
(a) (b)
Problem Figures
ing
(c) (d) (e)

(1) (2)
Answer Figures
(3) (4) (5)
? .ne
·
0
S C 0
=
?
· S·
C 0
? = C
S ? S C = S C
= 0
?
= ?
0 ·
·
(1) (2)
Answer Figures
(3) (4) (5)
t
(a) (b) (c) (d) (e)

(a) (b) (c) (d) (e)

DIRECTIONS (Qs. 6 to 10): In each of the following questions, a related pair of figures is followed by five lettered pairs of figures. Select
the pair that has relationship similar to that in the question figure. The best answer is to be selected from a group of fairly close choices.
6.

I II I II I II I II I II I II
(a) (b) (c) (d) (e)

RESPONSE 1. a b c d e 2. a b c d e 3. a b c d e 4. a b c d e 5. a b c d e
GRID

Downloaded From : www.EasyEngineering.net


Downloaded From : www.EasyEngineering.net

54 SPEED TEST 27
7.

I II I II I II I II I II I II
(a) (b) (c) (d) (e)
8.

I II I II I II I II I II I II
(a) (b) (c) (d) (e)
9.

I II I II I II I II I II I II
(a) (b) (c) (d) (e)
10.

I
ww II I II I II I II I II I II

w.E (a) (b) (c)

DIRECTIONS (Qs. 11-15): In each of the questions given below which one of the following five answer figures should come after the
(d) (e)

problem figures, if the sequence were continued?


11. Problem Figures asy Answer Figures
Z O E VP TD SA Z
En P U

P
P

E V P T D SA ZB
gin U U U
U

P
Answer Figures (a) (b) (c) (d) (e)
B B B = = 14.

eer
Problem Figures
C D C D C C
=
(a)
A
(b)
Z =
(c)
A
(d)
Z B
(e)
s Dz
z o Co
z
s s o
s
o ing DD s
z z o
12. Problem Figures
DZ T DO T
Answer Figures
C z o .ne
T
Answer Figures
Z
DO T DZ Z T O
DZ
D C
z o s
(a)
D s z C C
z o o D s D s
(b) (c) (d)
z
o tC
D s
(e)
DZ Z D O 15. Problem Figures
O DO T DZ T Z DZ T
T T O O
(a) (b) (c) (d) (e)
13. Problem Figures Answer Figures
U
P U
U
U

P
P
U

P
P

(a) (b) (c) (d) (e)

RESPONSE 6. a b c d e 7. a b c d e 8. a b c d e 9. a b c d e 10. a b c d e
GRID 11. a b c d e 12. a b c d e 13. a b c d e 14. a b c d e 15. a b c d e

Downloaded From : www.EasyEngineering.net


Downloaded From : www.EasyEngineering.net

Section Test :
Reasoning 28
13
28
Max. Marks : 40 No. of Qs. 40 Time : 25 min. Date : ........./......../. ..............
1. How many meaningful three letter English words can be 7. In a certain code DISPLAY is written as RHCQZBM. How is
formed with the letters AER, using each letter only once in GROUPED written in that code?
each word ? (a) PSHTEFQ (b) NQFVCDO
(a) None (b) One (c) NQFVEFQ (d) PSHTCDO
(c) Two (d) Three (e) None of these
(e) Four 8. Among P, Q, R, T and W each having different weight, T is
2.
ww
If in a certain language LATE is coded as 8 & 4 $ and HIRE
is coded as 7*3$ then how will HAIL be coded in the same
heavier than W and lighter than only P. Q is not the lightest.
Who among them is definitely the lightest ?
language ?
(a) 7 & 8*
(c) 7*& 8 w.E (b) &7*8
(d) 7&*8
(a) R
(c) R or W
(e) None of these
(b) W
(d) Data inadequate

3.
(e) None of these

asy
Four of the following five are alike in a certain way and so
form a group. Which is the one that does not belong to that
9. In a row of thirty children M is sixth to the right of R who is
twelfth from the left end. What is M’s position from the right
end of the row ?
group ?
(a) Stem (b) Tree En (a) Twelfth
(c) Fourteenth
(b) Thirteenth
(d) Data inadequate
(c) Root
(e) Leaf
(d) Branch
gin
10.
(e) None of these
What should come next in the following letter series?
4. Four of the following five are alike in a certain way and so
form a group. Which is the one that does not belong to that
group ?
(a) B
(c) M eer
B D F H J L N A C E G I K M B D F H J LA C E G I K B D F H J
(b) L
(d) F
(a) Clutch
(c) Break
(b) Wheel
(d) Car
(e) None of these
ing
DIRECTIONS (Qs. 11-15) : Study the following arrangement

5.
(e) Gear
How many such pairs of letters are there in word SENDING,
carefully and answer the questions given below
.ne
M, D, K, R, T, H, W and A are sitting around a circle facing at the
each of which has as many letters between its two letters as
there are between them in the English alphabets ?
(a) None
(c) Two
(b) One
(d) Three
second to the right of W.
11. Who is second to the right of A ?
t
centre. D is second to the right of M who is fifth to the left of T. K
is third to the right of R who is second to the right of D. H is

(e) More than three


(a) M (b) D
6. Four of the following five are alike in a certain way and so
(c) K (d) Data inadequate
form a group. Which is the one that does not belong to that
(e) None of the above
group?
12. Who is third to the left of M ?
(a) 169 (b) 441
(a) A (b) T
(c) 361 (d) 529
(c) H (d) D
(e) 289
(e) Data inadequate

1. a b c d e 2. a b c d e 3. a b c d e 4. a b c d e 5. a b c d e
RESPONSE 6. a b c d e 7. a b c d e 8. a b c d e 9. a b c d e 10. a b c d e
GRID 11. a b c d e 12. a b c d e

Downloaded From : www.EasyEngineering.net


Downloaded From : www.EasyEngineering.net

56 SPEED TEST 28
13. Who is fourth to the right of H ? 19. Statements:
(a) A (b) T All puppies are tigers.
(c) R (d) K All kittens are tigers.
(e) None of these Conclusions:
14. In which of the following combinations is the first person I. All puppies are kittens.
sitting between the second and the third person ? II. All tigers are puppies.
(a) KMW (b) MWD 20. Statements: Some doctors are nurses.
(c) RHT (d) TAK All nurses are patients.
(e) None of the above Conclusions:
15. In which of the following pairs is the second person sitting I. All doctors are patients.
to the immediate left of the first person ? II. Some patients are doctors.
(a) MW (b) AK DIRECTIONS (Qs.21-25) : Study the following information to
(c) TA (d) RH answer the given questions :
(e) WD Six plays – A, B, C, D, E and F are to be staged on six days of the
DIRECTIONS (Qs. 16 - 20): In each question below, there are week starting from Monday and ending on Saturday. Play C is

ww
three of two statements followed by two conclusions numbered
I and II. You have to take the three given statements to be true
even if they seem to be at variance from commonly known facts
staged on Tuesday. Plays A, F and B are staged one after the
other in the same order. Play D is not staged on Monday or
Wednesday.

from the three statements.w.E


and then decide which of the given conclusions logically follow

Give answer (a) if only conclusion I follows.


21. How many plays are staged after play A is staged ?
(a) One (b) Two

asy
Give answer (b) if only conclusion II follows.
Give answer (c) if either I or II follows.
(c) Three
(e) Cannot be determined
(d) Four

22. Four of the following five form a group based on the days
Give answer (d) if neither I nor II follows.
Give answer (e) if both I and II follow.
En that they are staged. Which one of them does not belong to
that group ?
16. Statements:
All fish are birds.
Some hens are fish.
gin (a) EC
(c) CA
(b) FD
(d) AF

Conclusion:
I. Some hens are birds.
(e) BD

eer
23. Which play is staged immediately before the day play E is
staged ?
II. No birds are hens;
17. Statements:
(a) B
(c) F ing
(b) A
(d) D
Some shoes are coats.
Some coats are buttons
Conclusions:
(e) There is no such play staged

.ne
24. If play D was staged on a Monday, which of the following

I. No button is shoe.
II. Some shoes are buttons.
18. Statements:
conditions given above remain the same) ?
(a) B
(c) E
(b) A
(d) E or B
t
plays would definitely be staged on a Saturday (all the other

All bats are boys. (e) Cannot be determined


All boys are gloves. 25. Which play is staged on Thursday ?
Conclusions: (a) B (b) E
I. Some gloves are bats. (c) D (d) F
II. All bats are gloves.. (e) Cannot be determined

13. a b c d e 14. a b c d e 15. a b c d e 16. a b c d e 17. a b c d e


RESPONSE 18. a c d e 19. a c d e 20. a c d e 21. a c d e 22. a c d e
b b b b b
GRID
23. a b c d e 24. a b c d e 25. a b c d e

Downloaded From : www.EasyEngineering.net


Downloaded From : www.EasyEngineering.net

SPEED TEST 28 57

DIRECTIONS (Qs.26-28) : In each question a group of letters is 30. Which of the following is an Assumption which is implicit in
given followed by four combinations of number/symbol numbered the facts stated in the above paragraph
(a), (b), (c) and (d). Letters are to be coded as per the scheme and (a) Whatever you do you cannot protect yourself from
conditions given below. You have to find out the serial number of fashion.
the combination, which represents the letter group. Serial number (b) Fashion changes the outward appearance and makes
of that combination is your answer. If none of the combinations is one feel better.
correct, your answer is (e) i.e. None of these. (c) Today's youth is highly influenced by fashion.
(d) Normal clothing is good as it keeps a person grounded.
Letters Q M S I N G D K A L P R B J E (e) Fashion is spoiling the minds of the youngsters.
Number/ 7 @ 4 # % $ 6 1 2 £ 5 * 9 8 3
Symbol
31. Which of the following is an Inference which can be made
from the facts stated in the above paragraph?
Conditions : (a) Moderate investment of time and money by youth is
(i) If the first letter is a consonant and the last a vowel, both are acceptable.
to be coded as the code of the vowel.
(b) Fashion brings with itself feelings of materialism.
(ii) If the first letter is vowel and the last a consonant, the codes
(c) All of us have to adopt ourselves to latest fashion to

ww
for the first and the last are to be interchanged.
(iii) If no vowel is present in the group of letters, the second and
the fifth letters are to be coded as ©.
some bit.
(d) Fashion helps in projecting a better image of one's
personality.
26. BARNIS
(a) 9 2 * % # 4
(c) 9 2 * # % 9
w.E (b) 9 2 4 # * %
(d) 4 2 * # % 4
(e) People who will avoid fashion completely will be
outdated.
DIRECTIONS (Qs. 32-35) % In the following questions, the symbols
(e) None of these
27. DMBNIA
(a) 6 @ 9 % # 2 (b) 2 @ 9 % # 6
asy @, #, $, © and % are used with the following meaning as illustrated
below :
(c) 2 @ 9 % # 6
(e) None of these
(d) 2 © 9 % # 2
En 'P@ Q' means ‘P is not smaller than Q’,

28. IJBRLG
(a) # 8 9 * £ $ (b) # 8 9 * £ # gin 'P# Q' means ‘P is not greater than Q’.
'P$ Q' means ‘P is neither greater than nor equal to Q’.
(c) $ 8 9 * £ #
(e) None of these
(d) $ 8 9 * £ $

eer
'P© Q' means ‘P is neither smaller than nor equal to Q’.
'P% Q' means ‘P is neither smaller than nor greater than Q’.
DIRECTIONS (Qs. 29-31) : Study the given information carefully
and answer the questions that follow:
Though the waste of time or the expenditure on fashions is very
ing
Now in each of the following questions, assuming the given
statements to be true, find which of the two conclusions I
and II given below them is/are definitely true.
large, yet fashions have come to stay. They will not go, come what
may. However, what is now required is that strong efforts should .ne
Give answer (a) if only conclusion I is true.
Give answer (b) if onlyconclusion II is true.
be made to displace the excessive craze for fashion from the minds
of these youngsters.
29. Which of the following is the conclusion which can be drawn
from the facts stated in the above paragraph?
true.
Give answer (e) if both conclusions I and II are true.
t
Give answer (c) if either conclusion I or conclusion II is true.
Give answer (d) if neither conclusion I nor conclusion II is

(a) Fashion is the need of the day. 32. Statements: % V $ W, W@T, T#H
(b) The excessive craze for fashion is detrimental to one's Conclusions: I. V©T
personality. II. H%W
(c) The hoard for fashion should be done away with so as 33. Statements: H © M, M @ E, E $ C
not to let down the constructive development.
Conclusions: I. C@M
(d) Work and other activities should be valued more than
the outward appearance. II. H©E
(e) None of these

26. a b c d e 27. a b c d e 28. a b c d e 29. a b c d e 30. a b c d e


RESPONSE
31. a b c d e 32. a b c d e 33. a b c d e
GRID

Downloaded From : www.EasyEngineering.net


Downloaded From : www.EasyEngineering.net

58 SPEED TEST 28
34. Statements: N@J, J%R, R©H 38. How many such symbols are there in the above arrangement
Conclusions: I. R#N each of which is immediately preceded by a number and
immediately followed by a letter?
II. N©H
(a) None (b) One
35. Statements: L@K, K©A, A$W
(c) Two (d) Three
Conclusions: I . W$L (e) More than three
II. L#W 39. If all the numbers are dropped from the above arrangement,
DIRECTIONS (Qs. 36-40) : Study the following arrangement which of the following will be the eleventh from the left end?
carefully and answer the questions given below (a) B (b) H
D 5 d R @A K © 3 9 B J E* F $ M PI 4 H 1 W 6 (c) $ (d) *
2 # UQ 8 T N (e) None of these
36. How many such numbers are there in the above arrangement 40. Four of the following five are alike in a certain way based on
each of which is immediately preceded by a symbol and their positions in the above arrangement and so form a group.
immediately followed by a letter? Which is the one that does not belong to that group?
(a) None (b) One (a) 41I (b) 6#W

37.
ww
(c) Two
(e) More than three
(d) Three

Which of the following is the ninth to the right of the twenty


(c) QTU
(e) J9E
(d) RAd

(a) E w.E
second from the right end of the above arrangement ?
(b) I
(c) D
(e) None of these
(d) N

asy
En
gin
eer
ing
.ne
t

34. a b c d e 35. a b c d e 36. a b c d e 37. a b c d e 38. a b c d e


RESPONSE
GRID 39. a b c d e 40. a b c d e

Downloaded From : www.EasyEngineering.net


Downloaded From : www.EasyEngineering.net

Number System - I 29
13
Max. Marks : 30 No. of Qs. 30 Time : 20 min. Date : ........./......../. ..............
1. Which is the smallest of the following numbers ? 8. How many of the following numbers are divisible by 132 ?
264, 396, 4, 762, 792, 968, 2178, 5184, 6336
2 5 (a) 4 (b) 5 (c) 6
(a) (b) – 1.5 (c)
3 3 (d) 7 (e) None of these
(d) – 1.375 (e) None of these 9. Find the number which, when added to itself 13 times, gives
2. Three-fourth of 68 is less than two-thirds of 114 by 112.

3.
(a) 12
(d) 48
ww (b) 25
(e) None of these
(c) 35

If one-third of a number is 3 more than one -fourth of the 10.


(a) 9
(d) 11
(b) 7
(e) None of these
(c) 8

The smaller number by which 3600 can be divided to make it

(a) 18
(d) 36 w.E
number, then the number is :
(b) 24
(e) None of these
(c) 30
a perfect cube is :
(a) 9
(d) 450
(b) 50
(e) None of these
(c) 300

4. Which is the smallest of the following numbers ?

(a) 7 (b)
1 asy
(c)
7
11. The least number having four digits which is a perfect square
is :
(a) 1004 (b) 1016 (c) 1036

1
7 7

En 12.
(d) 1226 (e) None of these
A positive integer, which added to 1000, gives a sum which is

5.
(d)
7
(e) None of these

If one-eighth of a pencil is black, half of the remaining is gin


greater than the product obtained when it is multiplied by
1000. The positive integer is :
(a) 1 (b) 5 (c) 2
1
yellow and the remaining 3 cm is blue, then the length of
2
(d) 3
eer (e) None of these
3 2
the pencil is :
(a) 6 cm
(d) 11 cm
(b) 7 cm
(e) None of these
(c) 8 cm
13.

ing
Of the 120 people in the room,
5
are women. If
3
of the

people are married, then what is the maximum number of

6. A boy was asked to write 25 ´ 9 2 but he wrote 2592. The


women in the room who are unmarried?
(a) 40
(d) 80
(b) 50
(e) None of these .ne
(c) 60

7.
numerical difference between the two is:
(a) 0
(d) 9
(b) 3
(e) None of these
(c) 2

In a group of buffaloes and ducks, the number of legs are


14. How many odd numbered pages are there in a book of 1089
pages ?
(a) 545
(d) 547
(b) 544
(e) None of these
(c) 543
t
24 more than twice the number of heads. What is the number
of buffaloes in the group ? 15. If p is a prime number greater than 3, then (p 2 - 1) is always
(a) 6 (b) 12 (c) 8
divisible by :
(d) 15 (e) None of these
(a) 6 but not 12 (b) 24 (c) 12 but not 24
(d) 16 (e) None of these

1. a b c d e 2. a b c d e 3. a b c d e 4. a b c d e 5. a b c d e
RESPONSE 6. a b c d e 7. a b c d e 8. a b c d e 9. a b c d e 10. a b c d e
GRID 11. a b c d e 12. a b c d e 13. a b c d e 14. a b c d e 15. a b c d e

Downloaded From : www.EasyEngineering.net


Downloaded From : www.EasyEngineering.net

60 SPEED TEST 29
16. A man has 1044 candles. After burning all of them he can 24. What is the least number to be added to 1500 to make it a
make a new candle from 9 stubs left behind. Find the maximum perfect square?
number of candles that can be made : (a) 20 (b) 21 (c) 22
(a) 116 (b) 120 (c) 130
(d) 23 (e) None of these
(d) 136 (e) None of these
17. Find the whole number which when increased by 20 is equal 25. How many pieces of 8.6 metres length cloth can be cut out
to 69 times the reciprocal of the number: of a length of 455.8 metres cloth?
(a) 7 (b) 5 (c) 3 (a) 43 (b) 48 (c) 55
(d) 2.5 (e) None of these (d) 53 (e) 62
18. A clock strikes 4 taking 9 seconds. In order to strike 12 at the 26. If an amount of ` 15,487 is divided equally among 76 students,
same rate, the time taken is approximately how much amount will each student get?
(a) 36 seconds (b) 27 seconds (c) 30 seconds (a) `206 (b) `210 (c) `204
(d) 33 seconds (e) None of these (d) `218 (e) `212
19. The sum of the place values of 3 in the number 50, 35, 35 is 27. The difference between a number and one fifth of it is 84.
(a) 3300 (b) 6 (c) 60 What is the number?
(d) 3030 (e) None of these (a) 95 (b) 100 (c) 105
20.

ww
How many numbers are there between 500 and 600 in which
9 occurs only once?
(a) 19 (b) 18 (c) 20
28.
(d) 108 (e) 112
A, B, C, D and E are five consecutive odd numbers The sum
of A and C is 146. What is the value of E?

21.
(d) 21

w.E
(e) None of these
The product of two consecutive even numbers is 12768.
What is the greater number ? 29.
(a) 75
(d) 79
(b) 81
(e) None of these
(c) 71

The sum of five consecutive even numbers of set A is 280.


(a) 110
(d) 112
(b) 108
(e) None of these asy
(c) 114 What is the sum of a different set B of five consecutive
numbers whose lowest number is 71 less than double the
lowest number of set A ?
22. The product of two consecutive odd numbers is 19043.
Which is the smaller number?
En (a) 182
(d) 175
(b) 165
(e) None of these
(c) 172

23.
(a) 137
(d) 129
(b) 131
(e) None of these
(c) 133

The product of two successive numbers is 8556. What is the gin


30. Deepak has some hens and some goats. If the total number
of animal heads is 90 and the total number of animal feet is
248, what is the total number of goats Deepak has ?
smaller number?
(a) 89 (b) 94 (c) 90
(a) 32
eer (b) 36
(d) Cannot be determined
(c) 34

(d) 92 (e) None of these (e) None of these

ing
.ne
t

16. a b c d e 17. a b c d e 18. a b c d e 19. a b c d e 20. a b c d e


RESPONSE 21. a b c d e 22. a b c d e 23. a b c d e 24. a b c d e 25. a b c d e
GRID 26. a b c d e 27. a b c d e 28. a b c d e 29. a b c d e 30. a b c d e

Downloaded From : www.EasyEngineering.net


Downloaded From : www.EasyEngineering.net

Number System - II 30
13
Max. Marks : 30 No. of Qs. 30 Time : 20 min. Date : ........./......../. ..............

2 8. Given, n = 1 + x, where x is the product of four consecutive


1. Farah got married 8 years ago. Today her age is 1 times
7 integers. Then which of the following is true ?
her age at the time of her marriage. At present her daughter's A. n is an odd integer
age is one-sixth of her age. What was her daughter's age 3 B. n is prime.
years ago? C. n is a perfect square
(a) Both A and C are correct.

2.
ww
(a) 6 years
(c) 3 years
(e) None of these
(b) 2 years
(d) Cannot be determined

If (12)3 is subtracted from the square of a number the answer


(b) Both A and B are correct.
(c) Only A is correct
(d) Only C is correct

(d) 52
w.E
so obtained is 976. What is the number?
(a) 58 (b) 56
(e) None of these
(c) 54
9.
(e) None of these
9 6 + 7, when divided by 8, would have a remainder :
3.
of 10 chairs and 16 tables?
(a) ` 15674 (b) ` 16435
asy
The cost of 5 chairs and 8 tables is ` 6,574. What is the cost

(c) ` 13148 10.


(a) 0
(d) 4
(b) 6
(e) None of these
(c) 5

The remainder when 7 84 is divided by 342 is :


(d) Cannot be determined
(e) None of these En (a) 0
(d) 340
(b) 1
(e) None of these
(c) 21

4. If (56)2 is added to the square of a number, the answer so


obtained is 4985. What is the number?
(a) 52 (b) 43 (c) 65
11.
gin
Students of a class are preparing for a drill and are made to
stands in a row. If 4 students are extra in a row, then there

5.
(d) 39 (e) None of these
The unit’s digit in the product 7 35 ´ 371 ´ 1155 is : eer
would be 2 rows less. But there would be 4 more rows if 4
students are less in a row. The number of students in the
class is :
(a) 1
(d) 9
(b) 3
(e) None of these
(c) 7
12.
(a) 96
(d) 65
ing
(b) 56
(e) None of these
(c) 69

Three friends divided some bullets equally. After each of them


6. If p be a number between 0 and 1, which one of the following
will be true ?
.ne
shot 4 bullets, the total number of bullets remaining is equal
to the bullets each had after division . Find the original number
(a)

(c)
p> p

p <
1
(b)

(d)
1
p2
> p

p3 > p 2
13.
of bullets:
(a) 15
(d) 18
(b) 17
(e) None of these
(c) 20
t
If x and y are negative, then which of the following statements
p is/are always true ?
I. x + y is positive
(e) None of these
II. xy is positive
7. In a certain shop, 9 oranges cost as much as 5 apples, 5
III. x – y is positive
apples cost as much as 3 mangoes, 4 mangoes cost as
(a) I only (b) II only (c) III only
much as 9 lemons. If 3 lemons cost 48 paise, the price of an
(d) I and II only (e) None of these
orange is :
14. A student was asked to divide a number by 6 and add 12 to
(a) 12 paise (b) 14 paise (c) 13 paise
the quotient. He, however, first added 12 to the number and
(d) 15 paise (e) None of these
then divided it by 6, getting 112 as the answer. The correct
answer should have been :
(a) 122 (b) 118 (c) 114
(d) 124 (e) None of these
1. a b c d e 2. a b c d e 3. a b c d e 4. a b c d e 5. a b c d e
RESPONSE 6. a b c d e 7. a b c d e 8. a b c d e 9. a b c d e 10. a b c d e
GRID 11. a b c d e 12. a b c d e 13. a b c d e 14. a b c d e

Downloaded From : www.EasyEngineering.net


Downloaded From : www.EasyEngineering.net

62 SPEED TEST 30
22. The letters L, M, N, O, P, Q, R, S and T in their order are
1
15. If Harry is rd the age of his father, George now, and was substituted by nine integers 1 to 9 but not in that order. 4 is
3 assigned to P. The difference between P and T is 5. The
1 difference between N and T is 3. What is the integer
th the age of his father 5 years ago, then how old will assigned to N?
4
(a) 4 (b) 5 (c) 6
George be 5 years from now ?
(d) 7 (e) None of these
(a) 20 years (b) 45 years (c) 40 years
23. A number when divided by 765 leaves a remainder 42. What
(d) 50 years (e) None of these
will be the remainder if the number is divided by 17 ?
16. On Monday, a certain animal shelter housed 55 cats and
(a) 8 (b) 7 (c) 6
dogs. By Friday, exactly 1/5 of the cats and 1/4 of the dogs
(d) 5 (e) None of these
had been adopted; no new cats or dogs were brought to the
24. If m and n are natural numbers such that 2m – 2n = 960, what
shelter during this period. What is the greatest possible
is the value of m ?
number of pets that could have been adopted from the animal
(a) 10 (b) 12 (c) 16
shelter between Monday and Friday ?
(d) Cannot be determined
(a) 11 (b) 12 (c) 13
(e) None of these
(d) 14 (e) None of these
25. At the first stop on his route, a driver uploaded 2/5 of the
17.

ww
The number 6n2 + 6n, for any natural number n, is always
divisible by :
(a) 6 only (b) 18 only (c) 12 only
packages in his van. After he uploaded another three
packages at his next stop, 1/2 of the original number of
packages remained. How many packages were in the van
18.
(d) 6 and 12

w.E
(e) None of these
A bus starts from city X. The number of women in the bus is
half of the number of men. In city Y, 10 men leave the bus and
five women enter. Now, the number of men and women is
before the first delivery?
(a) 25
(d) 36
(b) 10
(e) None of these
(c) 30

bus? asy
equal. In the beginning, how many passengers entered the
26. A divisor is
2
3
of the dividend and 2 times the remainder If

19.
(a) 15
(d) 45
(b) 30
(e) None of these
(c) 36

En
In a class, there are 18 boys who are over 160 cm tall. If these
the remainder is 75, then find the dividend.
(a) 85 (b) 145 (c) 225

constitute three-fourths of the boys and the total number of


boys is two-thirds of the total number of students in the gin
27.
(d) 65 (e) None of these
The quotient arising from the division of 24162 by a certain
number x is 89 and the remainder is 43. Find x.
class, then what is the number of girls in the class?
(a) 6
(d) 24
(b) 12
(e) None of these
(c) 18
28.
(a) 271
(d) 251
eer (b) 281
(e) None of these
(c) 261

A boy multiplied a certain number x by 13. He found that the


20. In a family, a couple has a son and a daughter. The age of the
father is three times of his daughter and the age of the son is smallest value of x. ing
resulting product consisted of all nines entirely. Find the

half of his mother. The wife is nine years younger to her


husband and the brother is seven years older than his sister.
What is the age of the mother? 29.
(a) 76913
(d) 75933
(b) 76933
(e) None of these
.ne(c) 76923

Find the unit’s digit in the product (2467)153 × (341)72.

21.
(a) 40 years
(d) 60 years
(b) 50 years
(e) None of these
(c) 45 years

Soma purchases National Savings Certificates every year


whose value exceeds the previous year’s purchase by Rs.
30.
(a) 6
(d) 9
(b) 7
(e) None of these
What is the digit in the unit place of 251 ?
(a) 2 (b) 8
(c) 8

(c) 1
t
400. After 8 years, she finds that she has purchased certificates (d) 4 (e) None of these
whose total face value is Rs. 48,000. What is the face value of
the certificates purchased by her in the first year ?
(a) Rs. 4,300 (b) Rs. 4,500 (c) Rs. 4,400
(d) Rs. 4,600 (e) None of these

15. a b c d e 16. a b c d e 17. a b c d e 18. a b c d e 19. a b c d e


RESPONSE 20. a b c d e 21. a b c d e 22. a b c d e 23. a b c d e 24. a b c d e
GRID 25. a b c d e 26. a b c d e 27. a b c d e 28. a b c d e 29. a b c d e
30. a b c d e

Downloaded From : www.EasyEngineering.net


Downloaded From : www.EasyEngineering.net

LCM, HCF and


Fraction
31
31
13
Max. Marks : 30 No. of Qs. 30 Time : 20 min. Date : ........./......../. ..............
1. If the L.C.M and H.C.F. of two numbers are 2400 and 16, 8. Four runners started running simultaneously from a point on
one number is 480; find the second number. a circular track they took 200 sec, 300 sec, 360 sec and 450 sec
(a) 40 (b) 80 (c) 60 to complete one round, after how much time do they meet at
(d) 50 (e) None of these the starting point for the first time?
2. The L.C.M. of two number is 630 and their H.C.F. is 9. If the (a) 1800 sec (b) 3600 sec (c) 2400 sec
sum of numbers is 153, their difference is (d) 4800 sec (e) None of these

3.
(a) 17
(d) 33
ww (b) 23
(e) None of these
(c) 27

The H.C.F and L.C.M of two numbers are 21 and 4641


9. Philip, Tom and Brad start jogging around a circular field and
complete a single round in 18, 22 and 30 seconds respectively,
In how much time, will they meet again at the starting point ?

w.E
respectively. If one of the numbers lies between 200 and
300, then the two numbers are
(a) 273, 357 (b) 273, 361 (c) 273, 359
(a) 3 min 15 sec
(c) 16 min 30 sec
(e) None of these
(b) 21 min
(d) 12 min

4.
(d) 273, 363 (e) None of these

asy
Suppose you have 108 green marbles and 144 red marbles.
You decide to separate them into packages of equal number
10. The numbers 11284 and 7655, when divided by a certain
number of three digits, leave the same remainder. Find that
number of three digits.
of marbles. Find the maximum possible number of marbles
in each package. En (a) 161
(d) 191
(b) 171
(e) None of these
(c) 181

5.
(a) 4
(d) 12
(b) 36
(e) None of these
(c) 9

One pendulum ticks 57 times in 58 seconds and another


11.

gin
The LCM of two numbers is 2079 and their HCF is 27. if one of
the numbers is189, find the other.
(a) 273 (b) 279 (c) 297
608 times in 609 seconds. If they started simultaneously,
find the time after which they will tick together. 12.
(d) 307
eer (e) None of these
Find the least number which, when divided by 18, 24, 30 and

(a)
211
19
s (b)
1217
19
s (c)
1218
19
s (a) 2521
(d) 2473 ing
42, will leave in each case the same remainder 1.
(b) 2556
(e) None of these
(c) 2456

(d)
1018
19
s (e) None of these
13.
.ne
The HCF and LCM of two numbers are 44 and 264 respectively.
If the first number is devided by 2, the quotient is 44. What is

6. From 3 drums of milk, 271, 331 and 451 respectively are to


be drawn out. To do it in a minimum time, the capacity of
the measuring can be
(a) 271 (b) 61 (c) 111
14.
the other number?
(a) 108
(d) 132
(b) 44
(e) None of these
(c) 124

The product of two number si 2160 and their HCF is 12. Find
the possible pairs of numbers.
t
(d) 31 (e) None of these (a) 1 (b) 2 (c) 3
7. Product of two co-prime numbers is 117. Their L.C.M. should (d) 4 (e) None of these
be: 15. Three bells toll at intervals of 9, 12 and 15 minutes respectively.
(a) 1 (b) 117 All the three begin to toll at 8 a.m. At what time will they toll
(c) equal to their H.C.F. together again?
(d) cannot be calculated (a) 8.45 a.m. (b) 10. 30 a.m. (c) 11.00 a.m.
(e) None of these (d) 1. 30 p.m. (e) None of these

1. a b c d e 2. a b c d e 3. a b c d e 4. a b c d e 5. a b c d e
RESPONSE 6. a b c d e 7. a b c d e 8. a b c d e 9. a b c d e 10. a b c d e
GRID 11. a b c d e 12. a b c d e 13. a b c d e 14. a b c d e 15. a b c d e

Downloaded From : www.EasyEngineering.net


Downloaded From : www.EasyEngineering.net

64 SPEED TEST 31
16. Four metal rods of lengths 78 cm, 104 cm, 117 cm and 169 cm 31
21 31
are to be cut into parts of equal length. Each part must be as (a) (b) (c)
long as possible. What is the maximum number of pieces 38 38 48
that can be cut? 17
(a) 27 (b) 36 (c) 43 (d) (e) None of these
48
(d) 400 (e) 402
17. The LCM and HCF of two numbers are 84 and 21, respectively. 9 2 8 5
24. If the fractions , , , are arranged in ascending
If the ratio of two numbers be 1 : 4, then the larger of the two 13 3 11 7
numbers is : order, then the correct sequence is :
(a) 21 (b) 48 (c) 84
(d) 108 (e) None of these 9 2 8 5 2 9 5 8
(a) , , , (b) , , ,
18. Which of the following pair of fractions adds up to a number 13 3 11 7 3 13 7 11
greater than 5 ?
2 8 5 9 5 8 2 9
13 11 11 8 7 11 (c) , , , (d) , , ,
(a) , (b) , (c) , 3 11 7 13 7 11 3 13
5 6 4 3 3 5 (e) None of these
5 3 25. If the numerator and the denominator of a proper fraction are

19.
(d) ,
3 4
ww (e) None of these
Which of the following fractions is the least ?
increased by the same quantity, then the resulting fraction is:
(a) always greater than the original fraction
(b) always less than the original fraction
(a)
12
119
7
w.E (b)
1
10
(c)
4
39
26.
(c) always equal to the original fraction
(d) Can’t be determined
(e) None of these
If x is a positive number, then which of the following fraction
(d)
69
(e) None of these

asy 4
has the greatest value ?
x ( x + 1) x
20.
æ 2ö
Which of the following is not the reciprocal of ç ÷ ?
è 3ø
En (a)
x
(b)
x
(c)
( x + 1)

(a)
æ 3ö
ç ÷
è 2ø
4
(b)
æ 2ö
ç ÷
è 3ø
-4
(c)
æ 3ö
ç ÷
è 2ø
-4
gin
27.
(d)
( x + 2)
(x + 3)
(e) None of these

Four bells begin to toll together and toll respectively at in-

(d)
3 4
(e) None of these eer
tervals of 6, 5, 7, 10 and 12 seconds. How many times they
will toll together in one hour excluding the one at the start ?

21.
24
A certain type of wooden board is sold only in lengths of
multiples of 25 cm from 2 to 10 metres. A carpenter needs a 28.
(a) 7 times
(d) 11 times
ing
(b) 8 times
(e) None of these
(c) 9 times

HCF of 3240, 3600 and a third number is 36 and their LCM is


large quantity of this type of boards in 1.65 meter length. For
the minimum waste, the lengths to be purchased should be: .ne
2 4 ´ 35 ´ 5 2 ´ 7 2 . The third number is :

22.
(a) 3.30 m
(d) 9.95 m
(b) 6.60 m
(e) None of these
(c) 8.25 m

Which of the following group of fractions is arranged in


ascending order ?
29.
(a) 2 4 ´ 53 ´ 7 2
(c) 2 3 ´ 35 ´ 7 2
(e) None of these
(b) 2 2 ´ 35 ´ 7 2
(d) 25 ´ 52 ´ 7 2

The LCM of two numbers is 4800 and their HCF is 160. If one
t
5 7 6 7 6 5
(a) , , (b) , , of the numbers is 480, then the other number is :
16 18 17 18 17 16 (a) 16 (b) 16000 (c) 160
5 6 7 6 7 5 (d) 1600 (e) None of these
(c) , , (d) , , 30. The traffic lights at three different road crossings change
16 17 18 17 18 16
after every 48 seconds, 72 seconds and 108 seconds
(e) None of these respectively. If they all change simultaneously at 8 : 20 hours,
23. What is the least fraction which, when added to or subtracted then at what time will they again change simultaneously ?
29 15 (a) 8 : 20 : 08 hrs (b) 8 : 24 : 10 hrs
from + , will make the result a whole number ? (c) 8 : 27 : 12 hrs (d) 8 : 30 : 15 hrs
12 16
(e) None of these

16. a b c d e 17. a b c d e 18. a b c d e 19. a b c d e 20. a b c d e


RESPONSE 21. a b c d e 22. a b c d e 23. a b c d e 24. a b c d e 25. a b c d e
GRID 26. a b c d e 27. a b c d e 28. a b c d e 29. a b c d e 30. a b c d e

Downloaded From : www.EasyEngineering.net


Downloaded From : www.EasyEngineering.net

Square, Cube,
Indices & Surds
32
Max. Marks : 40 No. of Qs. 40 Time : 25 min. Date : ........./......../. ..............

DIRECTIONS (Qs. 1-25): What should come in place of the 10. 7 ´ 447 ¸ 21 + 73 - 26 = ?
question mark (?) in the following questions? (a) 196 (b) 16 (c) 13
(d) 169 (e) None of these
1. 1190 ¸ 7225 ´ ? = 3094
(a) 221 (b) 121 (c) 214 ? 15 ´ 4 - 40
11. =
(d) 241 (e) None of these 25 2

2. ( ww
5 -1 )
2
= ?- 2 5
(a) 20
(d) 50
(b) 45
(e) None of these
(c) 25

(a) 6
(d) 6-2 5 w.E (b) 6+2 5
(e) None of these
(c) 6 5 12. ( 656 ¸ 164 )2 =
(a) 14
(d) 256
?
(b) 16
(e) None of these
(c) 64

3.
?
36
=
25
(11´ 3 - 18 ) asy 13. 255 ¸ 17 ¸ 5 = ( ? )
2

(a) 2
(d) 6
(b) 4
(e) None of these
(c) 5
En (a) 9
(d) 27
(b)
(e) 81
3 (c) 3

4. 12 ´ 145 ¸ 6 + 34 = ?
(a) – 18 (b) (324)2 (c) 18 gin
14. (32 × 42 × 5) ¸ 36 = (?)2 – 80
(a) (100)2 (b) 10 (c) 100

5.
(d) 18

(13)2 - ( 5)2 -
(e) None of these

676 + 7 = ( ? )
2 15.
(d) 10

( 6 +1 )
2
eer
= ?+ 2 6
(e) None of these

(a) 10
(d) (25)2
(b) 20 (c) 5 (a) 7
ing (b) 6 (c) 4 6+7

6. (16 ) 2
- 53 + 169 = ( ? )
2
(e) 5

16.
(d) 4 6
12 ´184 ¸ 23 + 26 - 73 = ?
(e) None of these
.ne
7.
(a) (12)2
(d) (144)2

225 + 2304 = ? - (12 )


(b) 144
(e) None of these
2
(c) 12 (a) 7
(d) (49)2
(b) ( 7 )2
(e) None of these
(c) 7
t
17. (13)2 - ( 4 )3 - 676 + 2 = ( ? )
2
(a) 205 (b) 207 (c) 206
(a) 3 (b) 9 (c) 81
(d) 208 (e) None of these
(d) 27 (e) 18
8. 450 + 890 + 685 = ?
(a) 43 (b) 45 (c) 55 18. ( 74 ´ ) (
676 - 42 ´ ? = 496 )
(d) 53 (e) None of these (a) 1024 (b) 1296 (c) 1156
9. 3 6859 = ? - 4 (d) 1089 (e) None of these
(a) 26 (b) 25 (c) 23
(d) 22 (e) None of these

1. a b c d e 2. a b c d e 3. a b c d e 4. a b c d e 5. a b c d e
RESPONSE 6. a b c d e 7. a b c d e 8. a b c d e 9. a b c d e 10. a b c d e

GRID 11. a b c d e 12. a b c d e 13. a b c d e 14. a b c d e 15. a b c d e


16. a b c d e 17. a b c d e 18. a b c d e

Downloaded From : www.EasyEngineering.net


Downloaded From : www.EasyEngineering.net

66 SPEED TEST 32
19. 30. A gardener plants 34969 mango trees in his garden and
97344 = ?
arranges them so that there are so many rows as there are
(a) 302 (b) 322 (c) 292
mango trees in each row. The number of rows is
(d) 342 (e) None of these
(a) 187 (b) 176 (c) 169
20. (2 ) (
392 - 21 + 8 -7 )2 = ( ? )2 (d) 158 (e) None of these
DIRECTIONS (Q. 31-40): What will come in place of question
(a) 4 (b) – 4 (c) 12 mark (?) in the following questions?
(d) 2 (e) 6
31. 93 × 812 ¸ 273 = (3)?
21. 7365 + (5.4)2 + ? = 7437.16
(a) 3 (b) 4 (c) 5
(a) 1894 (b) 1681 (c) 1764 (d) 6 (e) None of these
(d) 2025 (e) None of these
22. ( )
3 ? = 756 ´ 67 ¸ 804 32. ( 9 )3 ´ ( 81)5 ¸ (27)2 = (3)(?)
(a) 195112 (b) 250047 (c) 226981 (a) 5 (b) 4 (c) 7
(d) 274625 (e) None of these (d) 6 (e) None of these
33. 81.1 × 42.7 × 23.3 = 2?
23.
(a) 3
ww
17 + 51 + 152 + 289 = ?

(d) 11
(b) 5
(e) None of these
(c) 8
34.
(a) 7.1
(d) 9
(49)3 ¸ (7)2 = ?
(b) 14
(e) None of these
(c) 0.5

24. 217 + 52 + 12 = ?
(a) 18 w.E
(b) 16 (c) 12 35.
(a) 2401
(d) 7
643.1 × 84.3 = 8?
(b) 49
(e) None of these
(c) 343

25.
(d) 15
10201 - 3136 = ?
(a) 45
(e) 10

(b) 42
asy
(c) 46
36.
(a) 10.5
(d) 13.3
87 × 26 ¸ 82.4 = 8?
(b) 7.4
(e) None of these
(c) 1.2

(d) 49 (e) None of these


En (a) 10.6
(d) 6.6
(b) 9.6
(e) None of these
(c) 8.6

DIRECTIONS (Qs. 26-30): What approximate value will come in


place of the question mark (?) in the following questions? (You are
not expected to calculate the exact value.) gin
37. (31)31 × (31)– 27 = ?
(a) (961)2
(d) 29791
(b) 4
(e) None of these
(c) (31)2

26. If 21025 = 145 , then the value of 210.25 + 2.1025 = ?


(a) 0.1595 (b) 1.595 (c) 159.5 {(12) }
-2 2
eer
27.
(d) 15.95 (e) None of these
What is the least number to be added to 2000 to make it a
38.
{(12) }
2 -2
=?

ing
perfect square?
(a) 25
(d) 49
(b) 64
(e) None of these
(c) 36 (a) 12
(d) 144
(b) 4.8
.ne
(e) None of these
(c)
12
144

28. If (22)3 is subtracted from the square of a number, the answer


so obtained is 9516. What is the number?
(a) 144
(d) 136
(b) 142
(e) None of these
(c) 138
39. 64 × 362 ¸ 216 = 6(?)
(a) 3
(d) 6
(b) 4
(e) None of these
(c) 5 t
( 8)
6 3 ?
29. If the square of a number is subtracted from 4052 and the 40. ´ ( 64 ) ¸ 84 = ( 8 )
difference is multiplied by 15, the answer so obtained is
41340. What is the number? (a) 3 (b) 4 (c) 5
(a) 36 (b) 1024 (c) 32 (d) 7 (e) None of these
(d) 1296 (e) None of these

19. a b c d e 20. a b c d e 21. a b c d e 22. a b c d e 23. a b c d e


24. a b c d e 25. a b c d e 26. a b c d e 27. a b c d e 28. a b c d e
29. a b c d e 30. a b c d e 31. a b c d e 32. a b c d e 33. a b c d e
RESPONSE
34. a b c d e 35. a b c d e 36. a b c d e 37. a b c d e 38. a b c d e
GRID 39. a c d e 40. a c d e
b b

Downloaded From : www.EasyEngineering.net


Downloaded From : www.EasyEngineering.net

Simplification 33
Max. Marks : 30 No. of Qs. 30 Time : 20 min. Date : ........./......../. ..............
8. (78700 ÷ 1748) + (3.79 × 121.24) = ?
DIRECTIONS (Qs. 1 to 12) : What will come in place of qustions
mark (?) in the following questions? (a) 305 (b) 415
1. 19.99 × 9.9 + 99.9 = ? (c) 525 (d) 635
(a) 129. 79 (b) 297. 801 (e) 745
(c) 1009 (d) 296.91 9. 6080 ¸ 3 74000 + 4 6560 = ?

ww
(e) None of these
2. 456.675 + 35.7683 × 67.909 – 58.876 = ?
(a) 33382 (b) 3587
(a) 30
(c) 130
(b) 80
(d) 170
(c) 1540
(e) 2830
3. {(52)2 + (45)2} ¸ ? = 8
w.E (d) 2756

10.
(e) 210
1 2 3
of of of 1715 = ?
(a) 611.345
(c) 591.125
(b) 487.225
(d) 372.425 asy 8 3 5
(a) 80 (b) 85

4.
(e) None of these
(12.25)2 – 625 = ? En (c) 90
(e) 75
(d) 95

(a)
(c)
235.1625
375.2625
(b) 125.0625
(d) 465.3625
11.
gin
561 ÷ 35.05 × 19.99 = ?
(a) 320 (b) 330

(e) None of these


(c) 315
(e) 335
eer (d) 325

5. 572 + 38 × 0.50 – 16 = ?
(a) 289 (b) 305
12. (15.01)2 × 730 = ?
(a) 6125
ing (b) 6225
(c)
(e)
448
None of these
(d) 565 (c) 6200
(e) 6250
(d) 6075

.ne
6.
3 2 11
´ ´ ´ ? = 1056
7 5 13

(a) 7280 (b) 7296


DIRECTIONS (13-20): What will come in place of the question
mark (?) in the following questions?
13. (3325 ¸ 25) × (152 ¸ 16) = ?
(a) 1269.4 (b) 1264.9
t
(c) 1265.3 (d) 1263.5
(c) 7308 (d) 7316 (e) None of these
(e) 7324
1 2 2
14. 5 +2 +3 = ?
7. 6138 + 3 17576 = ? ×9 5 15 3
(a) 15 (b) 13
(a) 676 (b) 729
11
(c) 784 (d) 841 (c) (d) 12
15
(e) 961
(e) None of these

1. a b c d e 2. a b c d e 3. a b c d e 4. a b c d e 5. a b c d e
RESPONSE 6. a b c d e 7. a b c d e 8. a b c d e 9. a b c d e 10. a b c d e

GRID 11. a b c d e 12. a b c d e 13. a b c d e 14. a b c d e

Downloaded From : www.EasyEngineering.net


Downloaded From : www.EasyEngineering.net

68 SPEED TEST 33
15. – 15 – 27– 88 – 63 + 255 = ?
23. ? - 11 = 1521
(a) 55 (b) 74
(c) 62 (d) 59 (a) (b) (28)2
2500
(e) None of these
16. (2525 × 0.25 ¸ 5) × 7 = ? (c) 28 (d) 50
(a) 889.43 (b) 883.75 (e) None of these
(c) 886.45 (d) 881.75
(e) None of these 24. 8059 - 7263 = ? ´ 40
14 57 20 (a) 19.9 (b) 18.7
17. ´ ´ =? (c) 15.9 (d) 17.7
19 70 21
(e) None of these
2 4
(a) (b) 25. 4 ´ ? = 4062 ¸ 5
7 7
(a) 203.1 (b) 213.1
2 3 (c) 205.1 (d) 215.1
(c) (d)
9 7 (e) None of these

18.
(a) 231ww
(e) None of these
32% of 500 + 162% of 50 = ?
(b) 245
26. 3.5 ´ (80 ¸ 2.5) = ?
(a) 122 (b) 111

19.
(c) 237
(e) None of these
w.E (d)

45316 + 52131 – 65229 = ? + 15151


(a) 17063 (b)
247

17073 27.
(c) 222
(e) None of these
13% of 258 – ? = 10
(d) 212

(c) 17076
(e) None of these
(d)
asy
17067 (a) 23.45
(c) 23.54
(b) 24.53
(d) 24.35

20.
184 ´ 4
23 of 400
=?
En (e) None of these
4 3 5
(a) 7
(c) 8
(b) 9
(d) 5 gin
28.
5
´2 ¸ = ?
4 8
12 12
21.
(e) None of these
What will come in place of both the question marks (?) in
the following question ?
(a) 4
35
eer (b) 1
35

(?)
32
4/3
=
128
?5 / 3
(c)
11
35
2 (d)
ing 3
13
25

(a) 16
(c) 18
(b) 12
(d) 14
29.
(e) None of these
623.15 – 218.82 – 321.43 = ?
(a) 89.2 (b) .ne
82.2

22.
(e) None of these
16% of 450 ÷ ? % of 250 = 4.8
(a) 12 (b) 6 30.
(c) 89.9
(e) None of these
5437 – 3153 + 2284 = ? × 50
(d) 79.2
t
(c) 4 (d) 10 (a) 96.66 (b) 91.36
(e) None of these (c) 96.13 (d) 93. 16
(e) None of these

15. a b c d e 16. a b c d e 17. a b c d e 18. a b c d e 19. a b c d e


20. a b c d e 21. a b c d e 22. a b c d e 23. a b c d e 24. a b c d e
RESPONSE
25. a b c d e 26. a b c d e 27. a b c d e 28. a b c d e 29. a b c d e
GRID
30. a b c d e

Downloaded From : www.EasyEngineering.net


Downloaded From : www.EasyEngineering.net

Average 34
Max. Marks : 30 No. of Qs. 30 Time : 20 min. Date : ........./......../. ..............
1. The average of five numbers is 281. The average of the first 8. The average of 5 consecutive numbers A, B, C, D and E is 48.
two numbers is 280 and the average of the last two numbers What is the product of A and E?
is 178.5. What is the third number ? (a) 2162 (b) 2208
(a) 488 (b) 336 (c) 2024 (d) 2800
(c) 228 (d) 464 (e) None of these
(e) None of these 9. If the value of 16a + 16b = 672, what is the average of a and b?
2.

ww
The average age of 3 friends is 32 years. If the age of a
fourth friend is added, their average age comes to 31 years.
What is the age of the fourth friend ?
(a) 44
(c) 24
(e) None of these
(b) 21
(d) 42

3.
(a) 32 years
(c) 24 years
(e) None of these w.E (b) 28 years
(d) 26 years

Find the average of the following set of scores.


10. The sum of five numbers is 290. The average of the first two
numbers is 48.5 and the average of last two numbers is 53.5.
What is the third number?
965, 362, 189, 248, 461, 825, 524, 234
(a) 476 (b) 504 asy (a) 72
(c) 96
(e) None of these
(b) 84
(d) 108
(c) 461
(e) None of these
(d) 524
En 11. The average age of the family of five members is 24. If the
present age of youngest member is 8 yr, then what was the
4. If the value of 21a + 21b=1134, what is the average of a+b?
(a) 29
(c) 58
(b) 27
(d) 54 ginaverage age of the family at the time of the birth of the youngest
member ?
(a) 20 yr (b) 16 yr (c) 12 yr

5.
(e) None of these
Out of three given numbers, the first number is twice the 12.
(d) 18 yr
eer (e) 21 yr
The average of four consecutive odd numbers is 36. What is
second and thrice the third. If the average of the three num-
bers is 154, what is the difference between the first and the
third number?
(a) 31
(d) 47 ing
the smallest of these numbers ?
(b) 35
(e) None of these
(c) 43

(a) 126
(c) 166
(e) None of these
(b) 42
(d) 52
13.
.ne
Average score of Rahul, Manish and Suresh is 63. Rahul’s
score is 15 less than Ajay and 10 more than Manish. If Ajay
scored 30 marks more than the average scores of Rahul,
6.

is the average weight of P, T, R, F and G?


1
Average weight of three boys P, T and R is 54 kgs while
3
the average weight of three boys, T, F and G is 53 kgs. What
Manish and Suresh, what is the sum of Manish’s and Suresh’s
scores ?
(a) 120
(c) 117
(b) 111
(d) Cannot be determined
t
(a) 53.8kgs (b) 52.4kgs (e) None of these
(c) 53.2kgs (d) Cannot be determined 14. The total marks obtained by a student in physics, Chemistry
(e) None of these and Mathematics together is 120 more than the marks obtained
7. Find the average of the following set of scores by him in Chemistry. What are the average marks obtained by
221, 231, 441, 359, 665, 525 him in Physics and Mathematics together ?
(a) 399 (b) 428 (a) 60 (b) 120
(c) 40 (d) Cannot be determined
(c) 407 (d) 415 (e) None of these

(e) None of these


1. a b c d e 2. a b c d e 3. a b c d e 4. a b c d e 5. a b c d e
RESPONSE 6. a b c d e 7. a b c d e 8. a b c d e 9. a b c d e 10. a b c d e

GRID 11. a b c d e 12. a b c d e 13. a b c d e 14. a b c d e

Downloaded From : www.EasyEngineering.net


Downloaded From : www.EasyEngineering.net

70 SPEED TEST 34
15. A man drives to his office at 60 km/hr and returns home 23. The average of three numbers is 135. The largest number is
along the same route 30 km/hr. Find the average speed. 180 and the difference between the others is 25. The small-
(a) 50 km/hr (b) 45 km/hr (c) 40 km/hr est number is :
(d) 55 km/hr (e) None of these (a) 130 (b) 125 (c) 120
16. A cricketer has completed 10 innings and his average is 21.5 (d) 100 (e) None of these
runs. How many runs must he make in his next innings so as 24. There are 30 student in a class. The average age of the first
to raise his average to 24? 10 students is 12.5 years. The average age of the next 20
(a) 69 (b) 59 (c) 49 students is 13.1 years. The average age of the whole class
(d) 39 (e) None of these is:
17. One-third of a certain journey is covered at th rate of 25 km / (a) 12.5 years (b) 12.7 years (c) 12.8 years
hr, one-fourth at the rate of 30 km / hr and the rest at 50 km / (d) 12.9 years (e) None of these
hr. Find the average speed for the whole journey. 25. The weight in kilograms of 10 students are 52, 45, 31, 35, 40,
1 1 1 55, 60, 38, 44, 36. If 44 is replaced by 46 and 40 is replaced by
(a) 33 km / hr (b) 44 km / hr (c) 33 km/ hr 35, then the new median will be :
3 4 5 (a) 42 (b) 40.5 (c) 40
1 (d) 41.5 (e) None of these
(d) 44 km / hr (e) None of these 26. The average (arithmetic mean) of x and y is 40. If z = 10, then
5
18.
ww
The average salary of all the workers in a workshop is
` 8,000. The average salary of 7 technicians is `12,000 and
the average salary of the rest is Rs 6,000. The total number
what is the average of x, y and z ?
(a) 16
2
3
(b) 30 (c) 25

(a) 21
(d) 22
w.E
of workers in the workshop is :
(b) 20
(e) None of these
(c) 23
27.
(d) 17
1
2
(e) None of these
The average of 11 numbers is 10.9. If the average of the first
19.
asy
A batsman makes a scores of 98 runs in his 19th inning and
thus increases his average by 4. What is his average after
19th inning ?
six numbers is 10.5 and that of the last six numbers is 11.4,
then the middle number is :
(a) 22
(d) 26
(b) 24
(e) None of these
(c) 28
En 28.
(a) 11.5
(d) 11.0
(b) 11.4
(e) None of these
(c) 11.3

The average of 10 numbers is 40.2. Later it is found that two


20. The average weight of 45 students in a class is 52 kg. 5 of
them whose average weight is 48 kg leave the class and
other 5 students whose average weight is 54 kg join the gin numbers have been wrongly copied. The first is 18 greater
than the actual number and the second number added is 13
class. What is the new average weight (in kg) of the class ?

(a) 52.6 (b) 52


2
(c) 52
1
(a) 40.2
(d) 40.8 eer
instead of 31. Find the correct average.
(b) 40.4
(e) None of these
(c) 40.6

21.
(d) 62.5
3
(e) None of these
The batting average of 40 innings of a cricket player is 50
3 29.
ing
Last year, a Home Appliance Store sold an average (arithmetic
mean) of 42 microwave ovens per month. In the first 10
months of this year, the store has sold an average (arithmetic
runs. His highest score exceeds his lowest score by 172
runs. If these two innings are excluded, the average of the
.ne
mean) of only 20 microwave ovens per month. What was the
average number of microwave ovens sold per month during

22.
remaining 38 innings is 48. His highest score was :
(a) 172
(d) 176
(b) 173
(e) None of these
(c) 174

A car owner buys petrol at Rs 7.50, Rs 8.00 and Rs 8.50 per


litre for three successive years. What approximately is his
30.
the entire 22 months period ?
(a) 21
(d) 32
(b) 30
(e) None of these
(c) 31
t
The average temperature for the first four days of a week is
40.2°C and that of the last four days is 41.3°C. If the average
average cost per litre of petrol if he spends Rs 4000 each temperature for the whole week is 40.6°C, then the tempera-
year ? ture on the fourth day is :
(a) Rs 8 (b) Rs 9 (c) Rs 7.98 (a) 40.8°C (b) 38.5°C (c) 41.3°C
(d) Rs 8.50 (e) None of these (d) 41.8°C (e) None of these

15. a b c d e 16. a b c d e 17. a b c d e 18. a b c d e 19. a b c d e

20. a b c d e 21. a b c d e 22. a b c d e 23. a b c d e 24. a b c d e

RESPONSE 25. a b c d e 26. a b c d e 27. a b c d e 28. a b c d e 29. a b c d e

GRID 30. a b c d e

Downloaded From : www.EasyEngineering.net


Downloaded From : www.EasyEngineering.net

Percentage 35
Max. Marks : 30 No. of Qs. 30 Time : 20 min. Date : ........./......../. ..............
9. 36% of 245 – 40% of 210 = 10 – ?
DIRECTIONS (Qs. 1-18): What should come in place of the
question mark (?) in the following questions? (a) 4.2 (b) 6.8 (c) 4.9
(d) 5.6 (e) None of these
1. 76% of 1285 = 35% of 1256 + ?
(a) 543 (b) 537 (c) 547 1
10. of 3842 + 15% of ? = 24499
(d) 533 (e) None of these 2
2.
ww
(21.5% of 999)1/3 + (43% of 601)1/2 = ?
(a) 18 (b) 22 (c) 26
(a) 3520
(d) 3540
(b) 3250
(e) None of these
(c) 3350

3.
(d) 30
w.E (e) 33
64.5% of 800 + 36.4% of 1500 = (?)2 + 38
11. 57% of 394 – 2.5% of 996 = ?
(a) 215
(d) 180
(b) 175
(e) 205
(c) 200
(a) 32
(d) 48
(b) 38
(e) 34
asy (c) 42
12. 40% of 265 + 35% of 180 = 50% of ?
(a) 338 (b) 84.5 (c) 253.5
4. 41% of 601 – 250.17 = ? – 77% of 910
(a) 800 (b) 500 (c) 700
En (d) 169 (e) None of these

5.
(d) 650 (e) 550
40.005% of 439.998 + ?% of 655.011 = 228.5
13.
gin1 1
4 ´ 3 + ? = 20% of 120
5 3
(a) 8
(d) 20
(b) 17
(e) 5
(c) 12
(a) 10 eer
1
15
(b) 10 (c) 5
6. 25% of 84 × 24% of 85 = ?
(a) 424.2 (b) 488.4 (c) 482.8
14.
(d) 15
14% of 250 × ? % of 150 = 840ing
(e) None of these

7.
(d) 428.4 (e) None of these
20.06% of 599 + 10.01% of 901 = ?
(a) 15
(d) 12
(b) 18
.ne
(e) None of these
(c) 16

8.
(a) 150
(d) 280
(b) 210
(e) 300
14.2% of 5500 + 15.6% of ? = 1795
(c) 250
15. 18% of 609 + 27.5% of 450 = ?
(a) 220
(d) 248
(b) 233
(e) 274
(c) 267
t
(a) 6500 (b) 6200 (c) 5600
(d) 5800 (e) None of these

1. a b c d e 2. a b c d e 3. a b c d e 4. a b c d e 5. a b c d e
RESPONSE 6. a b c d e 7. a b c d e 8. a b c d e 9. a b c d e 10. a b c d e

GRID 11. a b c d e 12. a b c d e 13. a b c d e 14. a b c d e 15. a b c d e

Downloaded From : www.EasyEngineering.net


Downloaded From : www.EasyEngineering.net

72 SPEED TEST 35
16. 125% of 3060 – 85% of ? = 408 (a) 700 (b) 730
(a) 3890 (b) 3940 (c) 4020 (c) 745 (d) 765
(d) 4015 (e) None of these (e) None of these
17. If x% of 500 = y% of 300 and x% of y% of 200 = 60, then x = ?
24. The population of a town is 198000. It increases by 7% in the
(a) 10 2 (b) 20 2 (c) 15 2 1st year and decreases by 5% in the 2nd year. What is the
(d) 30 2 (e) None of these population of the town at the end of 2 years?
18. 185% of 400 + 35% of 240 = ? % of 1648 (a) 211860 (b) 201267
(a) 85 (b) 75 (c) 125 (c) 222453 (d) 198900
(d) 50 (e) None of these (e) None of these
2 25. Bovina spent ` 44,668 on her air tickets, ` 56,732 on buying
19. What is 25% of 50% of rd of 630 ? gifts for the family members and the remaining 22% of the
3
total amount she had as cash with her. What was the total
(a) 36.5 (b) 52.5 (c) 45.5
amount?
(d) 68.5 (e) None of these
(a) ` 28]600 (b) ` 1]30]000
20. There are 1225 employees in an organization, out of which
(c) ` 1]01]400 (d) ` 33]800

(a) 540 ww
40% got transferred to different places. How many such
employees got transferred ?
(b) 490 26.
(e) None of these
Rubina decided to donate 16% of her monthly salary to an
(c) 630
(e) None of these
w.E(d) 710

21. If the numerator of a fraction is increased by 500% and the


NGO. On the day of donation she changed her mind and
donated ` 6,567 which was 75% of what she had decided
earlier. How much is Rubina’s monthly salary?

is 2
4
. What was the original fraction? asy
denominator is increased by 300%, the resultant fraction (a) ` 8]756
(c) ` 6]56]700
(e) None of these
(b) ` 54]725
(d) ` 45]696

(a)
7
4
(b)
12
En 27. In a college election between two candidates, one candidate
got 55% of the total valid votes. 15% of the votes were

15
7
6
7
gin invalid. If the total votes were 15,200, what is the number of
valid votes the other candidate got ?
(c)
4
(e) None of these
(d)
5 (a) 7106
(c) 8360
eer (b) 6840
(d) 5814
22. If the numerator of a fraction is increased by 250% and the
7
denominator is increased by 300%, the resultant fraction is .
28.
(e) None of these

ing
If the radius of a circle is diminished by 10%, the area is
diminished by

What is the original fraction ?


9 (a) 36%
(e) None of these
(b) 20% (c) 19%
.ne (d) 10%

(a)
8
11
8
(b)

7
7
8
29.
per cent is B’s salary lower than A’s ?

(a)
1
16 % (b) 20% (c) 25% (d) 33 %
1
t
If A’s salary is 25% higher than B’s salary, then how much

(c) (d) 3 3
9 11
(e) None of these
(e) None of these
30. A reduction of 20% in the price of an apple enable a man to
23. In an examination it is required to get 40% of the aggregate buy 10 apple more for ` 54. The reduced price of apple per
marks to pass. A student gets 261 marks and is declared failed dozen is
by 4% marks. What are the maximum aggregate marks a
(a) ` 4.32 (b) ` 12.96
student can get?
(c) ` 10.80 (d) ` 14.40
(e) None of these

16. a b c d e 17. a b c d e 18. a b c d e 19. a b c d e 20. a b c d e

21. a b c d e 22. a b c d e 23. a b c d e 24. a b c d e 25. a b c d e


RESPONSE
26. a b c d e 27. a b c d e 28. a b c d e 29. a b c d e 30. a b c d e
GRID

Downloaded From : www.EasyEngineering.net


Downloaded From : www.EasyEngineering.net

Ratio & Proportion 36


Max. Marks : 30 No. of Qs. 30 Time : 20 min. Date : ........./......../. ..............
1. The total number of students in a school is 819. If the number (a) 10 years (b) 15 years
of girls in the school is 364, then what is the respective ratio (c) 24 years (d) Cannot be determined
of the total number of boys to the total number of girls in the
(e) None of these
school ?
8. In a college the students in Arts and Commerce faculties were
(a) 26 : 25 (b) 21 : 17
in the ratio of 4 : 5 respectively. When 65 more students joined
(c) 18 : 13 (d) 5 : 4 Commerce faculty the ratio became 8 : 11. How many students

2. ww
(e) None of these
If a dividend of ` 57,834 is to be divided among Meena,
Urmila and Vaishali in the proportion of 3:2:1, find Urmila's
are there in Arts faculty?
(a) 520 (b) 650

share.
(a) ` 19,281
(c) ` 23,133
w.E (b) ` 17,350
(d) ` 19,278
9-
(c) 715
(e) None of these
(d) Cannot be determined

The ratio between the boys and girls in a class is 6 : 5. If 8 more


boys join the class and two girls leave the class then the ratio

3.
(e) None of these
asy
The ratio of the ages of Richa and Shelly is 5 : 8. The ratio of
their ages 10 years hence would be 7:10. What is the present
becomes 11 : 7. What is the number of boys in the class now?
(a) 28 (b) 38
age of Shelly?
(a) 45years (b) 40years En (c) 44
(e) None of these
(d) 36

(c) 35years
(e) 25years
(d) 30years
10.

gin
At present Meena is eight times her daughter's age. Eight
years from now, the ratio of the ages of Meena and her
daughter will be 10 : 3. What is Meena's present age ?
4. A sum of money is to be divided equally amongst A, B, and
C in the respective ratio of 3:4:5 and another sum of money
is to be divided between E and F equally. If F got `1050 less
(a) 32 years
(c) 36 yearseer (b) 40 years
(d) Cannot be determined
than A, how much amount did B receive?
(a) ` 750 (b) ` 2000 11.
(e) None of these

ing
The ratio of the ages of a father and son is 17 : 7. 6 years ago
the ratio of their ages was 3 : 1 . What is the father’s present

5.
(c) ` 1500
(e) None of these
(d) Cannot be determined

The average age of a woman and her daughter is 42 years.


age?
(a) 64 (b) 51 .ne
The ratio of their ages is 2:1. What is the daughter's age?
(a) 28 years
(c) 52 years
(e) None of these
(b) 48 years
(d) 32 years 12.
(c) 48
(e) None of these
(d) Cannot be determined

t
The ratio of the money with Rita an Sita is 7 : 15 and that with
Sita and Kavita is 7 : 16. If Rita has ` 490, how much money
does Kavita have?
6. The difference between the present ages of Arun and Deepak
(a) 1050 (b) 2200 (c) 2400
is 14 years. Seven years ago the ratio of their ages was 5 : 7
(d) 2800 (e) None of these
respectively. What is Deepak’s present age?
13. In two alloys, the ratio of iron and copper is 4 : 3 and 6 : 1,
(a) 49 years (b) 42 years
respectively. If 14 kg of the first alloy and 42 kg of the second
(c) 63 years (d) 35 years alloy is mixed together to form a new alloy, then what will be
(e) None of these the ratio of iron to copper in the new alloy ?
7. Ratio of Rani’s and Komal’s age is 3 : 5. Ratio of Komal’s and (a) 11 : 3 (b) 11 : 8 (c) 8 : 1
Pooja’s age is 2 : 3. If Rani is two-fifth Pooja’s age, what is (d) 3 : 11 (e) None of these
Rani’s age?

1. a b c d e 2. a b c d e 3. a b c d e 4. a b c d e 5. a b c d e
RESPONSE 6. a b c d e 7. a b c d e 8. a b c d e 9. a b c d e 10. a b c d e

GRID 11. a b c d e 12. a b c d e 13. a b c d e

Downloaded From : www.EasyEngineering.net


Downloaded From : www.EasyEngineering.net

74 SPEED TEST 36
23. An amount of money is to be distributed among P, Q and R
1 1 1
14. The sides of a triangle are in the ratio : : and its in the ratio 3 : 1 : 5. The difference between Q’s and R’s share
2 3 4 is Rs 3600. What is the total of P’s and Q’s share ?
perimeter is 104 cm. The length of the longest side is: (a) Rs 5400 (b) Rs 3600 (c) Rs 2700
(a) 52 cm (b) 48 cm (c) 32 cm (d) Rs 1800 (e) None of these
(d) 26 cm (e) None of these 24. Mrs. X spends Rs 535 in purchasing some shirts and ties for
15. The ratio of two numbers is 4 : 7. If each of these numbers her husband. If shirts cost Rs 43 each and the ties cost Rs 21
increases by 30, their ratio will become 5 : 8 . What is the each, then what is the ratio of the shirts to the ties, that are
average of these two numbers? purchased ?
(a) 135 (b) 145 (c) 155 (a) 1 : 2 (b) 2 : 1 (c) 2 : 3
(d) 165 (e) 175 (d) 3 : 4 (e) None of these
1 2 3 25. If a : b = 2 : 5, then the value of (2a + 3b) : (7a + 5b) is :
16. If ` 782 be divided into three part, proportional to : : ,
2 3 4
then the first part is : 19 99 31
(a) (b) (c)
(a) ` 182 (b) ` 190 (c) ` 196 39 13 19
(d) ` 204 (e) None of these
19
17.
ww 3
The scale of map is of cm = 1 km. If the distance on the
4
map between two towns is 60 cm, then the actual distance is 26.
(d)
31
(e) None of these

If two numbers are in the ratio 6 : 13 and their least common


(a) 60 km
(d) 50 km
w.E
(b) 80 km
(e) None of these
A B C
(c) 75 km
multiple is 312, then the sum of the numbers is :
(a) 75
(d) 67
(b) 57
(e) None of these
(c) 76

18. If A : B : C = 2 : 3 : 4, then

(a) 4 : 9 : 16
B C A
(b) 8 : 9 : 12 asy
: : is equal to :

(c) 8 : 9 : 16
27. If
5a + 3b 23
2a - 3b
=
5
, then the value of a : b is :

19.
(d) 8 : 9 : 24 (e) None of these
The average age of three boys is 25 years and their ages are
En 28.
(a) 2 : 1
(d) 4 : 1
(b) 1 : 4
(e) None of these
(c) 1 : 2

A bag contains Rs 216 in the form of one rupee, 50 paise and


in the proportion 3: 5 : 7. The age of the youngest boy is:
(a) 21 years
(d) 9 years
(b) 18 years
(e) None of these
(c) 15 years
gin 25 paise coins in the ratio of 2 : 3 : 4. The number of 50 paise
coins is :
20. The income of A and B are in the ratio 3 : 2 and expenses are
in the ratio 5 : 3. If both save ` 200, what is the income of A?
29.
(a) 96
(d) 141
eer (b) 144
(e) None of these
(c) 114

The ages of two persons are in the ratio of 5 : 7. Sixteen years

21.
(a) ` 1000
(d) ` 1800
(b) ` 1200
(e) None of these
` 750 is distributed among A, B and C such that
(c) ` 1500
are: ing
ago, their ages were in the ratio of 3: 5. Their present ages

A’s share : B’s share = 2 : 3 and B’s share: C’s share = 6 : 5.


The share of A is,
(a) 30 years and 44 years
(b) 35 years and 52 years
(c) 40 years and 56 years .ne
22.
(a) ` 150
(d) ` 250
(b) ` 175
(e) None of these
(c) ` 200

Two number are in the ratio 5 : 4 and their difference is 10.


What is the larger number ? 30.
(d) 45 years and 60 years
(e) None of these t
In a mixture of 28 litres, the ratio of milk and water is 5 : 2.
(a) 30 (b) 40 (c) 50 Another 2 litres of water is added to the mixture. The ratio of
(d) 60 (e) None of these milk and water in the new mixture is:
(a) 1: 1 (b) 2: 1 (c) 3: 1
(d) 4: 1 (e) None of these

14. a b c d e 15. a b c d e 16. a b c d e 17. a b c d e 18. a b c d e


19. a b c d e 20. a b c d e 21. a b c d e 22. a b c d e 23. a b c d e
RESPONSE 24. a b c d e 25. a b c d e 26. a b c d e 27. a b c d e 28. a b c d e
GRID 29. a b c d e 30. a b c d e

Downloaded From : www.EasyEngineering.net


Downloaded From : www.EasyEngineering.net

Alligation and
Mixture
37
Max. Marks : 25 No. of Qs. 25 Time : 20 min. Date : ........./......../. ..............
1. One litre of water was mixed to 3 litres of sugar. Solution 8. An alloy contains copper and zinc in the ratio 5 : 3 and another
containing 4% of sugar. What is the percentage of sugar in alloy contains copper and tin in the ratio 8 : 5. If equal weights
the solution? of both the alloys are melted together, then the weight of tin
(a) 3 (b) 4 in the resulting alloy per kg will be:
(c) 6 (d) Insuffficient data
26 5 7
(e) None of these (a) (b) (c)
2.
ww
A trader mixes 80 kg of tea at ` 15 per kg with 20 kg of tea at
cost price of ` 20 per kg. In order to earn a profit of 25%,
(d)
5
40
26

(e) None of these


40

(a) ` 23.75
(d) ` 19.20 w.E
what should be the sale price of the mixed tea?
(b) ` 22
(e) None of these
(c) ` 20
9.
7
The ratio of milk and water in 55 litres of adulterated milk is
7 : 4. How much water must be added to make the mixture’s
3.

resulting mixture may be ` 2.75 per litre? asy


Alcohol cost ` 3.50 per litre and kerosene oil cost ` 2.50 per
litre. In what proportion these should be mixed so that the
ratio 7 : 6?
(a) 5 lt
(d) 25 lt
(b) 10 lt
(e) None of these
(c) 15 lt

(a) 2 : 5
(d) 2 : 3
(b) 1 : 3
(e) None of these
(c) 4 : 7
En 10. A and B are two alloys of gold and copper prepared by mixing
metals in the ratio 7 : 2 and 7 : 11 respectively. If equal
4. Pure milk costs ` 3.60 per litre. A milkman adds water to 25
litres of pure milk and sells the mixture at ` 3 per litre. How
many litres of water does he add? gin quantities of the alloys are melted to form a third alloy C, then
the ratio of gold and copper in alloy C will be

(a) 2 litres
(d) 11 litres
(b) 5 litres
(e) None of these
(c) 7 litres
11.
(a) 5 : 7
(d) 9 : 5
eer (b) 5 : 9
(e) None of these
(c) 7 : 5

Sameer bought 10 kg of tea at ` 45 per kg and 8 kg at ` 50 per


5. In what ratio must water be mixed with milk to gain 20% by
selling the mixture at cost price?
(a) 1 : 3 (b) 1 : 5 (c) 1 : 7
ing
kg. He mixed both the brands and sold it at a total profit of `
32. What was the selling price per kg of the mixture?

6.
(d) 1 : 10 (e) None of these
A chemist has 10 litres of a solution that is 10 per cent nitric 12.
(a) ` 48
(d) ` 47
(b) ` 50
(e) None of these
.ne
(c) ` 49

How many litres of pure alcohol must be added to 10 litres of


acid by volume. He wants to dilute the solution to 4 per
cent strength by adding water. How many litres of water
must he add ?
(a) 15 (b) 20 (c) 18
mixture which is 15% alcohol to make a mixture which will be
25% alcohol?
5 5
t
3
(a) (b) (c)
(d) 25 (e) None of these 4 2 4

2 4
7. In what ratio must water be mixed with milk to gain 16 % (d) (e) None of these
3 3
on selling the mixture at cost price?
(a) 1 : 6 (b) 6 : 1 (c) 2 : 3
(d) 4 : 3 (e) None of these

1. a b c d e 2. a b c d e 3. a b c d e 4. a b c d e 5. a b c d e
RESPONSE 6. a b c d e 7. a b c d e 8. a b c d e 9. a b c d e 10. a b c d e

GRID 11. a b c d e 12. a b c d e

Downloaded From : www.EasyEngineering.net


Downloaded From : www.EasyEngineering.net

76 SPEED TEST 37
13. How many kg of custard powder costing ` 40 kg must be 20. If 50% of a 2 : 3 solution of milk and water is replaced with
mixed with 16 kg of custard powder costing ` 55 kg so that water, then the concentration of the solution is reduced by:
25% may be gained by selling the mixture at ` 60 kg? (a) 25% (b) 33.33% (c) 50%
(a) 11 kg (b) 14 kg (c) 12 kg (d) 75% (e) None of these
(d) 20 kg (e) None of these 21. In a mixture of 45 litres, the ratio of milk and water is 3 : 2.
14. 300 gm of sugar solution has 40% sugar in it. How much How much water must be added to make the ratio 9 : 11?
sugar should be added to make it 50% in the solution? (a) 10 litres (b) 15 litres (c) 17 litres
(a) 40 gm (b) 50 gm (c) 60 gm (d) 20 litres (e) None of these
(d) 70 gm (e) None of these 22. Three containers A, B and C are having mixtures of milk and
15. Gold is 19 times as heavy as water and copper 9 times. In water in the ratio 1 : 5, 3 : 5 and 5 : 7, respectively. If the
what ratio should these metals be mixed so that the mixture capacities of the containers are in the ratio 5 : 4 : 5, then find
may be 15 times as beavy as water? the ratio of the milk to the water if the mixtures of all the three
(a) 1 : 2 (b) 3 : 2 (c) 2 : 3 containers are mixed together.
(d) 4 : 5 (e) None of these (a) 51 : 115 (b) 52 : 115 (c) 53 : 115
16. In a mixture of 60 litres, the ratio of milk to water is 2 : 1. If the (d) 54 : 115 (e) None of these
ratio of milk to water is to be 1 : 2, then amount of water to be 23. Five litres of water is added to a certain quantity of pure milk

ww
further added is ___________.
(a) 20
(d) 80
(b) 40
(e) None of these
(c) 60
costing Rs. 3 per litre. If by selling the mixture at the same
price as before, a profit of 20% is made, then what is the
amount of pure milk in the mixture ?
17.
w.E
In a mixture of milk and water the proportion of water by
weight was 75%. If in 60 gm of mixture 15 gm water was
added, what would be the percentage of water? (Weight in 24.
(a) 20 litres
(d) 35 litres
(b) 30 litres
(e) None of these
(c) 25 litres

How many kg of tea worth Rs 25 per kg must be blended


gm)
(a) 75% (b) 88% asy (c) 90%
with 30 kg of tea worth Rs 30 per kg so that by selling the
blended variety at Rs 30 per kg there should be a gain of

18.
(d) 100% (e) None of these
In what ratio must tea at Rs. 62 per kg be mixed with tea at
En 10%?
(a) 32 kg (b) 40 kg (c) 36 kg
Rs. 72 per kg so that the mixture must be worth Rs. 64.50 per
kg?
(a) 3 : 1 (b) 3 : 2 (c) 4 : 3 gin
25.
(d) 42 kg (e) None of these
How many kg of sugar costing Rs 5.75 per kg should be
mixed with 75 kg of cheaper sugar costing Rs 4.50 per kg so

19.
(d) 5 : 3 (e) None of these
Two vessels A and B contain spirit and water mixed in the (a) 350 kg eer
that the mixture is worth Rs 5.50 per kg ?
(b) 300 kg (c) 250 kg
ratio 5 : 2 and 7 : 6 respectively. Find the ratio in which these
mixture be mixed to obtain a new mixture in vessel C contain-
ing spirit and water in the ratio 8 : 5 ?
(d) 325 kg

ing
(e) None of these

(a) 4 : 3
(d) 7 : 9
(b) 3 : 4
(e) None of these
(c) 5 : 6
.ne
t

13. a b c d e 14. a b c d e 15. a b c d e 16. a b c d e 17. a b c d e

18. a b c d e 19. a b c d e 20. a b c d e 21. a b c d e 22. a b c d e


RESPONSE 23. a c d e 24. a c d e 25. a c d e
b b b
GRID

Downloaded From : www.EasyEngineering.net


Downloaded From : www.EasyEngineering.net

Profit, Loss &


Discount
38
Max. Marks : 30 No. of Qs. 30 Time : 20 min. Date : ........./......../. ..............
1. If the cost price is 96% of the selling price, then what is the 9. A man buys milk at Rs 6 per litre and adds one third of water
profit percent? to it and sells mixture at Rs 7.20 per litre. The gain is
(a) 4.5% (b) 4.2% (c) 4% (a) 40% (b) 80% (c) 60%
(d) 3.8% (e) None of these (d) 25% (e) None of these
2. If the manufacturer gains 10%, the wholesale dealer 15% 10. A dishonest fruit seller professes to sell his goods at the cost
and the retailer 25%, then find the cost of production of a

ww
table, the retail price of which is ` 1265?
(a) ` 800 (b) ` 1000 (c) ` 900
price but weighs 800 grams for a kg weight. Find his gain
percent.
(a) 100% (b) 150% (c) 50%

3.
(d) ` 600

w.E(e) None of these

A man sold his book for Rs 891, thereby gaining


1
10
of its
11.
(d) 200% (e) None of these
Two electronic musical instruments were purchased for
` 8000. The first was sold at a profit of 40% and the second at
cost price. Find his cost price.
(a) ` 850 (b) ` 810 asy
(c) ` 851
loss of 40%. If the sale price was the same in both the cases,
what was the cost price of two electronic musical instruments?
(a) ` 2000, ` 5000 (b) ` 2200, ` 5500
4.
(d) ` 840 (e) None of these
A man buys 50 pencils for Rs 100 and sells 45 pencils for
` 90. Find his gain or loss %. En (c) ` 2400, ` 5000 (d) ` 2400, ` 5600
(e) None of these
(a) 20% (b) 35%
(d) No gain or loss (e) None of these
(c) 25%
12.
gin
A tradesman is marketing his goods 20% above the cost price
of the goods. He gives 10% discount on cash payment, find
5. A dealer sold a mixer for ` 420 at a loss of 12.5%. At what
price should he have sold it to gain 12.5%.
(a) 12%
(d) 18% eer
his gain percent.
(b) 8%
(e) None of these
(c) 15%

6.
(a) ` 620
(d) ` 750
(b) ` 540
(e) None of these
(c) ` 650

A man sold 10 eggs for 5 rupees and gained 20%.How


13.
ing
A man sells his car for ` 5000 and loses something. Had he
sold it for ` 5600, his gain would have been double the former
many eggs did he buy for 5 rupees?
(a) 10 eggs (b) 12 eggs (c) 14 eggs
loss. Find the cost price.
(a) ` 5500 (b) ` 5100
.ne (c) ` 5400

7.
(d) 16 eggs (e) None of these
Coconuts were purchased at ` 150 per hundred and sold at
` 2 per coconut. If 2000 coconuts were sold, what was the
total profit made?
14.
(d) ` 5200 (e) None of these

he would have gained 5%. Find the cost price of the fan.
(a) ` 350 (b) ` 480 (c) ` 240
t
John sold a fan at a loss of 7%. If he had sold it for ` 48 more,

(d) ` 400 (e) None of these


(a) ` 500 (b) ` 1000 (c) ` 1500
15. The owner of an electronics shop charges his customer 22%
(d) ` 2000 (e) None of these
more than the cost price. If a customer paid ` 10,980 for a
8. A shopkeeper’s price is 50% above the cost price. If he
DVD player, then what was the cost price of the DVD player?
allows his customer a discount of 30% what profit does he
make? (a) ` 8000 (b) ` 8800 (c) ` 9500
(a) 5% (b) 10% (c) 15% (d) ` 9200 (e) None of these
(d) 20% (e) None of these

1. a b c d e 2. a b c d e 3. a b c d e 4. a b c d e 5. a b c d e
RESPONSE 6. a b c d e 7. a b c d e 8. a b c d e 9. a b c d e 10. a b c d e

GRID 11. a b c d e 12. a b c d e 13. a b c d e 14. a b c d e 15. a b c d e

Downloaded From : www.EasyEngineering.net


Downloaded From : www.EasyEngineering.net

78 SPEED TEST 38
16- Cost of 24 bats and 32 sticks is `5,600. What is the price of 24. If the cost of 12 pencils is equal to the selling price of 10
3 bats and 4 sticks? pencils, the profit percent in the transaction is :
(a) ` 1,400 (b) ` 2,800 1 1
(a) 16 % (b) 22 % (c) 20%
(c) ` 700 (d) Cannot be determined 3 2
(e) None of these (d) 25% (e) None of these
17. The profit earned after selling an article for ` 1,754 is the 25. Two motor cars were sold for Rs 9,900 each, gaining 10% on
same as loss incurr ed after selling the article for one and losing 10% on the other. The gain or loss percent in
` 1,492. What is the cost price of the article? the whole transaction is :
(a) ` 1,623 (b) ` 1,523 (c) ` 1,689 (a) Neither loss no gain
(d) ` 1,589 (e) None of these 1 100
18. Prathik sold a music system to Karthik at 20% gain and Karthik (b) % gain (c) % profit
99 99
sold it to Swasthik at 40% gain. If Swasthik paid `10,500 for
(d) 1% loss (e) None of these
the music system, what amount did Prathik pay for the same?
26. The retail price of a water geyser is Rs 1265. If the
(a) ` 8,240 (b) ` 7,500 (c) ` 6,250 manufacturer gains 10%, the wholesale dealer gains 15%
(d) Cannot be determined and the retailer gains 25%, then the cost of the geyser is :

19.
ww
(e) None of these
The cost of 5 pendants and 8 chains is ` 1,45,785. What
would be the cost of 15 pendants and 24 chains? 27.
(a) Rs 800
(d) Rs 650
(b) Rs 900
(e) None of these
(c) Rs 550

A man buys a watch for Rs. 1950 in cash and sells it for Rs.
(a) ` 3,25,285
(c) ` 5,50,000
(e) None of these
w.E(b) ` 4,39,355
(d) Cannot be determined
2200 on credit of 1 year. If the rate of interest is 10% per
annum, then the man
(a) Gains Rs. 55 (b) Gains Rs. 50
20.
asy
What price should a shopkeeper mark on an article costing
him Rs 153 to gain 20% after allowing a discount of 15% ?
(a) Rs 162 (b) Rs 621 (c) Rs 216 28.
(c) Loses Rs. 30
(e) None of these
(d) Gains Rs. 30

A shopkeeper marks up his goods to gain 35%. But he allows

21.
(d) Rs 226 (e) None of these
Three successive discounts of 10%, 12% and 15% amount En 10% discount for cash payment. His profit on the cash
transaction in percentage, is
to a single discount of :
(a) 36.28 % (b) 34.68% (c) 37 % gin (a) 13
1
2
(b) 25 (c) 21
1
2
22.
(d) 38% (e) None of these
A machine is sold at a profit of 10%. Had it been sold for Rs
80 less, there would have been a loss of 10%. The cost price (d) 31
1
2 eer (e) None of these
of the machine is :
(a) Rs 350
(d) Rs 520
(b) Rs 400
(e) None of these
(c) Rs 450
29.
ing
A man sold two steel chairs for Rs. 500 each. On one he
gains 20% and on other, he loses 12%. How much does he
23. If selling price is doubled then, the profit triples. What is
profit per cent :
gain or lose in the whole transaction?
(a) 1.5% gain
(d) 2% loss
(b) 2% gain
(e) None of these .ne
(c) 1.5% loss

(a) 66

(d) 125
2
3
(b) 100

(e) None of these


(c) 33
1
3
30.
selling price of two notebooks. What is his percentage
profit?
t
By selling 12 notebooks, the seller earns a profit equal to the

2
(a) 25% (b) 20% (c) 16 %
3
(d) Data inadequate
(e) None of these

16. a b c d e 17. a b c d e 18. a b c d e 19. a b c d e 20. a b c d e

21. a b c d e 22. a b c d e 23. a b c d e 24. a b c d e 25. a b c d e


RESPONSE 26. a c d e 27. a c d e 28. a c d e 29. a c d e 30. a c d e
b b b b b
GRID

Downloaded From : www.EasyEngineering.net


Downloaded From : www.EasyEngineering.net

Simple Interest 39
Max. Marks : 30 No. of Qs. 30 Time : 20 min. Date : ........./......../. ..............
1. Ms. Sandhya deposits an amount of ` 31,400 to obtain a 8. What total amount would Mithilesh get at the end of three
simple interest at the rate of 12 per cent per annum for 8 years if he invests an amount of ` 11200 in a scheme, which
years. What total amount will Ms. Sandhya get at the end offers simple interest 8.5% per annum for three years ?
of 8 years? (a) ` 14056 (b) ` 14348 (c) ` 13852
(a) ` 31,444 (b) ` 61,544 (c) ` 41,544 (d) ` 15064 (e) None of these
(d) ` 31,144 (e) None of these 9. A sum of Rs. 2600 is lent out in two parts in such a way that
2.

ww
Mr. Deepak invested an amount of ` 21,250 for 6 years. At
what rate of simple interest will he obtain the total amount
of ` 26,350 at the end of 6 years?
the interest on one part at 10% for 5 years is equal to that on
the other part at 9% for 6 years. The sum lent out at 10% is
________ .

3.
(a) 6 % p.a
(d) 12 % p.a
w.E (b) 5 % p.a
(e) None of these
(c) 8 % p.a

Asmita invests an amount of ` 9535 at the rate of 4 per cent


per annum to obtain a total amount of ` 11442 on simple
10.
(a) 1250
(d) 1550
(b) 1350
(e) 1650
(c) 1450

At a certain rate of simple interest, a certain sum doubles


itself in 10 years. It will treble itself in years ________ .

invest the amount to obtain the total sum?


(a) 10 years (b) 2 years
asy
interest after a certain period. For how many year did she

(c) 5 years
(a) 10
(d) 30
(b) 20
(e) 45
(c) 25

A sum of money at simple interest amounts to ` 600 in 4 years

4.
(d) 4 years (e) None of these
En
Girish invested a certain amount at the rate of 8% p.a. for 6
11.
and ` 650 in 6 years. Find the rate of interest per annum.
(a) 3% (b) 5% (c) 9%
year to obtain an amount of ` 28,046. How much amount
did Girish obtain as simple interest?
(a) `12,550 (b) `9,096 (c) `18,950
12.
gin
(d) 10% (e) 15%
A person lent at certain sum of money at 4% simple interest;

(d) Cannot be determined


(e) None of these (a) 500 eer
and in 8 years the interest amounted to ` 340 less than the
sum lent. Find the sum lent.
(b) 600 (c) 1000
5. Mr. Anuraag Awasthi deposits an amount of ` 56500 to
obtain a simple interest at the rate of 12% p.a. for 3 years.
13.
(d) 1500
ing
(e) 1700
1
In what time will ` 72 become ` 81 at 6 % p.a. simple interest?

.ne
What total amount will Mr. Anuraag Awasthi get at the end
4
of 3 year ?
(a) 1 year 6 months (b) 2 years (c) 1 years
(a) `75680 (b) `77540 (c) `76840

6.
(d) `73420 (e) None of these
Veena obtained an amount of ` 8, 376/- as simple interest
on a certain amount at 8% p.a. after 6 years. What is the
amount invested by Veena?
14.
1
(d) 2 years
2
(e) None of these
A sum of money lent out at simple interest amounts to
t
` 1008 in 2 years and ` 1164 in 3½ years. Find the rate % p.a.
(a) ` 17,180 (b) ` 18,110 (c) ` 16,660 1
(d) ` 17,450 (e) None of these (a) 13% (b) 14% (c) 12 %
2
7. The simple interest accrued on a sum of certain principal is
(d) 15% (e) None of these
` 2000 in five years at the rate of 4% per annum. What
would be the compound interest accrued on same principal 15. A sum of money lent out at simple interest amounts to ` 720
at same rate in two years ? after 2 years and to ` 1,020 after a further period of 5 years.
(a) ` 716 (b) ` 724 (c) ` 824 Find the sum and the rate %.
(d) ` 816 (e) None of these (a) ` 500, 10% (b) ` 600, 10% (c) ` 500, 12%
(d) ` 600, 12% (e) None of these

1. a b c d e 2. a b c d e 3. a b c d e 4. a b c d e 5. a b c d e
RESPONSE 6. a b c d e 7. a b c d e 8. a b c d e 9. a b c d e 10. a b c d e

GRID 11. a b c d e 12. a b c d e 13. a b c d e 14. a b c d e 15. a b c d e

Downloaded From : www.EasyEngineering.net


Downloaded From : www.EasyEngineering.net

80 SPEED TEST 39
16. On retirement, a person gets 1.53 lakhs of his provident fund 24. Nitin borrowed some money at the rate of 6% p.a. for the
which he invests in a scheme at 20% p.a. His monthly income first three years, 9% p.a. for the next five years and 13% p.a.
from this scheme will be for the period beyond eight years If the total interest paid by
(a) ` 2, 450 (b) ` 2,500 (c) ` 2, 550 him at the end of eleven years is ` 8160, how much money
(d) ` 2, 600 (e) None of these did he borrow?
17. A sum was put at simple interest at a certain rate for 4 years (a) ` 8000 (b) ` 10,000 (c) ` 12,000
Had it been put at 2% higher rate, it would have fetched ` 56 (d) Data inadequate (e) None of these
more. Find the sum. 25. An automobile financier claims to be lending money at simple
(a) ` 500 (b) ` 600 (c) ` 700 interest, but he includes the interest every six months for
(d) ` 800 (e) None of these calculating the principal. If he is charging an interest of 10%,
18. The simple interest on ` 200 for 7 months at 5 paise per the effective rate of interest becomes :
rupee per month is (a) 10% (b) 10.25% (c) 10.5%
(a) ` 70 (b) ` 7 (c) ` 35 (d) Data inadequate (e) None of these
(d) ` 30.50 (e) None of these 26. A lent ` 5000 to B for 2 years and ` 3000 to C for 4 years on
19. A father left a will of ` 68,000 to be divided between his two simple interest at the same rate of interest and received
sons aged 10 years and 12 years such that they may get ` 2200 in all from both of them as interest. The rate of interest
per annum is:

ww
equal amount when each attains the age of 18 years If the
money is reckoned at 10% p.a., find how much each gets at
the time of the will.
(a) ` 30,000, ` 38,000 (b) ` 28,000, ` 40,000
(a) 5% (b) 7% (c)
1
7 %
8

20.
(e) None of these w.E
(c) ` 32,000, ` 36,000 (d) cannot be determined.

In how many minimum number of complete years, the interest


27.
(d) 10% (e) None of these
The rates of simple interest in two banks A and B are in the
ratio 5 : 4. A person wants to deposit his total savings in two

rupees?
(a) 6 (b) 8
asy
on ` 212.50 P at 3% per annum will be in exact number of

(c) 9
banks in such a way that he received equal half-yearly
interest from both. He should deposit the savings in banks
A and B in the ratio.

21.
(d) 7 (e) None of these
En
What annual instalment will discharge a debt of ` 4,200 due
(a) 2 : 5
(d) 5 : 4
(b) 4 : 5
(e) None of these
(c) 5 : 2

in 5 years at 10% simple interest?


(a) ` 500 per year
(c) ` 700 per year
(b) ` 600 per year
(d) ` 800 per year
gin
28. If a certain sum of money becomes double at simple interest
in 12 years, what would be the rate of interest per annum ?

22.
(e) None of these
A certain amount earns simple interest of ` 1750 after 7 years
(a)

(d) 14
8
1
3
eer (b) 10

(e) None of these


(c) 12

Had the interest been 2% more, how much more interest


would it have earned?
(a) ` 35 (b) ` 245 (c) ` 350
29.
ing
Two equal sums were borrowed at 8% simple interest per
annum for 2 years and 3 years, respectively. The difference

(d) Cannot be determined


(e) None of these
was Rs 56. The sum borrowed were :
(a) Rs 690 (b) Rs 700 .ne
in the interests was Rs 56. The difference in the interests

(c) Rs 740
23. What will be the ratio of simple interest earned by certain
amount at the same rate of interest for 6 years and that for
9 years?
(a) 1 : 3 (b) 1 : 4 (c) 2 : 3
30.
(d) Rs 780 (e) None of these
How much interest will Rs 10,000 earn in 9 months at an
annual rate of 6 percent ?
(a) Rs 450 (b) Rs 460 (c) Rs 475
t
(d) Data inadequate (e) None of these (d) Rs 600 (e) None of these

16. a b c d e 17. a b c d e 18. a b c d e 19. a b c d e 20. a b c d e


RESPONSE 21. a b c d e 22. a b c d e 23. a b c d e 24. a b c d e 25. a b c d e
GRID 26. a b c d e 27. a b c d e 28. a b c d e 29. a b c d e 30. a b c d e

Downloaded From : www.EasyEngineering.net


Downloaded From : www.EasyEngineering.net

Compound Interest 40
Max. Marks : 25 No. of Qs. 25 Time : 20 min. Date : ........./......../. ..............
1. Sudhanshu invested ` 15,000 at interest @ 10% p.a for one 7. Rohit invested some amount at the rate of 6 pcpa and at the
year. If the interest is compounded every six months what end of 2 yr he got ` 8730 simple interest. How much compound
amount will Sudhanshu get at the end of the year? interest he will get on same amount and same rate of interest
(a) ` 16,537.50 (b) ` 16,5000 after 2 yr.
(c) ` 16,525.50 (d) ` 18,150 (a) ` 5820 (b) ` 5949.60 (c) ` 5900
(e) None of these (d) ` 5994.60 (e) None of these
2.
ww
What would be the compound interest obtained on an
amount of ` 1,250 at the rate of 8% p.a. after 2 year ?
8. Sonika invested an amount of ` 5800 for 2 years. At what rate
of compound interest will she get an amount of ` 594.5 at the
end of two years ?

3.
(a) ` 200
(d) `220
w.E (b) ` 208
(e) None of these
(c) `212

Shyam invests an amount of ` 5,690 at the rate of 5 per cent 9.


(a) 5 p.c.p.a.
(d) 8 p.c.p.a.
(b) 4 p.c.p.a.
(e) None of these
(c) 6 p.c.p.a.

What would be the compound interest accrued on an amount

(a) ` 854 (b) ` 799


asy
per annum for 3 years. What approximate amount of
compound interest will he obtain at the end of 3 years?
(c) ` 843
of ` 7400 @ 13.5 p.c.p.a. at the end of two years ?
(Rounded off to two digits after decimal)
(a) ` 2136.87 (b) ` 2306.81 (c) ` 2032.18

4.
(d) ` 787 (e) ` 897
En
The simple interest accrued on an amount of ` 84,000 at the 10.
(d) ` 2132.87 (e) None of these
If the compound interest accrued on an amount of ` 14500 in
end of three year is ` 30,240. What would be the compound
interest accrued on the same amount at the same rate in the gin
two years is ` 4676.25, what is the rate of interest p.c.p.a ?
(a) 11
(d) 18
(b) 9
(e) None of these
(c) 15
same period?
(a) ` 30,013.95
(c) ` 32,013.95
(b) ` 31,013.95
(d) ` 33,013.95
11.
eer
What will be the compound interest accrued on an amount of
` 10000 @ per annum in two years if the interest is

5.
(e) ` 34,013-95
What will be the difference between the compound interest
compounded half-yearly ?
(a) ` 4400
(d) ` 4680 ing
(b) ` 4600
(e) None of these
(c) ` 4641
and simple interest at the rate of 5% p.a. on an amount of
` 4,000 at the end of two years?
(a) ` 10 (b) ` 20
12.
.ne
What will be the difference between the simple interest and
compound interest earned on a sum of ` 985.00 at the rate of

6.
(c) ` 30
(e) None of these
(d) Data inadequate

Pamela invested an amount of ` 35,000 for two year at the


rate of 5% p.a. What amount of compound interest would 13.
14% per annum at the end of two years ?
(a) ` 16.408
(d) ` 17.405
(b) ` 14.214
(e) None of these
(c) ` 19.218

A sum of money at compound interest amounts in two years


t
she receive at the end of two year ? to ` 2809, and in three years to ` 2977.54. Find the rate of
(a) ` 3587.50 (b) ` 3500 (c) ` 3580.50 interest and the original sum.
(d) ` 3565.50 (e) None of these (a) 2000 (b) 2100 (c) 2200
(d) 2500 (e) 3000

1. a b c d e 2. a b c d e 3. a b c d e 4. a b c d e 5. a b c d e
RESPONSE 6. a b c d e 7. a b c d e 8. a b c d e 9. a b c d e 10. a b c d e

GRID 11. a b c d e 12. a b c d e 13. a b c d e

Downloaded From : www.EasyEngineering.net


Downloaded From : www.EasyEngineering.net

82 SPEED TEST 40
14. In what time will 6250 amount to ` 6632.55 at 4 p.c. compound 20. A sum of money invested at compound interest amounts in
interest payable half-yearly? 3 years to Rs 2,400 and in 4 years to Rs. 2,520. The interest
rate per annum is :
3 3 3
(a) (b) (c) (a) 6% (b) 5% (c) 10%
5 2 4 (d) 12% (e) None of these
5 9 21. A computer is available for Rs. 39,000 cash or Rs. 17,000 as
(d) (e) cash down payment followed by five monthly installments
2 11
of Rs. 4,800 each. What is the rate of interest under the
15. If the simple interest on a certain sum of money for 3 years at
instalment plan ?
5% is ` 150, find the corresponding CI.
(a) 35.71% p.a. (b) 37.71% p.a.
(a) 197 (b) 157.62 (c) 137.36
(c) 36.71% p.a. (d) 38.71% p.a.
(d) 117.17 (e) 127.34
16. A sum of money becomes eight times in 3 years if the rate is (e) None of these
compounded annually. In how much time, the same amount 22. Under the Rural Housing Scheme, the Delhi Development
at the same compound interest rate will become sixteen times? Authority (DDA) allotted a house to Kamal Raj for Rs.
(a) 6 years (b) 4 years (c) 8 years 1,26,100. This payment is to be made in three equal annual
(d) 5 years (e) None of these instalments. If the money is reckoned at 5% per annum
17.
ww
The difference between the simple interest and the compound
interest compounded annually at the rate of 12% per annum
on Rs 5000 for two years will be :
compound interest, then how much is to be paid by Kamal
Raj in each instalment ?
(a) Rs. 45,205 (b) Rs. 47,405 (c) Rs. 46,305

18.
(a) Rs 47.50
(d) Rs 72 w.E (b) Rs 63
(e) None of these
(c) Rs 45

Rahul borrowed a certain sum from Dhawan at a certain rate


23.
(d) Rs. 48,505 (e) None of these
A finance company declares that, at a certain compound
interest rate, a sum of money deposited by anyone will

asy
of simple interest for 2 years. He lent this sum to Ramesh at
the same rate of interest compounded annually for the same
period. At the end of two years, he received Rs 4200 as
become 8 times in three years. If the same amount is deposited
at the same compound rate of interest, then in how many
year will it become 16 times ?
compound interest but paid Rs 4000 only as simple interest.
Find the rate of interest. En (a) 5 years
(d) 7 years
(b) 4 years
(e) None of these
(c) 6 years

19.
(a) 12%
(d) 10%
(b) 25%
(e) None of these
(c) 35%

What will Rs 1000 be worth after three years if it earns interest


gin
24. Seema invested an amount of Rs. 16,000 for two years on
compound interest and received an amount of Rs. 17,640 on
maturity. What is the rate of interest ?
at the rate of 5% compounded annually ?
(a) Rs 1075 (b) Rs 1257 (c) Rs 1157
(a) 5 % pa
(c) 4 % pa eer (b) 8 % pa
(d) Data inadequate
(d) Rs 1300 (e) None of these
25.
(e) None of these
ing
Two friends A and B jointly lent out Rs. 81,600 at 4% per
annum compound interest. After 2 years A gets the same

.ne
amount as B gets after 3 years. The investment made by B
was
(a) Rs. 40,000
(d) Rs. 38,000
(b) Rs. 30,000
(e) None of these
t
(c) Rs. 45,000

14. a b c d e 15. a b c d e 16. a b c d e 17. a b c d e 18. a b c d e


RESPONSE 19. a b c d e 20. a b c d e 21. a b c d e 22. a b c d e 23. a b c d e
GRID 24. a b c d e 25. a b c d e

Downloaded From : www.EasyEngineering.net


Downloaded From : www.EasyEngineering.net

Distance, Speed
and Time
41
Max. Marks : 30 No. of Qs. 30 Time : 20 min. Date : ........./......../. ..............
1. How many seconds will a train 60 m in length, travelling at 8. A 200 meter long train crosses a platform double its length in
the rate of 42 km an hour, rate to pass another train 84 m 36 seconds. What is the speed of the train in km/hr ?
long, proceeding in the same direction at the rate of 30 km (a) 60 (b) 48 (c) 64
an hour?
(d) 66 (e) None of these
(a) 42 (b) 43.2 (c) 45
9. A 160 meter long train running at a speed of 90 km/h crosses
a platform in 18 seconds. What is the length of the platform in
2.
ww
(d) 50 (e) None of these
A train takes 5 seconds to pass an electric pole. If the length
of the train is 120 metres, the time taken by it to cross a
meters?
(a) 210 (b) 240 (c) 290

(a) 12.5
(d) 15.5
w.E
railway platform 180 metres long is _______ seconds.
(b) 13.5
(e) None of these
(c) 14.5 10.
(d) 310 (e) None of these
Excluding the stoppages, the speed of a bus is 64 km/hr and
including the stoppages the speed of the bus is 48 km/hr. For
how many minutes does the bus stop per hour?
3.
asy
A train is running at the rate of 40 kmph. A man is also
going in the same direction parallel to the train at the speed
of 25 kmph. If the train crosses the man in 48 seconds, the
(a) 12.5 minutes
(d) 18 minutes
(b) 15 minutes (c) 10 minutes
(e) None of these
length of the train is _______ metres.
(a) 100 (b) 150 (c) 200 En 11. The ratio between the speed of a train and a car is 18 : 13 .
Also, a bus covered a distance of 480 kms. in 12 hours. The
speed of the bus is five-ninth the speed of the train. How

4.
(d) 250 (e) None of these
A train speeds past a pole in 15 seconds and speeds past a gin
much distance will the car cover in 5 hours ?
(a) 250 km. (b) 280 km.
platform 100 metres long in 25 seconds. Its length in metres
is ________ .
(a) 50 (b) 100 (c) 150 12. eer
(c) 260 km.
(e) None of these
(d) Cannot be determined

A 300 meter long train moving with an average speed of 126

5.
(d) 200 (e) None of these
A bus covers a distance of 2,924 kms in 43 hours. What is ing
km/hr crosses a platform in 24 seconds. A man crosses the
same platform in 5 minutes. What is the speed of the man in
meters/second
the speed of the bus?
(a) 72 kmph
(c) 68kmph
(b) 60kmph
(d) Cannot be determined
(a) 1.8 m/s
(c) 1.5 m/s
(b) 1.2 m/s
.ne
(d) Cannot be determined

6.
(e) None of these
A bus travels at the speed of 49 kmph and reaches its
destination in 7 hours. What is the distance covered by the
bus?
13.
(e) None of these
t
A bike covers a certain distance at the speed of 64 km/hr in 8
hours. If a bike was to cover the same distance in
approximately 6 hours, at what approximate speed should the
(a) 343 km (b) 283 km (c) 353 km bike travel?
(d) 245 km (e) 340 km (a) 80 km./hr. (b) 85 km/hr. (c) 90 km./hr.
7. Nilesh goes to school from his village & returns at the (d) 75 km/hr. (e) 70 km./hr
speed of 4 km/hr. If he takes 6 hours in all, then what is the
distance between the village and the school?
(a) 6 km (b) 5 km
(c) 4 km (d) Cannot be determined
(e) None of these

1. a b c d e 2. a b c d e 3. a b c d e 4. a b c d e 5. a b c d e
RESPONSE 6. a b c d e 7. a b c d e 8. a b c d e 9. a b c d e 10. a b c d e

GRID 11. a b c d e 12. a b c d e 13. a b c d e

Downloaded From : www.EasyEngineering.net


Downloaded From : www.EasyEngineering.net

84 SPEED TEST 41
14. The driver of a car driving @ 36 kmph locates a bus 40 23. A train does a journey without stoppage in 8 hours, if it had
meters ahead of him. After 20 seconds the bus is 60 meters travelled 5 km/h faster, it would have done the journey in 6
behind. The speed of the bus is : hours 40 minutes. Find its original speed.
(a) 36 kmph (b) 20 m/sec. (c) 72 m/sec. (a) 25 km/h (b) 40 km/h (c) 45 km/h
(d) 18 kmph (e) None of these (d) 36.5 km/h (e) None of these
15. Two trains 100 meters and 120 meters long are running in the 24. A car travels 25 km an hour faster than a bus for a journey of
same direction with speeds of 72 km/h and 54 km/h. In how 500 km. If the bus takes 10 hours more than the car, then the
much time will the first train cross the second? speeds of the bus and the car are
(a) 50 sec (b) 44 sec (c) 38 sec (a) 25 km/h and 40 km/h respectively
(d) 42 sec (e) None of these (b) 25 km/h and 60 km/h respectively
16. A man covers a certain distance on a scooter. If the scooter (c) 25 km/h and 50 km/h respectively
moved 4 km/h faster, it would take 30 minutes less. If it moved (d) Cannot be determined
2 km/h slower, it would have taken 20 minutes more. Find the (e) None of these
distance. 25. A thief goes away with a Maruti car at a speed of 40 km/h.
(a) 60 km (b) 58 km (c) 55 km The theft has been discovered after half an hour and the
(d) 50 km (e) None of these owner sets off in another car at 50 km/h. When will the owner
17.
ww
A boat running downstream covers a distance of 16 km in 2
hours while for covering the same distance upstream, it takes
4 hours. What is the speed of the boat in still water?
overtake the thief from the start.

(a) 2
1
2
hours (b) 2 hr 20 min
(a) 4 km/h
(c) 8 km/h
(e) None of these
w.E (b) 6 km/h
(d) Data inadequate
26.
(c) 1 hr 45 min
(e) None of these
(d) Cannot be determined

In a flight of 600 km, an aircraft was slowed down due to bad


18.
asy
In a 800 m race around a stadium having the circumference
of 200 m, the top runner meets the last runner on the 5th
minute of the race. If the top runner runs at twice the speed
weather. Its average speed for the trip was reduced by 200
km/ hr and the time of flight increased by 30 minutes. The

finish the race ? En


of the last runner, what is the time taken by the top runner to
duration of the flight is:
(a) 1 hours
(d) 4 hours
(b) 2 hours
(e) None of these
(c) 3 hours

19.
(a) 20 min
(d) 5 min
(b) 15 min
(e) None of these
(c) 10 min

A long distance runner runs 9 laps of a 400 meters track


gin
27. A train covers 180 km distance in 4 hours. Another train
covers the same distance in 1 hour less. What is the difference
everyday. His timings (in minutes) for four consecutive days
are 88, 96, 89 and 87 resplectively. On an average, how many
(a) 45 km
(d) 25 km eer
in the distances covered by these trains in one hour ?
(b) 9 km
(e) None of these
(c) 40 km
meters/minute does the runner cover ?
(a) 40 m/min
(d) 49 m/min
(b) 45 m/min
(e) None of these
(c) 38 m/min
28.
ing
The jogging track in a sports complex is 726 metres in
circumference. Pradeep and his wife start from the same point

20. A cyclist covers a distance of 750 m in 2 min 30 sec. What is


the speed in km/h of the cyclist ? .ne
and walk in opposite directions at 4.5 km/h and 3.75 km/h,
respectively. They will meet for the first time in :

21.
(a) 18 km/h
(d) 22 km/h
(b) 15 km/h
(e) None of these
(c) 20 km/h

An aeroplane flies along the four sides of a square at the


speeds of 200, 400, 600 and 800 km/h. Find the average speed
29.
(a) 5.5 min
(d) 4.9 min
(b) 6.0 min
(e) None of these
t
(c) 5.28 min

The speed of a boat in still water is 15 km/h and the rate of


stream is 5 km/h. The distance travelled downstream in 24
minutes is
of the plane around the field. (a) 4 km (b) 8 km (c) 6 km
(a) 384 km/h (b) 370 km/h (c) 368 km/h (d) 16 km (e) None of these
(d) 378 km/h (e) None of these 30. A man makes his upward journey at 16 km/h and downward
22. R and S start walking each other at 10 AM at the speeds of journey at 28 km/h. What is his average speed ?
3 km/h and 4 km/h respectively. They were initially 17.5 km (a) 32 km/h (b) 56 km/h (c) 20.36 km/h
apart. At what time do they meet? (d) 22 km/h (e) None of these
(a) 2 : 30 PM (b) 11 : 30 AM (c) 1 : 30 PM
(d) 12 : 30 PM (e) None of these

14. a b c d e 15. a b c d e 16. a b c d e 17. a b c d e 18. a b c d e

RESPONSE 19. a b c d e 20. a b c d e 21. a b c d e 22. a b c d e 23. a b c d e

GRID 24. a b c d e 25. a b c d e 26. a b c d e 27. a b c d e 28. a b c d e

29. a b c d e 30. a b c d e

Downloaded From : www.EasyEngineering.net


Downloaded From : www.EasyEngineering.net

Time and Work /


Pipe and Cistern
42
Max. Marks : 30 No. of Qs. 30 Time : 20 min. Date : ........./......../. ..............
1. ‘A’ can complete a piece of work in 12 days. ‘A’ and ‘B’ together 7. Sunil and Pradeep can complete a work in 5 days and 15 days
can complete the same piece of work in 8 days. In how many respectively. They both work for one day and then Sunil
days can ‘B’ alone complete the same piece of work? leaves. In how many days will the remaining work be completed
by Pradeep ?
(a) 15 days (b) 18 days (c) 24 days
(a) 11 days (b) 12 days (c) 15 days
(d) 28 days (e) None of these (d) 8 days (e) None of these
2. George takes 8 hours to copy a 50 page manuscript while

ww
Sonia can copy the same manuscript in 6 hours. How many
hours would it take them to copy a 100 page manuscript, if
they work together ?
8. If 6 men and 8 boys can do a piece of work in 10 days while 26
men and 48 boys can do the same in 2 days, the time taken by
15 men and 20 boys in doing the same work will be:

(a) 6
6
7 w.E
(b) 9 (c) 9
5
7 9.
(a) 4 days
(d) 7 days
(b) 5 days
(e) None of these
(c) 6 days

12 men complete a work in 9 days. After they have worked for


6 days, 6 more men join them. How many days will they take to
3.
(d) 14 (e) None of these

asy
A can finish a work in 18 days and B can do the same work
in half the time taken by A. Then, working together, what
complete the remaining work?
(a) 2 days
(d) 5 days
(b) 3 days
(e) None of these
(c) 4 days
part of the same work can they finish in a day?
1 1 2 En 10. A and B can do a job in 16 days and 12 days respectively. B
has started the work alone 4 days before finishing the job, A
(a)

(d)
6
2
(b)
9

(e) None of these


(c)
5
gin
joins B. How many days has B worked alone?
(a) 6 days
(d) 7 days
(b) 4 days
(e) None of these
(c) 5 days

4.
7
A and B can finish a work in 10 days while B and C can do it
in 18 days. A started the work, worked for 5 days, then B
11.
eer
Three taps A, B and C can fill a tank in 12, 15 and 20 hours
respectively. If A is open all the time and B and C are open for

worked for 10 days and the remaining work was finished by


C in 15 days. In how many days could C alone have finished (a) 6 hrs. ing
one hour each alternately, then the tank will be full in :
2
(b) 6 hrs.
3
(c) 7 hrs.
the whole work ?
(a) 30 days
(d) 24 days
(b) 15 days
(e) None of these
(c) 45 days 1
(d) 7 hrs.
2
(e) None of these.ne
5. Two pipes A and B can fill a cistern in 10 and 15 minutes
respectively. Both fill pipes are opened together, but at the
end of 3 minutes, ‘B’ is turned off. How much time will the
cistern take to fill ?
12. Two pipes A and B when working alone can fill a tank in 36
min. and 45 min. respectively. A waste pipe C can empty the t
tank in 30 min. First A and B are opened. After 7 min., C is also
opened. In how much time will the tank be full ?
(a) 6 min (b) 8 min (c) 10 min (a) 1/60 (b) 1/30 (c) 7/20
(d) 12 min (e) None of these (d) 13/20 (e) None of these
6. A is 30% more efficient than B. How much time will they, 13. A man can do a piece of work in 10 days but with the assistance
working together, take to complete a job which A alone could of his son, the work is done in 8 days. In how many days, his
have done in 23 days? son alone can do the same piece of work?
(a) 11 days (b) 13 days (a) 15 days (b) 22 days (c) 30 days
3 (d) 40 days (e) None of these
(c) 20 days (d) Cannot be determined
17
(e) None of these

1. a b c d e 2. a b c d e 3. a b c d e 4. a b c d e 5. a b c d e
RESPONSE 6. a b c d e 7. a b c d e 8. a b c d e 9. a b c d e 10. a b c d e

GRID 11. a b c d e 12. a b c d e 13. a b c d e

Downloaded From : www.EasyEngineering.net


Downloaded From : www.EasyEngineering.net

86 SPEED TEST 42
3 22. A contractor undertook to do a piece of work in 9 days. He
14. A can do of a work in 12 days. In how many days can he employed certain number of laboures but 6 of them were
4 absent from the very first day and the rest could finish the
1 work in only 15 days. Find the number of men originally
finish of the work?
8 employed .
(a) 6 days (b) 5 days (c) 3 days (a) 15 (b) 6 (c) 13
(d) 2 days (e) None of these (d) 9 (e) None of these
15. A can do a piece of work in 25 days and B in 20 days. They 1
23. After working for 8 days, Anil finds that only of the work
work together for 5 days and then A goes away. In how 3
has been done. He employs Rakesh who is 60% efficient as
many days will B finish the remaining work ?
Anil. How many more days will Anil take to complete the job?
(a) 17 days (b) 11 days (c) 10 days (a) 15 days (b) 12 days (c) 10 days
(d) 15 days (e) None of these (d) 8 days (e) None of these
16. A contractor undertakes to built a walls in 50 days. He 24. 2 men and 3 boys can do a piece of work in 10 days while
employs 50 peoples for the same. However after 25 days he 3 men and 2 boys can do the same work in 8 days. In how
finds that only 40% of the work is complete. How many more many days can 2 men and 1 boy to the work ?
man need to be employed to complete the work in time? 1 1 1
(a) 25 (b) 30 (c) 35 (a) 12days (b) 11 days (c) 15 days

17.
ww
(d) 20 (e) None of these
A and B can finish a work in 10 days while B and C can do it
in 18 days. A started the work, worked for 5 days, then B
2
1
(d) 13 days
2
2

(e) None of these


2

w.E
worked for 10 days and the remaining work was finished by C
in 15 days. In how many days could C alone have finished
the whole work ?
25. A can do a certain job in 12 days. B is 60% more efficient than
A. How many days B alone take to do the same job?
1 1

18.
(a) 30 days
(d) 24 days
(b) 15 days
(e) None of these
asy (c) 45 days

12 men complete a work in 18 days. Six days after they had


(a)
(d) 8
7
2
(b) 11
(e) None of these
(c) 8

26. A man can do a piece of work in 5 days but with the help of
2

started working, 4 men joined them. How many days will all
of them take to complete the remaining work ?
En his son he can do it in 3 days. In what time can the son do it
alone?

19.
(a) 10 days
(d) 9 days
(b) 12 days
(e) None of these
(c) 15 days

10 men can complete a piece of work in 15 days and 15 women gin (a)
1
2
(d) 8 days
6days (b) 7 days
(e) None of these
(c) 7
1
2
can complete the same work in 12 days. If all the 10 men and
15 women work together, in how many days will the work get eer
27. x is 3 times as faster as y and is able to complete the work in
40 days less than y. Then the time in which they can complete
completed?

(a) 6 (b) 6
1
3
(c) 6
2
3
the work together?

(a) 15 days ing


(b) 10 days (c) 7
1
days

(d) 7
2
3
(e) None of these
(d) 5 days (e) None of these
.ne
28. Pipe A can fill a tank in 5 hours, pipe B in 10 hours and pipe C
2

20. A can do a piece of work in 10 days, while B alone can do it in


15 days. They work together for 5 days and the rest of the
work is done by C in 2 days. If they get ` 450 for the whole
work, how should they divide the money ?
the tank be filled ?
(a) 2
(d) 3.5
(b) 2.5
(e) None of these
(c) 3

29. Two taps can fill a tank in 12 and 18 minutes respectively.


t
in 30 hours. If all the pipes are open, in how many hours will

(a) ` 225, ` 150, ` 75 (b) ` 250, ` 100, ` 100 Both are kept open for 2 minutes and the first is turned off. In
(c) ` 200, ` 150, ` 100 (d) ` 175, ` 175, ` 100 how many minutes more will the tank be filled ?
(e) None of these (a) 15 min. (b) 20 min. (c) 11 min.
21. 10 men and 15 women together can complete a work in 6 (d) 13 min. (e) None of these
days. It takes 100 days for one man alone to complete the 30. A cistern has three pipes, A, B and C. The pipes A and B can
same work. How many days will be required for one woman fill it in 4 and 5 hours respectively and C can empty it in 2
hours. If the pipes are opened in order at 1, 2 and 3 a.m.
alone to complete the same work?
respectively, when will the cistern be empty ?
(a) 90 (b) 125 (c) 145 (a) 3 p.m. (b) 4 p.m. (c) 5 p.m.
(d) 105 (e) None of these (d) 6 p.m. (e) None of these

14. a b c d e 15. a b c d e 16. a b c d e 17. a b c d e 18. a b c d e

RESPONSE 19. a b c d e 20. a b c d e 21. a b c d e 22. a b c d e 23. a b c d e

GRID 24. a b c d e 25. a b c d e 26. a b c d e 27. a b c d e 28. a b c d e

29. a b c d e 30. a b c d e

Downloaded From : www.EasyEngineering.net


Downloaded From : www.EasyEngineering.net

Problem Based
on Ages 43
Max. Marks : 30 No. of Qs. 30 Time : 20 min. Date : ........./......../. ..............
1. The average age of a man and his son is 16 years. The ratio (a) 10 yrs (b) 30 yrs (c) 20 yrs
of their ages is 15 : 1 respectively. What is the son’s age? (d) 40 yrs (e) None of these
(a) 30 years (b) 32 years (c) 2 years 9. Ten yrs ago, A was half of B in age. If the ratio of their present
(d) 4 years (e) None of these ages is 3 : 4, what will be the total of their present ages?
2. The average age of a lady and her daughter is 28.5. The (a) 25 (b) 35 (c) 45
ratio of their ages is 14 : 5 respectively. What is the daugh- (d) 50 (e) None of these

ww
ters age?
(a) 12 years
(c) 18 years
(b) 15 years
(d) Cannot be determined
10. The sum of the ages of a mother and her daughter is 50 yrs.
Also 5 yrs ago, the mother’s age was 7 times the age of the
daughter. What are the present ages of the mother and the
3.
(e) None of these

w.E
Present age of Sudha and Neeta are in the ratio of 6 : 7
respectively. Five years ago their ages were in the ratio of
5 : 6 respectively. What is Sudha’s present age?
daughter?
(a) 35, 5
(d) 25, 15
(b) 40, 10
(e) None of these
(c) 30, 20

(a) 30 years
(c) 40 years
(e) None of these
(b) 35 years
asy
(d) Cannot be determined
11. The ratio of the father’s age to the son’s age is 4 : 1. The
product of their ages is 196. What will be the ratio of their
ages after 5 years?
4.
children joined whose average age is 16 years. What is theEn
Average age of 36 children of the class is 15 years. 12 more (a) 7 : 3
(d) 17 : 3
(b) 14 : 9
(e) None of these
(c) 11 : 4

average age of all the 48 children together?


(a) 15.25 years
(c) 15.3 years
(b) 15.5 years
(d) 15.4 years
12.
gin
The ratio of Rita’s age to the age of her mother is 3 : 11. The
difference of their ages is 24 yrs. What will be the ratio of their

5.
(e) None of these
Two years ago the ratio of the ages of Swati and Khyati (a) 1 : 2
(d) 2 : 5 eer
ages after 3 yrs?
(b) 1 : 3
(e) None of these
(c) 3 : 7
was 5 : 7 respectively. Two years hence the ratio of their
ages will be 7 : 9 respectively. What is the present age of
Khyati? 13. ing
A man’s age is 125% of what it was 10 years ago, but 83 %
1

(a) 16 years
(c) 12 years
(b) 14.5 years
(d) Cannot be determined
(a) 30 yrs (b) 40 yrs .ne (c) 50 yrs
3
of what it will be after 10 years. What is his present age?

6.
(e) None of these
The age of a man is 4 times that of his son. 5 yrs ago, the
man was nine times as old as his son was at that time. What
is the present age of the man?
(a) 28 yrs (b) 32 yrs (c) 40 yrs
14.
(d) 60 yrs (e) None of these
t
The age of a man is three times the sum of the ages of his two
sons. Five years hence, his age will be double of the sum of
the ages of his sons. The father's present age is
(d) 42 yrs (e) None of these (a) 40 years (b) 45 years (c) 50 years
7. After 5 yrs, the age of a father will be thrice the age of his (d) 55 years (e) 65 years
son, whereas five years ago, he was 7 times as old as his
15. The ratio between the present ages of P and Q is 3 : 4
son was. What are their present ages?
respectively. Four years hence Q will be 5 years older than P.
(a) 30 yrs (b) 40 yrs (c) 50 yrs
What is P’s present age?
(d) 60 yrs (e) None of these
8. 10 Yrs ago, Sita’s mother was 4 times older than her (a) 15 years (b) 20 years
daughter. After 10 yrs, the mother will be two times older (c) 25 years (d) Cannot be determined
than the daughter. What is the present age of Sita? (e) None of these

1. a b c d e 2. a b c d e 3. a b c d e 4. a b c d e 5. a b c d e
RESPONSE 6. a b c d e 7. a b c d e 8. a b c d e 9. a b c d e 10. a b c d e

GRID 11. a b c d e 12. a b c d e 13. a b c d e 14. a b c d e 15. a b c d e

Downloaded From : www.EasyEngineering.net


Downloaded From : www.EasyEngineering.net

88 SPEED TEST 43
16. Present ages of Rama and Shyama are in the ratio of 4 : 5 24. The sum of the ages of 5 children born at the intervals of 3
respectively. Five years hence the ratio of their ages be- years each is 50 years. What is the age of the youngest
comes 5 : 6 respectively. What is Rama’s present age? child?
(a) 25 years (b) 22 years (c) 20 years (a) 4 years (b) 8 years (c) 10 years
(d) 30 years (e) None of these
(d) 12 years (e) None of these
17. In a family, a couple has a son and daughter. The age of the
father is three times that of his daughter and the age of the 25. If 6 years are subtracted from the present age of Gagan and
son is half of his mother. The wife is nine years younger to the remainder is divided by 18, then the present age of his
her husband and the brother is seven years older than his grandson Anup is obtained. If Anup is 2 years younger to
sister. What is the age of the mother? Madan whose age is 5 years, then what is Gagan’s present
(a) 40 years (b) 45 years (c) 50 years age?
(d) 60 years (e) 65 years (a) 48 years (b) 60 years (c) 84 years
18. Ram’s present age is three times his son’s present age and (d) 96 years (e) 100 years
two-fifth of his father’s present age. The average of the
26. The ratio between the school ages of Neelam and Shaan is
present ages of all of them is 46 years. What is the difference
5 : 6 respectively. If the ratio between the one-third age of

ww
between the Ram’s son’s present age and Ram’s father’s
present age?
(a) 68 years (b) 88 years
Neelam and half of Shaan’s age is 5 : 9, then what is the
school age of Shaan?

19.
(c) 58 years
(e) None of these w.E (d) Cannot be determined

Abhay’s age after six years will be three-seventh of his


(a) 25 years
(b) 30 years

asy
father’s age. Ten years ago, the ratio of their ages was 1 : 5.
What is Abhay’s father’s age at present?
(c) Cannot be determined
(d) 35 years

(a) 30 yrs.
(d) 60 yrs.
(b) 40 yrs.
(e) 70 years
(c) 50 yrs.
En 27.
(e) None of these
A is two years older than his son. In two years, his age will
20. The present ages of three persons are in proportions
4 : 7 : 9. Eight years ago, the sum of their ages was 56. Find
their present ages (in years).
gin be twice the age of his son. The present age of the son is:
(a) 7 (b) 8 (c) 9

(a) 8, 20, 28
(d) 25, 30, 40
(b) 16, 28, 36
(e) None of these
(c) 20, 35, 45
28.
(d) 10
eer (e) 11
Eighteen years ago, a father was three times as old as his
21. Tanya’s grandfather was 8 times older to her 16 years ago.
He would be 3 times of her age 8 years from now. Eight years ing
son. Now the father is only twice as old as his son. Then the
sum of the present ages of the son and the father is:
ago, what was the ratio of Tanya’s age to that of her grand-
father?
(a) 54
(d) 108
(b) 72
(e) 116
.ne (c) 105

22.
(a) 1 : 2
(d) 11 : 53
(b) 1 : 5
(e) None of these
(c) 3 : 8

Q is as much younger than R as he is older than T. If the sum


of the ages of R and T is 50 years, what is definitely the
29. One year ago, Preeti was four times as old as her daughter
t
Sonal. Six years hence, Preeti’s age will exceed her daughter’s
age by 9 years. The ratio of the present ages of Preeti and
her daughter is :
difference between R and Q’s age?
(a) 1 year (b) 2 years (c) 25 years (a) 9 : 2 (b) 11 : 3 (c) 12 : 5
(d) Data inadequate (e) None of these (d) 13 : 4 (e) 17 : 7
23. The sum of the ages of a father and his son is 45 years. Five 30. The present age of the father and the son are in the ratio of
years ago, the product of their ages is 34. Find the present 8:3. After 12 years the ratio of their ages will be 2:1. What is
age of father. the sum of the present age of the father and the son?
(a) 32 years (b) 36 years (c) 38 years (a) 66yrs (b) 70yrs (c) 74yrs
(d) 40 years (e) 39 years (d) 78yrs (e) 80yrs.

16. a b c d e 17. a b c d e 18. a b c d e 19. a b c d e 20. a b c d e


21. a b c d e 22. a b c d e 23. a b c d e 24. a b c d e 25. a b c d e
RESPONSE
26. a b c d e 27. a b c d e 28. a b c d e 29. a b c d e 30. a b c d e
GRID

Downloaded From : www.EasyEngineering.net


Downloaded From : www.EasyEngineering.net

Permutation and
Combination
44
Max. Marks : 30 No. of Qs. 30 Time : 20 min. Date : ........./......../. ..............
1. In how many ways can six different rings be worn on four 8. In how many different ways can it be done so that the
fingers of one hand ? committee has at least one woman?
(a) 10 (b) 12 (c) 15 (a) 210 (b) 225 (c) 195
(d) 185 (e) None of these
(d) 16 (e) None of these
9. In how many different ways can it be done, so that the
2. In how many ways can 7 persons be seated at a round table
committee has at least 2 men?
if 2 particular persons must not sit next to each other ?

ww
(a) 5040
(d) 720
(b) 240
(e) None of these
(c) 480
10.
(a) 210
(d) 185
(b) 225
(e) None of these
(c) 195

In how many ways can 5 boys be chosen from 6 boys and 4


3.

come together ? w.E


In how many different ways can the letters of the word
‘MATHEMATICS’ be arranged so that the vowels always
girls so as to include exactly one girl?
(a) 252
(d) 90
(b) 210
(e) 60
(c) 126

4.
(a) 10080
(d) 12960
(b) 4989600
(e) None of these
asy (c) 120960

The number of ways in which four letters of the word


11. In how many different ways can the letters of the word
CORPORATION be arranged?
(a) 3326400 (b) 1663200 (c) 831600
‘MATHEMATICS’ can be arranged is
(a) 136 (b) 2454 (c) 1680 En 12.
(d) 415800 (e) 207900
In how many different ways can the letters of the word

5.
(d) 192 (e) None of these
In how many different ways can hte letters of the word
gin
"COUNTRY" be arranged in such a way that the vowels
always come together?
(a) 720 (b) 1440 (c) 2880
‘PRETTY’ be arranged?
(a) 120
(d) 720
(b) 36
(e) None of these
(c) 360
13.
(d) 5040
eer (e) None of these
In how many different ways can the letters of the word

6. In how many different ways can be letters of the word


‘CYCLE’ be arranged?
‘PROBLEM’ be arranged ?
(a) 5060
(d) 980 ing
(b) 720
(e) None of these
(c) 5040

(a) 120
(d) 80
(b) 4240
(e) None of these
(c) 30 14.
.ne
How many different ways can the letters in the word ATTEND
be arranged?
7. In how many different ways can the letters of the word
TRUST be arranged?
(a) 240
(d) 25
(b) 120
(e) None of these
(c) 80
15.
(a) 60
(d) 80
(b) 120
(e) None of these
(c) 240

In how many different ways can the letters of the word


t
DIRECTIONS (Qs.8 & 9): Answer these questions on the basis ‘OFFICES’ be arranged?
of the information given below :
From a group of 6 men and 4 women a Committee of 4 persons is (a) 2520 (b) 5040 (c) 1850
to be formed. (d) 1680 (e) None of these

1. a b c d e 2. a b c d e 3. a b c d e 4. a b c d e 5. a b c d e
RESPONSE 6. a b c d e 7. a b c d e 8. a b c d e 9. a b c d e 10. a b c d e

GRID 11. a b c d e 12. a b c d e 13. a b c d e 14. a b c d e 15. a b c d e

Downloaded From : www.EasyEngineering.net


Downloaded From : www.EasyEngineering.net

90 SPEED TEST 44
16. In how many different ways can the letters of the word 22. Letters of the word DIRECTOR are arranged in such a way
‘ARMOUR’ be arranged? that all the vowels come together. Find out the total number
of ways for making such arrangement.
(a) 720 (b) 300 (c) 640 (a) 4320 (b) 2720
(d) 350 (e) None of these (c) 2160 (d) 1120
17. In how many different ways can 4 boys and 3 girls be (e) None of these
arranged in a row such that all boys stand together and all 23. How many three digit numbers can having only two
consecutive digits identical is
the girls stand together?
(a) 153 (b) 162
(a) 75 (b) 576 (c) 288 (c) 168 (d) 163
(d) 24 (e) None of these (e) None of these
18. In how many different ways can be letters of the word 24. In how many ways can the letters of the word 'PRAISE' be
SOFTWARE be arranged in such a way that the vowels arranged. So that vowels do not come together?
always come together? (a) 720 (b) 576
(c) 440 (d) 144
(a) 13440 (b) 1440
(e) None of these
(c) 360 (d) 120

19. ww
(e) None of these
A bag contains 2 red, 3 green and 2 blue balls. 2 balls are to
be drawn randomly. What is the probability that the balls
25. The number of ways in which one or more balls can be
selected out of 10 white, 9 green and 7 blue balls is
(a) 892 (b) 881

(a)
5
w.E
drawn contain no blue ball?

(b)
10 26.
(c) 891
(e) None of these
(d) 879

How many 3-digit numbers, each less than 600, can be formed
7

2 asy21

11
from {1, 2, 3, 4, 7, 9} if repetition of digits is allowed?
(a) 216
(c) 144
(b) 180
(d) 120
(c)
7
(e) None of these
(d)
21
En 27.
(e) None of these
If a secretary and a joint secretary are to be selected from a
20. In how many different ways can the letters of the word
BOOKLET be arranged such that B and T always come gin committee of 11 members, then in how many ways can they
be selected ?
together?
(a) 360
(c) 480
(b)
(d)
720
5040
(a) 110
(c) 22
eer
(e) None of these
(b) 55
(d) 11

21.
(e) None of these
In a box there are 8 red, 7 blue and 6 green balls. One ball is
28.
ing
On a railway route there are 20 stations. What is the number
of different tickets required in order that it may be possible
picked up randomly. What is the probability that it is neither
red nor green? (a) 40 (b) 380
.ne
to travel from every station to every other station?

(a)

(c)
3
7
19
(b)

(d)
2
3

9
29.
(c) 400
(e) None of these
(d) 420

t
What is the number of five-digit numbers formed with 0, 1, 2,
3, 4 without any repetition of digits?
4 21 (a) 24 (b) 48
(e) None of these (c) 96 (d) 120
(e) None of these
30. What is the number of three-digit odd numbers formed by
using the digits 1, 2, 3, 4, 5, 6 if repetition of digits is allowed?
(a) 60 (b) 108
(c) 120 (d) 216
(e) None of these

16. a b c d e 17. a b c d e 18. a b c d e 19. a b c d e 20. a b c d e


RESPONSE 21. a b c d e 22. a b c d e 23. a b c d e 24. a b c d e 25. a b c d e
GRID 26. a b c d e 27. a b c d e 28. a b c d e 29. a b c d e 30. a b c d e

Downloaded From : www.EasyEngineering.net


Downloaded From : www.EasyEngineering.net

Probability 45
Max. Marks : 25 No. of Qs. 25 Time : 20 min. Date : ........./......../. ..............
1. Tickets numbered 1 to 20 are mixed up and then a ticket is 6. In a box, there are 8 red, 7 blue and 6 green balls. One ball is
drawn at random. What is the probability that the ticker picked up randomly. What is the probability that it is neither
drawn bears a number which is a multiple of 3? red nor green?
3 3 2 2 3 7
(a) (b) (c) (a) (b) (c)
10 20 5 3 4 19

ww
1 8 9
(d) (e) None of these (d) (e)
2 21 21
7. Two dice are tossed. The probability that the total score is a
2. In a lottery, there are 10 prizes and 25 blanks. A lottery is

(a)
1
10
w.E
drawn at random. What is the probability of getting a prize?

(b)
2
5
(c)
2
7
prime number is :

(a)
1
6
(b)
5
12
(c)
1
2

(d)
7
5
(e) None of these asy 8.
(d)
7
9
(e) None of these
In a simultaneous throw of two coins, the probability of getting
3. One card is drawn at random from a pack of 52 cards. What
is the probability that the card drawn is a face card? En at least one head is
1 1 2

(a)
1
13
(b)
4
13
(c)
1
4 gin (a)

3
2
(b)
3
(c)
3

(d)
9
52
(e) None of these 9.
(d)
4
eer (e) None of these
Three unbiased coins are tossed. What is the probability of
4. One card is drawn from a pack of 52 cards. What is the
probability that the card drawn is either a red card or a
king?
getting at least 2 heads?

(a)
1
4
(b) ing
1
2
(c)
1
3

(a)
1
2
(b)
6
13
(c)
7
13
(d)
1
8
(e) None of these .ne
5.
(c)
27
52
(e) None of these
A bag contains 6 black and 8 white balls. One ball is drawn
10.
number greater than 4?

(a)
1
(b)
1
(c)
2
t
In a single throw of a die, what is the probability of getting a

at random. What is the probability that the ball drawn is 2 3 3


white? 1
(d) (e) None of these
3 4 1 4
(a) (b) (c) 11. In a simultaneous throw of two dice, what is the probability
4 7 8 of getting a total of 7?
3 1 1 2
(d) (e) None of these (a) (b) (c)
7 6 4 3
3
(d) (e) None of these
4

1. a b c d e 2. a b c d e 3. a b c d e 4. a b c d e 5. a b c d e
RESPONSE 6. a b c d e 7. a b c d e 8. a b c d e 9. a b c d e 10. a b c d e

GRID 11. a b c d e

Downloaded From : www.EasyEngineering.net


Downloaded From : www.EasyEngineering.net

92 SPEED TEST 45
12. What is the probability of getting a sum 9 from two throws 19. What is the probability that a leap year selected at random
of a date? contains 53 Mondays?
1 1 1 1 2 7
(a) (b) (c) (a) (b) (c)
6 8 9 7 7 366
1 26
(d) (e) None of these (d) (e) None of these
12 183
13. In a simultaneous throw of two dice, what is the probability 20. What is the probability of getting a sum of 7 with two dice?
of getting a doublet?
1 1 1
1 1 2 (a) (b) (c)
(a) (b) (c) 6 3 12
6 4 3
5
3 (d) (e) None of these
(d) (e) None of these 36
7 21. A bag contains 5 white, 7 red and 8 black balls. If 4 balls are
14. In a simultaneous throw of two dice, what is the probability drawn one by one with replacement, what is the probability
of getting a total of 10 or 11? that all are white ?

(a)
ww
1
4
(b)
1
6
(c)
7
12
(a)
1
256
(b)
1
16
(c)
4
20

15.
(d)
5
36
w.E (e) None of these
A coin is tossed three times. What is the probability of 22.
(d)
4
8
(e) None of these
Two dice are tossed. The probability that the total score is a

(a) 1/4 (b) 1/5 asy


getting head and tail (HTH) or tail and head (THT)
alternatively ?
(c) 1/6
prime number is :

(a)
1
6
(b)
5
12
(c)
1
2

16.
(d) 1/8 (e) None of these
In a lottery, 16 tickets are sold and 4 prizes are awarded. If a
En (d)
7
9
(e) None of these
person buys 4 tickets,what is the probability of his winning
a prize?
4 175 1
gin
23. A bag contains 3 white balls and 2 black balls. Another bag
contains 2 white balls and 4 black balls. A bag is taken and a
(a)
164
(b)
256
(c)
4
eer
ball is picked at random from it. The probability that the ball
will be white is:
7 7 5

17.
(d)
81
256
(e) None of these
Two letters are drawn at random from the word ‘HOME’.
(a)
11
7 ing (b)
30
(c)
11

What is the probability that both the letters are vowels?


(a) 1/ 6 (b) 5/ 6 (c) 1/ 2 24.
(d)
15
.ne
(e) None of these
Out of 20 consecutive positive integers, two are chosen at

18.
(d) 1/ 3

(a)
1
(e) None of these
Three dice are thrown. What is the probability that the same
number will appear on each of them?

(b)
1
(c)
1 25.
random. The probability that their sum is odd is
(a) 19/20
(d) 9/19

a multiple of 3.
(b) 10/19
(e) None of these
(c) 1/20

An unbiased die is tossed. What is the probability of getting


t
6 18 24
1 1 2
1 (a) (b) (c)
(d) (e) None of these 2 3 5
36
1
(d) (e) None of these
6

12. a b c d e 13. a b c d e 14. a b c d e 15. a b c d e 16. a b c d e


RESPONSE 17. a b c d e 18. a b c d e 19. a b c d e 20. a b c d e 21. a b c d e
GRID 22. a b c d e 23. a b c d e 24. a b c d e 25. a b c d e

Downloaded From : www.EasyEngineering.net


Downloaded From : www.EasyEngineering.net

Area and Perimeter 46


Max. Marks : 35 No. of Qs. 35 Time : 25 min. Date : ........./......../. ..............
1. The circumference of a circle is 44 metres. Find the area of 10. How many plants will be there in a circular bed whose outer
the circle. edge measure 30 cms, allowing 4 cm2 for each plant ?
(a) 154 m2 (b) 160 m2 (c) 175 m2 (a) 18 (b) 750 (c) 24
(d) 168 m 2 (e) None of these (d) 120 (e) None of these
2. The length and breadth of a rectangle are in the ratio 9 : 5. 11. A rectangular plot 15 m ×10 m, has a path of grass outside it.
If its area is 720 m2, find its perimeter. If the area of grassy pathway is 54 m 2, find the width of the

ww
(a) 112 metre
(c) 110 metre
(e) None of these
(b) 115 metre
(d) 118 metre
path.
(a) 4 m
(d) 1 m
(b) 3 m
(e) None of these
(c) 2 m

3.

(a) 88 cm2
w.E
A circle and a rectangle have the same perimeter. The sides
of the rectangle are 18 cm and 26 cm. What is the area of the
circle ?
(b) 154 cm2 (c) 1250 cm2
12. If the area of a circle decreases by 36%, then the radius of a
circle decreases by
(a) 20%
(d) 64%
(b) 18%
(e) None of these
(c) 36%

4.
(d) 616 cm2
asy
(e) None of these
The cost of carpeting a room 18m long with a carpet 75 cm
wide at ` 4.50 per metre is ` 810. The breadth of the room is:
13. The floor of a rectangular room is 15 m long and 12 m wide.
The room is surrounded by a verandah of width 2 m on all

(a) 7 m
(d) 8.5 m
(b) 7.5 m
(e) None of these
(c) 8 m
En its sides. The area of the verandah is :
(a) 124 m2
(d) 58 m 2
(b) 120 m2
(e) None of these
(c) 108 m2

5. If the perimeter and diagonal of a rectangle are 14 and 5 cms


respectively, find its area.
(a) 12 cm2 (b) 16 cm2 (c) 20 cm2
gin
14. A circular grass lawn of 35 metres in radius has a path 7
metres wide running around it on the outside. Find the area of
path.

6.
(d) 24 cm2 (e) None of these
In an isoscele right angled triangle, the perimeter is 20 metre.
(a) 1694 m2
(d) 1900 m2eer (b) 1700 m2
(e) None of these
(c) 1598 m2

Find its area.


(a) 9,320 m2
(d) 8,150 m 2
(b) 8,750 m2
(e) None of these
(c) 7,980 m2
15.

ing
The radius of the wheel of a bus is 70 cms and the speed of
the bus is 66 km/h, then the r.p.m. (revolutions per minutes) of
the wheel is
7. The diameter of a garden roller is 1.4 m and it is 2 m long.
æ
How much area will it cover in 5 revolutions ? ç use p =
22 ö
(a) 200
(d) 330
(b) 250
.ne
(e) None of these
(c) 300

8.
(a) 40 m2
(d) 36 m2
(b) 44 m2
(e) None of these
è
(c) 48 m2

The area of a triangle is 615 m2. If one of its sides is 123


7 ø
÷ 16.

17.
and the perimeter is 32 cm. The area of the triangle is
(a) 72 cm2
(d) 67 cm 2
(b) 60 cm2
(e) None of these
(c) 66 cm2 t
The altitude drawn to the base of an isosceles triangle is 8 cm

The area of a square field is 576 km 2. How long will it take for
metre, find the length of the perpendicular dropped on that a horse to run around at the speed of 12 km/h ?
side from opposite vertex. (a) 12 h (b) 10 h (c) 8 h
(a) 15 metres (b) 12 metres (d) 6 h (e) None of these
(c) 10 metres (d) 9 metres 18. The area of a rectangular field is 144 m2. If the length had
(e) None of these been 6 metres more, the area would have been 54 m 2 more.
9. A horse is tethered to one corner of a rectangular grassy The original length of the field is
field 40 m by 24 m with a rope 14 m long. Over how much (a) 22 metres (b) 18 metres (c) 16 metres
area of the field can it graze? (d) 24 metres (e) None of these
(a) 154 cm2 (b) 308 m2 (c) 150 m2
(d) 145 cm 2 (e) None of these

1. a b c d e 2. a b c d e 3. a b c d e 4. a b c d e 5. a b c d e

RESPONSE 6. a b c d e 7. a b c d e 8. a b c d e 9. a b c d e 10. a b c d e

GRID 11. a b c d e 12. a b c d e 13. a b c d e 14. a b c d e 15. a b c d e


16. a b c d e 17. a b c d e 18. a b c d e

Downloaded From : www.EasyEngineering.net


Downloaded From : www.EasyEngineering.net

94 SPEED TEST 46
19. The ratio between the length and the breadth of a rectangular 28. The perimeter of a square is double the perimeter of a
park is 3 : 2. If a man cycling along the boundary of the park rectangle. The area of the rectangle is 240 sq cm. What is the
at the speed of 12km / hr completes one round in 8 minutes, area of the square ?
then the area of the park (in sq. m) is: (a) 100 sq cm (b) 36 cq cm
(a) 15360 (b) 153600 (c) 30720 (c) 81 sq cm (d) Cannot be determined
(d) 307200 (e) None of these (e) None of these
20. A wire can be bent in the form of a circle of radius 56 cm. If it 29. If the perimeter of a square is equal to the radius of a circle
is bent in the form of a square, then its area will be: whose area is 39424 sq. cm, what is the area of the square ?
(a) 3520 cm2 (b) 6400 cm2 (c) 7744 cm2 (a) 1225 sq. cm (b) 441 sq. cm
(d) 8800 cm 2 (e) None of these (c) 784 sq. cm (d) Cannot be determined
21. The length of a room is double its breadth. The cost of (e) None of these
colouring the ceiling at ` 25 per sq. m is ` 5,000 and the cost 30. Two poles, 15 m and 30 m high, stand upright in a playground.
of painting the four walls at ` 240 per sq. m is ` 64,800. Find If their feet be 36 m apart, find the distance between their
the height of the room. tops.
(a) 4.5 m (b) 4 m (c) 3.5 m (a) 35 cm (b) 39 cm (c) 45 cm
(d) 5 m (e) None of these (d) 50 cm (e) None of these
22. The surface area of a cube is 150 m2. The length of its 31. A semi-circle is constructed on each side of a square of

(a)
ww
diagonal is

5 3m (b) 5 m (c)
10
m
length 2m. Find the area of the whole figure.
(a) (5 + 3p) m2
(d) (4 + 2p) m2
(b) (4 + 3p) m2 (c) (4 + p) m2
(e) None of these

23.
(d) 15 m
w.E (e) None of these
The length and breadth of a playground are 36m and 21 m
respectively. Poles are required to be fixed all along the
3 32. In a quadrilateral, the length of one of its diagonal is 23 cm
and the perpendiculars drawn on this diagonal from other
two vertices measure 17 cm and 7 cm respectively. Find the
area of the quadrilateral.

asy
boundary at a distance 3m apart. The number of poles
required will be
(a) 250 cm2
(d) 325 cm 2
(b) 276 cm2
(e) None of these
(c) 300 cm2

24.
(a) 39
(d) 40
(b) 38
(e) None of these
(c) 37

What would be the area of a square whose diagonal measures En 33. A circular wire of radius 42 cm is cut and bent in the form of
a rectangle whose sides are in the ratio of 6 : 5. Find the
smaller side of the rectangle.
28 cm?
(a) 288 sq cm (b) 514 sq cm (c) 428 sq cm gin (a) 50 cm
(d) 80 cm
(b) 60 cm
(e) None of these
(c) 70 cm

25.
(d) 392 sq cm (e) None of these
The perimeter of a square is one-fourth the perimeter of a
rectangle. If the perimeter of the square is 44 cm and the
34.

eer
The following figure contains three squares with areas of
100, 16 and 49 sq. units respectively laying side by side as
shown. By how much should the area of the middle square
length of the rectangle is 51 cm, what is the difference
between the breadth of the rectangle and the side of the
square?
three squares is 19?
ing
be reduced in order that the total length PQ of the resulting

(a) 30 cm
(d) 32 cm
(b) 18 cm
(e) None of these
(c) 26 cm
.ne
26. The area of a rectangle is equal to the area of a circle with
circumference equal to 220 metres. What is the length of the
rectangle if its breadth is 50 metres?
(a) 56 metres
(d) 69 metres
(b) 83 metres
(e) None of these
(c) 77 metres
P
100

16
49
t
Q
27. A man riding a bicycle, completes one lap of a circular field (a) 12 (b) 4 (c) 3
along its circumference at the speed of 79.2 km/hr in 2 minutes (d) 2 (e) 6
40 seconds. What is the area of the field? 35. A rectangle has perimeter of 50 metres. If its length is 13
(a) 985600 sq metre (b) 848500 sq metre metres more than its breadth, then its area is:
(c) 795600 sq metre (d) Cannot be determined (a) 124 m2 (b) 144 m2 (c) 114 m2
(d) 104 m 2 (e) 117m 2
(e) None of these

19. a b c d e 20. a b c d e 21. a b c d e 22. a b c d e 23. a b c d e

RESPONSE 24. a b c d e 25. a b c d e 26. a b c d e 27. a b c d e 28. a b c d e

GRID 29. a b c d e 30. a b c d e 31. a b c d e 32. a b c d e 33. a b c d e

34. a b c d e 35. a b c d e

Downloaded From : www.EasyEngineering.net


Downloaded From : www.EasyEngineering.net

Volume and
Surface Area
47
Max. Marks : 25 No. of Qs. 25 Time : 20 min. Date : ........./......../. ..............
1. A cylindrical bucket of height 36 cm and radius 21 cm is 7. The length of a cold storage is double its breadth. Its height
filled with sand. The bucket is emptied on the ground and is 3 metres. The area of its four walls (including the doors) is
a conical heap of sand is formed, the height of the heap 108 m2. Find its volume.
being 12 cm. The radius of the heap at the base is : (a) 215 m3 (b) 216 m3
(a) 63 cm (b) 53 cm (c) 217 m 3 (d) 218 m3
(c) 56 cm (d) 66 cm (e) None of these

2.
ww
(e) None of these
A metal cube of edge 12 cm is melted and formed into three
smaller cubes. If the edges of two smaller cubes are 6 cm
8. A cube of 384 cm2 surface area is melt to make x number of
small cubes each of 96 mm2 surface area. The value of x is
(a) 80,000 (b) 8

(a) 10 cm
(c) 12 cm
w.E
and 8 cm, then find the edge of the third smaller cube.
(b) 14 cm
(d) 16 cm 9.
(c) 8,000
(e) None of these
(d) 800

A conical vessel, whose internal radius is 12 cm and height 50

3.
(e) None of these

asy
A well 22.5 deep and of diameter 7 m has to be dug out. Find
the cost of plastering its inner curved surface at ` 3 per sq.
cm, is full of liquid. The contents are emptied into a cylindrical
vessel with internal radius 10 cm. Find the height to which the
liquid rises in the cylindrical vessel.
metre.
(a) ` 1465 (b) ` 1485 En (a) 18 cm
(c) 24 cm
(b) 22 cm
(d) 20 cm

4.
(c) ` 1475
(e) None of these
(d) ` 1495

A copper sphere of radius 3 cm is beaten and drawn into a


10.
gin (e) None of these
A hollow sphere of internal and external diameters 4 cm and 8
cm respectively is melted into a cone of base diamater 8 cm.
wire of diametre 0.2 cm. The length of the wire is
(a) 9 m (b) 12 m (a) 12 cm eer
The height of the cone is:
(b) 14 cm

5.
(c) 18 m
(e) None of these
(d) 36 m

If the volume of a sphere is divided by its surface area, the 11.


(c) 15 cm
(e) None of these
ing (d) 18 cm

A cone of height 9 cm with diameter of its base 18 cm is


result is 27 cms. The radius of the sphere is
(a) 9 cms (b) 27 cms percentage of the wood wasted is: .ne
carved out from a wooden solid sphere of radius 9 cm. The

6.
(c) 81 cms
(e) None of these
(d) 243 cms

A cistern 6 m long and 4 m wide contains water up to a


depth of 1 m 25 cm. The total area of the wet surface is: 12.
(a) 25%
(c) 50%
(e) None of these
(b) 30%
(d) 75%
t
A monument has 50 cylindrical pillars each of diameter 50 cm
(a) 49 m2 (b) 50 m2 and height 4 m. What will be the labour charges for getting
(c) 53.5 m 2 (d) 55 m2 these pillars cleaned at the rate of 50 paise per sq. m?
(e) None of these (use p = 3.14)
(a) ` 237 (b) ` 157
(c) ` 257 (d) ` 353
(e) None of these

1. a b c d e 2. a b c d e 3. a b c d e 4. a b c d e 5. a b c d e
RESPONSE 6. a b c d e 7. a b c d e 8. a b c d e 9. a b c d e 10. a b c d e
GRID 11. a c d e 12. a c d e
b b

Downloaded From : www.EasyEngineering.net


Downloaded From : www.EasyEngineering.net

96 SPEED TEST 47
13. A conical vessel of base radius 2 cm and height 3 cm is filled 20. The curved surface of a right circular cone of height 15 cm
with kerosene. This liquid leaks through a hole in the bottom and base diameter 16 cm is :
and collects in a cylindrical jar of radius 2 cm. The kerosene (a) 120p cm2 (b) 60 p cm2
level in the jar is (c) 136 p cm 2 (d) 68p cm2
(a) p cm (b) 1.5 cm (e) None of these
(c) 1 cm (d) 3 cm 21. The weight of a solid cone having diameter 14 cm and vertical
(e) None of these height 51 cm is ....., if the material of solid cone weighs 10
14. In a swimming pool measuring 90 m by 40 m, 150 men take a grams per cubic cm :
dip. If the average displacement of water by a man is 8 cubic (a) 16.18 kg (b) 17.25 kg
metres, what will be the rise in water level? (c) 26.18 kg (d) 71.40 kg
(a) 33.33 cm (b) 30 cm (e) None of these
(c) 20 cm (d) 25 cm 22. A hemispherical bowl is made of steel 0.5 cm thick. The inside
(e) None of these radius of bowl being 4 cm. The volume of the steel used in
15. Three cubes of a metal are of edges 3 cm, 4 cm and 5 cm. making the bowl is :
These are melted together and from the melted material, (a) 55.83 cm2 (b) 56.83 cm2
another cube is formed. The edge of this cube is : (c) 57.83 cm 2 (d) 58.83 cm2

ww
(a) 8 cm
(c) 9 cm
(e) None of these
(b) 10 cm
(d) 6 cm 23.
(e) None of these
Consider the volumes of the following :
A. A parallelopiped of length 5 cm, breadth 3 cm and height
16.

w.E
A cylindrical bath tub of radius 12 cm contains water to a
depth of 20 cm. A spherical iron ball is dropped into the tub
and thus the level of water is raised by 6.75 cm. What is the
radius of the ball?
4 cm.
B. A cube having each side 4 cm.
C. A cylinder of radius 3 cm and length 3 cm.
D. A sphere of radius 3 cm.
(a) 8 cm
(c) 12 cm
(b) 9 cm
(d) 7 cm asy The volumes of these in the decreasing order is :
(a) A, B, C and D (b) A, C, B and D

17.
(e) None of these

En
The volume of a cube is numerically equal to the sum of the
lengths of its edges. What is its total surface area in square 24.
(c) D, B, C and A
(e) None of these
(d) D, C, B and A

Two solid spheres of radii 1 cm and 2 cm were melted and


units ?
(a) 64 (b) 144 gin combined to form a bigger sphere. The radius of the bigger
sphere is:

18.
(c) 36
(e) None of these
(d) 72

The volume of a cube whose surface area is 726m2, is :


(a)
1
23 eer (b)
1
33
(a) 1300 m3
(c) 1452 m 3
(b) 1331 m3
(d) 1542 m3
(c)
1
83 ing (d)
93
1

19.
(e) None of these
If a cistern is 3 m long, 2 m wide and 1 m deep, its capacity in
litres is :
25.
(e) None of these

.ne
The inner diameter of a circular building is 54 cm and the
base of the wall occupies a space of 352 cm2. The thickness
(a) 6
(c) 6,000
(e) None of these
(b) 600
(d) 60,000
of the wall is :
(a) 29 cm
(c) 4 cm
(e) None of these
(b) 2 cm
(d) 58 cm
t

13. a b c d e 14. a b c d e 15. a b c d e 16. a b c d e 17. a b c d e


RESPONSE 18. a b c d e 19. a b c d e 20. a b c d e 21. a b c d e 22. a b c d e
GRID 23. a c d e 24. a c d e 25. a c d e
b b b

Downloaded From : www.EasyEngineering.net


Downloaded From : www.EasyEngineering.net

Geometry 48
Max. Marks : 25 No. of Qs. 25 Time : 20 min. Date : ........./......../. ..............
1. In triangle ABC, angle B is a right angle. If (AC) is 6 cm, and (a) 8 cm (b) 4 cm
D is the mid-point of side AC. The length of BD is (c) 5 cm (d) 7 cm
(e) None of these
A
5. If the angles of a triangle are in the ratio 5 : 3 : 2, then the
triangle could be :
D (a) obtuse (b) acute

ww B C 6.
(c) right
(e) None of these
(d) isosceles

In the figure, AB = 8, BC = 7 m, ÐABC = 1200. Find AC.

(a) 4 cm
(c) 3 cm
w.E (b) 6cm
(d) 3.5 cm
A

2.
(e) None of these
asy
AB is diameter of the circle and the points C and D are on M
8
B
1200
C
the circumference such that ÐCAD = 30°. What is the
measure of ÐACD ?
En (a) 11
(c) 13
(b) 12
(d) 14

D
C
7.
gin (e) None of these
The perimeters of two similar triangles ABC and PQR are
36 cm, and 24 cm, respectively. If PQ = 10 cm, then the
A
70°
B
(a) 16 cm eer
length of AB is :
(b) 12 cm

(a) 40° (b) 50°


8.
(c) 14 cm
(e) None of these
ing
(d) 15 cm

Two isosceles triangles have equal vertical angles and their

3.
(c) 30°
(e) None of these
(d) 90°

The sum of the interior angles of a polygon is 1620°. The


heights is : .ne
areas are in the ratio 9 : 16. The ratio of their corresponding

number of sides of the polygon are :


(a) 9
(c) 15
(e) None of these
(b) 11
(d) 12
9.
(a) 3 : 4
(c) 2 : 1
(e) None of these
(b) 4 : 3
(d) 1 : 2

The circumcentre of a triangle is always the point of


t
intersection of the :
4. A point P is 13 cm. from the centre of a circle. The length of
(a) medians
the tangent drawn from P to the circle is 12cm. Find the
(b) angle bisectors
radius of the circle.
(c) perpendicular bisectors of sides
T (d) perpendiculars dropped from the vertices on the
12c opposite sides of the triangle.
r m.
(e) None of these
O 13cm. P

RESPONSE 1. a b c d e 2. a b c d e 3. a b c d e 4. a b c d e 5. a b c d e

GRID 6. a b c d e 7. a b c d e 8. a b c d e 9. a b c d e

Downloaded From : www.EasyEngineering.net


Downloaded From : www.EasyEngineering.net

98 SPEED TEST 48
10. In a triangle ABC, Ð A = x , ÐB = y and ÐC = y + 20 . 18. Number of degrees in two and half right angles is
__________ .
If 4x – y = 10, then the triangle is :
(a) 245 (b) 225
(a) Right-angled (b) Obtuse-angled
(c) 200 (d) 180
(c) Equilateral (d) Cannot be determined
(e) None of these
(e) None of these
19. Number of degrees in five and two-third of a right angle is
11. If the sides of a right triangle are x, x + 1 and x – 1, then its
(a) 510 (b) 490
hypotenuse is :
(c) 486 (d) 480
(a) 5 (b) 4
(e) None of these
(c) 1 (d) 0
20. An angle is two-third of its complement and one-fourth of
(e) None of these
its supplement, then the angle is
12. Two circles touch each other internally. Their radii are 2 cm
(a) 46º (b) 56º
and 3 cm. The biggest chord of the outer circle which is
(c) 36º (d) 40º
outside the inner circle is of length
(e) None of these
(a) 2 2 cm (b) 3 2 cm 21. The measures of the four angles of a quadrilateral are in the
(c) 2 3 cm (d) ratio of 1 : 2 : 3 : 4.
4 2 cm

13.
ww
(e) None of these

An angle is equal to
1
rd of its supplement. Find its measure.
What is the measure of fourth angle?
(a) 144º
(c) 125º
(b) 135º
(d) 150º

(a) 60°
(c) 90°
w.E
3
(b) 80°
(d) 45°
22.
(e) None of these
Find the length of a chord which is at a distance of 3 cm from
the centre of a circle of radius 5 cm
(a) 8 cm (b) 10 cm
14.
(e) None of these
An angle measuring 270° is an example of
(a) acute angle (d) obtuse angle asy (c) 12 cm
(e) None of these
(d) 6 cm

(c) right angle


(e) None of these
(d) reflex angle
En 23. Two adjacent angles of a parallelogram are in the ratio 2 : 3.
Find the measures of all angles
(a) 72o, 108o, 72o, 108o
15. Find the angle which is equal to its supplementary angle
(a) 45°
(c) 90°
(b) 180°
(d) 360° gin (b) 72o, 108o, 70o, 110o
(c) 80o, 100o, 80o, 100o

16.
(e) None of these
Three angles of a quadrilaterals are in the ratio
24. eer
(d) Cannot be determined
(e) None of these
If the angle of triangle are in the ratio of 4 : 3 : 2, then the
1 : 2 : 3. The sum of the least and the greatest of these angles
is equal to 180°. All the angles of the quadrilateral will be :
(a) 30°, 90°, 45°, 60°
triangle
ing
(a) is obtuse angled triangle
(b) 45°, 90°, 135°, 90°
(c) 120°, 150°, 210°, 360°
(b) has one angle greater than 80°
(c) is a right triangle
(d) is acute angled triangle .ne
17.
(d) Cannot be determined
(e) None of these
The number of degrees in four and one-third right angles is
(a) 405 (b) 390
25.
(e) None of these
t
One of the angles of a parallelogram is 45°. What will be the
sum of the larger angle and twice the smaller angle of the
parallelogram ?
(c) 395 (d) 400
(e) None of these (a) 228° (b) 224°
(c) 225° (d) 222°
(e) None of these

10. a b c d e 11. a b c d e 12. a b c d e 13. a b c d e 14. a b c d e

RESPONSE 15. a b c d e 16. a b c d e 17. a b c d e 18. a b c d e 19. a b c d e

GRID 20. a b c d e 21. a b c d e 22. a b c d e 23. a b c d e 24. a b c d e

25. a b c d e

Downloaded From : www.EasyEngineering.net


Downloaded From : www.EasyEngineering.net

Number Series - I 49
Max. Marks : 30 No. of Qs. 30 Time : 20 min. Date : ........./......../. ..............

DIRECTIONS (Qs. 1-5): What should come in place of question 9. 9 49 201 1009 ? 20209 80841
mark (? ) in the following number series ? (a) 4054 (b) 4049
1. 121 117 108 92 67 ? (c) 4050 (d) 4041
(a) 31 (b) 29
(c) 41 (d) 37 (e) None of these

2.
ww
(e) None of these
50 26 14 ? 5 3.5
(a) 6 (b) 8
10. 31
(a) 121
(c) 109
35 44 60 85 ?
(b) 111
(d) 97

3.
(c) 10
(e) None of these
3 23 43 ? 83 103 w.E (d) 12
(e) None of these
DIRECTIONS (Q.11-15) : What will come in place of the question
(a) 33
(c) 63
(e) None of these
(b) 53
(d) 73
asy mark (?) in the following number series?
11. 7 9 12 16 ?
4. 748 737 715 682 638 ?
(a) 594 (b) 572 En (a) 2 2
(c) 20
(e) None of these
(b) 19
(d) 21

5.
(c) 581
(e) None of these
1 9 25 49 81 ? 169
(d) 563
gin
12. 384 192 96 48 ?
(a) 36 (b) 28
(a) 100
(c) 81
(b) 64
(d) 121
(c) 24
(e) None of theseeer (d) 32

(e) None of these


DIRECTIONS (Qs. 6-10) : What should come in place of question
13. 5 6 14 45 ?
(a) 183
ing
(b) 185
mark (?) in the following number series?
6. 36 20 ? 8 6 5
(c) 138
(e) None of these
(d) 139

.ne
7.
(a) 10
(c) 14
(e) None of these
668 656 632 584 ? 296
(b) 12
(d) 16
14. 8 9 13 22 ?
(a) 30
(c) 34
(e) None of these
(b) 31
(d) 36 t
(a) 392 (b) 438 15. 6 11 21 41 ?
(c) 488 (d) 536 (a) 81 (b) 61
(e) None of these (c) 71 (d) 91
8. 1 121 441 961 1681 ? (e) None of these
(a) 2701 (b) 2511
(c) 2611 (d) 2801
(e) None of these

1. a b c d e 2. a b c d e 3. a b c d e 4. a b c d e 5. a b c d e
RESPONSE 6. a b c d e 7. a b c d e 8. a b c d e 9. a b c d e 10. a b c d e
GRID 11. a c d e 12. a c d e 13. a c d e 14. a c d e 15. a c d e
b b b b b

Downloaded From : www.EasyEngineering.net


Downloaded From : www.EasyEngineering.net

100 SPEED TEST 49


DIRECTIONS (Qs. 16-20): What will come in place of question 24. 4 2 3.5 7.5 26.25 118.125
mark (?) in the following number series? (a) 118.125 (b) 26.25
16. 7 13 25 49 ? (c) 3.5 (d) 2
(a) 99 (b) 97 (e) 7.5
(c) 89 (d) 87 25. 16 4 2 1.5 1.75 1.875
(e) None of these
(a) 1.875 (b) 1.75
17. 5 6 10 19 ?
(c) 1.5 (d) 2
(a) 28 (b) 37
(c) 36 (d) 35 (e) 4
(e) None of these DIRECTIONS (Q. 26-30) : In the following number series only
18. 8 9 20 63 ? one number is wrong. Find out the wrong number.
(a) 256 (b) 252 26. 4 6 18 49 201 1011
(c) 246 (d) 242 (a) 1011 (b) 201
(e) None of these
(c) 18 (d) 49
19. 11 13 16 20 ?
(a) 24 (b) 26 (e) None of these
(c) 28
ww
(e) None of these
(d) 27 27. 48 72 108 162 243 366
(a) 72 (b) 108
20. 608 304 152 76 ?
(a) 39
(c) 38
(e) None of these
w.E (b) 36
(d) 37
(c) 162
(e) None of these
(d) 243

asy
DIRECTIONS (Qs. 21-25) % In the following number series, a wrong
number is given. Find out that wrong number.
28. 2 54 300 1220 3674 7350
(a) 3674 (b) 1220

21. 2 11 38 197 1172 8227 65806


En (c) 300
(e) None of these
(d) 54

(a) 11
(c) 197
(e) 8227
(b) 38
(d) 1172
gin
29. 8 27 64 125 218 343
(a) 27 (b) 218

22. 16
(a)
19 21 30 46 71 107
19 (b) 21
(c) 125
eer
(e) None of these
(d) 343

(c)
(e)
30
71
(d) 46 30. 19 68 102 129 145 154
ing
23. 7 9
(a)
16 25 41 68 107 173
107 (b) 16
(a) 154
(c) 145
(b) 129
(d) 102
.ne
(c)
(e)
41
25
(d) 68
(e) None of these
t

16. a b c d e 17. a b c d e 18. a b c d e 19. a b c d e 20. a b c d e


RESPONSE 21. a b c d e 22. a b c d e 23. a b c d e 24. a b c d e 25. a b c d e
GRID 26. a c d e 27. a c d e 28. a c d e 29. a c d e 30. a c d e
b b b b b

Downloaded From : www.EasyEngineering.net


Downloaded From : www.EasyEngineering.net

Number Series - II 5
500
Max. Marks : 30 No. of Qs. 30 Time : 20 min. Date : ........./......../. ..............

DIRECTIONS (Qs.1-5) : What should come in place of the 8. 101 103 99 105 97 ? 95
question mark (?) in the following number series ? (a) 93 (b) 104
(c) 108 (d) 107
1. 353 354 351 356 349 ?
(a) 348 (b) 358 (e) None of these
(c) 338 (d) 385 9. 3 219 344 408 ? 443 444
(e) 340

(a) 83
(c) 61
ww
2. 1 5 13 29 ? 125 253
(b) 69
(d) 65
(a) 416
(c) 423
(b) 435
(d) 428

(e) 81
3. 45 57 81 117 165 ?
(a) 235
w.E
(b) 215
10. 7
(a)
(e) None of these
10 16 28 52 ? 196
100 (b) 90
(c) 205
(e) 225
4. 17 18 26 53 117 ? 458
(d) 245
asy (c)
(e)
160
None of these
(d) 150

(a) 342
(c) 257
(b) 142
(d) 262 En DIRECTIONS (Qs. 11-15) : What should come in place of the
question mark (?) in the following number series ?

5.
(e) 242
1 1 3 1 1 3
11 1 1 ? gin
11. 1 1 2 6 ? 120
(a) 24 (b) 60
4 2 4 4 2 4
(a) 2 (b) 4
(c) 100

eer
(e) None of these
12. 7 8 16 43 ? 232
(d) 30

(c) 6 (d) 1
1
5
(a) 204
(c) 119 ing
(b) 107
(d) 89
(e) 1
2
3
(e) None of these
13. 4 13 17 ? 30 39 .ne
DIRECTIONS (Qs.6-10) : What should come in place of the
question mark (?) in the following number series?
6. 4 19 49 94 154 ?
(a) 223 (b) 225
(a) 29
(c) 26
(e) None of these
14. 982 977 952 827 822 ?
(b) 21
(d) 19
t
(c) 229 (d) 239
(a) 779 (b) 817
(e) None of these
(c) 789 (d) 697
1 1 1
7. 1 1 2 2 3 ? (e) None of these
2 2 2 15. 41472 5184 576 72 8 ?
1 1
(a) 3 (b) 2 (a) 0 (b) 9
2 3
(c) 1 (d) 8
1
(c) 4 (d) 3 (e) None of these
4
(e) None of these

1. a b c d e 2. a b c d e 3. a b c d e 4. a b c d e 5. a b c d e
RESPONSE 6. a b c d e 7. a b c d e 8. a b c d e 9. a b c d e 10. a b c d e
GRID 11. a c d e 12. a c d e 13. a c d e 14. a c d e 15. a c d e
b b b b b

Downloaded From : www.EasyEngineering.net


Downloaded From : www.EasyEngineering.net

102 SPEED TEST 50


DIRECTIONS (Qs. 16-20) : What should come in place of the 23. 2 8 12 20 30 42 56
question mark (?) in the following number series? (a) 8 (b) 42 (c) 30
16. 64 54 69 49 74 44 ? (d) 20 (e) 12
(a) 89 (b) 69 24. 32 16 24 65 210 945 5197.5
(c) 59 (d) 99 (a) 945 (b) 16 (c) 24
(d) 210 (e) 65
(e) None of these
25. 7 13 25 49 97 194 385
17. 4000 2008 1012 ? 265 140.5 78.25
(a) 13 (b) 49 (c) 97
(a) 506 (b) 514
(d) 194 (e) 25
(c) 520 (d) 512
(e) None of these DIRECTIONS (Qs. 26 - 30) :In each of these questions, a number
series is given. In each series, only one number is wrong number.
18. 5 5 15 75 ? 4725 51975
Find out the wrong number.
(a) 520 (b) 450
26. 16 19 21 30 46 71 107
(c) 525 (d) 300
(a) 19 (b) 21 (c) 30

19.
(e)
52
(a)
ww
None of these
26 26 39 78 ? 585
195 (b) 156
27.
(d) 46 (e) 71
7 9 16 25 41 68 107 173

(c) 234
(e) None of these w.E
(d) 117
28.
(a) 107
(d) 68
(b) 16
(e) 25
32 16 24 65 210 945 5197.5
(c) 41

20. 29, 23, ?, 17, 13, 11, 7


(a) 19
(c) 23
(b) 21
(d) 27
asy (a) 945
(d) 210
(b) 16
(e) 65
(c) 24

(e) None of these


En 29. 850 600 550 500 475 462.5 456.25
(a) 600 (b) 550 (c) 500
DIRECTIONS (Qs. 21 to 25): In each of these questions, a number
series is given. In each series, only one number is wrong. Find out
the wrong number. gin
30.
(d) 4625
8 12 24 46 72 108 216
(a) 12
(e) None of these

(b) 24 (c) 46
21. 3601 3602 1803 604 154 36 12
(a) 3602 (b) 1803 (c) 604
(d) 72
eer (e) None of these

22.
(d) 154 (e) 36
4 12 42 196 1005 6066 42511 ing
(a) 12
(d) 196
(b) 42
(e) 6066
(c) 1005
.ne
t

16. a b c d e 17. a b c d e 18. a b c d e 19. a b c d e 20. a b c d e


RESPONSE 21. a b c d e 22. a b c d e 23. a b c d e 24. a b c d e 25. a b c d e
GRID 26. a c d e 27. a c d e 28. a c d e 29. a c d e 30. a c d e
b b b b b

Downloaded From : www.EasyEngineering.net


Downloaded From : www.EasyEngineering.net

Data Interpretation 51
Max. Marks : 20 No. of Qs. 20 Time : 20 min. Date : ........./......../. ..............

DIRECTIONS (Q. 1-5): Study the following graph carefully to answer these questions.
Quantity of Various Items Sold and Price per kg

30 60

25 50

ww 20 40
Price

Price in ` per kg
15 30 Quantity sold in quintals

w.E 10

5
20 Quantity

10

0
A
asy
B C D
Items
E F
0

1. If the quantity sold of item D increased by 50% and the


En DIRECTIONS (Qs. 6 - 10): Study the following graph carefully
price reduced by 10%. What was the total value of the
quantity sold for item D ?
(a) ` 675 (b) ` 6750 (c) `67550 gin
and answer the questions given below it.
Number of Students Studying in Various Colleges from
Various Faculties (Number in Thousands)
2.
(d) ` 67500 (e) None of these
Approximately, what is the average price per kg of items A,
B& C?
80 eer
(a) ` 9.50 (b) ` 8 (c) ` 7.50
70
60
65

ing 60 56
Number of Students

(d) ` 9 (e) ` 10.50 51.2 50

.ne
50 Arts
3. What is the ratio between the total values of quantity sold 40
44
for items E & F respectively ? 40 36.5 Science
33
30 30 Commerce
(a) 15 : 14 (b) 3 : 2 (c) 5 : 7 30

t
25
(d) 7 : 5 (e) None of these 20
4. Total value of the quantity sold for item C is what per cent 10
of the total value of the quantity sold for item E ? 0
H I J K
(a) 111 (b) 85 (c) 90 Colleges
(d) 87.5 (e) None of these
5. If the price as well as the quantity sold is increased by 20% 6. What is the difference between the total number of students
for item A, what is the total value of quantity sold for item studying in college H and those studying in college K ?
A? (a) 16100 (b) 15800 (c) 16300
(a) ` 48500 (b) ` 49000 (c) ` 42000 (d) 16700 (e) None of these
(d) ` 50400 (e) None of these 7. What is the total number of students studying in all the
colleges together ?
(a) 520900 (b) 520700 (c) 610200
(d) 510800 (e) None of these

1. a b c d e 2. a b c d e 3. a b c d e 4. a b c d e 5. a b c d e
RESPONSE 6. a b c d e 7. a b c d e
GRID

Downloaded From : www.EasyEngineering.net


Downloaded From : www.EasyEngineering.net

104 SPEED TEST 51


8. What is the respective ratio of the students from the faculty (a) 634 (b) 654 (c) 658
of Science from colleges H and I together to the students (d) 778 (e) None of these
from the same faculty from colleges J and K together ? 14. What is the approximate average number of teachers teaching
(a) 43 : 45 (b) 41 : 43 (c) 45 : 43 Economics, History and Biology together ?
(d) 43 : 41 (e) None of these (a) 400 (b) 420 (c) 450
9. The number of students from the faculty of Science from (d) 480 (e) 470
college I are approximately what per cent of the total number 15. What is the respective ratio of the number of teachers who
of students studying in that college ? teach Biology and the number of teachers who teach Physics?
(a) 34 (b) 37 (c) 29 (a) 6 : 7 (b) 4 : 7 (c) 3 : 5
(d) 31 (e) 39 (d) 4 : 5 (e) None of these
10. What is the average number of students from the faculty of
Commerce from all the colleges together ? DIRECTIONS (Q. 16-20) : Study the following table carefully to
(a) 36825 (b) 38655 (c) 35625 answer the questions that follow.
(d) 36585 (e) None of these Number of flights cancelled by five different
airlines in six different years
DIRECTIONS (Qs. 11-15) : Study the following Pie-chart carefully
to answer these questions.
Percentagewise Distribution of Teachers who Teach six Different Airline P Q R S T
Subjects
wwTotal Number of Teachers = 2000
Percentage of Teachers
Year
2005
2006
240
420
450
600
305
470
365
446
640
258

w.E
Biology
12%
English
7%
2007
2008
2009
600
160
140
680
208
640
546
708
656
430
550
250
610
586
654

asy 16.
2010 290 363 880 195

What was the difference between the highest number of


483

Economics
25%
History
27%
En flights cancelled by airline - Q and the lowest number of
flights cancelled by airline-T out of all the six years ?

gin (a) 446


(d) 442
(b) 456
(e) None of these
(c) 432

Physics Mathematics
17.

eer
What was the approximate percentage increase in number
of flights cancelled by airline-S in the year 2008 as compared
to previous year ?

11.
15% 14%

If five-seventh of the teachers who teach Mathematics are 18.


(a) 127
(d) 45
ing
(b) 27
(e) 117
(c) 150

What was the average number of flights cancelled by the


female, then number of male Mathematics teachers is
approximately what percentage of the total number of
airlines P, R, S and T in the year 2008 ?
(a) 551.5 (b) 501 .ne (c) 405

12.
teachers who teach English ?
(a) 57
(d) 69
(b) 42
(e) 51
(c) 63

What is the difference between the total number of teachers


19.
(d) 442.4 (e) None of these
t
In 2010, 40% flights are cancelled by airline-R due to bad
weather and technical fault. How many flights are cancelled
by airline-R due to technical fault ?
who teach English and History together and the total number
(a) 528 (b) 568 (c) 468
of teachers who teach Mathematics and Biology together ?
(d) 548 (e) None of these
(a) 146 (b) 156 (c) 180
20. What is the approximate percentage of cancelled flights by
(d) 160 (e) None of these
13. If the percentage of Biology teachers is increased by 40 per airline’s-P and R in 2007 compared to cancelled flights by
cent and percentage of History teachers decreased by 20 airline-S in 2005 ?
per cent then what will be the total number of Biology and (a) 356 (b) 280 (c) 265
History teachers together ? (d) 340 (e) 314

8. a b c d e 9. a b c d e 10. a b c d e 11. a b c d e 12. a b c d e


RESPONSE 13. a b c d e 14. a b c d e 15. a b c d e 16. a b c d e 17. a b c d e
GRID 18. a b c d e 19. a b c d e 20. a b c d e

Downloaded From : www.EasyEngineering.net


Downloaded From : www.EasyEngineering.net

Section Test :
Quantitative Aptitude 52
Max. Marks : 40 No. of Qs. 40 Time : 25 min. Date : ........./......../. ..............
DIRECTIONS (Q.1-10) : What will come in place of question 11. Cost of 12 belts and 30 wallets is Rs 8940. What is the cost of
mark (?) in the following questions ? 4 belts and 10 wallets?
(a) Rs 2890 (b) Rs 2980 (c) Rs 2780
1. 48% of 525 + ? % of 350 = 399 (d) Rs 2870 (e) None of these
(a) 42 (b) 46 (c) 28 12. Ghanshyam purchased an article for Rs 1850. At what price
(d) 26 (e) None of these should he sell it so that 30% profit is earned?
2.
3 4 5
7 5 8
ww
of of of 490 = ?
(a) 115 (b) 105 (c) 108 13.
(a) Rs 2450
(d) Rs 2425
(b) Rs 2245
(e) None of the above
(c) Rs 2405

What is the compound interest accrued on an amount of Rs

3.
(d) 116
2
? + 17 = 335
(a) 46
w.E
(e) None of these

(b) 42 (c) 1764 14.


8500 in two years @ interest 10% per annum?
(a) Rs 1875
(d) Rs 1765
(b) Rs 1885
(e) None of these
(c) Rs 1775

A train running at the speed of 60 kmph crosses a 200 m long

4.
(d) 2116 (e) None of these
125% of 560 + 22% of 450 = ?
(a) 799 (b) 700
asy
(c) 782
platform in 27 s. What is the length of the train ?
(a) 250 m
(d) 450 m
(b) 200 m
(e) None of these
(c) 240 m

(d) 749
28 ´ 5 - 15 ´ 6
(e) None of these
En 15. 10 men can complete a piece of work in 8 days. In how many
days can 16 men complete that work?
5.
7 + 256 + (13)2
2
=?
gin
16.
(a) 4 days
(d) 3 days
(b) 5 days
(e) None of these
Find the average of following set of numbers.
(c) 6 days

(a)
27
115
(b)
22
117
(c)
25
117 (a) 72
eer
76, 48, 84, 66, 70, 64
(b) 66 (c) 68

6.
(d)
22
115
(e) None of these
18.76 + 222.24 + 3242.15 = ?
17.
(d) 64

ing
(e) None of these
If the numerator of a certain fractions increased by 100% and
the denominator is increased by 200%; the new fraction thus
(a) 3384.15
(d) 3383.25
(b) 3483.15
(e) None of these
(c) 3283.25
formed is
4
21 .ne
. What is the original fraction?
7. 784 ÷ 16 ÷ 7 = ?
(a) 49
(d) 7
3 5
(b) 14
(e) None of these
(c) 21 (a)

(d)
4
2
7
(b)
3
7

(e) None of these


(c)
2
5 t
8. of 455 + of 456 = ? 7
2 8
(a) 448 (b) 476 (c) 480 18. In how many different ways can the letters of the word
(d) 464 (e) None of these 'SIMPLE' be arranged?
9. 1.05% of 2500 + 2.5% of 440 = ? (a) 520 (b) 120 (c) 5040
(a) 37.50 (b) 37.25 (c) 370.25 (d) 270 (e) None of these
(d) 372.50 (e) None of these 19. The ratio of the ages of A and B seven years ago was 3 : 4
10. 4900 ÷ 28 × 444 ÷ 12 = ? respectively. The ratio of their ages nine years from now
(a) 6575 (b) 6475 (c) 6455 will be 7 : 8 respectively. What is B’s age at present ?
(d) 6745 (e) None of these (a) 16 years (b) 19 years (c) 28 years
(d) 23 years (e) None of these

1. a b c d e 2. a b c d e 3. a b c d e 4. a b c d e 5. a b c d e
RESPONSE 6. a b c d e 7. a b c d e 8. a b c d e 9. a b c d e 10. a b c d e

GRID 11. a b c d e 12. a b c d e 13. a b c d e 14. a b c d e 15. a b c d e


16. a b c d e 17. a b c d e 18. a b c d e 19. a b c d e

Downloaded From : www.EasyEngineering.net


Downloaded From : www.EasyEngineering.net

106 SPEED TEST 52


20. The sum of three consecutive odd numbers is 1383. What 31. 4 women and 12 children together take four days to complete
is the largest number ? a piece of work. How many days will four children alone take
(a) 463 (b) 49 (c) 457 to complete the piece of work if two women alone can
(d) 461 (e) None of these complete the piece of work in 16 days?
DIRECTIONS (Q. 21-25) : What should come in place of ques- (a) 32 (b) 24 (c) 16
tion mark (?) in the following number series? (d) 12 (e) None of these
32. The average of four consecutive odd number A, B, C and D
21. 8 52 ? 1287 4504.5 11261.25 16891.875 respectively is 40. What is the product of B and D?
(a) 462 (b) 286 (c) 194 (a) 1599 (b) 1591 (c) 1763
(d) 328 (e) None of these (d) 1677 (e) None of these
22. 3 42 504 ? 40320 241920 967680 33. Anu walks 2.31 km in three weeks by walking an equal
(a) 6048 (b) 5544 (c) 4536 distance each day. How many metres does she walk each
(d) 5040 (e) None of these day?
23 403 400 394 382 358 310 ? (a) 110 m (b) 90 m (c) 140 m
(a) 244 (b) 210 (c) 214 (d) 120 m (e) None of these
(d) 256 (e) None of these 34. A man riding a bicycle completes one lap of a square field
24. 7 8 4 13 –3 22 ? along its perimeter at the speed of 43.2 km/hr in 1 minute 20
(a) –7 (b) –10 (c) –12
(d) –14
ww (e) None of these
25. 250000 62500 12500 3125 625 ? 31.25
(a) 156.25 (b) 172.25 (c) 125
seconds. What is the area of the field?
(a) 52900 sq m
(d) Can’t be determined
(e) None of these
(b) 57600 sq m (c) 48400 sq m

26.
(d) 150

Out of the fractions


w.E
(e) None of these
1 7 3 5 6
, , , and , what is the differ-
35. On Teacher’s Day, 4800 sweets were to be equally distributed
among a certain number of children. But on that particular
day 100 children were absent. Hence, each child got four

7
2 8 4 6

3
7

asy
ence between the largest and the smallest fraction?
4
sweets extra. How many children were originally supposed
to be there?
(a) 300 (b) 400 (c) 540
(a)
13
(b)
8
(c)
7
En 36.
(d) 500 (e) Can’t be determined.
The ratio of the monthly oncomes of Sneha, Tina and Akruti

27.
(d)
1
6
(e) None of these
What will come in place of both questions marks(?) in the gin is 95:110:116. If Sneha’s annual income is `3,42,000, what is
Akruit’s annual income?
(a) `3,96,900 (b) `5,63,500 (c) `4,17,600
following equation?
(?)0.6 26
37.
eer
(d) `3,88,000 (e) None of these
A truck covers a distance of 256 km at the speed of 32 km/hr.
What is the average speed of a car which travels a distance
104
(a) 58
=
(?)1.4
(b) –48 (c) –56
(a) 46 kmh–1 ing
of 160 km more than the truck in the same time?
(b) 52 kmh–1 (c) 49 kmh–1

.ne
(d) 64 kmh –1 (e) None of these
(d) 42 (e) –52 38. In an examination, the maximum aggregate marks is 1020. In
28. The perimeter of a square is thrice the perimeter of a rectange. order to pass the exam a student is required to obtain 663
If the perimeter of the square is 84 cm and the length of the
rectangel is 8 cm, what is the difference between the breadth
of the rectangle and the sidce of the square?
(a) 15 cm
(d) 8 cm
(b) 19 cm
(e) None of these
(c) 10 cm
marks out of the aggregate marks. Shreya obtained 612
marks. By what per cent did Shreya fail the exam?
(a) 5%
(d) Can’t be determined
(b) 8% (c) 7% t
(e) None of these
29. The area of a circle is equal to the area of a rectangel with 39. The average height of 21 girls was recorded as 148 cm. When
perimeter equal to 42 m and breadth equal to 8.5 m. What is the teacher’s height was included, the average of their
the area of the circle? heights increased by 1 cm. What was the height of the
(a) 116.25 sq m (b) 104.25 sq m (c) 146.25 sq m teacher?
(d) 128.25 sq m (e) None of these (a) 156 cm (b) 168 cm (c) 170 cm
30. The product of 5% of a positive number and 3% of the same (d) 162 cm (e) None of these
number is 504.6 What is half of that number? 40. What would be the area of a circle whose diameter is 35 cm?
(a) 290 (b) 340 (c) 680 (a) 962.5 sq cm (b) 875.5 sq cm (c) 981.5 sq cm
(d) 580 (e) None of these (d) 886.5 sq cm (e) None of these
20. a b c d e 21. a b c d e 22. a b c d e 23. a b c d e 24. a b c d e
25. a b c d e 26. a b c d e 27. a b c d e 28. a b c d e 29. a b c d e
RESPONSE 30. a b c d e 31. a b c d e 32. a b c d e 33. a b c d e 34. a b c d e
GRID 35. a b c d e 36. a b c d e 37. a b c d e 38. a b c d e 39. a b c d e
40. a b c d e

Downloaded From : www.EasyEngineering.net


Downloaded From : www.EasyEngineering.net

Reading
Comprehension - I 53
Max. Marks : 30 No. of Qs. 30 Time : 20 min. Date : ........./......../. ..............
Gandhi. He has rightly been called the Father of the Nation because
DIRECTIONS : Read the following passages carefully and answer
it was he who awakened in the people of this country a sense of
the questions given below it. Certain words are printed in bold to
national consciousness and instilled in them a high sense of
help you to locate them while answering some of the questions.
patriotism without which it is not possible to build a country into
Passage 1 nationhood. By the time the Constitution of India came to be enacted,
We find that today the unity and integrity of the nation is insurgent India, breaking a new path of non-violent revolution and

ww
threatened by the divisive forces of regionalism, linguism and
communal loyalties which are gaining ascendancy in national
life and seeking to tear apart and destroy national integrity. We
fighting to free itself from the shackles of foreign domination, had
emerged into nationhood and “the people of India” were inspired by
a new enthusiasm, a high and noble spirit of sacrifice and above all,

w.E
tend to forget that India is one nation and we are all Indians first
and Indians last. It is time we remind ourselves what the great
visionary and builder of modern India Jawaharlal Nehru said,
“Who dies if India lives, who lives if India dies?” We must realise,
a strong sense of nationalism and in the Constitution which they
framed. They set about the task of a strong nation based on certain
cherished values for which they had fought.
1. The author has quoted Jawaharlal Nehru to emphasise the

asy
and this is unfortunately what many in public life tend to overlook,
sometimes out of ignorance of the forces of history and
sometimes deliberately with a view to promoting their self-
point that (a) national interest must enjoy supreme im-
portance (b) India is going to survive even if the world is
under the

En
interest, that national interest must inevitably and forever prevail
over any other considerations proceeding from regional, linguistic
spell of destruction (c) the world will be destroyed if
India is on the threshold of
or communal attachments. The history of India over the past
centuries bears witness to the fact that India was at no time a
single political unit. Even during the reign of the Maurya dynasty, gin destruction (d) the survival of the world depends
only upon the well

though a large part of the country was under the sovereignty of


the Mauryan kings, there were considerable portions of the 2. eer
being of India
(e) None of these
What, according to the author, is the impact of the divisive
territory which were under the rule of independent kingdoms. So
also during the Mughal rule which extended over large parts of
the territory of India, there were independent rulers who enjoyed
forces on our nation?
ing
(a) They promote a sense of regional pride.
(b) They help people to form linguistic groups.
political sovereignty over the territories of their respective
kingdoms. It is an interesting fact of history that India was forged
into a nation, neither on account of a common language nor on
them. .ne
(c) They separate groups of people and create enmity among

account of the continued existence of a single political regime


over its territories but on account of a common culture evolved
over the centuries. It is cultural unity—something more
fundamental and enduring than any other bond which may unite 3.
(d) They encourage among people the sense of loyalty to
their community.
(e) They remind us of our national pride. t
“Communal loyalties” have been considered by the author as
the people of a country together which has welded this country (a) a good quality to be cherished (b) of no
into a nation. But until the advent of the British rule, it was not consequence to the nation (c) a very important aspect
constituted into a single political unit. There were, throughout for nation-building (d) a threat to the solidarity of the
the period of history for which we have fairly authenticated nation (e) None of these
accounts, various kingdoms and principalities which were 4. Which of the following was instrumental in holding the
occasionally engaged in conflict with one another. During the different people of India together?
British rule, India became a compact political unit having one (a) A common national language
single political regime over its entire territories and this led to the (b) A common cultural heritage
evolution of the concept of a nation. This concept of one nation (c) The endurance level of the people
took firm roots in the minds and hearts of the people during the (d) Fundamentalist bent of mind of the people
struggle for independence under the leadership of Mahatma (e) None of these

RESPONSE 1. a b c d e 2. a b c d e 3. a b c d e 4. a b c d e
GRID

Downloaded From : www.EasyEngineering.net


Downloaded From : www.EasyEngineering.net

108 SPEED TEST 53


5. The passage appears to have been written with the purpose of DIRECTIONS (Qs. 11-13): Choose the word/group of words which
(a) giving a piece of advice to politicians of free India is most nearly the SAME in meaning as the word given in bold as
(b) assessing the patriotic values and sacrifices made by used in the passage.
people for India’s freedom 11. awakened
(c) justifying the teaching of Mahatma Gandhi and its (a) moved (b) segregated
impact on the people (c) extracted (d) kindled
(d) giving a historical account of how India evolved as a (e) supported
nation 12. cherished
(e) None of these (a) maintained carefully
6. History shows that India, which was not a political unit (b) available abundantly
earlier, became so (c) managed tactfully
(a) during the reign of Maurya dynasty (d) accepted happily
(b) during the Mughal rule (e) protected lovingly
(c) after one-national-language policy was adopted 13. authenticated
(d) during the regime of independent rulers (a) established (b) documented
(e) during the British rule (c) hea rsay (d) audited
7.
ww
Why do people tend to overlook the paramount importance
of national interest?
(A) Because they are unaware of the imperative need of
(e) maintained
DIRECTIONS (Qs. 14 & 15) : Choose the word which is most
OPPOSITE in meaning of the word given in bold as used in the
the day

motives
w.E
(B) Because they give undue importance to their selfish
passage.
14. considerable
(a) inconsiderate
(c) unfathomable
(b) uncountable
(d) irresolute
(a) Only A
(c) Only C
(b) Only B
(d) A and B only asy
(C) Because historical events force them to do so
(e) negligible
15. deliberately

8.
(e) B and C only
En
The “people of India”, as highlighted by the author in the last
(a) reluctantly
(c) unauthorisedly
(d) notoriously
(b) unintentionally
(d) wrongly
sentence of the passage, refer to
(a) the people of one unified nation
(b) the subjects of several independent rulers
gin Passage - 2
In a reversal of the norm elsewhere, in India policymakers and
(c) the patriots who sacrificed themselves in the freedom
struggle eer
economists have become optimists while bosses do the worrying.
The country’s Central Bank has predicted that the country’s

ing
economy is likely to grow at a double digit rate during the next 20-
(d) the people who were instrumental in writing the
30 years. India has the capability with its vast labour and lauded
Constitution entrepreneurial spirit. But the private sector which is supposed to
(e) None of these
9. India’s insurgence was for
(a) breaking the path of non-violence .ne
do the heavy lifting that turns India from the world’s tenth largest
economy to its third largest by 2030 has become fed up. Business
people often carp about India’s problems but their irritation this
(b) having one common national language
(c) insisting on a unique cultural identity
(d) several independent sovereign rulers
(e) None of these
The economy may be slowing naturally as the low interest ratest
time has a nervous edge. In the first quarter of 2011, GDP grew at
an annual rate of 7.8 percent; in 2005-07 it managed 9-10 percent.

and public spending that got India through the global crisis are
10. Transformation of our country into nationhood was possible belatedly withdrawn. At the same time the surge in inflation caused
by exorbitant food prices has spread more widely, casting doubt
because of
over whether India can grow at 8-10 percent in the medium term
(A) People’s spontaneously referring to Mahatma Gandhi without overheating.
as the Father of the Nation In India, as in many fast growing nations, the confidence to invest
(B) People’s sense of national consciousness depends on the conviction that the long term trajectory is intact
(C) Generation of a high sense of dedication to the nation and it is that which is in doubt. Big Indian firms too sometimes
among the people seem happier to invest abroad than at home, in deals that are
(a) A and B only (b) A and C only often hailed as symbols of the country’s growing clout but
(c) B and C only (d) All the three sometimes speak to its weaknesses – purchases of natural
(e) None of these resources that India has in abundance but struggles to get out of
the ground. In fact a further dip in investment could be self

5. a b c d e 6. a b c d e 7. a b c d e 8. a b c d e 9. a b c d e
RESPONSE 10. a b c d e 11. a b c d e 12. a b c d e 13. a b c d e 14. a b c d e
GRID 15. a b c d e

Downloaded From : www.EasyEngineering.net


Downloaded From : www.EasyEngineering.net

SPEED TEST 53 109


fulfilling: if fewer roads, ports and factories are built, this will hurt 19. What is the author’s main objective in writing the passage?
both short term growth figures and reduce the economy’s long (a) Showcasing the potential of India’s growth potential to
term capacity. entice foreign investors .
There is a view that because a fair amount of growth is assured (b) Exhorting India to implement measures to live up to its
the government need not try very hard. The liberalization reforms potential.
that began in 1991 freed markets for products and gave rise to (c) Recommending India’s model of development to other
vibrant competition, at the same time what economists call factor developing countries
markets, those for basic inputs like land, power, labour etc remain (d) Berating the private sector for not bidding for
unreformed and largely under state control, which creates infrastructure development projects.
difficulties. Clearances today can take three to four years and (e) Criticising the measures taken by India during the global
many employers are keen to replace workers with machines economic crisis.
20. What impact has the GDP growth of 7.8 percent had?
despite an abundance of labor force. This can be attributed to
(1) Indian Industry is anxious about India’s economic
labor laws which are inimical to employee creation and an
growth.
education system that means finding quality manpower a major
(2) India has achieved status as the world’s third largest
problem. In fact the Planning Commission concluded that even economy at present.
achieving 9 percent growth will need marked policy action in (3) Foreign investment in India has drastically increased.

ww
unreformed sectors. Twenty years age it was said that yardstick
against which India should be measured was its potential and it
is clear that there remains much to do.
(a) Only (1)
(b) All (1), (2) and (3)
(c) Only (1) and (3)

economy at present?
w.E
16. Which of the following can be said about the Indian

(a) It can comfortably achieve double digit growth rare at


(d) Only (1) and (2)
(e) None of these
21. Which of the following is most similar in meaning to the
word CLOUT given in bold as used in the passage?
present.

asy
(b) High food prices have led to overheating of the
economy.
(a) Strike
(b) Standing
(c) Citizens are affluent owing to laxity in regulation.
(d) Private sector confidence in India’s growth potential
En (c) Force
(d) Launch
(e) Achieve
is high.
(e) Unreformed sectors are a drag on economic growth.
17. Why are employers reluctant to hire Indian labour force? gin
22. Which of the following is most opposite in meaning to the
word MARKED given in bold as used in the passage?
(1) India’s labour force is overqualified for the
employment opportunities available.
(b) Ignored
(c) Clear eer
(a) Decreased

(2) High attrition rate among employees stemming from


their entrepreneurial spirit.
(3) Labour laws are not conducive to generating
(d) Assessed
(e) Imperceptible
ing
23. What measures do experts suggest be taken to ensure targeted
emploment.
(a) Only (3)
economic growth?
.ne
(a) Loweing of interest rates to help industries hit by
(b) All (1), (2) and (3)
(c) Only (1) and (3)
(d) Only (1) and (2)
(e) None of these
recession.

t
(b) Prolonged financial support for basic input industries.
(c) Incentives to Indian companies to invest in infrastucture.
(d) Formulation of policies and their implementation in factor
markets
18. What is the state of India’s basic input sectors at present?
(a) These sectors attract Foreign Direct Investment (e) Stringent implementation of licensing system.
because of their vast potential. Passage - 3
(b) These sectors are lagging as projects are usually In many countries, a combustible mixture of authoritarianism,
awarded to foreign companies. unemployment and youth has given rise to disaffection with
(c) These sectors are stagnating and badly in need of strongmen rulers which has in turn spill over into uprising. Young
reforms. people in these countries are far better educated than their parents
(d) These sectors are well regulated as these are governed were. In 1990 the average Egyptian had 4.4 years of schooling; by
by the State. 2010 the figure had risen to 7.1 years. Could it be that education, by
(e) None of these making people less willing to put up with restrictions on freedom

RESPONSE 16. a b c d e 17. a b c d e 18. a b c d e 19. a b c d e 20. a b c d e

GRID 21. a b c d e 22. a b c d e 23. a b c d e

Downloaded From : www.EasyEngineering.net


Downloaded From : www.EasyEngineering.net

110 SPEED TEST 53


and more willing to question authority, promotes democratization. 25. Which of the following is most similar in meaning to the
Ideas about the links between education, Income and democracy word PROMOTES given in bold as used in the passage?
are at the heart of what social scientists have long studied. Since (a) Up grades (b) Prefers
then plenty of economists and political scientists have looked for (c) Recommends (d) Advocates
statistical evidence of a causal link between education and (e) Publicizes
democratization. Many have pointed to the strong correlation that 26. What conclusion can be drawn from the statistics cited about
exists between levels of education and measures like the pluralism Egypt’s education system?
of party politics and the existence of civil liberties. The patterns (a) Job prospects have been on the rise in Egypt in recent
are similar when income and democracy are considered. There are times.
outliers, of course – until recently, many Arab countries managed to (b) Authoritarian leaders have played a vital role in
reforming Egypt’s education system.
combine energy-based wealth and decent education with
(c) Egypt has one of the youngest and best educated
undemocratic political systems. But some deduce from the overall
demographies in the world.
picture that as China and other authoritarian states get more educated (d) Egypt is likely to be successful vibrant democracy.
and richer, their people will agitate for greater political freedom, (e) There has been a rise in education levels in Egypt in
culminating in a shift to a more democratic form of government. recent times.
This apparently reasonable intuition is shakier than it seems. Critics 27. In the context of the passage which of the following
of the hypothesis point out that correlation is hardly causation. characterize (s) democracies?

ww
The general trend over the past half century may have been towards
rising living standards, a wider spread of basic education and more
democracy, but it is entirely possible that this is being by another
(1) Active participation of majority of educated citizens in
electoral process.
(2) Fast paced economic growth and accountability of those

w.E
variable. Even if the correlation were not spurious, it would be difficult
to know which way causation ran. Does more education lead to
greater democracy? Or are more democratic countries better at
educating their citizens? A recent NBER paper compared a group of
in power.
(3) Better standards of living and access to higher education.
(a) All (1), (2) and (3)
(c) Only (3)
(b) Only (2) and (3)
(d) Only (1) and (2)

asy
Kenyan girls in 69 primary school whose students were randomly
selected to receive a scholarship with similar students in schools 28.
(e) None of these
What according to the author has led to uprisings in
authoritarian countries?
which received no such financial aid. Previous studies has shown
that the scholarship programme led to higher test scores and
increased the likelihood that girls enrolled in secondary school. En (a) Lack of access to education.
(b) Vast numbers of uneducated and unemployable youth.
Overall, it significantly increased the amount of education obtained.
For the new study the authors tried to see how the extra schooling gin (c) Frustration with the existing system of governance.
(d) Unavailability of natural energy resources like coal and
oil.
had affected the political and social attitudes of the women in
question. Findings suggested that education may make people more
interested in improving their own lives but they may not necessarily
29.
eer
(e) Government’s overambitious plans for development.
Which of the following is/are true about China in the context
of the passage?
see democracy as the way to do it. Even in established democracies,
more education does not always mean either more active political
participation or greater faith in democracy. Poorer and less educated
form of government. ing
(1) China’s citizens are in favor of a more representative

people often vote in larger numbers than their more educated


compatriots, who often express disdain for the messiness of
developments.
.ne
(2) China has made huge strides in infrastructure

(3) China is in the midst of a political revolution.


democracy, yearning for the kind of government that would deal
strongly with the corrupt and build highways, railway lines and
bridges at a dizzying pace of authoritarian China.
24. Which of the following most aptly describes the central
theme of the passage?
30.
(a) None
(c) Only (1) and (3)
(e) All (1), (2) and (3)
(b) Only (1)
(d) Only (2)

What does the phrase “messiness of democracy” convey


in the context of the passage?
t
(a) Democratic nations are richer and have a better track (a) Democratic nations are chaotic on account of individual
record of educating their citizens. freedoms.
(b) Education does not necessarily lead to greater (b) Most democratic countries frequently have violent
enthusiasm for a democratic form of government revolts among their citizens.
(c) Educated societies with autocratic form of government (c) The divide between the poor and educated is growing
enjoy a better quality of life than democracies. wider in democracies.
(d) Citizens can fulfill their personal aspirations only under (d) High levels of pollution on account of frantic pace of
a democratic form of government. infrastructure development.
(e) Democracy makes citizens more intolerant as it does (e) Resigned acceptance of intrinsic corruption in the
not restrict personal freedoms education system.

RESPONSE 24. a b c d e 25. a b c d e 26. a b c d e 27. a b c d e 28. a b c d e


GRID 29. a b c d e 30. a b c d e

Downloaded From : www.EasyEngineering.net


Downloaded From : www.EasyEngineering.net

Reading
Comprehension - II 54
Max. Marks : 30 No. of Qs. 30 Time : 20 min. Date : ........./......../. ..............

DIRECTIONS (Qs. 1-9): Read the following passage carefully 1. Why is the consumer likely to be swept off his feet?
and answer the questions given below it. Certain words/phrases (a) He is easily taken in by the deceptive publicity.
in the passage are printed in bold to help you locate them while (b) He is wooed by the charm of foreign brands readily
answering some of the questions.
available in the market.
In a country where consumers have traditionally had a raw
(c) He is not aware of the Law of Torts as practised abroad.

ww
deal, the Consumer Protection Act was one of the most
progressive acts of legislation introduced in 1986. Before this, a (d) He is not aware of the benefits of the consumer rights.
shop could get away easily with the line “goods once sold will (e) The Consumer Protection Act has been implemented and
not be taken back or exchanged” or a car parking contractor with

w.E
“park at your own risk”. It is not that things have changed now
but at least a legislation is in place and a forum is available to
seek redressal . One of the basic limitations of this act is its
2.
he can seek redressal.
What does ‘lack of... verdicts’ imply?
(a) A lack of the basis of the system, trained staff and

asy
mystification and general ignorance. No consumer agency or
group has made its provisions general, nor has any redressal
commission or forum. Restricted as it is by a lack of in frastructure
decisions based on fact
(b) A paucity of funds, jury and judgement (c)
and personnel and great verdicts to encourage consumers. The
legislation is comprehensive. It gives consumers the right to
En A lack of resources, employees and final decision based
on facts (d)
redress against defective goods, deficient services and unfair
trade practices. Consumer courts must deliver their judgements
within 40 days, but rarely is this deadline adhered to. This gin Not having the required manpower, economy and
decisive ruling
reviewer had a first-hand experience of the chairman of a consumer
court in Delhi who adjourned a case against a foreign airline for 3.
eer
(e) None of these
Which of the following statements is/are true?

ing
two years on the grounds that he did not have staff to type the A. Girimaji’s attempt is comprehensive but could have done
orders. His replacement found the backlog so shocking that he with an angle or two more.
dismissed several cases without applying his mind, in the process

.ne
working against the interests of consumers. But what is more B. Though the Act allows the consumer to approach the
important is that the law has it that a consumer can approach court on his own, yet a lawyer to represent him is insisted
court on his own without having to pay legal fees. In practice, upon.
this does not happen. The chairperson of the National
Commission, who is a sitting judge, is so attuned to delivering
judgments which can stand scrutiny in a civil court of law that it
is insisted upon that a consumer must be represented by a lawyer.
C. Despite the Act, much remains the same.
(a) Only A and C
(c) Only B and C
(b) Only A and B
(d) Only B and C
t
If not, cases are adjourned with impunity and set for another (e) None of these
day. Girimaji’s attempt is creditable in that it is the first of its kind
and has addressed almost all possible angles. She has discussed 4. What does the author mean by ‘mystification of the Act’?
redressals in complaints about housing, basic telephony, rail (a) The mysterious Act is yet to be resolved.
transportation, power supply, life insurance and medical (b) The consumer is wary of the Act.
negligence. There are even tips on how to file a complaint. But it
(c) The Act is not easily accessible.
is mired in the case files of the National/ State Commissions of
the Consumer Forum. A useful dimension would have been a (d) The consumer remains unaware of his rights and
comparison with the Law of Torts practised abroad. It is necessary privileges.
here also, especially in an era of economic liberalisation, when the (e) The plight of the consumer is yet to end.
consumer is like ly to be swept off his feet by free-market forces.

RESPONSE 1. a b c d e 2. a b c d e 3. a b c d e 4. a b c d e
GRID

Downloaded From : www.EasyEngineering.net


Downloaded From : www.EasyEngineering.net

112 SPEED TEST 54


5. Which of the following best describes the judge’s 11. Attuned
replacement? (a) Brought into harmony
(a) He was partial towards the airline as it was a foreign (b) Adjusted
one. (c) Hazardous
(b) He never bothered to safeguard the interests of the (d) Out of tune
reviewer. (e) Malpractice
(c) He dismissed cases without even giving a second 12. Adjourned
thought to what cases came to him. (a) Stopped (b) Postponed (c) Decided
(d) He was apathetic and uninterested about the direction (d) Cleared (e) Pended
the case might head in. DIRECTIONS (Qs. 13-15): Select the word which is most OPPOSITE
(e) He passed irrelevant verdicts indifferently. in meaning of the word printed in bold as used in the passage.
6. What does the Act broadly cover? 13. Impunity
(a) It protects the right to redress. (a) Penalised
(b) It is a forum that protects the redresser. (b) Fine
(c) It shields the consumer from deceptive and unfair trade (c) Sentence

ww
practices.
(d) It enables the plaintiff to fight his case free of cost.
(e) None of these
(d) Freedom from punishment
(e) None of these
14. Mired
(a) Buried (b) Muddy (c) Steeped
7.
w.E
Which of the following is a limitation of the Act?
(a) It does not cover the international law of torts.
(b) It is not comprehensive with regard to liberal economy.
(d) Free
15. Redressal
(l) Plea
(e) None of these

to light. asy
(c) No forum or commission has come forward to bring it (b) Justice
(c) Sue for compensation
(d) Not to compensate
(d) Its red-tapism
(e) None of these
En (e) Put right
DIRECTIONS (Qs. 16-23): Read the following passage carefully
8. How has Girimaji’s attempt been creditable?
(a) It has given the Act a new dimension.
(b) She has brought all the loopholes in the Act to the gin
and answer the questions given below it. Certain words / phrases
have been printed in bold to help you locate them while answering

consumer’s notice.
(c) She has looked at the Act in a very disinterested and eer
some of the questions.
Amartya Sen wrote about the Indian tradition of skepticism
and heterodoxy of opinion that led to high levels of intellectual
impersonal manner.
(d) She has discussed the law in the most explicit manner.
ing
argument. The power sector in India is a victim of this tradition at
its worst. Instead of forcefully communicating, supporting and
honestly and firmly implementing policies, people just debate

9.
(e) Her implicit dialogue with the consumer has made him
aware of his rights.
What is the functionary role of the chairman of the National .ne
them. It is argued that central undertakings produce power at
lower tariffs and must therefore build most of the required extra
Commission?
(a) To be the titular head of the commission
(b) To be accountable to the public
capacities. This is a delusion. They no longer have access to
low-cost government funds.
Uncertainty about payment remains a reason for the
t
hesitation of private investment. They had to sell only to SEBs
(c) To prevent any dissent arising out of his verdicts and (State Electricity Boards). SEB balance sheets are cleaner after the
Acts “securitisation” of the Rs 40,000 crore or so owed by SEBs to
(d) To adjourn the cases with impunity central government undertakings, now shown as debt instruments.
(e) None of these But state governments have not implemented agreed plans to
ensure repayment when due. The current annual losses of around
DIRECTIONS (Qs. 10-12): Choose the word which is most SIMILAR
Rs 28,000 crore make repayment highly uncertain. The central
in meaning to the word printed in bold as used in the passage. undertakings that are their main suppliers have payment security
10. Forum because the government will come to their help. Private enterprises
(a) Dias (b) Podium (c) Platform do not have such assurance and are concerned about payment
(d) Stage (e) None of these security, that must be resolved.

5. a b c d e 6. a b c d e 7. a b c d e 8. a b c d e 9. a b c d e
RESPONSE 10. a b c d e 11. a b c d e 12. a b c d e 13. a b c d e 14. a b c d e
GRID 15. a b c d e

Downloaded From : www.EasyEngineering.net


Downloaded From : www.EasyEngineering.net

SPEED TEST 54 113


By the late 1990s, improving the SEB finances was 18. Which of the following is the reason for apathy of private
recognised as fundamental to power reform. Unbundling SEBs, investors in power sector?
working under corporate discipline and even privatisation and (a) Their hesitation
not vertically integrated state enterprises, are necessary for (b) Uncertainly of their survival
efficient and financially viable electricity enterprises. Since (c) Cut-throat competition
government will not distance itself from managing them, (d) Lack of guarantee of timely returns
privatising is an option. The Delhi model has worked. But it (e) None of these
receives no public support. 19. What was the serious omission on the part of the State
Government?
The Electricity Act 2003, the APRDP (Accelerated Power
(a) Agreement for late recovery of dues
Reform and Development Programme) with its incentives and
(b) Reluctance to repay to private investors as per agreed
penalties, and the creation of independent r egulatory
plan
commissions, were the means to bring about reforms to improve
(c) Non-implementation of recovery due to unplanned and
financial viability of power sector. Implementation has been half-
haphazard polices
hearted and results disappointing. The concurrent nature of
(d) Lack of assurance from private enterprises
electricity in the Constitution impedes power sector improvement.
(e) None of these
States are more responsive to populist pressures than the central

ww
government, and less inclined to take drastic action against
electricity thieves.
20. Which of the following is/are considered necessary for
improving performance of electricity enterprises?
(A) Corporate work culture

w.E
Captive power would add significantly to capacity. However,
captive generation, three years after the Act enabled it, has added
little to capacity because rules for open access were delayed.
(B) Privatisation
(C) Properly integrated state enterprises
(a) All the three (b) (a) and (b) only

asy
Redefined captive generation avoids state vetoes on purchase
or sale of electricity except to state electricity enterprises.
Mandating open access on state-owned wires to power
21.
(c) (a) and (c) only
(e) None of these
(d) (b) and (c) only

The example of “Delhi Model” quoted by the author


regardless of ownership and customer would encourage
electricity trading. The Act recognised electricity trading as a
En underlines his feelings of
A. happiness about its success.
separate activity. A surcharge on transmission charges will pay
for cross-subsidies. These were to be eliminated in time. Rules
for open access and the quantum of surcharge by each state gin B. unhappiness for lack of public support
C. disgust towards privatisation.
commission (under broad principles defined by the central
commission) have yet to be announced by some. The few who
eer
(a) (a) and (b) only (b) (b) and (c) only
(c) (a) and (c) only
(e) None of these
(d) All the three
have announced the surcharge have kept it so high that no trading
can take place.
16. The author thinks it appropriate to
22.
ing
Which of the following was/were not considered as the
instrument(s) to accomplish financial well-being of power
(a) discuss any policy in details and make it fool proof
instead of implementing it hastily.
sector?
(a) The Electricity Act 2003
.ne
(b) The APRDP with its incentives and penalties
(b) follow Indian tradition meticulously as skepticism is
essential for major decisions.
(c) divert our energies from fruitlessly contracting policies
to supporting its implementation whole-heartedly.
23.
(c) Setting up of independent regulatory commissions
(d) States vulnerability to populist pressures
(e) Taking drastic action against electricity thieves.
Why were the results of the power sector reforms NOT as
t
(d) intellectual arguments and conceptualisation of every
had been anticipated?
policy is definitely better than its enforcement.
(a) The means to bring about reforms were illconceived.
(e) none of these
(b) The enforcement of the reform means was inadequate
17. Why are the central undertakings not capable of generating
and apathetic.
power at low cost?
(c) The Act and the reform measures were contradicting
(a) Due to paucity of low-cost funds
(b) Due to their access to Government funds with each other.
(c) Due to their delusion about government funds (d) The incentives on the one hand and penalties on the
(d) Because of their extra capacities other created dissatisfaction.
(e) None of these (e) None of these

RESPONSE 16. a b c d e 17. a b c d e 18. a b c d e 19. a b c d e 20. a b c d e


21. a b c d e 22. a b c d e 23. a b c d e
GRID

Downloaded From : www.EasyEngineering.net


Downloaded From : www.EasyEngineering.net

114 SPEED TEST 54


DIRECTIONS (Qs. 24-30) : Read the following passage and answer 25. The author thinks that openness in budget is essential as it
the questions given below it. Certain words/phrases are given in leads to
hold to bold you to locate them while answering some of the (a) prevention of tax implications
questions. (b) people’s reluctance to accept their moral duties
We have inherited the tradition of secrecy about the budget (c) exaggerated revelation of the strengths and
from Britain where also the system has been strongly attacked by weaknesses of economy
eminent economists and political scientists including Peter Jay. (d) making our country on par with Finland
Sir Richard Clarke, who was the originating genius of nearly every (e) None of these
important development in the British budgeting techniques during 26. The author seems to be in favour of
the last two decades, has spoken out about the abuse of budget (a) maintaining secrecy of budget
secrecy: “The problems of long-term tax policy should surely be (b) judicious blend of secrecy and openness
debated openly with the facts on the table. In my opinion, all (c) transparency in budget proposals
governments should have just the same duty to publish their (d) replacement of public constitution by secrecy
expenditure policy. Indeed, this obligation to publish taxation (e) None of these
policy is really essential for the control of public expenditure in 27. The secrecy of the budget is maintained by all of the
order to get realistic taxation implications.” Realising that following countries except

ww
democracy flourishes best on the principles of open government,
more and more democracies are having an open public debate on
A. Finland
B. India
C. United States

w.E
budget proposals before introducing the appropriate Bill in the
legislature. In the United States the budget is conveyed in a
message by the President to the Congress, which comes well in
28.
(a) Only A
(d) A and C
(b) Only B
(e) B and C
(c) Only C

Which of the following statements is definitely TRUE in the

asy
advance of the date when the Bill is introduced in the Congress.
In Finland the Parliament and the people are already discussing in
June the tentative budget proposals which are to be introduced in
context of the passage?
(a) The British Government has been religiously
the Finnish Parliament in September. Every budget contains a
En
cartload of figures in black and white - but the dark figures represent
maintaining budget secrecy.
(b) Budget secrecy is likely to lead to corrupt practices.
(c) Consulting unjustifiable taxes with public helps make
the myriad lights and shades of India’s life, the contrasting tones
of poverty and wealth, and of bread so dear and flesh and blood
so cheap, the deep tints of adventure and enterprise and man’s gin them accept those taxes.
(d) There should be no control on public expenditure in
ageless struggle for a brighter morning. The Union budget should
not be an annual scourge but a part of presentation of annual
29. eer
democratic condition.
(e) None of these
Sir Richard Clarke seems to deserve the credit for
accounts of a partnership between the Government and the people.
That partnership would work much better when the nonsensical
secrecy is replaced by openness and public consultations, resulting ing
(a) transformation in the British budgetary techniques.
(b) maintenance of secrecy of the British budget.
in fair laws and the people’s acceptance of their moral duty to pay.
24. How do the British economists and political scientists react .ne
(c) detection of abuse of transparency in budget.
(d) bringing down the tax load on British people.
to budget secrecy? They are
(a) in favour of having a mix of secrecy and openness.
(b) indifferent to the budgeting techniques and taxation
policies.
30.
(e) None of these

author is
(a) authoritarian in his approach.
t
From the contents of the passage, it can be inferred that the

(c) very critical about maintenance of budget secrecy. (b) a democratic person.
(d) advocates of not disclosing in advance the budget (c) unaware of India’s recent economic developments.
contents. (d) a conservative person.
(e) None of these (e) None of these

RESPONSE 24. a b c d e 25. a b c d e 26. a b c d e 27. a b c d e 28. a b c d e

GRID 29. a b c d e 30. a b c d e

Downloaded From : www.EasyEngineering.net


Downloaded From : www.EasyEngineering.net

Reading
Comprehension - III 55
Max. Marks : 30 No. of Qs. 30 Time : 20 min. Date : ........./......../. ..............

DIRECTION : Read the following passages to answer the given The political benefits of the loan waiver have also been
question bused on it. Some words/phrases are printed in bold to exaggerated since if only a small fraction of farm families benefit,
and many of these have to pay bribes to get the actual benefit, will
help you locate them while answering some of the questions.
the waiver really be a massive vote-winner? Members of joint families
Political ploys initially hailed as master-strokes often end will feel aggrieved that, despite having less than one hectare per
up as flops. The ` 60,000 crore farm loan waiver announced in head, their family holding is too large to qualify for the 100% waiver.
the budget writes off 100% of overdues of small and marginal All finance ministers, of central or state governments, give away

ww
farmers holding upto two hectares, and 25% of overdues of larger
farmers. While India has enjoyed 8%-9% GDP growth for the
past few years, the boom has bypassed many rural areas and
freebies in their last budgets, hoping to win electoral regards. yet,
four-fifth of all incumbent government are voted out. This shows
that beneficiaries of favours are not notably grateful, while those

w.E
farmer distress and suicides have made newspaper headlines.
Various attempts to provide relief (employment guarantee scheme,
public distribution system) have made little impact, thanks to
not so favoured may feel aggrieved, and vote for the opposition.
That seems to be why election budgets constantly fail to win
elections in India and the loan waiver will not change that pattern.

asy
huge leakages from the government’s lousy delivery systems.
So, many economists think the loan waiver is a worthwhile
alternative to provide relief.
1. Why do economists feel that loan waivers will benefit farmers
in distress?
(a) It will improve the standard of living of those farmers
However the poorest rural folk are landless labourers who
En
get neither farm loans nor waivers. Half of the small and marginal
who can afford to repay their loans but are exempted.
(b) Other government relief measures have proved
ineffective,
farmers get no loans from banks, and depend entirely on
moneylenders, and will not benefit. Besides, rural India is full of
the family holdings rather than individual holdings and family
gin (c) Suicide rates of farmers have declined after the
announcement of the waiver.
(d) Farmers will be motivated to increase the size of their
holdings will typically be much larger than two hectares even for
dirt-poor farmers, who will, therefore, be denied the 100% waiver. It eer
family holdings not individual holdings.
(e) The government will be forced to re-examine and improve
will thus fail in both economic and political objectives. IRDP loans
to the rural poor in the 1980s demonstrated that crooked bank
officials demand bribes amounting to one third the intended
2.
ing
the public distribution system.
What message will the loan waiver send to farmers who have
repaid loans?
benefits. Very few of the intended beneficiaries who merited relief
received it. After the last farm loan waiver will similarly slow down
the future.
.ne
(a) The Government will readily provide them with loans in

(b) As opposed to money lenders banks are a safer and


fresh loans to deserving farmers. While overdues to co-operatives
may be higher, economist Snrjit Bhalla says less then 5% of farmer
loans to banks are overdue ie overdues exist for only 2.25 million
out of 90 million farmers. If so, then the 95% who have repaid loans
more reliable source of credit.
(c) Honesty is the best policy. t
(d) It is beneficial to take loans from co-operatives since
their rates of interest are lower.
will not benefit. They will be angry at being penalised for honesty. (e) They will be angry at being penalised for honesty.
The budget thus grossly overestimates the number of 3. What is the author's suggestion to provide aid to farmers?
beneficiaries. It also underestimates the negative effects of the (a) Families should split their joint holding to take advantage
waiver encouraging wilful default in the future and discouraging of the loan waiver.
fresh bank lending for some years. Instead of trying to reach the (b) The government should increase the reach of the
needy, through a plethora of leaky schemes we should transfer employment guarantee scheme.
(c) Loans should be disbursed directly into bank accounts
cash directly to the needy using new technology like biometric
of the farmers using the latest technology.
smart cards, which are now being used in many countries, and (d) Government should ensure that loans waivers can be
mobile phones bank accounts. Then benefits can go directly to implemented over the number of years.
phone accounts operable only by those with biometric cards, (e) Rural infrastructure can be improved using schemes
ending the massive leakages of current schemes. which were successful abroad.

RESPONSE 1. a b c d e 2. a b c d e 3. a b c d e
GRID

Downloaded From : www.EasyEngineering.net


Downloaded From : www.EasyEngineering.net

116 SPEED TEST 55


4. What was the outcome of IRDP loans to the rural poor? 9. What impact will the loan waiver have on banks?
(a) The percentage of bank loan sanctioned to family (a) Banks have to bear the entire brunt of the write off.
owned farms increased. (b) Loss of trust in banks by big farmers.
(b) The loans· benefited dishonest moneylenders not (c) Corruption among bank staff will increase.
landless labourers. (d) Farmers will make it a habit to default on loans
(c) Corrupt bank officials were the unintended beneficiaries (e) None of these
of the loans. 10. According to the author what is the government's motive in
(d) It resulted in tne Government sanctioning thrice the sanctioning the loan waiver?
amount for the current loan waiver. (a) To encourage farmers to opt for bank loans from money
(e) None of these. lenders.
5. What are the terms of the loan waiver? (b) To raise 90 million farmers out of indebtedness.
(1) One-fourth of the overdue loans of landless labourers (c) To provide relief to those marginal farmers who have
will be written off. the means to but have not repaid their loans
(2) The ` 60,000 crore loan waiver has been sanctioned for (d) To ensure they will be re-elected
2.25 million marginal farmers. (e) None of these

ww
(3) Any farmer with between 26 per cent to 100 per cent of
their loan repayments overdue will be penalised.
(a) Only (1) (b) On1y (2)
DIRECTION (Q. 11-13) : Choose the word which is most nearly
the SAME in meaning to the word printed in bold as used in the
passage.

(e) None of these w.E


(c) Both (2) and (3) (d) All (1), (2) and (3) 11. incumbent
(a) mandatory
(c) incapable
(b) present
(d) tazy
6.

asy
What is the author's view of the loan waiver?
(a) It will have an adverse psychological impact on those
who cannot avail of the waiver.
(e) officious
12. ploys

En
(b) It is a justified measure in view of the high suicide rate
among landless labourers.
(a) surveys
(c) ruses
(b) entreaties
(d) sliders
(c) It makes sound economic and political sense in the
existing scenario. gin (d) assurances
13. aggrieved
(d) It will ensure that the benefits of India's high GDP are
felt by the rural poor.
(e) None of these
(a) vindicated
(c) offensive
(d) disputed eer (b) intimidated
(d) wronged

7. Which of the following cannot be said about loan waiver?


(1) Small and marginal farmers will benefit the most. ing
DIRECTIONS (Q. 14 & 15) : Choose the word which is most OPPOSITE
in meaning to the word printed in bold as used in the passage.
(2) The loan waiver penalises deserving farmers.
(3) A large percentage ie ninety five per cent of distressed
14. plethora
(a) dearth (b) missing .ne
farmers will benefit.
(a) Only (3)
(c) Only (1)
(e) None of these
(b) Both (1) and (3)
(d) Both (2) and (3)
(c) superfluous
(e) least
15. merited
(a) ranked·
(d) sufficient

(b) unqualified for


t
8. Which of the following will definitely be an impact of loan (c) lacked (d) inept at
waivers? (e) unworthy of
(1) Family holdings will be split into individual holdings DIRECTIONS (Q. 16-25): Read the following passage to answer
not exceeding one hectare. the given questions based on it, Some words/phrases are printed in
(2) The public distributipn system will be revamped. bold to help you locate them while answermg some of the questions.
(3) Opposition will definitely win the election. The e-waste (Management and Handling) Rules, 2011,
(a) None (b) Only (1) notified by the Ministry of Environmynt and Forests, have the
(c) Both (1) and (2) (d) Only (3) potential to turn a growing problem into a developmental
opportunity. With almost half-a-year to go before the rules take
(e) All (1), (2) and (3)

4. a b c d e 5. a b c d e 6. a b c d e 7. a b c d e 8. a b c d e
RESPONSE 9. a b c d e 10. a b c d e 11. a b c d e 12. a b c d e 13. a b c d e
GRID 14. a b c d e 15. a b c d e

Downloaded From : www.EasyEngineering.net


Downloaded From : www.EasyEngineering.net

SPEED TEST 55 117


effect, there is enough time to create the necessary infrastructure 17. Which of the following can be one ofthe by-products of
for collection, dismantling and recycling of electronic waste. The effective e-waste management? .
focus must be on sincere and efficient implementation. Only (a) India can guide other countries in doing so.
decisive action can reduce the pollution and health costs (b) It will promote international understanding.
associated with India's hazardous waste recycling industry. If (c) It will promote national integration.
India can achieve a transformation, it will be creating a whole (d) It will create a new employment sector.
new-employment sector that provides good wages and working (e) It will further empowet judiciary.
conditions for tens of thousands. The legacy response of the 18. Which of the following rules has not been indicated in the
States to even the basic law on urban waste, the Municipal Solid passage?
Wastes (Management and Handling) Rules has been one of (a) e-waste Rules 20 11
indifference; many cities continue to simply bum the garbage or (b) Pollution Check Rules
dump it in lakes. With the emphasis now on segregation of waste (c) Hazardous Wastes Rules, 2008
at source and recovery of materials. it should be feasible to (d) Municipal Solid Wastes Rules
implement both sets of rules efficiently. A welcome feature of the (e) All these have been indicated
new e-waste rules is the emphasis on extended producer 19. “Both sets of rules” is being referred to which of the following?
responsibility. In other words, producers must take responsibility

ww
for the disposal of end-of-life products. For this provision to
work, they must ensure that consumers who sell scrap get some
(a) Solid wastes and hazardous wastes
(b) e-waste and hazardous waste
(c) Solid waste and e-waste
form of financial incentive.

w.E
The e-waste rules, which derive from those pertaining to
hazardous waste, are scheduled to come into force on May 1, 20.
(d) e-waste and e-production
(e) Solid waste and recycling waste
e-waste rules have been derived from those pertaining to

asy
2012. Sound as they are, the task of scientifically disposing a few
hundred thousand tonnes of trash electronics annually depends
heavily on a system of oversight by State Pollutions Control
(a) Hazardous waste (b) PC waste
(c) Computer waste (d) Municipal solid waste
(d) National waste
Boards (PCBs). Unfortunately, most PCBs remain unaccountable
and often lack the resources for active enforcement. It must be En 21. Which of the following will help implement “both sets of
rules”?
pointed out that, although agencies handling e-waste must obtain
environmental clearances and be authorised and registered by
the PCBs even under the Hazardous Wastes (Management,
gin (a) Employment opportunities
(b) International collaboration

Handling and Transboundary Movement) Rules 2008, there has


been little practical impact. Over 95 per cent of electronic waste is eer
(c) Financial incentive
(d) Segregation of waste at source
(e) Health costs
collected and recycled by the informal sector. The way forward is
for the PCBs to be made accountable for enforcement of the e-
22.
ing
e-waste Rules came/come into force from
(a) 2008 (b) 2009
waste rules and the levy of penalties under environmental laws.
Clearly, the first order priority is to create a system that will absorb
the 80,000-strong workforce in the informal sector into the
(c) 2010
(e) 2012
(d) 2011
.ne
proposed scheme for scientific recycling. Facilities must be created
to upgrade the skills of these workers through training and their
occupational health must be ensured.
23. Which of the following best explains the meaning of the
phrase “which could only be imagined few years back” as
used in the passage?
(a) It was doomed.
t
Recycling of e-waste is one of the biggest challenges today. In (b) It took us few years.
such a time, when globalisation and information technology are (c) It took us back by few years
growing at a pace which could only be imagined few years back, (d) Imagination is better than IT.
e-waste and its hazards have become more prominent over a (e) None of these
period of time and should be given immediate attention. 24. Which of the following is true in the context of the passage?
16. What, according to the passage, is important now for (a) No city dumps its waste in lakes.
e-waste management? (b) Some cities burn garbage.
(a) Making rules (b) Reviewing rules (c) PCBs have adequate resources for'active enforcement.
(c) Implementing rules (d) Notifying rules (d) e-waste was a much bigger chajlenge in the past.
(e) Amending rules (e) None of these

RESPONSE 16. a b c d e 17. a b c d e 18. a b c d e 19. a b c d e 20. a b c d e


21. a b c d e 22. a b c d e 23. a b c d e 24. a b c d e
GRID

Downloaded From : www.EasyEngineering.net


Downloaded From : www.EasyEngineering.net

118 SPEED TEST 55


25. Which of the following is not true 10 the context of the 27. Turn
passage? (a) Throw (b) Chance
(a) Some fonn offmancial incentive is recommended for (c) Send (d) Transform
the producers. (e) Rotate
(b) Some financial incentive is recommended for the 28. Potential
consumers. (a) Intelligence (b) Aptitude
(c) e-waste will be a few hundred thousand tonnes. (c) Possibility (d) Portion
(d) The agencies handling e-waste have to obtain (e) Will
environmental clearances. DIRECTIONS (Q. 29 & 30): Choose the word which is most
(e) Those involved in e-waste management would need to opposite in meaning of the word printed in bold as used in the
upgrade their skills. passage.
29. Feasible
DIRECTIONS (Q. 26-28): Choose the word which is most nearly (a) Unattended (b) Physical
the same in meaning of the word printed in bold, as used in the (c) Practical (d) Unviabie
passage. (e) Wasteful
26. Clearance

ww
(a) Cleaning
(c) Sale
(b) Permisssion
(d) Remedy
30. Indifference
(a) Interest
(c) Ignorance
(b) Difference
(d) Rule-bound
(e) Clarity
w.E (e) Insignificance

asy
En
gin
eer
ing
.ne
t

RESPONSE 25. a b c d e 26. a b c d e 27. a b c d e 28. a b c d e 29. a b c d e

GRID 30. a b c d e

Downloaded From : www.EasyEngineering.net


Downloaded From : www.EasyEngineering.net

Synonyms 56
Max. Marks : 40 No. of Qs. 40 Time : 25 min. Date : ........./......../. ..............

DIRECTIONS (Qs. 1-25): Select the synonyms of the word given 9. PROTAGONIST
in CAPITAL letters. (a) Talented child (b) Reserved person
(c) Leading character (d) Fearless
1. VICARIOUS
(e) None of these
(a) Ambitious 10. FACTITIOUS
(b) Not experienced personally (a) Humorous (b) Truthful

ww
(c) Nostalgic
(d) Vindictive
(e) None of these 11.
(c) Artificial
(e) None of these
HOSPITABLE
(d) Causing fatigue

2. CRAVEN
(a) Greedy
(c) Flattering
w.E (b) Cowardly
(d) Restless
(a) Convivial
(c) Congenital
(e) None of these
(b) Liberal
(d) Welcoming

3.
(e) None of these
TEPID asy 12. SCARCELY
(a) Hardly
(c) Sometimes
(b) Always
(d) Frequently
(a) Irreversible
(c) Fast moving
(b) Causing fatigue
(d) Lukewarm
En 13.
(e) None of these
DISDAIN

4.
(e) None of these
TENUOUS
(a) Contentious (b) Dark gin
(a) Disown
(c) Hate
(e) None of these
(b) Condemn
(d) Criticise

(c) Slender
(e) None of these
(d) Malfunctioning 14. ABSURD
eer
(a) Senseless (b) Clean
5. PROBITY
(a) Integrity (b) Impudence
15.
(c) Abrupt
(e) None of these
PHILANTHROPY ing (d) Candid

(c) Profane
(e) None of these
(d) Preface
(a) Generosity
(c) Perjury .ne
(b) Perversity
(d) Flaunting
6. MUSTY
(a) Certainty
(c) Modern
(e) None of these
(b) Stale
(d) Mysterious
16.
(e) None of these
MUTUAL
(a) Reciprocal
(c) Common
(b) Agreed
(d) Conjugal
t
7. ALLEVIATE (e) None of these
(a) To release (b) To lessen 17. WEIRD
(c) To deprive (d) To deceive (a) Beastly (b) Unpleasant
(e) None of these (c) Frightening (d) Unnatural
8. MOROSE (e) None of these
(a) Humble (b) Morsel
(c) Sullen (d) Repugnant
(e) None of these

1. a b c d e 2. a b c d e 3. a b c d e 4. a b c d e 5. a b c d e
RESPONSE 6. a b c d e 7. a b c d e 8. a b c d e 9. a b c d e 10. a b c d e
GRID 11. a b c d e 12. a b c d e 13. a b c d e 14. a b c d e 15. a b c d e
16. a b c d e 17. a b c d e

Downloaded From : www.EasyEngineering.net


Downloaded From : www.EasyEngineering.net

120 SPEED TEST 56


18. PESSIMISTIC (c) Clear (d) Ambiguous
(a) Indifferent (b) Ascetic (e) None of these
(c) Unsettle (d) Not hopeful 30. Friends have always DEPLORED my unsociable nature.
(e) None of these (a) Deprived (b) Implored
19. ANALOGOUS (c) Denied (d) Regretted
(a) Unsuitable (b) Uncritical (e) None of these
(c) Similar (d) Disproportionate
31. People fear him because of his VINDICTIVE nature.
(e) None of these
20. EXAGGERATE (a) Violent (b) Cruel
(a) Bluff (b) Overstate (c) Revengeful (d) Irritable
(c) Explain (d) Underestimate (e) None of these
(e) None of these 32. He always has a very PRAGMATIC approach to life.
21. EVIDENT (a) Practical (b) Proficient
(a) Prominent (b) Seen (c) Potent (d) Patronizing
(c) Observed (d) Quite clear (e) None of these
(e) None of these 33. The song had a SOPORIFIC effect on the child.
22. PENALIZE
(a) Soothing (b) Terrific

ww
(a) Persecute
(c) Torture
(e) None of these
(b) Punish
(d) Ruin
(c) Supreme
(e) None of these
(d) Sleep-inducing

23. REMEDIAL
(a) Punitive
(c) Corrective
(e) None of these
w.E (b) Stringent
(d) Strict
34. His bad behaviour EVOKED punishment.
(a) Escaped
(c) Produced
(e) None of these
(b) Called for
(d) Summoned

24. TRIVIAL
(a) Unimportant asy
(b) Transparent
35. The boy said that pain had ABATED.
(a) Reduced (b) Vanished

25.
(c) Important
(e) None of these
INCREDIBLE
(d) Unexpected

En (c) Increased
(e) None of these
(d) Stabilised

(a) Hard to believe


(c) Inconsistent
(b) Considerable
(d) Unsatisfactory gin
36. The queen was aware of the INSOLENT behaviour of the
lords.
(a) Violent (b) Polite
(e) None of these
DIRECTIONS (Qs. 26-40): Select the synonym of the word
occuring in the sentence in CAPITAL letters as per the context.
(c) Insulting eer
(e) None of these
(d) Frivolous

26. After weeks of FRENETIC activity, the ground was ready


for the big match.
37.
(a) mischievous
(c) provocative ing
The newspaper reports were MENDACIOUS
(b) truthful
(d) false
(a) Strenuous
(c) Excited
(e) None of these
(b) Hurried
(d) Hectic
38.
(e) None of these
.ne
He INDUCES human beings to want things they don’t want.
27. The policemen kept on questioning the criminal but failed to
ELICIT the truth.
(a) Evoke
(c) Obtain
(b) Wrest
(d) Extort 39.
(a) Influences
(c) Persuades
(e) None of these
(b) Dictates
(d) Appreciates

His information is not AUTHENTIC.


t
(e) None of these (a) Real (b) Reliable
28. Teaching in universities is now considered a LUCRATIVE (c) Believable (d) Genuine
job. (e) None of these
(a) Risky (b) Profitable 40. It is the difficulty or SCARCITY of a thing that it makes it
(c) Honourable (d) Undignified precious.
(e) None of these (a) poverty (b) absence
29. His descriptions are VIVID. (c) insufficiency (d) disappearance
(a) Detailed (b) Categorical (e) None of these

18. a b c d e 19. a b c d e 20. a b c d e 21. a b c d e 22. a b c d e


23. a b c d e 24. a b c d e 25. a b c d e 26. a b c d e 27. a b c d e
RESPONSE 28. a b c d e 29. a b c d e 30. a b c d e 31. a b c d e 32. a b c d e
GRID 33. a b c d e 34. a b c d e 35. a b c d e 36. a b c d e 37. a b c d e
38. a b c d e 39. a b c d e 40. a b c d e

Downloaded From : www.EasyEngineering.net


Downloaded From : www.EasyEngineering.net

Antonyms 57
Max. Marks : 40 No. of Qs. 40 Time : 25 min. Date : ........./......../. ..............

DIRECTIONS (Qs. 1-20): Select the antonyms of the word given 11. EXPLOIT
in CAPITAL letters. (a) Utilize (b) Alert
(c) Support (d) Neglect
1. REINFORCING
(e) None of these
(a) contradicting (b) wishing
12. SHARP
(c) jolting (d) forcing
(e) re-inventing (a) Bleak (b) Blunt
2.
ww
BEFRIENDED
(a) recoiled
(c) accepted
(b) killed
(d) mistrusted 13.
(c) Bright
(e) None of these
CONDEMN
(d) Blond

3.
(e) ignored
FRAIL
(a) Unhealthy
w.E
(b) Massive
(a) Censure
(c) Recommend
(e) None of these
(b) Approve
(d) Praise

4.
(c) Rich
(e) Civilised
SPLENDIDLY
(d) Robust
asy 14. RELUCTANT
(a) Avoiding
(c) Refuse
(b) Anxious
(d) Eager
(a) Wisely
(c) Rudely
(b) Unfairly
(d) Reluctantly En 15.
(e) None of these
SCARCITY

5.
(e) Unimpressively
CHARMED
(a) Offended (b) Stunned gin (a) Plenty
(c) Facility
(e) None of these
(b) Prosperity
(d) Simplicity

(c) Repulsed
(e) Outraged
(d) Jealous 16. BLEAK
(a) Brighteer (b) Confusing
6. OBLIGATORY
(a) Doubtful
(c) Sincerely
(b) Voluntary
(d) Faithfully 17.
(c) Uncertain
(e) None of these
STERN ing
(d) Great

7.
(e) None of these
OBSCURE
(a) Suitable (b) Apt
(a) Violent
(c) Mild
(b) Generous
(d) Forgiving
.ne
8.
(c) Thalamus
(e) None of these
REPULSIVE
(a) Attractive
(d) Clear

(b) Colourful
18.
(e) None of these
SUPERFICIAL
(a) Profound
(c) Secretive
(b) Difficult
(d) Mystical
t
(e) None of these
(c) Unattractive (d) Striking 19. ELEGANCE
(e) None of these (a) Balance (b) Indelicacy
9. VITAL (c) Clumsiness (d) Savagery
(a) Trivial (b) Peripheral (e) None of these
(c) Optional (d) Superficial 20. COARSE
(e) None of these (a) Pleasing (b) Rude
10. INHIBIT (c) Polished (d) Soft
(a) Pamper (b) Breed
(e) None of these
(c) Accept (d) Promote
(e) None of these

1. a b c d e 2. a b c d e 3. a b c d e 4. a b c d e 5. a b c d e
RESPONSE 6. a b c d e 7. a b c d e 8. a b c d e 9. a b c d e 10. a b c d e
GRID 11. a b c d e 12. a b c d e 13. a b c d e 14. a b c d e 15. a b c d e
16. a b c d e 17. a b c d e 18. a b c d e 19. a b c d e 20. a b c d e

Downloaded From : www.EasyEngineering.net


Downloaded From : www.EasyEngineering.net

122 SPEED TEST 57


DIRECTIONS (Qs. 21-40): Select the antonym of the word (a) Active (b) Lazy
occuring in the sentence in CAPITAL letters as per the context. (c) Strong (d) Resolute
21. He is a man of EXTRAVAGANT habits. (e) None of these
(a) Sensible (b) Careful 31. The boy comes of an AFFLUENT family.
(a) Poor (b) Ordinary
(c) Economical (d) Balanced
(c) Infamous (d) Backward
(e) None of these
(e) None of these
22. They employ only DILIGENT workers.
32. The young athlete is ENERGETIC enough to run ten
(a) Unskilled (b) Lazy thousand meters at a stretch.
(c) Careless (d) Idle (a) inactive (b) dull
(e) None of these (c) gloomy (d) lethargic
23. The characters in this story are not all FICTITIOUS. (e) None of these
(a) Common (b) Factual 33. He has a SECURE position in the entrance examination.
(c) Real (d) Genuine (a) rigid (b) precarious
(e) None of these (c) static (d) secondary
24. He is a GENEROUS man. (e) None of these

ww
(a) Stingy
(c) Selfish
(e) None of these
(b) Uncharitable
(d) Ignoble
34. The criminal was known to the police by VARIOUS names.
(a) separate
(c) identical
(b) distinct
(d) similar

(a) Removed
(c) Set aside
w.E
25. The issue raised in the form can be IGNORED.
(b) Considered
(d) Debated
35.
(e) None of these
RECESSION is a major cause of unemployment.
(a) Education (b) Inflation
(e) None of these
asy
26. After swallowing the frog had become LETHARGIC.
(a) Aggressive (b) Dull 36.
(c) Poverty
(e) None of these
(d) Computerization

The accused emphatically DENIED the charge in the court.


(c) Active
(e) None of these
(d) Hungry
En (a) Accepted
(c) Asserted
(b) Agreed
(d) Affirmed
27. Dust storms and polluted rivers have made it HAZARDOUS
to breathe the air and drink the water. gin
37.
(e) None of these
The three states signed a pact to have COLLECTIVE
economy.
(a) Convenient
(c) Wrong
(e) None of these
(b) Risky
(d) Safe (c) individual
eer
(a) distributive (b) disintegrative
(d) divided

28. Only hard work can ENRICH our country.


(a) Impoverish (b) Improve
38.
(e) None of these

ing
The child was ABDUCTED when he was going to school.
(a) seized (b) set free
(c) Increase
(e) None of these
(d) Involve (c) kidnapped
(e) None of these
(d) ransomed
.ne
29. The story you have just told is INCREDIBLE.
(a) Credible
(c) Probable
(e) None of these
(b) Fantastic
(d) Believable
39. The government has ENHANCED the tuition fees in schools
and colleges.
(a) magnified
(c) decreased
(e) None of these
(b) aggravated
(d) augmented
t
30. The doctor found the patient INERT.
40. Children ADORE the cricket players.
(a) Discuss (b) Condemn
(c) Benefit (d) Check
(e) None of these

21. a b c d e 22. a b c d e 23. a b c d e 24. a b c d e 25. a b c d e


RESPONSE 26. a b c d e 27. a b c d e 28. a b c d e 29. a b c d e 30. a b c d e

GRID 31. a b c d e 32. a b c d e 33. a b c d e 34. a b c d e 35. a b c d e


36. a b c d e 37. a b c d e 38. a b c d e 39. a b c d e 40. a b c d e

Downloaded From : www.EasyEngineering.net


Downloaded From : www.EasyEngineering.net

Sentence
Completion - I 58
Max. Marks : 30 No. of Qs. 30 Time : 20 min. Date : ........./......../. ..............

DIRECTIONS: Select the correct word or phrase to complete a 8. Of the two assistants we employed last month, I find
grammatical sentence. In case of more than one blank, the different Raman ............. hard working.
(a) most (b) more
words given in the options shall fill in the corrosponding order.
(c) least (d) only
1. As soon as the visitor’s dishonest purpose was discovered (e) None of these
he was ........ the door. 9. It is earth’s gravity which .............. people their weight.

ww
(a) show with
(c) shown
(e) None of these
(b) shown to
(d) shown out of
(a) gives
(c) giving
(e) None of these
(b) give
(d) given

2.

w.E
As soon as my attention was . ....... the dangerous state of
the staircase, I got it repaired.
(a) drawn for (b) drawn upon
10. Total weight of all the ants in the world is much greater
than .............. .
(a) to all human beings (b)
that of all human beings (c)

3.
(c) drawn near
(e) None of these
(d) drawn to

asy
I wish my brother -........ here to listen to this entertaining
is of all human beings (d)
that of the all human beings
lecture.
(a) would be (b) has been
En 11.
(e) None of these
It is good form to use the name of the person . .............. .
(a) who are greeting (b)

4.
(c) is
(e) None of these
(d) were

Did you think you . ....... somewhere before? gin you are greeting (c)
which you are greeting
(a) have seen me
(c) had seen me
(b) saw me
(d) would see me
12. eer
(d) greeting for you (e)
None of these
. ............ that increasing numbers of compact disc players will
5.
(e) None of these
Do not force me to ........ you on this issue; I am not at all
convinced.
(a) They are anticipated ing
be bought by consumers in the years to come.

(a) agree upon


(c) join over
(b) concur with
(d) equate with
(b) In anticipation (c)
Anticipating (d)
It is anticipated (e) .ne
6.
(e) None of these
Having ........ only in salt water before, I found it a little
difficult to swim in fresh water.
(a) swam (b) swum
13.
None of these
He was frightened . .............
(a) to be killed (b) to being killed
(c) for being killed (d) of being killed
t
(c) had swam (d) swimming (e) None of these
(e) None of these 14. Capitalist society .............. profit as a valued goal.
7. If I were you, I ............... be careful with my words. (a) which regards (b) regarding
(a) will (b) would (c) regards (d) was regarded
(c) shall (d) should (e) None of these
(e) None of these 15. The impact of two vehicles can cause a lot of .............. to both.
(a) damage (b) damages
(c) damaging (d) damagings
(e) None of these

1. a b c d e 2. a b c d e 3. a b c d e 4. a b c d e 5. a b c d e
RESPONSE
6. a b c d e 7. a b c d e 8. a b c d e 9. a b c d e 10. a b c d e
GRID
11. a b c d e 12. a b c d e 13. a b c d e 14. a b c d e 15. a b c d e

Downloaded From : www.EasyEngineering.net


Downloaded From : www.EasyEngineering.net

124 SPEED TEST 58


16. The conditions necessary . ............ this project have not been DIRECTIONS : Select the correct word or phrase to complete a
met. grammatical and idiomatic sentence.
(a) of completion (b) for the complete of
(c) of complete (d) for the completion of 24. If you are really not feeling well. you . ......... a doctor.
(e) None of these (a) should better see (b) may Sec
17. The weather in the far north is not .............. it is down south. (c) had better see (d) would rather see
(a) like humid as (b) as humid as (e) None of these
(c) humid as (d) so humid that 25. If only I.......... his address, I would most certainly have told
(e) None of these you.
18. .............. the reactions of people with amnesia, scientists are (a) know (b) knew
learning about the process of memory of the brain.
(c) had known (d) off
(a) By studying (b) To study
(e) None of these
(c) They study (d) They are studying
(e) None of these 26. The marathon race is intended to test one’s endurance
19. After the election .............. a new stage. more..........
(a) the entering nation (a) than his speed (b) than how fast one runs
(b) the nation will enter (c) than one’s speed (d) lain off (e)

ww
(c) to enter the nation
(d) will the nation enter
(e) None of these
27.
None of these
She expects me to type the letter in five minutes . ......... is
impossible.
20.
(a) must come
(c) will come
w.E
I hope she . ............. .
(b) should come
(d) must be coming
(a) that
(c) what
(e) None of these
(b) which
(d) but

21.
(e) None of these
asy
An increase in population, without an increase in
economic level, .............. result in a lower standard of
28. If only you had spoken clearly, you (a)
would not be misunderstood (b) would not
living.
(a) tends to (b) tending to En have been misunderstanding. (c) would
not have been misunderstood. (d) would

22.
(c) will tend
(e) None of these
(d) tends

. ............. as President, a candidate must win a majority of


gin
29.
not have misunderstood.
(e) None of these
It is high time that he .......... himself.
votes.
(a) Elected (b) To be elected eer
(a) had reformed
(c) has to reform
(b) will reform
(d) reformed

23.
(c) Having elected (d) Electing (e)
None of these
Encounters between people from different countries can
30.
(e) None of these
ing
.........., a bus almost ran over him.

result in misunderstandings . ............. different conceptions


about space.
(a) Running across the road
(b) Running on the road
(c) When he ran across the road .ne
(a) because they (b) is because they
(c) is because their (d) of their
(e) None of these
(d) When he was running through the road.
(e) None of these t

16. a b c d e 17. a b c d e 18. a b c d e 19. a b c d e 20. a b c d e


RESPONSE 21. a b c d e 22. a b c d e 23. a b c d e 24. a b c d e 25. a b c d e
GRID 26. a c d e 27. a c d e 28. a c d e 29. a c d e 30. a c d e
b b b b b

Downloaded From : www.EasyEngineering.net


Downloaded From : www.EasyEngineering.net

Sentence
Completion - II 59
Max. Marks : 35 No. of Qs. 35 Time : 20 min. Date : ........./......../. ..............
DIRECTIONS: Select the correct word or phrase to complete a 8. His interest in the study of human behaviour is indeed very
grammatical and idiomatic sentence. .......
(a) strong (b) large
1. These essays are intellectually . ............. and represent (c) broad (d) vast
various levels of complexity. (e) deep
(a) revealing (b) modern 9. The improvement made by changes in the system was .... and

ww
(c) superior
(e) persistent 2.
(d) demanding
The soldiers are instructed to .
................................................................... restraint and handle
id not warrant the large expenses.
(a) large
(c) minute
(e) uncertain
(b) small
(d) marginal
the situation peacefully.
(a) exercise
(c) enforce
w.E (b) control
(d) remain
10. He is too . .... to be deceived easily.
(a) strong
(c) kind
(b) modern
(d) honest

3.
(e) None of these
asy
Since one connot read every book, one should be content
(e) intelligent 11. There has been a .
......................................... lack of efficiency in all the crucial areas
of the working of Public Sector Undertakings.
with making a . ...... selection.
(a) normal
(c) sample
(b) standard
(d) moderate En (a) positive
(c) conspicuous
(b) surprising
(d) stimulative

(e) judicious 4. Some people .


...................................... themselves into believing that they are gin (e) insignificant 12. I will be leaving for Delhi tonight
and ........................................................................ to return by this
week end.
indispensable to the organisation they work for.
(a) keep (b) fool
(a) waiting
(c) going
(e) making eer (b) plan
(d) likely
(c) force
(e) delude 5.
progress when her
(d) denigrate
How do you expect that country to (a) trial (b) ing
13. Ravi’s behaviour is worthy of ....... by all the youngsters.
emulation

government is corrupt, ..... and still lergely feudal?


(a) devalued (b) dwinding
(c) following
(e) experiment 14. The only way to .
.ne
(d) exploration

................................................ the country from the evils of

6.
(c) demobilised
(e) None of these
(d) demeaning

The truck was ...... the trafic and the policeman asked the
driver to move off.
communalism is to enforce the rule of law.
(a) eradicate
(c) extricate
(b) mobilise
(d) purge
(e) strengthen 15. Even at the risk of economic loss, he .
t
......................................................................... refused to take the
(a) failing (b) obstructing beaten track.
(c) obviating (d) hiding (a) repeatedly (b) stead fastly
(e) disturbing (c) regularly (d) continuously
7. The paternalistic attitude is so ingrained to the (e) None of these
managements that they will have to ........ try to change it. 16. On his sudden demise, may emotions were so complicated
that it was ......... how I felt.
(a) casually (b) slowly
(a) unreasonable (b) impossible
(c) subtly (d) inadvertently (c) inexplicable (d) unimaginable
(e) None of these (e) None of these

1. a b c d e 2. a b c d e 3. a b c d e 4. a b c d e 5. a b c d e
RESPONSE 6. a b c d e 7. a b c d e 8. a b c d e 9. a b c d e 10. a b c d e
GRID 11. a b c d e 12. a b c d e 13. a b c d e 14. a b c d e 15. a b c d e
16. a b c d e

Downloaded From : www.EasyEngineering.net


Downloaded From : www.EasyEngineering.net

126 SPEED TEST 59


17. Two of the fugitives managed to remain free by adeptly 27. I will write a letter to you tentatively ....... the dates of the
avoiding the ....... of the police. programme.
(a) torture (b) pursuit (a) involving (b) urging
(c) discovery (d) following (c) guiding (d) indicating
(e) None of these (e) propagating 28. Contemporary economic develop-
18. Experts fail to understand the ..... behind the decision to
ment differs .................................................................... form the
move coal by road when there is enough rail capacity in this
Industrial Revolution of the 19th century.
sector
(a) ideology (b) judgement (a) naturally (b) usually
(c) rationale (d) politics (c) literally (d) specially
(e) logistics 19. Automobile manufacturers are re- (e) markedly 29. The word gharana points to the .
viving up to launch a .................................................................... concepts of stylistic
compaign designed to increase consumer ........ about the individuality and handing down of tradition within family
new emmission control. confines.
(a) production (b) education (a) joint (b) conflicting
(c) capacity (d) knowledge (c) dual (d) contradictory
(e) awareness

ww
20. His logic ......... everyone, including the expects.
(a) teased
(c) surprised
(b) defied
(d) confounded
(e) extraordinary
30. It ws the help he got from his friends which . ..... him through
the tragedy.
(e) overwhelmed
w.E
21. The factory went into a state of suspended ....... today with
all its workers on strike.
(a) helped
(c) perked
(b) boosted
(d) supported
(e) sustained 31. The criminals managed to escape
(a) symbiosis
(c) ways
(e) mortification asy
(b) animation
(d) condition
from the prison even
through two armed policemen were ....... vigil over them.
(a) taking (b) putting
22. It is not fair to cast . ...... on honest and innocent persons.
(a) aspiration (b) aspersions
En (c) guarding
(e) looking
(d) keeping

(c) inspiration
(e) None of these
23. You must ....... your career with all seriousness.
(d) adulation

gin
32. The speaker did not properly use the time as he went on .....
on one point alone.
(a) direct
(c) follow
(b) complete
(d) manage
(a) dilating

eer
(c) deliberating
(b) devoting
(d) diluting
(e) distributing 33. Ravi had to drop his plan of going to
(e) pursue 24. The villagers .
......................................... the death of their leader by keeping all
the shops closed.
picnic as he had certain
ing
..... to meet during that period.
(a) announced
(c) mourned
(e) None of these
(b) protested
(d) consoled
(a) preparations
(c) urgencies
(e) commitments .ne
(b) observations
(d) transactions

25. These medicines are ........ for curing cold.


(a) proper
(c) effective
(e) powerful 26. The poor ones continue to .
(b) real
(d) capable
34. Even in today’s modern society, people ....... god to bring
rains.
(a) provoke (b) evoke
t
(c) propitiate (d) superimpose
................................................................. out a living inspite of
(e) None of these
economic liberalisation in that country.
35. The good is often ....... with their bones.
(a) find (b) go
(c) bring (d) manage (a) buried (b) covered
(e) None of these (c) exhumed (d) interred
(e) fleshed

17. a b c d e 18. a b c d e 19. a b c d e 20. a b c d e 21. a b c d e


RESPONSE 22. a b c d e 23. a b c d e 24. a b c d e 25. a b c d e 26. a b c d e
GRID 27. a b c d e 28. a b c d e 29. a b c d e 30. a b c d e 31. a b c d e
32. a b c d e 33. a b c d e 34. a b c d e 35. a b c d e

Downloaded From : www.EasyEngineering.net


Downloaded From : www.EasyEngineering.net

Similar
Substitution 60
Max. Marks : 25 No. of Qs. 25 Time : 20 min. Date : ........./......../. ..............

DIRECTIONS : In each of these questions, two sentences (I) 6. I. The truck stopped ___________.
and (II) are given. Each sentence has a blank in it. Five words (a), II. We take a ___________ walk every day.
(b), (c), (d) and (e) are suggested. Out of these, only one fits at (a) suddenly (b) long
both the places in the context of each sentence. Option of that (c) short (d) distant
word is the answer. (e) near
7. I. I got the grains ___________ in the machine.
1. I.
II.
ww
He is__________ with whatever little he has.
They kept the ____________ of the communication
a secret.
II.
(a)
I do not have any ___________ for doubting him.
done (b) basis
(a)
(c)
(e)
happy
gist
sense
w.E (b) matter
(d) content
8.
(c)
(e)
I.
ground
tune
(d) crushed

We were asked to design a ___________ of the dam.

2. I.
involved in this activity. asy
It is hard to believe the ___________ of operations
II.
(a)
(c)
This Institute is a ___________ of modern thinking.
picture
function
(b) type
(d) fabric
II. The map is drawn to a ___________ of 1 inch to 50
km.
En 9.
(e)
I.
model
Keep a ___________ grip on the railing.
(a)
(c)
magnitude
scale
(b) size
(d) proportion
gin II.
(a)
He was ___________ asleep.
fast (b) firm

3.
(e)
I.
II.
significance
Heavy snow did ___________ the rescue efforts.
The food was kept in a ___________. 10.
(c)
(e)
I.
deep
sure
eer (d) strong

He asked me to ___________ over the fence.


(a)
(c)
delay
basket
(b) bundle
(d) hamper
II.
(a) vault ing
We should keep the valuables in the ___________.
(b) cross

4.
(e)
I.
holder
They left ___________ after breakfast.
(c)
(e)
safe
locker
(d) tie
.ne
II.
(a)
(c)
(e)
It is difficult to find a ___________ person for this job.
right
suitable
soon
(b) immediately
(d) best
11. I.
II.
(a)
(c)
He has now become a ___________ to reckon with.

model
coercion
(b) force
(d) name
t
It is better not to use ___________ to prove one’s point.

5. I. He would always do ___________ was told by his (e) influence


superiors. 12. I. He tried his ___________ best to score distinction in
II. He appeared on stage ___________ a narrator of the this exam.
drama. II. It pays to keep ___________ head in an emergency.
(a) cool (b) utmost
(a) as (b) what
(c) very (d) possible
(c) about (d) whatever
(e) level
(e) always

1. a b c d e 2. a b c d e 3. a b c d e 4. a b c d e 5. a b c d e
RESPONSE 6. a b c d e 7. a b c d e 8. a b c d e 9. a b c d e 10. a b c d e
GRID
11. a b c d e 12. a b c d e

Downloaded From : www.EasyEngineering.net


Downloaded From : www.EasyEngineering.net

128 SPEED TEST 60


13. I. The system is working with ___________ to getting (a) edition (b) volume
things done. (c) channel (d) frequency
II. ___________ must be commanded and not (e) pitch
demanded. 20. I. It helps to rinse one’s mouth early morning with a
(a) status (b) relation
___________ of salt and water.
(c) attitude (d) respect
II. You can always refer to this reference material to find
(e) honour
the ___________ to these problems.
14. I. There is a complaint against him that he ___________
the mistakes of his juniors. (a) mixture (b) answers
II. A good thing about this house is that it ___________ (c) liquid (d) fix
the sea. (e) solution
(a) ignores (b) promotes 21. I. Our office decided to organize a party for the
(c) examines (d) overlooks ……………….. couple.
(e) faces II. She …………… him in conversation to while away
15. I. Out of the total loans ___________ by the bank, the some time.

II.
(a)
ww
largest share was for infrastructure.
The trees are ___________ throughout the area.
disbursed (b) covered
(a)
(c)
new
pledged
(b) engaged
(d) held

16.
(c)
(e)
I.
distributed
extended w.E (d) spanned

Boats take more time going against the ___________


22.
(e)
I.
encountered
It is required that you fill out these two ………………..
to register for the job.

II.
of the river.
asy
She keeps herself abreast of ___________ events.
II. This subject has many practical ……………........ in day
to day life.
(a)
(c)
low
water
(b) latest
(d) all
En (a)
(c)
forms
relevance
(b) applications
(d) statements (e) views

17.
(e)
I.
II.
current
While trying to open the door, the ___________ broke.
It is not difficult to ___________ tricky situations. gin
23. I.
II.
The next ……………….. of this case is after two months.
After the accident her ……………….. has been affected.

(a)
(c)
handle
bracket
(b) knob
(d) overcome
(a)
(c)
(e)
dates

eer
evidence
hearing
(b) balance
(d) health

18.
(e)
I.
win
This course teaches you not to ___________ to 24. I.
ing
As a last ………………. we had to accept these terms
and conditions.
II.
temptations.
We hope to increase our ___________ of rice this
year.
II.
(a) resort (b) step .ne
This place has become a good tourist ………… now.

(a)
(c)
(e)
succumb
yield
submit
(b) produce
(d) share 25.
(c)
I.
attraction (d) spot
The organization decided on an hourly minimum
…………………….. of ` 35.
t
(e) means

19. I. When you play your radio at high ___________ it II. Will he ……………a war on these fronts to improve
disturbs others. things?
II. We have just received a latest ___________ of this
(a) pay (b) declare (c) pose
encyclopedia.
(d) wage (e) campaign

13. a b c d e 14. a b c d e 15. a b c d e 16. a b c d e 17. a b c d e


RESPONSE 18. a b c d e 19. a b c d e 20. a b c d e 21. a b c d e 22. a b c d e
GRID 23. a b c d e 24. a b c d e 25. a b c d e

Downloaded From : www.EasyEngineering.net


Downloaded From : www.EasyEngineering.net

Correct Usage of
Preposition 61
Max. Marks : 35 No. of Qs. 35 Time : 20 min. Date : ........./......../. ..............

DIRECTIONS : Select the correct word or phrase to complete a (a) with, than (b) within, to
grammatical sentence. In case of more than one blank, the different (c) within, against (d) on, from
words given in the options shall fill in the corresponding order. (e) None of these
1. The patient was cheered . ....... by the news that she was 9. I doubt whether the accommodation in that place is
likely to be discharged in a day or two. adequate........our needs.
(a) on

ww
(c) out
(e) None of these
(b) up
(d) down
(a) according to
(c) for
(e) None of these
(b) with
(d) against

2.
.......them.
(a) on with w.E
The thieves had driven ten miles before the police caught

(b) upon
10. The teacher warned the students to desist ........making noise.
(a) from
(c) with
(b) about
(d) by

3.
(c) up to
(e) None of these
asy
(d) up with

The way he is currying favour ........ his rich neighbour is


11.
(e) None of these
It was customary ........ devotees going to that temple to take
a bath in the nearby tank first.
sickening.
(a) of (b) with
En (a) with
(c) for
(b) on
(d) about

4.
(c) for
(e) None of these
(d) to

Because of his distrust........ every one, he could never gain


12. gin
(e) None of these
Absorbed .
any good friend.
(a) by (b) of
(a)
(c) with, eer
his own thought, he paid scant attention to
what was happening ........ him.
by, toabout
(b) in, around
(d) of, besides

5.
(c) with
(e) None of these
(d) in

The new Twenty Point Programme is designed to be a


13.
(e) None of these
ing
Though young, he proved a worthy rival ........ his opponent
renewed assault . ....... poverty.
(a) for (b) into
who had much more experience.
(a) to
(c) for .ne
(b) against
(d) with

6.
(c) on
(e) None of these
(d) upon

If you have a good project but are lacking .finance, you


should enter into partnership ........an affluent person.
14.
(e) None of these
t
He was of a charitable disposition, but did not like a number
of his relatives trying to live ........ him without trying to earn
their living.
(a) of, with (b) for, with
(a) with (b) near
(c) in, with (d) in, of
(c) off (d) through
(e) None of these
(e) None of these
7. He was operated ........ an abscess in his leg.
15. The by-election, ........ closely fought, resulted ........ a surprise
(a) for (b) against
landslide for the ruling party.
(c) on for (d) on against
(a) if, at (b) since, with
(e) None of these
(c) whether, in (d) though, in
8. Living a simple life .
(e) None of these
....................... one’s won resources is preferable
........ leading a luxurious life on borrowed funds.

1. a b c d e 2. a b c d e 3. a b c d e 4. a b c d e 5. a b c d e
RESPONSE
6. a b c d e 7. a b c d e 8. a b c d e 9. a b c d e 10. a b c d e
GRID 11. a b c d e 12. a b c d e 13. a b c d e 14. a b c d e 15. a b c d e

Downloaded From : www.EasyEngineering.net


Downloaded From : www.EasyEngineering.net

130 SPEED TEST 61


16. You cannot be too sensitive. ...... criticism, if you have chosen 26. His claims of close acquaintance. ...... the high and the mighty
a political career. are unbelievable.
(a) about (b) with (a) about (b) towards
(c) for (d) to (c) with (d) from
(e) None of these (e) None of these
17. . ...... we are good friends his views differ ........mine on many 27. Compared. ......China, India’s progress in the agricultural field
issues. has been commendable.
(a) Despite, with (b) Because, off (a) against (b) over
(c) Although, from (d) Nevertheless, form
(c) towards (d) to
(e) None of these
(e) None of these
18. A peculiar custom prevailing.........Toads is the sacrifice of
28. The court found him guilty and imposed ........ him a fine of
buffaloes on ceremonial occasions.
Rs.500.
(a) over (b) with
(c) by (d) among (a) to (b) for
(e) None of these (c) upon (d) against
19. However poor one may be, one can be happy only if one has (e) None of these

ww
the right attitude to life; happiness lies........ contentment.
(a) through
(c) with
(b) in
(d) over
29. It is believed that hypertension is most often the cause ........
heart attack.
(a) towards (b) with

20.
(e) None of these
w.E
Being himself very quick ........ arithmetical calculations, he
did not need a calculator. 30.
(c) in
(e) None of these
(d) of

Ravi was . ...... Nagpur ........ 2nd Jan, 67 ........4 in the morning.
(a) ahout
(c) over
(e) None of these
(b) for
(d) in
asy (a) at, on, at
(c) in, in, about
(e) None of these
(b) in, on, at
(d) at, at, at

21.
had a clear conscience and believed in the correctness of En
Though accused of partiality........ his home team, the umpire 31. . ............. a very long time this city has been prosperous.
(a) Since (b) For
his decisions.
(a) with
(c) for
(b) towards
(d) against
gin (c) From
(e) None of these
(d) Till

22.
(e) None of these
Though I would not recommend it, I have no objection........
32.
grief.
(a) by eer
The mother of the dead child was overwhelmed .............

(b) with
your going to that movie.
(a) for
(c) to
(b) against
(d) upon
(c) from
(e) None of these ing (d) for

23.
(e) None of these
Do not force me to ........ you on this issue; I am not at all
33.
(a) on
.ne
There is no use discussing . ............. prohibition.
(b) about
convinced.
(a) agree upon
(c) join over
(e) None of these
(b) concur with
(d) equate with 34.
(c) of
(e) None of these
(d) for

t
.............. business, a merger is a combination of two or more
corporations under one management.
24. The court acquited him ........ all the charges. (a) At (b) In
(a) from (b) against (c) The (d) On
(c) for (d) of (e) None of these
(e) None of these 35. How is life enhanced? A beautiful passage from Tagore
25. There is no point in counting. ....... his support he is comes ............. mind.
notoriously undependable. (a) in (b) to
(a) for (b) on (c) from (d) out
(c) with (d) through
(e) None of these
(e) None of these

16. a b c d e 17. a b c d e 18. a b c d e 19. a b c d e 20. a b c d e


RESPONSE 21. a b c d e 22. a b c d e 23. a b c d e 24. a b c d e 25. a b c d e
GRID 26. a b c d e 27. a b c d e 28. a b c d e 29. a b c d e 30. a b c d e
31. a b c d e 32. a b c d e 33. a b c d e 34. a b c d e 35. a b c d e

Downloaded From : www.EasyEngineering.net


Downloaded From : www.EasyEngineering.net

Sentence
Improvement 62
Max. Marks : 30 No. of Qs. 30 Time : 20 min. Date : ........./......../. ..............

DIRECTIONS (Qs.1-20) : Which of the following phrases (a), 8. Belonged to this cadre, you are eligible for facilities such as
(b), (c) and (d) given below each sentences should replace the free air travel and accommodation.
phrase printed in bold in the sentence to make it grammatically (a) Since you belong to (b) Whoever belongs
correct? If the sentence is correct as it is given and ‘No correction (c) For belonging to (d) To belong in
is required’, mark (e) as the answer. (e) No correction required
1. Starting out my own business at this time would affect the 9. The bank has hired a consultant who will look into any issues

ww
financial stability of my family.
(a) Starting up my (b) For starting with
(c) To start out mine (d) By starting my
which arise during the merger.
(a) is looking over
(c) will look out
(b) will be looked after
(d) looks down on

2.
w.E
(e) No correction required
Use a tactic for mixing the inferior with good quality rice is
dishonest and you will lose your license.
10.
(e) No correction required
I had severe doubts about if I successfully run a company,
but my father encouraged me.
(a) if I am successful in (b) how should I successfully
(a) Using tacti as
(c) To use tactics
(e) No correction required asy
(b) Using a tactic like
(d) Used to tactics like
(c) whether I would successfully
(d) that I would succeed to
(e) No correction required
3. The company will invest more six hundred crores in the
next five years to expand its operations in Britain.
En 11. As it was a dark and stormy night, Lata was too scared to go
home alone.
(a) will future invest (b) has invested more than
(c) have invested over (d) will be invested above
(e) No correction required gin
(a) very scary to
(c) as scared to
(e) No correction required
(b) much scared to
(d) to scared too

4. Several of our projects have delayed because the equipment


we ordered was delivered late.
12.
eer
Since it was her engagement party, Riya was dress to kill.
(a) dresses to kill (b) dressed to kill

ing
(a) have been delayed when (c) dressed to killings (d) dressing to killed
(b) delayed because of (e) No correction required
(c) are delayed since (d) were delayed with 13. Ramesh worries endlessly about his son’s future as he was

5.
(e) No correction required
The committee has ruled out the possible raising taxes for
this financial year.
so poor in studies.
(a) worry endless
(c) worried endlessly .ne
(b) worried endless
(d) worries endless

6.
(a) possibly raised (b) possible rise of
(c) possibility to raise (d) possibility of raising
(e) No correction required
The company has set up a foundation which helps students
14.
(e) No correction required
Now that the actual criminal had been caught, Kunal was
happy that he was finally let of the hook.
(a) off the hook (b) of the hookings
t
(c) off the hooks (d) of the hooks
who do not have the necessary funds to study ahead. (e) No correction required
(a) further to study (b) of studying more 15. The little boy appeared all of a sudden out of nowhere and
(c) to study onward (d) for higher studies take everyone by surprise.
(e) No correction required (a) took everyone as surprised
7. If this land is used to cultivate crops it will be additionally (b) take everyone with surprised
source of income for the villagers. (c) took everyone by surprises
(a) a source of additional (b) an additionally source (d) took everyone by surprise
(c) an additional source (d) additionally the source (e) No correction required
(e) No correction required

1. a b c d e 2. a b c d e 3. a b c d e 4. a b c d e 5. a b c d e
RESPONSE 6. a b c d e 7. a b c d e 8. a b c d e 9. a b c d e 10. a b c d e
GRID
11. a b c d e 12. a b c d e 13. a b c d e 14. a b c d e 15. a b c d e

Downloaded From : www.EasyEngineering.net


Downloaded From : www.EasyEngineering.net

132 SPEED TEST 62


16. A young and successful executive was travelling down a 23. The course of events made it necessary for Joseph to start working.
neighbourhood street, going a bit to fast in his new car. (a) events that were planned.
(a) a bit too fastly (b) a bit as fast (b) long list of future events.
(c) a bit to fastly (d) a bit too fast (c) A succession of unexpected events.
(e) No correction required (d) nature of events that followed after Joseph joined work.
17. All she could think about was the beautiful dress and how (e) None of these
she could earn enough money to buy it. 24. The new law on “Right to Food Safety” will come into force
(a) All she can think (b) All she could thought next month.
(c) All she can thought (d) All she can thinking (a) be forced upon the people.
(e) No correction required (b) be associated from next month onwards.
18. He told his employer of his plans to leave the business to (c) be implemented next month.
lead a more leisure life. (d) be withdrawn next month.
(a) more leisurely life (b) many leisurely life (e) be widely rejected next month.
(c) many leisured life (d) more leisurely live 25. When the girl wanted to stay out past midnight, her father
(e) No correction required put his foot down.
19. Padma could convince anyone with her talks as she had the (a) gave in to her request

ww
gift of the gabbing.
(a) gifting of the gabbing (b) gift of the gab
(c) gifting of the gab (d) gift of the gab
(b) walked away disapprovingly.
(c) obstructed her from leaving the house.
(d) requested her to be home on time.

w.E
(e) No correction required
20. For countries undergoing a recession, large cuts in public
spending seem to be the ordering of the day.
(a) be the ordering of days
(e) None of these
26. In all likelihood the missing boy has run away to the forest
(a) with good intentions (b) there's no chance
(c) without doubt (d) in most probability
(b) being the order of the day
(c) be the order of the day asy (e) None of these
27. The parents were completely in the dark concerning their
(d) being the ordering of days
(e) No correction required
En daughter's plans.
(a) ignorant about
(c) pretending to be unaware
(b) ashamed of
DIRECTIONS (Qs.21-30) : In each of the following sentences,
an idiomatic expression or a proverb is highlighted. Select the
alternative which best describes its use in the sentence. gin(d) unhappy about (e) None of these
28. I am in touch with the police, and they will be here in ten minutes.
21. The team put their plan into execution the very next day.
(a) proposed a plan.
eer
(a) in communication with
(b) in close proximity with
(c) in good terms with (d) familiar with
(b) discussed their plan.
(c) started thinking about a plan
(d) started carrying out their plan.
(e) None of these
ing
29. I stumbled upon some interesting old letters in my
(e) None of these
22. Mrs. Nayak opened the discussion on the “alarming rate of
Grandfather's desk.
(a) deliberately went through
(b) surveyed .ne
(c) tripped over
poverty in India”.
(a) started the discussion.
(b) gave her opinion in the discussion.
(c) did not agree on the discussion.
(d) welcomed the people to the discussion.
(d) discovered by chance (e) None of these
30. The secretary made an entry of the arrangement.
(a) initiated discussion (b) made a record
(c) brought notice (d) showed approval
t
(e) None of these (e) None of these

16. a b c d e 17. a b c d e 18. a b c d e 19. a b c d e 20. a b c d e


RESPONSE
21. a b c d e 22. a b c d e 23. a b c d e 24. a b c d e 25. a b c d e
GRID 26. a c d e 27. a c d e 28. a c d e 29. a c d e 30. a c d e
b b b b b

Downloaded From : www.EasyEngineering.net


Downloaded From : www.EasyEngineering.net

Spotting the
Errors - I 63
Max. Marks : 41 No. of Qs. 41 Time : 25 min. Date : ........./......../. ..............

DIRECTIONS (Qs.1-41) : Read each sentence to find out whether 10. The agreement on (a)/ which all of us have (b)/ worked so
there is any grammatical error or idiomatic error in it. The error, if hard will (c)/ be sign tomorrow (d). No error (e).
any, will be in one part of the sentence. The number of that part 11. Nuclear waste will still being (a)/ radioactive even after twenty
is the answer. If there is no error, the answer is (e). (Ignore errors thousand years, (b)/ so it must be disposed (c)/ of very
of punctuation, if any.) carefully. (d)/ No error (e)
1.

2.
ww
His proposal had (a) / to be send to (b) / the President of
the company (c) / for her approval (d). No error (e).
Each tuesday evening we visited (a) / the farmers in the
12.

13.
My friend lived at the top (a)/ of an old house (b)/ which attic
had been (c)/ converted into a flat. (d)/ No error (e)
A public safety advertising (a)/ campaign in Russia (b)/ hope

w.E
area (b) / and held a meeting (c) / to discuss the problems
they faced (c). No error (e).
to draw attention (c)/ of pedestrians crossing the
road.
3.

asy
Though our training facilities (a) / are limited only a (b) /
few employees have been (c) / selected for training (d). No
error (e).
14.
(d)/ No error (e)
A cash prize was (a)/ award to the most (b)/ successful salesman
of the year (c)/ by the President of the company. (d)/ No error
4. During the interview (a) / the panel asked me (b) / several
technical questions (c) / and I answered all of it (d). No En 15.
(e)
The Renaissance was (a)/ a time to ‘re-awakening’ (b)/ in both

5.
error (e).
He decided to work for (a) / an NGO, but most of his (b) / gin
16.
the arts (c)/ and the sciences. (4)/ No error (e)
In times of crisis, (a) / the Bhagavad Gita gives light (b)/ and
classmates opted for high paid (c) / jobs in multinational
companies (d). No error (e). eer
guide to the mind tortured by doubt (c)/ and torn by conflict
of duties. (d)/ No Error (e)
6. It is necessarily to maintain (a)/ a record of all transactions
(b) / in case the auditors (c)/ want to see it. (d)/ No error (e).
17.
ing
It was not easy for late Raja Ram Mohan Roy (a)/ to root out
the custom of sati (b) / because a majority of (c)/ the educated
7. Very few young trainees (a)/ willingly undertake (b)/ a
posting to a branch (c)/ located in a rural area (d)/ No error 18.
.ne
class does not support him. (d) / No Error (e)
Deplete of the Ozone layer (a) / and the greenhouse effect (b)

8.

9.
(e).
He has travelled (a)/ all over the world (b)/ yet he speaks
(c)/ several languages fluently (d)/. No error (e).
A successful company is (a)/ any that makes a good (b)/
19.
t
/ are two long-term effects (c)/of air pollution. (d)/ No Error (e)
Most of the people which (a)/ have been victims (b) / of extreme
violence (c)/ are too frightened to report it to the police. (d)/
No Error (e)
profit and provides (c)/ high returns to its shareholders (d). 20. The doctor helps (a)/ to reducing human suffering (b)/by
No error (e). curing diseases (c)/ and improving health. (d)/ No Error (e)

1. a b c d e 2. a b c d e 3. a b c d e 4. a b c d e 5. a b c d e
RESPONSE 6. a b c d e 7. a b c d e 8. a b c d e 9. a b c d e 10. a b c d e
GRID 11. a b c d e 12. a b c d e 13. a b c d e 14. a b c d e 15. a b c d e
16. a b c d e 17. a b c d e 18. a b c d e 19. a b c d e 20. a b c d e

Downloaded From : www.EasyEngineering.net


Downloaded From : www.EasyEngineering.net

134 SPEED TEST 63


21. The shepherd counted (a) / his sheep and found (b) / that 32. We may have to await for (a) / a new political revival (b) / to
one of (c) / them is missing. (d) / No Error (e) eradicate the (c) / corruption from our economy. (d) / No
22. The teacher were (a) / impressed by her performance (b) / Error (e)
and asked her to (c) / participate in the competition. (d) No 33. When she was (a) / in jail (b) / she was debarred to send
Error (e) (c) / a letter even to her son. (d) / No Error (e)
23. She asked her (a) / son for help her (b) / find a place to bury 34. Despite of the best efforts (a) / put by the doctors (b) / the
(c) / the gold ornaments (d) No Error (e) condition of the patient (c) / is deteriorating from bad to
24. The painter was (a) / ask to paint a (b) / picture of the king, worse. (d) / No Error (e)
(c) / sitting on his throne (d) No Error (e) 35. The militant yielded for (a) / the temptation and fell (b) / into
25. The story was (a) / about how an (b) / intelligent man had the trap (c) / of police. (d) / No Error (e)
saving (c) / himself from being robbed (d) No Error (e) 36. Many people in India (a) / are dying from hunger (b) / but
26. The decline of her moral (a) / was caused by a lot of (b) / government seems (c) / to be ignorant of such crude fact. (d)
factors that were once (c) / fascinating to her. (d) / No Error / No Error (e)
(e) 37. In difficult time (a) she prefers keeping her counsel (b) rather

ww
27. He took me to a restaurant (a) / and ordered for two cups
(b) / of cold coffee (c) / which the waiter brought in an hour.
(d) / No Error (e) 38.
than wandering (c) / here and there for relief. (d) / No Error
(e)
The persons who are (a) / suffering from diabetes are (b) /

w.E
28. There are some animals (a) / that can live (b) / both in water
an land (c) / without any difficulty. (d) / No Error (e)
advised to substitute (c) / saccharine by sugar. (d) / No Error
(e)

asy
29. During his tour (a) / to the south (b) he visited not only to
Chennai (c) / but also Karnataka. (d) / No Error (e)
30. The President Mr. Kalam (a) / is much sought after (b) / by
39.

40.
He always says (a) / that he prefers to go (b) / home to stay
in (c) / a hotel at night. (d) / No Error (e)
Hardly had we settled down (a) / for the rest (b) / when we
school students and (c) / is invited for many functions.
(d) / No Error (e) En were startled by the (c) / strange sound of trumpets. (d) / No
Error (e)
31. His mother is not well (a) / but he (b) / does not look for her
(c) / properly. (d) / No Error (e) gin
41. He was able to (a) / free himself with (b) / the debts by (c) /
working day and night. (d) / No Error (e)

eer
ing
.ne
t

21. a b c d e 22. a b c d e 23. a b c d e 24. a b c d e 25. a b c d e


26. a b c d e 27. a b c d e 28. a b c d e 29. a b c d e 30. a b c d e
RESPONSE 31. a c d e 32. a c d e 33. a c d e 34. a c d e 35. a c d e
b b b b b
GRID 36. a b c d e 37. a b c d e 38. a b c d e 39. a b c d e 40. a b c d e
41. a b c d e

Downloaded From : www.EasyEngineering.net


Downloaded From : www.EasyEngineering.net

Spotting the
Errors - II
64
Max. Marks : 40 No. of Qs. 40 Time : 25 min. Date : ........./......../. ..............

DIRECTIONS : Read each sentence to find out whether there is 11. All three products help wean smokers (a) / from cigarettes by
any error in it. The error, if any, will be in one part of the sentence. providing small doses (b) / of nicotine designed to replace
The number of this part is the answer. If there is no error, the the basic level (c) / of average smoker's day's nicotine
answer is (e). consumption. (d) No error (e)

1. The banker’s association (a) / has submitted a memorandum 12. The main lesson is that (a) / the mosquito eradication

(e)
ww
(b) / for the fulfilment of (c) / their demands. (d) / No Error

13.
campaigns (b) / such as the one the French conducted (c) /
needs to be followed up. ( d) / No error (e)
By arresting the local criminals (a) / and encouraging good
2.

3.
w.E
Five quintals of wooden coal (a) / are (b) / his annual
requirement (c) / for the unit. (d) / No Error (e)
Dickens have (a) / vehemently criticised (b) / the philosophy
14.
people, (b) / we can end (c) / hostilities of that area (d) / No
error (e)
We admired thre way(a) / he had completed all his work (b) /

4.
(c) in ‘Hard Times’. (d) / No Error (e)

asy
All his money (a) / is spent (b) / and all his (c) / hopes
and appreciation the method (c) / adopted by him. (d) / No
error

5.
ruined. (d) / No Error (e)

En
This rule may (a) / and ought to be (b) / disregarded for (c)
15. I was being astonished (a) / when I heard that (b) / he had left
the country (c) / without informing anyone of us . (d) / No.

6.
/ the time being. (d) / No Error (e)
Why come people don't get (a) / what they deserve (b) / gin
16.
error (e)
The Head of the Department, along with his colleagues (a) /
and why others get what they don't deserve (c) / is a matter
decided by luck. (d)/No error(e)
eer
are coming to attend (b) / the conference which is (c) /
scheduled this afternoon. (d) / No error (e)
7. The committee is thankful to Mr. Roy (a) / for preparing not
only the main report (b) / but also for preparing (c) the
agenda notes and minutes. (d) / No error (e)
17.

ing
Govind loved his Guru immensely (a) / and gave him fullest
loyalty, (b) / yet he had his own (c) / independent way of
thinking (d) / No error (e)

8. In order to save petrol (a ) / motorists must have to (b) / be


very cautious (c) / while driving along the highways. (d) /
18.
.ne
In a very harsh tone, (a) / he shouted at his servants (b) / and
told them that (c) / he does not neEid their services (d) ./ No

9.
No error (e)
No country can long endure (a) / if its foundations (b) /
were not laid deep (c) / in the material prosperity. (d) / No
19.
error (e)
t
This is an important difference in (a) / that it marks the first
move towards (b) / an institution for money - earning
error (e) proposition. (d) / No error (e)

10. Due to certain inevitable circumstances (a) / the scheduled 20. The ultimate problem of physics (a)/ is to reduce matter by
programme had to be (b) / post poned indefinite (c) / but analysis (b) / to its lowest condition of divisibility (c) / No.error
(d)
the members could not be informed. (d) / No error (e)

1. a b c d e 2. a b c d e 3. a b c d e 4. a b c d e 5. a b c d e
RESPONSE 6. a b c d e 7. a b c d e 8. a b c d e 9. a b c d e 10. a b c d e
GRID 11. a b c d e 12. a b c d e 13. a b c d e 14. a b c d e 15. a b c d e
16. a b c d e 17. a b c d e 18. a b c d e 19. a b c d e 20. a b c d e

Downloaded From : www.EasyEngineering.net


Downloaded From : www.EasyEngineering.net

136 SPEED TEST 64


21. It was astonished (a) / to find that I scarcely had (b) / enough 32. Whether this happens, and whether the BIFR will (a) / once
money (c) / to pay the bills. (d) / No error (e) again reconsiger the Sirmour package (b) / are questions
those will be (c) / answered in the comming months. (d) / No
22. We can not handle (a)./ this complicated case to day (b) /
error (e)
unless full details are not given (c) / to us by now. (d) / No
error (e) 33. We now look forward for (a) / some great achievements (b) /
which to some extent (c) / can restore the country's prestige
23. Even after worked in the office (a) / for as many as fifteen
once again (d) / No error (e)
years, (b) / he still does not understand (c) / the basic
objectives of the work. (d) / No error (e) 34. Honesty and integrity are (a) / the qualityes which cannot
be (b) / done away with (c) / and hence assume a lot of
24. Neither the earthquake (a) / nor the subsequent fire ( b) /
importance (d) / No error (e)
was able to dampen (c) / the spirit of the residents, (d) / No
error (e) 35. The foreign funds are cheaper than those (a) / available the
domestic market and (b) / the company is competent that (c)
25. Our school is making (a) / every possible effort (b) / to provide
/ it will soon touch its earlier annual turnover, (d) / No error
best facilities (c) / land person attention for each child (d) /
(e)
No error (e)
26.
ww
Our neighbours had repeated (a) / the same illogical sequence
of activites (b) / if we had not brought the (c) /facts to their
notice. ( d) / No error (e)
36. Honesty, integrity and being intelligent (a) / are the qualities
which (b) / we look for when (c) / we interview applicants (d)
/ No error (e)

27.
w.E
This has forced them to focus at (a) I how to reach and serve
their customers, (b) / rather than, say, pumping money (c) /
into fancy graphics that look good in management meetings.
37. Not only the judges acquitted (a) / him of all the charges (b)
/ levelled against him, but (c) / also commended all his actions.
(d) /No error (e)

28.
(d) / No error (e)
asy
The loss of forests thus entail (a) / large social and economic
38. One of the most effective (a) / solutions is that (b) / she
should work on Sunday (c) / and complete the assignment.

En
costs; (b) the lives of more than one billion people (c) / are
already affected by loss of this forest cover (d) / No error (e) 39.
(d) / No error (e)
Our system of assigning (a) I different jobs to different people
29. Not one of the children (a) / has ever sang (b) / on any
occasion (c) / in public before. (d) / No error (e) gin (b) / should be based on (c) / their strengths and weaknesses
(d) / No error (e)

30. Reasonable ambition, if supported (a) / at persistent efforts,


(b) / is likely to yield (c) / the desied results. (d) / No error (e)
40.

eer
There he stood on the dais, (a) / debunked the manner in
which the company was run ( b) / by the former chairman
and managing director, (c) / and promising higher growth
31. I would have lost (a) / my luggage and other belonging (b) /
if I would have left the compartment (c) / and gone out to
ing
and industry. (d) No error (e)

.ne
fetch drinking water. (d) / No errro (e)

21. a b c d e 22. a b c d e 23. a b c d e 24. a b c d e 25. a b c d e


RESPONSE 26. a b c d e 27. a b c d e 28. a b c d e 29. a b c d e 30. a b c d e
GRID 31. a b c d e 32. a b c d e 33. a b c d e 34. a b c d e 35. a b c d e
36. a b c d e 37. a b c d e 38. a b c d e 39. a b c d e 40. a b c d e

Downloaded From : www.EasyEngineering.net


Downloaded From : www.EasyEngineering.net

Spelling Test 65
Max. Marks : 40 No. of Qs. 40 Time : 25 min. Date : ........./......../. ..............
DIRECTIONS: Choose the correct spelling of the given word. 11. (a) Entreprenuer (b) Entrepraneur
1. (a) Coimmission (b) Comision (c) Entrapreneur (d) Entrepreneur
(c) Comission (d) Commision (e) None of these
(e) None of these 12. (a) Skillful (b) Skillfull
2. (a) Jewelery (b) Jewellry (c) Skilful (d) Skilfull

ww
(c) Jwellry
(e) None of these
(d) Jewellery
13.
(e)
(a)
None of these
Varstile (b) Verstile
3. (a) Sattellite
(c) Sattelite
(e) None of these
w.E
(b) Satellite
(d) Satelite
14.
(c)
(e)
(a)
Versatile
None of these
Correspondant
(d) Vorstyle

(b) Corraspondent
4. (a) Ocasion
(c) Occasion
(b) Ocassion
(d) Occassion asy (c)
(e)
Corraspondant
None of these
(d) Correspondent

5.
(e) None of these
(a) Comettee (b) Committe En 15. (a)
(c)
Etiquete
Ettiquete
(b) Etiquette
(d) Ettiquette
(c) Comittee
(e) None of these
(d) Committee
gin
16.
(e)
(a)
None of these
Necessary (b) Necesarry
6. (a) Achievment
(c) Achievement
(b) Acheivment
(d) Achevement
(c)
(e) eer
Necesary
None of these
(d) Neccessary

7.
(e) None of these
(a) Hetrogenous (b) Hetrogeneous
17. (a)
(c)
Homeopathy
Homiopathy ing
(b) Homoepathy
(d) Homoeopathy
(c) Heterogenous (d) Heterogeneous
(e) None of these 18.
(e)
(a)
None of these
Lieutinant (b) Lieutenant .ne
8. (a) Foreigner
(c) Foriegnor
(e) None of these
(b) Forienor
(d) Foreiner
19.
(c)
(e)
(a)
Leutenant
None of these
Paralelogram
(d) Liutenant

(b) Paralellogram
t
9. (a) Colaboration (b) Collaberation (c) Parallelogram (d) Parallellogram
(c) Colaberation (d) Collaboration (e) None of these
(e) None of these 20. (a) Milennium (b) Millenium
10. (a) Acurrate (b) Accurate (c) Millennium (d) Milenium
(c) Acurate (d) Accuratte (e) None of these
(e) None of these

1. a b c d e 2. a b c d e 3. a b c d e 4. a b c d e 5. a b c d e
RESPONSE 6. a b c d e 7. a b c d e 8. a b c d e 9. a b c d e 10. a b c d e
GRID 11. a b c d e 12. a b c d e 13. a b c d e 14. a b c d e 15. a b c d e
16. a b c d e 17. a b c d e 18. a b c d e 19. a b c d e 20. a b c d e

Downloaded From : www.EasyEngineering.net


Downloaded From : www.EasyEngineering.net

138 SPEED TEST 65


DIRECTIONS (Qs. 21-40) : In each question below a sentence 30. Though the government initiated (a)/ a large sum (b)/ of
with four words printed in bold type is given. These are numbered money in the scheme (c)/ it was a failure (d)/. All correct (e).
as (a), (b), (c) and (d). One of these four words printed in bold may 31. It is not unusual (a)/for guests of the hotel to carry(b)/
be either wrongly spelt or inappropriate in the context of the souveniers(c)/ back with them when they return (d)/to their
sentence. Find out the word which is wrongly spelt or inappropriate homes. All correct (e)
if any. The number of that word is your answer. If all the words 32. She vested(a)/ her time in chatting(b)/ over the phone and
printed in bold are correctly spelt and also appropriate in the ultimately(c)/ ended up not finishing (d)/ her work. All
context of the sentence, mark (e) i.e. ‘All correct’ as your answer. correct (e)
21. Under existing (a) / regulations we are not permitted (b) / to 33. She had not eaten(a)/ anything(b)/ for a very long time now
owe (c) / more than a forty percent share (d) / of the family and her stomach(c)/ was groling(d)/ All correct(e)
business. All correct (e). 34. Half of the harm(a)/ that is done in this world(b)/ is due to
22. In case of any land dispute (a) / panchayat officials (b) / will people(c)/ who want to feel important(d)/ All correct(e)
determine (c) / how the property is to be dividend (d) / All 35. Life is like a mirror(a)/ smile at it and its charmeng(b)/
correct (e).
23.
ww
The World Bank has consented (a) / to sanction (b) / the
necessary (c) / finance (d) / for the project. All correct (e).
36.
frown(c)/ at it and it becomes sinister(d)/ All correct(e)
The Whole (a)/ time she walked with her child in her arms
the only thing (b)/ that worried (c)/ her was her son’s feature.
24.
w.E
To obtain (a) / a refund you will have to fill (b) / a claim (c)
/ with the appropriate (d) / authority. All correct (e). 37.
(d)/ All correct (e)
When the young artist returned (a)/ to his village, his family
held a festive (b)/ dinner on it’s lawn to celebrate his triumpant
25.

asy
Experts predict (a) / there will be shortage (b) / of investment
(c) / in the infrastructure (d) / sector. All correct (c).
38.
(c)/ homecoming. (d)/ All correct (e)
Had she not suppressed (a)/ all the details of her Company’s
26.
to constantly (c)/ improve (d)/. All correct (e).
En
In order to succeed (a)/ it is crucial (b)/ for an organisation
project (b)/ her Company would have bagged (c)/ the
contract. (d)/ All correct (e)
27. With some assistance (a)/ from her son she was enable (b)/
to settle (c)/ her debts (d)/ on time. All correct (e). gin
39. She trusted Mira with all her heart (a)/ and thus handled (b)/
over her life’s (c)/ savings to her instantly. (d)/ All correct (e).
28. We have prepared a detailed (a)/ report giving various (b)/
solutions (c)/ to resort (d)/ the problem. All correct (e).
40.
eer
It is difficullt (a)/ to see the picture (b)/ when you are inside
(c)/ the frame. (d)/ All correct (e)
29. RBI has attempted (a)/ to spend (b)/ financial (c)/ awareness
(d) / through this programme. All correct (e). ing
.ne
t

21. a b c d e 22. a b c d e 23. a b c d e 24. a b c d e 25. a b c d e


RESPONSE 26. a b c d e 27. a b c d e 28. a b c d e 29. a b c d e 30. a b c d e
GRID 31. a b c d e 32. a b c d e 33. a b c d e 34. a b c d e 35. a b c d e
36. a b c d e 37. a b c d e 38. a b c d e 39. a b c d e 40. a b c d e

Downloaded From : www.EasyEngineering.net


Downloaded From : www.EasyEngineering.net

One Word
Substitution
66
Max. Marks : 30 No. of Qs. 30 Time : 20 min. Date : ........./......../. ..............

DIRECTIONS (Qs.1-25) : In each of the following questions, 8. The original inhabitants of a country
out of the four alternatives, choose the one which can be (a) Aborigines (b) Citizens
substituted for the given words/sentence. (c) Natives (d) Primitive
(e) None of these
1. List of headings of the business to be transacted at a
9. One desirous of getting money
meeting
(a) Avaracious (b) Voracious

ww
(a) Schedule
(c) Proceedings
(e) None of these
(b) Agenda
(d) Excerpts
10.
(c) Garrulous
(e) None of these
(d) Greedy

Be the embodiment or perfect example of


2.
(a) Dull
(c) Obscure
w.E
Through which light cannot pass
(b) Dark
(d) Opaque
(a) Signify
(c) Personify
(e) None of these
(b) Characterise
(d) Masquerade

3.
(e) None of these
Stealing from the writings of others asy 11. Cutting for stone in the bladder
(a) Dichotomy (b) Tubectomy
(a) Copying
(c) Reproducing
(b) Reframing
(d) Plagiarism
En (c) Vasectomy
(e) None of these
(d) Lithotomy

4.
(e) None of these
Constant effort to achieve something
(a) Perseverance (b) Attempt
12.

gin
That which makes it difficult to recognise the presence of real
nature of somebody or something
(a) Cover (b) Mask
(c) Enthusiasm
(e) None of these
(d) Vigour
eer
(c) Pretence
(e) None of these
(d) Camouflage

5. A person not sure of the existence of God


(a) Theist
(c) Agnostic
(b) Atheist
(d) Cynic
13.
(a) Centenary
(c) Anniversary ing
Yearly celebration of a date or an event
(b) Jubilee
(d) Birthday

6.
(e) None of these
One who deserts his religion 14.
(e) None of these
.ne
One who has suddenly gained new wealth, Power or prestige

7.
(a) Deserter
(c) Fanatic
(e) None of these
(b) Turn-coat
(d) Apostate

One who uses fear as a weapon of power


15.
(a) Aristocrat
(c) Maverick
(e) None of these
(b) Affluent
(d) Parvenu

Code of diplomatic etiquette and precedence


t
(a) Formalism (b) Statesmanship
(a) Terrorist (b) Militant
(c) Protocol (d) Hierarchy
(c) Extremist (d) Anarchist
(e) None of these
(e) None of these

1. a b c d e 2. a b c d e 3. a b c d e 4. a b c d e 5. a b c d e
RESPONSE 6. a b c d e 7. a b c d e 8. a b c d e 9. a b c d e 10. a b c d e
GRID
11. a b c d e 12. a b c d e 13. a b c d e 14. a b c d e 15. a b c d e

Downloaded From : www.EasyEngineering.net


Downloaded From : www.EasyEngineering.net

140 SPEED TEST 66


16. Of outstanding significance 24. One who studies mankind
(a) Meaningful (b) Ominous (a) Anthropologist (b) Physicist
(c) Evident (d) Monumental (c) Pathologist (d) Philanthropist
(e) Rational (e) None of these
17. One who promotes the idea of absence of government of DIRECTIONS (Qs.26-30) : Choose the one word for the italic
any kind, when every man should be a law into himself sentences.
(a) Anarchist (b) Belligerent
25. An opinion contrary to popular belief
(c) Iconoclast (d) Agnostic
(a) Paradox (b) Orthodoxy
(e) None of these
(c) Hearsay (d) Heresy
18. Land so surrounded by water as to be almost an island
(e) None of these
(a) Archipelago (b) Isthmus
26. Rajesh was a hater of learning and knowledge.
(c) Peninsula (d) Lagoon
(a) misogynist (b) misologist
(e) None of these
19. That which cannot be done without (c) misanthropist (d) bibliophile
(a) Irrevocable (b) Impracticable (e) None of these

20.
ww
(c) Indispensable
(e) None of these
(d) Impossible

One who travels from place to place


27. The bus has to go back and forth every six hours.
(a) travel
(c) cross
(b) run
(d) shuttle
(a) Itinerant
(c) Journeyman
(e) None of these
w.E (b) Mendicant
(d) Tramp
(e) commute
28. A man can be sentenced to death for killing another human
being.
21. An act or notion to look back in the past
(a) Retrospective (b) Postnatalasy (a) fratricide
(c) homicide
(b) regicide
(d) genocide
(c) Retrogressive
(e) None of these
(d) Primitive

En (e) None of these


29. A careful preservation and protection of wildlife is the need
22. Medicine to counteract the effect of a poison
(a) Emetic
(c) Anti-venom
(b) Antidote
(d) Antiseptic gin of the hour.
(a) Embarkment
(c) Conservation
(b) Promotion
(d) Management

23.
(e) None of these
A collection of poems
(e) Enhancement
eer
30. The officer was not willing to take a definite stand on that
(a) Pathology
(c) Oncology
(b) Anthology
(d) Pedology
point.
(a) vague ing
(b) evasive
(e) None of these (c) ambiguous
(e) None of these
(d) complex
.ne
t

16. a b c d e 17. a b c d e 18. a b c d e 19. a b c d e 20. a b c d e


RESPONSE 21. a b c d e 22. a b c d e 23. a b c d e 24. a b c d e 25. a b c d e
GRID 26. a b c d e 27. a b c d e 28. a b c d e 29. a b c d e 30. a b c d e

Downloaded From : www.EasyEngineering.net


Downloaded From : www.EasyEngineering.net

Para Jumbles 67
Max. Marks : 30 No. of Qs. 30 Time : 20 min. Date : ........./......../. ..............

DIRECTIONS (Qs. 1 to 5): Rearrange the following five sentences C. The broad–based agitation against SEZs has demonstrated
(A), (B), (C), (D) and (E) in the proper sequence to form a meaningful the power of popular protest in the State.
paragraph, then answer the questions given below them. D. Those opposed to the projects had questioned the propriety
of the government acquiring large tracts of land and then
A. It will take extraordinary political commitment and liberal selling them to promoters at low prices.
public funding during the 11th Plan for affordable housing E. A coastal State with an area of 3,700 square kilometers and a
to become a credible goal.

ww
B. The National Urban Housing and Habitat Policy of the
United Progressive Alliance Government seeks to make
access to housing, long acknowledged as a fundamental
6.
population of about 1.4 million, Goa has always been extremely
sensitive to the impact of unrestrained economic development.
Which of the following should be the FIRST sentence?
(a) A (b) B (c) C

w.E
right, a reality for all.
C. The task is staggering even if we go by conservative estimates.
D. The housing shortage to be met during the Plan is 26.53
7.
(d) D (e) E
Which of the following should be the SECOND sentence?
(a) A (b) B (c) C
E.
asy
million units, which include the backlog from the 10th Plan.
If the existing stock of poor quality dwellings and the
growing urbanization–driven demand are taken into
8.
(d) D (e) E
Which of the following should be the THIRD sentence?
(a) A (b) B (c) C
1.
account, the real deficit will be even higher.
Which of the following should be the FIRST sentence?
(a) A (b) B (c) C En 9.
(d) D (e) E
Which of the following should be the FOURTH sentence?

2.
(d) D (e) E
Which of the following should be the SECOND sentence? gin
(a) A
(d) D
(b) B
(e) E
(c) C

10. Which of the following should be the FIFTH (LAST) sentence?

3.
(a) A
(d) D
(b) B
(e) E
(c) C

Which of the following should be the THIRD sentence?


(a) A
(d) D
eer (b) B
(e) E
(c) C

4.
(a) A
(d) D
(b) B
(e) E
(c) C

Which of the following should be the FOURTH sentence?


ing
DIRECTIONS (Qs. 11 to 15): Rearrange the following five sentences
(A), (B), (C), (D) and (E) in the proper sequence to form a meaningful
paragraph, then answer the questions given below them.
(a) A
(d) D
(b) B
(e) E
(c) C
.ne
A. The British government plans to insist that spouses should
have to learn English before they are allowed into Britain to
5. Which of the following should be the FIFTH (LAST)
sentence?
(a) A
(d) D
(b) B
(e) E
(c) C
join their husbands or wives have run into a barrage of
opposition and warnings that the idea could breach human
rights laws.
B. The responses to an official consultation on the proposal
published on Thursday was more than two to one against the
t
DIRECTIONS (Qs. 6 to10): Rearrange the following five sentences proposal, with many warning it could break up marriages
(A), (B), (C), (D) and (E) and in the proper sequence to form a because many cannot afford or access English lessons.
meaningful paragraph, then answer the questions given below them. C. Immigration lawyers have told ministers that spouses and
A. The upsurge of public activism against the setting up of fiances should not be barred from joining a partner in the
Special Economic Zones, which eventually forced the State U.K. for language reasons and that the plan could breach the
Government to announce the scrapping of all 15 such human rights convention's guarantees to the right to marry
projects, is an impressive case in point. and have a family life.
B. Early last year, a similar agitation coerced the government into D. The anonymised responses were 68 to 31 against the pre-
calling for a revision of the Goa Regional Plan 2011, a entry english test for spouses.
controversial document that opened up large swathes of land, E. Other immigration organizations said the measure would
including green belts and coastal stretches, for construction. discriminate against those from rural areas in South Asia,
where the opportunities to learn English are limited.

RESPONSE 1. a b c d e 2. a b c d e 3. a b c d e 4. a b c d e 5. a b c d e
GRID 6. a b c d e 7. a b c d e 8. a b c d e 9. a b c d e 10. a b c d e

Downloaded From : www.EasyEngineering.net


Downloaded From : www.EasyEngineering.net

142 SPEED TEST 67


11. Which of the following should be the FIRST sentence? (C) He rushed to his village and placed his humble offering of
(a) A (b) B (c) C milk in a bowl before the snake.
(d) D (e) E (D) Vishnu Raman was a poor Brahmin and a farmer by profession.
12. Which of the following should be the SECOND sentence? (E) The next day when he returned, he was rewarded with a gold
(a) A (b) B (c) C coin in the bowl he left behind.
(d) D (e) E (F) Just as he was preparing to lie down he saw a huge cobra
13. Which of the following should be the THIRD sentence? swaying with his hood open.
(a) A (b) B (c) C 21. Which of the following should be the SECOND sentence
(d) D (e) E after rearrangement?
14. Which of the following should be the FOURTH sentence? (a) B (b) C (c) E
(a) A (b) B (c) C (d) D (e) F
(d) D (e) E 22. Which of the following should be the FIRST sentence after
15. Which of the following should be the FIFTH (LAST) rearrangement?
sentence? (a) A (b) D (c) F
(a) A (b) B (c) C (d) C (e) E
(d) D (e) E 23. Which of the following should be the FIFTH sentence after
DIRECTIONS (Qs. 16 to 20) : Rearrange the following six sentences rearrangement?
(A), (B), (C), (D), (E) and (F) in the proper sequence to from a (a) F (b) D (c) C
meaningful paragraph; then answer the questions given below them. (d) B (e) E
24. Which of the following should be the SIXTH (LAST)

ww
(A) He immediately acknowledged Mohan’s good work and
invited him to his home for dinner.
(B) One day a wealthy merchant sent his son’s bicycle to the
shop for repair.
sentence after rearrangement?
(a) D
(d) E
(b) B
(e) F
(c) C

w.E
(C) The next day the merchant came to claim the bicycle and
noticed that it was shiny.
(D) After repairing the bicycle, Mohan cleaned it up and made it
look new.
25. Which of the following should be the FOURTH sentence
after rearrangement?
(a) E
(d) A
(b) F
(e) D
(c) B

worked as an apprentice in a bicycle shop. asy


(E) Once upon a time, there was a boy named Mohan who
(F) Other apprentices in the shop laughed at Mohan for doing
DIRECTIONS (Qs. 26 to 30) : In each of the following items some
parts have been jumbled up. You are required to rearrange these parts
which are labelled P, Q, R, S to produce the correct sentence. Choose
unnecessary work.
16. Which of the following should be the SECOND sentence
En the proper sequence and mark in your Answer Sheet accordingly.
26. Feeling flattered by praise of the fox to the piece of cheese
after rearrangement?
(a) A
(d) D
(b) B
(e) F
(c) C
17. Which of the following should be the THIRD sentence after
gin (P) / the crow began to crow (Q) / it held on its beak (R) /
unmindful of what will / happen (S).
The proper sequence should be
rearrangement?
(a) A
(d) D
(b) B
(e) E
(c) C
(a) SPRQ
(d) QSPR
eer (b) QSRP
(e) PQSR
(c) RSPQ
27. There is that the woman is a kitchen-maid and (P) / the
18. Which of the following should be the FIRST sentence after
rearrangement?
(a) A (b) B (c) C (a) QPSR
ing
traditional belief (Q) / an instrument of man’s pleasure (R) /
and a child-bearing machine (S).
The proper sequence should be
(b) RQPS (c) QSPR
(d) D (e) E
19. Which of the following should be the LAST (SIXTH)
sentence after rearrangement?
(d) RSPQ (e) PSRQ
.ne
28. The rapid endangerment and death of many minority
(a) A
(d) E
(b) B
(e) F
(c) D
20. Which of the following should be the FOURTH sentence
after rearrangement?
(a) B (b) C (c) D
languages not only among logistics and anthropologists

The proper sequence should be


(a) PSRQ (b) RQPS
t
(P) / with issues of cultural identity (Q) / is a matter of
widespread concern (R) / but among all concerned (S).
(c) RPSQ
(d) QRPS (e) SPQR
(d) E (e) F 29. Violence even before she is born (P) / and can happen
DIRECTIONS (Qs. 21 to 25) : Rearrange the following six throughout a woman’s life (Q) / against women (R) / takes
sentences (A), (B), (C), (D), (E) and (F) in the proper sequence to many forms (S).
from a meaningful paragraph; then answer the questions given The proper sequence should be
below them. (a) RPSQ (b) PQSR (c) RSQP
(d) SQRP (e) QPRS
(A) At first he got scared, but then he thought, “I have never 30. I saw two roads covered with the yellow (P) / directions in a
worshipped her; that is why I am not able to get anything forest (Q) / branching in two different (R) leaves of autumn (S).
from my land.” The proper sequence should be
(B) One day unable to tolerate the summer heat, he went to rest (a) PQRS (b) PQSR (c) PRSQ
under a big banyan tree. (d) PSRQ (e) QSRP

11. a b c d e 12. a b c d e 13. a b c d e 14. a b c d e 15. a b c d e


RESPONSE 16. a b c d e 17. a b c d e 18. a b c d e 19. a b c d e 20. a b c d e
GRID 21. a b c d e 22. a b c d e 23. a b c d e 24. a b c d e 25. a b c d e
26. a b c d e 27. a b c d e 28. a b c d e 29. a b c d e 30. a b c d e

Downloaded From : www.EasyEngineering.net


Downloaded From : www.EasyEngineering.net

Idioms and
Phrases 68
Max. Marks : 36 No. of Qs. 36 Time : 25 min. Date : ........./......../. ..............

DIRECTIONS (Qs. 1 to 36): In each of the following sentences, 10. We should give a wide berth to bad characters.
an idiomatic expression or a proverb is highlighted Select the (a) give publicity to (b) publicly condemn
alternative which best describes its use in the sentence. (c) keep away from (d) not sympathise with
(e) none of these
1. I have a bone to pick with you in this matter. 11. The authorities took him to task for his negligence.
(a) Am in agreement (b) Am angry (a) gave him additional work (b) suspended his assignment
(c) Am indebted (d) Will join hands (c) reprimanded him (d) forced him to resign
2.
ww
(e) None of these
The new CM stuck his neck out today and promised 10kgs.
free wheat a month for all rural families.
12.
(e) none of these
In spite of the immense pressure exerted by the militants, the
Government has decided not to give in.

3.
(a) took an oath
(c) extended help
(e) None of these w.E (b) took a risk
(d) caused embarrassment

Harassed by repeated acts of injustice he decided to put


(a) accede
(c) oblige
(e) none of these
(b) yield
(d) confirm

his foot down.


(a) not to yield (b) resign
(c) to accept the proposal unconditionally
asy 13. Their business is now on its last legs.
(a) About to fructify
(c) About to produce results
(b) About to perish

4.
(d) withdraw (e) none o fthese
The class could not keep a straight face on hearing the En 14.
(d) About to take off (e) none of these
He went back on his promise to vote for me.
(a) withdrew (b) forgot
strange pronunciation of the new teacher.
(a) remain silent
(c) remain mute
(b) remain serious
(d) remain disturbed gin
15.
(c) reinforced
(e) none of these
(d) supported

The old beggar ran amuck & began to throw stones at the passerby.
5.
(e) none of these
His speech went down well with the majority of the audience.
(a) found acceptance with eer
(a) became desperate
(c) become annoyed
(b) ran about wildly
(d) felt disgusted

ing
(e) none of these
(b) was attentively listened to by 16. Turban is in vogue in some communities.
(c) was appreciated by (d) was applauded by (a) in fashion (b) out of use
(e) none of these
6. Rohit has bitten off more than he chew.
(a) Is trying to do much (b) Is very greedy 17.
(c) vaguely used
(e) none of these
.ne
(d) never used

The old man was cut to the quick when his rich son refused

7.
(c) Is always hungry
(e) none of these

(a) took no pains


(d) Has little regard for others

The detective left no stone unturned to trace the culprit.


(b) did very irrelevant things
(c) resorted to illegitimate practices 18.
to recognise him.
(a) surprised
(c) annoyed
(e) none of these
(b) hurt intensely
(d) irritated

I requested him to put in a word for me.


t
(d) used all available means (a) introduce (b) assist
(e) none of these (c) support (d) recommend
8. He believes in the policy of making hay while the sun shines. (e) none of these
(a) giving bribes to get his work done 19. The dacoit murdered the man in cold blood.
(b) seeking advice from one and all (a) coldly (b) boldly
(c) helping those who help him (c) ruthlessly (d) deliberately
(d) making the best use of a favourable situation (e) none of these
(e) none of these 20. He is always picking holes in every project.
9. His friends advised him to be fair and square in his dealings. (a) creating problems in (b) finding fault with
(a) Careful (b) Considerate (c) suggesting improvement in
(c) Polite (d) Upright (d) asking irrelevant questions on
(e) none of these (e) None of these
1. a b c d e 2. a b c d e 3. a b c d e 4. a b c d e 5. a b c d e
6. a b c d e 7. a b c d e 8. a b c d e 9. a b c d e 10. a b c d e
RESPONSE 11. a b c d e 12. a b c d e 13. a b c d e 14. a b c d e 15. a b c d e
GRID 16. a b c d e 17. a b c d e 18. a b c d e 19. a b c d e 20. a b c d e

Downloaded From : www.EasyEngineering.net


Downloaded From : www.EasyEngineering.net

144 SPEED TEST 68


21. Pt. Nehru was born with a silver spoon in his mouth. 31. The speaker gave a bird's eye view of the political
(a) born in a middle class family conditions in the country.
(b) born in a wealthy family (a) a personal view (b) a general view
(c) born in a royal family (c) a biased view (d) a detailed presentation
(d) born in a family of nationalists
(e) None of these
(e) none of these
22. The arrival of the mother-in-law in the family proved a rift in 32. The stunt that I recently attempted was a piece of cake
the lute. (a) The stunt that I recently attempted was enjoyable to watch
(a) caused unnecessary worries (b) The stunt that I recently attempted was very challenging
(b) brought about disharmony (c) The stunt that I recently attempted was celebrated by all
(c) caused a pleasant atmosphere (d) The stunt that I recently attempted turned out to be a failure
(d) brought about a disciplined atmosphere (e) The stunt that I recently attempted was a simple task
(e) none of these 33. The boy broke the window and took to his heels.
23. Having sold off his factory, he is now a gentleman at large.
(a) The boy broke the window and fell on his heels
(a) Has no serious occupation
(b) Is living comfortably (b) The boy broke the window and ran away
(c) Is respected by everybody (c) The boy broke the window with his heels
(d) Is held in high esteem (e) none of these (d) The boy ran into the window
24.
ww
Though he has lot of money, yet all his plans are built upon sand.
(a) established on insecure foundations
(b) based on inexperience (c) resting on cheap material
34.
(e) The boy broke the window and robbed a pair of heels
I pledged myself to serve the king faithfully.
(a) I made a mistake by promising to serve the king

25.
(e) none of these
w.E
(d) resting on immature ideas

There has been bad blood between the two communities even
before shouting.
faithfully
(b) I made a fool of myself in order to serve the king
(c) I boasted about serving the king faithfully
(a) Impure blood
(c) Bloody fights
(e) none of these
(b) Ill feeling
(d) Quarrels asy (d) I was forcibly made to serve the king
(e) I made a solemn and formal promise to serve the king
faithfully
26. The curious neighbors were disappointed as the young
couple's quarrel was just a storm in a tea cup. En 35. There is a crying need for improvements to our public
transport system.
(a) violent quarrel
(c) brittle situation
(e) none of these
(b) fuss about a trifle
(d) quarrel about tea cups
gin (a) There is an obvious need for improvements to our public
transport system
27. My father strained every nerve to enable me to get settled in life.
(a) worked very hard
(c) tried all tricks
(b) spent a huge amount
(d) bribed several persons eer
(b) There is a well documented need for improvements to
our public transport system.
(c) There is a minor need for improvements to our public

28.
(e) none of these
Madhuri might scream blue murder, but I feel Deepali should
transport system.
ing
(d) There is a serious need for improvements to our public
get the promotion since she is better qualified for the job.
(a) Someone has been murdered with some blue liquid
(b) Someone is being murdered and has become blue
transport system.

.ne
(e) There is no urgency for improvements to our public
transport system.

29.
(c) Suffer from persecution complex
(d) Make a great deal of noise and object vehemently
(e) none of these
Why do you wish to tread on the toes?
(a) To give offence to them
36.
belonged to my grandfather.
(a) In an old bookshop I happened to discard a volume
that belonged to my grandfather.
t
In an old bookshop I happened to light upon a volume that

(b) To follow them grudgingly (b) In an old bookshop I happened to purchase a volume
(c) To treat them indifferently that belonged to my grandfather.
(d) To be kicked by them (e) None of these (c) In an old bookshop I happened to look for a volume
30. The autographed bat from the famous cricketer Sunil that belonged to my grandfather.
Gavaskar is worth a jew's eye. (d) In an old bookshop I happened to discover by chance
(a) Not a worthy possession
a volume that belonged to my grandfather
(b) unnecessary (c) A costly items
(d) A possession of high value (e) In an old bookshop I happened to reveal a volume that
(e) None of these belonged to my grandfather.

21. a b c d e 22. a b c d e 23. a b c d e 24. a b c d e 25. a b c d e


RESPONSE 26. a b c d e 27. a b c d e 28. a b c d e 29. a b c d e 30. a b c d e
GRID 31. a b c d e 32. a b c d e 33. a b c d e 34. a b c d e 35. a b c d e
36. a b c d e

Downloaded From : www.EasyEngineering.net


Downloaded From : www.EasyEngineering.net

Cloze Test - I 69
Max. Marks : 34 No. of Qs. 34 Time : 20 min.

DIRECTIONS (Qs.1-34) : In the following passage there are Date : ........./......../. ..............
blanks each of which has been numbered. These numbers are
Passage - 2
printed below the passage and against each, five words are
Today, twenty-two years after the bank (9), it has over a
suggested, one of which fits the blank appropriately. Find out
thousand branches all over the country and the staff (10) about
the approptiate words in each case.
twenty-three lakh borrowers. We decided to operate (11) from
Passage - 1

ww
When we (1) started thirty years ago in 1977, we did not
know anything about how to run a bank for the poor. We therefore
looked at how others ran their operations and (2) from their
conventional banks who would ask their clients to come to their
office. Many people in rural areas found this (12). Our bank is
therefore based on the (13) that people should not come to the

w.E
mistakes. In Bangladesh, conventional banks and credit co-
operatives always (3) lump sum repayments. This created (4)
problems because repaying in a lump sum was a mental hurdle
bank but that the bank should go to the people. Our loans are also
(14)- we give them for activities for candlemaking to tyre repair. We
also keep (15) checks on the borrower through weekly visits. We
do this(a)to make certain that the family
(b) of the borrower is (16) from

asy
for borrowers. They tended to delay repayment and get further
into debt in the (5). In the end they usually (6) totally on the loan,
which was a loss to the bank. In structuring our own loans, I
the loan.
9.
inaugurated
(c) commence
(e) began
origin
(d) existed

decided to ask for a daily payment, Monitoring repayment was


En
(7) and it filled people with (8) that they could repay their loans.
10. (a)
(c)
handle
cope
(b) assemble
(d) interact
1. (a) firstly
(c) foremost
(e) recently
(b) freshly
(d) initially
gin
11.
(e)
(a)
(c)
deal
identically
similar
(b) differently
(d) reverse
2. (a) copied
(c) learned
(b) observed
(d) understood 12.
(e)
(a) eer
opposite
threatening (b) worried

3.
(e) improving
(a) asked
(c) demanded
(b) insisted
(d) settled 13.
(c)
(e)
(a)
upset
anxious
advantage ing (d) panicking

(b) principle

4.
(e) lend
(a) severe (b) no
(c)
(e)
discipline
chance .ne
(d) opportunity

5.
(c) additionally
(e) plenty
(a) time
(c) return
(d) variety

(b) process
(d) event
14.

15.
(a)
(c)
(e)
(a)
diverse
vary
contrast
daily
(b) worth
(d) disburse

(b) consistently
t
(e) action (c) regular (d) often
6. (a) neglected (b) abandoned (e) frequently
(c) defaulted (d) depended 16. (a) progress (b) benefiting
(e) disappointed (c) serving (d) welfare
7. (a) benefit (b) easier (e) obliged
(c) reckless (d) disorganised Passage - 3
(e) secure The Government seems to be in right earnest to ensure more
8. (a) sense (b) confidence (17) in governance. The Prime Minister’s announcement that his
(c) challenge (d) doubt Government is (18) drafting legislation to establish the citizen’s
(e) believe right to information is indeed welcome. Though the talk on the
right to information is not new, we may (19) the bill to be brought

1. a b c d e 2. a b c d e 3. a b c d e 4. a b c d e 5. a b c d e
6. a b c d e 7. a b c d e 8. a b c d e 9. a b c d e 10. a b c d e
RESPONSE 11. a b c d e 12. a b c d e 13. a b c d e 14. a b c d e 15. a b c d e
GRID 16. a b c d e

Downloaded From : www.EasyEngineering.net


Downloaded From : www.EasyEngineering.net

146 SPEED TEST 69


early this time. The previous Government had set up a high-level $ 86 billion. (32) countries have now begun to notice the (33)
committee to prepare a draft bill. But nothing has been heard about available in Africa. China’s attitude has (34) the way the world
the matter since, (20) the committee did quite some work. The deals with poor countries. “Trade not aid” is the new mantra of
issue, however, has come to such a pass that a solution cannot be African nations.
(21) further. Sunlight is the best disinfectant, a foreign judge once 25. (a) belonging (b) similarly
said, while (22) the unwarranted secrecy in an administrative (c) compared (d) with
system. When those in authority know that people have the right (e) like
to ask questions and the government is under the (23) to provide 26. (a) efforts (b) practices
them with answers, (24) of authority, or of public finances, for (c) challenges (d) achievements
personal or party ends is less likely to happen. (e) attempt
17. (a) strictness (b) rudeness (c) leniency
27. (a) given (b) approved
(d) economy (e) transparency
(c) regular (d) often
18. (a) personally (b) busy (c) not (e) being
(d) reluctantly (e) absolutely 28. (a) grant (b) sanctioned
19. (a) expect (b) wait (c) try
(c) took (d) hired
(d) frustrate (e) appeal
(e) apply

ww
20. (a) even
(d) until
21. (a) found
(b) as
(e) though
(b) expected
(c) because

(c) delayed
29. (a) goal
(c) way
(e) dream
(b) fund
(d) skill
(d) looked
22. (a) nurturing
(d) appreciating
w.E (e) longed
(b) criticising
(e) upholding
(c) demanding
30. (a) countries
(c) abroad
(e) poor
(b) others
(d) neighbours
23. (a) pretention
(d) obligation
24. (a) misuse
(b) affect
(e) property
asy (c) substance

(b) governance (c) dishonour


31. (a) further
(c) more
(b) extra
(d) less
(d) curbing (e) breach
Passage - 4 En (e) high
32. (a) Recently (b) Any
Today the economies of African countries are growing at
5 per cent every year. gin (c) Friendly
(e) While
33. (a) differences
(d) Many

(b) supply
Rich countries usually help poorer ones (25) African
countries through donations and aid. Their (26) are not always
successful as loans are (27) not used for the projects for which
(c) quantity
eer
(e) opportunities
(d) people

they are (28). China, however, has found a different (29) to help
Africa - by trading more with the (30). In 2009 China’s trade with
34. (a) substitute
(c) exchanged
(e) convert ing
(b) changed
(d) transform
African countries was $ 90 billion - (31) than the U.S., which was

.ne
t

17. a b c d e 18. a b c d e 19. a b c d e 20. a b c d e 21. a b c d e


RESPONSE 22. a b c d e 23. a b c d e 24. a b c d e 25. a b c d e 26. a b c d e
GRID 27. a b c d e 28. a b c d e 29. a b c d e 30. a b c d e 31. a b c d e
32. a b c d e 33. a b c d e 34. a b c d e

Downloaded From : www.EasyEngineering.net


Downloaded From : www.EasyEngineering.net

Cloze Test - II 70
Max. Marks : 35 No. of Qs. 35 Time : 20 min. Date : ........./......../. ..............

DIRECTIONS : In the following passage there are blanks, each 6. (a) felt (b) said
of which has been numbered. These numbers are printed below (c) know (d) accept
the passage, against each, five words are suggested, one of which (e) saw
fits the blank appropriately. Find out the appropriate word in 7. (a) asks (b) chooses
each case. (c) look (d) find
(e) wish
Passage - 1 8. (a) sure (b) put

ww
Emperor Akbar was fond of (1) tricky questions to Birbal. One
day he asked Birbal what he would (2) if he were given a choice
between justice and a gold coin. “The gold coin,” said Birbal. 9.
(c)
(e)
(a)
shown
made
no
(d) seen

(b) rich

w.E
Akbar was (3) aback. He had known Birbal for many years and
he knew that Birbal was a just person. Then how could he choose
the gold coin. “You would prefer a gold coin to justice ?” He 10.
(c)
(e)
(a)
short
plenty
but
(d) poor

(b) not

asy
asked, incredulously. “Yes,” said Birbal. The other courtiers were
amazed by Birbal’s (4) of idiocy. For years they had been trying
to discredit Birbal in the emperor’s eyes but without success and
(c)
(e)
and
only
Passage - 2
(d) s o

now the man had gone and (5) it himself ! They could not
believe their good fortune. “I would have been dismayed if even
En Once upon a time, two friends were (11) through the desert. During
some point of the (12) they had an argument, and one friend slapped
the lowliest of my servants had said this,” continued the em-
peror. “But coming from you it’s shocking - and sad. I did not (6)
you were so debased ! I never expected this from you. How gin
the other one in the face. The one who got slapped was (13), but
without saying anything, he wrote in the sand, “Today my best
friend slapped me in the face.” They kept on walking (14) they
could you be so shallow ?”
One (7) for what one does not have, Your Majesty !” said Birbal,
quietly. “You have (8) to it that in our country justice is available eer
found an oasis, where they (15) to take a bath. The one, who had
been slapped, got (16) in the quicksand and started drowning, but
the friend saved him. After the friend (17) from the near drowning
to everybody. So as justice is already available to me and as I’m
always (9) of money I said I would choose the gold coin.” The ing
he wrote on a stone, “Today my best friend saved my life.” The
friend who had slapped and saved his best friend asked him, “After
I hurt you, you wrote in the sand and (18) you write on a stone,
emperor laughed. He thought to himself, ‘I should have known
that Birbal would come up with a witty reply as always.’ He was
so pleased with Birbal’s reply that he gave him (10) one but a .ne
why?” The other friend (19), “When someone hurt us, we should
write it down in sand where wind of forgiveness can erase it away.
thousand gold coins.
1. (a) showing
(c) naming
(e) telling
(b) asking
(d) finding
But, when someone does something good for us, we must (20) it in
stone where no wind can ever erase it.”
11. (a) crawling
(c) swimming
(e) dancing
(b) speaking
(d) walking
t
2. (a) look (b) said 12. (a) journey (b) sand
(c) think (d) choose (c) running (d) border
(e) find (e) hunt
3. (a) pushed (b) fallen 13. (a) dead (b) captured
(c) pulled (d) sent (c) presentable (d) missing
(e) taken (e) hurt
4. (a) idea (b) display 14. (a) as (b) until
(c) reply (d) place (c) from (d) with
(e) showing (e) through
5. (a) speak (b) thought 15. (a) decided (b) fell
(c) done (d) create (c) made (d) want
(e) told (e) left

1. a b c d e 2. a b c d e 3. a b c d e 4. a b c d e 5. a b c d e
RESPONSE
6. a b c d e 7. a b c d e 8. a b c d e 9. a b c d e 10. a b c d e
GRID 11. a b c d e 12. a b c d e 13. a b c d e 14. a b c d e 15. a b c d e

Downloaded From : www.EasyEngineering.net


Downloaded From : www.EasyEngineering.net

148 SPEED TEST 70


16. (a) home (b) stuck 24. (a) affect (b) ideas (c) practice
(c) blended (d) mixed (d) concept (e) procedure
(e) sitting 25. (a) benefit (b) merit (c) chance
17. (a) separated (b) leaked (d) basis (e) method
(c) died (d) recovered 26. (a) unless (b) until (c) executed
(e) saved (d) provided (e) exercised
18. (a) so (b) how 27. (a) other (b) any (c) two
(c) when (d) tomorrow (d) differ (e) after
(e) now 28. (a) on (b) of (c) often
19. (a) called (b) tell (d) taken (e) off
(c) replied (d) questioned 29. (a) soft (b) more (c) less
(e) asked (d) only (e) hard
20. (a) talk (b) push 30. (a) need (b) equilibrium
(c) engrave (d) add (c) expectation (d) attempt
(e) bury (e) aspects
Passage - 3 Passage - 4
The Right of Children to Free and Compulsory Education (RTE) The Bhagavad Gita is a poem of 700 verses which is a part of the

ww
Act, 2009, which came (21) effect in April this year, is meant to
transform the education sector and take India closer to the goal of
universal schooling. But with admissions to the new academic
Mahabharata. It is the only philosophical song existing in all
languages. Its popularity and influence have never waned. It (31)
light and guidance to the troubled mind in times of crisis. It is in

w.E
session just (22) the corner, it is fast becoming clear that (23) well
intentioned ideas into (24) will take some doing. For a start, the
guidelines for admissions under the RTE prohibit schools from
conducting any sort of student profiling. The stress on a random
the (32) of a dialogue between Arjuna and Krishna on the
battlefield. Arjuna’s mind is troubled at the thought of the killings
of his friends and relatives. He cannot conceive of any gain. Arjuna

asy
yet justifiable admission process means that schools will have to
resort to something as quirky as a lottery system. However, leaving
is the (33) of the tortured spirit of man tom by conflicting
obligations and molalities.
The dialogue proceeds and takes upto the higher level of
admission to a good school to pure (25) will only incentivise
manipulations, defeating the very essence of RTE.
The main problem facing the education sector is that of aEn individual duty and social behaviour, application of ethics to
practical life and social outlook that should govern all. An attempt
is (34) to reconcile the three paths of human advancement - the
resource crunch. The provisions for ensuring universal access to
education are all very well, (26) we have the infrastructure in place gin
path of knowledge, the path of action and the path of faith. But
more (35) is laid on faith. There is a call of action to meet the
first. Brick and mortar schools need to precede open admission
and not the (27) way around. In that sense, legislators’ assessment
of ground realities is (28) target when they endorse the closure of eer
obligations of life, keeping in view the spiritual background and
the large purpose of the universe.
31. (a) provides (b) shines
tens of thousands of low-cost private schools for not meeting the
minimum standards of land plot, building specifications and
playground area as laid out in the RTE Act. Instead of bearing
(c) enforces
(e) seeks
32. (a) programme
ing
(d) secures

(b) constitution
down (29) on private schools for failing to conform to abstract
bureaucratic criteria, efforts to bring about universal education
(c) part
(e) form
(d) formation
.ne
should focus on upgrading and expanding the existing
government school infrastructure to accommodate all. Only then
can we ensure the much needed supply-demand (30) in the
education sector.
21. (a) with (b) for (c) on
33. (a) conceived
(c) source
(e) symbol
34. (a) generated
(b) dream
(d) figures

(b) made
t
(c) established (d) coined
(d) into (e) in (e) given
22. (a) around (b) near (c) into 35. (a) important (b) Significant
(d) about (e) reaching (c) declaration (d) emphasis
23. (a) forming (b) translating (c) having (e) blessings
(d) taking (e) framing

16. a b c d e 17. a b c d e 18. a b c d e 19. a b c d e 20. a b c d e


RESPONSE 21. a b c d e 22. a b c d e 23. a b c d e 24. a b c d e 25. a b c d e
GRID 26. a b c d e 27. a b c d e 28. a b c d e 29. a b c d e 30. a b c d e
31. a b c d e 32. a b c d e 33. a b c d e 34. a b c d e 35. a b c d e

Downloaded From : www.EasyEngineering.net


Downloaded From : www.EasyEngineering.net

Section Test :
English Language 71
Max. Marks : 40 No. of Qs. 40 Time : 20 min. Date : ........./......../. ..............
DIRECTIONS : Read the following passage carefully and answer or start a manufacturing business. It can help those who have
the questions given below it. Certain words/group of words have found a job and are still nonetheless poor. It gives the victims of
been printed in bold to help you locate them while answering prejudice who would not be hired because of their colour or national
some of the questions. origin a chance to earn a living. The average cost of creating self-
Unemployment is the problem of every modern nation. Even employment is ten, twenty or hundred times lesser than creating
industrialised nations are not able to ensure a job for everyone. industry-based employment. It helps isolated poor people gain self-

ww
Following the conventional strategy of creating employment,
governments of many developing countries try to attract
employers (business houses/industrialists) by offering tax
confidence step by step.
Obviously self-employment has limits, but in many cases it is
the only solution to help those whom economies refuse to hire and

w.E
rebates and many other facilities so that they locate their
upcoming plants on their soil, and thereby create industrial
employment. But there is a limit to what industry can bring.
taxpayers do not want to carry on their shoulders. The policy needed
for the eradication of poverty must be much wider and deeper than
the policy for the provision of mere employment. The real eradication

asy
Also, industrial plants often create toxic waste which results in
air and water pollution and environmental problems which can
outweigh whatever employment benefit industrial employment
of poverty begins when people are able to control their own fate.
Poor people are like bonsai trees. When you plant the best seed of
the tallest tree in a flower-pot, you get a replica of the tallest tree,

En
brings. In addition, they don't bring as substantial relief to the
dwindling economy of the host country as they seem to promise,
only inches tall. There is nothing wrong with the seed you planted;
only the soil-base that is too inadequate. Poor people are bonsai
as the profits of such foreign investments are carried back to the
parent company and foreign shareholders abroad. gin
people. There is nothing wrong in their seeds. Simply, society never
gave them the base to grow on. All it takes to get the poor people
out of poverty is for us to create an enabling environment for them.
Self-employment has none of these drawbacks. The problem
is that self-employment is not as obviously glamorous as a shiny
new factory. But profits from self-employment remain in the eer
Once the poor can unleash their energy and creativity, poverty will
disappear very quickly.
country where they are produced. It is too small to create
environmental hazards. It also puts the poor person in charge of ing
DIRECTIONS (Q. 1-3) : Choose the word/group of words which is
MOST OPPOSITE in MEANING to the word/group of words printed
his or her own working hours and conditions. The hours are
flexible and can adapt to fit any family situation. It allows people
to choose between running a business full-time, or part-time when
in bold as used in the passage.
1. RIGID .ne
they face a crisis, or to put their business on hold and work full-
time for a salary. Self-employment is tailor-made for anyone who
is street-smart and has many acquired and inherited traditional
skills, rather than learning acquired from books and technical 2.
(a) Unstructured
(c) Soft
(e) Calm
STEP BY STEP
(b) Flexible
(d) Gentle
t
schools. This means the illiterate and the poor can exploit their (a) All at once (b) In quick succession
strengths, rather than be held back by their weaknesses. It allows (c) In slow motion (d) In a nutshell
a person to turn their hobbies into gainful employment. It allows (e) Once and for all
individuals who cannot work well in a rigid hierarchy to run their
3. OFFERING
own show.
(a) Stealing (b) Permitting
Financing the poor to start their own little ventures elevates
their sense of pride and self-respect. It offers a way out of welfare (c) Refusing (d) Protesting
dependency, not just to become wage slaves, but to open a store (e) Questioning

RESPONSE 1. a b c d e 2. a b c d e 3. a b c d e
GRID

Downloaded From : www.EasyEngineering.net


Downloaded From : www.EasyEngineering.net

150 SPEED TEST 71


DIRECTIONS (Q. 4-15): Choose the word/group of words which 10. Which of the following may be inferred about self-
is MOST SIMILAR in MEANING to the word printed in bold as employment?
used in the passage. (A) Self-employment slowly but steadily strengthens the
4. REMAIN economy of the country.
(a) Left-over (b) Stay (B) Self-employment checks unemployment.
(c) Stagnate (d) Continue (C) As a str ategy of providing employment, self-
(e) Linger employment is still unexplored.
5. HIRED (a) Only (B) (b) Only (B) and (C)
(a) Allowed (b) Rented (c) Only (A) (d) Only (A) and (B)
(c) Authorised (d) Employed (e) Only (A) and (C)
(e) Delegated 11. What does the author indicate by the example of a bonsai
tree?
6. LIMIT
(A) When provided the right kind of financial help, poor
(a) Maximum (b) Finish
people in can flourish.
(c) Cap (d) Decrease
(B) The poor people are as capable as the well-to-do class.

7.
ww
(e) Barrier
Which of the following is a reason foreign investments do
not strengthen the economies of host nations?
(C) Conventional (industrial) employment can help the
poor people create their own base.

nation. w.E
(a) The parent company pays all the profit as tax to its

(b) The profit of such enterprises does not remain in the


(a) Only (A)
(c) Only (A) and (B)
(e) Only (B) and (C)
(b) Only (B)
(d) Only (A) and (C)

asy
host nation; rather it goes back to the share holders
and owners of the parent company.
12. The author claims that self-employment is "tailor-made" for
people with certain qualities. Which of the following are the
qualities of such people?
(c) The employees of the parent company demand extra

En
pay from profits that the companies earn from factories (A) They have an unconventional approach to all things.
(B) They are street-smart.
in another nation.
(d) The profit earned by such enterprises is too less to
provide for anything beyond the salaries of employees. gin (C) They possess many acquired and traditional skills.
(a) Only (A) (b) Only (C)

8.
(e) None of these
What is the tone of the passage? (e) Only (B) eer
(c) Only (B) and (C) (d) Only (A) and (C)

(a) Offensive
(c) Analytical
(b) Satirical
(d) Humorous
13.
ing
Which of the following outweighs the employment benefits
that foreign industrialists bring?

9.
(e) Speculative
Which of the following is an advantage(s) that self- pollution.
.ne
(a) Huge industries set up by them cause environmental

(b) They employ more number of people belonging to their


employment has over industry-based employment?
(A) The work timings are highly flexible.
(B) Starting one's own venture is an easy task and needs
no investment as financiers are readily available.
native nations than the host nations' unemployed.
t
(c) They evade many taxes that could be a source of revenue
for the host nation.
(C) Self-employment makes one a master of other people (d) They manufacture products that have no market in the
and thus satisfies their need to control others. host nation.
(a) Only(C) (b) Only (A) (e) They practise discrimination on grounds of gender
when providing employment to host nations' residents.
(c) Only (B) (d) Only (A) and (B)
(e) All (A), (B) and (C)

RESPONSE 4. a b c d e 5. a b c d e 6. a b c d e 7. a b c d e 8. a b c d e

GRID 9. a b c d e 10. a b c d e 11. a b c d e 12. a b c d e 13. a b c d e

Downloaded From : www.EasyEngineering.net


Downloaded From : www.EasyEngineering.net

SPEED TEST 71 151


14. Which of the following may be an appropriate title for the 20. Each business activity ___________ employment to people
passage? who would otherwise be unemployed.
(a) Addressing conventional employment in developed (a) taking (b) finds
nations (c) creates (d) provides
(b) Varied strategies and approaches to eradicating (e) given
poverty DIRECTIONS (Q. 21-30) : In the following passage, there are
(c) Limitations of industrial employment blanks, each of which has been numbered. These numbers are
(d) How is poverty linked to conventional (industrial) printed below the passage and against each five words are
employment? suggested, one of which fits the blank appropriately. Find out the
(e) Role of self-employment in battling unemployment and appropriate word in each case.
eradication of poverty
15. Which of the following is TRUE as per the passage? The latest technology (21) put to use or about to arrive in
(a) Self-employment is beneficial only for developing market must be (22) to all entrepreneurs. The reason is that it may
economies. have an (23) effect on business. Valve radios gave way to transistor
(b) Self-employment is not as glamorous as conventional radios and with micro chips, technology is giving way to digital

ww
(industrial) employment.
(c) Finance for poor is readily available in the developed
nations of the world.
equipment. Business has (24) the same but the technology has
kept changing. A notable feature is that the size of the receivers
decreased (25) so did the use of its material and consequently its

w.E
(d) Small-scale industries produce as much toxic waste
as big industries.
price. The traditional flour mills are losing business (26) customers
now buy flour (27) from the market. As a result of this, the business
is (28). Following the same lines as technology, the social trends
(e) None is true

asy
DIRECTIONS (Q. 16-20): Pick out the most effective word from
the given words to fill in the blank to make the sentence
also go on changing and influence the market. The Indian sarees
are being taken (29) by readymade stitched clothes. Every

meaningfully complete.
16. The government is planning to set ___________ family En entrepreneur must note such changes in the environment and also
the technology and plan in (30)with these to ensure the success of
his endeavour.
welfare centres for slums in cities.
(a) another (b) with gin
21. (a) to
(c) decided
(b) needed
(d) besides
(c) for
(e) up
(d) in (e) being
22. (a) hoped eer (b) welcome
17. Economic independence and education have ___________
women more assertive.
(a) prepared (b) made
(c) released
(e) aware ing
(d) known

(c) marked
(e) adjusted
(d) resulted
23. (a) approximate
(c) uniform
(b) huge
(d) excellence
.ne
18. In the modern world, the ___________ of change and
scientific innovation is unusually rapid.
(a) supplies (b) context
(e) enormous
24. (a) maintained
(c) often
(e) become
(b) remained
(d) mentioned
t
(c) pace (d) fantasy
25. (a) mainly (b) and
(e) requirement
(c) how (d) also
19. The unprecedented economic growth of China has
(e) some
___________ worldwide attention.
26. (a) reason (b) due
(a) perceived (b) proposed
(c) young (d) as
(c) neither (d) astonished
(e) old
(e) attracted

14. a b c d e 15. a b c d e 16. a b c d e 17. a b c d e 18. a b c d e


RESPONSE 19. a c d e 20. a c d e 21. a c d e 22. a c d e 23. a c d e
b b b b b
GRID
24. a b c d e 25. a b c d e 26. a b c d e

Downloaded From : www.EasyEngineering.net


Downloaded From : www.EasyEngineering.net

152 SPEED TEST 71


27. (a) knowingly (b) ease 34. It is difficult to work with him because he is one of those
(c) cheap (d) directly persons who think he is always right.
(e) forcefully (a) think they are always
28. (a) shrinking (b) blooming
(b) always thinks he is (c) is always thinking they are
(c) returned (d) same
(d) always think his (e) No correction required
(e) small
29. (a) against (b) to 35. Foreign businesses in developing countries have usually
(c) over (d) up problems with lack of infrastructure and rigid laws.
(e) for (a) usual problems as (b) usually problems on
30. (a) lines (b) relativity (c) as usual problems like (d) the usual problems of
(c) accordance (d) proper (e) No correction required
(e) toning
DIRECTIONS (Qs. 36-40) : Read each sentence to find out whether
DIRECTIONS (Q. 31-35): Which of the phrases (a), (b), (c) and
(d) given below should replace the phrase given in bold in the there is any grammatical mistake/error in it. The error if any, will be

ww
following sentences to make the sentence grammatically correct?
If the sentence is correct as it is and there is no correction required
mark (e) ie ‘No correction required’ as the answer.
in one part of the sentence. Mark the number of the part with error
as your answer. If there is no error, mark (e).
36. The cost of constructing (a) / houses are increased (b) /

lay off workers.


(a) have the force to
w.E
31. During the recession many companies will be forced to

(b) be forced into


37.
because of the high (c) / price of cement. (d) / No error (e).
According to the Twelfth (a) / Five Year Plan, India should

(c) forcibly have


(e) No correction required
(d) forcefully
asy (b) / invest one trillion dollars (c) / in infrastructure projects.
(d) / No error (e)
32. He wanted nothing else expecting to sleep after a Stressful
day at work.
En 38. To increase the selling (a) / of products in rural areas (b) / the
company will hire (c) / over five hundred trainees. (d) / No
(a) nothing better than (b) anything else unless
(c) nothing but having (d) nothing else than
gin
39.
error (e)
We have spent (a) / most of the profits (b) / that we earn (c)
(e) No correction required
33. Ramesh took charge of the project, within a few days of
having appointed ?
40.
eer
/ last year on purchasing new computers, (d) / No error (e).
The Government has (a) / promised to revise (b) / the

(a) having an appointment(b) being appointed


(c) after being appointed (d) appointing
error (e)
ing
pension scheme for bank (c) / staff since next year. (d) / No

(e) No correction required


.ne
t

27. a b c d e 28. a b c d e 29. a b c d e 30. a b c d e 31. a b c d e


RESPONSE
32. a b c d e 33. a b c d e 34. a b c d e 35. a b c d e 36. a b c d e
GRID
37. a b c d e 38. a b c d e 39. a b c d e 40. a b c d e

Downloaded From : www.EasyEngineering.net


Downloaded From : www.EasyEngineering.net

Computer Fundamentals/
Binary System/
Operating System
72
Max. Marks : 30 No. of Qs. 30 Time : 20 min. Date : ........./......../. ..............

1. Most of the commonly used personal computers/laptops 9. How many megabytes make a gigabyte?
do not have a command key known as ________. (a) 1024 (b) 128
(a) Turnover (b) Shift (c) Alter (c) 256 (d) 512
(d) Delete (e) Insert
(e) 64
2. Which of the following is NOT a hardware of a computer?
10. Which of the following categories would include a keyboard?

ww
(a) Monitor
(c) Windows
(e) Mouse
(b) Key Board
(d) Central Processing Unit
(a) Printing Device
(c) Pointing Device
(b) Output Device
(d) Storage Device
3.
w.E
Most of the commonly available personal computers/laptops
have a keyboard popularly known as ________. 11.
(e) Input Device
Which of the following is hardware and not software ?

4.
(a) QWERTY
(d) UCLIF
(b) QOLTY
(e) None of these
asy
(c) ALTER

Computers send and receive data in the form of ________


(a) Excel (b) Printer driver
(c) Operating System (d) Power Point
(e) CPU
signals.
(a) Analog (b) Digital En 12. A _________ is an electronic device that process data,
converting it into information.
(c) Modulated
(e) All of these
(d) Demodulated
gin (a) computer
(c) case
(b) processor
(d) stylus
5. The smallest unit of information a computer can understand
and process is known as a_______.
13. eer
(e) None of these
A computer works on a _________ number system.
(a) digit
(d) kilobyte
(b) byte
(e) bit
(c) megabyte
(a) binary
(c) decimal
ing (b) octal
(d) hexadecimal
6. A byte can represent any number between 0 and_____
(a) 2
(d) 1024
(b) 255
(e) 1025
(c) 256 (e) None of these
.ne
The physical components of a computer system is _______
7. External devices such as printers, keyboards and modems
are known as
(a) add-on devices.
14.
(a) Software
(c) ALU
(e) None of these
(b) Hardware
(d) Control Unit
t
(b) peripherals.
15. Files deleted from the hard disk are sent to the ______.
(c) extra hardware devices.
(a) recycle bin (b) floppy disk
(d) PC expansion slot add-ons.
(c) clipboard (d) motherboard
(e) special-buys
8. The most common pointing input device is the (e) None of these
(a) trackball (b) touchpad 16. Which of the following is NOT a famous operating system?
(c) touchscreen (d) mouse (a) Windows Vista (b) Mac OS X (c) Linux
(d) Sun OS (e) Virtual Box
(e) scanner

1. a b c d e 2. a b c d e 3. a b c d e 4. a b c d e 5. a b c d e
RESPONSE 6. a b c d e 7. a b c d e 8. a b c d e 9. a b c d e 10. a b c d e

GRID 11. a b c d e 12. a b c d e 13. a b c d e 14. a b c d e 15. a b c d e


16. a b c d e

Downloaded From : www.EasyEngineering.net


Downloaded From : www.EasyEngineering.net

154 SPEED TEST 72


17. Window 95, Windows 98 and Windows NT are known as 25. Executing more than one program concurrently by one user
(a) processors (b) domain names (c) modems on one computer is known as
(d) operating systems (e) None of these (a) multi-programming (b) multi-processing
18. Two different files can have the same name if (c) time sharing (d) multi-tasking
(a) they are in different folders (e) multi-action
(b) they are on different drives 26. All computers must have
(c) Never (a) a word processing software
(d) the names are capitalised differently (b) an operating system
(e) None of these (c) an attached printer
19. Every computer has a(n) ____________; many also have (d) a virus checking program
__________.
(e) None of these
(a) operating system; a client system
27. When you instal a new program on your computer,it is typi-
(b) operating system; instruction sets
cally added to the _________ menu.

ww
(c) application programs; an operating system
(d) application programs; a client system
(a) All Programs
(c) Start Programs
(b) Select Programs
(d) Desktop Programs

20.
w.E
(e) operating system; application programs
Applications are often referred to as
(a) data file (b) executable files
28.
(e) None of these
The operating system, that is self-contained in a device and

(c) system software


(e) None of these asy
(d) the operating system
resident in the ROM .is
(a) Batch Operating System

21. The process of transferring files from a computer on the


Internet to your computer is called __________. En (b) Real-time Operating System
(c) Embedded Operating System
(a) downloading
(d) JPEG
(b) uploading
(e) downsizing
(c) FTP
gin (d) Mutli-Processor Operating System
(e) None of these
22. The operating system called UNIX is typically used for
(a) Desktop computers
29.
performing
eer
If you change Windows 98 to Windows XP, you are actually

(b) Laptop computers


(c) Super computers
(a) upstart
(c) update ing
(b) upgrade
(d) patch
(d) Web servers
(e) All of these 30.
(e) None of these
.ne
What happens when we try to delete the files on the floppy?
23. When data changes in multiple lists and all lists are not
updated, this causes
(a) data redundancy (b) information overload
(a) The files get moved to the Recycle Bin
(b) Files on a floppy cannot be deleted
(c) The files get deleted and can be restored again from
t
(c) duplicate data (d) data inconsistency Recycle Bin .
(e) data repetition (d) The files get deleted and cannot be restored again
24. Which process checks to ensure the components of the (e) The file gets copied on the Hard disk
computer are operating and connected properly?
(a) Booting (b) Processing (c) Saving
(d) Editing (e) Starting

17. a b c d e 18. a b c d e 19. a b c d e 20. a b c d e 21. a b c d e


RESPONSE 22. a b c d e 23. a b c d e 24. a b c d e 25. a b c d e 26. a b c d e
GRID 27. a b c d e 28. a b c d e 29. a b c d e 30. a b c d e

Downloaded From : www.EasyEngineering.net


Downloaded From : www.EasyEngineering.net

MS Office/ Commands
and Shortcut Keys 73
Max. Marks : 30 No. of Qs. 30 Time : 20 min. Date : ........./......../. ..............

1. The name of a Microsoft Office Word document is displayed 8. The .xls extension is used for ________ files.
in both the ___________ and the taskbar. (a) Windows (b) Access (c) PowerPoint
(a) menu bar (b) taskbar (d) Word (e) Excel
(c) Formatting toolbar (d) Standard toolbar 9. In word, when you indent a paragraph, you
(e) title bar (a) push the text in with respect to the margin
2.

ww
Microsoft Office is an example of a _________.
(a) closed-source software
(b) open-source software
(b) change the margins on the page
(c) move the text up by one line
(d) move the text down by one line

w.E
(c) horizontal-market software
(d) vertical-market software 10.
(e) None of these
Using Print Preview is useful when you want to-
(a) Colour the document

asy
(e) compiler
3. You cannot link Excel worksheet data to a Word document (b) Save the document
___________. (c) Delete the document
(a) with the right drag method
(b) with the hyperlink En (d) Copy the document
(e) View how trip document will appear when printed
(c) with the copy and paste special commands
(d) with the copy and paste buttons on the standard gin
11. In word, you can change Page Margins by
(a) Dragging the scroll box on the scroll bars
(b) Deleting the margin boundaries on the Ruler
commands
(e) All of these
eer
(c) Dragging the margin boundaries on the Ruler
(d) Clicking the right mouse button on the Ruler
4. In Excel, Charts are created using which option?
(a) Chart Wizard
(c) Pie Chart
(b) Pivot Table
(d) Bar Chart
12.
(e) None of these
ing
In Excel, this is a prerecorded formula that provides a shortcut

5.
(e) None of these
Each cell in a Microsoft Office Excel document is referred to
for complex calculations
(a) Value
.ne
(b) Data Series

t
(c) Function (d) Field
by its cell address, which is the (e) None of these
(a) cell’s column label 13. This is not a function category in Excel
(b) cell’s column label and worksheet tab name (a) Logical (b) Data Series
(c) cell’s row label (c) Financial (d) Text
(d) cell’s row and column labels (e) None of these
(e) cell’s contents 14. Text in a column is generally aligned _________
6. This Excel feature includes functions to calculate an Average, (a) justified (b) right
Minimum, Maximum and Count. (c) center (d) left
(a) Format (b) Number (c) AutoSum (e) None of the above
(d) Calculate (e) MIN 15. In Excel _________ contains one or more worksheets.
7. This is a set of values that you want to chart in Excel. (a) Template (b) Workbook
(a) Object (b) Numbers (c) Data Mart (c) Active cell (d) Label
(d) Formulas (e) Data series (e) None of these

1. a b c d e 2. a b c d e 3. a b c d e 4. a b c d e 5. a b c d e
RESPONSE 6. a b c d e 7. a b c d e 8. a b c d e 9. a b c d e 10. a b c d e
GRID 11. a b c d e 12. a b c d e 13. a b c d e 14. a b c d e 15. a b c d e

Downloaded From : www.EasyEngineering.net


Downloaded From : www.EasyEngineering.net

156 SPEED TEST 73


16. You click at B to make the text _________. 23. ________ is the key to close a selected drop - down list;
(a) Italics cancel a command and close a dialog box.
(b) Underlined (a) TAB (b) SHIFT
(c) Italics and Underlined (c) ESC (d) F10
(d) Bold (e) None of these
(e) None of these 24. ________ is the function key to display save-as box.
17. The shortcut key Ctrl + F in Word is used for (a) F5 (b) F6
(a) To view document in full view (c) F9 (d) F12
(b) To open the Formula dialog box (e) None of these
(c) To save the file 25. ________ is the Keyboard shortcut key to insert auto sum
(d) To open the Find and Replace dialog box (a) ALT (b) ALT=
(e) None of these (c) ALT+ (d) ALT–
18. To restart the computer ________ key is used. (e) ALT+CTRL
(a) Del + Ctrl (b) Backspace + Ctrl 26. To restart the computer the following combination of keys

ww
(c) Ctrl + Alt + Del
(e) None of these
(d) Reset

19. ctrl, shift and alt are called ________ keys.


is used
(a) Del + Ctrl
(c) Esc + Ctrl
(b) Backspace + Ctrl
(d) Insert + Esc
(a) adjustment
(c) modifier w.E (b) function
(d) alphanumeric 27.
(e) Ctrl + Alt + Del
To make a copy of the current document to disk
(a) Use the “save” command
(e) None of these
20. To select or unselect one word to the right
(a) CTRL + SHIFT asy (b) This cannot be done
(c) Use the “duplicate” command
(b) CTRL + SHIFT + UP Arrow
(c) CTRL + SHIFT + Down Arrow En (d) Copy the document
(e) Use the “save as” command
(d) CTRL + SHIFT + Right Arrow
(e) None of these gin
28. What type of keys are ‘ctrl’ and ‘shift’?
(a) adjustment (b) function
21. To maximize or restore a selected window
(a) CTRL + F7 (b) CTRL + F10
(c) modifier

eer
(e) None of these
(d) alphanumeric

(c) CTRL + F8
(e) None of these
(d) CTRL + F9 29.
grammar and spelling?
(a) F3 ing
Which among the following key is used for checking

(b) F5
22. To Copy a picture of the selected window to the clipboard
(a) ALT + TAB
(b) Ctrl + TAB
(c) F7
(e) None of these
(d) F2
.ne
(c) ALT + Print Screen
(d) Both (a) and (b)
(e) None of these
30.
used for full screen view?
(a) F3 (b) F5
t
While browsing the internet , which of the following key is

(c) F11 (d) F9


(e) None of these

RESPONSE 16. a b c d e 17. a b c d e 18. a b c d e 19. a b c d e 20. a b c d e


21. a b c d e 22. a b c d e 23. a b c d e 24. a b c d e 25. a b c d e
GRID 26. a b c d e 27. a b c d e 28. a b c d e 29. a b c d e 30. a b c d e

Downloaded From : www.EasyEngineering.net


Downloaded From : www.EasyEngineering.net

Softwares/
Programming 74
Max. Marks : 30 No. of Qs. 30 Time : 20 min. Date : ........./......../. ..............
1. What kind of software would you most likely use to keep 8. A sales clerk at a checkout counter scanning a tag on an
track of a billing account? item rather than keying it into the system, is using _______.
(a) Word processing (b) Electronic publishing (a) input automation (b) item data automation
(c) Spreadsheet (d) Web authoring (c) scanning automation
(e) None of these (d) source data automation
(e) None of these
2.

ww
The two major categories of software include
(a) operating system and utility
(b) Personal productivity and system
9. A document that explains how to use a software program is
called ______ manual
(a) User (b) System

w.E
(c) system and application
(d) system and utility
(e) None of these 10.
(c) Software
(e) Technical
(d) Program

The process that deals with the technical and management


3. A directory within a directory is called.
(a) Mini Directory asy
(b) Junior Directory
issues of software development is ______
(a) Delivery process (b) Control process
(c) Part Directory
(e) None of these
(d) Sub Directory

En (c) Software process (d) Testing process


(e) Monitoring process
4. Which of the following is not a common feature of software
applications?
(a) Menus (b) Windows gin
11. Linux is a type of ________ software.
(a) Shareware
(c) Proprietary
(b) Commercial
(d) Open Source
(c) Help
(e) None of these
(d) Search
12. eer
(e) Hidden type
The _________ of a system includes the programs or
5. Software applies __________, also called algorithms, to
process data.
instructions.
(a) hardware
(c) information
ing
(b) icon
(d) software

.ne
(a) arithmetic (b) procedures
(c) objects (d) rules (e) None of these
(e) None of these 13. The software that allows users to surf the Internet is called
6. Application software is designed to accomplish _________.
(a) real-world tasks (b) computer-centric tasks
(c) gaming tasks (d) operating-system tasks
a/ an _________
(a) Search engine
(c) Multimedia application
(d) Browser (e) None of these
t
(b) Internet Service Provider (ISP)

(e) None of these


14. The ______ of software contains lists of commands and
7. Which is the best definition of a software package?
options.
(a) An add-on for your computer such as additional
(a) menu bar (b) tool bar
memory
(c) title bar (d) formula bar
(b) A set of computer programs used for a certain function
(e) None of these
such as word processing
15. ____ is a procedure that requires users to enter an
(c) A protection you can buy for a computer
identification code and a matching password.
(d) The box, manual and license agreement that accom-
(a) Paging (b) Logging on
pany commercial software.
(c) Time-sharing (d) Multitasking
(e) None of these
(e) None of these

1. a b c d e 2. a b c d e 3. a b c d e 4. a b c d e 5. a b c d e
RESPONSE 6. a b c d e 7. a b c d e 8. a b c d e 9. a b c d e 10. a b c d e
GRID 11. a b c d e 12. a b c d e 13. a b c d e 14. a b c d e 15. a b c d e

Downloaded From : www.EasyEngineering.net


Downloaded From : www.EasyEngineering.net

158 SPEED TEST 74


16. A program that works like a calculator for keeping track of 23. A program that enables you to perform calculations
money and making budgets __________. involving rows and columns of numbers is called a
(a) calculator (b) scholastic (c) keyboard _________.
(d) spreadsheet (e) None of these (a) spreadsheet program
17. Compiling creates a(n) ___________. (b) word processor
(a) program specification (c) graphics package
(b) algorithm (c) executable program (d) window
(d) subroutine (e) None of these (e) None of the above
18. Multiprogramming systems: 24. ______ is a feature for scheduling and multiprogramming
(a) are easier to develop than single programming systems. to provide an economical interactive system of two or more
(b) execute each job faster. users
(c) execute more jobs in the same time period. (a) Time sharing (b) Multitasking
(d) use only one large mainframe computer. (c) Time tracing (d) Multiprocessing
(e) None of these (e) None of these
25. A programming language having a ______ is slow in
19.

ww
A ________ contains specific rules and words that express
the logical steps of an algorithm.
(a) programming language
execution
(a) Interpreter (b) Compiler

(c) syntax
(d) logic chart
w.E
(b) programming structure

26.
(c) Assembler
(e) none of these
Assembly language is
(d) Linker

20.
(e) None of these
asy
_________ is a set of keywords, symbols, and a system of
(a) Machine Language
(b) High-level programming language
rules for constructing statements by which humans can

En
communicate the instructions to be executed by a computer.
(c) A low-level programming language
(d) Language for assembling computers
(a) A computer program
(b) A programming language
(c) An assembler
gin
27.
(e) None of these
What is correcting erros in a program called?
(a) Compiling (b) Debugging
(d) Syntax
(e) None of these
(c) Grinding
eer
(e) None of these
(d) Interpreting

21. ________ is the process of finding errors in software code.


(a) Compiling
28.
(a) utility ing
A (n)___is a program that makes the computer easier to use.
(b) application
(b) Assembling
(c) Interpreting
(c) operating system
(e) None of these
(d) network
.ne
22.
(d) Debugging
(e) None of these
Documentation of computer programs is important so that
(a) users can learn how to use the program
29.
and guides the user through certain steps?
(a) Software
(c) Wiki
(b) Wizard
(d) Language
t
What is used in most programs that is a part of a program

(b) other programmers can know how to maintain the (e) None of these
program 30. Which of the following was used in programming the first
(c) the programmer can see why the code is written that computers?
way while hunting for sources of error (a) Object code (b) Source Code
(d) All of the above (c) Machine Language (d) Assembly Language
(e) None of the above (e) None of these

RESPONSE 16. a b c d e 17. a b c d e 18. a b c d e 19. a b c d e 20. a b c d e


21. a b c d e 22. a b c d e 23. a b c d e 24. a b c d e 25. a b c d e
GRID 26. a b c d e 27. a b c d e 28. a b c d e 29. a b c d e 30. a b c d e

Downloaded From : www.EasyEngineering.net


Downloaded From : www.EasyEngineering.net

Internet, Networking and


Computer Abbreviations
75
Max. Marks : 30 No. of Qs. 30 Time : 20 min. Date : ........./......../. ..............

1. Which of the following is a Web browser? 9. Which of the following is not a term pertaining to the
(a) Paint (b) Power Point Internet?
(c) Fire fox (d) Word (a) Keyboard (b) Link
(e) All are Web browsers (c) Browser (d) Search Engine
2. Junk e-mail is also called __________.

ww
(a) spam
(c) sniffer script
(e) None of these
(b) spoof
(d) spool
10.
(e) Hyperlink
In a web site, the ‘home’ page refers to –
(a) the best page (b) the last page
3.
w.E
Most World Wide Web pages contain commands in the
language _________.
(a) NIH (b) URL 11.
(c) the first page
(e) the oldest page
(d) the most recent page

An e-mail address typically consists of a user ID followed


(c) HTML
(e) FTP
(d) IRC
asy by the _________ sign and the name of the e-mail server
that manages the user’s electronic post office box.
4. Computers connected to a LAN (local Area Network)
can _________.
En (a) @
(c) &
(b) #
(d) «
(a) run faster
(b) go on line
(c) share information and/or share peripheral equipment gin
12.
(e) None of these
A Web __________ consists of one or more Web pages
(d) E-mail
(e) None of these (a) hub eer
located on a Web server.
(b) site
5. What does a Web site address uniquely specify?
(a) Web browser
(c) PDA
(b) Web site
(d) Storage
(c) story
(e) None of these
ing
(d) template

6.
(e) Hard-disk
Web pages are saved in __________ format.
13.
windows into the Web are called
(a) Hypertext (b) Networks .ne
Programs such as Internet Explorer that serve as navigable

7.
(a) http://
(c) DOC
(e) None of these
(b) HTML
(d) URL

What are the two parts of an E-mail address? 14.


(c) Internet
(e) None of these
(d) Web browsers

A word in a web page that, when clicked, opens another


t
(a) User name and street address document.
(b) Legal name and phone number (a) anchor (b) URL
(c) User name and domain name (c) hyperlink (d) reference
(d) Initials and password (e) None of these
(e) login name and password 15. WWW stands for ________.
8. An educational institution would generally have the (a) World Work Web (b) Wide Work Web
following in its domain name –
(c) Wide World Web (d) World Wide Web
(a) .org (b) .edu
(e) None of the above.
(c) .inst (d) .com
(e) .sch

1. a b c d e 2. a b c d e 3. a b c d e 4. a b c d e 5. a b c d e
RESPONSE 6. a b c d e 7. a b c d e 8. a b c d e 9. a b c d e 10. a b c d e
GRID 11. a b c d e 12. a b c d e 13. a b c d e 14. a b c d e 15. a b c d e

Downloaded From : www.EasyEngineering.net


Downloaded From : www.EasyEngineering.net

160 SPEED TEST 75


16. The collection of links throughout the Internet creates an 23. FAT stands for
interconnected network called the _________. (a) File Activity Table
(a) WWW (b) Web (b) File Allocation Table
(c) World Wide Web (d) All of the above (c) File Access Tape
(e) Wide Area Web (d) File Accommodation Table
17. ASCII stands for (e) None of these
(a) American Special Computer for Information Interaction 24. ‘IMAP’ in computer language stands for
(b) American Standard Computer for Information (a) Internet Message Access Protocol
Interchange (b) Internet Money Access Protocol
(c) American Special Code for Information Interchange (c) Intrusion Message Access Protocol
(d) Internal Message Access Protocol
(d) American Special Computer for Information
(e) International Marketing Authority Protocol
Interchange
25. The BIOS is the abbreviation of
(e) American Standard Code for Information Interchange
(a) Basic Input Output System
18. POST stands for
(b) Best Input Output System

ww
(a) Power on Self Test
(b) Program on Self Test
(c) Power on System Test
(c) Basic Input Output Symbol
(d) Base Input Output System
(e) None of these

19.
w.E
(d) Program on System Test
(e) Power Off System Test
CPU stands for _________
26. The computer abbreviation KB usually means
(a) Key Block
(c) Key Byte
(b) Kernal Boot
(d) Kit Bit
(a) Computer Processing Unit
(b) Central Processing Unit asy 27.
(e) Kilo Byte
URL stands for
(c) Computer Protection Unit
(d) Central Processing Upload
En (a) Universal Resource List
(b) Universal Research List

20.
(e) None of the above
OCR stands for _________ gin (c) Uniform Resource List
(d) Uniform Research Locater
(e) Uniform Resource Locater
(a) Optical Character Recognition
(b) Optical CPU Recognition
28.
eer
The meaning of double-click is
(a) pushing and releasing the main mouse button twice in
(c) Optimal Character Rendering
(d) Other Character Restoration
(e) None of these ing
rapid succession when the on-tree mouse pointer is
positioned over the desired item

21. Line printer speed is specified in terms of :


(a) LPM (Line per minute)
(b) appearance of an icon

.ne
(c) to take a selection from the document and move it to
the clipboard
(b) CPM (Character per minute)
(c) DPM
(d) Any of the above 29.
(d) All of the above
(e) None of these
BPS stands for
t
(e) None of these (a) Bits Per Second (b) Bits Per Season
22. What does ‘DOS’ stand for ? (c) Bytes Per Second (d) Bits Per System
(a) Disk Originating System (e) None of these
(b) Dynamic Operating System 30. Reusable optical storage will typically have the
acronym _________
(c) Disk Operating System
(a) CD (b) DVD
(d) Default Operating System
(c) ROM (d) RW
(e) None of these
(e) None of these

RESPONSE 16. a b c d e 17. a b c d e 18. a b c d e 19. a b c d e 20. a b c d e


21. a b c d e 22. a b c d e 23. a b c d e 24. a b c d e 25. a b c d e
GRID 26. a b c d e 27. a b c d e 28. a b c d e 29. a b c d e 30. a b c d e

Downloaded From : www.EasyEngineering.net


Downloaded From : www.EasyEngineering.net

76
Fundamentals of
Marketing,
Product and Branding
Max. Marks : 30 No. of Qs. 30 Time : 20 min. Date : ........./......../. ..............

1. Marketing is: (c) Lifelong relationship with the buyer


(a) Only selling (d) All of these (e)
(b) meeting human & social needs while earning profits None of these
(c) focus on customer 9. A Market Plan is ________.
(d) focus on producing goods/ service (a) company's prospectus (b)
(e) Both (b) and (c) Memorandum of Association (c)
2.
ww
Long term objective of marketing is
(a) customer satisfaction
(b) profit maximisation
document for marketing strategies
(d) business goals (e)
action plan for better production
(c) cost cutting
w.E
(d) profit maximisation with customer
(e) None of these
10. Marketing helps in _______.
(a) boosting production (b)
getting new clients (c)
3. Market information means
(a) knowledge of companies
(b) cross-country information asy interacting with strangers
(d) All of these (e)
None of these
(c) knowledge of related markets
(d) knowledge of current customers
(e) None of these En 11. Marketing is the art of ________.
(a) buying more (b) paying more
4. Marketing and Selling are
(a) not required if profit is high gin
12.
(c) selling more
(e) only (a) & (b)
Selling is _______.
(d) talking more

(b) not required if sales are high


(c) not required in monopolistic conditions
(d) All of the above eer
(a) different from Marketing
(b) a sub-function of marketing

5.
(e) None of these
Direct Marketing is neccessary for
(a) having a focussed approach
(c) same as Marketing
(d) more than Marketing
(e) All of these ing
(b) boosting sales
(c) better customer contacts
(d) All of the above
13. Social Marketing is _______.
(a) Share market prices .ne
6.
(e) None of these
Marketing is required for
(a) boosting production
(b) reducing costs
(b) Marketing by the entire society
(c) Internet Marketing
(d) Marketing for a social cause
(e) Society bye-laws
t
(c) boosting profits 14. Marketing is the combined study of
(d) improving customer service (a) Buyer’s behaviour and consumer tasks
(e) All of the above (b) Product demand and Product supply
7. Marketing is successful when (c) Brand building and Publicity
(a) demand exceeds supply (d) Sales force abilities and customer response
(b) supply exceeds demand (e) All of the above
(c) exports are heavy and costly 15. The sales process begins with
(d) salesmen are effective (a) customer identification
(e) All the above (b) lead generation
8. In marketing the benefits of selling extend to (c) sales presentation
(a) Only products and services (d) sales closure
(b) Only after sales services (e) sales meet

1. a b c d e 2. a b c d e 3. a b c d e 4. a b c d e 5. a b c d e
RESPONSE 6. a b c d e 7. a b c d e 8. a b c d e 9. a b c d e 10. a b c d e
GRID 11. a b c d e 12. a b c d e 13. a b c d e 14. a b c d e 15. a b c d e

Downloaded From : www.EasyEngineering.net


Downloaded From : www.EasyEngineering.net

162 SPEED TEST 76


16. XXX is selecting and analyzing a target market and 24. To investigate new markets _____ management function is
developing a marketing mix to gain long-run competitive important.
advantages. XXX is creating a. (a) Finance functions. (b) Marketing.
(a) Corporate strategy (b) Target design (c) Production. (d) HRM.
(c) Mix strategy (d) Marketing strategy (e) None of these
(e) None of these 25. _____ is a “category killer”.
17. Critical success factors for a firm includes ________. (a) Products which stock must have products that need to
(a) Changing lifestyles and attitudes be stocked by retailers due to consumer demand like
(b) Low-cost production efficiency Coca Cola & Kellogg's.
(c) Both (a) and (b) (b) These are speciality stores with a deep product line to
(d) Marketing strategy be sold in restricted shop space
(e) None of the above (c) These are retail outlets with a narrow product focus
18. A differentiated product may be unique by itself but it will but sell products at low prices by bulk buying, low
only be successful only ________ . margins and selling high volumes
(a) if it satisfies customers' needs (d) These are retail outlets with a wide product focus but

ww
(b) if price differential is minimal
(c) if brand can be classed as aspirational
(d) differentiated products will always be successful
with a wide width and depth to products
(e) These are retail outlets with a narrow product focus
but with wide width and depth

19.
(e) None of these

w.E
Establishing and maintaining a distinctive place in the market
for an organization/product is ________.
26. Product life cycle theory maximizes profit at.
(a) Developed Stage
(c) Matured Stage
(b) Early Stage
(d) Declined Stage
(a) Profiling
(c) Segmentation
(e) None of these
asy
(b) Profiling segmentation
(d) Positioning 27.
(e) Cannot be predicted
Marketing Plans are used for

20. _____ of ad means how frequently you should expose your


target group to your message. En (a) doing Research by Marketing students
(b) planning Departments

(a) Frequency
(c) Copy strategy
(b) Copy
(d) Media gin (c) purchase of consumable items from retail outlets
(d) All of the above
(e) None of these

21.
(e) None of these
Market is divided into groups on the basis of age, family
28.
eer
Product mix means
(a) distributing mix products
size, gender, income, occupation, education, religion, race,
generation, nationality, or social class is the best description
of _____ .
(c) satisfying the customer
ing
(b) collecting ideas to sell better

(a) Demographics
(c) Behavioral
(b) Psychographics
(d) Geographic
29.
(e) products designed by the Company
Buyer Resistance' means _____. .ne
(d) bundle of products required by the customer

22.
(e) None of these
What approach should a brand manager adopt to know the
status of a brand in terms of consumer perceptions?
(a) Compare two or three brands
(a) Buyer's interest in the product
(b) Buyer being aggressive with the seller
(c) Buyer's hesitation in buying the product
t
(b) Analyze market segmentations (d) Buyer becoming a seller
(b) Select potential target markets (e) Buyer purchasing the product
(c) Understand customer's needs
30. Direct Marketing is useful for _____ .
(e) None of these
(a) Designing Products
23. Introducing additional items in the same product category
by adding new flavors, forms, colors, ingredients or package (b) Sending e-mails
sizes, under the same brand name, is _____. (c) Increased production
(a) Line extensions (b) Product mix (d) Bigger job opportunities
(c) Interactive marketing (d) Service intangibility (e) None of these
(e) None of these

16. a b c d e 17. a b c d e 18. a b c d e 19. a b c d e 20. a b c d e


RESPONSE 21. a b c d e 22. a b c d e 23. a b c d e 24. a b c d e 25. a b c d e
GRID 26. a b c d e 27. a b c d e 28. a b c d e 29. a b c d e 30. a b c d e

Downloaded From : www.EasyEngineering.net


Downloaded From : www.EasyEngineering.net

Market Situations Based on


Price, Distribution,
Promotion and Advertising
77
Max. Marks : 30 No. of Qs. 30 Time : 20 min. Date : ........./......../. ..............
1. A company's own retail outlets are meant _____. 8. Advertising copy _____.
(a) To avoid the threat of distributors' power (a) Provides continuity in a brand's advertising (b)
(b) To own and control the distribution channel Help a brand achieve distinctiveness (c)
(c) Distribution is profitable Provides a common benchmark on which all concerned
(d) All of the above in the company and the agency can evaluate merits of

2.
ww
(e) None of the above
Advertising _____ can attract consumers only if it is based
on their needs.
advertising campaign
(d) All of the above
(e) None of these
(a) Reach
(c) Frequency
(e) None of these w.E (b) Copy
(d) Media
9. Duration of _____ should be short and should not be
repeated.
(a) Sales promos
(c) Brand promos
(b) Market promos
(d) Product promos
3.

(c) Distributor
(b) Brand
(d) Customer asy
Major source of power in a distribution channel is the _____.
(a) Company 10.
(e) None of these
An effective advertising campaign _____
(a) Revolves around a strong central idea

4.
(e) None of these
Factors affecting choice of distribution channel include
En (b) Should appeal to consumers self interest
(c) Must not be generalised
_____.
(a) Customer value (b) Sales revenues
(c) Both [a] and [b] (d) Customer services
gin
11.
(d) All of the above
(e) None of these
Prices of luxury product are explained by _____ .

5.
(e) None of the above
Air India runs TV commercials that show its staff going out eer
(a) Plus-one pricing
(b) Skim pricing
(c) Strategic account pricing
of their way to help customers. An important secondary
audience for these ads is:
(a) Civil aviation authority 12.
(d) Segment pricing
(e) None of these
ing
_____ about Place/Distribution decisions is CORRECT?
(b) Competitors
(c) Air India employees .ne
(a) Product classes are not related to Place objectives
(b) The product life cycle is not related to Place objectives

6.
(d) Air travelers
(e) None of these
Pricing models offers opportunity to set different prices for
different needs is _____.
(c) Place decisions are short-term decisions that are easy
to change
(d) Different market segments may require separate Place
arrangements.
(e) None of these
t
(a) Segment pricing 13. Addition of 2.25 liter bottle by Coca Cola will ____ .
(b) Skim pricing (a) Increase customer base and usage
(c) Value-in-use pricing (b) Enhance customer loyalty
(c) Generate more profit
(d) Strategic account pricing
(d) Develop brand image
(e) None of these (e) All of the above
7. Advertising helps sales promotion by creating awareness 14. Communication through a news story about an organization
and comprehension that creates _____. and its products that is transmitted through a mass medium
(a) Customer pull (b) Customer push at no charge is ____.
(c) Customer loyal (d) Customer image (a) Advertising (b) Sales promotion
(e) None of these (c) Personal selling (d) Publicity
(e) None of these

1. a b c d e 2. a b c d e 3. a b c d e 4. a b c d e 5. a b c d e
RESPONSE 6. a b c d e 7. a b c d e 8. a b c d e 9. a b c d e 10. a b c d e
GRID 11. a b c d e 12. a b c d e 13. a b c d e 14. a b c d e

Downloaded From : www.EasyEngineering.net


Downloaded From : www.EasyEngineering.net

164 SPEED TEST 77


15. _____ is a disadvantage of using an agent in a channel of 22. If Marketing is done effectively, _____ is not required
distribution? (a) Advertisment (b) Pubilicity
(a) Length of channel (b) Cost factor (c) Market Research (d) Market Segmentation
(c) Lack of control (d) Speed of distribution
(e) None of these
(e) Absence of market knowledge
23. Sales forecasting involves
16. _____ enables a company to contr ol channels of
(a) sales planning (b) sales pricing
distribution?
(c) distribution channels (d) consumer tastes
(a) Vertical marketing system
(e) All of these
(b) Franchising
24. Proper pricing is essential for ____.
(c) Exclusive dealing arrangements
(a) extra charges for extra services
(d) Vertical integration
(b) levy of VAT
(e) All of the above
(c) good customer service
17. Distribution where a limited number of outlets in a
(d) putting burden on the customer
geographical area to sell its products are used is called
(e) service with extra facilities
_____?
25. Selling skills are measured by ____.
(a) Exhaustive distribution
(a) number of goods sold

ww
(b) Exclusive distribution
(c) Intensive distribution
(d) Selective distribution
(b) amount of profit earned
(c) number of customers converted
(d) All of the above
18.
(a) Distributor
w.E
(e) Segmented distribution
Power in marketing channels is now with _____ ?
(b) Manufacturer
26.
(e) None of these
Promotion means
(a) additional responsibility

19.
(c) Retailer
(e) Consumer
(d) Wholesaler
asy
______ is a strength associated with franchising?
(b) undertaking research in marketing
(c) advertisement and publicity for marketing
(a) Reduces marketing promotional and administration
costs En (d) going up the promotional ladder
(e) All of the above
(b) Goal conflict does not arise
(c) Reduces levels of channel conflict within the channel gin
27. Negotiation skills help in ____.
(a) evolving a consensus
(b) breaking the ice
(d) Combine the strengths of a large sophisticated
marketing-oriented organization with the energy and
motivation of a locally owned outlet eer
(c) carry marketing further
(d) Mutual win-win situation

20.
(e) All of the above
Best method of advertisement is
28.
(e) All of these

ing
Relationship Marketing is useful for
(a) trade between relatives
(a) glow sign boards (b) internet
(c) Television
(e) customer satisfaction
(d) Consumer awareness
(b) trade between sister concerns
(c) cross-selling of products .ne
21. A form of distribution in which manufacturer makes an agreement
with a middleman in each market stipulating that the distribution
of the product will be confined in that area is:
(a) mass distribution
29.
(d) preparing a list of relatives
(e) There is no such term as Relationship Marketing
Good Public Relations means improved _____.
(a) Marketing skills (b) Brand Image
t
(c) Customer Service (d) All of these
(b) exclusive agency distribution
(e) None of these
(c) selective distribution
30. One method of Market Monitoring is ____.
(d) price based distribution
(a) Monitor Performance of sales staff.
(e) None of these
(b) Monitor the SENSEX
(c) Monitor Media Outlets
(d) Monitor profits
(e) None of these

15. a b c d e 16. a b c d e 17. a b c d e 18. a b c d e 19. a b c d e


20. a b c d e 21. a b c d e 22. a b c d e 23. a b c d e 24. a b c d e
RESPONSE
25. a b c d e 26. a b c d e 27. a b c d e 28. a b c d e 29. a b c d e
GRID
30. a b c d e

Downloaded From : www.EasyEngineering.net


Downloaded From : www.EasyEngineering.net

Market Segmentation,
Targeting and Positioning
78
Max. Marks : 30 No. of Qs. 30 Time : 20 min. Date : ........./......../. ..............
1. 4 P's of Marketing are ____. 9. Positioning has to stem from the point of view of _____.
(a) Primary Marketing Techniques (a) Customers' (b) Competitors'
(b) Person, Place, Product and Promotion (c) General Managers' (d) Brand Owners'
(c) Promoting Authority (e) None of these
(d) Purpose, Place, Passion, and Product 10. _____ is a marketer's major positioning tool that has a direct
(e) None of these impact on product performance and is linked to customer
2.
ww
Qualities essential in good marketing are ____.
(a) aggressiveness (b) pushy
value.
(a) Product quality (b) Social marketing

3.
(c) perseverance
(e) Only (c) and (d)
w.E (d) politeness

______ is not a part of 7 P's of Marketing. 11.


(c) Specialty marketing (d) Production quality
(e) None of these
______ determines why customers buy?
(a) Product
(c) Production
(e) People
(b) Price
(d) Promotion
asy (a) Customer needs analysis
(b) Brand-based customer model
(c) Good brand promise
4. Market Share means _____.
(a) share market (b) share prices
En (d) Brand management process
(e) None of these

5.
(c) IPOs (d) Scope for marketing
(e) Share of business among pproducers
If lots of customers like the brand and are inclined to be gin
12. Marketers need to position their brands clearly in target
customers' minds. The strongest brands go beyond attributes

bound into a contract, they would be known as _____.


(a) Loyal customer (b) Difficult customer
_____.
eer
or benefit positioning. They are positioned on the basis of

(a) Desirable benefit

6.
(c) Potential customer (d) Finicky customer
(e) None of these
_____ evokes a hierarchical set of customer response effects
(b) Good packaging
ing
(c) Strong beliefs and values

i.e. building awareness, comprehension, intentions, and


actions.
(d) Service inseparability
(e) None of these
.ne
7.
(a) Distribution
(c) Merchandizing
(e) None of these
(b) Communication
(d) Branding

Apart from the 'four Ps' of marketing mix, the three additional
13. Target marketing featuring customized marketing
programmes is _______.
(a) Individual marketing (b) Segment marketing
(c) Family marketing (d) Local marketing
t
(e) Niche marketing
elements of service brands are people, process, and ____ .
(a) Physical evidence 14. _____ segmentation divides the market into different units
(b) Physiological evidence like nations, states, regions, cities or neighbourhoods.
(c) Psychological evidence (a) Geographic (b) Demographic
(d) Packaging (c) Psychographic (d) Behavioral
(e) None of these (e) Socio-economic
8. Unique Selling Proposition (USP) started in _____. 15. _____ segmentation is where the market is divided on the
basis of age, family size, life cycle, gender, income,
(a) Advertising era (b) Image era
occupation, education and religion .
(c) Product era (d) Positioning era
(a) Geographic (b) Demographic
(e) None of these (c) Psychographic (d) Behavioral
(e) Socio-economic
1. a b c d e 2. a b c d e 3. a b c d e 4. a b c d e 5. a b c d e
RESPONSE 6. a b c d e 7. a b c d e 8. a b c d e 9. a b c d e 10. a b c d e
GRID 11. a b c d e 12. a b c d e 13. a b c d e 14. a b c d e 15. a b c d e

Downloaded From : www.EasyEngineering.net


Downloaded From : www.EasyEngineering.net

166 SPEED TEST 78


16. To be useful market segments must assess on _____ criteria. 24. Motivation is essential to effective marketing. What other
(a) five (b) two qualities are required for marketing?
(c) three (d) four (a) Confidence
(e) Six (b) Effective Communication skills
17. Market Segmentation means (c) Team work
(a) dividing the market into groups (d) Perseverance
(b) segmenting by age (e) All of the above
(c) segmenting by tastes 25. Market Penetration is possible through
(d) geographic segmenting
(a) more calls to the same buyers
(e) None of the above
(b) more calls to potential buyers
18. In Marketing, Market penetration means
(c) surrogate marketing
(a) entering customers houses
(b) covering stores and shops (d) alternate marketing
(c) covering a wide market (e) All of these
(d) All of the above 26. Market penetration connotes
(e) None of these (a) covering a wider area
(b) entering sellers' houses
19.

ww
Rural Marketing is more effective if arranged through.
(a) melas
(b) village fairs
(c) covering all shops and business houses
(d) All of these

20.
(d) All of these
(e) None of these w.E
(c) door to door campaign

‘Buyer Resistance’ means _____.


27.
(e) None of these
Lead generation means ____.
(a) Tips for selling tactics

(a) Buyer’s interest in the product


(b) Buyer fighting with the seller asy (b) Tips for more efficient production
(c) Develop leaders
(d) sources for prospective clients
(c) Buyer’s hesitation in buying the product
(d) Buyer becoming a seller
(e) Buyer buying the product En 28.
(e) None of these
Marketing orientation focuses on _____?
21. Opportunities for growth and expansion are identified by
finding _____ gin (a) Customers
(c) Competitors
(e) All of the above
(b) Suppliers
(d) Employees
(a) Customers' beliefs about the segment
(b) Customers believe about competitors
(c) Customer's perceptions about the brand
29.
eer
‘Push’ marketing style requires _____.
(a) Proper planning
(d) Customer's response about the products
(e) None of these
(b) Good pushing strength
(c) Teamwork ing
22. What explains Maslow's Motivation Theory best?
(a) Importance of motivation of customer development
(b) When people are driven by particular need at particular 30.
(d) Ability to identify products
(e) Aggressive marketing
.ne
CRM (Customer Relationship Management) is _____.
times
(c) When human needs are arranged in a hierarchy
(d) All of these
(e) Only (b) and (c)
(a) A pre-sales activity
(b) A tool for lead generation
(c) An on going daily activity
t
(d) Task of a DSA
23. Customer Database is used by
(e) Customer complaint cell
(a) individuals (b) institutions
(c) builders (d) marketing experts
(e) None of these

16. a b c d e 17. a b c d e 18. a b c d e 19. a b c d e 20. a b c d e


RESPONSE 21. a b c d e 22. a b c d e 23. a b c d e 24. a b c d e 25. a b c d e
GRID 26. a b c d e 27. a b c d e 28. a b c d e 29. a b c d e 30. a b c d e

Downloaded From : www.EasyEngineering.net


Downloaded From : www.EasyEngineering.net

Modern Marketing /
Marketing in
Banking Industry
79
Max. Marks : 30 No. of Qs. 30 Time : 20 min. Date : ........./......../. ..............
1. Modern marketing includes (b) Tele-marketing (c)
(a) publicity on internet Door-to-door marketing
(b) advertisement on internet
(d) E-mail solicitation (e)
(c) Bulk emails
None of these
(d) telemarketing
(e) All of these 9. Web marketing involves
(a) Selling web cameras (b) Web advertisements
2.
ww
e-marketing is same as _______.
(a) virtual marketing (b) digital marketing
(c) real time marketing (d) All of these
(c) e-mail chatting
(e) Door-to-door canvassing
(d) Browsing the web

3.
(e) None of these

w.E
Digital marketing is selling ______.
(a) digital goods (b) calculators
10. Online Marketing is mostly useful for marketing of
(a) saving accounts
(c) home loans
(e) business accounts
(b) credit cards
(d) NRI deposits

4.
(c) through internet
(e) None of these
Online Marketing is useful for ______. asy
(d) All of these
11. The best promotional tool in any marketing is _______
(a) Pamphlets
(a) Selling Old Products
(b) Sending e-mails
(c) Increasing production En (b) Newsletters
(c) Word of mouth publicity

(d) Additional job opportunities


(e) Higher expenses gin
12.
(d) Regional Advertisement
(e) Viral marketing
Internet marketers are using ________ as a form of word of
5. Online value proposition should _______.
(a) Be communicated to site visitors and in all marketing
communications eer
mouth, or word of mouse, to draw attention to their sites.
(a) event marketing (b) subliminal marketing
(b) Be a clear differentiator from online competitors
(c) Target market segment(s) that the proposition will
13.
(c) viral marketing
(e) public relations
ing
(d) place marketing

The major marketing developments as we enter the new


appeal to
(d) Given financial back up
(e) All of the above
(a) innovation .ne
millennium can be summed up in a single theme:
(b) the Internet
6. Achieving marketing objectives through use of electronic
communications technology is _______.
(a) E-marketing (b) E-business
14.
(c) virtuality
(e) None of these
Today marketing is:
(a) Product driven
(d) connectedness
t
(c) Internet marketing (d) E-commerce
(b) Services driven
(e) None of the above
(c) Improvement of bottom line (Profitability)
7. Direct online contribution effectiveness is the _______.
(d) Cost conscious
(a) Reach of audience volume of a site
(e) Customer driven market
(b) Proportion of sales influenced by the web site
15. Modern marketing calls for more than developing a good
(c) Proportion of business turnover ach ieved by
product, pricing it attractively, and making it accessible.
e-commerce transactions
Companies must also ________ with present and potential
(d) First and third option above
stakeholders, and the general public.
(e) None of the above
(a) attract (b) reach
8. Modern marketing EXCLUDES _______.
(c) relate to (d) advertise to
(a) Digital marketing
(e) communicate
1. a b c d e 2. a b c d e 3. a b c d e 4. a b c d e 5. a b c d e
RESPONSE 6. a b c d e 7. a b c d e 8. a b c d e 9. a b c d e 10. a b c d e
GRID 11. a b c d e 12. a b c d e 13. a b c d e 14. a b c d e 15. a b c d e

Downloaded From : www.EasyEngineering.net


Downloaded From : www.EasyEngineering.net

168 SPEED TEST 79


16. In today's changing banking scenario, aggressive promotion 24. Difference between Direct and indirect Bank Marketing is
of business is necessary where the competition exists on (a) Direct Marketing is to Bank's employees. Indirect is to
(a) branch up-keep outsiders
(b) expeditious service (b) Direct Marketing is to outsiders. Indirect is to
(c) use of advanced digital technology employees
(d) good customer service (c) Direct Marketing is to Bank's owners. Indirect is to
(e) All of these outsiders
17. Bank marketing means (d) Direct Marketing is to other Bank's employees. Indirect
(a) selling by banks is to outsiders
(b) buying by banks (e) None of these
(c) merger of banks 25. Target market for Home Loans is
(d) selling bank's products and services (a) builders (b) housing societies
(e) selling products in banks (c) agriculturists (d) All of these
18. Marketing is not required in ________. (e) None of these
26. The USP of a Credit Card is

ww
(a) Selling Credit/ Debit Cards
(b) Net Banking
(d) Retail Loans
(c) Corporate Loans
(e) All of these
(a) cashless operations
(b) only for HNIs
19.
w.E
Marketing of Internet Banking means
(a) meeting of Banks on the net
(b) net practice
(c) only for men
(d) only for employed persons
(e) transactions through cheque book

asy
(c) marketing usage of Banking transactions through
internet
27. EMI can be a marketing tool if
(a) EMI is increasing
(d) transactions with foreign banks
(e) All of the above En (b) It is very high
(c) It is very low
20. Marketing in banks is
(a) a one-day function (b) a one-man function gin (d) EMI has no impact on marketing
(e) EMI is a flat rate
(c) a one-off affair
(e) None of these
(d) All of these 28.

eer
Banks sell insurance for
(a) increasing deposits
(b) increasing loans
21. Target group for marketing Internet Banking is _______.
(a) all customers
(b) all literate customers
(c) increasing clients
(d) earning more profits ing
(c) all computer literate customers
(d) only borrowers 29.
(e) taking over insurance companies
Savings Accounts can be opened by _______.ne
22.
(e) All of these
Digital Banking is available through _______.
(a) Mobile phones (b) Internet
(a) All individuals fulfilling KYC norms
(b) All tax payers only
(c) All individuals above the age of 18
t
(c) Telephones (d) All of these (d) All businessmen only
(e) None of these (e) All students below the age of 18
23. Bancassurance can be sold to _____. 30. A short term loan is repayable within _______
(a) Banks (b) Insurance companies (a) 20 years
(c) Insurance agents (d) Bank customers (b) 3 years
(e) All of the above (c) As per the borrowers’ wish
(d) As per the guarantor’s wish
(e) There is no need to repay short term loans

16. a b c d e 17. a b c d e 18. a b c d e 19. a b c d e 20. a b c d e


RESPONSE 21. a b c d e 22. a b c d e 23. a b c d e 24. a b c d e 25. a b c d e
GRID 26. a b c d e 27. a b c d e 28. a b c d e 29. a b c d e 30. a b c d e

Downloaded From : www.EasyEngineering.net


Downloaded From : www.EasyEngineering.net

Section Test :
Computer Knowledge/
Marketing Aptitude
80
Max. Marks : 40 No. of Qs. 40 Time : 25 min. Date : ........./......../. ..............
1. Devices that enter information and let you communicate with the (a) virtual private network (b) LAN
computer are called _______. (c) intranet (d) extranet
(a) Software (b) Output devices (e) internet
(c) Hardware (d) Input devices 10. Tangible, physical computer equipment that can be seen and
(e) Input/Output devices touched is called _______
2. What is the function of the Central Processing Unit of a Computer? (a) hardware (b) software
(a) Creates invoices (c) storage (d) input/output

ww
(b) Performs calculations and processing
(c) Deletes Data
(d) Corrupts the data
11.
(e) None of these
Which of the following is the second largest measurement of RAM?
(a) Terabyte (b) Megabyte

w.E
(e) None of these (c) Byte (d) Gigabyte
3. All the characters that a device can use is called its ? (e) Mega Hertz
(a) Skill Set (b) Character Alphabet 12. What resides on the motherboard and connects the CPU to other
(c) Character Codes (d) Keyboard Characters components on the motherboard ?

4.
(e) Character Set

asy
If your computer keeps rebooting itself, then it is likely that
_______
(a) Input Unit
(c) ALU
(e) None of these
(b) System Bus
(d) Primary Memory

(a) It has a virus


(b) It does not have enough memory
(c) There is no printer En 13. Supercomputers _______
(a) are smaller in size and processing capability than mainframe
computers

5.
(d) There has been a power surge
(e) It needs a CD-ROM
What utility to you use to transfer files and exchange messages ?
gin (b) are common in majority of households
(c) contain thousands of microprocessors
(d) are rarely used by researchers due to their lack of computing
(a) Web browsers
(c) Email
(b) WWW
(d) Hypertext
14. eer
capacity
(e) are of the same size as laptops
The basic computer processing cycle consists of _______
6.
(e) search engines
Which unit controls the movement of signals between CPU and I/
O?
(b) systems and application
ing
(a) input, processing and output

(c) data, information and applications


(a) ALU
(c) Memory Unit
(e) None of these
(b) Control Unit
(d) Secondary Storage
15.
(d) hardware, software and storage
(e) None of these
The system unit .ne
7. The three main parts of the processor are _______
(a) ALU, Control Unit and Registers
(b) ALU, Control Unit and RAM
(c) Cache, Control Unit and Registers
(d) Control Unit, Registers and RAM
(a) coordinates input and output devices
(b) is the container that houses electronic components
(c) is a combination of hardware and software
(d) controls and manipulates data
t
(e) does the arithmetic operations
(e) RAM, ROM and CD-ROM 16. System software
8. Which of the following does not relate to Input Unit ? (a) allows the user to diagnose and troubleshoot the device
(a) If accepts data from the outside world. (b) is a programming language
(b) It converts data into binary code that is understandable by (c) is part of a productivity suite
the computer (d) is an optional form of software
(c) It converts binary data into the human readable form that is (e) helps the computer manage internal resources
understandable to the users. 17. Microsoft's Messenger allows users to
(d) It sends data in binary form to the computer for further (a) bypass a browser to surf the Web
processing (b) create a blog
(e) None of these (c) communicate via direct live communication
9. Which of the following terms is just the collection of networks that (d) identify and eliminate spam
can be joined together ? (e) make graphic presentations

1. a b c d e 2. a b c d e 3. a b c d e 4. a b c d e 5. a b c d e
RESPONSE 6. a b c d e 7. a b c d e 8. a b c d e 9. a b c d e 10. a b c d e
GRID 11. a b c d e 12. a b c d e 13. a b c d e 14. a b c d e 15. a b c d e
16. a b c d e 17. a b c d e

Downloaded From : www.EasyEngineering.net


Downloaded From : www.EasyEngineering.net

170 SPEED TEST 80


18. What is the short-cut key to highlight the entire column? (d) Place, People, Product, Policy
(a) Ctrl + Page up (b) Ctrl + Page down (e) Promotion, Product, Price, People
(c) Ctrl + Enter (d) Ctrl + Space bar 31. For an economic organization like bank, MIS means
(e) Ctrl + C (a) Middle Income Scheme
19. To save the current document or to open a previously saved (b) Management Information System
document, _________ is used. (c) Management of Information & Science
(a) file menu (b) tools menu (d) Marketing Information System
(c) view menu (d) edit (e) Only (b) and (c)
(e) review menu 32. Market segmentation is done by firms for all of the following
20. In a computer system, _________ device is functionally opposite reasons, except
of a keyboard. (a) to increase market share.
(a) joystick (b) mouse (b) to assist new product development.
(c) trackball (d) printer (c) so that they can develop mu lti-purpose advertising
(e) scanner campaigns.
21. Marketing in banks is defined as (d) to extend products into new markets.
(a) Negotiable Instruments Act (e) None of these
(b) Banking Regulation Act 33. Which among the following is not a tool for marketing planning
(c) Reserve Bank of India Act appraisal?
(d) Companies Act (a) External appraisal (b) Internal appraisal
(e) None of these (c) Gap analysis (d) SWOT analysis

ww
22. In marketing terms, attitude can best be defined as a (e) PCOT analysis
(a) rude behaviour of salesperson 34. Cross-selling covers
(b) rude behaviour of consumer (a) identifying customer needs
(c) mental state of consumer (b) matching the products to customer needs

23.
(e) None of these
w.E
(d) ego of the marketing executive

According to product life cycle theory, the profit is maximum in


(a) developed Stage (b) early stage 35.
(c) convincing the customers of product benefits
(d) responding to questions and objections of customers
(e) All of these
Market information means

24.
(c) matured Stage
(e) None of these
In banks, loans & advances are considered as asy
(d) declined Stage (a) knowledge of shops and bazaars
(b) knowledge of shopping malls
(c) knowledge of customer profile and product mix
(d) knowledge of various languages
(a) assets
(c) resources
(e) None of these
(b) liabilities
(d) cause of expenditure
En 36.
(e) None of these
Market research is needed for
25. A firm is productively efficient when
(a) it is producing its product or service at the lowest unit cost
that it can. gin (a) deciding the market area
(b) deciding the right product to be sold
(c) making proper marketing decisions
(d) deciding right time to sell
(b) it is selling at the lowest price possible.
(c) it has the highest labour productivity that it can.
(d) it is making what its customers want. 37.
eer
(e) All of these
A product life cycle
(a) shows how a products sales or profits, depending on the
26.
(e) None of these
Vegetable market in India is nearly an example of
(a) perfect Competition (b) monopoly ing
units used, may rise and fall over its life.
(b) tells you how long a product will sell for and make a profit.
(c) is divided into three stages.

.ne
(c) oligopoly (d) imperfect competition
(d) shows how profitable a product will be.
(e) high monopsony
(e) None of these
27. Marketing in banks is
38. The long term objective of marketing is

t
(a) a one-day function (b) a one-man function
(a) customer satisfaction (b) profit maximisation
(c) a one-off affair (d) All of these
(c) cost cutting
(e) None of these
(d) profit maximisation with customer
28. Modern methods of marketing include
(e) None of these
(a) publicity on the net (b) advertisement on the net
39. Proper pricing is needed for
(c) soliciting business through e-mails (a) extra charges for extra services
(d) telemarketing (e) All of these (b) levy of VAT
29. In a selling process in today’s world (c) good customer service
(a) only standard products are sold (d) putting burden on the customer
(b) no customization required (e) service with extra facilities
(c) the seller need not have product knowledge 40. All of the following are examples of unfair competition, except
(d) the seller should aim at customer satisfaction (a) two firms agreeing to fix their prices.
(e) only quantum of sales matters (b) three companies agreeing to share a market between them.
30. In Marketing Mix 4P’s imply (c) a monopolist charging excess prices for its product.
(a) Product, Price, Place, Promotion (d) exploiting a patent that the firm has on a product it has
(b) Product, Price, Policy, Place developed itself.
(c) Product, Place, Promotion, Policy (e) None of these

18. a b c d e 19. a b c d e 20. a b c d e 21. a b c d e 22. a b c d e

RESPONSE 23. a b c d e 24. a b c d e 25. a b c d e 26. a b c d e 27. a b c d e


28. a b c d e 29. a b c d e 30. a b c d e 31. a b c d e 32. a b c d e
GRID 33. a b c d e 34. a b c d e 35. a b c d e 36. a b c d e 37. a b c d e
38. a b c d e 39. a b c d e 40. a b c d e

Downloaded From : www.EasyEngineering.net


Downloaded From : www.EasyEngineering.net

History of Banking
and its Development 81
Max. Marks : 30 No. of Qs. 30 Time : 20 min. Date : ........./......../. ..............
1. Reserve Bank of India was established on 9. Lead Bank system was started on the recommendations of
(a) April 12, 1932 (b) April 1, 1935 (a) Raja Challaia Committee
(c) May 2, 1943 (d) November 13, 1941 (b) Kelkar Committee
(e) None of these (c) Nariman Committee
2. Reserve Bank of India was nationalised on (d) Malhotra Committee
(a) 21 May, 1948 (b) 13 July, 1951 (e) None of these

3.
ww
(c) 1 January, 1949
(e) None of these
(d) 12 October, 1951

Six Commercial banks were nationalised by the Government


10. Lead Bank system was started in
(a) 1967
(c) 1969
(b) 1968
(d) 1970
on
(a) April 15, 1980
(c) July 21, 1982
w.E (b) May 10, 1981
(d) May 13, 1984
11.
(e) 1972
Reserve Bank of India (RBI) follows the Minimum Reserves
System in issuing currency since

4.
(e) None of these
asy
The 14 major banks were nationalised by the Government
(a) 1950
(c) 1954
(e) 1951
(b) 1952
(d) 1956

on
(a) June 19, 1967 (b) July 19, 1969
En 12. Reserve Bank of India started the system of ombudsman to
resolve grievances of customers in

5.
(c) May 21, 1962
(e) None of these
(d) May 12, 1963

Which of the following is the first commercial bank? gin (a) 1994
(c) 1996
(e) 1998
(b) 1995
(d) 1997

(a) State Bank of India


(b) Oudh Commercial Bank a
13.
eer
The credit control methods adopted by the Reserve Bank are
(a) Quantitative controls
(c) Union Bank of India
(d) Indian Bank
(e) None of these
(b) Qualitative controls
(c) Fixed controls
(d) Both (a) and (b) ing
6. State Bank of India was established on
(a) May 2, 1951 (b) June 21, 1952 14.
(e) None of these
.ne
Which of the following Committees were appointed by the
Government for r estr uctur ing the Regional Rural

7.
(c) July 1, 1955
(e) None of these
(d) August 12, 1956

NABARD (National Bank for Agriculture and Rural


Development) was established in
Development Banks?
(a) Bhandar Committee (b) K.Basu Committee
(c) Raj Committee
(e) None of these
(d) Both (a) and ((b)
t
(a) 1979 (b) 1980
15. The concept of 'Universal Banking' was implemented in India
(c) 1981 (d) 1982 on the recommendations of:
(e) 1985 (a) Abid Hussain Committee
8. Regional Rural Banks were established on (b) R H Khan Committee
(a) July 3, 1970 (b) April 14, 1971 (c) S Padmanabhan Committee
(c) October 2, 1975 (d) November 23, 1978 (d) Y H Malegam Committee
(e) None of these (e) None of these

1. a b c d e 2. a b c d e 3. a b c d e 4. a b c d e 5. a b c d e
RESPONSE 6. a b c d e 7. a b c d e 8. a b c d e 9. a b c d e 10. a b c d e
GRID 11. a b c d e 12. a b c d e 13. a b c d e 14. a b c d e 15. a b c d e

Downloaded From : www.EasyEngineering.net


Downloaded From : www.EasyEngineering.net

172 SPEED TEST 81


16. Oudh Commercial Bank was established in Which of the following are correct?
(a) 1878 (b) 1879 (a) 1 and 3 (b) only 2
(c) 1880 (d) 1881 (c) 2 and 3 (d) only 1
(e) None of these (e) All the above are correct.
17. Which of the following is the first Private bank established 24. Security printing press was established in 1982 at?
based on the recommendations of the Narasimhan (a) Kolkata (b) New Delhi
Committee? (c) Bombay (d) Hyderabad
(a) UTI Bank Ltd. (b) Union Bank (e) Nasik
(c) Bank of Baroda (d) Dena Bank 25. Six private sector banks were nationalised on April 15, 1980,
(e) None of these whose reserves were more than?
18. In 1993, a Nationalized bank was merged with Punjab National
(a) 100 Crores (b) 200 crores
Bank (PNB), what is the name of that Bank?
(c) 300 crores (d) 400 crores
(a) Bank of Baroda (b) Global Trust Bank
(e) 500 crores
(c) New Bank of India (d) Bank of India
26. Scheduled banks are those?
(e) None of these
(a) Includued in the 2nd schedule of the Banking
19.
ww
The biggest commercial bank in India is
(a) RBI
(c) IDBI
(b) SBI
(d) Exim Bank of India
Regulation Act-1949
(b) Includued in the 2nd schedule of the Companies Act-

20.
(e) None of these
ICICI is a w.E
(a) Financial Institution
1956
(c) Includued in the 2nd schedule of the Reserve Bank of
India Act -1934
(b) Rural Development Bank
(c) Cooperative Bank asy (d) Bank Nationalization Act -1969
(e) None of these
(d) Space Research Institute
(e) None of these
En 27. When RBI has decided to circulate 'Plastic Currency Notes'
in the market ?
21. Banking amendment bill reduced SBI holding in its seven
subsidiary banks from
(a) 90% to 75% gin (a) July 1, 1999
(c) July 1, 2011
(e) None of these
(b) July 1, 2010
(d) Sept. 1, 2011

(b) 85% to 50%


(c) 75% or more to 51%
28.
eer
When was addopted, New strategy for Rural Lending :
Service Area Approach ?
(d) 65% or more to 49%
(e) None of these
(a) April 1, 1989
(c) April 1, 2010 ing
(b) March 1, 2007
(d) April 1, 2011
22. When was paper currency first started in India?
(a) 1810
(c) 1635
(b) 1715
(d) 1542
29.
(e) None of these

.ne
India Brand Equity Fund' was established in the year:

23.
(e) 1902
Consider the following statements.
(1) Scheduled Banks are those Banks which are included
30.
(a) 1992
(d) 1996
(b) 1998
(e) 1997
(c) 1995

t
Which was the first Indian Bank to introduce credit card?
(a) State Bank of India
in the Second Scheduled of the Reserve Bank Act, 1934.
(b) Central Bank of India
(2) There are 10 Non-Scheduled commercial Banks
(c) Union Bank of India
operating in the country.
(d) ICICI
(3) Co-operative banks are organized &managed on the
(e) None of these
principle of co-operation, self-help and mutual help.

RESPONSE 16. a b c d e 17. a b c d e 18. a b c d e 19. a b c d e 20. a b c d e


21. a b c d e 22. a b c d e 23. a b c d e 24. a b c d e 25. a b c d e
GRID 26. a b c d e 27. a b c d e 28. a b c d e 29. a b c d e 30. a b c d e

Downloaded From : www.EasyEngineering.net


Downloaded From : www.EasyEngineering.net

RBI & its


Monetary Policies 82
Max. Marks : 25 No. of Qs. 25 Time : 20 min. Date : ........./......../. ..............
1. The accounting year of RBI occurs between the months of ? (a) ` 85 crore (b) ` 115 crore
(a) April – March (b) March – February (c) ` 200 crore (d) None of above
(c) July – June (d) August – July (e) None of these
(e) None of these 8. In India which agency is entrusted with the collection of
2. In which article RBI permitted to the co-operative Banks for data of capital formation ?

ww
special Account Supervision?
(a) Art – 30
(d) Art – 32
(b) Art – 31
(e) None of these
(c) Art – 33
(a) RBI and CSO (b) RBI and SBI
(c) RBI and Other Bank (d) CSO and NSSO

3.
w.E
Open market operations of RBI refers to?
(a) buying and selling of shares
9.
(e) None of these
The Bank rate is the rate at which ?
(a) a bank lends to the public
(b) auctioning of foreign exchange
(c) trading in securities
(d) transactions in gold.
asy (b) the RBI lends to the public
(c) RBI gives credit to the Commercial Banks

(e) None of these


En (d) the Government of India lends to other countries.
(e) None of these
4. Monetary policy in India is formulated and implemented by?
(a) Government of India
gin
10. An increase in CRR by the Reserve Bank of India result in ?
(a) decrease in debt of the government
(b) Reserve Bank of India
(c) Indian Banks Association
eer
(b) reduction in liquidity in the economy
(c) attracting more FDI in the country
(d) FICCI
(e) None of these
ing
(d) more flow of credit to desired sectors
(e) None of these
5. Reserve bank of India follows which system for the issue of
currency?
11.

.ne
Which of the following provides the largest credit to agri-
culture and allied sectors ?
(a) Minimum Reserve System
(b) Proportionate Reserve System
(c) Both of the above
(d) None of the above
(a) Co-operative Banks with RBI
(b) Regional Rural Banks
(c) Commercial Banks
(d) Co-operative and Regional Rural Banks
t
(e) None of these (e) None of these
6. Which of the following controls credit creation by the com- 12. Who published the financial report on currency and finance ?
mercial Banks in India? (a) RBI (b) CSO (c) WTO
(a) Ministry of Finance (b) Reserve Bank of India (d) NSSO (e) None of these
(c) Government of India (d) State Bank of India 13. Who is the custodian of India's foreign exchange Funds?
(a) RBI (b) SBI
(e) None of these
7. Note issuing department of Reserve Bank of India should (c) ICICI (d) Central Bank
always passes the minimum gold stock worth ? (e) None of these

1. a b c d e 2. a b c d e 3. a b c d e 4. a b c d e 5. a b c d e
RESPONSE 6. a b c d e 7. a b c d e 8. a b c d e 9. a b c d e 10. a b c d e
GRID 11. a b c d e 12. a b c d e 13. a b c d e

Downloaded From : www.EasyEngineering.net


Downloaded From : www.EasyEngineering.net

174 SPEED TEST 82


14. Which of the following authority sanctions foreign exchange (c) Banks holding debt ridden Country Electricity Boards
for the import of goods ? bonds
(a) Any Nationalised Bank (d) Both a and b
(e) Neither (a) and (b)
(b) Exchange Bank
20. Which of the following are reasons for adverse impact on
(c) Reserve Bank of India card use in India?
(d) Ministry of Finance (1) Less ATMs
(e) None of these (2) Less number of cards
(3) Less Point of Sales terminals
15. How many posts of Deputy Governor in Reserve Bank of
(a) 1, 2 (b) 1, 3
India ?
(c) 2 (d) All of the above
(a) 1 (b) 2 (c) 3 (e) None of these
(d) 4 (e) None of these 21. RBI will purchase back indexed bonds maturing in which
16. Which among the following institutions regulates the exter- year?
nal commercial borrowings ? (a) 2022 (b) 2023

ww
(a) SEBI
(b) Ministry of Finance 22.
(c) 2024
(e) None of these
(d) 2025

Which of the following statements is/are correct about

w.E
(c) Ministry of Commerce
(d) Reserve Bank of India
Deputy Governor of RBI?
1. The tenure of Deputy Governor is five years or till the
age of 62, whichever is earlier.

17.
(e) None of these
asy
Which of the following statements is not correct ?
2. RBI has provision for three deputy governors.
3. RBI has provision for four deputy governors.
(a) RBI is the Central Bank of the country
(b) RBI is the Banker of the Central and the state Govern-
En (a) Only 1
(c) Only 3
(b) Only 2
(d) Both 1 and 2
ments
(c) RBI is the custodian of the country’s Foreign Exchange gin
23.
(e) None of these
Urijit Patel, Deputy Governor of RBI has been reappointed
for 3 years. How many deputy governors of RBI are there?
Reserve
(d) RBI was established in 1949.
(a) 3
(c) 5 eer (b) 4
(d) 6

18.
(e) None of these
RBI has tweaked the rules for lending rates and stipulated 24.
(e) None of these
ing
Since when has the Reserve Bank of India been successfully
that fixed rate loans of up to how many years will be offered
by lenders based on marginal cost of funding?
country?
(a) 1939 (b) 1951 .ne
operating the instrument of selective credit control in this

19.
(a) 2
(c) 4
(e) None of these
(b) 3
(d) 5

RBI has relaxed norms for which category of banks?


25.
(c) 1956
(e) None of these
(d) 1961
t
India is a member of the International Monetary Fund since
_____.
(a) Banks part of the cleaning up exercises of power utilities (a) 1934 (b) 1935
(b) Banks holding debt ridden State Electricity Boards (c) 1947 (d) 1949
bonds (e) None of these

RESPONSE 14. a b c d e 15. a b c d e 16. a b c d e 17. a b c d e 18. a b c d e


19. a b c d e 20. a b c d e 21. a b c d e 22. a b c d e 23. a b c d e
GRID 24. a b c d e 25. a b c d e

Downloaded From : www.EasyEngineering.net


Downloaded From : www.EasyEngineering.net

Banking Product
and Services
83
Max. Marks : 21 No. of Qs. 21 Time : 15 min. Date : ........./......../. ..............

1. Which of the following is not the part of the Scheduled 6. Funded Services under corporate banking does not include?
Banking structure in India? (a) Working Capital Finance
(a) Money Lenders (b) Bill Discounting
(b) Public Sector Banks (c) Export Credit
(c) Private Sector Banks (d) Letters of Credit

2.
ww
(d) Regional Rural Banks
(e) State Cooperative Banks
Small Savings Scheme like national savings certificates,
7.
(e) None of these
''Swabhiman'' Scheme is related-

w.E
Public Provident Fund, Monthly Income Schemes are popular
among the salaried people. Which financial institutions
manage these schemes?
(a) Rich Customers of the Bank
(b) RRBs
(c) To provide basic banking services to bankless villages

(c) Post Offices asy


(a) Public Sector Banks (b) Commercial Banks
(d) Co-operative Banks
(d) Both (b) and (c)
(e) None of these

3.
(e) None of these
Which of the following is/are the right(s) of customer En 8. A centralized database with online connectivity to branches,
internet as well as ATM-network which has been adopted
towards his banker?
(a) To receive a statement of his account from a banker gin by almost all major banks of our country is known as?
(a) Investment Banking
(b) To sue the bank for any loss and damages
(c) To sue the banker for not maintaining the secrecy of
eer
(b) Core Banking
(c) Mobile Banking
his account
(d) All of the above
(e) None of these
(d) National Banking
(e) Specialized Banking
ing
4. Which of the following factors is not required to be
considered to analyze the repayment capacity of a borrower?
9.

.ne
The Reverse Mortgage scheme is launched to give benefit
to which of the following groups of society?

(a) Working capital management


(b) Personal educational qualifications
(c) Financial leverage
(a) Persons below 60 yrs
(b) Senior Citizens
(c) Unemployed youth
(d) Orphans
t
(d) Interest rate risk management (e) All of the above
(e) None of these
10. Which of the following scheme is not meant for investment
5. Which of the following is a facilitating service of core loan
purposes?
products of retail banking services?
(a) National saving certificate
(a) Current or savings accounts
(b) Infrastructure bonds
(b) Legal services for documentation
(c) Mutual funds
(c) Delivery of loan at promised time period
(d) Letter of credit
(d) Flexibility in prepayment of loan
(e) None of these
(e) None of these

RESPONSE 1. a b c d e 2. a b c d e 3. a b c d e 4. a b c d e 5. a b c d e

GRID 6. a b c d e 7. a b c d e 8. a b c d e 9. a b c d e 10. a b c d e

Downloaded From : www.EasyEngineering.net


Downloaded From : www.EasyEngineering.net

176 SPEED TEST 83


11. Systematic investment Plans relates to: 16. If a bank ties up with a retail vendor and then both of them
(a) Mutual Funds sponsor a credit card, then such card would be known as __:
(b) Life Insurance Companies (a) Retail Credit Card (b) Vendor Card
(c) Commercial Banks (c) Co-branded Card (d) Cash back Card
(d) Post office savings schemes (e) None of these
(e) None of these 17. In terms of Banking Terminology, Affinity Card refers to __:
12. Which of the following is an example of cash less purchase? (a) The Credit Cards that are linked to special organizations
(a) Debit card (b) Credit card (b) The Credit Cards with zero interest rate on repayments
(c) ATM withdrawal (d) All of the above (c) The Credit Cards exclusively for the Bank's employees
(e) None of these (d) All of above
13. Which one of the following is not an electronic banking (e) None of these
delivery channel? 18. From which country, the concept of Credit Card originated?
(a) Mobile Vans (a) United Kingdom (b) United States
(d) Mobile Phone Banking (c) France (d) Australia

ww
(c) Internet Banking
(d) Tele Banking
(e) ATM
19.
(e) None of these
Which is the first credit card facility to be recognised

14.
w.E
Now-a-days Banks are selling third party products. Example
of third party product is:
worldwide?
(a) Visa card
(c) MasterCard
(b) Maestro card
(d) Diner Card
(a) Mutual funds
(b) Term deposits
(c) Credit cards
asy 20.
(e) None of these
In terms of Credit Cards, what is a Hot Card?
(a) Newly issued Card (b) Invalid Card
(d) All of these
(e) None of these En (c) Stolen Card (d) Unpaid Card Hide

15. What are White Label ATMs


(a) ATMs set up and run by non-banking entities gin
21.
(e) None of these
What does the cirrus logo on ATM / debit cards signifies__?
(a) have cash access facility anywhere in India only
(b) ATMs set up and run by banking entities
(c) ATMs in rural areas eer
(b) have cash access facility outside the India only
(c) have cash access facility in or outside the India
(d) ATMs in Defence areas
(e) ATM set up in Uttaranchal ing
(d) have cash access facility in and outside the India
(e) None of these

.ne
t

RESPONSE 11. a b c d e 12. a b c d e 13. a b c d e 14. a b c d e 15. a b c d e


16. a b c d e 17. a b c d e 18. a b c d e 19. a b c d e 20. a b c d e
GRID 21. a b c d e

Downloaded From : www.EasyEngineering.net


Downloaded From : www.EasyEngineering.net

Banking Term/
Terminology
84
Max. Marks : 30 No. of Qs. 30 Time : 20 min. Date : ........./......../. ..............
1. What is the full form of ‘FINO’, a term we see frequently in (a) Company Social Representation
financial newspapers? (b) Company Service Responsibility
(a) Financial Investment Network and Operations (c) Corporate Social Responsibility
(b) Farmers’ Investment in National Organisation (d) Corporate Security Responsibility
(c) Farmers Inclusion News and Operations (e) None of these
(d) Financial Inclusion Network and Operations 8. In a bid to standardise and enhance the security features in

2.
ww
(e) None of these
What does the letter 'L' denote in the term ‘LAF’ as referred
every now and then in relation to monetary policy of the
cheque forms, it has now been made mandatory for banks
to issue new format of cheques called
(a) CTS-2010 (b) CTS-2011
RBI?
(a) Liquidity
(c) Leveraged
(e) Linear
w.E (b) Liability
(d) Longitudinal 9.
(c) CTS-2012
(e) None of these
(d) CTS-2013

The “Four Eyes” principle (mentioned by the Reserve Bank

3.
(a) Investment asy
Which of the following terms is NOT a financial term?
of India) refers to:
(a) Lenders (b) Borrowers
(c) Wealth Managers (d) Micro-Fananciers
(b) El Nino effect
(c) Core banking Solution
(d) RTGS En 10.
(e) None of these
NRE deposit is

4.
(e) All are financial terms
The term 'Smart Money" refers to __________ . gin (a) Non Resident External deposit
(b) Non Resident Extra deposit
(c) Non Resident Exchange deposit
(a) Foreign Currency (b) Internet Banking
(c) US Dollars
(e) Credit Cards
(d) Travelers' cheques
11. eer
(d) Non Refundable External deposit
(e) Non Resident Extended deposit
Which of the following is NOT a banking-related term?
5. We often come across the term SWIFT in financial
newspapers. What is the expanded form of this term?
(a) Society for Worldwide Interbank Financial
(a) SME Finance
(c) Drawing power ing
(b) Overdraft
(d) Sanctioning Authority
Telecommunication
(b) Secure Worldwide Interbank Financial Telecommunica-
12.
(e) Equinox
What does the acronym LAF stand for?
(a) Liquidity Adjustment Fund .ne
tion.
(c) Society for Worldwide Intr a-bank Finan cial
Transaction.
(d) Security for Worldwide Interbank Financial
(b) Liquidity Adjustment Facility
(c) Liquidity Adjustment Finance
(d) Liquidity Adjustment Factor
(e) None of these
t
Transaction
13. Which of the following terms is used in Banking Field?
(e) None of these
6. Many a time we read in financial newspapers about the (a) Interest rate swap (b) Input devices
performance of the “core sectors” in the economy. Which (c) Sedimentary (d) Zero hour
of the following is NOT included in the same? (e) Privilege motion
(a) Coal (b) Automobiles 14. What is “wholesale banking”?
(c) Steel (d) Cement (a) It is a bank-to-bank or B2B dealing.
(e) Oil & Petroleum (b) It is a bank-to-customer dealing.
7. The Standing Committee on Finance headed by Yashwant (c) It is a bank-to-trustworthy customer dealing.
Sinha has recommended that CSR should be mandatory for (d) It is a bank-to-government dealing
all the companies. CSR stands for (e) None of these

1. a b c d e 2. a b c d e 3. a b c d e 4. a b c d e 5. a b c d e
RESPONSE 6. a b c d e 7. a b c d e 8. a b c d e 9. a b c d e 10. a b c d e
GRID 11. a b c d e 12. a b c d e 13. a b c d e 14. a b c d e

Downloaded From : www.EasyEngineering.net


Downloaded From : www.EasyEngineering.net

178 SPEED TEST 84


15. Trade between India and China is in a state of “Payment (d) Industrial Research and Demands Agency
imbalance”. What does this mean in real terms? (e) None of these
(1) China imports less from India but India imports more 22. Hard Currency is defined as currency :
from China. (a) which can hardly be used for international transactions
(2) China delays payments to exporters. (b) which is used in times of war
(3) India wants payments in US Dollars but wants to pay (c) which loses its value very fast
in Yuan. (d) traded in foreign exchange market for which demand
(a) Only 1 (b) Only 2 is persistently relative to the supply
(c) Only 3 (d) All 1, 2 and 3 (e) None of these
(e) None of these 23. The terms “bull” and “bear” are used in the :
16. Banks are promoting “Branch less Banking” which means? (a) Bihar Government’s Animal Husbandry Department
(1) Banks will not reduce number of branches. Number of (b) Income Tax Department
branches will be restricted and will concentrate on (c) CBI
specified core business. (d) Stock Exchange
(2) Banks will launch/operate multiple delivery channels (e) None of these
like ATMs, Mobile Banking/Internet Banking etc making 24. We read a term 'ECB' in the financial newspapers. What is
visit to a branch unnecessary. the full form of ECB?
(3) Banks will issue only debit or credit cards for daily (a) Essential Credit and Borrowing

ww
financial transactions. Cheques/Cash payment will not
be allowed.
(a) Only 1 (b) Only 2
(b) Essential Commercial Borrowing
(c) External Credit and Business
(d) External Commercial Borrowing

17.
(c) Only 1 and 2
(e) All 1, 2 and 3
w.E (d) Only 2 and 3

NBFCs are an important part of the Indian financial system.


What is the full form of this term?
25.
(e) None of these
Green Banking means
(a) financing of irrigation projects by banks
(a) New Banking Financial Companies
(b) Non-Banking Financial Companies
(c) Neo Banking Financial Confederation
asy (b) development of forestry by banks
(c) financing of environment friendly projects by banks
(d) development of Railway by banks.
(d) Non-Banking Fiscal Companies
(e) All of these En 26.
(e) None of these
DTAA stands for
(a) Direct Tariff Avoidance Agreement
18. BCSBI stands for
(a) Banking Codes and Standards Board of India
(b) Banking Credit and Standards Board of India gin (b) Double Taxation Avoidance Agreement
(c) Direct Taxation Avoidance Agreement
(c) Banking Codes and Service Board of Inida.
(d) Banking Credit and Service Board of India.
(e) None of these 27. eer
(d) Double Tariff Avoidance Agreement
(e) None of these
The rate at which the Reserve Bank of India lends to the
19. What does the term ‘bancassurance’ mean ?
(a) Assurance from the bank to its account holder ing
commercial banks in very short term against the backing of
the Government securities is known as?
(a) Bank rate (b) Repo rate
regarding safety of his money
(b) A special product designed by the bank
(c) Selling of insurance policies by banks
(c) Reverse Repo
(e) None of these
.ne
(d) Discount rate

20.
(d) Understanding between banks and insurance
companies
(e) None of these
SEZ stands for:
(a) Southern Economic Zone
28.

(a) Back to back LC (b) Red clause LC


(c) Back to front LC (d) Revolving LC
(e) None of these
t
A letter of credit (L(C) wherein the credit available to the
customer gets reinstated after the bill is paid is known as?

(b) South European zone 29. Which of the following is not an imperfect note?
(c) Special Economic Zone (a) Washed note (b) Bleached note
(d) Special Eastern Zone (c) Mutilated note (d) Oiled note
(e) None of these (e) None of these
21. The abbreviation IRDA stands for: 30. Which of the following is not shown as an asset in the
(a) Industrial Research and Development Authority of balance sheet of a BanK?
India (a) Investment (b) Advances
(b) Insurance Research and Development Authority of (c) Cash Balances with other banks
India (d) Borrowings
(c) Insurance Regulation Development Authority of India (e) None of these

15. a b c d e 16. a b c d e 17. a b c d e 18. a b c d e 19. a b c d e


RESPONSE 20. a b c d e 21. a b c d e 22. a b c d e 23. a b c d e 24. a b c d e
GRID 25. a b c d e 26. a b c d e 27. a b c d e 28. a b c d e 29. a b c d e
30. a b c d e

Downloaded From : www.EasyEngineering.net


Downloaded From : www.EasyEngineering.net

Micro Finance and


Economics 85
Max. Marks : 30 No. of Qs. 30 Time : 20 min. Date : ........./......../. ..............
1. Which sector of Indian economy contributes longest to Gross (c) Finance Ministry
National Product ? (d) Reserve Bank of India
(a) Primary sector (b) Secondary sector (e) None of these
(c) Tertiary sector (d) Public sector 9. In which of the following financial years the devaluation of
(e) None of these rupee in India took place twice ?
2. The main source of National Income in India is- (a) 1966-67 (b) 1991-92

ww
(a) Service sector (b) Agriculture
(c) Industrial sector (d) Trade sector
(e) None of these 10.
(c) 1990-91
(e) None of these
VAT is imposed-
(d) 1989-90

3.
w.E
Which one of the following is not a tax levied by the
government of India ?
(a) Service tax (b) Education
(a) On first stage of production
(b) Directly on consumer
(c) On all stages between production and final sale

4.
(c) Custom duty
(e) None of these
(d) Toll tax

asy
The most appropriate measure of a country’s economic 11.
(d) On final stage of production
(e) None of these
Term Balance of payment is used in relation to which of the
growth is its-
(a) Gross Domestic Product (GDP)
En following ?
(a) Annual sale of a factory
(b) Net Domestic Product (NDP)
(c) Net National Product (NNP)
(d) Per Capita Product (PCP) gin (b) Tax collection
(c) Exports and imports
(d) None of the above

5.
(e) None of these
FEMA (Foreign Exchange Management (Act) was finally 12. eer
(e) None of these
The Indian economy can be described as
implemented in the year
(a) 1991
(c) 2002
(b) 1997
(d) 2007
(b) a developing economy ing
(a) a backward and stagnant economy

6.
(e) None of these
The most common measure of estimating inflation in India is-
(c) an underdeveloped economy
(d) a developed economy
(e) None of these .ne
(a) Price Index
(b) Wholesale Price Index
(c) Consumer Price Index
(d) Price Index of Industrial Goods
13. Loans to poor people by banks have many limitations
including lack of security and high operating cost. So to
help them which type of finance system developed ?
(a) Ponzi schemes
t
(e) None of these (b) Micro Finance System
7. The National Income of India is estimated by- (c) Money Laundering Schemes
(a) National Sample Survey Organization (d) Money tampering finance
(b) Ministry of Finance (e) None of these
(c) Reserve Bank of India 14. The beneficiaries of Micro finance business are _______.
(d) Central Statistical Organization (a) Land Less labour
(e) None of these (b) Marginal farmers
8. In India, which one among the following formulates the fiscal (c) Vendors in the small markets
policy?
(d) All the above
(a) Planning Commission
(e) None of these
(b) Finance Commission

1. a b c d e 2. a b c d e 3. a b c d e 4. a b c d e 5. a b c d e
RESPONSE 6. a b c d e 7. a b c d e 8. a b c d e 9. a b c d e 10. a b c d e
GRID 11. a b c d e 12. a b c d e 13. a b c d e 14. a b c d e

Downloaded From : www.EasyEngineering.net


Downloaded From : www.EasyEngineering.net

180 SPEED TEST 85


15. One of the delivery channel for Micro Finance is SHG model. 24. Which among the following is considered to be the most
SHG means ? liquid asset?
(a) Soar Help Group (b) Sake Help Group (a) Gold (b) Money
(c) Self Hope Group (d) Self Help Group (c) Land (d) Treasury bonds
(e) None of these (e) None of these
16. In the Not-For-Profit Micro Finance Institutes, which among 25. Who is author of the ancient book on economics,
the following are included ? Arthashastra?
(a) Societies (a) Kautilya (b) Chanakya
(b) Public Trusts (c) Sushrut (d) Bhattacharya
(c) Non-Profit Companies (e) None of these
(d) All of these
26. Currency notes and coins are called as:
(e) None of these
(a) Flat money (b) Legal tenders
17. Non-banking financial companies, producer companies and
LAB come under the category of For-Profit-MFIs. LAB (c) Fiat money (d) Both b and c
means ? (e) None of these
(a) Loan Area Banks (b) Legal Area Banks 27. What is the currency deposit ratio (cdr)?

18. ww
(c) Local Axis Banks (d) Local Area Banks
(e) None of these
On which among the following dates FERA was replaced by
(a) ratio of money held by the public in currency to that of
money held in bank deposits
(b) ratio of money held by public in bank deposits to that
of money held by public in currency
FEMA?
(a) June 1, 2000
(c) April 1, 2001 w.E (b) June 11, 2000
(d) April 1, 2002
(c) ratio of money held in demand drafts to that of money
held in treasury bonds
(d) ratio of money held in demand drafts to that of money

19.
(e) None of these

asy
What is the minimum net owned funds (NOF) mandatory
for a Infrastructure Finance Company (IFC) in India?
28.
held in mutual bonds
(e) None of these
What is the reserve deposit ratio (rdr)?
(a) ` 200 Crore
(c) ` 500 Crore
(b) ` 300 Crore
(d) ` 100 Crore
En (a) the proportion of money RBI lends to commercial banks
(b) the proportion of total deposits commercial banks keep
20.
(e) None of these
In India, Infrastructure Debt Fund can be established as a
Trust or a company. Which of the following regulated the gin as reserves
(c) the total proportion of money that commercial banks
Infrastructure Debt Fund set up as a trust?
(a) SEBI (b) RBI
eer
lend to the customers
(d) the total proportion of money that commercial banks
lend to the money RBI tends

21.
(c) IrDA
(e) None of these
(d) Ministry of Corporate Affairs

What is the cap on loan amount given out by a Non-Banking 29.


(e) None of these
ing
What is the Cash Reserve Ratio (CRR)?
Financial Company - Micro Finance Institution (NBFC-MFI)?
(a) ` 50000 (b) ` 60000 to the customers
.ne
(a) the fraction of the deposits that commercial banks lend

(b) the fraction of the deposits that RBI must keep with

22.
(c) ` 70000
(e) None of these
(d) ` 75000

Which of the following is are the parties in Factoring


Business?
commercial banks
t
(c) the fraction of the deposits that commercial banks must
keep with RBI
(d) the fraction of the deposits that private banks must
(a) buyer
(b) seller keep with RBI
(c) buyer and seller (e) None of these
(d) buyer, seller and financial institution 30. In monetary terminology, what is called the 'monetary base'
or 'high powered money'?
(e) None of these
23. What is the minimum tenure of deposits to be taken by (a) the total assets of RBI
NBFCs? (b) the total liability of RBI
(A) 6 months (b) 12 months (c) the total debt of the government
(c) 2 years (d) 3 years (d) the total foreign exchange of RBI
(e) None of these (e) None of these

15. a b c d e 16. a b c d e 17. a b c d e 18. a b c d e 19. a b c d e


RESPONSE 20. a b c d e 21. a b c d e 22. a b c d e 23. a b c d e 24. a b c d e
GRID 25. a b c d e 26. a b c d e 27. a b c d e
28. a b c d e 29. a b c d e 30. a b c d e

Downloaded From : www.EasyEngineering.net


Downloaded From : www.EasyEngineering.net

Foreign Trade 86
Max. Marks : 30 No. of Qs. 30 Time : 20 min. Date : ........./......../. ..............
1. ‘Eco mark’ is given to the Indian products that are ? 9. TRIPS and TRIMS are the term associated with ?
(a) pure and unadulterated (a) IMF (b) WTO
(b) rich in proteins
(c) IBRD (d) IDA
(c) Environment Friendly
(d) Economically viable (e) None of these
(e) None of these 10. SEZ act was passed by the parliament in the year ?
2.
ww
The earlier name of WTO was ?
(a) UNCTAD
(c) UNIDO
(b) GATT
(d) OECD
(a) 2004
(c) 2006
(b) 2005
(d) 2007

3.
(e) None of these
w.E
‘World Development Report’ is an annual publication of ?
(a) UNDP (b) IBRD
11.
(e) None of these
How many members recently in WTO ?
(a) 158 (b) 159

4.
(c) WTO
(e) None of these
(d) IMF
asy
India has the maximum volume of foreign trade with ?
(c) 160
(e) None of these
(d) 162

(a) USA
(c) Germany
(b) Japan
(d) UAE En 12. Which of the following is a part of capital account?
(a) Private capital (b) Banking capital

5.
(e) None of these
Participatory notes (PNs) are associated with which one of gin (c) Official capital
(e) None of these
(d) All the above

the followings ?
(a) Consolidated food
(b) Foreign Institutional Investors
13.
eer
The investment in productive assets and participation in
management as stake holders in business enterprises is
(c) UNDP
(d) Kyoto protocal
(a) FDI
(b) FII ing
6.
(e) None of these
What is the purpose of India Brand-Equity Fund ?
(a) To promote in bound tourism
(c) Balance of payment
(d) SDR
.ne
(b) To make ‘Made in India’ a label of quality
(c) To organise trade fairs
(d) To provide venture capitals to IT sector
(e) None of these
14.
(e) None of these
The portfolio investment by foreign institutional investors
is called
(a) FDI
t
7. A trade policy consists of : (b) FII
(a) Export-Import policy
(c) Balance of payment
(b) Licencing policy
(c) Dumping (d) SDR
(d) Double pricing (e) None of these
(e) None of these 15. Which of the following is international trade:
8. FERA in India has been replaced by ? (a) Trade between provinces
(a) FEPA (b) FEMA (b) Trade between regions
(c) FENA (d) FETA (c) Trade between countries
(d) (b) and (c) of above
(e) None of these
(e) None of these

1. a b c d e 2. a b c d e 3. a b c d e 4. a b c d e 5. a b c d e
RESPONSE 6. a b c d e 7. a b c d e 8. a b c d e 9. a b c d e 10. a b c d e
GRID 11. a b c d e 12. a b c d e 13. a b c d e 14. a b c d e 15. a b c d e

Downloaded From : www.EasyEngineering.net


Downloaded From : www.EasyEngineering.net

182 SPEED TEST 86


16. Which is NOT an advantage of international trade: 23. Special Economic Zones are :
(a) Export of surplus production (a) situated outside India, but subject to RBI control
(b) Import of defence material (b) treated as foreign territory and not Indian laws
(c) Dependence on foreign countries (c) governed by international and not Indian laws
(d) Availability of cheap raw materials (d) prohibited from buying from Domestic Tariff Area
(e) None of these
(e) None of these
24. The Imports and Exports (Control) Act came into enforcement
17. Trade between two countries can be useful if cost ratios of
from
goods are:
(a) 1947 (b) 1950.
(a) Equal (b) Different (c) 1951 (d) 1955
(c) Undetermined (d) Decreasing (e) None of these
(e) None of these 25. The IEC number is issued by the
18. Foreign trade creates among countries: (a) Central Government
(a) Conflicts (b) Cooperation (b) State Government

ww
(c) Hatred
(e) None of these
(d) Both (a) & (b) (c) Director General of Foreign trade
(d) Ministry of commerce
(e) None of these
19.

w.E
All are advantages of foreign trade EXCEPT:
(a) People get foreign exchange
(b) Nations compete
26. The IEC number is a number with
(a) 7 digits (b) 8 digits

(c) Cheaper goods


asy
(d) Optimum utilisation of country's resources
27.
(c) 9 digits
(e) None of these
(d) 10 digits

The Director General of Foreign trade is appointed by

20.
(e) None of these
Govt. policy about exports and imports is called: En (a) Central Government
(b) State Government
(a) Monetary policy
(b) Fiscal policy gin (c) Ministry of commerce
(d) Chief justice of the Supreme Court
(c) Commercial policy
(d) Finance policy 28. eer
(e) None of these
The foreign Trade (Regulation) Rules was passed in the

21.
(e) None of these
What would encourage trade between two countries:
year
(a) 1991 ing
(b) 1992
(a) Different tax system
(b) Frontier checks
(c) 1993
(e) None of these
(d) 1994
.ne
(c) National currencies
(d) Reduced tariffs
(e) None of these
29. The apex body of the Foreign Trade is
(a) The Central Government
(b) The State Government
(c) The Ministry of Commerce
t
22. Foreign trade: (d) All the above
(a) Increases employment opportunities (e) None of these
(b) Increases international mobility of labour 30. The tenure of the Foreign Trade policy is
(c) Increases competition (a) 3 years (b) 5 years
(d) All of the above (c) 1 year (d) 7 years
(e) None of these (e) None of these

16. a b c d e 17. a b c d e 18. a b c d e 19. a b c d e 20. a b c d e


RESPONSE 21. a b c d e 22. a b c d e 23. a b c d e 24. a b c d e 25. a b c d e
GRID 26. a b c d e 27. a b c d e 28. a b c d e 29. a b c d e 30. a b c d e

Downloaded From : www.EasyEngineering.net


Downloaded From : www.EasyEngineering.net

Socio-Eco-Political
Environment of India 87
Max. Marks : 25 No. of Qs. 25 Time : 20 min. Date : ........./......../. ..............
1. Swabhiman scheme launched in India is associated with 8. Which one of the following is the objective of National Food
(a) Rural women rights Security Mission ?
(b) Rural old people care (a) To increase production of rice
(c) Rural banking (b) To increase production of wheat
(d) Rural food security (c) To increase production of pulses

2.
ww
(e) None of these
In which one of the following years the unorganised workers
social security Act was passed ? 9.
(d) All the above
(e) None of these
Which one of the following durations is related to XII Five
(a) 2004
(c) 2008
(e) None of these
w.E (b) 2006
(d) 2010
Year Plan in India ?
(a) 2007-12
(c) 2012-17
(b) 2005-10
(d) 2010-15
3. Swadhar is the scheme for
(a) unique identification
(b) self employment for males
asy 10.
(e) None of these
Which of the following is not a measure of reducing
inequalities ?
(c) women in difficult circumstances
(d) common home of senior citizens. En (a) Minimum needs programme
(b) Liberalization of economy

4.
(e) None of these
The aim of Pradhan Mantri Gramodaya Yojana is gin (c) Taxation
(d) Land reforms
(e) None of these
(a) Meeting rural needs like primary education, health care,
drinking water, housing, rural roads.
11.
eer
Which one of the following scheme subsumed the Valmiki
Awas Yojana ?
(b) Alleviating poverty through microenterprises.
(c) Generating employment in rural areas.
(d) Strengthening Panchayati Raj system in rural areas. ing
(a) Integrated Housing and slum development Programme.
(b) Sampoorna Grameen Swarozgar Yojana
(c) Rajiv Awas Yojana

5.
(e) None of these
Twenty Point Economic Programme was first launched in
12.
(d) Integrated Rural development Programme
(e) None of these
.ne
Poverty level in India is established on the basis of-
the year
(a) 1969
(c) 1977
(e) None of these
(b) 1975
(d) 1980
(a) Per capita income in different states
(b) House hold average income
(c) House hold consumer expenditure
(d) Slum population in the country.
t
6. In India disguised unemployment is a prominent feature (e) None of these
mainly of 13. Nirmal Bharat Abhiyan Yojana is associated with-
(a) Primary sector (b) Secondary sector (a) Development of villages
(c) Tertiary sector (d) Social sector (b) Community toilets in slum areas
(e) None of these (c) Construction of house far low income groups
7. Golden Quadrangle project is associated with the (d) Development of roads
development of (e) None of these
14. Crop Insurance Scheme in India was started in
(a) Highways (b) Ports
(a) 1945 (b) 1980
(c) Power Grids (d) Tourism Network (c) 1985 (d) 1988
(e) None of these (e) None of these

1. a b c d e 2. a b c d e 3. a b c d e 4. a b c d e 5. a b c d e
RESPONSE 6. a b c d e 7. a b c d e 8. a b c d e 9. a b c d e 10. a b c d e
GRID 11. a b c d e 12. a b c d e 13. a b c d e 14. a b c d e

Downloaded From : www.EasyEngineering.net


Downloaded From : www.EasyEngineering.net

184 SPEED TEST 87


15. Which of the following scheme is not for rural development ? 21. India is implementing "Capacity Building for Industrial
(a) PMGSY (b) SGSY Pollution Management (CBIPM)" project with the support
(c) RGGVY (d) SJSRY of __?
(a) Asian Development Bank
(e) None of these
(b) Japan Central Bank
16. Who is the chairman of 21st law commission ? (c) World Bank
(a) Justice A.R. Lakshmanan (d) Reserve Bank of India
(b) Justice P.V. Reddy (e) None of these
(c) Justice D.K. Jain 22. Who is the chairman of the Panel set up to look into various
(d) Justice Balbir Singh Chauhan contentious issues relating to inter-linking of rivers?
(a) B N Navalawala (b) Hardip Singh Puri
(e) None of these
(c) Prodipto Ghosh (d) Gopalakrishnan
17. According to the Integrated child development services
(e) None of these
which age group of children are focused ?
23. Who has been appointed as new Chief Election
(a) upto 4 years (b) upto 5 years Commissioner of India?
(c) upto 6 years (d) upto 8 years

18. ww
(e) None of these
Which age group of women are eligible for Indira Gandhi
(a) Nasim Zaidi
(c) H Hari Shankar
(e) None of these
(b) Sayed Nazim
(d) V S Chikkamata

(a) 40-50
(c) 40-59
w.E
widow Pension Scheme ?
(b) 50-60
(d) 40-79
24. What is the name of the new scheme to empower and enable
minority youths, announcement for which was made in the
budget?

19.
(e) None of these
asy
Under which project India is developing guided missile
(a) New Udaan
(c) Nayi Umang
(e) None of these
(b) Nayi Soch
(d) Nayi Manzil

destroyers?
(a) Project 20B (b) Project 15B
En 25. Union Human Resource and Development (HRD) Minister
Smriti Irani introduced the logo winner of for the New

20.
(c) Project 75B
(e) None of these
(d) Project 40A

E-Samiksha is an online project monitoring system launched gin Education Policy (NEP) who emerged as the winner in the
competition was held through MyGov Platform. What is his
by ____?
(a) Indian Railways (b) NHAI
name?

eer
(a) Nawab Shaikh
(c) Nawaj Shaikh
(b) Ali Ahmed
(d) Mansood Pataudi
(c) RBI
(e) None of these
(d) FIPB
(e) None of these
ing
.ne
t

15. a b c d e 16. a b c d e 17. a b c d e 18. a b c d e 19. a b c d e


RESPONSE
20. a b c d e 21. a b c d e 22. a b c d e 23. a b c d e 24. a b c d e
GRID
25. a b c d e

Downloaded From : www.EasyEngineering.net


Downloaded From : www.EasyEngineering.net

Appointment/
Election/Resignation 88
Max. Marks : 30 No. of Qs. 30 Time : 20 min. Date : ........./......../. ..............
1. Who among the following has been appointed as permanent 7. Who among th e following has been appointed as
representative of India to UNESCO? Chairperson of National School of Drama?
(a) Ruchir Kamboj (a) Girish Carnad (b) Ratan Thiyam
(b) V S Oberoi (c) U R Anantha Murthy(d) Anupam Khair
(c) Vijay Keshav Ghokhale (e) None of these
(d) Sudeer Kulkarni 8. Who is appointed as the chairman of The Association of

2.
ww
(e) None of these
Who among the followings is the chairman of “National
Commission for Backward Classes”?
Mutual Funds in India (AMFI).
(a) Sundeep Sikka
(c) Pawan Kumar
(b) Sandesh Kirkire
(d) S. M. Patel

(b) Justice M N Rao


w.E
(a) Justice V Eswaraih

(c) Justice Narendra Babu


9.
(e) Leo Puri
Who took charge as the new commissioner of Delhi Police
on February 2016?

3.
(d) Justice Gopala Gowda
(e) None of these
asy
Who among the followings is the Chairperson of the National
(a) BK Gupta (b) Neeraj Kumar
(c) Bhim Sain Bassi (d) G.V. Prakash Kumar
(e) Alok Kumar Verma
Commission for Minorities?
(a) K N Daruwall
En 10. Who is recently appointed as the Chief Justice of Meghalaya
High Court?
(b) Tsering Namgyal Shanoo
(c) Wajahat Habibullah
(d) Ajaib Singh gin (a) M. Subhashini
(b) Kamal Ganzouri
(c) Prafulla Chandra Pant

4.
(e) Shri Naseem Ahmed
Who among the following is the current Chairperson of
eer
(d) P. Sathasivam
(e) T.Nand kumar Singh
“National Green Tribunal” of India?
(a) Justice Jyothimani
(b) Justice Kingaonkar
11.

ing
Who among the following is the CEO of PepsiCo Asia,
Middle East and Africa region?
(a) Abhinav Gupta (b) Sanjay Singh
(c) Justice Swatanter Kumar
(d) Justice U D Salvi
(c) Sanjeev Chadha (d) Nanda Kishore
(e) Indira Nooyi
.ne
5.
(e) None of these
Who among the following is the Chairman of “National
Ganga River Basin Authority”?
(a) President (b) Prime Minister
12. Who is appointed as the Vice Chairman of NITI Aayog?
(a) Jagdish Bhagwati (b) T. N. Srinivasan
(c) Arvind Panagariya (d) Montek Singh Ahluwalia
(e) None of these
t
(c) Vice-president (d) Home Minister 13. Who replaced Sujatha Singh as India's Foreign Secretary
(e) None of these (a) Nirupama Rao (b) Ranjan Mathai
6. Who among the followings has been appointed as Chief (c) Ajit Doval (d) S Jaishankar
Information Commissioner? (e) None of these
(a) Mrs Deepak Sandhu 14. _____ has been appointed as an Election Commissioner of
(b) Mrs Sushma Singh India ?
(c) Rajiv Mathur (a) Anil Swarup (b) C.R. Viswanath
(d) Vijay Sharma (c) Achal Kumar Jyoti (d) Ashok Kumar Gupta
(e) None of these (e) None of These

1. a b c d e 2. a b c d e 3. a b c d e 4. a b c d e 5. a b c d e
RESPONSE 6. a b c d e 7. a b c d e 8. a b c d e 9. a b c d e 10. a b c d e
GRID 11. a c d e 12. a c d e 13. a c d e 14. a c d e
b b b b

Downloaded From : www.EasyEngineering.net


Downloaded From : www.EasyEngineering.net

186 SPEED TEST 88


15. Who is the incumbent Managing Director of the 24. Who among the following has taken over charge as Deputy
International Monetary Fund (IMF)? Chairman of Kolkata Dock System?
(a) Christine Lagarde (b) Dominique Strauss-Kahn (a) Sudheer K Juneja
(c) Ban Ki-moon (d) Jim Yong Kim (b) Amit Kundan
(e) None of these (c) S Balaji Arunkumar
16. Who is the incumbent chairman of the National Bank for (d) K P Oli
Agriculture and Rural Development (NABARD)? (e) None of these
(a) Atulesh Jindal 25. Who among the following has been appointed as Chief
(b) Satynarayan Mohanty Executive Officer of its agri-machinery business by Escort
(c) Harsh Kumar Bhanwala Limited?
(d) Upendra Kumar Sinha (a) Ravi A Menon
(e) None of these (b) Ajit Kumar Sinha
17. Who has been appointed as an Asia-Pacific member of the (c) O P Juneja
International Coordinating Committee Bureau of NHRI-ICC? (d) Ram Kumar Adhikari
(a) Arun Jaitley (b) HL Dattu (e) None of these
(c) R M Lodha (d) H L Narayanaswamy 26. Consider the following statements:
18.
ww
(e) None of these
Who has been appointed as the advisor to the Khadi and
Village Industries Commission (KVIC) for promotion of khadi
1. Nita Ambani has been named the most powerful
businesswoman in Asia.
2. Arundhati Bhattacharya has been ranked third on the
(a) Payal Singhal
(c) Tarun Tahiliani
(e) None of these
w.E
within India and globally?
(b) Ritu Beri
(d) Manish Arora
2016 'Asia's 50 Power Businesswomen' list
3. The list has been released by Forbes.
Which of the above statements are correct?
19.
asy
Who is the newly appointed Governor of Central Bank of
Bangladesh?
(a) Only 1 & 2
(c) Only 2 & 3
(e) None of these
(b) Only 1 & 3
(d) All are correct
(a) Fazle Kabir
(c) AMA Muhith
(e) None of these
(b) Abdul Hamid
(d) Shahana Rahman
En 27. Who among the following persons has appointed as the
CEO of Suzlon Group?
20. Which Indian scholar has been appointed as the Vice
Chancellor of the University of Canberra? gin (a) Anita Ghosh
(b) J P Chalasani
(a) H Deep Saini (b) Sagar Mal
(c) Mandeep Singh (d) Stephen Parker
(e) None of these eer
(c) Ajit Chaudhary
(d) K S Sharma
(e) None of these
21. Who has been appointed as the new chairman of Bombay
Stock exchange (BSE)?
28.
ing
Who among the following has taken charge as the General
Manager (GM) of Pune unit of Bharat Electronics Ltd ?
(a) Sudhakar Rao
(c) Prem Kumar
(e) none of these
(b) Sethurathnam Ravi
(d) S Ramadorai
(a) M D Yadav
(b) M K Sharma
(c) Dinesh Kumar Batra .ne
22. Who of the following Indian-American investor took charge
as Alternate Executive Director at International Monetary
Fund (IMF)?
(a) Nikhil Chopra (b) Sunil Sabharwal
29.
(d) Advesh Singh Sharma
(e) None of these
Who among the following has been appointed as non-
official Director of CONCOR?
t
(c) Vikrant Singh (d) Dheerendra Payasi (a) R K Mishra (b) Sanjeev Shah
(e) None of these (c) M K Chaubey (d) Ajit Shah
23. Which Indian umpire has been promoted as the International (e) None of these
Outdoor Umpire by the International Hockey Federation 30. Who assumed charge as the new secretary of Railway
(FIH)? Board?
(a) Kumar Dixit (b) Jitendra Rai (a) Sanjoy Mookerjee (b) Gangaram Agarwal
(c) Nepoleon Singh (d) Mahendra Singh (c) R. K. Verma (d) P. C. Gajbhiye'
(e) None of these (e) None of these

15. a b c d e 16. a b c d e 17. a b c d e 18. a b c d e 19. a b c d e


RESPONSE 20. a b c d e 21. a b c d e 22. a b c d e 23. a b c d e 24. a b c d e
GRID 25. a b c d e 26. a b c d e 27. a b c d e 28. a b c d e 29. a b c d e
30. a b c d e

Downloaded From : www.EasyEngineering.net


Downloaded From : www.EasyEngineering.net

Events/Organisation/
Summits
89
Max. Marks : 30 No. of Qs. 30 Time : 20 min. Date : ........./......../. ..............

1. Which of the following countries is not a member of ASEAN ? 9. In which country the G-20 2016 summit will be organised ?
(a) Thailand (b) Vietnam (a) Mexico (b) South Korea
(c) Myanmar (d) Brazil (c) China (d) USA
(e) None of these (e) None of these
2.
ww
Project ‘Sankalp’ is associated with the eradication of -
(a) Polio (b) HIV/AIDS
10. The 42nd summit of G-7 will be held in ________ .
(a) USA
(c) Canada
(b) UK
(d) Japan

3.
(c) Illiteracy
(e) None of these
w.E
(d) Tuberculosis

The 19th SAARC summit will be which held in country- 11.


(e) None of these
Which of the following countries became the new
(a) India
(c) Dhaka
(b) Nepal
(d) Pakistan asy members of NAM ?
(a) Azerbaijan and Fiji

4.
(e) None of these
The 21th summit of ASEAN will be organised in- En (b) Syria and Comoras
(c) Surinam and Guyana
(a) UAE
(c) Mascow
(b) Brunei
(d) Laos gin (d) Columbia and Cyprus
(e) None of these

5.
(e) None of these
8th BRICS summit held in which country-
12.
(a) Russiaeer
The 2016 NATO summit will be organised in which country ?
(b) Poland
(a) Brazil
(c) S. Africa
(b) Russia
(d) India
(c) Canada
(e) None of these ing
(d) France

6.
(e) None of these
In which country the 2016 Annual Meeting of WEF organised ?
13.
(a) North Korea (b) South Korea .ne
In which country first woman elected as a president ?

(a) Switzerland
(c) Germany
(e) None of these
(b) France
(d) USA
14.
(c) Japan
(e) None of these
(d) China

Who is elected as a new pope of Roman Catholic Church


t
7. How many members in the APEC nations organisation ? recently ?
(a) 20 (b) 21 (a) Mario Bergoylio (b) George Allensary
(c) 22 (d) 23 (c) Telesfor Toppo (d) Ivan Dias
(e) None of these (e) None of these
8. The 17th NAM summit will be held in the country- 15. The new organisation named “UN-women” created by
(a) Iran (b) Kahira
united nations came into existence on ?
(c) India (d) Venezuela (a) 1 July 2010 (b) 1 July 2011
(e) None of these (c) 1 July 2012 (d) 1 July 2013
(e) None of these

1. a b c d e 2. a b c d e 3. a b c d e 4. a b c d e 5. a b c d e
RESPONSE
6. a b c d e 7. a b c d e 8. a b c d e 9. a b c d e 10. a b c d e
GRID
11. a b c d e 12. a b c d e 13. a b c d e 14. a b c d e 15. a b c d e

Downloaded From : www.EasyEngineering.net


Downloaded From : www.EasyEngineering.net

188 SPEED TEST 89


16. How many members are associated with BIMSTEC ? 25. 13th Edition of World Spice Congress was held in which
(a) 7 (b) 8 state?
(c) 9 (d) 10 (a) Gujarat (b) Rajasthan
(e) None of these (c) Kerala (d) Karnataka
17. What is the Ranking of India in Global Hunger Index list ? (e) None of these
(a) 63rd (b) 66th 26. The 12th Conference of Central Council of Health and Family
(c) 67th (d) 68th Welfare was held to enforce which policy?
(e) None of these (a) Draft National Health Policy
18. How many members are associated with the organisation (b) Draft National Family Welfare Policy
‘MERCOSUR’ ? (c) Draft National Welfare Policy
(a) 5 (b) 6 (d) Draft National Health Welfare Policy
(c) 7 (d) 8 (e) None of these
(e) None of these 27. Annual Conference of State Minorities Commission which
19.
ww
The 9th world Hindi Conference held in which country ?
(a) S. Africa (b) Sri Lanka
commenced on 24th February 2016 has which theme(s)?
(a) "Sabka Saath,Sabka Vikas"
(b) "Minority Welfare Schemes of Government of India-

20.
(c) Russia
(e) None of these
w.E (d) USA

How many points are mentioned to the achievement of


An Overview"
(c) "Functioning of State Minorities Commissions-
Millionium Development Goal LMDG-2015 of UN?
(a) 6 (b) 7 asy Problems and Challenges
(d) Both (b) and (c)

(c) 8
(e) None of these
(d) 9
En 28.
(e) None of these
Which maritime summit is being held for the first time in

21. Where is the headquarter of F.A.O. ?


(a) Italy (b) Paris gin India in April?
(a) Maritime India Summit

(c) Jeneva
(e) None of these
(d) Moscow
eer
(b) Maritime Cluster Development Summit
(c) Maritime Smart Cities Summit

22. (SAMPRITI-III), a special security forces exercise organised


between the countries of - (e) None of these ing
(d) Maritime Inland Waterways Summit

(a) India-Sri Lanka


(c) India-Russia
(b) India-Bangladesh
(d) India-USA
29.
2016?
.ne
Which website was launched for the Maritime Indian Summit

23.
(e) None of these
What does EAEU stand for?
(a) Eurasian Economic Union
(a) www.maritimeinvestment.com
(b) www.indianmaritime.in
(c) www.indiamaritimeinvestment.com
t
(b) European Economic Union (d) www.maritimeinvest.in
(c) European Energy Union (e) None of these
(d) Eurasian Energy Union 30. 10th WTO Ministerial Conference was held in which city?
(e) None of these (a) Nairobi
24. Nuclear Security Summit 2016 took place on which day? (b) Durban
(a) April 1, 2016 (b) April 2, 2016 (c) London
(c) April 3, 2016 (d) April 4, 2016 (d) New York
(e) None of these (e) None of these

16. a b c d e 17. a b c d e 18. a b c d e 19. a b c d e 20. a b c d e


RESPONSE
21. a b c d e 22. a b c d e 23. a b c d e 24. a b c d e 25. a b c d e
GRID
26. a b c d e 27. a b c d e 28. a b c d e 29. a b c d e 30. a b c d e

Downloaded From : www.EasyEngineering.net


Downloaded From : www.EasyEngineering.net

Awards and
Honours 90
Max. Marks : 30 No. of Qs. 30 Time : 20 min. Date : ........./......../. ..............

1. Jnanpith Award is given for which field? 9. ‘Ashoka Chakra’ is awarded for
(a) Journalism (b) Music (a) the most conspicuous bravery or self sacrifice on land,
(c) Science (d) Literature air or sea but not in the presence of the enemy
(e) None of these (b) acts of gallantry in the presence of enemy
2. Highest award given to civilian in India is (c) gallantry by children

ww
(a) Bharat Ratna
(c) Sharam Award
(e) None of these
(b) Padma Vibhushan
(d) Padma Bhushan
10.
(d) outstanding contribution to literature
(e) None of these
Shanthi Swaroop Bhatnagar awards are given for
3.

(c) 1954
w.E
In which year National Film Awards were initiated?
(a) 1952 (b) 1953
(d) 1955
(a) exploring new dimensions in creative writing in Indian
languages
(b) outstanding contribution to science

4.
(e) None of these
Vyas Samman is awarded annually by
(a) Azim Premji Foundation
asy (c) creating mass awareness on environmental issues
(d) excellence in film direction
(e) None of these
(b) Times Group
(c) KK Birla Foundation En 11. The prestigious Ramon Magsaysay Award was conferred
upon Mr.Arvind Kejriwal in which of the following category?
(d) Ministry of Culture
(e) None of these gin (a) Emergent Leadership
(b) Literature
5. Saraswati Samman is given to which field?
(a) Sanskrit Literature
(b) Science eer
(c) Community Welfare
(d) Government Service
(e) None of these
(c) Literature
(d) Social Harmony
12.
ing
B. C. Roy Award is given in the field of
(a) Medicine (b) Music

6.
(e) None of these
The second highest Gallantry award is
(a) Mahavir Chakra 13.
(c) Journalism
(e) None of these
(d) Environment

.ne
The Pampa Prashasti is the highest literacy award given by
(b) Vir Chakra
(c) Arjuna Award
(d) Ashok Chakra
(e) None of these
which of the following states?
(a) Karnataka
(b) Kerala
(c) Andhra Pradesh
t
7. Which of the following states conferred the Bihari Puraskar? (d) Maharashtra
(a) Uttar Pradesh (e) None of these
(b) Bihar 14. Which among the following states has won the 10th Na-
(c) Madhya Pradesh tional Award for Excellence work in Mahatma Gandhi Na-
(d) Rajasthan tional Rural Employment Guarantee Act (MGNREGA)?
(e) None of these (a) Madhya Pradesh
8. Tansen Samman is conferred in the field of : (b) Karnataka
(a) Music (b) Literature (c) West Bengal
(c) Science (d) Journalism (d) Haryana
(e) None of these (e) None of these

1. a b c d e 2. a b c d e 3. a b c d e 4. a b c d e 5. a b c d e
RESPONSE 6. a b c d e 7. a b c d e 8. a b c d e 9. a b c d e 10. a b c d e
GRID 11. a b c d e 12. a b c d e 13. a b c d e 14. a b c d e

Downloaded From : www.EasyEngineering.net


Downloaded From : www.EasyEngineering.net

190 SPEED TEST 90


15. Which of the following famous financial journals of interna- 23. Golden Globe award is given by
tional repute confers ‘Finance minister of the year’ Award? (a) UK (b) France
(a) Dalal Street (c) USA (d) China
(b) Euromoney (e) None of these
(c) Business Standard 24. Palme d’or prize is given by
(d) Money Matters (a) France (b) USA
(e) None of these (c) UK (d) Indonesia
16. The Nobel prize was instituted by which country? (e) None of these
(a) USA (b) UK 25. Which of the following is an award instituted by UNESCO?
(c) Russia (d) Sweden (a) Kalinga Award
(e) None of these (b) Pulitzer prize
17. When did the Nobel prize in the Economics Sciences (c) Stirling prize
launched? (d) Pritzker prize
(a) 1901 (b) 1942 (e) None of these

ww
(c) 1967
(e) None of these
(d) 1975 26. International Gandhi Peace prize is instituted in
(a) 1995
(c) 1997
(b) 1996
(d) 1998
18.
(a) Oscar Award
w.E
The Academy award is also known as
(b) BAFTA Award
(c) Matthews Award (d) Palm d’ore
27.
(e) None of these
The Stirling prize is a British prize for excellence in

19.
(e) None of these
Confucius peace prize is given by asy (a) Medicine
(c) Architecture
(e) None of these
(b) Science
(d) Literature

(a) Sri Lanka


(c) China
(b) India
(d) S. Korea
En 28. Which of the following award is given by World Economic
Forum?

20.
(e) None of these
Pulitzer prize was established in gin (a) Crystal Award
(b) Kalinga prize
(a) 1917
(c) 1922
(b) 1918
(d) 1928
eer
(c) Pulitzer Award
(d) Abel prize

21.
(e) None of these
Nobel prizes are distributed annually at 29.
(e) None of these
Magsaysay award is given by
ing
(a) Manila
(c) Stockholm
(b) New York
(d) Geneva
(a) USA
(c) Malaysia
(b) UK
(d) Philippines
.ne
22.
(e) None of these
BAFTA prize is distributed by
(a) UK
(c) India
(b) Russia
(d) USA
30.
(e) None of these
Booker prize is given to the field of :
(a) Fiction
(c) Drama
(b) Poetry
(d) Essay
t
(e) None of these (e) None of these

15. a b c d e 16. a b c d e 17. a b c d e 18. a b c d e 19. a b c d e


RESPONSE 20. a b c d e 21. a b c d e 22. a b c d e 23. a b c d e 24. a b c d e
GRID 25. a b c d e 26. a b c d e 27. a b c d e 28. a b c d e 29. a b c d e
30. a b c d e

Downloaded From : www.EasyEngineering.net


Downloaded From : www.EasyEngineering.net

Books and Authors 91


Max. Marks : 30 No. of Qs. 30 Time : 20 min. Date : ........./......../. ..............

1. Who is the author of “An Uncertain Glory: India and its 8. Who among the following is the author of “How to Get
Contradictions”? Filthy Rich in Rising Asia”?
(a) Amatya Sen & Michael Bush (a) Mohsin Hamid (b) Tahir Mahmood
(b) Amartya Sen & Satya Paul (c) Mohd Hamid (d) Mohd.Qazi
(c) Amartya Sen & Jean Dreze

ww
(d) Amartya Sen & Zeenat Shaukat
(e) Amartya Sen & Salman Rushdie
9.
(e) Mohamed Yusuf
Who has written the book “Walking with Lions: Tales from
a Diplomatic Past”?
2.
(a) Dr Dheerendra
(c) Dr Kiran Yadav
w.E
Who is the author of “Ambedkar Speaks (Triology)”?
(b) Dr Satyendra Singh
(d) Dr Narendra Jadhav
(a) P.Chidambaram (b) K.Natwar Singh
(c) Sashwant Sinha (d) Jaswant Singh

3.
(e) Akhilesh Yadav
Pax Indica is recently authored book by? asy 10.
(e) Manmohan Singh
A book ‘Fault Lines’ is written by -
(a) Raghuram Rajan (b) Subba Rao
(a) Narendra Modi
(c) Shashi Tharoor
(b) Atal Bihari Vajpayee
(d) A P J Abdul Kalam En (c) Vimal Jalan (d) Montek Singh Ahluwalia

4.
(e) None of these
Who is the author of “The Outsider”? gin
11.
(e) None of these
Which of the following books has been written by Vikram
Seth?
(a) Jimmy Connors (b) Amartya Sen
(c) Jean Dreze (d) Zeenat Shaukat eer
(a) My God Died Young
(b) Islamic Bomb (c) Look Back in Anger
5.
(e) Salman Rushdie
Name the book authored by Garima Sanjay which was
12.
(d) A Suitable Boy
ing
(e) None of these
Who is the author of the book India 2020?
released by Hamid Ansari. The book highlights how a man
blames the luck or someone for his failures? (a) Nibal Singh
(c) Sidney Shelton
(b) R.K.Narayan
.ne
(d) Dr.A.P.J.Abdul Kalam

6.
(a) Yaadein
(c) Divine Journey
(e) Man by Nature
(b) Smritiyan
(d) Lucky Me

Who is the author of “Women of Vision”?


13.
(e) None of these
The book ‘My Nation My Life’ was written by
(a) L.K.Advani (b) T.N.Seshan
t
(a) Amish Tripathi (b) Preeti Shenoy (c) Fervez Musharaf (d) Manmohan Singh
(c) Durjoy Dutta (d) Alam Srinivas (e) None of these
(e) Ravinder Singh 14. The famous book ‘Anandmath’ was authored by
7. Who among the following is the author of “Religion, Law & (a) Sarojini Naidu
Society - Across the Globe”? (b) Bankim Chandra Chottapadhya
(a) Salman Rushdie (b) Tahir Mahmood (c) Sri Aurobindo
(c) Jim Herley (d) Mohd.Raza (d) Rabindrnath Tagore
(e) None of these (e) None of these

1. a b c d e 2. a b c d e 3. a b c d e 4. a b c d e 5. a b c d e
RESPONSE 6. a c d e 7. a c d e 8. a c d e 9. a c d e 10. a c d e
b b b b b
GRID 11. a c d e 12. a c d e 13. a c d e 14. a c d e
b b b b

Downloaded From : www.EasyEngineering.net


Downloaded From : www.EasyEngineering.net

192 SPEED TEST 91


15. Who is the author of “INDIA : The Future is Now”. 23. Which of the following books is written by Panini?
(a) Robin Sharma (b) Shashi Tharoor (a) Mudrarakshas (b) Ashtadhyayi
(c) Amartya Sen (d) Arundhati Roy (c) Prem Vatika (d) Bijak
(e) None of these (e) None of these
16. ‘Freedom from Fear’ is a book written by ?
24. The book ‘Mitakshara’ is written by
(a) Benzir Bhutto (b) Corazon Aquino
(a) Vatsyayana (b) Jeemaatwahan
(c) Aung san suu Kyi (d) Nayantara Seghal
(e) None of these (c) Vigyaneshwar (d) Shudrak
17. ‘India of our Dreams’ is a book written by (e) None of these
(a) Dr. S. Radhakrishnan 25. Which of the following books is written by Kautilya?
(b) Dr. C. Subramanian (a) Daybhag (b) Rajtarangini
(c) M.V. Kamath (c) Arthashastra (d) Mitakshara
(d) Dr. Rajendra Prasad (e) None of these
(e) None of these 26. Post Office’ is written by?
18. Which of the following books is written by Sunil Gavaskar? (a) R.K. Narayan

ww
(a) A Brief History of Time
(b) A Sense of Time (c) Sunny Days
(d) Great Expectations (e) None of these
(b) Mulk Raj Anand
(c) R.K Laxman
19.
Premchand?
(a) Gaban
w.E
Which of the following is NOT written by Munshi

(b) Guide 27.


(d) Rabindra Nath Tagore
(e) None of these
Who is called the Father of English Poetry?
(c) Godan
(e) None of these
(d) Manasorovar
asy (a) Charles Dickens (b) Milton
(c) Chaucer (d) Wordsworth
20. Who is the author of the book ‘Beyond the Lines : An
Autobiography’ ? En 28.
(e) None of these
Who wrote Jungle Book?
(a) General J. J. Singh (b) Kuldip Nayar
(c) Ray Bradbury (d) Khushwant Singh
gin (a) Mohd. Salim
(b) Rudyard Kipling

21.
(e) None of these
Who among the following is the author of the book “India
and Malaysia: Intertwined Strands”?
(c) Sibhu
eer
(d) R.K. Narayan
(a) Veena Sikri
(c) Sushma Singh
(b) Naveen Bandopadhya
(d) Ravichandran Nayak 29.
(e) None of these
Who wrote ‘War and Peace’? ing
22.
(e) None of these
The Global Competitiveness Report is published from time
(a) Leo Tolstoy
(c) Charles Dickens (d) Kipling .ne
(b) Mahatma Gandhi

to time by ___?
(a) World Bank
(b) International Monetary Fund
(c) Yale University
30.
(e) None of these
Who wrote the book ‘The Prince’?
(a) Bernard Shaw
(c) V.S. Sharma
(b) Niccolo Machiaveli
(d) Emile Zola
t
(d) World Economic Forum
(e) None of these
(e) None of these

15. a b c d e 16. a b c d e 17. a b c d e 18. a b c d e 19. a b c d e


RESPONSE 20. a b c d e 21. a b c d e 22. a b c d e 23. a b c d e 24. a b c d e
GRID 25. a b c d e 26. a b c d e 27. a b c d e 28. a b c d e 29. a b c d e
30. a b c d e

Downloaded From : www.EasyEngineering.net


Downloaded From : www.EasyEngineering.net

Sports and Games 92


Max. Marks : 30 No. of Qs. 30 Time : 20 min. Date : ........./......../. ..............
1. The Olympic Museum was opened at which of the following 10. ‘Ashes’ is the term associated with which of the following
places? sports?
(a) Rome (b) Berlin (a) Cricket (b) Badminton
(c) Lausanne (d) Athens (c) Basketball (d) Football
(e) None of these (e) None of these
2.

ww
With which sport the term’ Caddie’ is associated?
(a) Polo
(c) Bridge
(b) Golf
(d) Billiards
11. National Sports Day is celebrated on
(a) 29th Aug.
(c) 14th Nov.
(b) 4th Dec.
(d) 28th Oct.

3.
(e) None of these
w.E
Champion Trophy is associated with
(a) Football (b) Hockey
12.
(e) None of these
The term ‘bogey’ is associated with

(c) Cricket
(e) None of these
(d) Chess
asy (a) Cricket
(c) Golf
(e) None of these
(b) Chess
(d) Baseball
4. Rangaswami Cup is associated with
(a) Wrestling (b) Football
En 13. FINA is governing body of which sports?
(a) Cricket (b) Archery
(c) Hockey
(e) None of these
(d) Golf

gin (c) Water polo


(e) None of these
(d) Polo
5. ‘Grand Slam’ is associated with the game of
(a) Lawn Tennis
(c) Football
(b) Hockey
(d) Swimming
14.
eer
Which of the following trophies/cups associated with the
game of Hockey?

6.
(e) None of these
First youth Olympic games was held in
(a) Derby
(c) Merdeka ing
(b) Aga Khan Cup
(d) Vizzy Trophy
(a) Japan
(c) North Korea
(b) China
(d) Singapore 15.
(e) None of these
The ‘Dronacharya Award’ is given to
.ne
7.
(e) None of these
‘Subroto Cup’ is associated with
(a) Badminton
(c) Chess
(b) Cricket
(d) Football 16.
(a) Coaches
(c) Umpires
(e) None of these
(b) Sportspersons
(d) Sports Editors

When did the Wimbledon Grand Slam Tennis tournament


t
(e) None of these start?
8. Wankhede Stadium is situated in (a) 1857 (b) 1877
(a) Mumbai (b) Delhi (c) 1897 (d) 1898
(c) Lucknow (d) Bangalore (e) None of these
(e) None of these 17. In which year, the Grand Master title of Chess started?
9. The term ‘Gambit’ is associated with (a) 1971 (b) 1972
(a) Chess (b) Tennis (c) 1973 (d) 1974
(c) Basketball (d) Baseball (e) None of these
(e) None of these

1. a b c d e 2. a b c d e 3. a b c d e 4. a b c d e 5. a b c d e
RESPONSE 6. a b c d e 7. a b c d e 8. a b c d e 9. a b c d e 10. a b c d e
GRID 11. a b c d e 12. a b c d e 13. a b c d e 14. a b c d e 15. a b c d e
16. a b c d e 17. a b c d e

Downloaded From : www.EasyEngineering.net


Downloaded From : www.EasyEngineering.net

194 SPEED TEST 92


18. Duleep Trophy is associated with the game of 25. Indian Sports Research Institute is located at
(a) Hockey (b) Badminton (a) Patiala (b) Delhi
(c) Football (d) Cricket (c) Cochin (d) Poona
(e) None of these (e) None of these
19. Which game is associated with Queensberry rules? 26. Who was the first Indian to win an individual medal in Olym-
(a) Weight lifting (b) Boxing pics?
(c) Golf (d) Polo (a) PT Usha
(e) None of these (b) Karnam Malleshwari
20. How many players are there in Kho-Kho? (c) Deepika Kumari
(a) 9 (b) 10 (d) Sania Nehwal
(c) 8 (d) 7 27. Which county did Ravi Shastri play for?
(e) None of these (a) Glamorgan (b) Leicestershire
21. What is the National Game of Russia? (c) Gloucestershire (d) Lancashire
(a) Chess (b) Hockey (e) None of these

22.
ww
(c) Table Tennis
(e) None of these
(d) Baseball

Hurlington stadium is associated with


28. The first World Cup Hockey was played in
(a) Amsterdom, 1972
(b) Barcelona, 1971
(a) Polo
(c) Boxing w.E (b) Cricket
(d) Golf
(c) Kualalumpur, 1975
(d) Mumbai, 1976
(e) None of these
23.
(e) None of these
First Olympic Games were held in-
(a) 776 BC. (b) 798 BC. asy 29. The normal length of a football ground must be
(a) 110 – 120 m (b) 100 – 110 m
(c) 876 BC.
(e) None of these
(d) 898 BC.
En (c) 90 – 100 m
(e) None of these
(d) 120 – 130 m

24. With which game is Bully associated?


(a) Cricket (b) Football gin
30. India first won the Olympic Hockey gold at
(a) Amsterdam (b) Los Angeles
(c) Golf
(e) None of these
(d) Hockey (c) Mumbai
eer
(e) None of these
(d) Tokyo

ing
.ne
t

18. a b c d e 19. a b c d e 20. a b c d e 21. a b c d e 22. a b c d e


RESPONSE 23. a b c d e 24. a b c d e 25. a b c d e 26. a b c d e 27. a b c d e
GRID 28. a b c d e 29. a b c d e 30. a b c d e

Downloaded From : www.EasyEngineering.net


Downloaded From : www.EasyEngineering.net

Science and
Technology 93
Max. Marks : 25 No. of Qs. 25 Time : 20 min. Date : ........./......../. ..............

1. Line connecting the points on a map that have the same 8. What is the name of Australia’s most powerful super
temperature is called? computer ?
(a) Isobar (b) Isotherm (a) Spirit (b) Raijin
(c) Isohyet (d) Isohels (c) Aizen (d) Shinto
(e) None of these
(e) None of these
2.

ww
Rearing of silkworms for the production of raw silk is called?
(a) Horticulture
(c) Viticulture
(b) Sericulture
(d) Apiculture
9. Which was the first atomic submarine of India?
(a) INS Chakra (b) INS Vikrant

3.
(e) None of these
w.E
In which of the following seas India has building Tsunami
warning device? 10.
(c) INS Dhanush
(e) None of these
(d) INS Viraat

Vikram Sarabhai Space Centre (VSSC) is at


(a) Arabian Sea
(c) Bay of Bengal
(b) South China Sea
(d) Indian Ocean asy (a) Thiruvananthapuram
(b) Mumbai

4.
(e) None of these
GlaxoSmithKline developed world’s first malaria vaccine
En (c) Hyderabad
(d) Bengaluru
called ___?
(a) RTS, S
(c) RMS, S
(b) MTS, S
(d) MRS, S gin
11.
(e) None of these
The first fertilizer plant in India was established in

5.
(e) None of these
With which of the following Kepler’s Laws are related to (c) Alwayeeer
(a) Trombay (b) Nangal
(d) Sindri
_______?
(a) Motion of Milkyway
12.
(e) None of these
ing
Name of first indigenously developed Super Computer of
(b) Motion of planets around sun
(c) Rotation of Earth on Its own axis
India is?
(a) Param (b) Aryabhatt .ne
6.
(d) Motion of Moon around earth
(e) None of these
Where is located the Centre for Wind Energy Technology
(C-WET)? 13.
(c) Apsara
(e) None of these
(d) Tejas

Which of the following is the principal ore of Mercury?


t
(a) Hyderabad (b) Chennai (a) Bauxite (b) Cinnabar
(c) Kochi (d) Kolkata (c) Hematite (d) Galena
(e) None of these
(e) None of these
7. Which of the following statements is true about “TomTato”?
14. Name the Chinese supercomputer which is declared the
(a) A new mammal species discovered in UK
fastest computer of the world-
(b) A plant produces both tomatoes and potatoes
(c) A man eating fish species (a) Tianhe-2 (b) Chinhane 1
(d) New element discovered under deep sea (c) Kisova (d) Techo-1
(e) None of these (e) None of these

1. a b c d e 2. a b c d e 3. a b c d e 4. a b c d e 5. a b c d e
RESPONSE 6. a b c d e 7. a b c d e 8. a b c d e 9. a b c d e 10. a b c d e
GRID 11. a b c d e 12. a b c d e 13. a b c d e 14. a b c d e

Downloaded From : www.EasyEngineering.net


Downloaded From : www.EasyEngineering.net

196 SPEED TEST 93


15. Name the organization that launched a series of satellites 21. The Rotavirus vaccine ROTOVAC developed by Indian
which is supposed to provide fast, cheap Internet and phone scientists cures which disease?
service to remote rural areas in 180 countries. (a) Diarrohea (b) Cancer
(a) A2B Networks (b) R2R Networks (c) Diabetes (d) Arthritis
(c) O3B Networks (d) M2M Networks (e) Pneumonia
(e) B2B Networks
22. India's first DNA Forensic Laboratory is established in which
16. Name the place in India where Early Tsunami Warning city
System have been installed
(a) Gurgaon (b) Mumbai
(a) Rangachang (b) Kanyakumari
(c) Delhi (d) Kanpur
(c) Chilka (d) Mysore
(e) Banglore
(e) Ootkamandalam
23. "A Boy and His Atom" - World's smallest movie made with
17. Researchers have developed Eco-friendly batteries which
can be used in power plants or to store solar energy.These one of the smallest particles of any element in the universe:
batteries are made up using which material atoms is developed by which organization
(a) Carbon, Tin, Sodium (a) Microsoft

ww
(b) Wood, Tin, Carbon
(c) Wood, Tin and Sodium
(b) Infosys
(c) IBM

18.
w.E
(d) Sodium, carbon, Tin
(e) Nickel, Carbon, Zinc
Which of the following is an indigenously built light combat 24.
(d) Toshiba
(e) Sony
Which among the following is the First cruise missile test
aircraft of India?
(a) Akash (b) Vikrant asy fired by India?
(a) Aakash
(c) Tejas
(e) None of these
(d) Arjun

En (b) Nirbhay
(c) Agni-3
19. Name the broadband and telecommunications provider
which launched world's first Firefox OS smartphone?
(a) Telefonica (b) Apple
gin (d) Aakash-2
(e) Prithvi-3

(c) Samsung
(e) None of these
(d) Telenor
25.
eer
First Indian to go into space
(a) Mohan Sharma

20. The First Navigation Satellite launched by ISRO


(a) PSLV C 2 (b) IRNSS-1A
(b) Rakesh Chaudhary
(c) Rakesh Sharma ing
(c) ISS-1A
(e) Edusat
(d) INSAT
(d) Suresh Sharma
(e) None of these
.ne
t

15. a b c d e 16. a b c d e 17. a b c d e 18. a b c d e 19. a b c d e


RESPONSE
20. a b c d e 21. a b c d e 22. a b c d e 23. a b c d e 24. a b c d e
GRID
25. a b c d e

Downloaded From : www.EasyEngineering.net


Downloaded From : www.EasyEngineering.net

Current Banking 94
Max. Marks : 23 No. of Qs. 23 Time : 15 min. Date : ........./......../. ..............
1. India's first Aadhaar enabled ATM has been launched by 8. Which bank has launched 'Japan Desk' to facilitate Japanese
which bank? corporates investing in India?
(a) State Bank of India (b) ICICI Bank (a) State Bank of India (b) Punjab National Bank
(c) DCB Bank (d) Axis Bank (c) Bank of Baroda (d) Dena Bank
(e) Indian Bank (e) ICICI Bank
2.

ww
Which is the first bank to commence the sale of Indian Gold
Coin (IGC) in the domestic market?
(a) State Bank of India (b) Indian Overseas Bank
9. Which of the following agreement is signed between Reserve
Bank of India (RBI) and UAE Central Bank?
(a) Monetary Exchange Agreement

3.
(e) Indian Bank w.E
(c) Punjab National Bank (d) Bank of Baroda

Who is the newly appointed Governor of Central Bank of


(b) Currency Swap Agreement
(c) Anti-Money Laundering Agreement
(d) Rupee Exchange Agreement
Bangladesh?
(a) Fazle Kabir asy
(b) Abdul Hamid
10.
(e) Money Exchange Agreement
Who has been appointed as Vice-President of Asian
Infrastructure Investment Bank (AIIB)?
(c) AMA Muhith
(e) None of these
(d) Shahana Rahman

En (a) Kamal Vatta


(c) Mitali Saran
(b) D J Pandian
(d) Sreenivasan Kumar
4. India's first contact-less mobile payment solution "iTap" has
been launched by which bank?
(a) State Bank of India (b) HDFC Bank
gin
11.
(e) None of these
Who has been appointed from India as Senior Director in

(c) Axis Bank Ltd


(e) None of these
(d) ICICI Bank
eer
the World Bank?
(a) Amitabh Singh
(c) Suvarna Kumar
(b) Saroj Kumar Jha
(d) Nikhil Srivastav
5. Which bank has launched iWork@home programme for its
women employees? 12.
(e) None of these
ing
Which bank has recently raised `900 crore by way of issuing
(a) Yes bank
(c) ICICI Bank
(b) Exim Bank
(d) Axis Bank Ltd.
Basel-III compliant tier-II bonds?
(a) IDBI Bank .ne
(b) Central Bank of India

6.
(e) HDFC Bank
Which of the following banks will become the India's first
Small Finance Bank (SFB)?
(a) Ujjivan Financial Services Pvt. Ltd
13.
(c) Syndicate Bank
(e) State Bank Of India
(d) Canara Bank

Which public sector bank has bagged three awards at the


'National Payments Excellence Awards 2015, organized by
t
(b) Capital Local Area Bank Ltd National Payments Corporation of India (NPCI)?
(a) Bank of Baroda (b) State Bank of India
(c) Au Financiers Ltd
(c) Allahabad Bank (d) Corporation Bank
(d) Janalakshmi Financial Services Pvt. Ltd
(e) None of these
(e) None of these
14. Which bank has recently launched Green PIN facility, under
7. Which bank has won the first ever Green Bond Pioneer Award
which a customer can obtain duplicate PIN for debit card
2016?
instantly through SMS request?
(a) State Bank of India (b) Bank of Baroda
(a) Punjab National Bank (b) ICICI Bank
(c) Punjab National Bank (d) Yes Bank
(c) Dena Bank (d) HDFC Bank
(e) ICICI Bank
(e) None of these

1. a b c d e 2. a b c d e 3. a b c d e 4. a b c d e 5. a b c d e
RESPONSE 6. a b c d e 7. a b c d e 8. a b c d e 9. a b c d e 10. a b c d e
GRID 11. a b c d e 12. a b c d e 13. a b c d e 14. a b c d e

Downloaded From : www.EasyEngineering.net


Downloaded From : www.EasyEngineering.net

198 SPEED TEST 94


15. Reserve Bank has imposed a penalty of Rs. 1 crore on which (a) South Korea (b) China
associate bank of SBI for violation of its instructions (c) Brazil (d) South Africa
including reporting of data to Central Repository of (e) None of these
Information on Large Credits (CRILC)? 20. Under what name, State Bank of India has launched a
(a) State Bank of Patiala specialised branch in Bengaluru that caters to startups?
(b) State Bank of Bikaner and Jaipur (a) InCube (b) Foster
(c) State Bank of Hyderabad (c) KickStart (d) SBIHatch
(d) State Bank of Travancore (e) None of these
(e) None of these 21. Which public sector bank has tied up with e-commerce major
16. Who among the following has been reappointed as Deputy Snapdeal to offer instant working capital loans to its online
Governor of RBI for three year term? e-commerce sellers?
(a) Rakesh Mohan (b) Urjit Patel (a) Bank of Baroda (b) State Bank of India
(c) H R Khan (d) S S Mundra (c) Corporation Bank (d) Syndicate Bank
(e) None of these (e) None of these
17. In January 2016, which bank had launched Immediate 22. The 2016 World Development Report by World Bank noted

ww
Payment Service (IMPS) across branches to provide inter-
bank electronic transfer service, capable of processing
person to account remittances?
that almost 1.063 billion Indians were offline even though
India ranked among the top five nations in terms of the total

(c) Andhra Bank w.E


(a) Central Bank of India (b) Canara Bank
(d) Corporation Bank
number of Internet users. What is the title of 2016 World
Development Report?
(a) 'Technology Inclusion'

18.
(e) None of these

asy
Which private sector bank will partner with FINO PayTech
to foray into the payments bank space?
(b) 'Digital Dividends'
(c) 'Sustainable Technology'
(d) 'Bridging Technology Divide'
(a) Axis Bank
(c) Yes Bank
(b) HDFC Bank
(d) ICICI Bank En 23.
(e) None of these
Which Central Bank has imposed negative interest rate in

19.
(e) None of these
Which country has recently allowed Six central banks and gin its monetary policy statement of January 2016?
(a) Bank of England (b) People's Bank of China
international financial institutions, including Reserve Bank
of India (RBI), to enter China's interbank foreign exchange
market from January 2016? eer
(c) Bank of Japan
(e) None of these
(d) European Central bank

ing
.ne
t

RESPONSE 15. a b c d e 16. a b c d e 17. a b c d e 18. a b c d e 19. a b c d e

GRID 20. a b c d e 21. a b c d e 22. a b c d e 23. a b c d e

Downloaded From : www.EasyEngineering.net


Downloaded From : www.EasyEngineering.net

Current Affairs 95
Max. Marks : 30 No. of Qs. 30 Time : 15 min. Date : ........./......../. ..............
1. The Pradhan Mantri Ujjwal Yojana will provide - 8. Where was the fourth Nuclear Security Summit (NSS) held?
(a) free electricity to the rural households. (a) London (b) Hong Kong
(b) free higher education to the girl child who belongs to a (c) Washington (d) New Delhi
poor family. (e) None of these
(c) LPG to women of all households below poverty line. 9. Who was appointed as the advisor of the Andhra Pradesh
(d) scholarships to the children of the people below Government by Chief Minister Chandrababu Naidu?

ww
poverty line.
(e) None of these
(a) Baba Ram dev
(b) Kodali Venkateswara Rao
(c) Hari Krishna
2.
(a) A P Shan
(b) Arvind Pangariyaw.E
Who is the Vice Chairman of NITI AAYOG in India?

10.
(d) Chaganti Koteswara Rao
(e) None of these
Who was sworn-in as the President of Kosovo in April 2016?
(c) Rakesh Tripath
(d) B.P. Jeevan Reddy
(e) None of these asy (a) Jakup Krasniqi
(c) Atifete Jahjaga
(e) None of these
(b) Hashim Thaci
(d) Fadil Hoxha

3. Who is the winner of the prestigious 2016 Abel Prize?


(a) Endre Szemeredi (b) Andrew Wiles
En 11. Which team topped the April 2016 edition of FIFA or Coca-
Cola World Rankings of Soccer?

4.
(c) John Rognes
(e) None of these
(d) Michael Atiyah

What is the rank of India in the United Nations, World


gin (a) Argentina
(c) France
(e) None of these
(b) Brazil
(d) Italy

Happiness Index 2016?


(a) 118 (b) 105
12.
eer
Which online furniture store was acquired by the Future
Group in the first week of April 2016?
(c) 98
(e) None of these
(d) 125 (a) FabFurnish.com
(c) Pepperfry.com
(e) None of these ing(b) UrbanLadder.com
(d) Ezaara.com
5. Where was 13th meeting of Broadband Commission for
Sustainable Development held?
(a) New York (b) Dubai
13.
.ne
The book 'Endurance: My Year in Space and Our Journey to
Mars' as announced by the publisher Alfred A. Knopf, is a

6.
(c) Switzerland
(e) None of these
(d) Mexico

March 15 is celebrated across the world as?


story of which famous personality?
(a) Scott Kelly
(c) Mikhail Korniyenko
(e) None of these
(b) Mark Kelly
(d) Timothy Kopra t
(a) World Human Rights Day 14. Who is going to succeed Vijay Mallya as the chairman of
(b) World Health Day United Spirits Limited?
(c) World Consumer Rights Day (a) Anand Kripalu (b) M.K. Sharma
(d) World Sleep Day (c) Nicholas Blazquez (d) Angus Mc Dowell
(e) None of these (e) None of these
7. The fifth edition of Indian Aviation show was conducted in 15. What is the theme of 2016 World Health Day (WHD)?
which city? (a) Healthy heart beat, Healthy blood pressure
(a) Odisha (b) Mumbai (b) Beat Diabetes
(c) Bangalore (d) Hyderabad (c) Healthy blood pressure
(e) None of these (d) Save lives: Make hospitals safe in emergencies
(e) None of these

1. a b c d e 2. a b c d e 3. a b c d e 4. a b c d e 5. a b c d e
RESPONSE 6. a b c d e 7. a b c d e 8. a b c d e 9. a b c d e 10. a b c d e
GRID 11. a b c d e 12. a b c d e 13. a b c d e 14. a b c d e 15. a b c d e

Downloaded From : www.EasyEngineering.net


Downloaded From : www.EasyEngineering.net

200 SPEED TEST 95


16. Who won the 2016 Formula 1 Rolex Australian Grand Prix at 23. India extended 2 billion US dollar line of Credit to which
Melbourne Grand Prix? country for implementing Socio-economic development
(a) Sebastian Vettel (b) Nico Rosberg projects?
(c) Daniel Ricciardo (d) Lewis Hamilton (a) Bangladesh (b) Nepal
(e) None of these (c) Sri Lanka (d) Myanmar
17. Which unique Biosphere Reserve of India in the Western (e) None of these
Ghats is among 20 new sites added by the UNESCO to its 24. Who was appointed as the Chairman of Central
World Network of Biosphere Reserves? Administrative Tribunal in April 2016?
(a) Great Nicobar (a) Justice (retd) Permod Kohli
(b) Simlipal Biosphere Reserve (b) Justice Syed Rafat Alam
(c) Agasthyamala Biosphere (c) Justice A P Shah
(d) Nanda Devi Biosphere Reserve (d) None of them
(e) None of these (e) None of these
18. Who was conferred with skoch Lifetime Achievement Award 25. Which has become the first state to pass land title bill?
in 2016? (a) Rajasthan (b) Bihar
(a) Venkaiah Naidu (b) Arun Jaitley (c) Punjab (d) Karnataka

ww
(c) Rajnath Singh
(e) None of these
(d) L K Advani
26.
(e) None of these
Who on 6 April 2016 was appointed the Head of Lamborghini
India Operations?
19.

w.E
Bedaquiline drug launched by the Health Ministry will be
used to treat which disease?
(a) Dengue (b) Tuberculosis
(a) Vincet Parker
(c) Himanshu Sharma
(e) None of these
(b) Sharad Agarwal
(d) Vivek Agnihotri

20.
(c) Malaria
(e) None of these
asy
(d) Zika Virus

Which veteran singer was awarded the Global Green Hero


27. Who was awarded the UNESCO or Guillermo Cano World
Press Freedom Prize 2016?
Award 2016?
(a) Stevie Wonder (b) Lionel Richie
En (a) Mazen Darwish
(c) Ahmet ??k
(e) None of these
(b) Khadija Ismayilova
(d) None of the above

21.
(c) Whitney Houston
(e) None of these
(d) None of the above

What was the theme of International women's Day in 2016?


gin
28. Who wrote the book titled "India's War: The Making of
Modern South Asia 1939-1945"?
(a) Equality for women is progress for all
(b) Planet 50-50 by 2030: Step It up for Gender Equality eer
(a) Srinath Raghavan
(c) Sreedhar Chandan
(b) Mini Kapoor
(d) Suhasini Haidar
(c) Empowering women, Empowering Humanity: Picture it!
(d) Empowerment of women for sustainable Development
29.
(e) None of these

ing
Who has won the 2016 US Masters golf tournament?
(a) Jordan Spieth (b) Lee Westwood

22.
(e) None of these
The Indian women's team on 6 March 2016 defeated which
country to bag gold medal at the Second Division of the
(c) Danny Willett
(e) None of these
.ne
(d) J Donaldson

World Team Table Tennis Championship?


(a) China
(c) Luxembourg
(e) None of these
(b) Brazil
(d) Japan
30. Whose birth anniversary was celebrated on April 13, 2016
at the UN headquarters?
(a) B R Ambedkar
(c) Florence Nightingale
(b) Annie Besant
(d) George Washington
t
(e) None of these

16. a b c d e 17. a b c d e 18. a b c d e 19. a b c d e 20. a b c d e


RESPONSE
21. a b c d e 22. a b c d e 23. a b c d e 24. a b c d e 25. a b c d e
GRID
26. a b c d e 27. a b c d e 28. a b c d e 29. a b c d e 30. a b c d e

Downloaded From : www.EasyEngineering.net


Downloaded From : www.EasyEngineering.net

96
Section Test :
General & Financial
Awareness
Max. Marks : 50 No. of Qs. 50 Time : 30 min.
Date : ........./......../. ..............
1. Which of the following games is not included in Olympic
games? 8. Which of the following is not(b)a Public Sector
United Bank Bank?
of India
(a) Football (b) Golf (a) Corporation
(c) Vijaya Bank Bank (d) Bank of Maharashtra
(c) Badminton (d) Hockey (e) Federal Bank
(e) Table Tennis 9. A form of intentional weather modification by changing to
2.
ww
Which of the following awards is given only to individuals
and organisations of Asian countries for excellence in their
respective fields?
trying the amount or type of precipitation that falls from clouds
by dispersing substances into air that serve as cloud

(a) Booker Prize


(c) Templeton Prize w.E (b) Nobel Prize
(d) Ramon Magsaysay Award
condensation is called ___________.
(a) Cloud Computing
(c) Cloud Seeding
(b) Cloud Technology
(d) Cloud Control

3.
(e) Oscar awards

asy
Which Indian city is known as the ‘Garden City’?
(a) Udaipur (b) Chandigarh
10.
(e) Cloud Engineering
The North Atlantic Treaty Organisation (NATO) is
(c) Srinagar
(e) Mysore
(d) Bangaluru
En headquartered at _________.
(a) Camp David, Maryland USA
4. Who amongst the following is the author of the English novel
‘The White Tiger’? gin (b) Geneva
(c) New York
(a) Kiran Desai
(c) Shashi Tharoor
(b) V. S. Naipaul
(d) Shobha De
(d) Sweden

eer
(e) Brussels

5.
(e) Aravind Adiga
Under the RTI Act the time for disposal of request for
information in cases concerning life and liberty is _________.
11.

ing
Which of the following is the abbreviated name of a rural
sports initiative introduced by the Government of India to
promote youth and social development through sports?
(a) 30 days
(c) 10 days
(b) 15 days
(d) 7 days
(a) NLMP
(c) CGHS
(b) PYKKA
(d) AABY .ne
6.
(e) 48 hours
World ‘No Tobacco Day’ is observed every year on
__________.
12.
(e) SGSRY
National Sample Survey Organisation is an organisation of
(a) Ministry of statistics and programme implementation
t
(a) 31st January (b) 31st May
(b) Planning Commission
(c) 31st July (d) 31st October
(c) Home Affairs
(e) 31st December
(d) RBI
7. Which of the following is not a Consumer Right as per
(e) None of these
Consumer Protection Act 1986?
13. License to establish a commercial bank in India is issued
(a) Right to Consumer Education
(b) Right to Seek Redressal by—
(c) Right to Safety (a) Ministry of Finance (b) Registrar of companies
(d) Right to be Informed (c) Reserve Bank of India (d) Planning Commission
(e) Right to Negotiate (e) None of these

1. a b c d e 2. a b c d e 3. a b c d e 4. a b c d e 5. a b c d e
RESPONSE 6. a b c d e 7. a b c d e 8. a b c d e 9. a b c d e 10. a b c d e
GRID 11. a b c d e 12. a b c d e 13. a b c d e

Downloaded From : www.EasyEngineering.net


Downloaded From : www.EasyEngineering.net

202 SPEED TEST 96


14. On which one of the following issues can SEBI penalize any 20. India vision-2020 is prepared by
company in India? (a) S.C. Gupta Committee
(A) Violation of Banking Regulation Act. (b) Panth Committee
(B) Violation of foreign portfolio investment guidelines. (c) Malhotra Commitee
(C) For violation of Negotiable Instrument Act.
(d) Narasimha Committee
(a) Only (A) (b) All (A), (B) & (C)
(e) Kelkar Committee
(c) Only (A) & (B) (d) Only (B) & (C)
21. The National Maritime Day of India is celebrated on which
(e) Only (B)
day?
15. Which one the following is the west flowing river?
(a) Narmada (b) Krishna (a) April 3 (b) April 4
(c) Godavari (d) Cauvery (c) April 5 (d) April 6
(e) Ganga (e) None of these
16. Which of the following best defines a floating-rate bond? 22. The "Mullaperiyar Dam" has been a subject of controversy

ww
(a) A bond with a fixed interest rate and has better yield
than varying interest rate bond
between which of the following two states?
(a) U.P and Uttarakhand

w.E
(b) A bond with a fixed interest rate and has lower yield
than varying interest rate bond
(c) A bond with a varying interest rate and has better yield
(b) Kerala & Tamil Nadu
(c) Telangana and Andhra Pradesh

than fixed interest rate bond


asy
(d) A bond with a varying interest rate and has lower yield
(d) Kerala & Karnataka
(e) None of these

than fixed interest rate bond


(e) None of these En 23. The headquarters of the Confederation of Indian Industry
(CII) is located at_____.
17. Once a Budget has been presented in the Parliament, the
government has to get all money bills related to the union gin (a) Mumbai
(c) New Delhi
(b) Chandigarh
(d) Chennai
budget passed within __?
(a) 30 Days (b) 60 Days (c) 75 Days 24. eer
(e) None of these
Which of the following is India's first state to link government

18.
(d) 90 Days (e) None of these
Which among the following regulate the commodity markets (a) Rajasthan ing
colleges to national cancer grid?
(b) Maharashtra
in India?
1. RBI
(c) Madhya Pradesh
.ne
(d) Odisha Hide

2. SEBI
3. Forward Market Commission
Choose the correct option from the codes given below:
25.
(e) None of these

t
Which of the following villages has become the first liquor-
free village in Rajasthan?
(a) Only 1 & 2 (b) Only 2 & 3 (c) Only 1 & 3 (a) Dausa (b) Kachhabali
(d) Only 3 (e) None of these (c) Churu (d) Achalpura
19. The credit policy of a bank does not consists of? (e) None of these
(a) Lending policies 26. Kakrapara Atomic Power Station' is located in which state
(b) Quality control of India?
(c) Loan product mix (a) Gujarat (b) Karnataka
(d) Advertising of loan products (c) Maharashtra (d) Rajasthan
(e) None of these (e) None of these

14. a b c d e 15. a b c d e 16. a b c d e 17. a b c d e 18. a b c d e


RESPONSE 19. a b c d e 20. a b c d e 21. a b c d e 22. a b c d e 23. a b c d e
GRID 24. a b c d e 25. a b c d e 26. a b c d e

Downloaded From : www.EasyEngineering.net


Downloaded From : www.EasyEngineering.net

SPEED TEST 96 203

27. Which committee has recommended change in the structure 34. Gurmeet Singh is associated with which sports?
and ecosystem in the Indian cricket board? (a) Race walk (b) Swimming
(a) Kirit Parikh committee (c) Hockey (d) Kabaddi
(b) RM Lodha committee (e) None of these
(c) Bibek Debroy committee 35. Which is the first bank to commence the sale of Indian Gold
(d) Naresh Chandra committee Coin (IGC) in the domestic market?
(e) None of these (a) State Bank of India
28. Shuklaphanta Wildlife Reserve' is located in which country? (b) Indian Overseas Bank
(a) Nepal (b) Bhutan (c) Punjab National Bank
(d) Bank of Baroda
(c) Myanmar (d) Sri Lanka
(e) None of these
(e) None of these
36. Which organisation is providing loan support for "Madhya
29. Which of the following virus is responsible for diarrhoea
Pradesh Citizen Access to Responsive Services Project"?
among infants and young children?

ww
(a) Parvovirus
(c) Megavirus
(b) Norwalk virus
(d) Rotavirus
(a) World Bank (WB)
(b) International Monetary Fund (IMF)
(c) Asian Development Bank (ADB)
(e) None of these
w.E
30. The International Day of Remembrance of the Victims of
Slavery and the Transatlantic Slave Trade", is observed on
(d) Asian Infrastructure Investment Bank (AIIB)
(e) None of these

which date?
(a) March 23 (b) March 24
asy 37. Which is the National Intelligence Agency of Israel?
(a) GRU (b) NSIS

(c) March 25 (d) March 26


En (c) CISEN
(e) None of these
(d) Mossad

(e) None of these


31. Swayam Shikshan Prayog (SSP) is an organisation which
gin
38. Which organisation has recently released the e-Atlas of
Gender Inequality in Education?
aims to:
(a) promote empowerment of women
(a) UNESCO
(c) UNFPA
eer (b) UNICEF
(d) UNSDN
(b) protect the rights of children
(c) protect the human rights of people 39.
(e) None of these

ing
Songkran festival is celebrated in which country?
(d) take care of Mentally or Physically Challenged Persons
(e) None of these
(a) India
(c) Indonesia (d) China
.ne
(b) Thailand

32. What does BEPZA stands for?


(a) Bangladesh Export Processing Zones Authority
(b) Brazil Export Processing Zones Authority
40.
(e) None of these

t
India's first contact-less mobile payment solution "iTap" has
been launched by which bank?
(a) State Bank of India
(c) Belgium Export Processing Zones Authority
(d) Bahamas Export Processing Zones Authority (b) HDFC Bank
(c) Axis Bank Ltd
(e) None of these
(d) ICICI Bank
33. Which is the India's oldest paramilitary force?
(e) None of these
(a) Indian Coast Guard
41. Which state government has launched awareness
(b) Special Frontier Force
programme to protect sparrows?
(c) Assam Rifles
(a) Madhya Pradesh (b) Uttar Pradesh
(d) None of the above (c) Odisha (d) Punjab
(e) None of these (e) None of these

27. a b c d e 28. a b c d e 29. a b c d e 30. a b c d e 31. a b c d e


RESPONSE 32. a b c d e 33. a b c d e 34. a b c d e 35. a b c d e 36. a b c d e
GRID 37. a b c d e 38. a b c d e 39. a b c d e 40. a b c d e 41. a b c d e

Downloaded From : www.EasyEngineering.net


Downloaded From : www.EasyEngineering.net

204 SPEED TEST 96


42. What is the new name of the LED based Domestic Efficient 47. Which state has launched Mukhyamantri Santwana Harish
Lighting Programme (DELP)? Yojana?
(a) Jyoti (b) Ujala (a) Himachal Pradesh
(c) Disha (d) Prakash
(b) Tamil Nadu
43. Who is the first Indian classical musician to perform at
(c) Karnataka
United Nations?
(a) Pt. Bhimsen Joshi (b) Pt. Jasraj (d) Odisha
(c) Ustad Jakir Hussain (d) MS Subbulakshmi (e) None of these
(e) None of these 48. Which of the following banks will become the India's first
44. Which of the following two seas surround the Crimean Small Finance Bank (SFB)?
Peninsula? (a) Ujjivan Financial Services Pvt. Ltd
(a) Black Sea and Sea of Azov (b) Capital Local Area Bank Ltd
(b) Wadden Sea and Black Sea
(c) Au Financiers Ltd
(c) Caspian Sea and Sea of Azov
(d) Janalakshmi Financial Services Pvt. Ltd

45.
ww
(d) Black Sea and Mediterranean Sea
(e) None of these
Which of the following is not a correct statement about
(e) None of these
49. India's first solar powered ferry will come up in which state?
Infrasound?
w.E
(a) Infrasound has a frequency lower than 50 hertz
(a) Gujarat
(b) Kerala

whales
asy
(b) Infrasound is produced / detected by elephants and

(c) Infrasound can travel longer distances than high


(c) Goa
(d) Andhra Pradesh

frequency sound waves


(d) All are correct En (e) None of these
50. India's first self cleaning smart toilets have been installed in

46.
(e) None of these
Which bank has launched iWork@home programme for its gin which city?
(a) Chennai
women employees?
(a) Yes bank (b) Exim Bank eer
(b) Bengaluru
(c) Kolkata
(c) ICICI Bank
(e) None of these
(d) Axis Bank Ltd. (d) Mumbai a
(e) None of these ing
.ne
t

RESPONSE 42. a b c d e 43. a b c d e 44. a b c d e 45. a b c d e 46. a b c d e

GRID 47. a b c d e 48. a b c d e 49. a b c d e 50. a b c d e

Downloaded From : www.EasyEngineering.net


Downloaded From : www.EasyEngineering.net

Prelim Test - 1 97
Max. Marks : 100 No. of Qs. 100 Time : 1 hr. Date : ........./......../. ..............

Part-I : Numerical Ability 9. 78.45 + 128.85 + 1122.25 = ?


(a) 1329.55 (b) 1239.55
DIRECTIONS (Qs. 1-15): What will come in place of the question (c) 1329.45 (d) 1239.45
mark (?) in the following questions. (e) None of these
5 4 11 10. 8729 – 4376 + 1245 = ? + 2785
1. of of of 848=? (a) 2713 (b) 2823

ww
11 5
(a) 216
(c) 208
6
(b) 222
(d) 212
(c) 2833
(e) None of these
(d) 2733

2.
(e) None of these
w.E
1.4% of 750 + 2.2% of 480 = ?
(a) 21.06 (b) 21.16
11. 17
2
5
4
5
8
3
? 46
7
8
3
(c) 20.88
(e) None of these
(d) 21.18
asy (a) 32

2
5
(b) 33
5
2
3.
3
4
of 116
(a) 31
2
3
of 87=?
(b) 27 En (c) 33
5
(e) None of these
(d) 32
5

(c) 29
(e) None of these
(d) 26
gin
12. 5616 ÷ 18 ÷ 8 = ?
(a) 36 (b) 76
4. 6.96 ÷ 1.2 – 18.24 ÷ 7.6 = ?
(a) 3.4 (b) 3.14
(c) 49
eer
(e) None of these
(d) 39

(c) 3.04
(e) None of these
(d) 3.24 13. 420 ÷ 28 × 288 ÷ 32 = ?
(a) 235
ing (b) 236

.ne
5. 32.25 × 2.4 × 1.6 = ? (c) 138 (d) 132
(a) 128.84 (b) 123.84 (e) None of these
(c) 112.88 (d) 112.84 222

t
14. ? 516
(e) None of these (a) 1029 (b) 1024
6. 136% of 250 + ? % of 550 = 670 (c) 1124 (d) 1128
(a) 64 (b) 55 (e) None of these
(c) 56 (d) 65 15. 45% of 660 + 28% of 450 = ?
(e) None of these (a) 413 (b) 428
7. 448 ÷ 16 × 35 = ? (c) 423 (d) 418
(e) None of these
(a) 850 (b) 890
(c) 950 (d) 980 DIRECTIONS (Qs. 16–20) : What will come in place of the
(e) None of these question mark (?) in the following number series.
16. 12 16 24 40 ?
14 × 25 – 53 (a) 76 (b) 72
8. ?
24 5 8 9 (c) 84 (d) 88
9 64 (e) None of these
(a) 1 (b)
64 75 17. 9 19 39 79 ?
(a) 139 (b) 129
11 11
(c) 1 (d) 1 (c) 159 (d) 149
64 75
(e) None of these (e) None of these

Downloaded From : www.EasyEngineering.net


Downloaded From : www.EasyEngineering.net

206 SPEED TEST 97


18. 8 17 42 91 ? 28. Number obtained by interchanging the digit of a two digit
(a) 170 (b) 142 number is more than the original number by 27 and the sum
(c) 140 (d) 172 of the digits is 13. What is the original number?
(e) None of these (a) 58 (b) 67
19. 7 8 18 57 ? (c) 76 (d) 85
(a) 244 (b) 174
(e) None of these
(c) 186 (d) 226
(e) None of these 29. 22 men can complete a job in 16 days. In how many days will
20. 3840 960 240 60 ? 32 men complete that job?
(a) 20 (b) 18 (a) 14 (b) 12
(c) 12 (d) 22 (c) 16 (d) 9
(e) None of these (e) None of these
21. 75% of a number is equal to three seventh of another number. 30. A, B, C, D and E are five consecutive odd numbers. Average
What is the ratio between the first number and the second of A and C is 59. What is the smallest number?
number respectively? (a) 65 (b) 63
(a) 4 : 7 (b) 7 : 4 (c) 61 (d) 57
(c) 12 : 7 (d) 7 : 12
(e) None of these

ww
(e) None of these
22. A 275 metre long train crosses a platform of equal length in Direction (Qs. 31-35) : Study the following table carefully and
33 seconds. What is the speed of the train in kmph ? answer accordingly :

w.E
(a) 66 (b) 60
(c) 64 (d) 72 The distribution of marks (out of 150) obtained by 180 students in
(e) None of these each of the five subjects.
23. What is compound interest accrued on an amount of ` 45,000
in two years at the rate of 9 p.c.p.a?
(a) ` 8,600
(c) ` 8,464.50
asy
(b) ` 8,565.40
(d) ` 8,540
Marks
Sub
0-29 30-59 60-89 90-119 120-150

(e) None of these


En Maths
Science
22
39
47
38
74
67
25
22
12
14

24. If the fractions


9 7 5 4
, , , and
11 9 6 5
11
13
are arranged in
gin Hindi
Englis h
19
24
59
41
47
58
36
34
19
23

eer
ascending order, which one will be the fourth?
Geography 42 32 52 41 13
9 7
(a) (b) Average of
27 45 60 31 17

ing
11 9 five s ubjects
5 4
(c) (d)
6 5 31. If for passing, the student has to obtain minimum 60% marks

(e)
11
13
pass? .ne
in the average of five subjects, how many students will

25. Srikant and Vividh started a business investing amounts of


` 1, 85,000 and ` 2, 25,000 respectively, If Vividh’s share in
the profit earned by them is ` 9,000, what is the total profit
earned by them together? 32.
(a) 108
(c) 48
(e) None of these
(b) 58
(d) 72

How many students will pass in Geography if minimum


t
(a) ` 17,400 (b) ` 16,400 passing marks is 40%?
(c) ` 16,800 (d) ` 17,800 (a) 74 (b) 106
(e) None of these (c) 96 (d) Can’t say
26. Present ages of father and son are in the ratio of 6 : 1 (e) None of these
respectively. Four years after the ratio of their ages will 33. How many students have obtained 60 or more marks in at
become 4 : 1 respectively. What is the son’s present age? least one of the five subjects?
(a) 10 years (b) 6 years
(a) 111 (b) 103
(c) 4 years (d) 8 years
(e) None of these (c) 108 (d) 106
(e) Data inadequate
2
27. 65% of a number is more than its th by 140. What is 30% 34. If the criteria for distinction is minimum 75% marks in Maths,
5 how many students will get distinction?
of that number? (a) 37 (b) 27
(a) 186 (b) 168 (c) 12 (d) Can’t say
(c) 164 (d) 182
(e) None of these
(e) None of these

Downloaded From : www.EasyEngineering.net


Downloaded From : www.EasyEngineering.net

SPEED TEST 97 207


35. The no. of students who obtained more than or equal to 43. Four of the following five are alike in a certain way and so
40% marks in Science is what per cent less than that of form a group. Which is the one that does not belong to that
those who scored less than 60% in Hindi? group ?
(a) 17.60% (b) 15.40% (a) Stem (b) Tree
(c) 19.80% (d) 24.30% (c) Root (d) Branch
(e) None of these (e) Leaf
44. If ‘Apple’ is called ‘Orange’, ‘Orange’ is called ‘Peach’,
Part-II : Reasoning Ability ‘Peach’ is called ‘Patato’, ‘Potato’ is called ‘Banana’,
36. How many meaningful three letter English words can be ‘Banana’ is called ‘Papaya’ and ‘Papaya’ is called ‘Guava’,
formed with the letters AER, using each letter only once in which of the following grows underground ?
each word ? (a) Potato (b) Guava
(a) None (b) One (c) Apple (d) Banana
(c) three (d) two (e) None of these
(e) Four 45. How many such pairs of letters are there in word ENGLISH,
37. Each vowel of the word ADJECTIVE is substituted with the each of which has as many letters between its two letters as
next letter of the English alphabetical series, and each there are between them in the English alphabets ?

ww
consonant is substituted with the letter preceding it. How (a) None (b) One
many vowels are present in the new arrangement ? (c) Two (d) Three
(a) Four (b) One (e) More than three

38.
(c) Two
(e) None of these
w.E (d) Three

In a certain code ‘na pa ka so’ means ‘birds fly very high’,


DIRECTIONS (Qs. 46-50) : In each of the questions below are
given three statements followed by two conclusions numbered I
and II. You have to take the given statements to be true even if

asy
‘ri so la pa’ means ‘birds are very beautiful’ and ‘ti me ka
bo’ means ‘the parrots could fly’. Which of the following is
the code for ‘high’in that language ?
they seem to be at variance from commonly known facts. Read
both of the conclusions and then decide which of the given
(a) na
(c) bo
(b) k a
(d) so En conclusions logically follows from the given statements
disregarding commonly known facts.

39.
(e) None of these
If the digits in the number 86435192 are arranged in gin
Read the statements and the conclusions which follow it and give
answer
ascending order, what will be the difference between the
digits which are second from the right and fourth from the
left in the new arrangement ? eer
(a) If only conclusion I is true.
(b) If only conclusion II is true.
(c) If either conclusion I or conclusion II is true.
(a) One
(c) Three
(b) Two
(d) Four ing
(d) If neither conclusion I nor conclusion II is true.
(e) If both conclusions I and II are true.

40.
(e) None
If it is possible to make only one meaningful word with the
46. Statements :
All stars are suns.
.ne
Third, Seventh, Eighth and Tenth letters of the word
COMPATIBILITY, which of the following would be the last
letter of that word ? If no such word can be made, give ‘X’
as your answer and if more than one such word can be
Some suns are planets.
All planets are satellites.
Conclusions :
I. Some satellites are stars.
t
formed, give your answer as ‘Y’. II. No star is a satellite.
(a) I (b) B 47. Statements :
(c) L (d) X All curtains are rods.
(e) Y Some rods are sheets.
41. In a certain code FINE is written HGPC. How is SLIT written Some sheets are pillows.
in that code ? Conclusions:
(a) UTGR (b) UTKR I. Some pillows are rods.
(c) TUGR (d) RUGT II. Some rods are curtains.
(e) None of these 48. Statements :
42. If in a certain language LATE is coded as 8 & 4 $ and HIRE All switches are plugs.
is coded as 7*3$ then how will HAIL be coded in the same Some plugs are bulbs.
language ? All bulbs are sockets.
(a) 7 & 8* (b) &7*8 Conclusions:
(c) 7*& 8 (d) 7&*8 I. Some sockets are plugs.
(e) None of these II. Some plugs are switches.

Downloaded From : www.EasyEngineering.net


Downloaded From : www.EasyEngineering.net

208 SPEED TEST 97


49. Statements : (iii) If no vowel is present in the group of letters, the second and
All fishes are birds. the fifth letters are to be coded as ©.
All birds are rats. 56. BARNIS
All rats are cows. (a) 9 2 * % # 4 (b) 9 2 4 # * %
Conclusions : (c) 9 2 * # % 9 (d) 4 2 * # % 4
I. All birds are cows. (e) None of these
II. All rats are fishes. 57. DMBNIA
50. Statements : (a) 6 @ 9 % # 2 (b) 2 @ 9 % # 6
Some walls are windows. (c) 2 @ 9 % # 6 (d) 2 @ 9 % # 2
Some windows are doors. (e) None of these
All doors are roofs.
58. IJBRLG
Conclusions :
(a) # 8 9 * £ $ (b) # 8 9 * £ #
I. Some doors are walls.
(c) $ 8 9 * £ # (d) $ 8 9 * £ $
II. No roof is a window.
(e) None of these
59. BKGQJN
DIRECTIONS (Qs. 51-55) : Read the following information
(a) 9 © $ 7© % (b) © 9 $ 7 % ©
carefully and answer the questions, which follow :
(c) 9 1 $ 7 8 % (d) % 1 $ 7 8 9

ww
‘A - B’ means ‘A is father of B’.
‘A + B’ means ‘A is daughter of B’.
‘A ÷ B’ means ‘A is son of B’.
60.
(e) None of these
EGAKRL

w.E
(a) # £ $ 2 1 * (b) £ $ 2 1 * 3
‘A × B’ means ‘A is wife of B’. (c) £ $ 2 1 * # (d) # £ $ 2 1 #
51. How is P related to T in the expression ‘P + S – T’ ? (e) None of these
(a) Sister (b) Wife
(c) Son
(e) None of these
(d) Daughter
asy
52. In the expression ‘P × Q – T’ how is T related to P ?
DIRECTIONS (Qs. 61-66) : Study the following information
carefully to answer these questions.
Eight persons A, B, C, D, E, F, G and H work for three different
(a) Daughter
(c) Mother
(b) Sister
(d) Can’t be determined En companies namely X, Y and Z. Not more than three persons work
for a company. There are only two ladies in the group who have
(e) None of these
53. Which of the following means T is wife of P ?
gin
different specialisations and work for different companies. Of the
group of friends, two have specialisation in each HR, Finance and

eer
(a) P × S ÷ T (b) P ÷ S × T Marketing. One member is an engineer and one is a doctor. H is
(c) P – S ÷ T (d) P + T ÷ S an HR specialist and works with a Marketing specialist B who
(e) None of these does not work for company Y. C is an engineer and his sister
54. Which of the following means P is grandson of S ?
(a) P + Q – S
(c) P ÷ Q + S
(b) P ÷ Q × S
(d) P × Q ÷ S ing
works in company Z. D is a specialist in HR working in company
X while her friend G is a finance specialist and works for company

.ne
Z. No two persons having the same specialisation work together.
(e) None of these Marketing specialist F work for company Y and his friend A who
55. In the expression ‘P + Q × T’ how is T related to P ? is a Finance expert works for company X in which only two
(a) Mother
(c) Son
(e) None of these
(b) Father
(d) Brother

DIRECTIONS (Qs. 56-60) : In each question a group of letters is


61. Which of the following combinations is correct ?
(a) C - Z - Engineer (b) E - X - Doctor
(c) H – X – HR (d) C – Y – Engineer
t
specialists work. No lady is a marketing specialist or a doctor.

given followed by four combinations of number/symbol numbered (e) None of these


(a), (b), (c) and (d). Letters are to be coded as per the scheme and 62. For which of the following companies does C work?
conditions given below. You have to find out the serial number of (a) Y (b) X
the combination, which represents the letter group. Serial (c) Z (d) Data inadequate
number of that combination is your answer. If none of the (e) None of these
combinations is correct, your answer is (e) i.e. None of these. 63. Which of the following pairs represents the two ladies in
Letters Q M S I N G D K A L P R B J E the group ?
Number/ 7 @ 4 # % $ 6 1 2 £ 5 * 9 8 3 (a) A and D (b) B and D
Symbol (c) D and G (d) Data inadequate
(e) None of these
Conditions :
64. Which of the following represents the pair working in the
(i) If the first letter is a consonant and the last a vowel, both are
same company ?
to be coded as the code of the vowel.
(a) D and C (b) A and B
(ii) If the first letter is vowel and the last a consonant, the codes
(c) A and E (d) H and F
for the first and the last are to be interchanged.
(e) None of these

Downloaded From : www.EasyEngineering.net


Downloaded From : www.EasyEngineering.net

SPEED TEST 97 209


65. Who amongst the friends is a doctor ? Part-III : English Language
(a) H (b) E
(c) C (d) Either E or C DIRECTIONS (Qs. 71-80) : Read the following passage carefully
(e) None of these and answer the questions given below it. Certain words/phrases
66. Four of the following five are alike in a certain way and so have been printed in bold to help you locate them while answering
form a group. Which is the one that does not belong to the some of the questions.
group?
(a) 56 (b) 35 Once upon a time a dishonest king had a man called the
(c) 49 (d) 42 Valuer in his court. The Valuer set the price which ought to be
(e) 51 paid for horses and elephants and the other animals. He also set
67. If it is possible to make only one meaningful English word the price on jewellery and gold, and things of that kind. This man
from the second, the fifth, the seventh and the eighth letters was honest and just, and set the proper price to be paid to the
of the word PHYSICAL, using each letter only once, second owners of the goods. The king, however, was not pleased with
letter of that word is your answer. If more than one such this Valuer, because he was honest. "If I had another sort of a
word can be formed your answer is M. If no such word can man as Valuer, I might gain more riches," he thought.
be formed your answer is N. One day the king saw a stupid, miserly peasant come into
(a) I (b) A the palace yard. The king sent for the fellow and asked him if he
(c) L (d) M would like to be the Valuer. The peasant said he would like the
(e) N

ww
DIRECTIONS (Qs. 68-70) : Study the following paragraph and
answer the questions that follow :
position. So the king had him made Valuer. He sent the honest
Valuer away from the palace.
Then the peasant began to set the prices on horses and

w.E
Culture in the literal sense of the term is the art and
manifestation of human intellectual achievement regarded
elephants, upon gold and jewels. He did not know their value, so
he would say anything he chose. As the king had made him Valuer,
the people had to sell their goods for the price he set. By and by
a horse-dealer brought five hundred horses to the court of this

asy
collectively. It refers to the cumulative deposit of knowledge,
experience, beliefs, values, attitudes, meanings and material king. The Valuer came and said they were worth a mere measure
objects and possessions acquired by a group of people in the of rice. So the king ordered the horse-dealer to be given the measure

En
course of generations through individual and group striving. The of rice, and the horses to be put in the palace stables.
culture of India refers to the religions, beliefs, customs, traditions, The horse-dealer then went to see the honest man who had
languages, ceremonies, arts, values and the way of life of the been the Valuer, and told him what had happened. "What shall I
people of India. Heritage on the other hand denotes or relates to
things of special architectural, historical or natural value that are gin
do?" asked the horse-dealer. "I think you can give a present to
the Valuer which will make him do and say what you want him to
preserved and sustained as valued objects such as historic
buildings and monuments as valued objects such as historic
buildings and monuments eer
do and say," said the man. "Go to him and give him a fine present,
then say to him: "You said the horses are worth a measure of rice
but now tell what a measure of rice is worth! Can you value that
68. Which of the following conclusion which can be drawn
from the facts stated in the above passage?
ing
standing in your place by the king?" If the says he can, go wth
him to the king, and I will be there, too."

.ne
(a) Culture is an imaginary term by the people of that The horse-dealer thought this was a good idea. So he gave
country. a fine present to the Valuer, and said what the other man had told
(b) Culture is related with our political activities. him to say., The stupid Valuer took the present, and said: "Yes, I
(c) Culture is an integrated sense of art, knowledge,
customs, beliefs, languages, values life-styles etc.
(d) Culture is mostly affected with economic activities and
thoughts.
t
can go before the king with you and tell what a measure of rice is
worth. I can value that now." Well, let us go at once," said the
horse-dealer. So they went before the king and his ministers in
the palace.
(e) None of these The horse-dealer bowed down before the king, and said: "O
69. Which statement strengthens the conclusion of the King, I have learned that a measure of rice is the value of my five
paragraph? hundred horses. But will the king be pleased to ask the Valuer
(a) Life-style, clothing and fooding is also a part of culture. what is the valuer of the measure of rice". The king, not knowing
(b) Geographical situation does not affect the culture. what had happened, asked, "How now, Valuer, what are five
(c) Scientific knowledge is also included in culture. hundred horses worth?" "A measure of rice, O King!" said he.
(d) Culture does not affect one’s life. "Very good, then! If five hundred horses are worth a measure of
(e) None of these rice, what is the measure of rice worth?" "The measure of rice is
70. Which statement weakens the conclusion of the paragraph? worth your whole city," replied the foolish fellow.
(a) Our heritage is also a part of our culture. The ministers clapped their hands, laughing, and saying,
(b) Methods of worship is an important part of culture. "What a foolish Valuer! How can such a man hold that office? We
(c) Culture is a hypothetical and imaginary term. Which used to think this great city was beyond price, but this man says
can not be determined by some points. it is worth only a measure of rice, "Then the king was ashamed,
(d) Culture has an extensive sense which is determined by and drove out the foolish fellow. "I tried to please the king by
life-style, custums, beliefs, languages, values etc. setting a low price on the horses, and now see what has happened
(e) None of these to me!" said the Valuer, as he ran away from the laughing crowd.

Downloaded From : www.EasyEngineering.net


Downloaded From : www.EasyEngineering.net

210 SPEED TEST 97


71. Who did the king appoint as the new Valuer? 79. Why did the horse-dealer go to meet the old Valuer?
(a) A minsister (b) A horse merchant (a) As the new Valuer had set a very inappropriate price
(c) Himself (d) A stingy peasant for his five hundred horses
(e) None of these (b) As his five hundred horses were stolen from him by
72. Why was the king not happy with the old Valuer? the king
(a) As the Valuer was not good at his work (c) As he was a very good friend of the old Valuer
(b) As he had dishonoured the king (d) As the king head requested him to do so
(c) As the Valuer had been dishonest with the king about (e) None of these
the prices that he set for goods 80. What advice did the old Valuer give to the horse-dealer?
(d) As the king believed that he was not earning much (a) He asked the horse-dealer to inquire with the king about
because of the Valuer's honesty the worth of a measure of rice
(e) None of these (b) He asked the horse-dealer to bribe the new Valuer and
73. Which of the following words can be used to describe the get his horses back
king? (c) He asked the horse-dealer to forget about his horses
(1) Smart (2) Dishonest and go on with his life
(3) Cheat (d) He asked the horse-dealer to publicize his plight and
(a) Only (1) (b) Only (2) thus get his horses back
(e) None of these

74. ww
(c) Only (2) and (3) (d) Only (1) and (3)
(e) All the three (1), (2) and (3)
What can possibly be the moral of the story?
DIRECTIONS (Qs. 81-85) : Rearrange the following six
sentences (A), (B), (C), (D), (E) and (F) in the proper sequence to

w.E
(a) Slow and steady wins the race
(b) Change is the only permanent thing in life
(c) An honest answer is the sign of true friendship
form a meaningful paragraph; then answer the questions given
below them.

asy
(d) Haste makes waste (A) The woodcutter thankfully broke off from work and sat down
(e) No legacy is so rich as honesty to eat the delicious meal that his wife had sent for him.
75. Why did the Ministers laugh at the new Valuer? (B) He was in a good mood that particular morning and soon

En
(a) As he had sold the king's city at a very low price
(b) As he had displayed his stupidity by quoting an
started singing as he swung his axe at the log of wood in
front of him.
(C) After he had eaten his meal and taken rest for a while the
abysmally low price on the king's city
(c) As he had cheated the horse dealer
(d) As he had not claculated the price of the five hundred ginwoodcutter got back to work.
(D) The hours passed and the sun became hotter than ever and
horses correctly
(e) None of these eer
very soon perspiraton started breaking out on the
woodcuter's hands and face.
(E) One hot summer's morning a woodcutter was hard at work,
76. What did the new Valuer do when he got the present from
the horse dealer?
(a) He accepted the present and resigned from his post as ing
chopping wood into small pieces, so that he could sell them
in the market.

.ne
(F) As it neared afternoon, his wife sent their little son to him
was requested by the horse-dealer with food for the afternoon.
(b) He accepted the present and agreed to state the worth

t
of a measure of rice in the presence of the King 81. Which of the following should be the FOURTH sentence
after rearrangement?
(c) He accepted the present and immediately returned the
(a) F (b) E
horse-dealer's horses
(c) D (d) C
(d) He refused to accept the present from the horse-dealer
(e) B
and asked him to leave the premises
82. Which of the following should be the FIRSTsentence after
(e) None of these rearrangement?
77. Which of the following can be said about the old Valuer? (a) A (b) B
(1) He was honest. (2) He was intelligent. (c) C (d) D
(3) He was revengeful. (e) E
(a) Only (1) (b) Only (3) 83. Which of the following should be the SECOND sentence
(c) Only (1) and (2) (d) Only (1) and (3) after rearrangement?
(e) All the three (1), (2) and (3) (a) A (b) B
78. What was the worth of a measure of rice according to the (c) C (d) D
new Valuer? (e) F
(a) The king's entire city 84. Which of the following should be the LAST (SIXTH)
(b) The king's life sentence after rearrangement?
(c) Two horses (a) A (b) B
(d) Not mentioned in the passage (c) C (d) D
(e) None of these (e) E

Downloaded From : www.EasyEngineering.net


Downloaded From : www.EasyEngineering.net

SPEED TEST 97 211


85. Which of the following should be the THIRD sentence after he was amused to see that the monkeys also did (93) what he did.
rearrangement? The gardener was happy that he had so much unpaid help.
(a) A (b) B One day the gardener wanted to (94) a fair in the city. He
(c) C (d) D had an idea. He called the chief of the monkeys and said to him, "I
(e) E have to go out for the day. Can you and your family water my
DIRECTIONS (Qs. 86-88) : Which of the phrases (a), (b), (c) and plants like you (95) do? I promise you that if you help me I will
(d) given below each sentence should replace the phrase printed (96) sweets for you from the fair.: The monkeys agreed. But after
in bold in the sentence to make it grammatically correct? If the the gardener had left, they had a (97). How much water were they
sentence is correct as it is given and no correction is required, to pour for each plant? then one of them said. "For plants with big
mark (e) as the answer. roots, we must pour (98) of water and for the ones with small
roots, we pour only a litle water." So, the monkeys (99) out each
86. As it was already afternoon, Rohan decided to check out of
plant and then pushed it back again after looking at the root. As
the hotel and go home.
a result, many plants (100) and died. On his return, the gardener
(a) for checking out (b) to checking out
realized that he had been very foolish to trust a bunch of mere
(c) to check outing (d) to checked out
monkeys to do his job.
(e) No correction required
87. Five people which ignored an evacuation order were trapped 91. (a) took (b) was
in a mountain region encircled by a wildfire. (c) great (d) handle

ww
(a) who ignored an
(c) who ignores a
(e) No correction required
(b) those ignoring an
(d) that ignored a
(e) mended
92. (a) try (b) told
88.
w.E
Since she was the most popular model on the ramp, she
thought no end to herself.
(a) no ending to herself (b) no ends of herself
(c) were
(e) learnt
93. (a) main
(d) bent

(b) exactly
(c) no end of herself
(e) No correction required
asy
(d) no end with herself (c) many
(e) too
(d) because

En
DIRECTIONS (Qs. 89-90) : In each question below, a sentence 94. (a) call (b) make
with four words printed in bold type is given. These are numbered
as (a), (b), (c) and (d). One of these four words printed in bold may (c) stall (d) go
be either wrongly spelt or inappropriate in the context of the
sentence. Find out the word which is wrongly spelt or gin (e) visit
95. (a) forcefully (b) hardly
inappropriate, if any. The number of that word is your answer. If
all the words printed in bold are correctly spelt and also
appropriate in the context of the sentence, mark (e) i.e., All
(e) truly
eer
(c) usually (d) costly

correct' as your answer.


89. The city's fashion-conscious ladies (a)/ came together at a
96. (a) ask
(c) got
ing (b) bring
(d) throw
city hotel to check out an exibition (b)/ by various (c)/
designers (d)/ and labels. All correct (e)
(e) create
97. (a) party (b) time
.ne
90. The ministry's proposal (a)/ for an autonomous (b)/
overarching authority (c)/ for higher education and research
was finally approval. (d)/ All correct (e)
DIRECTIONS (Qs. 91-100) : In the following passage there are
(c) answer
(e) water
98. (a) body
(c) lots
(d) doubt

(b) many
(d) weight
t
blanks, each of which has been numbered. These numbers are
(e) quantity
printed below the passage and against each, five words are
suggested, one of which fits the blank appropriately. Find out the 99. (a) thrashed (b) saw
appropriate word in each case. (c) stick (d) pulled
There was once a gardener who (91) care of the king's (e) splashed
garden. In the garden, lived a family of monkeys. Seeing the 100. (a) withered (b) crushed
gardener at work, the monkeys soon (92) to imitate him. As the (c) killed (d) grew
gardener tended the plants and weeded and watered the garden, (e) smiled

Downloaded From : www.EasyEngineering.net


Downloaded From : www.EasyEngineering.net

212 SPEED TEST 97

RESPONSE SHEET
Test Code : . ............................. Time taken : ......................... Date : ..........................
(a) (b) (c) (d) (e) (a) (b) (c) (d) (e) (a) (b) (c) (d) (e) (a) (b) (c) (d) (e)
1. 26. 51. 76.
2. 27. 52. 77.
3. 28. 53. 78.
4. 29. 54. 79.
5. 30. 55. 80.
6. 31. 56. 81.
7.
8. ww 32.
33.
57.
58.
82.
83.
9.
10. w.E 34.
35.
59.
60.
84.
85.
11.
12.
36.
37. asy 61.
62.
86.
87.
13. 38.
En 63. 88.
14.
15.
39.
40. gin
64.
65.
89.
90.
16.
17.
41.
42.
66.
67.
eer 91.
92.
18. 43. 68.
ing 93.
19.
20.
44.
45.
69.
70.
94.
95. .ne
21.
22.
46.
47.
71.
72.
96.
97.
t
23. 48. 73. 98.
24. 49. 74. 99.
25. 50. 75. 100.

Downloaded From : www.EasyEngineering.net


Downloaded From : www.EasyEngineering.net
98
Prelim Test - 2 98
Max. Marks : 100 No. of Qs. 100 Time : 1 hr. Date : ........./......../. ..............

Part-I : Numerical Ability 7 3 1


9. 3 7 1 ?
11 11 2
DIRECTIONS (Qs. 1-10): What will come in place of question
10 6
mark (?) in the following questions? (a) 13 (b) 14
11 11
1.
3
5
4
of of
7
(a) 220
ww5
12
of 1015 = ?

(b) 340
(c) 14
9
11
(d) 10
17
22

w.E
(e) None of these
(c) 240 (d) 145 10. 1080 12 10 = ?
(e) None of these (a) 900 (b) 90

asy
2. 1.5 × 0.025 + (?)2 = 0.1 (c) 120 (d) 12
(a) 0.28 (b) 0.27 (e) None of these
(c) 0.25 (d) 0.235 11. The number zero (0) is surrounded by the same 2-digit
(e) None of these
En number on both (left and right) the sides; for example, 25025,
67067, etc. The largest number that always divides such a
3. 1.52 0.0225
(a) 0.3375
?
(b) 3.275
gin number is
(a) 7 (b) 11
(c) 32.75
(e) None of these
(d) 0.0375 (c) 13

eer
(e) None of these
(d) 1001

4. 0.0289 12 1.5 ?
(a) 1.36 (b) 2.06
12.
(a) 181
(c) 179 ing
The number of 3-digit number exactly divisible by 5 is
(b) 180
(d) 199
(c) 13.90
(e) None of these
(d) 14.80

13.
(e) None of these
.ne
If a certain sum of money becomes double at simple interest
5. 125% of 260 + ?% of 700 = 500
(a) 32
(c) 23
(e) None of these
(b)
(d)
56
46 (a) 8 1
3
(b) 10
t
in 12 years, what would be the rate of interest per annum ?

6. 45% of 750 – 25% of 480 = ? (c) 12 (d) 14


(a) 216 (b) 217.50 (e) None of these
(c) 245 (d) 236.50 14. Three successive discounts of 10%, 12% and 15% amount
(e) None of these to a single discount of
(a) 36.28 % (b) 34.68%
7. 758.5 753.8 75 ? (c) 37 % (d) 32.68%
(a) 4.9 (b) 3.6 (e) None of these
(c) 3.3 (d) 4.7 15. The ratio of the prices of two houses A and B was 4 : 5 last
(e) None of these year. This year, the price of A is increased by 25% and that
of B by ` 50000. If their prices are now in the ratio 9 : 10, the
8. 39798 + 3798 + 378 = ?
price of A last year was
(a) 49532 (b) 43984 (a) ` 3,60,000 (b) ` 4,50,000
(c) 43974 (d) 43576
(c) ` 4,80,000 (d) ` 5,00,000
(e) None of these (e) None of these

Downloaded From : www.EasyEngineering.net


Downloaded From : www.EasyEngineering.net

214 SPEED TEST 98


16. During a journey of 80 km a train covers first 60km with a 23. 18.4% of 656 + 12.7% of 864 = ?
speed of 40 km/h and completes the remaining distance with (a) 253 (b) 231 (c) 211
a speed of 20 km/h. What is the average speed of the train (d) 241 (e) None of these
24. (98.4)2 + (33.6)2 = ?
during the whole journey?
(a) 10812 (b) 18012 (c) 10910
(a) 30 km/h (b) 32 km/h
(d) 18102 (e) None of these
(c) 36 km/h (d) 40 km/h 25. 8959 ? 4 5 = 26.35
(e) None of these (a) 15 (b) 25 (c) 30
17. An aeroplane takes off 30 minutes later than the scheduled (d) 17 (e) None of these
time and in order to reach its destination 1500 km away in 26. 3739 + 164 × 27 = ?
time, it has to increase its speed by 250 km/h from its usual (a) 10540 (b) 4000 (c) 8400
(d) 8200 (e) None of these
speed. Find its usual speed.
27. Rajeev consistently runs 415 meters every day except on
(a) 1000 km/h (b) 750 km/h
Sunday when he runs 500 meters. How many kilometers will
(c) 850 km/h (d) 650 km/h

18. ww
(e) None of these
A man arranges to pay off a debt of Rs 3,600 in 40 annual
he run in two weeks-? (in this question week starts from
Monday)
(a) 5.98 kms. (b) 5.86 kms. (c) 5.96 kms.

w.E
instalments which form an AP. When 30 of the instalments
are paid, he dies leaving one-third of the debt unpaid. Find 28.
(d) 5.88 knis. (e) None of these
Amit got 44 marks in Hindi. 55 marks in Science. 77 marks in
the value of the first instalment.
(a) 55 (b) 53
asy Maths. 79 marks in Social Science and 76 marks in English.
The maximum marks of each subject are 100. How much
overall percentage of marks did he get?
(c) 51
(e) None of these
(d) 49

En (a) 66.2 (b) 64.2 (c) 72.2


19. A tank 30 m long, 20 m wide and 12 m deep is dug in a field
500 m long and 30 m wide. By how much will the level of the gin (d) 74.2 (e) None of these

DIRECTIONS (Qs. 29 - 30) : What will come in place of question


field rise, if the earth dug out of the tank is evenly spread
over the field? 29. eer
mark (?) in the following number series?
7 11 19 35 67 (?)
(a) 0.33 m
(c) 0.25 m
(b) 0.5 m
(d) 0.4 m
(a) 121
(d) 133 ing
(b) 153
(e) None of these
(c) 141

(e) None of these


DIRECTIONS (Qs. 20-21) : Find the next term in the given series
30. 5 6 10 19 35 (?)
(a) 55 (b) 65 .ne
(c) 60
in each of the questions below.
20. 198, 194, 185, 169, ....
(a) 136 (b) 144
(d) 70 (e) None of these

Directions (Qs. 31-35) : Study the following data carefully and


answer accordingly.
t
(c) 9 (d) 92
Following chart shows the number of students in
(e) None of these
different universities
21. 6, 9, 7, 10, 8, 11, ....
(a) 12 (b) 13 Delhi Uni
Indraprastha 35%
(c) 9 (d) 14
Uni 13%
(e) None of these
DIRECTIONS (Qs. 22-28): Find out the approximate value which
should replace the question mark (?) in the following questions. Hamdard
14%
(You are not expected to find out the exact value).
J.N.U
22. 10609 7938.81 ? 20%
Jamia
(a) 9200 (b) 81973. 18%
(c) 8553.3 (d) 8682.7
(e) None of these Total no. of students = 120,000

Downloaded From : www.EasyEngineering.net


Downloaded From : www.EasyEngineering.net

SPEED TEST 98 215


Percentage of listeners of different FM channels in 38. If each of the digits in the number 92581473 are arranged in
National Capital Region ascending order, what will be the difference between the
digits which are fourth from the right and third from the left
FM Chann els in the new arrangement ?
Un ivers ities Rad io Radio Red FM (a) One (b) Two
Rain bo w (c) Three (d) Four
M irch i City FM Go ld
Ind rap ras tha 76% 72% 46% 54% 48% (e) None
39. In a certain code ‘ja ki mo pe’ means ‘at a frog’s leap’, ‘mo la
Hamd ard 63% 64% 59% 47% 53%
ki so’ means ‘take a leap ahead’ and ‘re bo ja na’ means
JNU 52% 65% 64% 51% 54% ‘insects are frog’s diet’. Which of the following is the code
DU 82% 44% 32% 35% 45% for ‘at’ in that language ?
Jamia 75% 32% 36% 52% 64% (a) ja (b) pe
(c) bo (d) re
31. How many students of JNU listen to Radio city? (e) None of these
(a) 15200 (b) 15600 40. If in a certain language WEAK is coded as 9%2$ and SKIT
(c) 14400 (d) 14600 is coded as #$7@, then how will WAIT be coded in the
same language ?
(e) None of these
(a) 9267 (b) 9276

ww
32. The no. of Indraprastha students listening to Rainbow is
(c) 92 @ 6 (d) 9 @ 67
what per cent of the no. of Jamia students listening FM (e) None of these
Gold? 41. How many such pairs of letters are there in word SENDING,
(a) 65
(c) 68
(e) None of these w.E (b) 56
(d) 58
each of which has as many letters between its two letters as
there are between them in the English alphabets ?
(a) None (b) One

asy
33. From which of the following universities, the no. of students (c) Two (d) Three
liking Red FM is minimum? (e) More than three
(a) Indraprastha (b) Jamia 42. Each vowel of the word GLADIOLUS is substituted with
(c) JNU
(e) Hamdard
(d) DU
En the next letter of the English alphabetical series, and each
consonant is substituted with the letter preceding it. How

gin
34. How many students of Indraprastha and Jamia together many vowels are present in the new arrangement ?
(a) Four (b) One
listen to Red FM?
(c) Two (d) Three
(a) 12562 (b) 12872
(c) 14952
(e) None of these
(d) 14272
eer
(e) None of these

DIRECTIONS (Qs. 43-47) : In each of these questions a group of


35. Which of the following channels is the most popular among
the students of Hamdard and JNU?
(a) Radio Mirchi (b) Radio city ing
letters is given followed by four combinations of number/symbol
numbered (a), (b), (c) and (d). Letters are to be coded as per the

.ne
scheme and conditions given below. You have to find out the serial
(c) Red FM (d) FM Gold number of the combination, which represents the letter group.
(e) Rainbow Serial number of the combination is your answer. If none of the

36.
Part-II : Reasoning Ability
If ‘football’ is called cricket, ‘cricket’ is called ‘basketball’
‘basketball’ is called ‘badminton’, ‘badminton’ is called
Letters M B D K Q L I R J S N P A E G
Number/
Symbol
@ 3 7 % * 4 # 1 2 £ 8 5 9 $ 6
t
combinations is correct, your answer is (e) i.e. ‘None of these’.

‘volleyball’, ‘volleyball’ is called ‘hockey’ and ‘hockey’ is


called ‘golf’, which of the following games is not played Conditions :
using a ball ? (i) If the first letter is a vowel and the last a consonant, both are
(a) Volleyball (b) Basketball to be coded as the code for the consonant.
(ii) If the first letter is a consonant and the last a vowel, the
(c) Hockey (d) Cricket
codes for the first and the last are to be interchanged.
(e) None of these
(iii) If no vowel is present in the group of letters, the first and
37. If it is possible to make only one meaningful word with the
the last letters are to be coded as ©.
First, Second, Third and Fifth letters of the word 43. GQRDBN
TECHNOLOGY, which of the following would be the third (a) ©* 173© (b) 6*1738
letter of that word ? If no such word can be made, give ‘X’ as (c) 6*1736 (d) 8*1738
your answer and if more than one such word can be formed, (e) None of these
give your answer as ‘Y’. 44. IPEBQS
(a) C (b) T (a) #5$3*£ (b) #53$*#
(c) N (d) X (c) £5$3*£ (d) £5$3*#
(e) Y (e) None of these

Downloaded From : www.EasyEngineering.net


Downloaded From : www.EasyEngineering.net

216 SPEED TEST 98


45. RMAPSI 55. If in each number, all the three digits are arranged in
(a) 1@95£# (b) 1@95£1 descending order, which of the following will be the third
(c) #@95£# (d) #@95£1 highest number?
(e) None of these (a) 972 (b) 682
46. AREMQN (c) 189 (d) 298
(a) 91$@*8 (b) 81$@*8 (e) 751
(c) 81$@*9 (d) 91$@*9
(e) None of these DIRECTIONS (Qs. 56-60) : In each of the questions below are
47. KJBPRD given three statements followed by two conclusions numbered I
(a) 923517 (b) 723517 and II. You have to take the given statements to be true even if
(c) %23519 (d) ©2915© they seem to be at variance from commonly known facts. Read
(e) None of these both of the conclusions and then decide which of the given
conclusions logically follows from the given statements
DIRECTIONS (Qs. 48-52) : Read the following information
disregarding commonly known facts.
carefully and answer the questions, which follow :
‘A – B’ means ‘A is daughter of B,. Read the statements and conclusions which follow it and
‘A + B’ means ‘A is wife of B’ Give answer (a) if only conclusion I is true.
‘A ÷ B’ means ‘A is father of B’ Give answer (b) if only conclusion II is true.

ww
‘A × B’ means ‘A is son of B’.
48. In the expression ‘P × R – S’ how is P related to S ?
Give answer (c) if either conclusion I or conclusion II is true.
Give answer (d) if neither conclusion I nor conclusion II is true.

w.E
(a) Father (b) Grandfather Give answer (e) if both conclusions I and II are true.
(c) Grandson (d) Sister 56. Statements :
(e) None of these No pen is a mobile.

asy
49. Which of the following means S is son-in-law of P? Some mobiles are bottles.
(a) P + R × S (b) P ÷ R × S All bottles are papers.
(c) P + R ÷ S (d) P ÷ R + S Conclusions :
(e) None of these
50. In the expression ‘P – Q + S’ how is S related to P?
En I. Some papers are pens.
II. All bottles are pens.
(a) Mother
(b) Father
(c) Brother gin
57. Statements :
All computers are radios.
All radios are televisions.
(d) Cannot be determined
(e) None of these eer
Some televisions are watches.
Conclusions :
51. How is P related to S in the expression ‘P × R ÷ S’?
(a) Brother
(c) Son
(b) Wife
(d) Sister ing
I. Some watches are computers.
II Some televisions are computers.

.ne
58. Statements :
(e) None of these Some desks are chairs.
52. How is S related to P in the expression ‘P + R ÷ S’? Some chairs are doors.
(a) Son
(c) Daughter- in - law
(e) None of these
(b) Daughter
(d) Sister

DIRECTIONS (Qs. 53-55) : Study the sets of numbers given


Some doors are walls.
Conclusions :
I. Some walls are chairs.
II. No chair is a wall.
t
below and answer the questions, which follow : 59. Statements :
972 682 189 298 751 All stars are fishes.
53. If one is added to the lowest number and two is added to Some fishes are moons.
the highest number, what will be the difference between the All moons are birds.
second digit of the smallest number and third digit of the Conclusions :
highest number ? I. Some birds are fishes.
(a) 5 (b) 7 II. Some stars are moons.
(c) 9 (d) 8 60. Statements :
(e) None of these All leaves are roots.
54. If in each number, first and the last digits are interchanged, All stems are roots.
which of the following will be the third highest number ? All roots are flowers.
(a) 972 (b) 682 Conclusions :
(c) 189 (d) 298 1. Some flowers are stems.
(e) 751 II. Some flowers are leaves.

Downloaded From : www.EasyEngineering.net


Downloaded From : www.EasyEngineering.net

SPEED TEST 98 217


DIRECTIONS (Qs. 61-67) : Study the following information 68. The passage best support the statement that–
carefully to answer these questions. (a) History has a short and lucid curriculum to study so it
A group of people has six family members and an advocate. attracts the students.
These are L, M, N, O, P, Q and R and having different professions. (b) History has an extensive curriculum and its questions
are time taking.
Each one of them is a journalist, businessman, architect, doctor
(c) History is lucid and easy discipline. So, it does not
and pilot but not necessarily in this order. There are three males
need a comprehensive strategic to ensure a sound
and three females in the family out of which there are two married
preparation.
couples. M is a businessman and is the father of P. N is a housewife
(d) History is an important subject in the examination point
and is daughter-in-law of O. L is neither a pilot nor a journalist. R
of view.
is an advocate. N is not the mother of P and O is not married to M. (e) None of these
No lady is a journalist. 69. Which of the following assumption which can be implicit in
61. Which of the following groups represents the three ladies the facts stated in the above passage?
in the group ? (a) History is an important subject for the competition
(a) N, P, L (b) P, L, N points of view.
(c) L, N, O (d) O, P, L (b) Nature of this subject is static.
(e) None of these (c) A comprehensive strategy is needed to ensure a sound
62. Who is married to Q ?

ww
preparation of history.
(a) N (b) O (d) History attracts the student with its easy nature.
(c) L (d) Can’t be determined (e) None of these

w.E
(e) None of these 70. Which of the following is a conclusion which can be drawn
63. Who among the following family members is an architect ? from the facts stated in the above passage.
(a) L (b) O (a) History is an easy subject nevertheless students do
(c) P (d) Can’t be determined not like to opt it as an optional paper.
(e) None of these
asy
64. Which of the following is the profession of P ?
(b) History is a boring and wearisome subject.
(c) There is not enough study material for the sound
(a) Architect
(c) Architect or pilot
(b)
(d)
Pilot
Journalist
En preparation of history.
(d) Questions are not being asked from history in the
competitive exams.

gin
(e) None of these
65. How is Q related to O ? (e) None of these
(a) Father (b) Mother
(c) Mother-in- law
(e) None of these
(d) Son - in - law

eer
Part-III : English Language
DIRECTIONS (Qs. 71-80) : Read the following passage carefully

ing
66. If in the number 5608391467, the position of the first and the
sixth digits are interchanged, the second and seventh digits and answer the questions given below it. Certain words have
are interchanged and so on upto the fifth and the tenth been printed in bold to help you locate them while answering
digits. Then which will be the fourth digit from the left end
after interchange?
some of the questions.

.ne
Govind’s father was a rich landlord, who was loved and

t
(a) 1 (b) 6 respected by all his tenants. When he died, he left large tracts of
(c) 8 (d) 3 land to Govind. But Govind did not spend a single day looking
(e) None of these after his land. He had a funny idea, that there existed a magic
67. What will come next in the following series? potion which, if it was poured on any object, would turn it into
ababcabcdabcdeabcdefabcd gold. He spent all his time trying to learn more about this potion.
(a) g (b) f People took advantage of him and cheated him. His wife grew
(c) e (d) a anxious. Given the amount of money Govind was spending, she
(e) None of these was sure that they would soon be paupers.
One day, a widely respected sage who had been to the
DIRECTIONS (Qs. 68-70) : Study the following paragraph and
Himalayas came to their town. Govind asked him about the potion.
answer the questions that follow :
To his surprise the sage answered, “I have learnt how to brew
History as a discipline has always attracted the imagination such a potion. But it is a difficult process.” “Tell me!” insisted
of the students because of its simple, lucid and easy to Govind, hardly able to believe his luck. “You have to collect the
comprehend nature. At the same time it is vast and time consuming dew which settles on the leaves of a banana tree every morning
therefore it becomes imperative to plan a comprehensive strategy during winter. There is a condition, though. The tree should be
to ensure a sound preparation. The nature of the subject is itself planted and watered regularly with your own hands. Store the
static however the questions asked over the last few years compels collected dew in an earthen vessel and when you have five litres,
us to understand the dynamics of the discipline as well as the bring it to me. I will recite a sacred mantra to transform the dew
forces, trends and patterns including institutions that have shaped into the potion. A drop of the potion will be sufficient to change
history at any given point of time. any object into gold.”

Downloaded From : www.EasyEngineering.net


Downloaded From : www.EasyEngineering.net

218 SPEED TEST 98


Govind was worried “Winter is only for a few months in the 75. Why did Govind decide to cultivate a banana crop?
year. It will take me years to collect the dew.” “You can plant as (a) The soil of his land was suitable only for cultivating bananas
many trees as you want,” replied the sage. Govind went home (b) It was the most highly priced commodity in the region
and after talking to his wife, began clearing the large fields which (c) It could be grown at any time of the year including winter
(d) His wife pressurised him to do so
has been lying vacant for years. He planted rows of banana
(e) The ingredient for the magic potion could only be
saplings. He tended them with great care. His wife helped him too. obtained from a banana tree
She would take the banana crop to market and get a good price. 76. What made Govind angry with the sage?
Over the years the plantation grew and finally after six years (a) The sage had conspired with Govind’s wife against
Govind had five litres of dew. He went to the sage who smiled, him
uttered a mantra and sprinkled a few drops of dew on a copper (b) He had forgotten the magic spell and all Govind’s hard
vessel. To Govind’s dismay, nothing happened. “you have cheated work was in vain
me!” he shouted at the sage. (c) He had lost a good deal of money in cultivating
The sage however smiled. Govind’s wife then came forward bananas
with a box. The sage opened it and revealed stacks of gold coins (d) The sage had made a fool of him in front of other
inside. Turning to Govind he said, “you worked hard on your villagers
land and created a plantation. Your wife sold the produce in the (e) None of these
market. It was your hard work which created this wealth, not magic.

ww
If I had told you this earlier, you would not have listened.” Govind
understood the wisdom behind the sage’s words and worked
DIRECTIONS (Qs. 77-78) : Choose the word which is most similar
in meaning to the word printed in bold as used in the passage.
77. SPEND

w.E
even harder from that day on.
71. Why did Govind’s father give him large tracts of land? (a) pay (b) bought
(a) It was his way of instilling a sense of responsibility in (c) devote (d) settle
(e) empty
his son

asy
(b) Govind was his only son and sole heir
(c) To provide Govind with sufficient funds to pursue his
78. LYING
(a) sleeping (b) dishonest
interest of discovering a magic potion
En
(d) He wanted Govind to continue to look after the tenants
(c) relaxing
(e) untruthful
(d) remaining

(e) None of these


72. Which of the following can be said about the sage? gin
DIRECTIONS (Qs. 79-80) : Choose the word which is most
opposite in meaning to the word printed in bold as used in the
(a) He was cunning and plotted with Govind’s wife to cheat
him.
passage.
79. DISMAY
eer
ing
(b) He had no magical powers as such and used to swindle (a) joy (b) interest
people (c) desire (d) humour
(c) He was a good judge of people (e) luck
(d) He did not deserve his good reputation
(e) He was dishonest because he had cheated Govind out
80. TENDED
(a) negligible (b) watched
.ne
of his gold
73. Why was Govind’s wife worried ?
(a) Govind had no knowledge of farming and could not
cultivate the land he had inherited from his father
(c) inclined
(e) spoil
(d) ignored

DIRECTIONS (Qs. 81-85) : Rearrange the following six


t
sentences (A), (B), (C), (D), (E) and (F) in the proper sequence
(b) Govind had not friends because he was obsessed with
to form a meaningful paragraph; then answer the questions given
finding a potion which would turn any thing into gold
below them.
(c) Govind was only interested in studying under different
sages and neglected his family duties A. The hall was filled with children, teachers, students, family
(d) Since Govind had devoted all his time and wealth to members and those who were close to him.
finding a magic potion, they would soon be poor B. Normally such ceremonies are attended by important people
(e) Govind’s experiments to find a magic potion were like industrialists, politicians and VIP’s.
dangerous C. What I saw when I stepped into the hall amazed me.
74. Why did Govind’s wife help him in the fields? D. I went home with the feeling that it was a most unusual oath
A. To support her husband in his endeavour to find a taking ceremony with only those who were ‘important’ to
magic potion.
him present.
B. The sage had advised her to help her husband succeed.
E. When he was elected President, he invited me to the swearing
C. He needed someone to help him collect the dew.
(a) Only (B) (b) Only (A) in ceremony in the Central Hall of Parliament.
(c) Both (A) and (B) (d) All (A), (B) and (C) F. However in this case everyone who attended the ceremony
(e) None of these seemed to know him personally.

Downloaded From : www.EasyEngineering.net


Downloaded From : www.EasyEngineering.net

SPEED TEST 98 219


81. Which of the following should be the FIRST sentence after mistakes. In Bangladesh, conventional banks and credit co-
rearrangement ? operatives always 93 lump sum repayments. This created 94
(a) A (b) B problems because repaying in a lump sum was a mental hurdle for
(c) C (d) D borrowers. They tended to delay repayment and get further into
(e) E debt in the 95. In the end they usually 96 totally on the loan,
82. Which of the following should be the SECOND sentence which was a loss to the bank. In structuring our own loans, I
after rearrangement ? decided to ask for a daily payment, Monitoring repayment was 97
(a) B (b) C and it filled people with 98 that they could repay their loans.
(c) D (d) E 91. (a) firstly (b) freshly
(e) F (c) foremost (d) initially
83. Which of the following should be the THIRD sentence after (e) recently
rearrangement ?
92. (a) copied (b) observed
(a) A (b) B
(c) C (d) D (c) learned (d) understood
(e) E (e) improving
84. Which of the following should be the FIFTH sentence after 93. (a) asked (b) insisted
rearrangement ? (c) demanded (d) settled
(a) B (b) C (e) lend

85.
(c) D
(e) F
ww (d) E

Which of the following should be the LAST (SIXTH)


94. (a) severe
(c) additionally
(b) no
(d) variety

(a) A
(c) C w.E
sentence after rearrangement ?
(b) B
(d) D
(e) plenty
95. (a) time
(c) return
(b) process
(d) event
(e) E

asy
DIRECTIONS (Qs. 86-90) : Read each sentence to find out
whether there is any grammatical error or idiomatic error in it.
(e) action
96. (a) neglected
(c) defaulted
(b) abandoned
(d) depended

En
The error, if any, will be in one part of the sentence. The number
of that part is the answer. If there is no error, the answer is (e).
(e) disappointed
97. (a) benefit (b) easier
(Ignore errors of punctuation, if any.)
86. His proposal had (a) / to be send to (b) / the President of the
company (c) / for her approval (d). No error (e). gin (c) reckless
(e) secure
(d) disorganised

87. Each tuesday evening we visited (a) / the farmers in the


area (b) / and held a meeting (c) / to discuss the problems
98. (a) sense

eer
(c) challenge
(e) believe
(b) confidence
(d) doubt
they faced (c). No error (e).
88. Though our training facilities (a) / are limited only a (b) / few
employees have been (c) / selected for training (d). No error ing
DIRECTIONS (Qs. 99 to 100): In each of the following sentences,
an idiomatic expression or a proverb is highlighted. Select the
(e).
89. During the interview (a) / the panel asked me (b) / several
technical questions (c) / and I answered all of it (d). No error .ne
alternative which best describes its use in the sentence.
99. Having sold off his factory, he is now a gentleman at large.
(e).
90. He decided to work for (a) / an NGO, but most of his (b) /
classmates opted for high paid (c) / jobs in multinational
companies (d). No error (e).
(a) Has no serious occupation
(b) Is living comfortably
(c) Is respected by everybody
(d) Is held in high esteem
t
DIRECTIONS (Qs. 91-98) : In the following passage there are (e) None of these
blanks each of which has been numbered. These numbers are 100. Though he has lot of money, yet all his plans are built upon
printed below the passage and against each, five words are sand.
suggested, one of which fits the blank appropriately. Find out the (a) established on insecure foundations
approptiate words in each case. (b) based on inexperience
When we 91 started thirty years ago in 1977, we did not (c) resting on cheap material
know anything about how to run a bank for the poor. We therefore (d) resting on immature ideas
looked at how others ran their operations and 92 from their (e) None of these

Downloaded From : www.EasyEngineering.net


Downloaded From : www.EasyEngineering.net

220 SPEED TEST 98

RESPONSE SHEET
Test Code : . ............................. Time taken : ......................... Date : ..........................
(a) (b) (c) (d) (e) (a) (b) (c) (d) (e) (a) (b) (c) (d) (e) (a) (b) (c) (d) (e)
1. 26. 51. 76.
2. 27. 52. 77.
3. 28. 53. 78.
4. 29. 54. 79.
5. 30. 55. 80.
6. 31. 56. 81.
7.
8. ww 32.
33.
57.
58.
82.
83.
9.
10. w.E 34.
35.
59.
60.
84.
85.
11.
12.
36.
37. asy 61.
62.
86.
87.
13. 38.
En 63. 88.
14.
15.
39.
40. gin
64.
65.
89.
90.
16.
17.
41.
42.
66.
67.
eer 91.
92.
18. 43. 68.
ing 93.
19.
20.
44.
45.
69.
70.
94.
95. .ne
21.
22.
46.
47.
71.
72.
96.
97.
t
23. 48. 73. 98.
24. 49. 74. 99.
25. 50. 75. 100.

Downloaded From : www.EasyEngineering.net


Downloaded From : www.EasyEngineering.net

Prelim Test - 3 99
Max. Marks : 100 No. of Qs. 100 Time : 1 hr. Date : ........./......../. ..............

Part-I : Numerical Ability 8. 3889 + 12.952 – ? = 3854.002


(a) 47.95 (b) 47.752
DIRECTIONS (Q. 1-10) : What should come in place of the (c) 47.095 (d) 47.932
question mark (?) in the following questions? (e) None of these
1. ? 75 9. ? + 72.64 = 74.64
(a) 145.28 (b) –2.00

ww
(a) –5625 (b) 75
(c) 1500 (d) Cannot be determined (c) –145.28 (d) 147.28
(e) None of these
(e) None of these
10. 6.25 0.0025 = ?
2.
21 7 1
8 72 171
?
w.E (a) 1800
(c) 1700
(e) None of these
(b) 2300
(d) 2500

(a)
9
19
(b)
1
3
asy 11. Which is the smallest of the following numbers ?

(a) (b)
1

En
5 3 7
7
(c) (d)
19 19
(e) None of these
1 2 1 gin (c)
7
7
(d)
1
7

eer
3. 4 6 5 ? (e) None of these
2 3 3 12. Two equal sums were borrowed at 8% simple interest per
2 annum for 2 years and 3 years, respectively. The difference
1
(a) 15
2
(b) 16
3
ing
in the interests was `56. The difference in the interests was
`56. The sum borrowed were
(a) `690 (b) `700

.ne
1
(c) 16 (d) 17 (c) `740 (d) `780
2
(e) None of these

t
(e) None of these
13. A machine is sold at a profit of 10%. Had it been sold for ` 80
4. 792.02 + 101.32 - 306.76 = ?
less, there would have been a loss of 10%. The cost price of
(a) 893.34 (b) 1200.10 the machine is
(c) 997.11 (d) 586.58 (a) `350 (b) `400
(e) None of these (c) `450 (d) `520
5. 300% of 150 = ?% of 600 (e) None of these
(a) 75 (b) 45 14. A jar of oil was four fifths full. When six bottles of oil were
1 taken out and four bottles of oil were poured into, it was
(c) 450 (d) 133 three fourths full. How many bottles of oil were contained
2
by the jar ?
(e) None of these (a) 10 (b) 20
6. 34.95 + 240.016 + 23.9800 = ? (c) 30 (d) 40
(a) 299.09 (b) 298.0946 (e) None of these
(c) 298.111 (d) 298.946 15. During a journey of 80 km a train covers first 60km with a
(e) None of these speed of 40 km/h and completes the remaining distance with
7. 48.95 – 32.006 = ? a speed of 20 km/h. What is the average speed of the train
(a) 16.089 (b) 16.944 during the whole journey?
(c) 16.35 (d) 16.89 (a) 30 km/h (b) 32 km/h
(e) None of these (c) 36 km/h (d) 40 km/h
(e) None of these

Downloaded From : www.EasyEngineering.net


Downloaded From : www.EasyEngineering.net

222 SPEED TEST 99


16. An aeroplane takes off 30 minutes later than the scheduled 25. 3
4096 ?
time and in order to reach its destination 1500 km away in
(a) 16 (b) 26
time, it has to increase its speed by 250 km/h from its usual
(c) 18 (d) 24
speed. Find its usual speed.
(a) 1000 km/h (b) 750 km/h (e) None of these
(c) 850 km/h (d) 650 km/h 26. 6 men can complete a piece of work in 20 days. In how many
(e) None of these days will 8 men complete the same piece of work?
17. In an examination 35% of the candidates failed in one subject (a) 12 days (b) 14 days
and 42% failed in another subject. While 15% failed in both (c) 15 days (d) 16 days
the subjects. If 2500 candidates appeared at the examination, (e) None of these
how many students passed in either subject but not in both? 27. What will come in place of both the question marks (?) in
the following equation?
(a) 325 (b) 1175
(c) 2125 (d) 1230 363 (?)
(e) None of these (?) 3
18. If the length of a certain rectangle is decreased by 4 cm and (a) 43 (b) 33
the width is increased by 3 cm, a square with the same area (c) 37 (d) 47

ww
as the original rectangle would result. The perimeter of the
original rectangle (in centimetres) is :
(a) 44 (b) 46
28.
(e) None of these
Raju decided to marry 3 years after he gets a job. He was 17
years old when he passed class 12th. After passing class
(c) 48
(e) None of these
w.E (d) 50 12th', he had completed his graduation course in 3 years
and PG Course in 2 years. He got the job exactly 1 year after
completing his PG Course. At what age will he get married?

asy
DIRECTION (Qs. 19-21): What will come in place of the question (a) 27 years (b) 26 years
mark (?) in the following number series? (c) 28 years (d) 23 years
(e) None of these

En
19. 2 9 30 105 ? 2195
(a) 432 (b) 426 DIRECTIONS (Qs. 29-33): Study the following table carefully to
answer the questions that follow:

gin
(c) 440 (d) 436
(e) None of these Number of Students studying in Six Different Colleges over the
20. 3 4 12 45 ? 1005 Years
(a) 152
(c) 144
(b) 198
(d) 192
College eer
21.
(e) None of these
1 3 9 31 ? 651 Year
2004
P Q

ing R S T

2500 2250 2450 2150 2020 2300


U

.ne
(a) 97 (b) 127
(c) 129 (d) 109 2005 2040 2300 2400 2200 2090 2120
(e) None of these 2006 2100 2150 2330 2250 2180 2260
DIRECTIONS (Qs. 22-23): Find out the approximate value which
should replace the question mark (?) in the following questions.
(You are not expected to find out the exact value.)
2007
2008
2009
2280 2600 2260 2340 2250 2490
2540 2240 2120 2380 2310 2520
2320 2440 2500 2480 2400 2440
t
22. 953.7 950.9989 95?
29. What is the total number of students from all the colleges
(a) 1.9 (b) 3 together in the year 2005?
(c) 2.99 (d) 3.6
(a) 10350 (b) 13150
(e) 2.7 (c) 15310 (d) 11350
3.001 (e) None of these
23 1000 of 1891.992 = ? 30. What is the per cent increase in the number of students in
4.987
College T in the year 2007 from the previous year? (rounded
(a) 2500 (b) 1230
off to two digits after decimal)
(c) 1640 (d) 1525
(a) 8.33 (b) 5.18
(e) 2130
(c) 6.63 (d) 3.21
24. 12.25 × ? × 21.6 = 3545 .64 (e) None of these
(a) 20 (b) 12 31. The number of students in college P in the year 2008 forms
(c) 15 (d) 13 approximately what per cent of the total number of students
(e) None of these in the college in all the year together.

Downloaded From : www.EasyEngineering.net


Downloaded From : www.EasyEngineering.net

SPEED TEST 99 223


(a) 11 (b) 31 41. Four of the following five are alike in a certain way and so
(c) 18 (d) 26 form a group. Which is the one that does not belong to that
(e) 23 group?
32. What is the ratio of the total number of students in College (a) 169 (b) 441
S in the years 2006 and 2009 together to the total number of (c) 361 (d) 529
students in College U in the same years? (e) 289
(a) 473 : 470 (b) 470 : 473 42. Among P, Q, R, T and W each having different weight, T is
(c) 371 : 390 (d) 390 : 371 heavier than W and lighter than only P. Q is not the lightest.
(e) None of these Who among them is definitely the lightest ?
33. What is the average number of students in all the colleges (a) R (b) W
together in the year 2004? (rounded off to the nearest integer) (c) R or W (d) Data inadequate
(a) 2208 (b) 2196 (e) None of these
(c) 2144 (d) 2324
(e) 2278 DIRECTIONS (Q. 43-47) : In each question below are three
34. The angles of a triangle are in the ratio of 5 : 6 : 7. respectively. statements followed by three conclusions numbered I, II and III.
What is the sum of the smallest angle and the largest angle You have to take the three given statements to be true even if they
together? seem to be at variance from commonly known facts and then

ww
(a) 130°
(c) 110°
(e) None of these
(b) 100°
(d) 140°
decide which of the given conclusions logically follows from the
three given statements disregarding commonly known facts. Then
decide which of the answers (a), (b), (c), (d) and (e) is the correct
35.

(a) 7
w.E
What least number should be subtracted from 536 to make it
a perfect square?
(b) 5
answer and indicate it on the answer sheet.
43. Statements : Some desks are chairs. All chairs are tables.
Some tables are mats.
(c) 23
(e) None of these
(d) 18
asy Conclusions : I. Some mats are desks.
II. Some tables are desks.

Part-II : Reasoning Ability


En III.Some mats are chairs.
(a) Only I follows (b) Only II follows

gin
36. How many such pairs of letters are there in the word (c) Only III follows (d) II and III follow
CHANNEL each of which has as many letters between them (e) None of the above
in the word as in the English alphabet? 44. Statements : All sweets are fruits. No fruit is pencil. Some
(a) None
(b) One
eer
pencils are glasses.
Conclusions : I. Some glasses are sweets.
(c) Two
(d) Three
(e) More than three ing
II. Some pencils are sweets.
III.No glass is sweet.

37. How many meaningful English words can be made with the
letters ATLE using each letter only once in each word ?
(a) Only I follows
(c) Only III follows
(e) None of the above .ne
(b) Only II follows
(d) either I or III follows

38.
(a) None
(c) Two
(e) More than three
(b) One
(d) Three

In a certain code GROWN is written as 7 @ % 36 and NAME


45.
chains. Some chains are hammers.
Conclusions : I. Some hammers are flowers.
t
Statements : Some books are flowers. Some flowers are

II. Some chairs are books.


is written as 64 $. How is GEAR· written in that code?
III.Some hammers are books.
(a) 74$@ (b) 7$4@
(a) None follows (b) Only I follows
(c) 7%4@ (d) 7@$4
(e) None of these (c) Only II follows (d) Only III follows
39. How many pairs of digits are there in the number 6315784 (e) II and III follow
each of which has as many digits between them in the number 46. Statements : All roofs are cameras. Some cameras are
as when the digits are rearranged in descending order? photographs.
(a) None (b) One Some photographs are stores.
(c) Two (d) Three Conclusions : I. Some stores are cameras.
(e) More than three II. Some stores are roofs.
40. What should come next in the following letter series? III.Some cameras are roofs.
B D F H J L N A C E G I K M B D F H J LA C E G I K B D F H J (a) Only I follows (b) Only II follows
(a) B (b) L
(c) Only III follows (d) II and III follow
(c) M (d) F
(e) None of the above
(e) None of these

Downloaded From : www.EasyEngineering.net


Downloaded From : www.EasyEngineering.net

224 SPEED TEST 99


47. Statements : Some nails are horses. All horses are tablets. (a) None is true (b) Only I is true
All tablets are crows. (c) Only II is true (d) Only III is true
Conclusions : I. Some crows are nails. (e) II and III are true
II. Some tablets are nails. 54. Statements : F @ T, T % M, M # R
III.Some crows are horses. Conclusions :
(a) Only I follows (b) I and II follows I. R © T II. F @M
(c) I and III follow (d) II and III follow III. F©M
(e) All I, II and III follow (a) Only I is true (b) Only II is true
(c) Only III is true (d) either II or III is true
DIRECTIONS (Q. 48-52) : Study the following arrangement (e) II and III are true
carefully and answer the questions given below 55. Statements : J H, H @ B, B % N
D5 R@AK© 3 9 BJE F$ MPI4 H1W Conclusions :
6 2 # UQ8 T N I. N H II. N @ J
48. How many such numbers are there in the above arrangement III. J B
each of which is immediately preceded by a symbol and (a) I and II are true (b) II and III are true
immediately followed by a letter? (c) I and III are true (d) All I, II and III are true
(a) None (b) One (e) None of the above

49. ww
(c) Two
(e) More than three
(d) Three

Which of the following is the ninth to the right of the twenty


56. Statements : B # T, T © K, K % M
Conclusions :
I. K # B

(a) E
(c) D w.E
second from the right end of the above arrangement ?
(b) I
(d) N
II. M # T
III. B # M
(a) Only I is true
50.
(e) None of these

asy
How many such symbols are there in the above arrangement
each of which is immediately preceded by a number and
(b) Only II is true
(c) Only III is true
immediately followed by a letter?
(a) None (b) One
En (d) II and III are true
(e) None of the above

51.
(c) Two
(e) More than three
(d) Three

If all the numbers are dropped from the above arrangement, gin
57. Statements : D % F, F K, K @ R
Conclusions :
which of the following will be the eleventh from the left
end ?
I. R % F
II. R % D
III. R@ D eer
ing
(a) B (b) H
(c) $ (d) (a) Only I is true (b) Only II is true
(e) None of these (c) Only III is true (d) I and II are true
52. How many such consonants are there in the above
arrangement each of which is immediately preceded by a
number and immediately followed by another consonant ?
(e) None of the above
.ne
t
DIRECTIONS (Q. 58-62) : Study the following arrangement
(a) None (b) One carefully and answer the questions given below
(c) Two (d) Three
(e) More than three M, D, K, R, T, H, W and A are sitting around a circle facing
at the centre. D is second to the right of M who is fifth to the left
DIRECTIONS (Q. 53-57) : In the following questions, the symbols of T. K is third to the right of R who is second to the right of D. H
#, %, @, © and are used with the following meanings illustrated. is second to the right of W.
‘P % Q’ means ‘P is not greater than Q’. 58. Who is second to the right of A ?
‘P Q’ means ‘P is not smaller than Q’. (a) M (b) D
‘P # Q’ means ‘P is neither equal to nor smaller than Q’. (c) K (d) Data inadequate
‘P © Q’ means ‘P is neither equal to nor greater than Q’. (e) None of the above
‘P @ Q’ means ‘P is neither smaller than nor greater than Q’. 59. Who is third to the left of M ?
In each question, three statements showing relationships have (a) A (b) T
been given, which are followed by three conclusions I, II and III.
(c) H (d) D
Assuming that the given statements are true, find out which
conclusion(s) is/are definitely true. (e) Data inadequate
53. Statements : M © K, K T, T © J 60. Who is fourth to the right of H ?
Conclusions : (a) A (b) T
I. J # K II. T # M (c) R (d) K
III. M # J (e) None of these

Downloaded From : www.EasyEngineering.net


Downloaded From : www.EasyEngineering.net

SPEED TEST 99 225


61. In which of the following combinations is the first person (d) Department of defense production provides the social
sitting between the second and the third person ? security to its ex-servicemen.
(a) KMW (e) None of these
(b) MWD 66. Four of the following five are alike in a certain way and so
(c) RHT form a group. Which is the one that does not belong to that
(d) TAK group?
(a) Water (b) Juice
(e) None of the above
(c) Petrol (d) Sugar
62. If A and W interchange their positions who will be third to
the left of R ? (e) None of these
(a) M (b) D 67. How many meaningful English words can be made with the
letters EIND using each letter only once in each word ?
(c) A (d) K
(a) None (b) One
(e) None of these
(c) Two (d) Three
DIRECTIONS (Qs. 63-65) : Study the following paragraph and (e) None of these
answer the questions that follow : 68. Four of the following five are alike in a certain way and so
The Government of India is responsible for ensuring the form a group. Which of the following does not belong to

ww
defense of India and every part thereof. The Supreme Command of that group?
the Armed Forces vests in the President. The responsibility for (a) 343 (b) 64
national defense rests with the Cabinet. This is discharged through (c) 75 (d) 27

w.E
the Ministry of Defense, which provides the policy framework and (e) 216
wherewithal to the Armed Forces to discharge their responsibilities
69. How many such digits are there in the number 586972 each
in the context of the defense of the country. Ministry of Defense
of which is as far away from the beginning of the number as

asy
comprises of four Departments viz. Department of Defense (DOD),
when the digits are arranged in discending order.
Department of Defense Production (DDP), Department of Defense
(a) None (b) One
Research & Development (DDR&D) and Department of Ex-

En
Servicemen Welfare and also Finance Division (c) Two (d) Three
63. Which of the following is a conclusion which can be drawn (e) More than Three

gin
from the facts stated in the given passage? 70. What should come next in the following number series ?
(a) President of India is responsible for the defense of the 98 7654 3218 7654 3217 6543 21
country. (1) 9 (2) 8
(b) The government of India is responsible for defense of
the country and ministry of defense performs the duty
(c) 7
eer
(e) None of these
(d) 6

with help of its various departments.


(c) People of India are themselves responsible four their
security. ing
Part-III : English Language
(d) Indians are so bold that there is no need of security.
(e) None of these .ne
DIRECTIONS (71-80) : Read the following passage carefully
and answer the questions given below it. Certain words/phrases
64. Which statement strengthens the conclusion of the
passages?
(a) All ministries of the government co-operate to each
other and prime minister co-ordinates them.
some of the questions.
t
have been printed in bold to help you locate them while answering

Once upon a time there lived a vicious king, Raja Shankara–


short-tempered and temperamental. "God I am" he said to his
(b) There are many falts in our defense system which image as he stared into the mirror everyday, many times a day, He
come-out time to time. was obsessed with himself. He loved no one but himself. He was
(c) Now-a-days the ministry of defense is not working blinded towards the injustice in his kingdom because he had little
properly consequently internal and external security time for his subjects. He wasted most of his time in pouring milk
is in danger. and honey over himself.
(d) People of the country is themselves aware for their Interruption in his possessed life was dealt with stern
security. reprimanding and sometimes on petty issues he would behead his
(e) None of these servants. Provoked by his evil advisor Twishar, he went on with
65. Which statement weakens the conclusion of the passage. his self indulged life, unaware of the plot his very devoted advisor
(a) President of India is the supreme commander of the was planning. A plot to dethrone the king, rule the kingdom with
Armed Forces. his wicked ways only to harness wealth and the reputation of a
(b) Due to lack of co-ordination ministry of defense is king.
unable to determine the security of people. One morning the king went on his usual moring horseback
(c) Ministry of defense is divided in four various rounds but returned with a very sad look on his face. He locked
departments. himself inside his platial room only to unlock it at sundown. Just

Downloaded From : www.EasyEngineering.net


Downloaded From : www.EasyEngineering.net

226 SPEED TEST 99


as the doors creaked open and Raja Shankara emerged from it, his (d) That his subjects were not good enough to deserve
wife rushed to embrace him. She feared a damaging incident had better treatment than what was already being meted
occurred. out to them
The king spoke seldom that day and awoke the next day to (e) None of these
make a proclamation to his servants and subjects. The whole 74. What did Raja Shankara's wife think about the Raja's peculiar
kingdom feared what was in store for them from their angry king. behaviour that particular day?
But to their surprise he said to all gathered, "From now on I will be (a) She was afraid that something really bad had happened
a different king. A softer and a patient king." (b) She was afraid that the Raja would beat her up because
True to his words from that day on, the king had truly turned of his unusual mood
on a new leaf; he cleaned out the corruption and injustice in a (c) She thought that he was in his usual sour mood
tender manner with punishments aimed to renew the person from (d) She thought that the Raja had received threats to his
within. life from his servants
One fine day his evil advisor gathered courage to ask the (e) None of these
reason for his paradigm shift. And the king answered. When I
75. What was the reason for Raja Shankara's change in
went on horseback that morning a month ago, I noticed a dog
behaviour?
brutally chasing a cat. The cat managed to sneak into a hole only
(a) His advisor's words had made him realize his mistake
after the dog bit her leg, maiming her for life. soon afterwards, the

ww
dog barked at a farmer who picked up a sharp stone and hit it
straight in the dog's eye. Bleeding profusely, the dog yelped in
pain. As the farmer walked on, he slipped on the edge of the road
(b) He had felt bad for a poor family on his tour around his
kingdom
(c) His wife had betrayed him and hence he was upset
and broke his head.
w.E
All this happened in a matter of minutes before me and then
I realized that evil begets evil. I thought about it deeply and was
(d) He had realized that doing good to people would bring
good to him
(e) He had realized that evil begets evil.

asy
ready to give up my worldly life for the betterment of my subjects.
I wanted to give up the evil in me as I did not want evil to encounter
me.
DIRECTIONS (76-78) : Choose the word/group of words which
is most similar in meaning to the word/group of words printed in

En
bold as used in the passage.
Sniggering away the immoral advisor thought what a perfect 76. BEGETS
time it was to dethrone the king, because the Raja had grown kind
hearted and patient and would not endeavour a combat. Thinking
how he would plan his attack, he stumbled over a step that took gin (a) produces
(c) expects
(e) calls
(b) loses
(d) avoids
him hurling down the remaining steps, bringing him to a stop with
a crash. He howled in pain only to discover he had broken the
bones in both his legs.
77. STERN
(a) hard eer (b) tall
71. How can Raja Shankara be described before his
transformation?
(c) easy
(e) severe ing (d) tight

(1) He was unjust


(2) He was preoccupied with himself
78. GATHERED
(a) partied
.ne
(b) assembled

t
(3) He was cruel (c) dispersed (d) pooled
(a) Only (1) (b) Only (2) (e) collated
(c) Only (3) (d) Only (1) adn (2) DIRECTIONS (79-80) : Choose the word/group of words which
(e) All the three (1), (2) and (3) is most opposite in meaning to the word/group of words printed
72. Why was the king not happy with the old Valuer? in bold as used in the passage,
(a) As the Valuer was not good at his work 79. PETTY
(b) As he had dishonoured the king (a) tremendous (b) huge
(c) As the Valuer had been dishonest with the king about (c) vast (d) important
the prices that he set for goods (e) frugal
(d) As the king beloeved that he was not earning much 80. BRUTALLY
because of the Valuer's honesty (a) cruelly (b) partly
(e) None of these (c) gently (d) rarely
73. What proclamation did the Raja make to his subjects? (e) harmfully
(a) That he was giving up his throne for the betterment of DIRECTIONS (81-83) : Which of the phrases (a), (b), (c) and (d)
the kingdom given below each sentence should replace the phrase printed in
(b) That his advisor would be the king from then on bold in the sentence to make it grammatically correct? If the
(c) That he would be a better king to them than he had sentece is correct as it is given and no correction is required,
been all this while mark (e) as the answer.

Downloaded From : www.EasyEngineering.net


Downloaded From : www.EasyEngineering.net

SPEED TEST 99 227


81. Her entry to the office party was restrict as an official walking (92) they found an oasis, where they (93) to take a both.
enquiry had ben constituted against her. The one, who had been slapped, got (94) in the quicksand and
(a) was restricting started drowing, but the friend saved him. After the friend (95)
(b) is restricted from the near drowning, he wrote on a stone, "The friend who had
(c) was restricted slapped and saved his best friend asked him, "After I hurt you,
you wrote in the sand and (96) you write on a stone, why?" The
(d) is restricting
other friend (97), "When someone hurts us, we should write it
(e) No correction required
down in sand where winds of forgiveness can erase it away. But,
82. Rima was at her wit's end trying to figure out what to buy when someone does something good for us, we must (98) it in
for her frind's birthday. stone where no wind can ever erase it."
(a) at her witting end 89. (a) crawling (b) speaking
(b) at her wit ends
(c) swimming (d) walking
(c) to her wit's end
(e) dancing
(d) so wit's end
90. (a) journey (b) sand
(e) No correction required
(c) running (d) border
83. Pritesh while away his time in playing games on the computer
(e) hunt
instead of studying.

ww
91. (a) dead (b) captured
(a) whiled away his time
(c) presentable (d) missing
(b) whiled against his time
(e) hurt
(c) whiling away his time

w.E
(d) while awayed his time
(e) No correction required
92. (a) as
(c) from
(e) through
(b) until
(d) with

asy
DIRECTIONS (84-88) : In each question below, a sentence with
four words printed in bold type is given. These are numbered as
93. (a) decided
(c) made
(b) fell
(d) want

En
(a), (b), (c) and (d). One of these four words printed in bold may be (e) left
either wrongly spelt or inappropriate in the context of the 94. (a) home (b) stuck
sentence. Find out the word which is wrongly spelt or
inappropriate, if any. The number of that word is your answer. If
all the words printed in bold are correctly spelt and also
gin (c) blended
(e) sitting
95. (a) separated
(d) mixed

(b) leaked

eer
appropriate in the context of the sentence, mark (e) i.e., 'All
correct' as your answer. (c) died (d) recovered
(e) saved
84. Discussion (a)/ is an exchange of knowledge (b)/ whereas
arguement (c)/ is a depiction (d)/ of ignorance. All correct
(e).
96. (a) s o
(c) when ing (b) how
(d) tomorrow
85. He was arrested (a)/ for the crime (b)/ and was charged (c)/
with attempt (d)/ to murder. All correct (e)
(e) now
97. (a) called (b) tell .ne
86.

87.
commit (a)/ yourself to lifelong learning (b)/ as the most
valuable (c)/ aset (d)/ you will have is your mind.
All correct (e)
Belive (a)/ that life is worth (b)/ living and your belief will
(c) replied
(e) asked
98. (a) talk
(c) engrave
(d) questioned

(b) push
(d) add
t
create (c)/ the fact. (d)/ All correct (e)
(e) bury
88. The best educated (1)/ human bing (2)/ is the one who
understands (3)/ most about the life in which (4)/ he is DIRECTIONS (Qs. 99 to 100): In each of the following sentences,
placed. All correct (e). an idiomatic expression or a proverb is highlighted. Select the
alternative which best describes its use in the sentence.
DIRECTIONS (89-98) : In the following passage there are blanks,
99. Mohan always keeps himself to himself.
each of which has been numbered. These numbers are printed
below the passage and against each, five words are suggested, (a) Is too busy (b) Is selfish
one of of which fits the blank appropriately. Find out the (c) Is unsociable (d) does not take sides
appropriate word in each case. (e) None of these
Once upon a time, two friends were (89) through the desert. 100. While the ladies continued their small talk in the drawing
During some point of the (90) they had an argument, and one room, I felt bored.
friend slapped the other one in the face. The one who got slapped (a) whispering (b) backbiting
was (91), but without saying anything, he wrote in the sand, (c) gossip (d) light conversation
"Today my best friend slapped me in the face." They kept on (e) None of these

Downloaded From : www.EasyEngineering.net


Downloaded From : www.EasyEngineering.net

228 SPEED TEST 99

RESPONSE SHEET
Test Code : . ............................. Time taken : ......................... Date : ..........................

(a) (b) (c) (d) (e) (a) (b) (c) (d) (e) (a) (b) (c) (d) (e) (a) (b) (c) (d) (e)
1. 26. 51. 76.
2. 27. 52. 77.
3. 28. 53. 78.
4. 29. 54. 79.
5. 30. 55. 80.
6. 31. 56. 81.
7.
8. ww 32.
33.
57.
58.
82.
83.
9.
10.
w.E 34.
35.
59.
60.
84.
85.
11.
12.
36.
37.
asy 61.
62.
86.
87.
13. 38.
En 63. 88.
14.
15.
39.
40. gin
64.
65.
89.
90.
16. 41. 66.
eer 91.
17.
18.
42.
43.
67.
68. ing 92.
93.
19.
20.
44.
45.
69.
70.
94.
95. .ne
21.
22.
46.
47.
71.
72.
96.
97.
t
23. 48. 73. 98.
24. 49. 74. 99.
25. 50. 75. 100.

Downloaded From : www.EasyEngineering.net


Downloaded From : www.EasyEngineering.net

Full Main Test - 4 100


Max. Marks : 200 No. of Qs. 190 Time : 2 hr 40 min. Date : ........./......../. ..............

Part-I : General and Financial Awareness (a) Amul-Gujarat


(b) Parag-Uttar Pradesh
1. Currency Swap is an instrument to manage (c) Gopal-Rajasthan
(a) currency risk (d) Verka-Punjab
(b) interest rate risk (e) None of these
(c) currency and interest rate risk 9. Which of the following terms is used in Banking Field?

ww
(d) cash flows in different currencies
(e) All of the above
(a) Interest Rate Swap
(c) Sedimentary
(b) Input Devices
(d) Zero Hour
2.

elections in India? w.E


Which of the following voting systems has been adopted
for the election of Lok Sabha and Legislative assembly 10.
(e) Privilege Motion
The term 'Smart Money" refers to __________ .
(a) Foreign Currency (b) Internet Banking

asy
(a) Single transferable vote system (c) US Dollars (d) Travelers' cheques
(b) First-past-the-post system (e) Credit Cards
(c) Hare-clark system 11. Which one of the following is not a 'Money Market
(d) Single non transferable vote system
(e) None of these
En Instrument' ?
(a) Treasury Bills (b) Commercial Paper
3. Money Laundering normally involves
(a) placement of funds (b) layering of funds
(c) integration of funds (d) All of (a), (b) and (c) gin
12.
(c) Certificate of Deposit (d) Equity Shares
(e) None of these
Which one of the following is a retail banking product ?

4.
(e) None of these
Capital Market Regulator is eer
(a) Home Loans (b) Working capital finance
(c) Corporate term loans (d) Infrastructure financing
(a) RBI
(c) NSE
(e) SEBI
(b) IRDA
(d) BSE 13.
(e) Export Credit

ing
Which of the following is NOT a function of the Reserve
Bank of India ?
5. Speaker of Lok Sabha is
(a) Fiscal Policy Functions
(b) Exchange Control Functions .ne
t
(a) appointed by the President of India on the advice of
(c) Issuance, Exchange and destruction of currency notes
the Cheif Justice of the supreme court
(d) Monetary Authority Functions
(b) appointed by the President of India
(e) Supervisory and Control Functions
(c) elected by the members of the Lok Sabha
14. With reference to a cheque which of the following is the
(d) elected by the members of Lok Sabha and Rajya Sabha
“drawee bank” ?
(e) None of these
(a) The bank that collects the cheque
6. Which of the following is the Regulator of the credit rating
(b) The payee's bank
agencies in India?
(c) The endorsee's bank
(a) RBI (b) SBI
(d) The endorser's bank
(c) SIDBI (d) SEBI
(e) The bank upon which the cheque is drawn
(e) None of these
15. What is a Debit Card ?
7. Lot of Banks in India these days are offering M-Banking
(a) It is a card issued by a Rating Agency
Facility to their customers. What is the full form of 'M' in 'M-
(b) It is a card which can be used for withdrawing cash or
Banking'
(a) Money (b) Marginal making payment even in the absence of any balance in
(c) Message (d) Mutual Fund the account
(e) Mobile phone (c) It is a card which can be used for withdrawing cash or
making payment if there is balance in the account
8. Which of the following is mis-matched in relation to leading (d) It is a card which carries prepaid balance
brands of milk and milk products in India? (e) It is a card which can be used for making STD calls

Downloaded From : www.EasyEngineering.net


Downloaded From : www.EasyEngineering.net

230 SPEED TEST 100


16. Bad advances of a Bank are called __________ . 27. The Indian space agency is soon opening a 100-acre Space
(a) Bad debt (b) Book debt Park in which of the following cities?
(c) Non Performing Asset (d) Out of order accounts (a) Bengaluru (b) Nagpur
(e) Overdrawn accounts (c) Mumbai (d) New Delhi
17. The "Sunil Gangopadhyay Memorial Award" is given for (e) None of these
excellence in ___? 28. The World Day of War Orphans is observed on which of
(a) Telugu (b) Bengali the following day?
(c) Odisi (d) Rajasthani (a) 7th January (b) 6th January
(e) None of these (c) 8th January (d) 9th January
18. National Girl Child Day was observed across India by the (e) None of these
Union Government on 29. Which of the following companies has won 26th Lal Bahadur
(a) 24 January (b) 25 January Shastri Hockey tournament?
(c) 27 January (d) 21 January (a) Coal India Limited
(e) None of these (b) Oil and Natural Gas Corporation (ONGC)
19. Who is the author of the book "My Journey: Transforming (c) Indian Oil Corporation Limited
Dreams into actions"? (d) Steel Authority of India Limited
(a) A P J Abdul Kalam (b) Sachin Tendulkar (e) None of these

20.
ww
(c) Pranab Mukherjee
(e) None of these
(d) Amartya Sen

Where is the headquarter of United Nations Children's Fund


30. Which of the following cities have made it to a top 30 list of
the world's most powerful, productive and connected cities,
as per the study conducted by international real estate
(UNICEF) located?
(a) New York
(c) Italy
w.E (b) Berlin
(d) Washington D.C
consultancy JLL?
(a) Chennai and Bangalore
(b) Kolkata and pune

21.
(e) None of these
asy
In which of the following states the Ganga Gram Yojana has
(c) Delhi and Mumbai
(d) Delhi and Bangalore
(e) None of these
been launched?
(a) Uttarakhand (b) Bihar
En 31. Which of the following organisations has launched Jeevan
Labh Scheme?

gin
(c) West Bengal (d) Uttar Pradesh (a) New India Insurance Ltd
(e) None of these (b) National Insurance Company Ltd
22. For what purpose, the Stand Up India Scheme has been (c) State Bank of India
launched?
(a) To promote entrepreneurship among SC/ST
eer
(d) Life Insurance Corporation of India
(e) None of these
(b) To promote entrepreneurship among Rural Youth
(c) To promote entrepreneurship among Woman
(d) To promote entrepreneurship among SC/ST and
32.
ing
Who of the following is the author of the book titled 'Jinnah
Often Came to Our House?

.ne
(a) Jaswant Singh (b) Kiran Doshi
Women (c) Nisid Hajari (d) Jairam Ramesh
(e) None of these (e) None of these
23. Who of the following is the author of the novel titled - Go
Set a Watchman?
(a) Cynthia Lord
(c) Marlon James
(b) Bill Clegg
(d) Harper Lee
33. On what date, the World Braille Day is observed?
(a) January 4
(c) November 9
(e) None of these
(b) December 30
(d) April 11
t
(e) None of these 34. Which of the following states became the first state in the
24. Which of the following companies publishes Purchasing country to sign MoU for UDAY scheme?
Managers' Index (PMI)? (a) Rajasthan (b) Gujarat
(a) National Stock Exchange of India Limited (c) Uttar Pradesh (d) Jharkhand
(b) BSE Institute Limited (c) Markit (e) None of these
(d) HSBC Analytics (e) None of these 35. The 14 km long Zojila tunnel will be constructed in which of
25. The 'Fountain of Oneness' is located in which of the the following states?
following cities? (a) Sikkim (b) Jammu & Kashmir
(a) Ajmer (b) Delhi (c) Uttarakhand (d) Himachal Pradesh
(c) Agra (d) Khajuraho (e) None of these
(e) None of these 36. Which of the following person appointed as Chairman of
26. Which of the following cities to get India's first river gallery? Film and Television Institute of India (FTII)?
(a) Bengaluru (b) Mysore (a) Neha Gupta (b) Hrishikesh Kanitkar
(c) Chennai (d) Hyderabad (c) Gajendra Chauhan (d) Arvind Uppal
(e) None of these (e) None of these

Downloaded From : www.EasyEngineering.net


Downloaded From : www.EasyEngineering.net

SPEED TEST 100 231


37. Who has been appointed as the new chairman of 47. Which of the following companies has acquired Shifu for $8
Competition Commission of India (CCI)? million to enrich customer experience?
(a) DK Sikri (b) Amitabh Kant (a) Paytm (b) Flipkart
(c) Ashok Chawla (d) Dhanendra Kumar (c) Amazon (d) Myntra
(e) None of these
(e) None of these
38. In which of the following cities the world's first slum museum
48. Which of the following financial entity has been penalised
will be set up?
(a) Indore (b) Mumbai by the Securities and Exchange Board of India (SEBI) for
(c) Lucknow (d) Ahmedabad breaching the stock broker regulations?
(e) None of these (a) R. V. Verma (b) Ashwini Kumar
39. Who has been conferred with the 2016 Global and Asia - (c) Ravindra Kumar (d) Anand Rathi
Pacific Central Bank Governor of the Year? (e) None of these
(a) Janet Yellen (b) Mark Carney 49. A friendship bus service between India and was flagged off
(c) Raghuram Rajan (d) Haruhiko Kuroda via Champawat district in Uttarakhand.
(e) None of these (a) Nepal (b) Japan
40. Who has been honoured with the second Harikrishna (c) Australia (d) Canada

ww
Devsare Baalsahitya Award 2015?
(e) None of these
(a) Gangesh Gunjan (b) Sheela Jhunjhunwala
50. Who has been crowned as the FBB 50. Famina Miss India
(c) Balkrishna Garg (d) Sherjung Garg
world 2016?
41.
(e) None of these

w.E
In which state, the Union govt has decided to set up National
Organic Farming Research Institute (NOFRI) in 2016?
(a) Namrata Sharma
(b) Pankuri Gidwani
(a) Mizoram
(c) Assam
(e) None of these asy
(b) Meghalaya
(d) Sikkim
(c) Priyadarshini chatterjee
(d) Sushruthi Krishna
(e) None of these
42. Who was appointed as the CEO of Wipro?
(a) Abid Ali Z. Neemuchwala En Part-II : Quantitative Aptitude
(b) T. Kurien
(c) Lalitha Kumaramangalam
gin
DIRECTIONS (Qs. 51-61): What will come in place of the question
mark (?) in the following questions ?

eer
(d) Ramesh Sippy
(e) None of these 51. 3 × ? + 30 = 0
43. Who of the following has won the 2015 Costa Novel Prize? (a) – 15 (b) 15
(a) Frances Hardinge
(c) Andrea Wulf
(b) Kate Atkinson
(d) Helen Macdonald
(c) 10
(e) None of these ing (d) – 30

44.
(e) None of these
Vilmos Zsigmond, who passed away recently, belong to
52. 40.83 × 1.02 × 1.2 = ?
(a) 49.97592
(c) 58.7952 .ne
(b) 41.64660
(d) 42.479532

t
which of the following fields?
(a) Politician (b) Scholar (e) None of these
(c) Sportsperson (d) Cinematographer 1 3 1 22
(e) None of these 53. 3 6 1 ?
3 7 2 7
45. National Bank for Agriculture and Rural Development
(NABARD) signed an Memorandum of Understanding 22
(a) 4.4 (b)
(MoU) with which of the following for monitoring Watershed 7
Projects?
5
(a) Central Glass and Ceramic Research Institute (CGCRI) (c) (d) 40.5
(b) National Remote Sensing Centre (NRSC) 22
(c) Central Institute of Technology (CIT) (e) None of these
(d) Indian Association for the Cultivation of Science 54. 1.5625 ?
(IACS) (a) 125 (b) 12.5
(e) None of these (c) 1.05 (d) 1.25
46. Raghunandan Mandal has passed away recently, he related (e) None of these
to? 55. 3978 + 112 × 2 = ? 2
(a) Politics (b) Scholar (a) 8400 (b) 8406
(c) Sportsperson (d) Cinematographer (c) 8600 (d) 8404
(e) None of these (e) None of these

Downloaded From : www.EasyEngineering.net


Downloaded From : www.EasyEngineering.net

232 SPEED TEST 100


2 66. A, B and C enter into a partnership with investments of
56. 103.7 101.3 10? ` 3500, ` 4500 and ` 5500, respectively. In the first six months,
profit is ` 405. What is A’s share in the profit ?
(a) 6 (b) 7 (a) ` 200 (b) ` 105
(c) 5 (d) 3 (c) ` 250 (d) ` 151
(e) None of these (e) None of these
57. 7589 – ? = 3434 67. A tap can fill a cistern in 8 hours and another tap can empty
(a) 721 (b) 4055 it in 16 hours. If both the taps are opened simultaneously,
(c) 3246 (d) 11023 the time taken (in hours) to fill the cistern will be :
(e) None of these (a) 8 (b) 10
(c) 16 (d) 24
58. 1225 ? 5
(e) None of these
(a) 3 (b) 2 68. Pipes A and B can fill a tank in 5 and 6 hours, respectively.
(c) 35 (d) 7 Pipe C can empty it in 12 hours. The tank is half full. All the
(e) None of these three pipes are in operation simultaneously. After how much
59. 300 + 102 × 2 = ? time, the tank will be full ?
(a) 450 (b) 800
9
(c) 550 (d) 320 (a) 3 h (b) 11 h

ww
(e) None of these
5 1.6 2 1.4 (c) 2
8
17

h
13
(d) 1 h

w.E
60. ? 11 17
1.3
(a) 4 (b) 0.4 (e) None of these
(c) 1.4 (d) 1.2 69. If the sum of the digits of an even number is divisible by 9,
(e) None of these
2 1 1 asy then that number is always divisible by :
(a) 24
(c) 18
(b) 12
(d) 27

En
61. 3 7 5 ?
5 5 4 (e) None of these
70. A water tank in the form of a cuboid has its base 20 m long,

gin
3 3
(a) 5 (b) 5 7 m wide and 10 m deep. Initially, the tank is full but later
10 20
when water is taken out of it, the level of water in the tank

eer
7 11 reduces by 2 m. The volume of water left in the tank is :
(c) 5 (d) 5 (a) 1120 m3 (b) 400 m3
10 20
(c) 280 m 3 (d) 140 m3
62.
(e) None of these
If one-third of a number is 3 more than one -fourth of the
number, then the number is :
(e) None of these
ing
DIRECTIONS (Qs. 71-75) : Find the next term in the given series
(a) 18
(c) 30
(b) 24
(d) 36
in each of the questions below.
71. 41, 31, ?, 17, 11, 5 .ne
63.
(e) None of these
A boy was asked to write 25 9 2 but he wrote 2592. The
numerical difference between the two is:
(a) 0 (b) 3
(a) 19
(c) 23
(e) None of these
72. 8, 15, 28, 53, ?
(b) 21
(d) 27
t
(a) 106 (b) 98
(c) 2 (d) 9
(c) 100 (d) 102
(e) None of these
(e) None of these
64. If the two numbers are respectively 20% and 50% of a third
73. 24, 49, ?, 94, 15, 31, 59, 58
number, what is the percentage of the first number to the
(a) 51 (b) 63
second ?
(c) 77 (d) 95
(a) 10 (b) 20
(e) None of these
(c) 30 (d) 40
74. 5, 10, 13, 26, 29, 58, ?, 122
(e) None of these
(a) 60 (b) 61
65. A man gains 10% by selling a certain article for a certain
(c) 111 (d) 91
price. If he sells it at double the price, then the profit made
is: (e) None of these
(a) 120% (b) 60% 75. 2, 3, 10, 15, 26, ?, 55
(c) 100% (d) 80% (a) 32 (b) 33
(e) None of these (c) 34 (d) 35
(e) None of these

Downloaded From : www.EasyEngineering.net


Downloaded From : www.EasyEngineering.net

SPEED TEST 100 233


DIRECTIONS (Qs. 76-80) : What approximate value should come DIRECTION (Qs. 86-90) : Study the following graph carefully to
in place of the question mark (?) in the following questions? answer the question given below it.
(You are not expected to calculate the exact value). Production of paper (in lakh tonnes) by 3 different
76. 3
860000 ? companies A, B & C over the years
(a) 75 (b) 80
(c) 110 (d) 125 70
60 60 60 60
(e) 95 60 55 55 55 55
50 50 50 50 50
5 1 2 50 45 45 45
77. 1 5 2 ? 40 40
8 3 5 40
(a) 15 (b) 4 30
(c) 19 (d) 9 20
(e) 21
10
78. 8769 82 4 ?
0
(a) 27 (b) 44 2010 2011 2012 2013 2014 2015
(c) 429 (d) 12

79. ww
(e) 512
? % of 45.999 × 16% of 83.006 = 116.073
A B C

(a) 6
(c) 19
(e) 11 w.E (b) 24
(d) 30
86. What is the difference between the production of company
C in 2010 and the production of Company A in 2015?
(a) 50,000 tonnes (b) 5,00,00,000 tonnes

asy
80. 12.998 × 27.059 × 17.999 = ? (c) 50,00,000 tonnes (d) 5,00,000 tonnes
(a) 6020 (b) 6320
(e) None of these
(c) 6800 (d) 6540

81.
(e) 6150
En
The area of a circular plot is twice the area of a rectangular
87. What is the percentage increase in production of CompanyA
from 2011 to 2012?

gin
plot. If the area of the rectangular plot is 11088 sq. metres., (a) 37.5 (b) 38.25
what is the perimeter of the circular plot? (c) 35 (d) 36
(a) 484 metres (b) 572 metres (e) None of these
(c) 528 metres
(e) None of these
(d) 440 metres
eer
88. For which of the following years the percentage of rise/fall
in production from the previous year the maximum for
82. The sum of the two digits of a two-digit number and the
difference between the two digits of the two-digit number is
Company B?
(a) 2011 (b) 2012 ing
8. What is the two digit number?
(a) 80
(3) 44
(b) 88
(d) Cannot be determined
(c) 2013
(e) 2015
(d) 2014

.ne
83.
(e) None of these
The total number of students studying in a college is 4220.
If the number of girls studying in the college is 2420, what is
the respective ratio of the number of boys to the number of
89. The total production of Company C in 2012 and 2013 is
what percentage of the total production of Company A in
2010 and 2011?
(a) 95 (b) 90
t
girls studying in the college? (c) 110 (d) 115
(a) 90 : 131 (b) 90 : 121 (e) None of these
(c) 121 : 70 (d) 121 : 80 90. What is the difference between the average production per
(e) None of these year of the company with highest average production and
84. In how many different ways can the letters of the word that of the company with lowest average production in lakh
'SCENIC' be arranged? tonnes?
(a) 120 (b) 720 (a) 3.17 (b) 4.33
(c) 60 (d) 360 (c) 4.17 (d) 3.33
(e) None of these (e) None of these
85. The cost of 4 bags and 12 purses is ` 1,520, what is the cost 91. The 63% of a number is 2583. What will be 45% of this
of 10 bags and 30 purses? number?
(a) ` 3,600 (b) `3,500 (a) 1845 (b) 1763
(c) `4,000 (d) `3,900 (c) 2255 (d) 1927
(e) None of these (e) None of these

Downloaded From : www.EasyEngineering.net


Downloaded From : www.EasyEngineering.net

234 SPEED TEST 100


column than the number of children seated in each row.
4 6 2 9 3
92. If the fractions , , , and be arranged in descending How many children are there in each row?
5 7 9 11 8
(a) 9 (b) 7
order, the fourth fraction will be (c) 11 (d) 13
9 4 (e) None of these
(a) (b)
11 5 Part-III : Reasoning Ability
3 6 DIRECTIONS (Qs. 101-103) : Study the following paragraph
(c) (d)
8 7 and answer the questions that follow :
(e) None of these
93. A shopkeeper purchased 245 pieces of an article at `30 per In our society, corruption and abuse of office has been
piece. He spent `980 on transport and `1470 on packing the aggravated by three factors. First, there is a colonial legacy of
articles. He sold the articles at the rate of `50 per piece. unchallenged authority and propensity to exercise power
What is the per cent profit earned? arbitrarily.
In a society which worships power, it is easy for public
(a) 25 (b) 20
officials to deviate from ethical conduct. Second, there is enormous
(c) 28 (d) 22.5
asymmetry of power in our society. Nearly 90% of our people are
(e) None of these
94.
ww
Difference between the compound interest and simple
interest accrued in two years at 8% per annum is `128.
What is the principal amount?
in the unorganized sector. Quite a number of them lead a precarious
existence, depending on subsistence wages with no job security.
And nearly 70% of the organized workers with job security and
(a) `18000
(c) `20000
(e) None of these
w.E (b) `16000
(d) Cannot be determined
regular monthly wages are employed by the state directly or
through public sector undertakings. Almost all these employees
are 'educated' in a largely illiterate and semiliterate society and
95.
asy
The number obtained by interchanging the digits of a two
digit number is less than the original number by 18. If sum
economically even the lowliest of public servants are better off
than most people in the country. What is more, their employment
in government comes with all the trappings of power. Such

En
of the digits is 6, what was the original two digit number?
asymmetry of power reduces societal pressure to conform to
(a) 51 (b) 24 ethical behaviour and makes it easy to indulge in corruption
(c) 42 (d) 15

96.
(e) None of these
Samar spends 52% of his monthly salary on household gin
101. The passage best support the statement that–
(a) Corruption is a consequence of hundred, gears slavary
of the country.
expenditure and 23% on miscellaneous expenditure. If he is
left with `4500, what is his monthly salary?
eer
(b) Among the many causes socio-economic disparity is
the main cause of corruption.

ing
(a) `16000 (b) `17500 (c) Corruption is a status symbol in our society.
(c) `17000 (d) `18500 (d) Mostly it is found that people themselves incourage
(e) None of these the corruption.
97. Number of girls in a class is 44 which is 55% of the total
number of students. How many boys are there in the class?
(e) None of these
.ne
102. Which of the following is an inference which can be made

98.
(a) 75
(c) 84
(e) None of these
(b)
(d)
76
36

Average marks obtained in History by student in ‘A’ division


from the facts stated in the above passage?
t
(a) Corruption is the consequences of can challenged
authority, asymmetry of power and illiteracy etc.
(b) People are helpless before the corruption.
are 75 and the average marks obtained by 40 students in ‘B’ (c) The government should make a strict law against the
division are 78. What are the overall average marks obtained corruption.
in History by the students in both the divisions (rounded (d) People should stir a movement against corruption.
off to two digits after decimal)? (e) None of these
(a) 76.81 (b) 77.11 103. Which of the following is the most effective statement in
(c) 77.41 (d) 76.41 the context of the above passage?
(e) None of these (a) Asymmetry of power reduces societal pressure to
99. The ratio between length and breadth of a rectangular plot conform to ethical behaviour and makes it easy to
is 5 : 3 respectively and its perimeter is 48 m. What will be its indulge in corruption.
area in sq m? (b) Society itself nourishes the corruption.
(a) 120 (b) 116 (c) Power is worshiped in our society which increases the
(c) 115 (d) 76.41 corruption.
(e) None of these (d) 90% people are in unorganised sector and they always
100. In a class of 63 children, the children are seated in rows and feel the insecurity of jobs.
columns such that there are two more children in each (e) None of these.

Downloaded From : www.EasyEngineering.net


Downloaded From : www.EasyEngineering.net

SPEED TEST 100 235


104. If ‘+’ means ‘divided by’; ‘–’ means ‘added to’; ‘×’ means 113. How many such vowels are there in the above arrangement
‘substracted from’ and ‘÷’ means ‘multiplied by’; then 26 – each of which is immediately preceded by a symbol and
15 + 5 × 4 ÷ 2 = ? immediately followed by a number ?
(a) 15 (b) 20 (a) None (b) One
(c) 25 (d) 18 (c) Two (d) Three
(e) None of these (e) More than three
105. Four of the following five are alike in a certain way and so form DIRECTIONS (Qs. 114-119) : In the following questions, the
a group. Which is the one that does not belong to that group. symbols , $, *, @ and © are used with the following meanings
(a) RPN (b) WSU illustrated.
(c) HDF (d) LHJ
(e) QMO ‘P $ Q’ means ‘P is neither equal to nor greater than Q’.
106. In a row of forty students R is the fifth from right end and ‘P © Q’ means ‘P is neither equal to nor smaller than Q’.
there are ten students between R and D. What is D’s position ‘P Q’ means ‘P is neither greater to nor smaller than Q’.
from the left end of the row ? ‘P @ Q’ means ‘P is not smaller than Q’.
(a) 26th (b) 23rd ‘P * Q’ means ‘P is not greater than Q’.
(c) 24th (d) 25th In each question three statements showing relationship have been

ww
(e) Data inadequate given, which are followed by three conclusions I, II and III.
107. In a certain code ORBITAL is written as CSPHMBU. How is Assuming that the given statements are true, find out which
CHARGER written in that code ? conclusion(s) is/are definitely true.
(a) BIDQSFH
(c) BIDQQDF
(e) None of these w.E (b) BIDSSFH
(d) DIBQSFH
114. Statements : M @ D, D K, K © R
Conclusions : I. R $ M
II. K M

asy
108. Among A, B, C, D and E each having scored different marks
in an examination, B scored more than C and E and less than
A and D. C’s marks are not the lowest. Who scored the
III. K $ M
(a) Only I is true
(c) Only III is true
(b) Only II is true
(d) Either II or III and I are true
lowest marks ?
(a) O (b) C
En (e) Either II or III is true
115. Statements : F * T, T $ N, N @ R
(c) B
(e) None of these
(d) Data inadequate

109. How many such pairs of letters are there in the word gin Conclusions : I. R $ T
II. N © F

OVERWHELM each of which has as many letters between


them in the word as in the English alphabet? eerIII. F $ R
(a) None is true (b) Only I is true

ing
(c) Only II is true (d) Only III is true
(a) None (b) One
(e) II and III are true
(c) Two (d) Three
116. Statements : B © N, N @ R, F * R
(e) More than three
110. In a certain code BROWN is written as 531 @% and MEAN
is written as 26©%. How is ROBE written in that code ?
Conclusions : I. B © M
II. F * N .ne
(a) 3@16
(c) 3156
(e) None of these
(b) 3516
(d) 3©16
III. R $ B
(a) I and II are true
(c) II and III are true
(e) None of the above
(b) I and III are true
t
(d) All I, II and III are true
DIRECTIONS (Qs. 111-113) : Study the following arrangement
117. Statements : D $ M, M * B, B J
carefully and answer the questions given below :
Conclusions : I. J © D
R5# 3$MD P14F ©A6EWJ2@ K8Q7
II. B * D
% UT I * V9
III. J $ M
111. Which of the following is the seventh to the left of the
nineteenth from the left end of the above arrangement ? (a) I and II are true (b) I and III are true
(a) D (b) U (c) II and III are true (d) All I, II and III are true
(c) © (d) (e) None of the above
(e) None of these 118. Statements : W K, K © F, F $ M
112. What should come next in the following series based on the Conclusions : I. M © K
above arrangement ? II. W @ F
53$ P4F 6WJ 8Q ? III. F @ W
(a) UI * (b) UT * (a) Only I is true (b) Only II is true
(c) UIV (d) UTV (c) Only III is true (d) II and III are true
(e) None of these (e) None is true

Downloaded From : www.EasyEngineering.net


Downloaded From : www.EasyEngineering.net

236 SPEED TEST 100


119. Statements : F @ T, T K, K * D 127. Statements : Some bikes are cars. Some cars are trains. Some
Conclusions : I. D @ F trains are buses.
II. F @ K Conclusions : I. Some buses are cars.
III. D @ T II. Some trains are bikes.
(a) II and III are true (b) I and III are true III. Some buses are bikes.
(c) I and II are true (d) All I, II and III are true (a) None follows (b) Only I follows
(e) None of the above (c) Only II follows (d) Only III follows
DIRECTIONS (Q. 120-125) : Study the following information (e) I and II follow
carefully and answer the questions given below 128. Statements : All dogs are cats. Some cats are rats. All rats
are mats.
P, A, D, Q, T, M, R and B are sitting around a circle facing at
Conclusions : I. Some mats are cats.
the centre. D is third to the left of T who is fifth to the right of P. A
is third to the right of B, who is second to the right of D. Q is II. Some mats are dogs.
second to the left of M. III. Some rats are cats.
120. Who is to the immediate right of D ? (a) Only I follows (b) Only II follows
(c) Only III follows (d) I and III follow
(a) M (b) Q
(e) None of the above
(c) B (d) Data inadequate
129. Statements : All pens are sticks. All sticks are rings. All

ww
(e) None of these
121. Who is second to the right of M ?
(a) B (b) R
rings are rods.
Conclusions : I. Some rings are pens.

w.E
II. Some rods are sticks.
(c) T (d) Q
III. Some rods are pens.
(e) None of these
(a) I and II follow (b) I and III follow
122. Who is second to the left of D ?
(c) II and III follow (d) All I, II and III follow
(a) A
(c) B
(e) Data inadequate
(b) Q
(d) P
asy (e) None of the above
130. Statements : Some tables are chairs. All chairs are houses.

123. Who is third to the right of P ?


(a) D (b) M En All houses are tents.
Conclusions : I. All houses are chairs.

gin
II. Some tents are chairs.
(c) R (d) Data inadequate III. Some houses are tables.
(e) None of these (a) I and II follow (b) I and III follow
124. Which of the following pairs represents the immediate
neighbours of A ?
(a) PT (b) PB eer
(c) II and III follow
(e) None of the above
(d) All I, II and III follow

(c) TQ
(e) None of these
(d) PD
ing
131. Statements : Some boxes are walls. No wall is road. All roads
are rivers.
Conclusions : I. Some rivers are walls.
125. In which of the following pairs the first person is sitting to
the immediate right of the second person ?
.ne
II. Some roads are boxes.
III. No wall is river.
(a) DM
(c) RA
(e) PA
(b) BT
(d) PQ

DIRECTIONS (Qs. 126-131) : In each of question below are three


(a) Only I follows
(c) Only III follows
(e) II and III follow
(d) Only II follows
t
(b) Either I or III follows

132. In a certain code MAIN is written as '9364' and DEAR is


statements followed by three conclusions numbered I, II and III. written as '8532'. How is MEND written in that code?
You have to take the three given statements to be true even if they (a) 9548 (b) 9458
seem to be at variance from commonly known facts and then (c) 9538 (d) 9528
decide which of the given conclusions logically follows from the (e) None of these
three given statements disregarding commonly known facts. Then 133. In a certain code DREAMING is written as BFSEFMHL.
decide which of the answers (a), (b), (c), (d) and (e) is the correct 'How is TREATISE written in that code?
answer and indicate it on the answer sheet. (a) USFBDRHS (b) BFSUDTHS
126. Statements : All cups are benches. Some benches are drums. (c) BFSUSHRD (d) BDQSDRHS
All drums are kites. (e) None of these
Conclusions : I. Some kites are cups. 134. The positions of how many digits in the number 5314697
II. Some kites are benches. will remain unchanged if the digits are rearranged in
III. Some drums are cups. ascending order within the number?
(a) None follows (b) Only I follows (a) None (b) One
(c) Only II follows (d) Only III follows (c) Two (d) Three
(e) II and III follow (e) More than three

Downloaded From : www.EasyEngineering.net


Downloaded From : www.EasyEngineering.net

SPEED TEST 100 237


135. Among A, B, C, D and E each having different amount of 144. A hard copy of a file created on a computer refers to data
money, C has more money than only E. B has more money (a) saved on a floppy disk
than D but less than A. Who among them has the highest (b) printed on a printer
amount of money? (c) backed up on a tape drive
(a) B (b) A (d) sent as an e-mail
(c) D (d) Data inadequate (e) None of these
(e) None of these 145. When you quickely press and release the left mouse button
136. Prakash walked 30 metres towards West, took a left turn and twice, you are
walked 20metres. He again took a left turn and walked 30 (a) Primary – clicking (b) Pointing
metres. He then took a right turn and stopped. Towards (c) Double – clicking (d) Secondary – clicking
which direction was he facing when he stopped? (e) None of these
(a) South (b) North 146. Underlined text, such as text and folder names is referred to
(c) East (d) Data inadequate as a
(e) None of these (a) icon (b) hyperlink
137. How many meaningful English words can be made with the (c) menu (d) source drive
letters RTOU using each letter only once in eachword? (e) None of these
147. To centre a paragraph using shortcut keys, press

ww
(a) None
(c) Two
(e) More than three
(b) One
(d) Three (a) CTRL + C
(c) CTRL + L
(b) CTRL + E
(d) CTRL + R

w.E
(e) None of these
138. If 'P' denotes '–'; 'Q' denotes ' ', 'R' de notes '×' and 'W'
148. Word processing, spreadsheet, and photo-editing are
denotes '+' then-
examples of
48 Q 12 R 10 P 8 W 4=? (a) application software
(a) 56
(c) 52
(b) 40
(d) 44
asy (b) system software
(c) operating system software

En
(e) None of these (d) platform software
139. Which of the following is the middle digit of the second (e) None of these
highest number among the five three-digit numbers given

gin
149. Which of the following displays to the right of the space
below ? where the text will be inserted when you type?
512 739 428 843 654 (a) Screen tip (b) Insertion point
(a) 1
(c) 2
(b) 3
(d) 4
(c) Rulers

eer
(e) None of these
(d) Office assistant

ing
(e) 5 150. In a spreadsheet, a cell is defined as the
140. In a certain code language 'green grass everywhere' is written (a) intersection of a table and a tuplet
as 'dik pa sok' and 'cow eats grass' is written as 'nok ta pa'. (b) intersection of a file and a database
How is 'cow' written in that code language?
(a) nok (b) ta
(c) intersection of a row and column
.ne
(d) intersection of a field and a record
(c) nok or ta
(e) None of these
(d) Data inadequate

141. A program designed to destroy data on your computer which


can travel to “infect” other computers is called a
(e) None of these

Part-IV : English Language


DIRECTIONS (Q. 151-160) : Read the following passage
t
(a) disease (b) torpedo carefully and answer the questions given below it. Certain words/
(c) hurricane (d) virus phrases have been printed in bold to help you locate them while
(e) None of these answering some of the questions.
142. What is the most common way to get a virus in your
There was a country long time ago where the people would
computer’s hard disk ?
change a king every year. The person who would become the
(a) By installing games from their CDROMS
king had to agree to a contract that he would be sent to an island
(b) By uploading pictures from mobile phones to the after one year of his being a king.
computer
One king had finished his term and it was time for him to go
(c) By opening e-mails to the island and live there. The people dressed him up in expensive
(d) By sending e-mails clothes and put him on an elephant and took him around the cities
(e) None of these to say goodbye to all the people. This was a moment of sadness
143. What is the main folder on a storage device called ? for all kings who ruled for one year. After bidding farewell, the
(a) platform (b) interface people took the king to a remote island in a boat and left him there.
(c) root directory (d) device driver On their way back, they discovered a ship that had sunk just
(e) None of these recently. They saw a young man who had survived by holding on

Downloaded From : www.EasyEngineering.net


Downloaded From : www.EasyEngineering.net

238 SPEED TEST 100


to a floating piece of wood. As they needed a new king, they (b) He would be gifted with expensive clothes and
picked up the young man and took him to their country. They jewellery
requested him to be king for a year. First he refused but later he (c) He would be sent on an island which required a lot of
agreed to be the king. People told him about all the rules and work to be done
regulations and that how he would be sent to an island after one (d) He would be asked to buy an elephant and go to the
year. remote island himself
After three days of being a king, he asked the ministers if (e) None of the above
they could show him the island where all the other kings were 153. What did the young man notice on his visit to the remote
sent. They agreed and took him to the island. The island was island after three days of being king?
covered with a thick Jungle and sounds of vicious animals were (a) That the animals on the island were too many to be
heard coming out of it. The king went a little bit further to check. killed
Soon he discovered dead bodies of all the past kings. He (b) That the island was very big
understood that as soon as they were left on the island, the wild (c) That the jungle on the island was full of animals that
animals had come and killed them. had killed all the previous kings placed on the island
The king went back to the country and collected 100 strong (d) That the island was very beautiful and clean
workers. He took them to the island and instructed them to clean (e) Not mentioned in the passage
the jungle, remove all the deadly animals and cut down all excess 154. What happened to the island in the first month of the young
trees. He would visit the island every month to see how the work king’s tenure?

ww
was progressing. In the first month, all the animals were removed
and many trees were cut down. In the second month, the whole
island was cleaned out. The king then told the workers to plant
(A) The wild animals were removed from the island.
(B) The whole island was cleaned.
(C) Many unnecessary trees were cut down.

w.E
gardens in various parts of the island. He also took with himself
useful animals like chickens, ducks, birds, goats, cows etc. In the
third month, he ordered the workers to build big houses and
(a) Only A
(c) Only C
(e) A and C
(b) Only B
(d) B and C

asy
docking stations for ships. Over the months, the island turned
into a beautiful place. The young king would wear simple clothes
and spend very little from his earnings as a king. He sent all the
155. What could be said about the island after the young king
had made his servants work on it ?
(a) The dangerous island had been turned to a beautiful

En
earnings to the island for storage. When nine months passed like
this, the king called the ministers and told them: “I know that I
place
(b) The island still remained the same despite all the young
have to go the island after one year, but I would like to go there
right now.” But the ministers didn’t agree to this and said that he
had to wait for another three months to complete the year. gin king’s efforts
(c) The island was barely tolerable now
(d) The island was safe from all types of attacks
Three months passed and now it was a full year. The people
dressed up the young king and put him on an elephant to take him 156.
eer
(e) None of the above
Which of the following describes the young king correctly?
around the country to say goodbye to others. However, this king
was unusually happy to leave the kingdom. People asked him,
“All the other kings would cry at this moment. Why is it that you
(A) He was intelligent
(B) He had foresight
(C) He was cunning ing
are laughing?” He replied, “Don’t you know what the wise people
say? They say that when you come to this world as a baby, you
(a) Only A
(c) Only C
.ne
(b) A and B
(d) B and C
are crying and everyone else is smiling. Live such a life that when
you die, you will be smiling and everyone around you will be
crying. I have lived that life. While all the other kings were lost
into the luxuries of the kingdom, I always thought about the future
157.
(e) All A, B and C

t
What was the king’s request to the ministers after the
completion of nine months?
(a) That he should not be sent to the island after
and planned for it. I turned the deadly island into a beautiful completion of one year
abode for me where I can stay peacefully.” (b) That he would like to go to the island immediately
151. Why did the people of the kingdom change the king every (c) That he should be paid more in order to improve work
year? on the island
(a) As their first king had invented this system and had (d) That he should be sent to the island after a year
recorded it in the form of a contract (e) None of the above
(b) As they believed that the new king would bring better 158. How did the young king arrange for money on the island?
ideas to the kingdom. (a) By selling a large amount of his property
(c) As they wanted their king to relax on an island after (b) By spending all the money derived from his income as
one year of hard work a king
(d) Not mentioned in the passage
(c) By borrowing money from the ministers and sending it
(e) None of the above
to the island
152. What would happen to the king once his term of one year
(d) By spending very less of his income as a king and
was over ?
sending it to the island for storage
(a) He would be paraded in the cities as a farewell and
(e) None of the above
then be taken to a remote island

Downloaded From : www.EasyEngineering.net


Downloaded From : www.EasyEngineering.net

SPEED TEST 100 239


159. Why were the people of the kingdom puzzled when the (C) The ducklings hurried towards the lake and the mother duck
young king was taken around the country to say goodbye began to walk back and forth dragging one wing on the
to everyone? ground.
(a) As they could not believe that one year had elapsed (D) A mother duck and her little ducklings were on their way to
so soon the lake one day.
(b) As they were nor aware that the young king was (E) The fox stared in disbelief at the mother duck and her
actually a wise sage ducklings as he could not reach the ducklings because they
(c) As the young king was happy to go to the island unlike were in the middle of the lake by now.
the previous kings (F) When the fox saw her he became happy as he thought that
(d) Not mentioned in the passage the mother duck was hurt and couldn’t fly and that he could
(e) None of the above easily catch and eat her!
160. What can possibly be the moral of the story? 166. Which of the following should be the FIRST sentence after
(a) Always put others before yourself rearrangement?
(b) Give respect to others (a) A (b) B
(c) Live in the present and forget about the future (c) C (d) D
(d) Do not put things off until tomorrow (e) E
167. Which of the following should be the SECOND sentence

ww
(e) Always think and plan ahead
after rearrangement ?
DIRECTIONS (Q. 161-163): Choose the word/group of words (a) A (b) B
which is most similar in meaning to the word/group of words (c) C (d) D

161. CONTRACT w.E


printed in bold as used in the passage.
168.
(e) F
Which of the following should be the THIRD sentence after
rearrangement?
(a) work
(c) deal
(e) get asy
(b) signature
(d) temporary (a) A
(c) C
(b) B
(d) D
162. BIDDING
(a) wishing (b) auctioning En 169.
(e) E
Which of the following should be the FOURTH sentence
(c) wasting
(e) talking
(d) playing
gin after rearrangement?
(a) B
(c) D
(b) C
(d) E
163. ABODE
(a) stop (b) mountain
170.
(e) F
eer
Which of the following should be the LAST (SIXTH)

ing
(c) plenty (d) house
sentence after rearrangement?
(e) dwelling
(a) A (b) B
DIRECTIONS (Q. 164-165) : Choose the word / group of words (c) D (d) E
which is most opposite in meaning to the word / group of words
printed in bold as used in the passage.
(e) F
.ne
DIRECTIONS (Q. 171-175) : Which of the phrases (a), (b), (c)
164. SURVIVED
(a) scratched
(c) lived
(e) suffered
(b) died
(d) fell
t
and (d) given below each sentence should replace the phrase
printed in bold in the sentence to make it grammatically correct?
If the sentence is correct as it is given and ‘No correction is
required’, mark (e) as the answer.
165. VICIOUS 171. As it was a dark and stormy night, Lata was too scared to go
(a) simple (b) small home alone.
(c) tough (d) harmless (a) very scary to (b) much scared to
(e) ferocious (c) as scared to (d) to scared too
DIRECTIONS (Q. 166-170) : Rearrange the following six (e) No correction required
sentences (A), (B), (C), (D), (E) and (F) in the proper sequence 172. Since it was her engagement party, Riya was dress to kill.
to form a meaningful paragraph; then answer the questions given (a) dresses to kill (b) dressed to kill
below them. (c) dressed to killings (d) dressing to killed
(A) All of a sudden the mother duck saw a fox in the distance, (e) No correction required
was frightened and shouted, “Children, hurry to the lake, 173. Ramesh worries endlessly about his son’s future as he was
there’s a fox !” so poor in studies.
(B) The mother duck ran, leading the fox away from the lake and (a) worry endless (b) worried endless
as soon as the fox came very close, the mother duck quickly (c) worried endlessly (d) worries endless
spread her wings and rose up in the air. (e) No correction required

Downloaded From : www.EasyEngineering.net


Downloaded From : www.EasyEngineering.net

240 SPEED TEST 100


174. Now that the actual criminal had been caught, Kunal was 181. (a) believe (b) thought
happy that he was finally let of the hook. (c) wished (d) smiled
(a) off the hook (b) of the hookings (e) rejoiced
(c) off the hooks (d) of the hooks 182. (a) make (b) let
(e) No correction required (c) pay (d) allow
175. The little boy appeared all of a sudden out of nowhere and
(e) request
take everyone by surprise.
183. (a) idea (b) luck
(a) took everyone as surprised
(c) necklace (d) cry
(b) take everyone with surprised
(e) presence
(c) took everyone by surprises
184. (a) opened (b) snatch
(d) took everyone by surprise
(e) No correction required (c) decorated (d) took
(e) ring
DIRECTIONS (Q. 176-185) : In the following passage there are
blanks, each of which has been numbered. These numbers are 185. (a) smiling (b) hurried
printed below the passage and against each, five words are (c) barking (d) much

ww
suggested, one of which fits the blank appropriately. Find out
the appropriate word in each case.
(e) friendly
DIRECTION (Qs. 186 to 190): In each of the following sentences,

w.E
A lamb was (176) with a flock of sheep one day. She soon
found some sweet grass at the (177) of the field. Farther and
farther she went, away from the others. She was enjoying herself
an idiomatic expression or a proverb is highlighted. Select the
alternative which best describes its use in the sentence.
186. He went to his friend's house in the evening as was his

asy
so much that she did not (178) a wolf coming nearer to her.
However, when it (179) on her, she was quick to start pleading,
wont.
(a) as usual (b) as he wanted

En
“Please, please don’t eat me yet. My stomach is full of grass. If
you wait a while, I will (180) much better.” The wolf (181) that
(c) as his want was
(e) none of these
(d) as he wanted that day

was a good idea, so he sat down and waited. After a while, the
lamb said, “If you (182) me to dance, the grass in my stomach will
gin
187. Why do you wish to tread on the toes?
(a) To give offence to them
be digested faster.” Again the wolf agreed. While the lamb was
dancing, she had a new (183). She said, “Please take the bell from
eer
(b) To follow them grudgingly
(c) To treat them indifferently
around my neck. If you ring it as hard as you can, I will be able to
dance even faster.” The wolf (184) the bell and rang it as hard as
he could. The shepherd heard the bell ringing and quickly sent
(d) To be kicked by them
(e) None of these ing
his dogs to find the missing lamb. The (185) dogs frightened the
wolf away and saved the lamb’s life. (a) To establish himself .ne
188. He intends setting up as a lawyer in the adjoining district.
(b) To migrate
176. (a) watching
(c) willing
(e) grazing
(b) laughing
(d) tiring
(c) To join
(e) None of these
(d) To settle

189. The autographed bat from the famous cricketer Sunil


t
177. (a) height (b) edge Gavaskar is worth a jew's eye.
(c) midst (d) first (a) Not a worthy possession
(e) base
(b) unnecessary
178 (a) notice (b) trust
(c) A costly items
(c) saw (d) worry
(d) A possession of high value
(e) maintain
(e) None of these
179. (a) lunge (b) visited
190. The speaker gave a bird's eye view of the political conditions
(c) ate (d) stand
(e) pounced in the country.
180. (a) walk (b) die (a) a personal view (b) a general view
(c) taste (d) mix (c) a biased view (d) a detailed presentation
(e) reveal (e) None of these

Downloaded From : www.EasyEngineering.net


Downloaded From : www.EasyEngineering.net

RESPONSE SHEET
Test Code : . ............................. Time taken : ......................... Date : ..........................
(a) (b) (c) (d) (e) (a) (b) (c) (d) (e) (a) (b) (c) (d) (e) (a) (b) (c) (d) (e)
1. 51. 101. 146.
2. 52. 102. 147.
3. 53. 103. 148.
4. 54. 104. 149.
5. 55. 105. 150.
6. 56.
106. 151.
7. 57.
8. 107. 152.
58.
9. 59. 108. 153.
10. 60. 109. 154.
11. 61. 110. 155.
12.
13.
14.
ww 62.
63.
64.
111.
112.
113.
156.
157.
158.
15.
16.
17.
w.E 65.
66.
67.
114.
115.
159.
160.
18.
19.
20.
68.
69.
70.
asy 116.
117.
118.
161.
162.
163.
21.
22.
71.
72. En 119.
120.
164.
165.
23.
24.
25.
73.
74.
75.
gin
121.
122.
166.
167.
26.
27.
76.
77.
123.
124.
eer 168.
169.
28.
29.
30.
78.
79.
80.
125.
126.
127. ing 170.
171.
172.
31.
32.
81.
82.
128.
129.
173.
174. .ne
33.
34.
35.
36.
83.
84.
85.
86.
130.
131.
132.
133.
175.
176.
177.
178.
t
37. 87.
38. 88. 134. 179.
39. 89. 135. 180.
40. 90. 136. 181.
41. 91. 137. 182.
42. 92. 138. 183.
43. 93. 139. 184.
44. 94. 140. 185.
45. 95.
46. 141. 186.
96.
47. 97. 142. 187.
48. 98. 143. 188.
49. 99. 144. 189.
50. 100. 145. 190.

Downloaded From : www.EasyEngineering.net


Downloaded From : www.EasyEngineering.net

242 SPEED TEST 100

ww
w.E
asy
En
gin
eer
ing
.ne
t

Downloaded From : www.EasyEngineering.net


Downloaded From : www.EasyEngineering.net

Full Main Test - 5 101


Max. Marks : 200 No. of Qs. 190 Time : 2 hr. 40 min. Date : ........./......../. ..............

Part-I : General Awareness 8. Which of the following statement is true?


(a) Banks cannot accept demand and time deposite from
1. On which one of the following issues can SEBI penalize any public.
company in India?
(b) Banks can accept only demand deposits from public.
(A) Violation of Banking Regulation Act.
(c) Banks can accept only time deposits from public.

ww
(B) Violation of foreign portfolio investment guidelines. (d) Banks can accept both demand and time deposits from
(C) For violation of Negotiable Instrument Act. public.
(a) Only (A) (b) All (A), (B) & (C) (e) Banks can accept demand and time deposits only from

2.
(c) Only (A) & (B)
(e) Only (B)
w.E (d) Only (B) & (C)

Expand the term ALM as used in Banking/Finance sector?


9.
government.
Interest payable on saving bank accounts is
(a) not regulated by RBI.
(a) Asset Liability Mismatch
(b) Asset Liability Maturity
(c) Asset Liability Management asy (b) regulated by State Governments
(c) regulated by Central Government
(d) Asset Liability Manpower
(e) None of the above En (d) regulated by RBI
(e) regulated by Finance minister.
3. What is an ISO series?
(a) Documentation of production processes
gin
10. Which of the following is the correct statement?
(a) State bank of India is the sole authority to issue and
(b) Engineering process flowchart
(c) Quality management and quality assurance standards
(d) All of the above eer
manage currency in India.
(b) A nationalized bank is the sole authority to Issue and
manage currency in India

4.
(e) None of these
The Federation of Indian Chambers of Commerce and ing
(c) A cooperative bank is the sole authority to issue and
manage currency in India.
Industry (FICCI) was founded in 1927 by
(a) Birla and Tata (b) Tata and Thakurdas in India. .ne
(d) RBI is the sole authority to issue and manage currency

t
(c) Thakurdas and Birla (d) Tata and Godrej
(e) None of these
(e) None of these
11. Accounts are allowed to be operated by cheques in respect
5. Every year March 20 is celebrated as what day?
of
(a) World Sparrow Day
(b) International Women’s Day (a) Both Savings bank accounts and fixed deposit accounts
(c) World Cuckoo Day (b) Savings bank accounts and current accounts
(d) International Child Day (c) Both Savings bank accounts and loan accounts
(e) International Mother’s Day (d) Both Savings bank accounts and cash accounts only
6. Which of the following is not a Central Government tax? (e) Both Current accounts and fixed deposit accounts
(a) Income tax (b) Customs 12. Which of the following is correct statement?
(c) Land revenue (d) Corporation tax (a) Normally no interest is paid on current deposit
(e) None of these accounts
7. The expansion for the BIS, in the context of the banking (b) Interest is paid on current accounts at the same rate as
industry is term deposit accounts
(a) Bank for International Settlements
(c) The rate of interest on current account and savings
(b) Bank for Industrial Settlements account are the same
(c) Bank for Industrial Sectors
(d) No interest is paid on any deposit by the bank
(d) Bank for International Services
(e) Savings deposits are the same as current deposits.
(e) None of these

Downloaded From : www.EasyEngineering.net


Downloaded From : www.EasyEngineering.net

244 SPEED TEST 101


13. Which of the following terms is not associated with the 23. The RV Easwar Committee gave recommendations on:
game of cricket? (a) Simplification of income tax laws
(a) China man (b) Hook (b) Goods and Services Tax in India
(c) Pull (d) Love (c) Banking reforms
(e) None of these (d) Crude oil and gas pricing
14. Larry page and Sergey Brin are well known as (e) None of these
(a) Creators of Bluetooth device
24. The RV Easwar Committee gave recommendations on
(b) Founders of Google simplification of income tax laws. . 10. Who among the
(c) Stem cell researchers following is popularly known as "Krishi ka Rishi"?
(d) Scientists (a) Subhash Palekar
(e) None of these (b) Mankombu Sambasivan Swaminathan
15. The book ‘A Bend in the River’ is written by
(c) Subrahmanyan Chandrasekhar
(a) VS Naipaul (b) Chetan Bhagat
(d) Norman Ernest Borlaug
(c) Vikram Seth (d) Arundhati Roy
(e) None of these
(e) None of these

ww
16. When a bank returns a cheque unpaid. It is called 25. "Maa Annapurna Yojna" has been launched in which state
of India?
(a) payment of the cheque
(b) drawing of the cheque (a) Odisha (b) Tamil Nadu

w.E
(c) cancelling of the cheque
(d) dishonour of the cheque
26.
(c) Gujarat
(e) None of these
(d) Kerela

What is the maximum speed of the India's first semi high-

asy
(e) taking of the cheque.
17. Distribution of Insurance products and Insurance policies speed train "Gatimaan Express"?
by banks as corporate agents is known as (a) 160 kmph (b) 170 kmph
(a) General Insurance
(c) Bancassurance
(b) Non-life Insurance
(d) Insurance banking
En (c) 180 kmph
(e) None of these
(d) 190 kmph

18.
(e) Deposit Insurance
What does the letter 'L' denote in term 'LAF' as referred
gin
27. World's first 'White Tiger Safari' has opened in which state
of India?
every now and then in relation to monetary policy of the
RBI?
(a) Liquidity (b) Liability eer
(a) Rajasthan
(c) Gujarat
(b) Madhya Pradesh
(d) West Bengal
(c) Leveraged
(e) Linear
(d) Longitudinal
28.
(e) None of these
ing
Unakoti hill,an ancient Shaivite place of worship, is located
19. POSCO is in the process of establishing its plants in India.
What does the letter 'P'. denote in the name POSCO?
in which state of India?
(a) Uttarakhand
.ne
(b) Tamil Nadu

20.
(a) Popular
(c) Pohang
(e) Petersburg
(b) Pallin
(d) Paradeep

The remuneration payable to the Attorney General of India


29.
(c) Odisha
(e) None of these
(d) Tripura

t
The Central Industrial Security Force ( CISF) Raising Day is
celebrated on which date?
is determined by? (a) February 11 (b) March 10
(a) President of India (b) Prime Minister
(c) April 26 (d) May 3
(c) Law Minister (d) Parliament
(e) None of these
(e) None of these
21. Farzad B gas field is located in which of the following 30. The world's tallest Jain statue is located in which state of
country? India?
(a) Iran (b) Oman (a) Maharashtra (b) Gujarat
(c) Saudi Arabia (d) Afghanistan (c) Karnataka (d) Uttar Pradesh
(e) None of these (e) None of these
22. "Mission Bhagiratha" is the prestigious water grid project 31. Which is the India's first digital state?
in which state of India? (a) Odisha (b) Tamil Nadu
(a) Telangana (b) Uttarakhand (c) Kerala (d) Karnataka
(c) Bihar (d) Uttar Pradesh
(e) None of these
(e) None of these

Downloaded From : www.EasyEngineering.net


Downloaded From : www.EasyEngineering.net

SPEED TEST 101 245


32. The National Science Day is celebrated on which date in 41. The World Day of Social Justice is observed on which date?
India? (a) February 20 (b) March 12
(a) March 1 (b) April 21 (c) April 23 (d) June 14
(c) February 28 (d) June 23 (e) None of these
(e) None of these 42. Which state government has launched the Pucca Ghar
33. The NDDB foundation for Nutrition (NFN) has launched Yojana for construction workers?
the 'Gift Milk' initiative in which state? (a) West Bengal (b) Gujarat
(a) Telangana (b) Bihar (c) Odisha (d) Madhya Pradesh
(c) Uttar Pradesh (d) Odisha (e) None of these
(e) None of these 43. Which of the following states is also known as 'The land of
34. The world's first fast-acting anti-rabies drug "RMAb" will dwan lit mountains'?
be launched in which country?
(a) Manipur (b) Arunachal Pradesh
(a) China (b) France
(c) Nagaland (d) Mizoram
(c) India (d) Japan
(e) None of these
(e) None of these
44. First 'New Silk Road' train from China has completed its trip
35.

ww
Who has been conferred with the Germany's prestigious
'Cross of the Order of Merit'?
(a) Arun Joshi
in which city?
(a) Tehran (b) Kabul

(c) Goverdhan Mehta


(d) Sushmita Sharma
w.E
(b) Narendra Singh Rathore

45.
(c) Islamabad
(e) None of these
(d) Ankara

The Indian Institute of Petroleum and Energy (IIPE) has

36.
(e) None of these
asy
Which company has bagged the Global CSR Excellence &
been established at which of the following city?
(a) Visakhapatnam (b) Kochi

En
Leadership Awards in the category of Best Environment-
Friendly Project?
(c) Kolkata
(e) None of these
(d) Chennai

(a) GVK Biosciences


(b) Dr. Reddy's Laboratories gin
46. Who has won the 2016 Formula 1 Gulf Air Bahrain Grand
Prix?
(c) Shantha Biotechnics
(d) Bharat Biotech
eer
(a) Kimi Raikkonen
(c) Nico Rosberg
(b) Lewis Hamilton
(d) Daniel Ricciardo

ing
(e) None of these (e) None of these
37. Nargol port is located in which state? 47. In which of the following industry CASA ratio is used?
(a) Merchandise Trade (b) Banking & Finance

.ne
(a) Maharashtra (b) Gujarat
(c) West Bengal (d) Odisha (c) Software Industry (d) Travel & Tourism
(e) None of these

t
(e) None of these
38. The Khajuraho Dance Festival is celebrated in which state 48. Which among the following is a opposite activity of hedging
of India? in share / currency /future markets?
(a) Rajasthan (b) Uttar Pradesh (a) arbitrage (b) speculation
(c) Madhya Pradesh (d) Chhattisgarh (c) spread (d) short
(e) None of these (e) tall
39. Which of the following committee is related to rejuvenation 49. Which among the following correctly categorizes
of Godavari river? collateralized borrowing and lending obligation?
(a) Ashok Kumar committee (a) A market operation
(b) Nandan Rai committee (b) A money market instrument
(c) Rekha Somalia committee (c) A clearing system
(d) Deepak Naik committee (d) A scheme of Reserve Bank of India
(e) None of these (e) A policy initiative of Government of India
40. India's first ever Gender Park has been inaugurated in which 50. Which of the calling is the oldest joint stock Bank of India
state? (a) Allahbad Bank (b) Bank of Baroda
(a) Odisha (b) Assam (c) Patiala (d) Bank of India
(c) Kerala (d) Tripura
(e) None of these
(e) None of these

Downloaded From : www.EasyEngineering.net


Downloaded From : www.EasyEngineering.net

246 SPEED TEST 101


Part-II : Quantitative Aptitude 61. Which of the following fractions is the least ?
12 1
DIRECTIONS (Qs. 51-69): What will come in place of the question (a) (b)
119 10
mark (?) in the following questions ?
5 4 3 4 7
(c) (d)
51. of of of 222 = ? 39 69
8 9 5
(a) 42 (b) 43 (e) None of these
(c) 39 (d) 37 62. A number of points are marked on a plane and are connected
(e) None of these pairwise by a line segment. If the total number of line
52. 56% of 450 + ? = 300 segments is 10, how many points are marked on the plane ?
(a) 52 (b) 48 (a) 4 (b) 10
(c) 42 (d) 56 (c) 5 (d) 9
(e) None of these (e) None of these
63. A sum of money becomes eight times in 3 years if the rate is
53. 271.5 273.5 27?
compounded annually. In how much time, the same amount
(a) 5 (b) 7 at the same compound interest rate will become sixteen

ww
(c) 3 (d) 2
times?
(e) None of these
(a) 6 years (b) 4 years
54. 27.06 × 25 – ? = 600
(c) 8 years (d) 5 years
(a) 76.3
(c) 76.5
(e) None of these w.E (b) 76.7
(d) 76.2
64.
(e) None of these
A machine is sold at a profit of 10%. Had it been sold for
` 40 less, there would have been a loss of 10%. What was
55. 4
7
2
8 13
4
?
asy the cost price ?
(a) ` 320 (b) ` 200

(a) 11
1
3
(b) 11
1
13 En (c) ` 225
(e) None of these
(d) ` 250

(c) 11
4
(d) 11
3
gin
65. Ram spends ` 3620 for buying pants at the rate of ` 480
each and shirts at the rate of ` 130 each. What will be the
ratio of pants to shirts when maximum number of pants are
13
(e) None of these
8
to be bought ?
(a) 7 : 2 eer (b) 7 : 3

56. 84 1
83
85 2
8 ?
8
(c) 2 : 7
(e) None of these
ing (d) 4 : 5

(a) 7
(c) 3
(b) 2
(d) 4
66.

.ne
Two trains each of 120 m in length, run in opposite directions
with a velocity of 40 m/s and 20 m/s respectively. How long
will it take for the tail ends of the two trains to meet each

57.
(e) None of these
– (a – b) × ? = b – a
(a) – 1
(c) – a
(b) 1
(d) a
other during the course of their journey ?
(a) 20 s
(c) 4 s
(b) 3 s
(d) 5 s
t
(e) None of these
(e) None of these 67. Ramesh is twice as good a workman as Sunil and finishes a
58. (a + b) = ? × (– a – b) piece of work in 3 hours less than Sunil. In how many hours
(a) 1 (b) – a they together could finish the same piece of work ?
(c) – 1 (d) –b
(e) None of these 1
(a) 2 (b) 2
59. |? + 14| = 11 3
(a) –3 (b) –25 2
(c) 25 (d) 3 (c) 1 (d) 8
3
(e) Either – 3 or –25
(e) None of these
60. 16 + 26 × 2 =?
68. Fifteen years hence, a man will be four times as old as he
(a) 84 (b) 44
was fifteen years ago. His present age is:
(c) 40 (d) 832
(a) 25 years (b) 20 years
(e) None of these
(c) 30 years (d) 45 years
(e) None of these

Downloaded From : www.EasyEngineering.net


Downloaded From : www.EasyEngineering.net

SPEED TEST 101 247


69. The floor of a rectangular room is 15 m long and 12 m wide.
3 1 1
The room is surrounded by a vrandah of width 2 m on all its 80. 1 2 7 ?
sides. The area of the vrandah is : 5 7 3
(a) 124 m2 (b) 120 m2 (a) 17 (b) 3
(c) 108 m 2 (d) 58 m2 (c) 9 (d) 29
(e) None of these (e) 25
DIRECTIONS (Qs. 70-74) : Find the next term in the given series 81. The height of four boys is recorded as 142 cms., 156 cms.,
in each of the questions below. 162 cms. and 178 cms. What is the average height of all four
70. 2, 4, ?, 16, 32 boys?
(a) 6 (b) 10 (a) 160 cms. (b) 158.5 cms.
(c) 8 (d) 12 (c) 159.5 cms. (d) 162 cms.
(e) None of these (e) None of these
71. 0, 7, 26, ?, 124, 215 82. Pratul's monthly income is onefourth of Manoj's monthly
(a) 37 (b) 51 income. Manoj's annual income is ` 2.16 lacs. What is
(c) 63 (d) 88 Pratul's annual income? (In some cases monthly income and
(e) None of these

ww
in some cases annual income are used.)
72. 4, 15, 16, ?, 36, 63, 64
(a) ` 54.000 (b) ` 5.4 thousand
(a) 25 (b) 30
(c) ` 4.500 (d) ` 45.000

w.E
(c) 32 (d) 35
(e) None of these
(e) None of these
73. 1, 8, 9, ?, 25, 216, 49 DIRECTIONS (83-85) : What will come in place of the question
mark (?) in the following number series?

asy
(a) 60 (b) 64
(c) 70 (d) 75 83. 17 98 147 172 181 (?)
(e) None of these (a) 180 (b) 192
74. 336, 210, 120, ?, 24, 6, 0
(a) 40 (b) 50 En (c) 184
(e) None of these
(d) 182

(c) 60
(e) None of these
(d) 70
gin
84. 11 19 31 47 67 (?)
(a) 80 (b) 81
DIRECTIONS (Qs. 75-82): Find out the approximate value which
is closest to the value that should replace the questions mark (?)
(c) 86
eer
(e) None of these
(d) 96

ing
in the following questions. (You are not expected to find out the
85. 748 460 316 244 208 (?)
exact value.)
(a) 180 (b) 190
75.

.ne
1223.9975 ? (c) 172 (d) 182
(a) 110 (b) 144 (e) None of these
(c) 34 (d) 12.55

76.
(e) 125
503 × 201 = ?
(a) 101100 (b) 1000000
the questions that follow:
Number of candidates (in lakhs) appearing in an entrance
t
DIRECTIONS (Q. 86-90): Study the tables carefully to answer

examination from six different cities and the ratio of candidates


(c) 110000 (d) 100003
passing and failing the same
(e) 1000103
77. 1205 2.5 = ? City A B C D E F
(a) 3000 (b) 4800 Number of
1.25 3.14 1.08 2.27 1.85 2.73
(c) 300 (d) 480 Candidates
(e) 500 Ratio of candidates passing and failing within the city.
78. 22020 0.011 = ?
City Passing Failing
(a) 20020 (b) 2002000
A 7 3
(c) 200200 (d) 20002
B 5 3
(e) 2000020
C 4 5
79. 20800 ? D 1 3
(a) 12 (b) 120 E 3 2
(c) 140 (d) 102 F 7 5
(e) 1020

Downloaded From : www.EasyEngineering.net


Downloaded From : www.EasyEngineering.net

248 SPEED TEST 101


86. What is the ratio of number of candidates failing the exam 96. If the value of x + y = 18 and xy = 72, what is the valueof (x)2
from City D to that of those failing the exam from City A? + (y)2 ?
(a) 289 : 42 (b) 42 : 289 (a) 324 (b) 54
(c) 227 : 50 (d) 50 : 227 (c) 180 (d) Cannot be determined
(e) None of these (e) None of these
87. The number of candidates appearing for the exam from City 97. Rashmi obtained a total of 484 marks out of 750 in an
examination. What is her approximate percentage in the
C is what per cent of the number of candidates appearing
examination?
for the exam from City B? (rounded off to the nearest integer)
(a) 61 (b) 56
(a) 27 (b) 34 (c) 72 (d) 65
(c) 42 (d) 21 (e) 70
(e) 38 98. What approximate amount of compound interest can be
88. The number of candidates passing in the exam from City F is obtained on an amount of `5000 at the rate of 3% per annum
what per cent of the total number of candidates appearing at the end of 3 yr?
from all the cities together ? (rounded off to two digits after (a) `482 (b) `464
the decimal). (c) `450 (e) `425
(a) 12.93 (b) 14.46 (e) `478

ww
(c) 10.84 (d) 11.37 99. Girish attends to 7 customer calls daily. Approximately, how
(e) None of these many calls would he attend to in the span of two months?
(a) 8400 (b) 420
89. Which city has the highest number of students failing the

w.E
(c) 217 (d) 7012
entrance exam?
(e) 336
(a) F (b) C 100. Two number are such that the sum of twice the first number
(c) B (d) D and thrice the second number is 126 and the sum of thrice

90.
(e) None of these
asy
What is the number of candidates passing the exam from
the first number and twice the second number is 144. What
is the smaller number?

En
City E? (a) 18 (b) 24
(a) 13,000 (b) 11,10,000 (c) 32 (d) 36
(e) None of these

gin
(c) 1,13,000 (d) 11,000
(e) None of these
91. Sitaram invested an amount of `7450@ 6% per annum rate Part-III : Reasoning Ability
of simple interest. After how many years will he obtain the
total amount of `8791?
eer
101. In a certain code language ‘tree is very beautiful’ is written
as ‘ka na da ta’ and ‘this is stong tree’ is written as ‘na pa sa

ing
(a) 5 yr (b) 3 yr ka’. How is beautiful written in that code language?
(c) 2 yr (d) 6 yr (a) da (b) ta
(e) None of these (c) sa (d) Data inadequate
92. The difference between 36% of a number and 12% of the
same number is 82.32. What is the number?
(e) None of these
.ne
102. In a certain code ‘GIVE’ is written as 51@© and ‘FAIL’ is

93.
(a) 324
(c) 343
(e) None of these
(b) 382
(d) 336

The cost of 16 watches and 21 calculators is 30480. What is


the cost of 32 watches and 42 calculators?
written as ‘% 219’. How is LEAF written in that code?
(a) 5©2%
(c) 9@2%
(e) None of these
(b) 9©2%
(d) 9©1%
t
(a) `60970 (b) `60480 103. The positions of the first and the sixth digits in the number
(c) `80960 (d) Cannot be determined 5109238674 and interchanged, similarly the positions of the
(e) None of these second and the seventh digits are interchanged and so on.
94. If (66)2 added to the square of a number, the answer so Which of the following will be the third digit from the right
obtained is 4840. What is the number? end after the rearrangement?
(a) 32 (b) 28 (a) 9 (b) 0
(c) 24 (d) 18 (c) 6 (d) 3
(e) None of these (e) None of these
95. A sum of money is divided amongst A, B and C in the ratio 104. Four of the following five are alike in a certain way and so
of 3 : 4 : 5. Another amount is divided amongst P and Q in form a group. Which is the one that does not belong to that
the respective ratio of 2 : 1. If Q got `1050 less than B, what group?
is the amount received by C? (a) Wheel (b) Tyre
(a) `2850 (b) `1000 (c) Car (d) Door
(c) `1840 (d) Cannot be determined
(e) Gear
(e) None of these

Downloaded From : www.EasyEngineering.net


Downloaded From : www.EasyEngineering.net

SPEED TEST 101 249


105. In a certain code SUBSTANCE is written as RATRUFDOB. DIRECTIONS (114-118): In the following questions, the symbols
How is TENTHOUSE written in that code? @, S, , © and # are used with the following meaning as
(a) SMDSIFTVP (b) UOFUIDRTN illustrated below:
(c) UOFUIFTVP (d) SMDSIDRTN ‘P Q’ means ‘P is neither greater than nor smaller than Q’.
(e) None of these
‘P S Q’ means ‘P is neither greater than nor equal to Q’.
DIRECTIONS (106-108): Following questions are based on the ‘P @ Q’ means ‘P is not smaller than Q’.
five three digit numbers given below: ‘P © Q’ means ‘P is not greater than Q’.
519 364 287 158 835
‘P # Q’ means ‘P is neither smaller than nor equal to Q.’
106. If the positions of the first and the third digits within each
number are interchanged, which of the following will be the Now is each of the following questions assuming the given
third digit of the second lowest number? statements to be true. Find which of the three conclusions I, II
(a) 9 (b) 4 and III given below them is/are definitely true and give your answer
(c) 7 (d) 8 accordingly.
(e) 5 114. Statements: W @ T, T © M, M S D
107. If the positions of the first and the third digits within each Conclusions: I. W # D
number are interchanged, which of the following will be the II W @ M

ww
middle digit of the second highest number?
(a) 1
(c) 8
(b) 6
(d) 5
III. D # T
(a) Only I is true

(e) 3
w.E
108. Which of the following is the difference between the second
digits of the highest and the lowest of these numbers?
(b) Only II is true
(c) Only III is true
(d) Only II and III are true
(a) 3
(c) 2
(b) 1
(d) 0 asy (e) None of these
115. Statements: F R, R © M, M S D
(e) None of these
DIRECTIONS (109-113) : Study the following information
En Conclusions: I. D # R
II. D # F
carefully and answer the questions given below:
A, M, P, D, Q, R, W and B are sitting around a circle facing at the
centre. D is fourth to the left of A who is third to the right of M. P gin III. M @ F
(a) Only I and II are true

is third to the left of Q who is third to the left of M. R is third to the


right of W who is second to the right of B. eer
(b) Only I and III are true
(c) Only II and III are true

ing
(d) All I, II and III are true
109. Who is second to the left of D?
(e) None of these
(a) W (b) B
116. Statements: V © M, M B, B S F
(c) Q
(e) None of these
(d) Data inadequate
Conclusions: I. F # M
II. B @ V .ne
110. Who is third to the left of P?
(a) M
(c) R
(e) None of these
(b) D
(d) Data inadequate
III. F # V
(a) Only I and II are true
(b) Only II and III are true
t
111. Who is to the immediate right of Q? (c) Only I and III are ture
(a) W (b) D (d) All I, II and III are true
(c) B (d) Data inadequate (e) None of these
(e) None of these
117. Statements: D # N, N @ B, B F
112. Which of the following pairs represents the first and second
Conclusions: I. F S D
respectively to the right of W?
II. N # F
(a) DM (b) QB
III N F
(c) MR (d) Data inadequate
(e) None of these (a) Only I is true
113. In which of the following pairs is the second person sitting (b) Only II is true
to the immediate right of the first person? (c) Only III is true
(a) MD (b) RM (d) Only either II or III is true
(c) AB (d) QB (e) Only I and either II or III are true
(e) None of these

Downloaded From : www.EasyEngineering.net


Downloaded From : www.EasyEngineering.net

250 SPEED TEST 101


118. Statements: R S T, T # K, K @ M Conclusions :
Conclusions: I. R S M I. No bag is key.
II. T # M II. Some bags are keys.
III. R S K III. Some toys are keys.
(a) None follows (b) Only I follows
(a) None is true (b) Only I is true
(c) Only II follows (d) Only III follows
(c) Only II is true (d) Only III is true
(e) Only I and II follow
(e) Only II and III are true 125. Statements :
DIRECTIONS (Qs. 119-123): Study the following arrangement Some days are nights.
carefully and answer the questions given below: Some nights are months.
W3# R@ EJ KT 4B 91 DU8 1 H% AV 5 7MP 2Q S6 Some months are years.
119. Which of the following is the sixth to the right of the twenty Conclusions :
first from the right end of the above arrangement? I. Some years are nights.
(a) 8 (b) D II. Some months are days.
(c) P (d) @ III. No year is night.
(a) Only I follows (b) Only II follows

ww
(e) None of these
120. How many such vowels are there in the above arrangement (c) Only III follows (d) Only either I or II follows
each of which is immediately preceded by a symbol and (e) None of these

(a) None
(c) Two
w.E
immediately followed by a consonant?
(b) One
(d) Three
126. Statements :
All cycles are tyres.
Some tyres are wheels.

(e) Four
asy
121. How many such symbols are there in the above arrangement,
All wheels are buses.
Conclusions :
I. Some buses are tyres.

immediately preceded by a number?


En
each of which is immediately followed by a number but not
II. Some wheels are tyres.
IIi. Some buses are cycles.
(a) None
(c) Two
(b) One
(d) Three
gin (a) Only I and II follow (b)
(c) Only II and III follow (d)
Only I and III follow
All I, II and III follow
(e) more than three
122. How many such consonants are there in the above 127.
eer
(e) None of these
Statements :

ing
arrangement, each of which is immediately preceded by a Some dogs are cats.
number and immediately followed by a symbol? Some cats are horses.
(a) None (b) One All horses are tigers.
(c) Two
(e) More than three
(d) Three Conclusions :
I. Some tigers are cats. .ne
123. Four of the following five are alike in a certain way based on
their positions in the above arrangement and so form a group.
Which is the one that does not belong to that group?
(a) 8 1 D (b) 7 5
II. Some horses are dogs.
III. Some tigers are dogs.
(a) None follows
(c) Only II follows
(b)
(d)
Only I follows
Only III follows
t
(c) P 2 7 (d) E J R (e) Only II and III follow
(e) T 4 J 128. Statements :
All ropes are sticks.
DIRECTIONS (124-128) : In each of the questions below are
given three statements followed by three conclusions numbered Some sticks are hammers.
I, II and III. You have to take the given statements to be true even Some hammers are lakes.
if they seem to be at variance from commonly known facts. Read Conclusions :
all the conclusions and then decide which of the given conclusions I. Some lakes are ropes.
logically follows from the given statements disregarding
II. Some hammers are ropes.
commonly known facts.
III. Some lakes are sticks.
124. Statements :
(a) None follows (b) Only I follows
All keys are locks.
(c) Only II follows (d) Only III follows
No lock is toy.
(e) Only I and III follow
All bags are toys.

Downloaded From : www.EasyEngineering.net


Downloaded From : www.EasyEngineering.net

SPEED TEST 101 251


DIRECTIONS (129 – 130) : In each of the questions given below 132. Which of the following statement strengthens the
which one of the five answer figures on the right should come conclusion of paragraph?
(a) Political parties should patch-up the disputes amongst
after the problem figures on the left, if the sequence were
them.
continued?
(b) Election disputes should be disposed the parliament
129. Problem Figures or concerned house of states.
C C S S = (c) The supreme Court should establish a special judicial
Z S C = S authority for the disposal of election petitions.
S Z = C Z (d) Existing system is appropriate for the disposal of
= = Z Z C election-petitions.
Answer Figures (e) None of these
133. Which of the following statement weakens the conclusion
= = S S = of the paragraph?
S Z = = Z (a) The government should follow the high level
Z S C C S committees and eminent persons.
C C Z Z C (b) The government should follow the recommendation
(a) (b) (c) (d) (e) of NCRWC.

ww
130. Problem Figures
C = = C
(c) The new system may be unsuccessful so, the
government should continue the existing system.

S
=
= C
w.E C =
(d) Fast disposal of election petitions is necessary.
(e) None of these
134. In a certain code OMNIBUS is written as SUBINMO. How
Answer Figures

D CD D C asy =
is TROUBLE written in that code?
(a) ELBUORT
(c) EBOURLT
(b) ELRTBOU
(d) ELBTROU
=C = = =C
C En (e) None of these
135. How many such pairs of letters arethere in the word
(a) (b) (c) (d) (e)
gin CORPORATE each of which has as many letters between
them in the word as in the English alphabet?
DIRECTIONS (Qs. 131-140) : Study the following paragraph
and answer the questions that follow :
(a) None
(c) Two
eer (b) One
(d) Three

ing
Election petitions in India are at present to be filed in the (e) More than three
High Court. Under the Representation of the People Act, such 136. If it is possible to make only one meaningful word with the
petitions should be disposed of within a period of 6 months. In first, second, fifth, and sixth letters of the word
actual practice however, such petitions remain pending for years
and in the meanwhile, even the full term of the House expires thus
.ne
'EDUCATION', which would be the last letter of the word?
If more than one such word can be formed, give X as the
rendering the election petition infructuous. There have been
suggestions from other high level committees and eminent persons
that a separate judicial set-up may be required. The National
Commission to Review the Working of the Constitution (NCRWC)
answer. If
no such word can be formed, give K as your answer.
(a) T
(c) A
(b) X
(d) E
t
recommended that special election benches should be constituted
(e) K
in the High Courts earmarked exclusively for the disposal of
137. Rohan walks a distance of 10 km towards North, then turns
election petitions.
to his left and walks 20 km. He again turns left and walks 10
131. Which of the following conclusion which can be drawn
km and then, he takes a right turn and walks 5 km. How far
from the facts stated in the above paragraph?
he is from the starting point?
(a) The election commission should itself. make the
(a) 10km (b) 20 km
disposal of election disputes.
(c) 30 km (d) 25 km
(b) The election commission should hand-over the
(e) 15 km
disputes to an authority.
138. If 'a' means '–', 'B' means '×', 'A' means ' 'and 'F' means '+',
(c) The election petitions remain pending for years while
then-
it should be disposed within six years.
(d) To fast disposal of election petitions a separate judicial 12 B 20 A 4 Q 10 F 30 = ?
set-up should be made or special benches should be (a) 60 (b) 80
constituted in the High Courts. (c) 70 (d) 90
(e) None of these (e) None of these

Downloaded From : www.EasyEngineering.net


Downloaded From : www.EasyEngineering.net

252 SPEED TEST 101


139. The positions of the first and eighth digits of the number (a) repeater (b) router
92753481 are interchanged. Similarly, the positions of second (c) gateway (d) switch
and the seventh digits are interchanged and so on, which of (e) None of these
the following will be the fifth digit to the left of the second 150. Something which has easily understood instruction is said
digit from the right end after the rearrangement? to be
(a) 8 (b) 2 (a) user friendly (b) information
(c) 4 (d) 1 (c) word processing (d) icon
(e) 3 (e) None of these
140. What should come next in the following letter series?
Z Y X W V U T Z Y X W V U Z YX W V Z Y X W Part-IV : English Language
(a) V (b) Y
(c) Z (d) U DIRECTIONS (Q. 151-165): Read the following passage carefully
(e) None of these and answer the questions given below it. Certain words/ phrases
141. Most of the commonly available personal computers / have been printed in bold to help you locate them while answering
laptops have a keyboard popularly known as some of the questions.
(a) QWERTY (b) QOLTY Long ago there was a poor Brahmin named Krishnan. He

ww
(c) ALTER
(e) None of these
(d) UCLIF

142. What are the default file extension for all word documents ?
could not find enough work to do. Sometimes, he and his family
had to go without food. At last Krishnan decided to leave his
village in search of work. Early next morning, he left the house. He
(a) WRD
(c) DOC w.E (b) TXT
(d) FIL
walked the whole day until he came to a thick jungle. He was tired,
thirsty and hungry. While looking around for water to drink, he
found a well. He went to the well and looked in. There he saw a

asy
(e) None of these jaguar, a monkey, a snake and a man. They had all fallen into the
143. Manipulating data to create information is known as well. “O, noble Brahmin”, the jaguar called out to him, “Please
(a) feedback (b) programming help me out, so that I can go back to my family.”
(c) processing
(e) None of these
(d) analysis
En “But you are a jaguar”, said Krishnan. “How do I know you
will not kill me?” “Don’t be afraid of me, I promise I will not do you
144. A _________ is an organised colection of data about a
single entity. gin
any harm”, replied the jaguar. Krishnan reached into the well and
pulled out the jaguar. The jaguar thanked him and said, “I’m
(a) file
(c) database
(b) library
(d) dictionary eer
Shersingh. I live in a cave in the mountains. I shall be most delighted
if I can repay my debt to you someday.” Krishnan then heard the
monkey calling out to him from the well. The Brahmin at once
(e) None of these
145. The horizontal and vertical lines on a worksheet are called ing
pulled the monkey out. The monkey thanked the Brahmin. “If you
are ever in need of food, just drop in at my place below that big
(a) cells
(c) blocking
(b) sheets
(d) gridlines
.ne
mountain. Bali is my name.” Now the snake called out to him for
help. “Help you!” exclaimed Krishnan. “You are a snake. What if
(e) None of these
146. _________ is when the computer is turned on and the
operating system is loading.
(a) Booting (b) Flashing
t
you bite me?” “I shall never bite you”, said the snake. So Krishnan
pulled the snake out of the well. The snake said, “Remember, if
you are ever in any difficulty, just call out my name-Naagesh, and
wherever you are, I shall find you.” The jaguar, the monkey and
the snake took leave of the Brahmin. But before they left, they
(c) Tracking (d) Taping
spoke to him about the man in the well. “Please do not help him,”
(e) None of these said Shersingh. “If you do”, said Naagesh, “you will be in trouble
147. To print a document, press ________ then press ENTER yourself.” As soon as they left, the man in the well began to call
(a) SHIFT + P (b) CTRL + P out for help. Krishnan felt sorry for the man and pulled him out of
(c) ALT + P (d) ESC + P the well. “Thank you for your kindness”, said the man. “I am Seth
(e) None of these Ghanshyamdas. I am a goldsmith. If you ever need my help, don’t
148. Which of the following are valid Min. & Max. zoom sizes in hesitate to visit my humble house near the city.” The goldsmith
MS Office then left for home.
(a) 10, 100 (b) 20, 250 After some time, the Brahmin continued his journey. But he
could not find any work. He then remembered Shersingh, Bali,
(c) 10, 500 (d) 10, 1000
Naagesh and Seth Ghanshyamdas. He thought it was time to
(e) None of these seek their help. He first went to Bali. The monkey was overjoyed
149. If you wish to extend the length of the network without to see him. He gave him a warm welcome and offered him some
having the signal degrade, you would use a really delicious fruits. The Brahmin told him how grateful he was.

Downloaded From : www.EasyEngineering.net


Downloaded From : www.EasyEngineering.net

SPEED TEST 101 253


Now Krishnan went to see Shersingh, the jaguar. As soon as (c) As his village people had asked him to leave their village
Shersingh saw Krishnan coming, he ran out to welcome him. He and look for work somewhere else.
gave Krishnan a beautiful gold necklace and other precious (d) As he wanted to search for food in a village different
jewellery. Krishnan thanked Shersingh for the jewellery and from his own.
departed. His journey had at last brought him luck, he thought. He (e) None of the above
would be able to sell the ornaments for a good price. But who could 152. Why did the jaguar, the monkey and the snake tell Krishnan
help him to sell the ornaments? He then remembered Seth not to save the man in the well?
Ghanshyamdas. He went to him. The goldsmith was glad to see (a) As the man in the well was a goldsmith
Krishnan. “I have come to ask for your help”, said Krishnan. “Here (b) As the man in the well had cheated the snake, the
are some ornaments. Please give me a good price for them.” Seth monkey and the jaguar
Ghanshyamdas took the jewellery and examined it carefully. “I shall (c) As the man in the well was a thief
certainly help you”, he said. “But let me show them to another (d) As the snake, the monkey and the jaguar hated the man
goldsmith. Please wait here, I will be right back.” He then went out as they had known him for a very long time
with the ornaments. Seth at once rushed to the Palace of the King. (e) None of the above
He said, “A man brought these ornaments to me and asked me to 153. Why was krishnan afraid to save Naagesh from the well?
sell them. But they are the ornaments I made for the Prince who is (a) As Naagesh had threatened him with dire
missing.” “Who is this man? Where is he?”, thundered the King. consequences.

ww
This rogue must have murdered my little Prince and robbed his
jewels!” “He is a Brahmin named Krishnan, your Majesty”, replied
(b) As he thought Naagesh would eat him.
(c) As he thought Naagesh would bite him once he was
out of the well.

w.E
the goldsmith, and he is there, in my house. The king called for his
most dreaded soldiers. “Arrest the Brahmin who is in the goldsmith’s (d) As he thought that Naagesh would capture him as soon
house and throw him into the darkest dungeons of the kingdom”, as he got out of the well.
roared the King. The King’s guard stormed into the goldsmith’s (e) None of the above.

asy
house and seized Krishnan. Krishnan was thrown into a dark
dungeon to await his execution. He then remembered the words of
154. Why did Krishnan go to meet Seth Ghanshyamdas?
(a) As he thought that Seth Ghanshyamdas could help

En
Naagesh, the snake. So he called out to him. him in selling the ornaments gifted to him by Shersingh.
(b) As he knew that Seth Ghanshyamdas had contact with
Suddenly, almost like magic, Naagesh slithered his way
the King which could prove to be beneficial.
down a narrow window into the dingy cell. “O, Lord!” hissed
Naagesh, “how did you manage to get yourself arrested?”
Krishnan cried and then told the snake what had happened. “I gin (c) As Seth Ghanshyamdas had requested krishnan to sell
ornaments only to him
have a plan”, hissed Naagesh. “I shall creep into the Queen’s
room and bite her”, said Naagesh. “She will faint. No matter what
155. eer
(d) As Krishnan was extremely fond of Seth Ghanshyamdas
(e) None of the above.
What did Bali do after seeing Krishnan at his house?
they do, she will remain asleep. The poison will remain in her
body until you place your hand on her forehead”, explained
Naagesh. He then left Krishnan and went to the palace. He crept ing
(1) He gave Krishnan directions to Shersingh’s house.
(2) He welcomed Krishnan to his house.
into the Queen’s room and bit her. The Queen fainted. The sad
news that the Queen had been bitten by a snake spread all over
(a) Only 1 (b) Only 2
.ne
(3) He offered tasty fruits to Krishnan.

t
(c) Only 3 (d) Only 2 and 3
the Kingdom. Vaidyas came from far and near, but their medicines
(e) 1 and 3
had no effect. No one could revive the Queen. Finally, the King
156. What plan did Naagesh have to save Krishnan from the
declared that anyone who could cure the Queen would be
dungeon?
handsomely rewarded. Many people went to the palace but all of
(a) That he would sneak Krishnan out of the dungeon
them failed. “I can cure the Queen”, Krishnan told the guards. At
without anyone noticing
once they took him to the Queen. Krishnan sat beside the Queen
(b) That he would bite the King and make him unconscious
and placed his hand on her forehead. Soon, she opened her eyes
(c) That he would bite Krishnan and make everyone believe
and sat up. The King was overjoyed and shed tears of happiness. that he was dead
He embraced Krishnan and thanked him. “Your Majesty”, said (d) That he would enter the Queen’s chamber and scare her
Krishnan. “I was sent to prison for a crime I did not commit.” (e) None of the above
Krishnan told the King the whole story. The King was fuming 157. What did Seth Ghanshyamdas tell the King about Krishnan?
with rage when he heard what the goldsmith had done. He at once (a) That Krishnan had brought fake ornaments for selling
had the goldsmith arrested. The King then presented Krishnan (b) That krishnan was an honest Brahmin who had left his
with a large house and a thousand pieces of gold. Krishnan sent
village
for his family and they all lived happily ever after.
(c) That Krishnan had killed the Prince
151. Why did Krishnan decide to leave his village?
(d) That Krishnan had brought those ornaments for selling
(a) As he could not find much work in his own willage and
his family had to starve sometimes because of it. which had been made for the missing Prince
(b) As his family had requested him to do so. (e) None of the above

Downloaded From : www.EasyEngineering.net


Downloaded From : www.EasyEngineering.net

254 SPEED TEST 101


158. What did the King do on learning the truth about Krishnan DIRECTIONS (Q. 166-170): Which of the phrases (a), (b), (c)
and Seth Ghanshyamdas?
and (d) given below each sentence should replace the phrase
(a) He put Krishnan back in the dungeon as he still held
printed in bold in the sentence to make it grammatically correct?
Krishnan responsible for the Prince’s death
If the sentence is correct as it is given and no correction is
(b) He called for Krishnan’s wife and family
required, mark (e) as the answer.
(c) He presented gold to Krishnan and also a house to live in
166. A young and successful executive was travelling down a
(d) He congratulated the snake on his efforts to save
neighbourhood street, going a bit to fast in his new car.
Krishnan
(a) a bit too fastly (b) a bit as fast
(e) None of the above
(c) a bit to fastly (d) a bit too fast
159. What did the King do to save the Queen after even the
(e) No correction required
Vaidyas failed to revive her? 167. All she could think about was the beautiful dress and how
(a) He punished the snake for having harmed the Queen she could earn enough money to buy it.
(b) He announced a reward to anyone who could cure the (a) All she can think (b) All she could thought
Queen (c) All she can thought (d) All she can thinking
(c) He immediately called for Krishnan to cure the Queen (e) No correction required

ww
(d) He asked his guards to immediately look for someone
who could cure the Queen
168. He told his employer of his plans to leave the business to
lead a more leisure life.

w.E
(e) None of the above (a) more leisurely life (b) many leisurely life
160. What can possibly be the moral of the story? (c) many leisured life (d) more leisurely live
(a) Trust oneself before trusting overs (e) No correction required
(b) A good deed never goes in vain

asy
(c) You cannot change people but you can change yourself
169. Padma could convince anyone with her talk as she had the
gift of the gabbing.
(a) gifting of the gabbing (b) gift of the gab
(d) Try and try until you succed

En
(e) One must be the change one wishes to see in this world
(c) gifting of the gab
(e) No correction required
(d) gift of the gab

DIRECTIONS (Q. 161-163): Choose the word/group of words


which is most similar in meaning to the word/group of words gin
170. For countries undergoing a recession, large cuts in public
spending seem to be the ordering of the day.
printed in bold as used in the passage.
161. GO eer
(a) be the ordering of days
(b) being the order of the day
(a) report
(c) send
(b) live
(d) leave ing
(c) be the order of the day
(d) being the ordering of days

.ne
(e) depart (e) No correction required
162. HUMBLE DIRECTIONS (Q. 171-175): In each question below, a sentence
(a) elegant
(c) modest
(e) vast
163. SEEK
(b) polite
(d) real
t
with four words printed in bold type is given. These are numbered
as (a), (b), (c) and (d). One of these four words printed in bold may
be either wrongly spelt or inappropriate in the context of the
sentence. Find out the word which is wrongly spelt or
(a) hunt for (b) watch for inappropriate, if any. The number of that word is your answer. If
(c) mention (d) ask for all the words printed in bold are correctly spelt and also
(e) force appropriate in the context of the sentence, mark (e) ie. ‘All
DIRECTIONS (Q. 164-165): Choose the word/group of words correct’ as your answer.
which is most opposite in meaning to the word/group of words 171. The whole (a)/ time she walked with her child in her arms,
printed in bold as used in the passage. the only thing (b)/ that worried (c)/ her was her son’s
164. CONTINUED feature. (d)/ All correct (e)
(a) remanded (b) presented 172. When the young artist returned (a)/ to his village, his family
(c) rested (d) carried on held a festive (b)/ dinner on its lawn to celebrate his
(e) stopped triumpant (c)/ homecoming. (d)/ All correct (e)
165. HANDSOMELY 173. Had she not suppressed (a)/ all the details of her Company’s
(a) Meagrely (b) tidily project (b)/ her Company would have bagged (c)/ the
(c) ugly (d) raggedly contract. (d)/ All correct (e)
(e) plenty

Downloaded From : www.EasyEngineering.net


Downloaded From : www.EasyEngineering.net

SPEED TEST 101 255


174. She trusted Mira with all her heart (a)/ and thus handled 183. (a) more (b) scene
(b)/ over her life’s (c)/ savings to her instantly. (d)/ All (c) whole (d) last
correct (e). (e) lucky
175. It is difficullt (a)/ to see the picture (b)/ when you are inside 184. (a) servants (b) mother
(c)/ the frame. (d)/ All correct (e)
(c) computers (d) relatives
DIRECTIONS (Q. 176-190): In the following passage there are (e) man
blanks, each of which has been numbered. these numbers are 185. (a) minds (b) selves
printed below the passage and against each, five words are (c) property (d) pillars
suggested, one of which fits the blank appropriately. Find out the
(e) country
appropriate word in each case.
One day a father of a very wealthy family (176) his son on a DIRECTIONS (Qs. 186-190): In each of the following sentences,
trip to the country with the purpose of (177) his son how the poor an idiomatic expression or a proverb is highlighted. Select the
people live so he could be thankful for his wealth. They spent a alternative which best describes its use in the sentence.
(178) of days and nights on the farm of what would be considered
186. He resigned the post of his own accord.
a (179) poor family. On their (180) from the trip, the father asked

ww
his son, “How was the trip?" "It was great, Dad.” “Did you see
how poor people can be?”, the father asked. “Oh yeah”, said the
(a) which he liked
(b) according to his convenience

w.E
son. So what did you (181) from the trip?”, asked the father. The (c) voluntarily and willingly
son answered, “I saw that we have one dog and they had four. (d) according to his judgement
We have a pool that (182) in the middle of our garden and they (e) None of these

asy
have a creek that has no end.” “We have imported lanterns in our
garden and they have the stars at night.” “Our patio reaches to
the front yard and they have the (183) horizon.” “we have a small
187. As a politician he is used to being in the limelight all the
time.

En
piece of land to live on and they have fields that go beyound our
sight.” “We have (184) who serve us, but they serve others.”
(a) giving speeches
(b) the object of admiration
“We buy our food, but they grow theirs.” “We have walls around
our (185) to protect us; they have friends to protect them.”
gin (c) the centre of attraction
(d) an object of public notice
With this the boy’s father was speechless. Then his son
added, “Thanks dad for showing me how poor we are”.
eer
(e) None of these
188. I ran out of money on my European tour.

ing
176. (a) took (b) beat
(a) exhausted my stock of
(c) drag (d) mould
(b) did not have enough
(e) showed
177. (a) presenting
(c) tell
(b) requesting
(d) trusting
(c) lost
(d) carried a lot
.ne
(e) showing
178. (a) two
(c) much
(b) couple
(d) few
(e) None of these
t
189. Madhuri might scream blue murder, but I feel Deepali
should get the promotion since she is better qualified for
the job.
(e) many
179. (a) major (b) some (a) someone has been murdered with some blue liquid
(c) sorrow (d) very (b) someone is being murdered and has become blue
(e) astutely (c) suffer from persecution complex
180. (a) lane (b) journey (d) make a great deal of noise and object vehemently
(c) leave (d) return (e) None of these
(e) walking 190. In modern democratic societies lynch law seems to have
181. (a) reveal (b) think become the spheres of life.
(c) saw (d) believe (a) law of the mob
(e) learn (b) law of the underworld
182. (a) stands (b) reaches (c) law of the constitution

(c) swims (d) leak (d) law of the parliament

(e) watery (e) None of these

Downloaded From : www.EasyEngineering.net


Downloaded From : www.EasyEngineering.net

256 SPEED TEST 101

RESPONSE SHEET
Test Code : . ............................. Time taken : ......................... Date : ..........................
(a) (b) (c) (d) (e) (a) (b) (c) (d) (e) (a) (b) (c) (d) (e) (a) (b) (c) (d) (e)
1. 51. 101. 151.
2. 52. 102. 152.
3. 53. 103. 153.
4. 54. 104. 154.
5. 55. 105. 155.
6. 56. 106. 156.
7. 57. 107. 157.
8. 58. 108. 158.
9. 59. 109. 159.
10. 60. 110. 160.
11. 61. 111. 161.
12.
13.
14.
ww 62.
63.
64.
112.
113.
114.
162.
163.
164.
15.
16.
17.
w.E 65.
66.
67.
115.
116.
117.
165.
166.
167.
18.
19.
20.
68.
69.
70.
asy 118.
119.
120.
168.
169.
170.
21.
22.
71.
72. En 121.
122.
171.
172.
23.
24.
25.
73.
74.
75.
gin
123.
124.
125.
173.
174.
175.
26.
27.
76.
77.
126.
127. eer 176.
177.
28.
29.
30.
78.
79.
80.
128.
129.
130. ing 178.
179.
180.
31.
32.
81.
82.
131.
132.
181.
182. .ne
33.
34.
35.
36.
83.
84.
85.
86.
133.
134.
135.
136.
183.
184.
185.
186.
t
37. 87. 137. 187.
38. 88. 138. 188.
39. 89. 139. 189.
40. 90. 140. 190.
41. 91. 141. 191.
42. 92. 142. 192.
43. 93. 143. 193.
44. 94. 144. 194.
45. 95. 145. 195.
46. 96. 146. 196.
47. 97. 147. 197.
48. 98. 148. 198.
49. 99. 149. 199.
50. 100. 150. 200.

Downloaded From : www.EasyEngineering.net


Downloaded From : www.EasyEngineering.net

ww
w.E
asy
En
gin
eer
ing
.ne
t

Downloaded From : www.EasyEngineering.net


Downloaded From : www.EasyEngineering.net

ww
w.E
asy
En
gin
eer
ing
.ne
t

Downloaded From : www.EasyEngineering.net


Downloaded From : www.EasyEngineering.net

SOLUTIONS 1
SPEED TEST 1 19. (c) First term = 32 – 1.
Second term = 33 + 1
1. (a) A square is a two-dimensional figure consisting of sides Third term = 42 – 1
whereas a cube is a three- dimensional figure. Similarly, \ Fourth term = 43 + 1
circle is a two-dimensional figure and a sphere is a 20. (b) First term = 23.
three-dimensional figure. Second term = 32
B R I G H T Third term = 43
\ Fourth term = 52
+1 –1 21. (d) Second term = 4 × First term
2. (e) \ Fourth term = 4 × Third term
J S C S G F 22. (a) Second term = (First term)2 – 1
Similarly, \ Fourth term = (Third term)2 – 1
23. (e) Second term = (First term)3
J O I N E D \ Fourth term = (Third term)3.
24. (a) Second term = First term + 1/8 First term.
+1 –1
\ Fourth term = Third term + 1/8 Third term.
25. (e)
J P K C D M 26. (c) 1 and 4 are what happens after a disease. 5 is its
3.

4.
(a)

(d)
ww
The second number is the product of the digits of the
first.
The first is found in the form of the second.
symptom but not a definite one. 2 is a probable cause.
27. (c) The first is the force fighting on/in the second.
28. (e) More of a test of your English.
5. (c)
w.E
In first term, two letters are missing between first two
letters while last two are continous but in second term
first two letters are continuous and two letters are
29. (c) Secretly is the opposite of openly, and silently is the
opposite of noisily. Choices a and b are clearly not the
opposites of silently. (Choice d) means the same thing

6. (a)
missing between last two letters.
asy
The third letter of second term is the next letter
according to alphabet to the third letter of first term.
as silently.
30. (b) A spring forms or has the shape of a coil, and a ring
forms a loop.
7.
8.
(a)
(d)
The letters of each group are in reverse order.
En
First two letters of the first term are in reverse order in SPEED TEST 2

9. (b)
the second term and so are the next two letters.
Fifth and third letters of the first term are first and
second letters of the second term and first two letters
gin
1. (a) +2
H ¾¾® J ¾¾® G
+1
-3

-3
of the first term are third and fourth letters of the second
term. eer
P ¾¾® Q ¾¾® N
+1
D ¾¾® E ¾¾® B
-3

10. (a) There is a gap of one letter between third and fourth,
fourth and first, and first and second letters of each
group.
+1
T ¾¾® U ¾¾® R
+1
K ¾¾® L ¾¾® I ing
-3

-3

11. (a) The letters in first and second terms are in reverse order
of alphabet.
2.
3.
(e)
(b)
Lotus is grown in water (Mud).
P R O B L E M .ne
12. (e)

13. (d)
There is a gap of one letter between each corresponding
letters of 'QYGO' and 'SAIQ'
There is a gap of three letters between each
corresponding letters of 'YAWC' and 'UESG'.
¯
2
B
¯
9
O
¯
4
R
¯ ¯ ¯
8 3 7 5
E M O
¯

E P
t
14. (c) There is a gap of two letters between the two ¯ ¯ ¯ ¯ ¯ ¯ ¯ ¯
consecutive letters of each term. ;
8 4 9 7 5 9 (4) 7 2
15. (e) First, Second, and third each term is one more than the
L B O R O M E P
square of prime number. Hence the fourth term
¯ ¯ ¯ ¯
= (19)2 + 1 ;¯ ¯ ¯ ¯
16. (a) First term = (6)2 + 6. 3 8 4 9 4 5 7 2
Second term = (7)2 + 7 E R O L
Third term = (10)2 + 10
¯ ¯ ¯ ¯
\ Fourth term = (11)2 + 11
17. (d) First term = 32 + 3. 7 9 4 3
Second term = 42 + 4 4. (e)
Third term = 52 + 5 5. (d) River is a water body.
\ Fourth term = 62 + 6 6. (d) Except 255 all other numbers are one more than perfect
18. (c) First term = 22 – 1 square.
Second term = 32 + 1 50 = (7)2 + 1, 65 = (8)2 + 1;
Third term = 32 – 1 170 = (13)2 + 1, 290 = (27)2 + 1
\ Fourth term = 42 + 1 But, 255 = (16)2 – 1

Downloaded From : www.EasyEngineering.net


Downloaded From : www.EasyEngineering.net

2 101 SPEED TEST


7. (e) The number 49 is a perfect square of a natural 23. (d) In each pair if first letter is mth from the beginning of
number. alphabet the second letter is mth from end.
8. (e) Except Brinjal, all others grow underground. 24. (e) In all others,
9. (d) All others are parts of a car. 1st letter + 1 = 4th letter.
10. (c) Except 529, all others are perfect squares of even 4th letter + 1 = 2nd letter
numbers. The number 529 is a perfect square of an odd
and 2nd letter +2 = 3rd letter
number.
196 = 14 × 14; 256 = 16 × 16 25. (c) In all others
529 = 23 × 23; 576 = 24 × 24 4th letter + 1 = 1st letter
324 = 18 × 18 1st letter + 2 = 2nd letter
-2 -2 2nd letter + 1 = 3rd letter.
11. (a) R ¾¾® P ¾¾® N 26. (b) All others are synonyms.
-4 +2 27. (d) All others are synonyms.
W ¾¾® S ¾¾® U
28. (a) All others imply ‘UP’.
-4 +2
H ¾¾® D ¾¾® F 29. (c) ‘Large’ is an adjective whereas others are noun.
-4 +2
30. (d) All others are negative.
L ¾¾® H ¾¾® J

12. (b)
ww -4
Q ¾¾® M ¾¾® O
+2

The number 441 is a multiple of 3


SPEED TEST 3

13. (e) -3

-3
w.E
P ¾¾® M; E ¾¾® B;

T ¾¾® Q; I ¾¾® F;
-3

-3
1. (a) 1050

0
¸ 2.5
6
420 168
¸ 2.5
24
¸ 2.5
60
67.2 26.88
¸ 2.5
120
¸ 2.5
210
10.752

?
+3
V ¾¾® Y asy 2. (e)
+6 +18 +36 +60 +90 +126
14. (d) 115 = 5 × 23 ;
85 = 5 × 17;
95 = 5 × 19; En +12 +18 +24 +30 +36
155 = 5 × 31;
But, 75 = 5 × 15 gin
3. (a)
\ ? = 210 + 126 = 336
The pattern of the series is :
19 – 15 = 4 = 22
15. (e)
One factor of 75 is not a Prime Number.
Except number 345, all other numbers are product of 23
and a Prime Number. eer
83 – 19 = 64 = 43
119 – 83 = 36 = 62
631 – 119 = 512 = 83
115 = 23 × 5;
161 = 23 × 7
253 = 23 × 11;
4. (c) ing
\ ? = 631 + 102 = 631 + 100 = 731
The pattern of the series is :
391 = 23 × 17
But 345 = 23 × 15.
19 + 1 × 7 = 19 + 7 = 26
26 + 2 × 7 = 26 + 14 = 40
40 + 4 × 7 = 40 + 28 = 68 .ne
16. (c)
The number 15 is not a Prime Number.
-2
O ¾¾® M ¾¾® Q
-2
H ¾¾® F ¾¾® J
+4

+4
5. (d)
68 + 8 × 7 = 68 + 56 = 124
124 + 16 × 7 = 124 + 112 = 236
t
The pattern of the number series is as given below:
11 × 1 – 1 = 10
10 × 2 – 2 = 18
-4
T ¾¾® P ¾¾® R
+2 18 × 3 – 3 = 51
51 × 4 – 4 = 200
-2 +4 200 × 5 – 5 = 995
T ¾¾® R ¾¾® V
\?= d
-2 +4 6. (b) The pattern of the number series is as given below:
V ¾¾® T ¾¾® X
14 + 10 = 24
17. (c) In all others, 1st letter – 1 = 2nd letter, and 2nd letter 24 + 19 (= 10 + 9) = 43
– 2 = 3rd letter. 43 + 28 (= 19 + 9) = 71
18. (a) Except Diabetes, all others are infectious diseases. 71 + 37 (= 28 + 9) = 108
19. (a) Except Mustard, all others are grains. Mustard is an 108 + 46 (= 37 + 9) = 154
oilseed. 7. (e) The pattern of the number series is as given below:
20 (d) All the numbers are multiples of 5. But 25 is a perfect 144 + 29 = 173
square. 173 – 33 = 140
21. (d) All others are synonyms 140 + 29 = 169
169 – 33 = 136
22. (c) All others are synonyms
136 + 29 = 165

Downloaded From : www.EasyEngineering.net


Downloaded From : www.EasyEngineering.net

SOLUTIONS 3
8. (a) The pattern of the number series is as given below: 25. (e) The letters are in reverse order while one letter is
656 missing between two consecutive letters.
+ 24 = 328 + 24 = 352 26. (b) There is a gap of one letter between two consecutive
2
letters. Besides, this the letters are capital and lower
352 respectively.
+ 24 = 176 + 24 = 200
2 27. (b) m m l l
mmll
200
+ 24 = 100 + 24 = 124 mmll
2
28. (e) 1, 12, 123, 1234, 12345, 123456, 123456 7
124
+ 24 = 62 + 24 = 86 29. (c) ABCD, ABCDE, ABCDEF, PQRS, PQRST, PQRST U
2
86 30. (e) N + 3 = Q, Q + 3 = Z, Z + 3 = S
+ 24 = 43 + 24 = 67 D – 2 = W, W – 2 = F, F – 2 = V
2
9. (b) The pattern of the number series is as given below: P + 3 = B, B + 3 = R, R + 3 = I Hence, ? = SVI
12 × 4 – 30 = 48 – 30 = 18
18 × 4 – 36 = 72 – 36 = 36 SPEED TEST 4

ww
36 × 4 – 42 = 144 – 42 = 102
102 × 4 – 48 = 408 – 48 = 360
360 × 4 – 54 = 1440 – 54 = 1386 1. (e) E X C U R S I O N
10. (c) 71 78 99

w.E 134 183 246


+(7 × 1) +(7 × 3) +(7 × 5) +(7 × 7) +(7 × 9)
342 337.5 328.5 315 297 274.5
2.

3.
(c)

(a)
G O L D E N

11. (b)

12. (c) 161 164 179 242


asy
–(4.5× 1) –(4.5 × 2) –(4.5 × 3) –(4.5 × 4) –(4.5 × 5)
497 1520
4.
S T R I V E
(b) Meaningful words are % TAN and ANT
C L I E N T
+3 +15 +63 +255 +1023
En 5. (a)
C E I L N T
(3 × 4)+3 (15 × 4)+3 (63 × 4)+3 (255 × 4)+3
13. (b) 3 + 32, 5 + 52, 7 + 72.
14. (a) 92 + 10, (19)2 + 20, (29)2 + 30 15. (e) gin
6.
7.
8.
(c) ELAN, LEAN and LANE
(b) Meaningful words are % ARE, ART, ATE
(a) Given word : A R G U M E N T
100 × 1.1, 200 × 2.2, 300 × 3.3, 400 × 4.4 16.
(b) + 12 – 02, + 32 – 22, + 52 – 42, + 72 – 62. eer
Alphabetically the sequence is
A E G M+1 N+1 R+1 T+1 U+1
17. (e) × 3 + 1, × 3 + 3, × 3 + 5, × 3 + 7 . ......
18. (e) The given series is ¸ 2 – 6
Reqd no. = 50 ¸ 2 – 6 = 25 – 6 = 19
+1 +1

B F H N O S U V
ing
+1

19. (d) (1015 + 1) ¸ 2 = 508;


(508 + 2) ¸ 2 = 255; (255 + 3) ¸ 2 = 129;
(129 + 4) ¸ 2 = 66.5; (66.5 + 5) ¸ 2 = 35.75;
9. (d) New order of letters : GANCARROE

.ne
10. (c) According to english alphabet, resultant group will be
as follows:
(35.75 + 6) ¸ 2 = 20.875
20. (a) The series is
× 4, ¸ 8, × 12, ¸ 16, × 20
21. (d) The series is
P R I N C E
C E I N P R
t
Only two letters 'I and N' will remain unchanged.
–1.1, –2.2, –4.4, –8.8, –17.6
22. (e) The series is 11. (e) Meaningful words are : ROSE, SORE, EROS and ORES.
+112, + 122, + 132, + 142, + 152. 3 15 14 19 20 1 2 12 5
23. (d) The series is abcab, bcabc, cabca. 12. (d) C O N S T A B L E
24. (c) First two letters of each term are in reverse order.
Similarly third and fourth letters are also in reverse order. 13. (c) P H Y S I C A L
Besides this, second letter of the second term is the
next letter after the first letter of the first term. Second 14. (d) S H I F T E D
Method -
15. (b) Meaningful words are : GO and TO
–1 –1 –1 –1 –1 –1 –1 –1
16. (a) Meaningful words are : TON, TOE, TEN
D C X W, F E V U, H G T S, ...
J ...
I ...
R Q... 17. (b) According to order of alphabet
–1 –1
C E F I L M R U
+1 B D E H K L Q T
+1
+1 18. (d) Meaningful word are : TIRE, TIER and RITE.

Downloaded From : www.EasyEngineering.net


Downloaded From : www.EasyEngineering.net

4 101 SPEED TEST


19. (a) D I S T A N C E 5. (b) As, Similarly,
+2
N ¬¾¾ L ü G ¾¾® I
-2
C J R S B M B F ï +2
-2
A ¬¾¾ Y ï L ¾¾® N
B B C F J M R S +2
X ¬¾¾ V ïï R ¾¾® T
-2
20. (d) FOLK ® FKLO ® EJKN +2
-2
A ¬¾¾ Y ï C ¾¾® E
21. (c) MULE = 1 + 3 + 3 + 2 = 9 ýÞ +2
22. (e) Meaningful words are : TEARS, STARE, RATES and -2
L ¬¾¾ J ï E ¾¾® G
+2
I ¬¾¾ G ï P ¾¾® R
ASTER. -2
23. (b) Word : WEBPAGE ï +2
-2
T ¬¾¾ R ï Y ¾¾® A
Changed word : +2
-2 ï R ¾¾® T
VFAOBFF E ¬¾¾ C þ +2
C ¾¾® E
So, F appears thrice.
6. (a) Going through information provided, we get codes for
24. (c) Words : TIPS, SPIT and PITS.
G ® 3, R ® 8, E ® 1, C ® 9.
25. (b) SKILL, KILLS Therefore, Greece will be coded as 381191.
26. (c) Original word : 7. (b) CAT ® SATC & DEAR ® SEARD
HABITUAL

ww
Changed word :
GBAJSVBK
So, fourth from the left is J.
Clearly, the first letter is transferred to last place and S
taken its place. In the same way SINGS would be coded
as SINGS.

27. (c) Word :


WA LK I N G
Alphabetical order :
w.E 8.

9.
(a)

(d)
PROMISE
1 2 3 4 567
Þ MISER will be coded as 45672.
In the code, night is called sunshine. As we sleep in
A G I K LN W
asy
So, the positions of K and N remain unchanged. 10. (a)
night, the correct answer is sunshine.
Position of D alphabetically = 4
Position of F alphabetically = 6
28. (d) Clearly, we have :
COMPREHENSION ® (COM) (PREHENS) (ION)
® COMIONSNEHERP En Thus D + F = 10 = position of J in alphabet.
(d) ‘ 2 4 7 ’ ® ‘spread red carpet ’

29. (d)
The middle letter is the seventh letter, which is S.
gin
11.

‘ 2 3 6 ’ ® ‘dust one carpet ’

D E P R E S S I O N eer
‘ 2 3 4 ’ ® ‘ one red carpet ’
Hence, ‘2’ ® ‘carpet’; ‘4’ ® ‘red’; ‘7’ ® ‘spread’;
E D
1 2
R P
3 4
S E
5 6
I S
7 8
N O
9 10
ing
‘3’ ® ‘one’; ‘6’ ® ‘dust’.
12. (b) A person sits on a chair. Since ‘chair’ is called ‘cot’, our

30. (a) Random, Restaurant, Restrict, Robber, Rocket.


answer is ‘cot’.
O V E R V I S T.ne
1.
SPEED TEST 5
(c) Second and fourth letters of the word PROSE are moved
two steps backwards in alphabatical order. Similarly,
13. (a) $ # % * # +

From above table, SORE is coded as :


´ –

S
´
O
$
t
R
*
E
%
LIGHT can be coded as LGGFT.
14. (a) As,
2. (d) Z = 52 = 26 × 2
P U L S E and N E W
ACT = 1 × 2 + 3 × 2 + 20 × 2 = 48 [Alphabetical position
numbers has been -1 -1 -1 -1 -1 -1 -1 -1
doubled] O T K R D M D V
Þ BAT = 2 × 2 + 1 × 2 + 20 × 2 = 46 reverse order reverse order
3. (c) Here, 2 ® A, 3 ® L, 5 ® G,, 4 ® U and 9 ® T..
D R K T O V D M
Hence, 23549 will be having the code ALGUT. Similarly,
4. (a) RBM STD BRO PUS º the cat is beautiful ...(i)
P R O B E S
TNH PUS DIM STD º the dog is brown ...(ii)
-1 -1 -1 -1 -1 -1
PUS DIM BRO PUS CUS º the dog has the cat ....(iii)
(i) and (ii) Þ STD PUS º is O Q N A D R
(ii) and (iii) Þ PUS DIM º the dog reverse order
(i) and (iii) Þ PUS BRO º the cat R D A N Q O
\ From (iii), CUS º has Hence, required code : RDANQO

Downloaded From : www.EasyEngineering.net


Downloaded From : www.EasyEngineering.net

SOLUTIONS 5
15. (b) As, 21. (a) As,
R E M I T +2
D ¾¾ ®F +2
I ¾¾ ®K
+2 and
O ¾¾ ®Q +2
N ¾¾ ®P
£ 3 7
Similarly,
and
+2
C O N S U L A ¾¾ ®C
+2
T ¾¾ ®V
= % 8 b $ 5 22. (b) W R O M B T ® 7 1 9 4 8 3
Similarly, 23. (b) As,
+1
P ¾¾® and B ¾¾® C
+1
O C E L O T Q
-1 -1
I ¾¾® H O ¾¾ ®N
% = £ 5 % 7 +1
N ¾¾®
+1
O L ¾¾ ®M
16. (c) -1 -1
K ¾¾® T ¾¾ ®S
1
A ww
2
M
3
O
4
N
5
G ®
4
N
1
A
3
O
5
G
2
M
J
Similarly,
1
S
Hence,
2
P
3
I
4
N
w.E
5
E ®
4
N
1
S
3
I
5
E
2
P
+1
M ¾¾
-1
U ¾¾
+1
S ¾¾
®N
®T
®T
1
L
2
A
3
M
4
O
5
N ®
4
O asy
1
L
3
M
5
N
2
A
-1
T ¾¾ ®S

17. (a) S E A L and D O S E


En 24. (b) As,
L O C K
¯ ¯ ¯ ¯
$ 7 5 @
¯
#
¯ ¯ ¯
8 $ 7
gin +1

M
+1

P
-1

B
-1

J
Hence,
S O L D
and
B eerL O W
¯
$ 8 @ #
¯ ¯ ¯
+1

C
+1

M
-1

N
ing
-1

18. (a) Teacher write on blackboard with chalk, here chalk is


called book, hence here the code of chalk is book.
Similarly,
W I N E
.ne
t
+1 +1 -1 -1
19. (e) As,
X J M D
M O T H E R
25. (b) SOLDIER JFSCRNK
+1 +1 +1
+1
O M H U R F +1
–1
Similarly, –1
–1
A N S W E R –1

+1 +1 Similarly,

N A W T R F GENIOUS PVTHFDM
+1
G R O W W I T H I N +1
+1
20. (a) and –1
= @ % # –1
# ÷ + © ÷ r –1
W I N G –1

26. (e) The colour of blood is red and here red means orange.
# ÷ r =

Downloaded From : www.EasyEngineering.net


Downloaded From : www.EasyEngineering.net

6 101 SPEED TEST


7. (c) The only son of Vineet’s grandfather is Vineet’s
(27-28) : colours of the sky father. The lady is the daughter of Vineet’s father.
Thus, the lady in the park is the sister of Vineet.
Þ ki la fa so 8. (a) As X is the son of Y’s father and Y is the sister of X and
thus, he has to be the brother of Y.
rainbow colours Þ ro ki Father
9. (b)
Only Daughter
sky high rocket Þ la pe jo
Monika
the rocket world Þ pe so ne
Mother
27. (d) colours sky high Þ ki la jo
28. (c) The code of ‘the’ is ‘so’. Rahul
D R E A M I N G 10. (a)
29. (e) +1 –1 Daizy Binny Aruna
B F S E F M H L (Binny’s sister (Binny’s sister)

ww
Similarly,
T R E A
+1
T I S E
–1
and Chinky’s Mother)

30.
B F S U
w.E
D R H S
(b) A person sits on a chair. Since ‘chair’ is called ‘cot’, our
answer is ‘cot’. 11. (d)
Chinky

SPEED TEST 6 asy Saroj


mother
Rajesh
father
1. (c) Clearly, Arun is uncle of the lady in the photograph.
En
Arun — Brother
|
Son + Wife mother = lady gin Vani
Sister-in-law
Deepak
only brother
Ramesh

|
daugher
eer wife

Mother — B
|
A-C 12. (c) ing
2.

3.
(a)

(b)
A’s mother = B’s sister
Clearly, A can be a neice of B.
The only son of Mahesh’s father is Mahesh himself. .ne
4. (d)
Father of Kamla is Mahesh and Mahesh is father of
Kamla.
Clearly, the grandson of Anil’s mother is son of Anil
and wife of Anil’s son is daughter in-law of Anil. Thus,
13. (c)

Mot
h er-in-
law
t
Mother/Photograph

Anil is the father-in-law of the girl.


5. (b) Clearly, from the relationship diagram. Y is the brother- Husband Sister Man Brother Wife Son
in-law of B.
Ne

Married couple
A B
ph
ew

Son
Brother Brother-in-law
Brother
X Y
Brothers For Qs. 14 to 16
6. (c) Woman’s Mother’s husband
(Couples) (Couples)
Roma Mohan Madhu Jeevan
(Couples) (Couples)
Smita Devika Aman Krishna Sunil
Woman’s father
Woman’s father’s sister ¾¾® Woman’s Aunt. Daughter
Anuj Ankur
Since, woman’s aunt is man’s aunt Romila
\ woman is sister of man. There are in all four married couples.

Downloaded From : www.EasyEngineering.net


Downloaded From : www.EasyEngineering.net

SOLUTIONS 7
14. (a) Since, Devika is wife of Aman and Krishna is sister 25. (a) R(+) V(–) S
of Aman, therefore, Krishna is the sister-in-law of
Devika.
15. (d) Since, Anuj is son of Krishna and Romila is daughter
T(+)
of Aman and Aman is brother of Krishna, therefore
Anuj and Romila are cousins. Hence V is the aunt of T.
16. (c) Since, Madhu is mother of Krishna who is wife of 26. (c) K(+) T(–)
Sunil. Therefore, Madhu is Sunil’s mother-in-law.
For Qs. 17-20.
The information given in the question can be
summarised as: D(+) B M(+)

(Father) Wife (Mother)


For Qs. 27 to 30
E D
Adhir Mishra Mr. & Mrs. Mohan
Married
Urmila Raghu Sumit Roma Roshan Bimla Shilla
(eldest) (youngest)

ww Wife Sons
C B (Son)
Mr. & Mrs Sharma
Sohan Shivendar
Daughter

F w.E A
Son/daughter
Shilla Sandeep

Leela (daughter)
Shaifali

17. (c)
18. (d)
C is the mother to A and F.

therefore (d). asy


Since we do not know whether A is male or female, 27.
28.
(b) Sumit’s mother-in-law = Mrs. Mohan.
(c) As Sohan is son of Sumit and Sumit is son of Mishra.
19. (d) We cannot tell from the given facts whether A is a male
or female.
En 29.
The surname of Sohan is Mishra.
(a) Leela is the grand daughter of Mr. Sharma. Hence, the
20. (b)

21. (e)
The two couples in the question are DE (grandmother
and grandfather) and BC (father and mother).
S gin
30.
surname of Leela is Sharma.
(d) Shivendar is son of Roma. Therefore, he is grandson
of Roma’s father.

eer SPEED TEST 7


T(–) R(+)
Hence T is the daughter of S.
V(–)
1.
ing
(c) Clearly, I am to the north of my house.

.ne
22. (b) V(–) 5 N

t
5 W E
T(–) R(+) S
10 5
23. (d) T(+) R(–)
End Point S

Starting point
V(+) S (My house)
S is either the nephew or niece of T.
2. (b) 70 m
24. (c) T(+) R(–) T(–) V(+)(–) S
100 m
B' A'
A B
V(+) S R(+) 50 m 10 30 My friend
My self 10
(1) (2)
20
S is nephew or niece of T. Absurd relationship.
When my friend reaches on the previous track
T(–) S(+) (i.e. on B') again, he had travelled a distance of
(30 + 10 + 20 + 10) = 70 m. As I walk with the same speed
R(+) V(+)
as that of my friend I have walked 70m, but on the
(3) straight track. Now, he is just [100 – (30 + 20)] = 50m
S is husband of T. from my starting point.
We need not go further. Hence, the distance between us = (70 – 50) = 20m

Downloaded From : www.EasyEngineering.net


Downloaded From : www.EasyEngineering.net

8 101 SPEED TEST


3. (a) Clearly the direction of the hour’s hand is North-east. 8. (d) Left of Kunal is Atul.
North East of Atul is Prashant (obvious from the above
H N diagrams).
2 3 4 9. (c) Total distance travelled = 25 + 40 + 60 + 90 = 215 mts.
1 5
12 6 M W E 10. (d) West North
8 km/h
Rohan
B A
S
3 N
1
90 m
W E
4. (b) 3
3
S C D
1 Rahul
Required distance = 3 + 1 = 4 km 10 km/h
South East
5. (a) Here, O is the starting point.

ww Rohan takes the route BADCB at 8 km/hr =


20
9
m/sec.

4
w.E 3
Rahul takes the route CDABC at 10 km/hr =
AB = BC = CD = DA = 90 m.
First time they meet when Rohan travels 120 m and
25
9
m/sec.

A 3 O 4
asy B Rahul travels 150 m.
For the second time, Rohan is 10 m towards B from C or
Both A and B are 32 + 42 = 5 km from the starting
point.
En 11. (a)
Rahul is 80 m towards C from B. Hence on BC at a
distance of 10 m from C.
6. (b) Clearly the school is in north-east
5 gin E

40
School N N

eer S

Ram’s house W E W
ing Reeta Kavita

25
S
.ne
7. (c)
West
t
Kavita’s shadow is right to Reeta i.e., Kavita’s shadow
is left to Kavita.
\ Kavita is facing North.
12. (a) 45°+180° = 225° clockwise direction
River is finally flowing in east direction. 270° anticlockwise direction.
For (Qs. 8-9) 225 – 270° = – 45°
Prashant 1st ent
vem
Mo

180°
30m
45°
Initial
Position 45°
Nitin Atul
25m 40m Kunal
Mo Fina 2nd
60m vem l Movement
en
t

i.e., 45° anticlockwise from initial position.


Dinesh Hence, the required direction is south-west.

Downloaded From : www.EasyEngineering.net


Downloaded From : www.EasyEngineering.net

SOLUTIONS 9

13. (d) 50 m 18. (b) P O


N
20 m
10m

10 km
10 km
30 m P
Bhavika W E
Sunaina
50 m Q 100 m

6 km S
70 m
19. (a) 15 m
When Bhavika is at P, Sunaina who is walking at the N
same rate will be at Q which is at a distance of 70 m from
Sunaina’s point of start.

10 m
\ Required distance between them is PQ W E
PQ = (70 + 50) – 100 = 20 m
O
5 km S
14. (c)
N 20. (a) PQ = 10 + 6 = 16 km
A F B 21. (d)
1 km 1 km P

ww
W E
C 2 km E
D 22. (a)
5 km
S N
3 km Q

w.E
5 km

P (Starting point) 5 km
W E

Find the distance PB.


By Pythagorus theorem, we have
PB2 = PF2 + FB2
PB2 = (4)2 + (3)2 = 25
asy R
Obviously, Q lies North of S.
5 km
S S

Þ PB = 5 km
En 23. (d)
N

gin
15. (c) M 15 m X D M
W E

eer
20 m

R K

15 m Y S
N

Z
12 m 24. (b) O
ing
XZ = XY + YZ = MN + YZ = 20 m + 12 m = 32 m
.ne
30 m

16. (d) C 15 km D
N

t
12 km

15 m
W E

B 15 km A S
20 m

Obviously, CB = AD = 12 km and B is south of C.


17. (b) B 7 km C

P
15 m
5 km

F OP = 30 m + 20 m = 50 m
8 km
3 km

25. (b) D A 40 m
O
N
E 7 km D
30 m

30 m

W E
A C B
20 m
BF = AB – (AE + EF) S
OD = OA + AD = OA + BC
= AB – {(AB – CD) + EF} = 40 m + 20 m = 60 m
= 8 – {(8 – 5) + 3} = 8 – {3 + 3}= 8 – 6 = 2 km

Downloaded From : www.EasyEngineering.net


Downloaded From : www.EasyEngineering.net

10 101 SPEED TEST


26. (d) 21 km 2. (b) Time between 1 p.m. on Tuesday to 1 p.m. on Thursday
B C = 48 hrs.
The watch gains (1 + 2) = 3 minutes in 48 hrs.
Þ it gains 1 min, in 16 hrs.
5 km

5 km
Hence, it will show correct time at 5 a.m. on Wednesday.
3. (b) Anuj reached at = 8 : 15 AM
Time when the other man came = 8:15 + 0:30=8:45 AM
A D
8 km 13 km (who was 40 minutes late)
We have to find out C to D. \ scheduled time of meeting = 8:45 – 0: 40 = 8 : 05 AM
27. (a) N 4. (d) First clock will gain 11 × 2 minutes in 11 hrs., and second
15 km clock will lose 11 × 1 minutes in 11 hrs.
Q
Hence difference will be 33 minutes.
P
5. (d) His birthday will be in common date of 15 to 18 and 16
W E 12km to 19 i.e., 17th
18 km
O 6. (b) The heights of A, B, C, D and E in ascending order
is E < D < A < B < C
R
S Clearly, E is the shortest.

ww
let O be the starting point and P, Q and R the positions
after every movement. Hence, Distance from the starting
point =
7. (a) Position of Kiran from the top = [35 – 7] + 1 = 29 th
Position of Sohan from the top = 9th.
Difference of their positions = 29 – 9 = 20

28. (b)
South
N w.E
Distance of final position R from O = OR = 18 – 12 = 6 km.

West
\ Mohan’s position from top = 9 + 10 = 19th
Hence, Kiran’s position from Mohan = 29 – 19 = 10th
For (Qs. 8 to 11)

W E asy According to the information provided, the order in which


the boys stand according to their heights is as follows :

East
S
North
En D E C A F B
From the figure, it is clear that ‘S’ becomes ‘North-east’
in the new figure (dotted line) gin
8.
9.
(c)
(b)
Clearly, form the above diagram, E is between D and C.
D is the tallest.

29. (e)
5 km

2 km
N
10. (d)

11. (c) eer


Counting from the shortest, C is the fourth one in the
line.
Clearly, from the above diagram, B is the shortest.
8 km School
5 km
W E
12. (b)
ing
Even if we cannot determine the exact sequence of the
weights of the children, we can conclude on the basis

5 km
1 km
S
13. (c) The arrangement in the parking
.ne
of the information provided that D is the heaviest.

C + S + C + 2S + C + 3S + C + 4S + C + 5S + C +

30. (c)
Remaining distance = 8 – (2 + 1) = 5 km
Hence, the drivers require to travel 5 km towards north
to reach the school again.
6S + C + 7S + C = 36 vehicles
Q in the second half of the row = 18 vehicles
\ C + 7S + C + 6S + 2S = 18 vehicles
Hence, no. of scooters= 7 + 6 + 2 = 15
t
14. (b) After interchanging their positions, position of A from
25 m left = 11
then positions of A form right = 9.
\ The total no. of people in the row = (9 + 11) – 1 = 19.
15 m 15. (a) Clearly, C comes at the second place – in the
descending order.
South- west direction 1B
25 m
2 C
3 A ¯ Decreasing weights
4E
SPEED TEST 8 5 D
1. (b) Next train for N. Delhi leaves at 8:30 p.m. Since time For (Qs. 16 & 17) :
interval between two trains for N. Delhi is 45 minutes. A The age wise order is as follows :
train for New Delhi has left 15 minutes ago. Gita > Kusum > Arti > Archana > Suman
\ Time of information = 8:30 – 45 + 15 = 8 P.M. 16. (b) 17. (c)

Downloaded From : www.EasyEngineering.net


Downloaded From : www.EasyEngineering.net

SOLUTIONS 11
18. (b) The order in which the five boys reach the finishing
SPEED TEST 9
line is Gaurav, Raj, Mohit, Ashish, Sanchit.
Hence Gaurav won the race. 1. (b) The sitting order of the persons are as follows :
19. (b) From the information given in the question, the E
newspaper was read in the following order
N S
B, C, E, A, D.
Hence B passed the newspaper to C. BEGFDCA W
20. (d) There are 18 persons in a row. Thus, A and B are sitting at the extreme ends.
Previous B M 2. (c) Clearly, J is sitting in the middle.
3. (d) At one end, we have k and at the other end, we have N.
4. (b) Only statement (b) is true.
Present M B 5. (a) The sitting arrangement can be shown as follows:
7th from left 15th from left C
12th from right 4th from right
(Motilal’s position) B D
21. (d) Positions as given in the question are
E F

ww 7th
My friend
23rd
exactly
I
A
Clearly, F is opposite to B.

6
persons
w.E
7th
in between

23rd 15
persons
39th
6. (b) Sitting arrangement is as follows :
G1 B1 G 2 B 2 G 3 B 3 G 4
1st 2 nd 3 rd 4 th 5 th 6 th 7 th

22. (c) According to Pratap: 20, 21 or 22 asy


\ My position is 6 + 1 + 15 + 1 + 15 + 1 = 39.
...(i)
The number of boys are less than girls, so we should
begin with girl.
(Qs. 7-10) : Seating arrangement is as follows:

23. (a)
According to his sister: not 22 ...(ii)
From (i) and (ii), the birthday falls on Apr 20 or 21.
R's position = 11th from right En R
T
H
\ M's position = (11 + 15 + 1 = ) 27th from right
= (40 – 27 + 1 =) 14th from left gin D W
24. (a)

25. (e)
R = 16th from the right.
\ w = (16 – 5 =) 11th from the right.
Time at which the train arrived = 7 : 14 PM eer M
Y

26. (c)
But Seema's watch is 6 minutes fast, hence the time in
the Seema's watch = 7 : 20 PM.
7. (a) 8. (b)
ing
9. (c)
(Qs. 11-15): Seating arrangement is as follows:
10. (c)

12th
R
16th
S
J
K
P .ne
27. (d) B > C > D, A, E
28. (a) R < M; Q < R, N; N < M
22nd
Total number of children in the row = 16 + 22 – 1 = 37 Q

L M
O
t
M>N / R>Q
N
29. (e) Total number of boys = 41.
11. (b) 12. (c) 13. (e) 14. (a)
30. (d) Geeta > Shilpa
Deepa > Geeta 15. (d)
Reepa > Gayatri (Qs. 16-20): Seating arrangement is as follows:
J
Fatima is the seniormost
But no other data is there to find who is the juniormost. L O
31. (e) Correct order can’t be determined.
32. (c) 3
Q P
Neela
Kamal Sunita
K M
11th 22nd N
(From front) 19th (From front) 16. (e) 17. (c) 18. (d) 19. (a)
So there are seven girls between Kamal and Neela. 20. (b)

Downloaded From : www.EasyEngineering.net


Downloaded From : www.EasyEngineering.net

12 101 SPEED TEST


For (Qs. 21 to 25) : Seating arrangement is as follows: 8. (d) Clearly, B and D are the females members in the family.
H 9. (b) From the above table C and D the married couple.
A G 10. (c) A, C and E are the male members in the family.
11. (d) D, the wife of the trader C is the doctor in the family.
E F 12. (c) C is the trader in the family.
For (Qs. 13-16)
D From the given facts, we can summarize,
C
B
21. (a) 22. (b) 23. (e) 24. (c) Friends Game Subjects
For (Qs.25 to 30) A Football Maths, Physics / Accounts
V B Football Maths, Accounts / Physics
S Q C Cricket Chemistry, Biology
D Boxing Maths, Accounts.

R W 13. (c) 14. (d) 15. (a) 16. (d)


For (Qs. 17-21)

25. (d) ww 26.


T
(b) 27.
P
(c) 28. (d)
The Organization of the plays will be as follows

5th 6th 7th 8th 9th 10th


29. (a)

For (Qs. 1 to 5) :
30.

SPEED TEST 10 w.E


(c) Mon
D
Tue
B
Wed
E
Thu
C
Fri
F
17. (c) The organisation would start from play D which is clear
Sat
A

The given information can be summerised in a table


that follows:
asy from above.
18. (b) E is to be organised on 7th.
Students Math

Asha
Athletics Studies GK

×
En
Arts

×
19. (a) The organisation would end with the play A on 10th.
20. (a) The play B is organised on 6th which is a Tuesday.
Charu
Deepa
Beena
×
×
× ×

× gin
21. (d) The correct sequence of the plays is DBECFA.
For (Qs. 22-25)

Ela
shows good
× ×
eer
Teacher
A
Subjects
Hindi, Eng, Math
1. (d)
× shows not good
From the table above, it is clear that Deepa is good
in Studies, General Knowledge and Arts.
B
C Eng, Geo ing
Hindi, Eng, Geo, History, French

2.

3.
(b)

(a)
Clearly, Beena is good in Studies, General Knowledge
and Mathematics.
Obviously, Asha is good in Studies, Mathematics and
D
E
Math, Hindi
History, French .ne
4.

5.
(c)

(d)
Athletics.
Charu is good in Athletics, General Knowledge and
Mathematics.
From the last row of the above table, it is clear that
22. (c)
(Qs. 26-30) :
23. (d) 24. (c) 25. (b)

The given information can be summarized as follows :


t
Ela is good in Studies, General Knowledge and Arts
but not in Athletics.
6. (c) A maximum of five cources can be taken – History,
Chemistry, Pschycology, Astronomy and Flights Day
Mathematics. Jet Airways Tuesday
7. (a) There is only one way – History and Psychology.
For (Qs. 8 to 12) British Airways Thursday
A Delta Wednesday
(Farmer) Quantas Saturday
Emirates Monday
Lufthansa Sunday
Daughter Sons Air India Friday

B (Unmarried)
(Teacher) 26. (c) 27. (c) 28. (a)
Husband E (Unmarried)
D C 29. (e) 30. (d)
(lawyer)
(Doctor) (Trader)

Downloaded From : www.EasyEngineering.net


Downloaded From : www.EasyEngineering.net

SOLUTIONS 13
For questions 21 to 25:
SPEED TEST 11
Employee Department Sport
For questions 1 to 5:
A Pers TT
B Admin Football
Friend Bank Occupation
C Admin Hockey
A S Forex
D Admin Basketball
B M Agriculture
E Mktg Cricket
C N Economist
F Pers Volleyball
D L TO
G Mktg LT
E R IT
H Mktg Badminton
F Q Clerk
21. (c) 22. (b) 23. (e)
G P Research 24. (a) 25. (d)
(26-30) : Ascending order of fatness :-
1. (b) 2. (c) 3. (a) Q < T < V < W < R < S/P < P/S
E555F
4. (d)
ww 5.
For questions 6 to 10:
(e)

26.
Ascending order of height
V<W< T<S<R< P<Q
(c)
Mon
Tue w.E P
R
27.

28.
(e) Even all the conditions are not sufficient to find the

(b)
thickest.
Wed
Thu
Fri
W
V
S
asy 29.
30.
(d) P or S can come.
(c) R is 5th from the left in both arrangements.

Sat Q
En SPEED TEST 12

6. (c)
Sun

7. (a) 8.
T

(b) gin
1.
2.
(b) Area common to singer and poets.
(b)

9. (c) 10. (e)


For questions 11 to 15:
3.
4.
5. eer
(c) Area common to
(b) circle only
and

(c) Area not common to rectangle but common to square

Friend Shift Day off 6.


and circle.
ing
(a) Area common to Rectangle, Circle, Square.
P
Q
II
I
Tuesday
Monday
7.
8.
9.
(a) Only area of square.
.ne
(d) Area common to Rectangle and Circle.

(c) Area common to rectangle, square and circle.


R
S
T
II
I
III
Wednesday
Sunday
Friday
10. (d) Sun is star. Moon is a satellite.
Star t
V III Thursday Sun
Moon
W I Saturday

11. (c) 12. (d) 13. (a) 11. (a) The required set of students is denoted by region
14. (d) 15. (c) common to any three circle only
For questions 16 to 20: \ Required number = (13 + 13 + 18 + 18) = 62.
A > D > G ...(ii); C > E > H ...(iii) 12. (a) The required set of students is denoted by regions
D > B > F ...(iv); G > C ...(v); F > G ...(vii) lying inside the circles representing History,
Combining these, we get Mathematics and Science. \ Required number = (9 +
A > D > B> F> G >C > E > H 14 + 18 + 15 + 16 + 13 + 13 + 20 + 18 + 13 + 16 + 19) = 183.
16. (e) 13. (b) The required set of students is denoted by the regions
common to the circles representing History and
17. (b) A > D > B > F > G. > C , J > E > H
Geography.
18. (a) \ Required number = (20 + 13 + 12 + 18) = 63.
19. (b) G, C, E and H 14. (b) Number of students who took History
20. (e) = (16 + 12 + 18 + 20 + 18 + 14 + 13) = 111.

Downloaded From : www.EasyEngineering.net


Downloaded From : www.EasyEngineering.net

14 101 SPEED TEST


Number of students who took Geography Fingers
= (9 + 16 + 13 + 20 + 13 + 12 + 18) = 101. Neclaces
Number of students who took science
= (19 + 15 + 18 + 20 + 18 + 16 + 13) = 119. Rings Ears
Number of students who took mathematics
= (9 + 14 + 13 + 20 + 13 + 15 + 18) = 102.
15. (c) both students and teachers are parts of college and is
differents. Conclusions :
16. (b) I. True
II. Not True

3. (d) Bottles
Cups
Mother Homosapien

Woman Plates

Spoon
17.
18.
19.
(b)
(d)
(a) ww
Home minister is a minister, minister is part of cabinet.
All Parrots are birds, but mice is entirely different.
Some professor may be scientist or researcher.
Bottles
or

20.

21.
(c)

(c)
beings.
w.E
Men, Rodents are entirely differents, but both are living

Mother and Father are entirely different but both are


parents.
Cups

22.
23.
(d)
(a) All three items are partly related. asy
Nitrogen is air but ice is differents.
Plates
Spoon

24.
25.
(d)
(b)
Tiger is a Carnivore, while elephants is not.

En
Herring is type of fish, fish belongs to the class of
animals.
Conclusions :
I. Not True
26. (c) Nurse and Patient are differents but both are parts of
Hospitals. gin II. Not True
Both conclusions form complementary pair.
27.
28.
29.
(c)
(b)
(d)
Nose and hand are differents but both are parts of body.
All diamonds rings are rings, all rings are ornaments.
Table are furniture but book are differents.
4. (a)

eer Erasers

30. (c) Chess and table tennis are differents but both are indoor
games.
Pens

ing Sharpners

SPEED TEST 13 Staples


or .ne
1. (c)

Phones
Radios
Computers Televisions
Pens
Erasers

Sharpners
t
Staples

Conclusions :
Conclusions : I. Not True
I. Not True II. Not True
II. True
5. (d) Jungles Bushes
2. (b) Fingers Trees
Neclaces
Hills
Rings

Ears Conclusions :
I. True
or II. True

Downloaded From : www.EasyEngineering.net


Downloaded From : www.EasyEngineering.net

SOLUTIONS 15

or,
Grills
6. (d) Windows or
Glasses holidays trip vacations
If neither conclusion I and nor II follow.
12. (d) According to statement
Grills
Windows
Glasses

kites bird kites aeroplane


Conclusion-I : False
II: False or

Painters Artists kites bird Aeroplane


7. (d) Painters Artists or

wwDancers
Dancers
If neither conclusion I and II follows.
13. (a) According to statement

w.E
Conclusion-I : False
plastics
metals
fibres
plastics

II: False

Buildings asy or,


fibres
plastics
Room
En metal

8. (b) Cabins
gin Only conclusion I follows.

eer
14. (a) According to statement
streets
roads
Conclusion-I : False
II: True
ing street highway
Necklace

Rings
or, streets
.ne
9. (a)

Conclusion-I : True
Bracelet
roads
Hence only conclusion I follow.
15. (b) According to statement I
highway

t
II: False Rocks
animals plants plants
Arms
Rocks
Hands Muscles
10. (a) plants
or,
animals
Conclusion-I : True Hence, only conclusion II follows.
II: False
11. (d) According to statement
16. (d) Holiday
Vacation
Holiday
or Trips
Trips
holidays vacation holidays trip Vacation

Downloaded From : www.EasyEngineering.net


Downloaded From : www.EasyEngineering.net

16 101 SPEED TEST


Conclusions : 22-23.
I. Not True II. Not True
Kites Birds Power
17. (d)
Kites Birds or
Energies Heat

Aeroplane
Force
Aeroplane
Conclusions :
I. Not True II. Not True
Fibres
18. (a)

22. (b) Conclusion I : False


Metals Conclusion II : True
23. (a) Conclusion I : True
Plastics Conclusion II : False

ww
Conclusions :
I. True II. Not True
24-25.
Coin
Metals

19. (b)
Animals
w.ERocks

Plants
Note

asy Plastics

Conclusions :
I. Not True II. True En Note
OR
Metals Coin
20. (b) Institutes
gin
Banks
eer
School
ing
Plastics
OR

Conclusions :
I. Not True
Academics

II. True
Note Metals

.neWin

21. (d)
Test Exams
Plastics
t
24. (e) Conclusion I : True
Conclusion II: True
25. (d) Conclusion I : False
Question Conclusion II : False
OR
26. (e)
Test Exams Picture

Symbols
Figures
Question
Graphics
Conclusion I : False Conclusion II : False

Downloaded From : www.EasyEngineering.net


Downloaded From : www.EasyEngineering.net

SOLUTIONS 17
OR 28. (a) Hence, only conclusion I follows.
Symbols 29. (b) Hence, conclusion II follows.
30. (d) According to statements.
Figures

Mails Updates

Chats
Picture
or

Graphics
Conclusion I : True Conclusion II : True Mails
27. (d) Jobs or
Mails

ww Chats

Hence, conclusion I follows.


Chats

w.E Occupations
SPEED TEST 14

Vacancies
OR
Jobs
asy 1. (e) Flowers
Bins

Occupations

En
gin Handles

Sticks
Vacancies
eer Bins
Conclusion I : False
For questions 27 -29 :
According to statements:
Conclusion II : False

or
ing
Flowers Handles
.ne
Gliders

Parachutes
Airplane

Helicopters
Conclusions
Sticks
I : True
III : Not true
II : Not true

Houses
t
or 2. (b)

towers
Gliders Airplane windows
temples
Parachutes s
opte r
Helic Houses
or
temples

Gliders Airplane towers


windows
Parachutes Conclusions I : Not true II : True
III : Not true

Downloaded From : www.EasyEngineering.net


Downloaded From : www.EasyEngineering.net

18 101 SPEED TEST


3. (c) cots Pens
6. (d)

walls Toys
doors chairs
Desks
walls
doors Roads
or Roads
Pens
or
cots
chairs
walls doors

or
ww Toys Desks
Conclusions I : Not true
II : Not true

cots
w.E chairs 7. (c) Bangles

Conclusions I : Not true


II : Not true
III : Not true asy Table Ring
As conclusions I and III complement to each other.

En Huts
Ring

4. (d) gin or

trees
fences eer
Bangles

gardens stones
Conclusions I : True
Table
ing Huts

5. (a)
II : True
III : True
or
.ne Bangles

Box

Books
Leaves
Table Huts
Conclusions I : Not true
II : Not true
Ring
t
Since, conclusion I and II form complementary pair.
Therefore, either (I) or (II) follows.
Jungles
8. (e)
Trees
or Jungles Chairs
Poles
Rooms
Books Conclusions I : True
Leaves
II : True
(Qs. 9-10) House Flats
Box
Conclusions I : Not true
II : Not true Buildings
III : Not true
Appartment

Downloaded From : www.EasyEngineering.net


Downloaded From : www.EasyEngineering.net

SOLUTIONS 19
Flats 16. (d) Conclusions I : Not true
II : Not true
or
17. (b) Circles
Rings ellipse
Buildings Appartment
House Squares
9. (b) Conclusions I : Not true
II : True
10. (d) Conclusions I : Not true
Conclusions I : Not True
II : Not true
(Qs. 11-12) II : True
Rivers 18. (d)
Appar- Bun-
House glows
Canal tment
or
Oceans Bun- Appar-
House
or
ww Seas
Seas
glows tment

Conclusions I : Not True


II : Not True
Oceans
Rivers w.E Canal
19. (a)

Gases Liquids
11.

12.
(d)

(e)
Conclusions I : Not true
II : Not true
Conclusions I : True asy or
Water

(Qs. 13-14)
II : True
En
evening
Noon
gin Gases Liquids

Day eer Water

Conclusions I : True

or
Night
Noon 20. (b)
II : Not True
Hours ing
Seconds

Day .ne
Day Night

evening
or
t
13. (d) Conclusions I : Not true Hours
II : Not true Seconds
14. (a) Conclusions I : True
II : Not true Day

(Qs. 15-16)
card

Conclusions I : Not True


papers boards II : True
or papers (21 -22)
boards r s
so
Teachers

fes
o
card Pr
rs
15. (c) Conclusions I : Not true ture
Lec
II : Not true
or
Since conclusions I and II form complementary pair.

Downloaded From : www.EasyEngineering.net


Downloaded From : www.EasyEngineering.net

20 101 SPEED TEST


3. (d) Letter series T M E I U F
Professors
Lecturers
Teachers Code 7 $ 2 % d 9

or According to
9 $ 2 % d 9
condition (ii)
rs
so
fes Teachers, 4. (a) Letter series J T A E R I
o
Pr Lecturers
Code 3 7 1 2 @ %
21. (a) Conclusions I : True II : Not True
According to
22. (b) Conclusions I : Not True II : True
condition (i) % 7 1 2 @ 3
(23 - 24) : Possible venn diagram are
5. (c) Letter series U K T M I H
Roses Flower Red

ww
or
Code d # 7 $ % 6

w.E
Red

Roses Flower
According to
condition (iii) «
2 8 % 9
#

5
7

6
$ % «

or asy 6. (d)
R N A X H S
Red

Flowers Red En Condition (iii) is applicable.


@ 6 2 + 7 4

23. (d) gin


7. (a)
P S R G F T
24.
25.
(a)
(a) A possible Venn diagram is +
eer
Condition (i) is applicable.
5 9 6 3 %

High
Hills
8. (a)
G H X S ing
D G

9.
Condition (ii) is applicable.
(a) As, M O D E and D E A F
.ne
# 8 % 6
Similarly, F O A M
% 6 7 $
t
SPEED TEST 15 $ 8 7 #
10. (e) W E A K W H E N
1. (a) Letter series E R W H K A

5 % 9 $ 5 * % 7
Code 2 @ © 6 # 1 Therefore,
without H A N K
condition
2 @ © 6 # 1

2. (e) Letter series M P E K D U * 9 7 $


11. (c) 12. (a) 13. (e)
14. (b) 15. (d)
Code $ 4 2 # 5 d 16. (b) 2 7 0 5 1 4

According to
condition (i) d 4 2 # 5 $ $%L T KQ
condition (ii) is applied

Downloaded From : www.EasyEngineering.net


Downloaded From : www.EasyEngineering.net

SOLUTIONS 21
17. (a) 3 6 4 2 7 9 8. (a) New arrangement
H F 3 U 6 G I T P L 8 9 S 2 7 A M K .......
©H$ Q%©
condition (i) is applied 12th from left
18. (c) 8 7 5 3 0 6
L +2 –1
9. (d) S 8
H%T # L J +2 –1
condition (ii) is applied A K M
19. (e) 5 9 2 4 7 6 @ +2 –1
I F
6 +1 +1
T@Q $%H % G
20. (d) 4 6 8 9 1 0 J +2 –1
D ©

$ H J @ K« F +5 +6 +7
% L
7
21.
22.
(a)
(b) ww
condition (iii) is applied
2976581®B@$©P=B
7 2 6 9 5 3 4 ® K B © @P T $
10. (c) 3

U
+5

+5
I

T
+6

+6
S

^
+7

+7
A
M
23.
24.
25.
(c)
(e)
(a)
81 3524 6®©CTP BK©
4352718®= TPB$ C=
4 3 5 9 7 1 w.E 11. (a)

12. (e)
D F J T $ # PR ZQ * C MAB @ H K LS + ?
only $ # P is the required answer.
* Q Z RP# $ TJ FD C MAB@ HK LS + ?
? + % « # $
asy 13. (d) Number of total symbols = 6; Number of total letters
= 16. Since, all the symbols are denoted by 7 and all
letters are denoted by 5, sum of the elements of the
1. (e) -3
2 ¾¾® 3 ¾¾® #
SPEED TEST 16
+4
En 14. (b)
sequence = 6 × 7 + 16 × 5 = 122
When all the symbols are dropped the series becomes
-3
O ¾¾® I ¾¾® C
-3
+4

+4 gin as follows:
D F JT PR ZQ C MAB HK LS
Now, seventh to the right of twelfth letter from the right
K ¾¾® O ¾¾® 5
-3
# ¾¾® P ¾¾® I
+4
15. (d) eer
= (12 – 7 =) 5th letter from the right, i.e., B.
Compare ‘DJ’ and ‘?S’. ‘D’ is the first element from left

2. (a)
-2
B ¾¾® $ ¾¾® <
+3
+3

+3
P ¾¾® # ¾¾® 7 ¾¾® @ ¾¾® D
+3 +3 ing
end of the series and ‘?’ is the first element from right
end. Similarly, ‘J’ and ‘S’ are third elements from left
and right end respectively. Hence, ‘FT’ is related
+3
R ¾¾® L ¾¾® I ¾¾® O ¾¾® K
+3 +3 +3
16. (b)
to' + L'.
.ne
D F J T $ #P R ZQ * C M A B @ H K L S + ?

3.
+3 +3
J ¾¾® 3 ¾¾® 2 ¾¾® N ¾¾® C
+3 +3

(c) 11th to the left of 16th from left means 5th from the left.
But the sequence has been reversed. Therefore, required
element will be 5th from right in the original sequence.
17. (e)
ABC DE F G HI JK LMNO P QRS T UV

t
Except it there is only one element between first and
second letter of each group of words when the position
of the letters in the series is taken into consideration.
18. (c) Eighth element from right = (22 + 1 – 8 =)15th element
5th from right Þ B from left.
4. (c) Consonant Number Symbol Hence, the required element which is exactly midway
Such combinations are between 5th element from left and 15th element from
R3P , N7O , K5D , Q4 æ 15 + 5 ö
left, is ç = ÷ 10th element from left, i.e., Q.
è 2 ø
5. (e) Number Symbol Consonant 19. (d) Clearly, the given letters, when arranged in the order 5,
There is no such combination. 1, 2, 3, 4 from the word ‘TRACE’.
6. (b) Ninth to the right of the 20th from the right means 11th 20. (b) Clearly the given letters, when arranged in the order 4,
from the right, i.e., M. 5, 2, 3, 1, 6 form the word 'STRAVE'.
Symbol Symbol 21. (d) We have to look for Vowel-Number and Number-Vowel
7. (c) Consonant
sequences.
Such combinations are : J 1 # P 4 E K 3 A D $ R U M 9 N 51 % T V * H 2 ¸ F 6
@ F ! : + J C G8QW
4, 3 and 5 are the required numbers.

Downloaded From : www.EasyEngineering.net


Downloaded From : www.EasyEngineering.net

22 101 SPEED TEST


(d) 4 + 2 K – 51; 4+2=K–5=1
22. (d) D $ RU M 9 N 5 1 % T V * H 2 (e) 6 + 3 Q – 5 ¸ ; 6 +2=8–5=2
E5555555555F E555555555F
7 elements 7 elements Note that the difference between two successive
23. (a) After the changing, the series becomes as follows; elements in 5 is not similar to others.
J 1 # P4E K3AD$ RUM9 NWQ8 G6 F ¸2H 26. (d) After changing the series becomes as follows :
* V T % 15 I D 7 1 J P $ 3 E RT 5 £ M 2 N A4 F H 6 H U9 # VB@ W
Now, ninth to the right of the eleventh element from the Now, twenty-second element from the right end is 3.
left® (11 + 9 =) 20th element from the left, i.e., G. 27. (c) We have to look for
24. (b) We have to look for Symbol Consonant – Consonant Vowel-number-consonant sequence.
sequence and Symbol–Consonant–Symbol sequences. M £ 5TRE 3$ PJ1 7D I 2NA4 F H6 H U9 #VB@W
J I # P 4 E K 3 A D $ R U M 9 N 51 % T V H Only 2 and 4 are such numbers.
2¸ F6G8QW 28. (e) D2 J
Only T is such a consonant. R +4 P +4 D +4A
25. (e) See the difference between each two successive 3 +4 1 +4 2 +4F
element. £ +4 E +4 J +4I
(a) A +2 $ – 5 E; A+2=$–5=E 29. (d) M £ 5T RE 3 $ PJ 17 D I 2 NA4 F H 6 U9 # VB@ W
(b) % + 2V – 5 N; %+2=V–5=N 30. (b) Fifth element towards right of the seventeenth element

ww
(c) 2 + 2 F – 5 V; 2+2 = F–5 =V from the right end implies twelfth element from the right
end. Hence, the required element is 4.

Solution (1 to 5) w.E SPEED TEST 17

1. (c) input : won 13 asy


After analysis of the given input and various steps of rearrangement it is clear that words and numbers are being rearranged
alternately. The words get rearranged in alphabetical order while the numbers get rearranged in descending order.
now 25 72 please go 47
Step I
Step II
:
:
go
go
won
72
13
won
Ennow
13
25
now
72
25
please
please
47
47
Step III
Step IV :
: go
go
72
72
now
now
won
47
gin
13
won
25
13
please
25
47
please
Step V
Step VI
:
:
go
go
Step VI is last step.
72
72
now
now
47
47
please
please
won
25
eer 13
won
25
13

2. (c) Step III


Step IV :
: car
car
81
81
desk
desk
15
42
42
15
39
39
tall
tall
ing more
more
Step V
Step VI
Step VII :
:
:
car
car
car
81
81
81
desk
desk
desk
42
42
42
more
more
more
15
39
39
39
15
tall
tall
tall
15 .ne
3.
Step VII is last steps.
(b) Step II
Step III
:
:
bell
bell
53
53
town
hall
hall
town
near
near
27
27
43
43
12
12
t
Step IV : bell 53 hall 43 town near 27 12
Step V : bell 53 hall 43 near town 27 12
Step VI : bell 53 hall 43 near 27 town 12
Step VI is last steps.
4. (d) Step II : box 93 25 year end 41 32 value
input :
Hence step II can not determine input.
5. (a) input : paper dry 37 23 height call 62 51
Step I : call paper dry 37 23 height 62 51
Step II : call 62 paper dry 37 23 height 51
Step III : call 62 dry paper 37 23 height 51
Step IV : call 62 dry 51 paper 37 23 height
Step V : call 62 dry 51 height paper 37 23
Step VI : call 62 dry 51 height 37 paper 23
Step VI is last step.

Downloaded From : www.EasyEngineering.net


Downloaded From : www.EasyEngineering.net

SOLUTIONS 23
(Sol. 6-10): 16. (c) Step II : 18 task bear cold dish 81 63 31
In step I, the word that comes last in the alphabetical order Step III : 18 task 31 bear cold dish 81 63
comes to the first place, pushing the rest of the line rightward. Step IV : 18 task 31 dish bear cold 81 63
In step II, the largest number comes at the second place, Step V : 18 task 31 dish 63 bear cold 81
pushing the line rightward. Thus, words and numbers get Step VI : 18 task 31 dish 63 cold bear 81
arranged alternately till all the words are in reverse Step VII : 18 task 31 dish 63 cold 81 bear
alphabetically order and numbers in descending order. 17. (d) Input : 72 59 37 go for picnic 24 journey
6. (d) Input: glass full 15 37 water now 85 67 Step I : 24 72 59 37 go for picnic journey
Step I : water glass full 15 37 now 85 67 Step II : 24 picnic 72 59 37 go for journey
Step III : 24 picnic 37 72 59 go for journey
Step II : water 85 glass full 15 37 now 67
Step IV : 24 picnic 37 journey 72 59 go for
Step III : water 85 now glass full 15 37 67
Step V : 24 picnic 37 journey 59 72 go for
Step IV : water 85 now 67 glass full 15 37 Step VI : 24 picnic 37 journey 59 go 72 for
Step V : water 85 now 67 glass 37 full 15 18. (a) Input : nice flower 34 12 costly height 41 56
Since the arrangement has been done, there will be no Step I : 12 nice flower 34 costly height 41 56
step VI. Step II : 12 nice 34 flower costly height 41 56
7. (d) Step II : ultra 73 12 16 mail sort 39 kite Step III : 12 nice 34 height flower costly 41 56
Step III : ultra 73 sort 12 16 mail 39 kite 19. (d) Step II : 16 victory 19 36 53 store lake town

ww
Step IV : ultra 73 sort 39 12 16 mail kite
Step V : ultra 73 sort 39 mail 12 16 kite
Step VI : ultra 73 sort 39 mail 16 12 kite
Step III : 16 victory 19 town 36 53 store lake
Step IV : 16 victory 19 town 36 store 53 lake
Since the line is already arranged, there will be no fifth

8.
9.
w.E
Step VII : ultra 73 sort 39 mail 16 kite 12
Hence, Step VI will be the last but one.
(d) We can't work backward in an arrangement type.
(c) Step II : tube 83 49 34 garden flower rat 56
20.
21.
step.
(d) We can't work out backward.
(b) Input : milk pot 18 24 over goal 36 53
Step I : 18 milk pot 24 over goal 36 53

asy
Step III : tube 83 rat 49 34 garden flower 56
Step IV : tube 83 rat 56 49 34 garden flower
Step V : tube 83 rat 56 garden 49 34 flower
Step II : 18 pot milk 24 over goal 36 53
Step III : 18 pot 24 milk over goal 36 53
Step IV : 18 pot 24 over milk goal 36 53
Step VI : tube 83 rate 56 garden 49 flower 34
10. (a) Input : hunt for 94 37 good 29 48 book.
En Step V : 18 pot 24 over 36 milk goal 53
Step VI : 18 pot 24 over 36 milk 53 goal
Step I : hunt 94 for 37 good 29 48 book
Step II : hunt 94 good for 37 29 48 book
Step III : hunt 94 good 48 for 37 29 book gin
22.
Hence step V is the last but one.
(a) Step III : 36 win 44 95 86 ultra box queen
Step IV : 36 win 44 ultra 95 86 box queen
Step IV : hunt 94 good 48 for 37 book 29
(Sol. 11-15):
Input : for 52 all 96 25 jam road 15 hut 73 bus stop 38 46 eer
Step V : 36 win 44 ultra 86 95 box queen
Step VI : 36 win 44 ultra 86 queen 95 box
Hence 6 – 3 = 3 more steps will be required
Step I : all for 52 25 jam road 15 hut 73 bus stop 38 46 96
Step II : bus all for 52 25 jam road 15 hut stop 38 46 96 73
Step III : for bus all 25 jam road 15 hut stop 38 46 96 73 52
23.
ing
(a) Input : new 22 model 27 pump 38 11 join
Step I : 11 new 22 model 27 pump 38 join
Step II : 11 pump new 22 model 27 38 join
Step IV : hut for bus all 25 jam road 15 stop 38 96 73 52 46
Step V : jam hut for bus all 25 road 15 stop 96 73 52 46 38
.ne
Step III : 11 pump 22 new model 27 38 join
Step IV : 11 pump 22 new 27 model 38 join
Step VI : road jam hut for bus all 15 stop 96 73 52 46 38 25
Step VII : stop road jam hut for bus all 96 73 52 46 38 25 15
11. (b) Step IV : hut for bus all 25 jam road 15 stop 38 96 73 52

12. (c)
46 Eighth from the right – road.
13. (c) 1. (a)
+Þ – – Þ´
¸Þ + ´Þ ¸
SPEED TEST 18
t
14. (a) Step V : jam hut for bus all 25 road 15 stop 96 73 52 46 38
Sixth from the left is 25 10 × 5 ¸ 3 – 2 + 3 = ?
15. (d) or, ? = 10 ¸ 5 + 3 × 2 – 3
(Sol. 16-23): or, ? = 2 + 6 – 3 = 5
In step I the least number comes to the left most position,
pushing the rest of the line rightward . In step II the word +Þ ¸ – Þ´
2. (b)
that comes last in the alphabetical order shifts to second ¸Þ + ´Þ –
from left, pushing again the rest of the line rightward. Similarly,
in step III the second least number shifts to third from left. In 63 × 24 + 8 ¸ 4 + 2 – 3 = ?
step IV the second from last in the alphabetical order comes or, ? = 63 – 24 ¸ 8 + 4 ¸ 2 × 3
to the fourth position. And this goes on alternately till all the or, ? = 63 – 3 + 2 × 3
numbers are arranged in ascending order and the words in or, ? = 66
reverse alphabetical order. 3. (b) 6 × 4 – 9 = 15
Step V : jam hut for bus all 25 road 15 stop 96 73 52 46 38 4. (c)
Step VI : road jam hut for bus all 15 stop 96 73 52 46 38 25 5. (a) 6 × 5 = 30, 30 × 3 + 1 = 91, 8 × 7 = 56, 56 × 3 + 1 =
Step VIII : stop road jam hut for bus all 96 73 52 46 38 25 15 169, 10 × 7 = 70, 70 × 3 + 1 = 211
Similarly 11 × 10 = 110, 110 × 3 + 1 = 331

Downloaded From : www.EasyEngineering.net


Downloaded From : www.EasyEngineering.net

24 101 SPEED TEST


6. (a) 21. (b) Since, 20 × 10 = 200, therefore, – means ×
– Þ +, + Þ ´ 8 + 4 = 12, therefore, ÷ means +.
+ Þ –, ´ Þ + 6 – 2 = 4, therefore, × means – .
and 12 ÷ 3 = 4, therefore, + means ÷.
Option (a) : 6 ¸ 20 + 12 + 7 – 1 = 70
Now, given expression
L.H.S. = 6 – 20 + 12 × 7 ¸ 1 = 6 – 20 + 84
= 100 × 10 – 1000 + 1000 ÷ 100 – 10
= 90 – 20 = 70 R. H.S.
= 100 0 – 1000 + 10 – 10 = 0
7. (c) + Þ ´, ´ Þ – 22. (b) Using the given symbols, we have:
Given expression = 8 + 7 × 8 ÷ 40 – 2
¸ Þ +, – Þ ¸
1
Given expression ® 175 – 25 ¸ 5 + 20 × 3 + 10 =8 + 7× –2
5
After conversion Þ 175 ¸ 25 + 5 × 20 – 3 × 10
= 7 + 100 – 30 = 77 37 2
= =7 .
8. (a) Using the proper signs, we get 5 5
Given expression = 14 × 10 + 42 ¸ 2 – 8 23. (c) 9 × 8 + 8 ÷ 4 – 9 = 65
= 140 + 21 – 8 = 153 24. (b) 20 + 12 – 4 ÷ 8 × 6 = 29
9. (c) Using proper notations, we have: 25. (d) 40 + 12 ÷ 3 × 6 – 60 = 4

ww
(a) Given statement is 3 ¸ 2 + 4 < 9 ¸ 3 – 2 or
11
2
< 1 not true
(b) 3 + 2 + 4 < 18 ¸ 3 – 1 or 9 < 5, which is not true.
26. (d) Using the correct symbols, we have
Given expression = 24 x 12 + 18 ÷ 9 = 288 + 2 = 290.
27. (d) Using the proper notations in (4) we get the statement

w.E
(c) 3 + 2 – 4 > 8 ¸ 4 – 2 or 1 > 0, which is true

(d) 3 ¸ 2 – 4 > 9 ¸ 3 – 3 or -
5
> 0, which is not true.
as 2 × 5 – 6 + 2 = 6 or 10 – 6 + 2 = 6 or 6 = 6, which is true.
28. (c) Using the proper notations in (3), we get the statement
as 5 × 2 ¸ 2 < 10 – 4 + 8 or 5 × 1 < 18 – 4 or 5 < 14,
which is true.
10. (d)
2
asy
Using the proper notation in (d), we get the statement
as:
29. (a) Using the correct symbols, we have
Given expression

11. (c)
8 × 8 + 8 ¸ 8 – 8 = 64 + 1 – 8 = 57
En
Using the proper notations in (c), we get the statement =
(36 - 4 ) ¸ 8 - 4
4x8 - 2x16 + 1
=
32 ¸ 8 - 4
32 - 32 + 1
=
4-4
0 +1
= 0.
as:-
5 × 2 ¸ 2 < 10 – 4 + 8
or, 5 × 1 < 18 – 4 gin
30. (b) Using the correct symbols, we have
Given expression = (3 × 15 + 19) ÷ 8 – 6
= 64 ÷ 8 – 6 = 8 – 6 = 2.
12. (b)
or 5 < 12 ® which is true.
Using the correct symbols, we have
Given expression = (23 + 45) × 12 = 68 × 12 = 816. eer SPEED TEST 19
13. (c) Given expression = 32 × 5 – (15 – 3) × 3
= 160 – 12 × 3
1. (b)
\ ing
No of right angles in one hour = 2
No of right angles in 24 hours = 24 × 2 = 48

.ne
= 160 – 36 = 124 = bce 2. (a) Hour hand covers an angle of 360° in 12 hours.
14. (c) Given expression = 105 + 56 – (20 × 7/14) \ Time taken to cover an angle of 135°
12
= 105 + 56 – 10 = 151 = ´135 =4.5 h \ Required time = 3 + 4.5 = 7.5 = 7:30
15.
16.
(b)
(a)
Given expression = 105 ¸ 15 × 3 = 7 × 3 = 21 = cb
Using the proper signs in the given expressions, we
get
175 ¸ 25 + 5 × 20 – 3 × 10
3. (d)
360
Angle made by hour hand for 12 hours = 360°
Angle made by hour hand for 1 hour =
360°
12
360°
t
= 7 + 5 × 20 – 3 × 10 = 7 + 100 – 30 = 107 – 30 = 77 \ Angle made by hour hand for 6 hours = 12 × (6) = 180°
17. (c) Using the proper signs, we get: 4. (b) In a year, number of weeks = 52 extra day = 1
36 – 8 4 + 6 ¸ 2× 3 = 36 – 2 + 3 × 3 From 2002 to 2008, there are 6 years.
= 36 – 2 + 9 = 45 – 2 = 43 So number of extra days = 6 (1) = 6
18. (d) Interchanging (+ and ÷) and (2 and 4), we get : While 2004 and 2008 are leap years, having one more extra
day apart from the normal extra day.
(1) 4 ÷ 2 + 3 = 3 or 5 = 3, which is false
Thus, number of extra days = 6 + 1 + 1 = 8
(2) 2 ÷ 4 + 6 = 1.5 or 6.5 = 1.5, which is false. Out of these 8 extra days, 7 days form a week and so 1 day
10 remains.
(3) 2 + 4 ÷ 3 = 4 or = 4, which is false. Hence, March 1, 2002 is 1 day less then March 1, 2008 i.e.,
3 it is Friday.
(4) 4 ÷ 2 + 6 = 8 or 8 = 8, which is true. 5. (c) In one hour, hour hand and minute hand are at right angles 2
19. (d) Using the correct symbols, we have: times.
Given expression = 8 + 36 ÷ 6 – 6 ÷ 2 × 3 Time = 10 p.m – 1 p.m = 9 hr.
\ No. of times, when both hands are perpendicular to each
=8+6–3×3=5
other in 9 hr = 9 × 2 = 18
20. (b) Using the proper notations in (2), we get the statement 6. (b) Here H × 30 = 4 × 30 = 120 0 .
as 5 × 2 ÷ 2 < 10 – 4 + 2 or 5 < 8 , which is true. (Since initially the hour hand is at 4. \ H = 4).

Downloaded From : www.EasyEngineering.net


Downloaded From : www.EasyEngineering.net

SOLUTIONS 25
Required angle A = 90 0 and since, H × 30 > A° so, 15. (c) The year 2004 is a leap year. It has 2 odd days.
there will be two timings. \ The day on 8th Feb, 2004 is 2 days before the day on 8th
2 Feb, 2005. Hence, this day is Sunday.
Required time T = (H × 30 ± A) minutes past H. 16. (d) Count the number of odd days from the year 2007 onwards
11
from the year 2007 onwards to get the sum equal to 0 odd day.
2
\ One timing = (4 × 30 + 90) minutes past 4 Year 2007 2008 2009 2010 2011 2012 2013 2014 2015 2016 2017
11
Odd day 1 2 1 1 1 2 1 1 1 2 1
2
= 38 minutes past 4. Or 4 : 38 approx. 17. (b) Each day of the week is repeated after 7 days
11
7. (a) Since, in one hour, two hands of a clock coincide only once, So, after 63 days, it will be Monday.
so, there will be value. \ After 61 days, it will be Saturday.
18. (c) 17th June, 1998 = (1997 years + Period from 1.1.1998 to
2 17.6.1998)
Required time T = (H ´ 30 + A°) minutes past H.
11 Odd days in 1600 years = 0
Here H = initial position of hour hand = 3 Odd days in 300 years = (5 × 3) º 1
(Since 3 o’clock) 97 years has 24 leap years + 73 ordinary years.
A° = required angle = 0° (Since it coincides) Number of odd days in 97 years = (24 × 2 + 73) = 121 = 2
2 odd days.
T= (3 ´ 30 + 0) minutes past 3 Jan. Feb. March April May June
11
(31 + 28 + 31 + 30 + 31 + 17) = 168 days

8. (b) ww = 16
4
11
minutes past 3.
At 5 o’clock, the hands are 25 min. spaces apart.
To be at right angles and that too between 5.30 and 6, the
= 24 weeks = 0 odd day
Total number of odd days = (0 + 1 + 2 + 0) = 3
Given day is Wednesday.

w.E
minute hand has to gain (25 + 15) = 40 min. spaces
55 min. spaces are gained in 60 min.
æ 60 ö 7
40 min. spaces are gained in ç ´ 40 ÷ min . = 43 min .
19. (d) No. of days between 21st July, 1947 and 21 st July, 1999
= 52 years + 366 days.
= 13 beap years + 39 ordinary years + 366 days
= (13 × 2) odd days + 39 odd days + 2 odd days

\ Required time = 43
7
è 55 ø

asy
min. past 5
11 = (26 + 39 + 2) odd days = 67 odd years = 4 odd days.
= (7 – 4) = 3 days before the week day on 21st July,
1999 = Saturday.

En
11 20. (b) Next train for N. Delhi leaves at 8:30 p.m. Since time
9. (d) At 4 o’clock, the hands of the watch are 20 min. spaces apart. interval between two trains for N. Delhi is 45 minutes.
To be in opposite directions, they must be 30 min. spaces apart. A train for New Delhi has left 15 minutes ago.
\ Minute hand will have to gain 50 min. spaces
55 min. spaces are gained in 60 min.
æ 60 ö 6 gin
21. (b)
Time of information = 8:30 – 45 + 15 = 8 P.M.
Time between 1 p.m. on Tuesday to 1 p.m. on Thursday
= 48 hrs. The watch gains (1 + 2) = 3 minutes in 48 hrs.
50 min. spaces are gained in ç ´ 50 ÷ min . or 54 min .

6
è 55
\ Required time = 54 min. past 4
ø 11
22. (b) eer
it gains 1 min, in 16 hrs.
Hence, it will show correct time at 5 a.m. on Wednesday.
A reverse flow chart will look as follows:

10. (a)
11
55 min. spaces are covered in 60 min.
æ 60 ö 5 ing
Desk officer - Friday
Senior clerk - Friday
Senior clerk’s leave -Thursday
Same day

60 min. spaces are covered in ç ´ 60 ÷ min. = 65 min.


è 55 ø
Loss in 64 min. = æ 65 5 - 64 ö = 16 min .
11
23. (b)
Inward clerk - Wednesday
Ashish leaves his house at 6:40 AM..ne Next day

11. (c)
ç
è 11

è 11 64
100 years contain 5 odd days.
÷
ø 11

Loss in 24 hrs. = çæ ´ ´ 24 ´ 60 ÷ö min = 32 min .


16 1
ø
8
11
24. (b)
Ashish reaches Kunal’s house at 7:05 AM.
They finish Breakfast at 7:05 + 0:15 = 7:20 AM.
t
That’s the time when they leave Kunal’s house for thier
office.
Anuj reached at = 8 : 15 AM
\ Last day of 1st century is Friday Time when the other man came = 8:15 + 0:30=8:45 AM
200 years contain (5 × 2) º 3 odd days. (who was 40 minutes late)
\ Last day of 2nd century is Wednesday. \ scheduled time of meeting = 8:45 – 0: 40 = 8 : 05 AM
300 years contain (5 × 3) = 15 º 1 odd day. 25. (d) First clock will gain 11 × 2 minutes in 11 hrs., and second
\ Last day of 3rd century is Monday. clock will lose 11 × 1 minutes in 11 hrs.
400 years contain 0 odd day. Hence difference will be 33 minutes.
\ Last day of 4th century is Sunday. 26. (d) 1st of month was Tuesday, hence the date on first Saturday
This cycle is repeated. was 5th.
\ Last day of a century cannot be Tuesday or Thursday or Hence the other Saturdays of the month are 12, 19, 26.
Saturday. Rama met her brother on 26th.
12. (a) The century divisible by 400 is a leap year. 1
\ The year 700 is not a leap year. 27. (d) 2 hrs = 150 min
2
13. (b) x weeks x days = (7x + x) days = 8x days

14. (c) On 31st December, 2005 it was Saturday. \ Angle covered by hour hand in 1 min =
Number of odd days from the year 2006 to the year 2009 2
= (1 + 1 + 2 + 1) = 5 days \ Angle covered by hour hand in 2½ hrs.
\ On 31st December 2009, it was Thursday. 1°
Thus, on 1st Jan, 2010 it is Friday. = 150 min =150 × = 75°
2

Downloaded From : www.EasyEngineering.net


Downloaded From : www.EasyEngineering.net

26 101 SPEED TEST


28. (c) Total no. of days between 27.3.1995 7. (d) From both the statements:
and 1.11. 1994 = 27 + 28 + 31 + 31 + 29 = 146 B D
Now, 146 is not completely divided by 7. It we have one + 10
day more then we have 147 days which is completely divided
by 7. Thus, the days of the week on 1 Nov, 1994 was Left 20th Right
Monday. But the day will be Tuesday ( We have 146 days)
29. (a) 16 - 1- 1997—Thursday. D’s position from the left end cannot be determined.
Number of normal year between 1997 and 2000 = 2 8. (e) From both the statements:
We know every year has 1odd day.
N
Now, number of leap year = 1
K
Leap year has 2 odd days NW NE
\ odd days = 2 + 2 = 4 \ 4 Jan, 2000 was Tuesday.
30. (b) Day is on 28th Feb = Tuesday W E D
Since, the leap year is excluded
\ The day is on 28th March = Tuesday SW SE
S
M
SPEED TEST 20
Clearly, Town M is towards South-East direction of Town K.
1. (c)
ww
Age of C = Total age – age of (A + B + D + E)
= Total age – 2 × Average ages of (A + B) – 2 × Average ages
of (D + E)
9. (d)
10. (c)
From both the statements K, M and T are siblings of P.
From I. M
PT
2. (c) From I

Q
w.E N
Thus, it can be found that M is to the north-west of T. So, I
alone is sufficient to answer the question.
From II. M K
T

asy Thus, it can be deduced that M is to the north-west of T. So,


II alone is also sufficient to answer the question. Thus, the
answer can be found by using either of the statements alone.
W E
En 11. (e) From I. It is clear that K, D and R are siblings while K and D
are females. But no relation can be found between D and M.

P H
S-W
S gin From II. It can be found that R’s father is certainly the father
of K and D as well. Since M is married to R’s father, it means
M is the mother of R.

P is to the South-West of Q.
From II eer
Thus, after combining I and II, we can definitely say that M
is the mother of D. So, both statements I and II are required
to answer.

F Q
12. (e) D’s position = 15th from right

ing
\ R’s position = (15 – 10 =) 5th from right
= (40 – 5 + 1 = ) 36th from left

P
13. (c) 20 m
.ne
3. (e)
Again, P is to the South-West of Q.
From I :

(+)N
K

M(–)
30 m
t
From II : F (+) Û K (–) D was facing East when he started his journey, from
Combining both: F(+) Û K(–) statement I.

N(+) M(–)
Hence K is the mother of N. 30 m
4. (e) Statement II:
Rani’s age = x yrs.
Þ 2x + x = 72 Þ 3x = 72 Þ x = 24 yrs. 20 m
Using this with Statement I, we get
D was facing East when he started his journey, from statement
24 II.
Age of Nidhi = 3 times younger than Rani’s age = = 8 yrs.
3 14. (d) From statement I and II.
\ Both statements I and II are Sufficient. A
5. (a)
6. (e) Since the ages of none of them is given, no conclusion can be – – +
drawn through both the statements. B C D

Downloaded From : www.EasyEngineering.net


Downloaded From : www.EasyEngineering.net

SOLUTIONS 27
15. (c) From Statement I
B D

R 10 km S From Right
10 Girls
20th
or
2 km 2 km D B
T 10 Girls
P 5 km Q
North
From Right
20th
West East Both statements are not sufficient to answer the
question.
South 22. (e) According to statement I
Town K is in north-west with respect to town D.
Required distance = PT = PQ + QT = (5 + 10) km N
= 15 km From Statements II N-W N-E
K

ww P
2 km

2 km
Q
T

2 km
W E

w.E
R 13 km S D
W-S
S
S-E

16. (e) From the both statements


4th asy
Require distance = PT = PQ + QT = (2 + 13) km = 15 km
According to statement II.
Town M is in S-E w.r.t to D

En
21
R S
15th N
N-W N-E
17. (d)
There are 21 students between R and S.
From the the statements
Suneeta has two children. However, she may have more sons. gin D

E
18. (c) From Statement I

eer W

H
M

ing S-E
S
S-E

Combining statement I and II

.ne
N

X
From statement II
X
Y

R
NW
W
N
NE
E
K

D W
W-N N-E

E
t
SW SE
S
M S-W S-E
S
Town M is in S-E w.r.t town, K.
Y Both statements required to answer the question
23. (d) From statement I
K and M are sister of T
19. (e) T
20. (a)
21. (d) According to statements I sister sister
D is the 20th from right end.
According to statement (ii) K M
10 girls are in between B and D. From statement II.
Combining statement (i) and (ii). T’s father is husband of P’s mother.

Downloaded From : www.EasyEngineering.net


Downloaded From : www.EasyEngineering.net

28 101 SPEED TEST


husband \ F £ N >R ³ H
Conclusion : I. H d N Þ H > N [not true]
II. F # R Þ F < R [not true]
father mother If neither conclusion I not II is true.
6. (b) Accordingly,
M # T Þ M< T
T P T@ K Þ T £ K
From statements - I and II K$N Þ K ³ N
husband \ M<T £ K ³ N
Conclusion : I. M # N Þ M < N [not true]
II. K d M Þ K > M [not true]
father mother Only conclusion II is true.
brother or sister 7. (c) Accordingly,
T%H Þ T=H
T P
H$W Þ H ³ W
sister sister
\ T=H ³ W

ww K M
Both statements are not sufficient to answer the
Conclusion : I. W # T Þ W < T
II. W % T Þ W = T
If either conclusion I or II is true.
[true]
or
[true]
question.
w.E
24. (b) According to Statement I
Arun did not went London on sunday.
8. (a) Accordingly,
NdK Þ N>K
K# D Þ K<D
According to statement II:-
Arun’s brother went London on Friday
asy
Hence only statement II are sufficient to answer the
D%M Þ D=M
\ N> K< D= M
Conclusion : I. M d K Þ M > K [true]
question.
25. (e) From statement I
En II. D d N Þ D > N
Only conclusion I is true.
[not true]

new good clothes Þ 5 3


From statement II
9
gin
9. (e) Accordingly,
J$ B Þ J³B
B% RÞ B=R
good clothes are costly Þ 9 6 7 3
Combining statement - I and II eer RdF Þ R> F
\ J ³ B= R> F
new Þ 5
Both statements are required to answer the question. ing
Conclusion : I. F # B Þ F < B
II. R @ J Þ R £ J
Both conclusion I and II are true.
[true]
[true]

SPEED TEST 21 10. (e) V # S Þ V £ S


S© LÞS<L .ne
1. (a) H > K ³ R > T > L
Conclusions:
I. H > L : True
II. K > T : Not True
L© JÞL<J
hence V £ S < L < J
Conclusions
t
2. (e) P = N > D ³ G < B = J I. V © L Þ V < L (True)
Conclusions:
I. G < P : True II. S © J Þ S < J (True)
II. G < J : True 11. (b) M # R Þ M £ R
3. (d) F £ C ³ V = Z > X = U R©JÞR<J
Conclusions: J#HÞJ£H
I. V < U : Not True
II. Z < F : Not True hence M £ R < J £ H
4. (b) Q £ E = 1 > N ³ R ³ S Conclusions
Conclusions : I. M # H Þ M £ H (False)
I. E = S : Not True
II. S £ N : True II. R © H Þ R < H (True)
5. (d) Accordingly, 12. (a) H $ F Þ H ³ F
F@ N Þ F £ N F@ GÞF=G
NdR Þ N >R G «MÞG>M
H@ R Þ H £ R hence H ³ F = G > M

Downloaded From : www.EasyEngineering.net


Downloaded From : www.EasyEngineering.net

SOLUTIONS 29
Conclusions 5. (c) It is mentioned in the statement that Mr. X has been
I. H « M Þ H > M (True) declared successful in the preliminary screening for
II. H « G Þ H > G ( False) the post of Director of KLM Institute. Therefore, either
he will be selected or will not be selected as Director of
13. (d) R©JÞR<J KLM Institute.
J«TÞJ>T 6. (d) II may be an assumption which the professor is
T#LÞT£L assuming before passing his statement but it definitely
hence R < J > T £ L cannot be a conclusion. Hence II does not follow. I
may or may not be possible. Hence I does not follow.
Conclusions 7. (b) It is clear that either there is no facility for health
I. R @ T Þ R = T ( False) insurance available or it is available for only affluent
II. J @ L Þ J = L (True) sections. Hence I cannot be definitely concluded. II
14. (a) W@TÞW=T follows from the given statement, as ‘limited resources’
of the person suggests that he will go to a hospital
T $ K ÞT ³ K
which provides treatment on nominal charges or free.
K«FÞK>F 8. (a) Only conclusion I follows. The statement talks about
hence W = T ³ K > F dedicated ordinary doctros but that in no way infers
Conclusions that extra ordinary specialists are not dedicated to their

15. (d)
ww
I. W $ K Þ W ³ K (True)
II. W @ K Þ W = K (False)
R £ D ...(i); D > W ...(ii); B ³ W ...(iii)
9.
profession. So conclusion II follows. I is true in the
context of the scenario prevailing in the country.
(c) Either I or II can follow. As the government would be
reviewing the diesel prices in light of the spurt in the

16. (c)
w.E
None of the inequations can be combined.
However, either I (W < R) or III (W ³ R) must be true.
H ³ V ...(i); V = M ...(ii); K > M ...(iii) 10.
international oil prices, the gove can either decide to
increase or keep the price stagnant (increasing subsidy.)
(d) The availability of vegetables is not mentioned in the

Hence K > V and I follows.


Also, M £ H and II follows. asy
Combining these, we get H ³ V = M < K given statement. So, I does not follow. Also, II is not
directly related to the statement and so it also does not
follow. Probably the demand is surpassing the supply.

17. (a) K < T ...(i); T ³ B ...(ii); B £ F ...(iii)


En
But H and K can't be compared. Hence III does not follow.
11.
12.
(d) I and II are assumptions and not conclusions.
(d) The statement does not say why the poor societies
suffer. Hence I does not follow. II also does not follow
18. (a)
Clearly, the inequations can't be combined.
Z < F ...(i); R £ F ...(ii); D > R ...(iii)
Clearly, the inequations can't be combined. gin
13.
because the statement merely states a fact; it does not
look into the merits of the fact.
(c) As Praveen has not yet returned, he might have got
19. (b) M > R ...(i); R = D ...(ii); D £ N ...(iii)
Combining these, we get M > R = D £ N
I does not follow as M and N can't be related. 14. eer
killed or might have survived. Hence (c) is the correct
option.
(b) Nothing has been said in the statements which imply
N ³ R and II follows.
M > D and III follows. ing
that VCRs and being now manufactured indigenously.
Therefore, I is invalid. Since import licence on VCR’s
has been withdrawn, they can be now freely imported.
20. (b) 21. (d)

SPEED TEST 22
22. (e)
15.
.ne
(d) Nothing of the sort can be concluded as given in two
conclusions on the basis of the statements.

1. (a) Only conclusion I seems to be reasonable. Considering


the different nature of IT Companies different
parameters should be employed for rating. It is not
necessary that if separate rating agency is established
16.
17.
(d) Both of the conclusion are invalid.

t
(b) Unless absolute figures are given, no conclusin of the
type I can be made. Since average no. of students per
teacher (60) in rural areas is higher than the average
no. of students per teacher (50) in urban areas, we can
for IT companies the investors will get protection of conclude that more students study with the same
their investment. Therefore, conclusion II does not teacher in the rural areas as compared to those in the
follow. urban areas.
18. (d) This statement does not mention anything about
2. (a) By increasing the manufacturing capacity the Company healthy people. Neither does it mention about evening
"Y" would compete reasonably on the cost front. And, walks. Hence none of the conclusions follows.
as such it can improve the quality of its products. Hence, 19. (a) Only this can balance the equation.
conclusion I follows. Conclusion II seems to be an 20. (e) All the given choices would lead to an increase in the
assumption. number of visits to health facilities.
3. (b) Considering the amount of loss incurred by Public
Sector Units it seems to be true that the Government SPEED TEST 23
did not take care in the matter of investment in the
Public Sector Units. The use of term "only" in the 1. (b) Read the last two sentences of the paragraph.
conclusion I makes it invalid. 2. (b) Read the line-"If the agriculture sector does well and
4. (b) Clearly, only conclusion II follows. It is not clear how world trade conditions improve."
the population of developing countries will not increase 3. (c) The Indian economy depends on agricultural sector
in the future. which depends a lot on monsoon.

Downloaded From : www.EasyEngineering.net


Downloaded From : www.EasyEngineering.net

30 101 SPEED TEST


4. (c) Process of poverty measurement needs to take into
account various factors to tackle its dynamic nature. 3. (a) 1 R
5. (a) It may not be possible to have an accurate poverty G B
measurement in India. W Y 2
6. (b) Increase in number of persons falling into poverty varies O 3
considerably across the country over a period of time. G– O
7. (c) 8. (e) 9. (c) 10. (b) R–W
11. (d) 12. (e) B– Y
13. (c) The argument boils down to the following, including 4. (a) B & K can't opposite to A
the unstated assumption provided by (c): M & K '' '' to H
Premise: Students get enough reading practice already. B & P '' '' to H
Unstated assumption (c): The reading program From above statements
provides only reading practice. H can’t be opposite to B, K, M, P
Conclusion: The reading program is unnecessary. Thus H will in opposite of A.
(a) is not a necessary assumption. The argument is not (Qs. 5-8). Since, there are 64 smaller cubes of equal size, therefore,
concerned with whether improved reading skills would n = no. of divisions on the face of undivided cube = 4
help the students learn history and science. Rather, the 5. (c) no. of cubes with no face coloured = (n – 2)3

ww
argument involves whether the new program would
help improve reading skills.
(b) is not a necessary assumption. The argument is
6. (d)
= (4 – 2)3 = 8
no. of cubes with one face painted = (n – 2) 2 × 6
= (4 – 2)2 × 6 = 24

w.E
that no additional reading practice is needed, regardless
of which program provides that practice.
(d) is not a necessary assumption. The argument does
not aim to compare the importance of one discipline
7. (a) Number of cubes with two red opposite faces = 0
(none of the cubes can have its opposite faces
coloured)

14.
over another.
asy
(a) (a) is the correct choice as the passage says that
8. (c) Number of cubes with three faces coloured
= 4(cubes at top corners) + 4(cubes at bottom corners)
=8
pervading”. The passage does not suggest that
En
“efficiency is present everywhere, this makes it all

efficiency does not pay or can be more of a torture.


9. (d)
10. (d) From figure, 6 is opposite 4
15. (c) Only this follows by combining the two statements.
gin
11. (c)
1 is opposite 2
3 is opposite 5
is opposite to =

1. (d) 2. (b)
SPEED TEST 24

3. (b) eer
× will opposite to +
¸ will opposite to –
hence (a) (b) (d) are not identical.
4. (c) 3 goes in the opposite direction: it talks about the utility
of fashion whereas the author does not talk of fashion
approvingly. ing
12. (a) When the sheet shown in fig. (X) is folded to from a
cube, then the face bearing the dot lies opposite to the
5.
6.
(b) Read the last sentence of the paragraph.
(d) 7. (a) 8. (a) 9. (a) .ne
shaded face, the face bearing a circle (With ‘+’ sign
inside it) lies opposite to a blank face and the remain-
ing two blank faces lie opposite to each other. Clearly,
10.
14.
15.
(b)
(d)
11. (d) 12. (d)

(d) The use of word ‘hassle-free’ suggests that the


company assumed that people seek convenience and
comfort.
13. (c) the cubes shown in figures (B) and (D) cannot be
t
formed since they have the shaded face adjacent to the
face bearing a dot and the cube shown in fig. (C) can-
not be formed since it shown all the three blank face
adjacent to each other. Hence, only the cube shown in
fig.(A) can be formed.
SPEED TEST 25
1. (d) 1 is correct as it clearly shows that 1 and 5 are the top 13. (c) The symbols adjacent to D are d , ×, and .
and bottom. 2 and 6 are on the sides of 4. So 2 is opposite Hence, the remaining symbol (?) will be opposite
6. 2 is correct on the same lines. 3 is opposite 4. 3 is to D.
correct as I & IV clearly tells that 4 is opposite 3. All the 14. (a) The numbers adjacent to 6 are 2, 3, 4 and 5 (from the
three statements regarding the figure given in the first three figure). Hence, number 1 will be opposite
question are correct. Hence, the answer is (d). to 6.
2. (a) As it is clear from the figure that face 2 is adjacent to
face 3. 15. (a)

2
5
1 4 3
The total no. of all such smaller cubes
6 = 4 × 2 (layers) = 8.

Downloaded From : www.EasyEngineering.net


Downloaded From : www.EasyEngineering.net

SOLUTIONS 31
16. (b) There are 10 cubes. SPEED TEST 27
17. (c) The number on the fall opposite to face having 1 is
6 because 3, 2 and 4 are adjacent faces of 6. 1. (a) From problem figure (4) to (5) the lower design is
18. (a) The number opposite to 4 is 2 because 5, 1, 6 and 3 reversed laterally while the other design moves to the
are adjacent faces of 4 opposite side. Similar changes would occur from
19. (a) Bottom face of figure I means opposite face of 5. It will problem figure (2) to (3).
be 3 because 4, 1, 6 and 2 are adjaceent faces of 5. 2. (a) Symbol changes positions as shown in both the
20. (a) Symbols adjacent to are –, ×, ¸ , +. Therefore, diagrams alternately and symbols in place of the sign
symbol D will be opposite to . ‘·’ remain unchanged in each of the successive figure.
SPEED TEST 26 · · ·
1. (d) The shifting of the elements takes place in such a way
that the change is completed in four steps. So the
· ·
change from figure five to six will be similar to the Figure (1) to (2) Figure (2)
change from figure one to figure two. 3. (c) Each symbol of the figure rotates in Anticlockwise and
2. (a) In each step, two pairs of elements get changed, a new symbol replaces the symbol at the top which is
beginning from the upper left and lower right. The inverted alternatively in each of the successive figures.
3.
ww
change takes place in three successive steps.
(c) The main design rotates respectively through 45°
clockwise, 180° and 90° anticlockwise after every two
figures. The shaded leaflet rotates through 45°
4. (e) From problem figure (1) to (2), first and second figure
interchange their position and are reversed at the new
positions and at the same time, third figure remains

4.
w.E
clockwise, 90° clockwise, 135° clockwise, 45°
anticlockwise and 90° anticlockwise.
(d) Watch the rotation of each element separetely. The
unchanged. From problem figure (2) to (3), second and
third figures interchange their positions and are
reversed at the new position and the figure (1) remain
unchanged. The same problem is repeated alternatively.

asy
triangel rotates by 90° ACW and 180°. The circle rotates
45°, 135°, 90° CW 45° CW ... ‘C’ rotates 45°, 90°, 135°,
180° CW.
5. (a) In each step, both the line segments close to the sides
of the hexagon, move to the adjacent sides in a
Clockwise. Also the line segments at the corner move
5.
En
(b) In each step elements interchange in pairs while one
element beginning from one end is replaced by a new
to the adjacent corner Clockwise and their number
increases by one, in the first, third, fifth steps.

6.
one. The line of orientation rotates by 45° ACW.
(d) In each step the elements of the upper row shift from
left to right in cyclic order while elements of the lower gin
6.
7.
(e) From element I to II the design rotates through 180°
(b) From element I to II the design is mirror image after
being rotated through 90° clockwise.

7.
row shift from right to left in cyclic order.
(b) In each step, the whole figure rotates by 45° ACW. The
middle element interchanges with elements on either
8.

9. eer
(c) From element I to II the design is enclosed by another
design.
(e) From element I to II the upper design encloses the lower
side alternately while the third element is replaced by a
new one. 10.
design.
ing
(d) From element I to II the design is divided into four
8. (c) In each step the whole figure rotates by 90° ACW while
one of the end elements is replaced alternately on either
side.
11.
.ne
equal parts and the lower left part becomes shaded.
(c) For the square follow if 1 = 5 then 2 = 6 rule. In alternate
steps the lower-row elements go to the upper row and
9. (a) In the first step the elements shift from the upper left to
lower right ® middle left ® upper right ® lower left ®
upper left. In the next step the elements shift one step
CW in cyclic order.
12.
13.
(b)
new elements appear in the lower row.
t
(b) In each step the corner elements rotate 90° CW and
shift one side ACW alternately. The middle figure
rotates 180° and 90° CW alternately.
10. (e) In each step the upper element rotates by 90° ACW.
The lower element gets inverted and a curve is added 14. (e) In alternate steps the upper left shifts to centre ® lower
to it on the upper side. right ® upper left. The upper right shifts to right middle
11. (d) In alternate steps the elements shift one-and-a-half sides ® upper-middle ® upper right and lower left ® left
CW while one of the elements beginning from the ACW middle ® lower middle ® lower left.
end gets replaced by a new one in each step. 15. (e) In each step the side arrow rotates 90° CW and shifts
12. (b) In each step the whole figure rotates by 90° CW while one side CW. The smaller arrow rotates 45° ACW and
one element is added in each step alternately on CW the larger arrow 45° CW alternately.
and ACW end. SPEED TEST 28
13. (b) In each step the whole figure rotates by 90° ACW and
1. (c) Meaningful words : ARE, EAR
an arc is added on the CW side.
14. (b) In each step the triangles rotate by 90° CW. The shading 2. (d) As L®8 and H ® 7
of the right triangle changes alternately. The shadings A®& I ®*
of the middle and left triangles change in each step in a
set order. T®4 R ®3
15. (a) In each step the quadrilateral rotates by 90° ACW while E®$ E®$
it shifts half a side CW alternately.

Downloaded From : www.EasyEngineering.net


Downloaded From : www.EasyEngineering.net

32 101 SPEED TEST


Similarly, 21. (c) Play A is staged on Wednesday. So plays F, B and D are
H ¾¾ ®7 A ¾¾
®& staged after play A is staged.
I ¾¾ ®* L ¾¾
®8 22. (b) All others are staged one after the other.
3. (b) Others relate to ‘parts of tree’. 23. (e) Play E is staged on Monday, the first day.
4. (d) All others are parts of a car. 24. (c) 25. (d)
5. (d) DG, IG and SN 26. (a) B ® 9; A ® 2; R ® *; N ® %; I ® #; S ® 4
6. (b) 7. (c) 27. (b) D ® 2; M ® @; B ® 9; N ® %; I ® #; A ® 6
8. (c) 9. (b) Condition (ii) is applied.
10. (e) 11. (a) 28. (c) I ® $; J ® 8; B ® 9; R ® *; L ® £; G ® #
12. (b) 13. (d) Condition (ii) is applied.
14. (c) 15. (d) 29. (c) 30. (c)
16. (a) Some hens are fish. (I-Type) 31. (a)
32. (d) V $ W ÞV < W
W@TÞW³T
T#H ÞT£H
All fish are birds. (A - Type) hence V < W ³ T £ H

ww
I + A Þ I-type
\ Some hens are birds.
This is Conclusion I.
Conclusions %
I. V © T Þ V > T ( False)
II. H % W Þ H = W ( False)

w.E
17. (c) Both the Premises are Particular Affirmative. No
Conclusion follows from Particular Premises.
Conclusions I and II form Complementary Pair. Therefore,
33. (b) H © M Þ H > M
M@ EÞM³E
E$ CÞE<C
either Conclusion I or II follows.
18. (e) All bats are boys. (A-Type) asy hence] H > M ³ E < C
Conclusions %

En I. C @ M Þ C ³ M ( False)
II. H © E Þ H > E (True)
34. (e) N @ J Þ N ³ J
All boy are gloves. (A-Type)
A + A Þ A type Conclusion. gin J%RÞJ=R
R©HÞR>H
\ All bats are gloves.
This is Conclusion II.
eer
hence] N ³ J = R > H
Conclusions %

19. (d)
Conclusion I is Converse of this Conclusion.

20. (b) Some doctors are nurses. (I-Type) ing


I. R # N Þ R £ N (True)
II. N © H Þ N > H (True)
35. (d) L @ K Þ L ³ K
K ©AÞ K >A
A$WÞA<W .ne
All nurses are patients. (A-Type)
I + A Þ I -type Conclusion.
\ Some doctors are patients.
36. (a)
hence, L ³ K > A < W
Conclusions % I. W $ L Þ W < L ( False)
II. L # W Þ L £ W ( False)
37. (b)
t
Conclusion II is Converse of this Conclusion. 38. (c) 39. (d)
(Qs. 21-25) : 40. (e)
The given information can be summarized as follows.

Plays Days
A Wednesday
B Friday
C Tuesday
D Saturday
E Monday
F Thursday

Downloaded From : www.EasyEngineering.net


Downloaded From : www.EasyEngineering.net

SOLUTIONS 33

SPEED TEST 29 11. (d) None of options gives us a four digit perfect square.
12. (a) Clearly, 1000 + 1> 1000 × 1
1. (b) – 1.5 will lie leftmost on the number line, hence it is the Hence, the required positive integer is 1.
smallest. 3
13. (a) Number of women = 120 ´ = 72
2. (b) According to the question, 5
2 3 Number of men = 48
the required value = ´ 114 - ´ 68 = 76 - 51 = 25 Þ number of married persons = 80
3 4
3. (d) Let the number be x. and the number of unmarried persons = (120 – 80) = 40
If all the men are assumed to be married, then the number
x x x of married women could be 80 - 48 = 32
Now - = 3 or = 3 or x = 36
3 4 12 Þ maximum number of unmarried women
= 72 - 32 = 40
4. (d) Clearly, 1 < 1 = 7 < 7
7 7 7 1089 + 1
14. (a) Number of odd numbered pages = = 545
2
1
Þ is the smallest number.. 15. (b) Let p = 5, 7,11,13, ............

5. (c) ww
1
8
7

th part is black. Half of the remaining


For p = 5,
For p = 7,
(p2 - 1) = 24

(p2 - 1) = 48

i.e.
1 7 7
´ =
2 8 16 w.E
is yellow.. For p = 11,

For p= 13,
(p2 - 1) = 120

(p2 - 1) = 168
Therefore, the part left =
7
16
,
asy ...........................................................
...........................................................
1
which is equal to 3 = 3.5 cm.
2 En 16.
...........................................................
Clearly, all the above numbers are divisible by 24.
(a) 9 stubs are used to make 1 candle

Hence, length of the pencil =


3.5
7
= 8 cm
gin \ 1044 stubs will be used to make
1044
9
or 116 candles

6. (a) 25 ´ 92 = 32 ´ 81 = 2592 .
16 17.
eer
(c) Let the whole number be x
According to question

\ Difference = 25 × 92 – 2592 = 2592 – 2592 = 0


Hence, the numerical difference is 0.
Þ
x + 20 =
69
x
x2 + 20x = 69
ing
7. (b) Let the number of buffaloes be x and number of ducks
be y. Then number of legs = 4x + 2y and the number of
heads = x + y
Þ
Þ
x2 + 20x – 69 =0
x2 + 23x – 3x – 69 = 0 .ne
8.
Now, 4x + 2y = 2(x + y) + 24
or 2x = 24 or x = 12
(a) The numbers– 264, 396, 792, and 6336 are divisible by
132.
18.
Þ
\
(b)

\
x = 3 or – 23, Hence, 3 is only whole number.
A clock strikes 4 in 9 seconds.

clock strikes 1 in
9
seconds.
t
x ( x + 23) – 3 ( x + 23) = 0 Þ ( x + 23) ( x – 3) = 0

9. (c) Let the number be x. 4


112 9
Now, x + 13x = 112 or 14x = 112 or x = =8 Þ clock strikes 12 in ´ 12 seconds
14 4
10. (d) We test i.e., clock strikes 12 in 27 seconds.
19. (c) Given, numbers are 50, 35 and 35.
3600 Now, place value of 3 is 30 and 30 in the numbers 35
= 400, which is not a perfect cube.
9 and 35 respectivvely.
\ Sum of the place values = 30 + 30 = 60
3600 20. (b) Below given are those numbers between 500 and 600 in
= 72, which is not a perfect cube.
50 which 9 occurs only once.
3600 509, 519, 529, 539, 549, 559, 569, 579, 589, 590, 591, 592,
= 12, which is not a perfect cube 593, 594, 595, 596, 597, 598.
300 These are 18 numbers.
3600 21. (c) From the given alternatives,
Now, = 8 = (2)3 , which is a perfect cube
450 112 × 114 = 12768
Hence, (d) is the correct option. \ Larger number = 114

Downloaded From : www.EasyEngineering.net


y
Downloaded From : www.EasyEngineering.net

o
u
rs
m
a
h
34 101 SPEED TEST

b
o
o
22. (a) Out of the given alternatives, 2. (d) Let the number be x.

b
Q x2 – (12)3 = 976

.w
137 × 139 = 19043
\ x2 = 976 + 1728 = 2704

o
\ Required smaller number = 137

rd
23. (d) Let the numbers be x and (x + 1), \ x = 2704 = 52

p
\ x(x + 1) = 8556 3. (c) Q 5 chairs + 8 tables = `6574

re
or, x2 + x – 8556 = 0 \ 10 chairs + 16 tables = 6574 × 2 = ` 13148

s
s
or, x2 + 93x – 92 x – 8556 = 0

.c
4. (b) Let the number be x.

o
or, (x2 + 93) (x – 92) = 0 Q x2 + (56)2 = 4985

m
\ x = 92 Þ x2 = 4985 – 3136 = 1849
24. (b) 382 = 1444 \ x = 1849 = 43
392 = 1521
\ Required number = 1521 – 1500 = 21 5. (a) Units digit in (7 4 ) = 1 . Therefore, units digit in (7 4 )8
455.8 i..e. 7 32 will be 1. Hence, units digit in
25. (d) Number of pieces = = 53
8.6 (7)35 = 1´ 7 ´ 7 ´ 7 = 3
26. (c) Amount received by each student
Again, units digit in (3) 4 = 1

ww
=
15487
76
= » `204 Therefore, units digit in the expansion of
(34 )17 = (3) 68 = 1
27. (c)
w.E
æ 1ö
ç 1 - ÷ of the number = 84
è 5ø
æ 84 ´ 5 ö
Þ Units digit in the expansion of
(371 ) = 1´ 3 ´ 3 ´ 3 = 7
\ number = ç
28. (d) A + C = 146
è 4 ø
÷ = 105
asy and units digit in the expanison of (1135 ) = 1
Hence, units digit in the expansion of
or A + A + 4 = 146
146 - 4 En 7 35 ´ 371 ´ 1155 = 3 ´ 7 ´1 = 1
1
or A =
2
= 71
\ E = A + 8 = 71 + 8 = 79 gin
6. (b,c) Here, 0 < p < 1, so let p =

1 æ 1 1 1
2
ö
29. (d) Lowest number of set A =
280
5
– 4 = 52
eer
Clearly, p <
p
çèQ <
2 1/ 2
or < 2÷
2 ø

Lowest number of other set = 52 × 2 – 71 = 33


\ Required sum = 33 + 34 + 35 + 36 + 37 = 175
30. (c) Let total number of goats be x.
Also,
ing
1
p2
> p Q
1
æ 1ö
çè ÷ø
2
>
1
2
or 4 > 0.707

Then, total number of hens = (90 – x)


So, x × 4 + (90 – x) × 2 = 248
Þ 4x – 2x = 248 – 180 7. (a) 9 lemons cost =
48 ´ 9
2

.ne
= 144 paise = cost of 4 mangoes

x=
68
2
= 34
3

Þ cost of 3 mangoes =
144
4
´ 3 = 108 paise
t
SPEED TEST 30 = cost of 5 apples
or cost of 9 oranges = 108 paise
1. (c) Let Farah’s age at the time of her marriage be x. 108
Þ cost of one orange = = 12 paise
9
Then, (x + 8) = x ´ 9 8. (a) Let x = 1 × 2 × 3 × 4 = 24
7
Therefore, n = 1 + 24 = 25 = odd
9x Clearly, n is an odd integer and a perfect square.
Þ -x=8 This is true for all values of x (product of any four
7
consecutive integers).
8´7 9. (a) By remainder theorem,
Þx= = 28 years
2 96 will have the remainder 1 as 9 has the remainder 1.
\ Farah’s present age = 28 + 8 = 36 years
96 + 7
1 Also will have the same remainder as
\ Daughter’s age 3 years ago = 36 ´ -3 8
6
= 3 years (1) 6 + 7
which has the remainder equal to 0.
8

Downloaded From : www.EasyEngineering.net


y
Downloaded From : www.EasyEngineering.net

o
u
rs
m
a
h
SOLUTIONS 35

b
o
o
It means that number of cats must be divisible by 5 and
784 (73 )28 (343)28

b
10. (b) = = number of dogs must be divisible by 4.

.w
342 342 342 This condition follow, if cat, C = 15, dog, D = 40

o
or C = 35, D = 20.

rd
(343) 28 In other cases, (1) will not be satisfied.

p
By remainder theorem, will have the same

re
342 If C = 15, D = 40, then 3 + 10 = 13 pets were adopted.

s
If C = 35, D = 20, then 7 + 5 = 12 pets were adopted.

s
128

.c
remainder as i.e. the remainder is 1. But 13 > 12.

o
342 17. (d) 6n2 + 6n = 6(n2 + n) = 6n(n + 1)

m
Alternatively : n(n + 1) is always divisible by 2, as the product of two
consecutive natural numbers is always divisible by 2.
(343) 28 (342 + 1) 28
= Þ (6n2 + 6n) is divisible by 6 and 12.
342 342 Hence, option (d) is correct.
(342)28 + 28 C1 (342) 27 + ... + 28C 27 342 + 1 M
= 18. (d) W= , M – 10 = W + 5
342 2
Clearly, 1 is the remainder. [where M ® men, W ® women]
On solving, we get M = 30, W = 15.

ww
11. (a) Let the number of students in each row is n and the
number of row is r. Then the number of students in the
class will be nr.
According to the question,
19.
\ M + W = 45.
(b) Let the number of boys = B.

w.E
(n + 4) (r – 2) = nr
and (n – 4) (r + 4) = nr
............(1)
............(2)
on simplifying equations (1) and (2), we get the system
3
4
2
\ B = 18 Þ B = 24 = of class
3
\ Strength of the class = 36
\ No. of girls = 36 – 24 = 12.
of equations
n – 2r + 4 = 0
n–r–4=0 asy 20. (d) Suppose husband’s age be H years.
Then wife’s age W = H – 9
On solving this system, we obtain r = 8; n = 12
Hence, nr = 96
En Son’s age S =
H-9
2
12. (d) Let each of them had x bullets after division. Then,
total number of bullets they had after using 4 bullets
each = (x - 4) + (x - 4) + (x - 4) = 3x - 12 gin Daughter’s age D =
H
3

Now, 3x - 12 = x or x = 6
Original no. of bullets = 6 × 3 = 18 H eer
According to question,

+7 =
H -9
Þ 2H + 42 = 3H - 27
13. (b) I is false as sum of two negative integers is always
negative.
II is true, as product of two negative integers is always
3
ing 2
Þ H = 42 + 27 = 69
positive.
III is not always true as x –y can be positive or negative
21. (d)
\ W = 60.

.ne
Let the face value of the National Savings Certificates
purchased by Soma in the first year be Rs.x.
according as x > y or x < y.
14. (a) Let the number be x. Then, as per the operation
undertook by the student, we have
x + 12
= 112 Þ x = 660
Þ 8x + 11200 = 48000
Þ 8x = 36800 Þ x = 4600
t
\ x + (x + 400) + (x + 800) + (x + 1200) + (x + 1600) +
(x + 2000) + (x + 2400) + (x + 2800) = 48000

6 22. (c) T = P + 4 = 9; N = T – 3 = 6.
23. (a) Let the Number be N
660 \ N = 765x + 42
Hence, the correct answer = + 12 = 122
6 765 is divisible by 17
15. (d) Let the present age of Harry and George be x and y, \ if N is divided by 17, remainder will be 42 – 34 = 8.
respectively. 24. (a) From option (a),
Then, 210 – 2n = 960 Þ 1024 – 2n = 960 Þ 2n = 64
\ n = 6.
1
x= y ...........(i) Similarly we can try for options (b) and (c). Hence, any
3 other option does not satisfy the given equation.
1 25. (c) Let the total number of packages be x.
and ( x - 5) =( y - 5) ...........(ii)
4 2
After uploading x packages remaining packages are
From (i) and (ii), y = 45 years 5
Hence, the required age = (5 + y) = 50 years
2 3
16. (c) Total no. of cats and dogs = 55 .............. (1) x– x= x
Out of these, 1/5 of cats + 1/4 of dogs had been adopted. 5 5

Downloaded From : www.EasyEngineering.net


y
Downloaded From : www.EasyEngineering.net

o
u
rs
m
a
h
36 101 SPEED TEST

b
o
o
According to the question, 6. (d) To find the capacity we have to take the HCF of 279,

b
341 and 465.

.w
1
When he uploaded another 3 packages then of 279 = 31 × 9 × 3

o
2 341 = 31 × 11

rd
original no. of packages remained. 465 = 31 × 3 × 5

p
re
3x x 3 1x Þ HCF (279, 341, 465) = 31
\ –3= Þ x – =3

s
Capacity of the measuring can be = 31 ml.

s
5 2 5 2

.c
7. (b) H.C.F of co-prime numbers is 1.
Þ 6x – 5x = 30 Þ x = 30

o
So, L.C.M. = 117/1 = 117.

m
Hence, 30 packages were in the van before the first delivery.
8. (a) Required time = LCM of 200, 300, 360, 450 sec
26. (c) Given : Divisor = 2 × remainder
= 1800 sec.
\ Divisor = 2 × 75 = 150 9. (c) The LCM of 18, 22, 30 is 990.
2 So, they will meet each other after 990, ie, 16 min and 30 sec.
Also, Divisor = ´ dividend
3 10. (d) The required number must be a factor of (11284 – 7655) or
3 3629.
Þ dividend = 150 ´ = 225 Now, 3629 = 19 × 191
2 \ 191 is the required number.
27. (a) 24162 = 89x + 43
LCM ´ HCF 2079 ´ 27
Þ x = (24162 – 43) ÷ 89 = 271 11. (c) The required number = = = 297
28.

29.
ww
(c) By actual division, we find that 999999 is exactly divisible
by 13. The quotient 76923 is the required number.
(b) Clearly, unit’s digit in the given product = unit’s digit in
12. (a)
First number 189
Clearly, the required number must be greater than the LCM
of 18, 24, 30 and 42 by 1.

w.E
7153 × 172.
Now, 74 gives unit digit 1.
\ 7153 gives unit digit (1 × 7) = 7. Also 172 gives unit
Now, 18 = 2 × 32
24 = 23 × 3
30 = 2 × 3 × 5
42 = 2 × 3 × 7

30.
digit 1.

asy
Hence, unit’s digit in the product = (7 × 1) = 7.
(b) The digit in the unit’s place of 251 is equal to the remainder 13. (d)
\ LCM = 32 ×23 × 5 × 7 = 2520
\ the required number = 2520 + 1 = 2521
The first number = 2 × 44 = 88

En
when 251 is divided by 10. 25 = 32 leaves the remainder 2
when divided by 10. Then 250 = (25)10 leaves the remainder
210 = (25)2 which in turn leaves the remainder 22 = 4. Then
\ The second number =
HCF ´ LCM 44 ´ 264
88
=
88
= 132

251 = 250×2, when divided by 10, leaves the remainder 4×2 = 8.


gin
14. (b) HCF = 12. Then let the numbrs be 12x and 12y.
Now 12x × 12y = 2160 \ xy = 15
Possible values of x and y are (1, 15); (3, 5); (5, 3); (15, 1)
\ the possible pairs of numbers (12, 180) and (36, 60)

eer
SPEED TEST 31
15. (c) Bells will toll together again at a time, which is obtained by
1. (b) Q Product of numbers = (LCM × HCF) taking L.C.M. of their individual tolling intervals.

ing
Þ 480 × second number = 2400 × 16 L.C.M. of 9, 12 and 15 = 180 min
Þ second number = 80 They will toll together again after 180 min, i.e. 3 hours.
Time = 8 + 3 = 11 a.m.
2. (c) Let numbers be x and y. 16. (b) Since each rod must be cut into parts of equal length and each
Q Product of two numbers = their (LCM × HCF)
Þ xy = 630 × 9
Also, x + y = 153 (given)
HCF of 78, 104, 117 and 169 = 13.
.ne
part must be as long as possible, so HCF should be taken.

since x – y = = (x + y) 2 - 4xy

Þ x - y = (153) - 4(630 ´ 9)
2
No. of parts from 78cm. rod =
78
13

No. of parts from 104 cm. rod =


=6

104
13
=8
t
= 23409 - 22680 = 729 = 27 117
No. of parts from 117 cm. rol = =9
3. (a) Product of the numbers 13
= HCF × LCM = 21 × 4641 169
= 21 × 3 × 7 × 13 × 17 No. of parts from 169 cm. rod = = 13 .
13
= 3 × 7 × 3 × 7 × 13 × 17 \ Maximum no. of pieces = 6 + 8 + 9 + 13 = 36
\ The required numbers can be 17. (c) Let the numbers be x and 4x.
3 × 7 × 13 and 3 × 7 × 17 = 273 and 357 Then, 84 ´ 21 = x ´ 4x
4. (b) Required number = HCF of 108 and 144 = 36
5. (c) Time gap between two consecutive ticks or 4 x 2 = 1764
58 609 or x 2 = 441 or x = 21
sec. and sec.
57 608 Þ 4x = 4 × 21 = 84
58 609 Thus the larger number = 84
\ Required time = LCM of and 18. (b) Let us check each of the options here starting with (a)
57 608
LCM of 58 and 609 1218 13 11 133 11 8 65
= = sec (a) + = <5 (b) + = >5
HCF of 57 and 608 19 5 6 30 4 3 12

Downloaded From : www.EasyEngineering.net


y
Downloaded From : www.EasyEngineering.net

o
u
rs
m
a
h
SOLUTIONS 37

b
o
o
12 1 Therefore, in one hour (60 minutes), then will fall

b
19. (b) = 0.1008, = 0.1

.w
119 10 æ 60 ö
together 8 times ç ÷ excluding the one at the start.

o
è 7 ø

rd
4 7
= 0.102 and = 0.101 28. 2
(b) HCF = 2 × 3 2

p
39 69

re
LCM = 24 × 35 × 52 × 72

s
1 Ist number = 23 × 34 × 5

s
Thus, is the least.

.c
10 2nd number = 24 × 32 × 52

o
observing the above situation, we conclude that the

m
4 4 -4
æ2ö æ3ö 34 æ 2ö third number must be
20. (c) Reciprocal of ç ÷ = ç ÷ = 4 = ç ÷
è3ø è2ø 2 è 3ø x = 22 ´ 32 ´ 33 ´ 7 2 = 22 ´ 35 ´ 7 2
29. (d) Product of numbers = HCF × LCM
34
(a), (b), (d) are all equivalent to 4800 ´ 160
24 Þ The other number = = 1600
480
21. (c) 1.65m = 165 cm
Required length = LCM of 25 and 165 30. (c) The traffic lights will again change at three different
= 825 cm = 8.25 m road crossings simultaneously after the LCM of 48, 72

22. (c)
ww
5
16
= 0.312 ,
6
17
= 0.352 ,
7
18
= 0.388
and 108
i.e., after every (432 sec) 7 minutes and 12 seconds, i.e.
the earliest at 8 : 27 : 12 hours.

w.E
Therefore,
5
<
6
<
16 17 18
29 15 116 + 45 161
7

1190
SPEED TEST 32

1190 ´ ?
= 3094
23. (d) +
12 16
=
48
=
48
161 31 192 asy 1. (a)
7225
or, ? =
´ ? = 3094 or,
3094 ´ 85
1190
= 221
85

Therefore,

161 17
+ =
48 48 48
= 4 = a whole number

En 2. (a) Q ?- 2 5 = ( 5 -1 )2 = 5 + 1 - 2 5

And -
48 48
= 3 = whole number

31 17 17 gin
3. (a)
\?=6
?
=
25
36 (11 ´ 3 - 18)
?
Þ =
5 5 1
= =
6 33 - 18 15 3
Between
48
and ;
48 48
is the least fraction.
Clearly, the least fraction among the given fractions in
6
eer
\= = 2
3

ing
4. (c)
17
options is . 5. (e) 169 – 25 – 26 + 7 = (?)2 = 125 = ?2 Þ ? = 125 = 5 5
48 2 2
(16) - 5 + 169 = (? )
3

.ne
24. (b) Correct asecending order is 6. (e)
256 – 125 + 13 = (?)2
2 9 5 8 144 = (?)2
< < <
3 13 7 11

25. (a) Let us take a proper fraction, such as


1
2
. 7.

8.
(b)

(b)
? = ± 12
2
? = 225 + 2304 + (12 ) = 15 + 48 + 144 = 207

? = 450 + 890 + 685 = 2025 = 45


t
1+ 2 3
Now, the new fraction = = 9. (c) ? = ( 6859 )
13
+ 4 = 19 + 4 = 23
2+ 2 4
3 1 447
Thus, > 10. (e) ? = 7´ + 73 - 26 = 149 + 73 - 26 = 196 = 14
4 2 21
x æ 60 - 40 ö 20
26. (b) As x is a positive number, we have =1 ? = 5´ ç = 5´ = 50
x
11. (d) è 2 ÷ø 2
x +1 1 x 12. (d) ? = 42 = 16
=1+ >1, <1
x x x +1 \ ? = 256
x + 2 x + 3 -1 1 255
= =1- <1 13. (b) (?)2 = =3 \ ?= 3
x+3 x +3 x+3 17 ´ 5
27. (b) LCM of 6, 5, 7, 10 and 12 = 420 seconds 9 ´ 16 ´ 5
14. (d) ( ?) 2 = 80 + = 80 + 20 = 100
420 36
= = 7 minutes .
60 Q ? = 100 = 10

Downloaded From : www.EasyEngineering.net


y
Downloaded From : www.EasyEngineering.net

o
u
rs
m
a
h
38 101 SPEED TEST

b
o
o
( )2 - 2 Þ 34969 = 187

b
15. (a) ?= 6 +1 6 = 6 +1+ 2 6 - 2 6 = 7

.w
Hence, the number of rows = 187
31. (c) 93 × 812 ¸ 273 = (3)?

o
184
32 × 3 × 34 × 2 ¸ 33 × 3 = (3)?

rd
16. (c) 12 ´ + 26 - 73 = ?
23 36 × 38 ¸ 39 = (3)? or 36 + 8 – 9 = (3)?

p
re
= 96 + 26 - 73 = ? = 122 - 73 = 49 = 7 35 = (3)? or ? = 5

s
( 9 ) ´ ( 81 ) ¸ ( 27 ) = ( 3 )( )
3 5

s
17. (b) 169 – 64 – 676 + 2 = (?)2 3 ?
32. (c)

.c
= 169 – 64 – 26 + 2 = (?)2 = 171 – 90 = 81

o
(3)3 × (9)5 ¸ (3)3 × 2 = (3)? or (3)3 × (3)2 × 5 ¸ (3)6 = (3)?

m
\?=9
(3)3 + 10 – 6 = (3)? or (3)7 = (3)?
18. (c) (74 ´ )
676 - 42 ´ ? = 496
33. (e)
?=7
81.1 × 42.7 × 23.3 = 2?
( 74 ´ 26 - 496) 2? = (23)1.1 × (22)2.7 × 23.3 or 2? = (2)3.3 × (2)5.4 × 23.3
?= = 1428 ¸ 42 = 34 = (34)2 = 1156 2? = (2)3.3 + 5.4 + 3.3 or 2? = (2)12.0
42
19. (e) 312 ? = 12
49 ´ 49 ´ 49
20. (e) ?2 = 2 48 ´ 8 = – 21 + 8 + 49 – 14 8 34. (a) ? = (49)3 ¸ (7)2 = = 2401
7´7
= 14 8 – 21 + 57 – 14 8 = 36 = 62 35. (a) 643.1 × 84.3 = 8?

21. (e)
ww
\?=6
7365 + 29.16 +
? = 473.16 – 7394.16
? = 7437.16
Þ (82)3.1 × (8)4.3 = 8? Þ 86.2 × 84.3 = 8?
Þ 86.2 + 4.3 = 8? Þ 810.5 = 8?
? = 10.5
8? = 87 × 26 ¸ 82.4

22. (b)
w.E
? = 43 = 1849

æ 756 ´ 67 ö
çè ÷
3
= 250047
36. (d)

8? = 87 ×
82.4
82
or 8? = 87 + 2 – 2.4

23. (b)
804 ø

17 + 51 + 152 + 289 asy 37. (a)


8? = 86.6 or ? = 6.6
? = (31)31 × (31)– 27
? = (31)31 – 27
= 17 + 51 + 152 + 17 = 17 + 51 + 169
En ? = (31)4
? = (961)2

{(12) }
gin
= 17 + 51 + 13 = 17 + 8 = 25 = 5 -2 2
(12 )-4
24. (d) 217 + 64 Þ 217 + 8 = 225 = 15 38. (e) = =1
{(12) } (12 )-4
-2
2

eer
25. (a)
210.25 21025 145 145
26. (d) + Þ + Þ 14.5 + 1.45 = 15.95
100 10000 10 100 64 ´ 64
27. (a) 4 2000 45
4 16
85 400
39. (c)
6 3
ing
( 8)
= 6? Þ 68– 3 = 6? Þ 6? = 65 Þ ? = 5

6 3 3 6 4 3+ 6- 4
= 85
5 425
-25
40. (c) 8? =
\?=5
´ ( 64 ) ¸ 84 = 8 ´ 8 ¸ 8 = 8

.ne
28. (b)
Clearly, the required least number is 25.
Let the number be x.
Now, according to the question,
x2 – (22)3 = 9516
or, x2 = 9516 + (22)3 = 9516 + 10648 = 20164
1.
2.
(b)
(e)
SPEED TEST 33

? = 456.675 + 35.7683 × 67.909 – 58.876


t
Þ x = 20164 = 142 = 456.675 + 2428.98 – 58.876
29. (a) Let the number be x. = 2885.66 – 66 – 58.876 = 2826.78 » 2830
According to the question.
(4052 – x2) × 15 = 41340 3. (c) ?=
{(52) 2
+ ( 45 )
2
} = 2704 + 2025 = 4729 = 591.125
8 8 8
41340
Þ 4052 - x 2 = = 2756 Þ x2 = 4052 – 2756 = 1296
15 4. (b) ?= (12.25)2
– 625 = – 25 (12.25)2
\x= = 150.0625 – 25 = 125.0625
1296 = 36
5. (e) ? = 572 + 38 × 0.50 – 16
30. (a) Number of rows = 34969 = 572 + 19 – 16 = 591 – 16 = 575

187 1056 ´ 7 ´ 5 ´ 13
6. (a) ?= = 7280
3 ´ 2 ´ 11
1 34969
1
249
7. (b) ? ´ 9 = 6318 + 26 = 243
28
224 243
2569 \ ? = = 27
367 9
2569 2
\ ? = (27) = 729
´

Downloaded From : www.EasyEngineering.net


y
Downloaded From : www.EasyEngineering.net

o
u
rs
m
a
h
SOLUTIONS 39

b
o
o
78700 24. (a) ? × 40 = 8059 – 7263 = 796
+ 4 ´ 120 = 45 + 460 = 505 ; 525

b
8. (c) ?»

.w
1750 796
\ ?= = 19.9
9. (c) ? ; 78 + 42 + 9 = 129 ; 130

o
40

rd
1 2 3 1

p
10. (b) ´ ´ ´1715 = 85.75 » 85 4 ´ ? = 4062 ¸ 5 = 4062 ´ = 812.4
25. (a)

re
8 3 5
5

s
561

s
11. (a) ´ 20 = 320.5 » 320

.c
812.4
35 \ ?= = 203.1

o
4

m
12. (d) (15)2 ´ 730 = 225 ´ 27 = 6075
æ 1 ö
3325 152 26. (e) ? = 3.5 ´ (80 ¸ 2.5) = 3.5 ´ ç 80 ´ ÷
13. (d) Given expression implies ? = ´ è 2.5 ø
25 16
= 133 × 9.5 = 1263.5 = 3.5 ´ 32 = 112
27. (c) 13% of 258 – ? = 10
1 2 2 \ ? = 13% of 258 – 10
14. (e) ? =5+ + 2 + + 3+
5 15 3
13
1 2 2 = 258 ´ - 10 = 33.54 - 10 = 23.54
= 10 + + + 100

ww
= 10 +
5 15 3
3 + 2 + 10
= 10 +
15 28. (d) ?=
4
5
3 5 4 11 5
´2 ¸ = ´ ¸
4 8 5 4 8

15. w.E 15
= 10 + 1 = 11
15

(c) ? = – 15 – 27 – 88 – 63 + 255
= –193 + 255 = 62
4 11 8 88
=´ ´ =
5 4 5 25
=3
13
25
29. (e) ? = 623.15 – 218.82 – 321.43 = 623.15 – 540.25 = 82.9
16. (b) Given expression can be written as

?=
2525 ´ 0.25 ´ 7 asy 30. (b) ? × 50 = 5437 – 3153 + 2284 = 7721 – 3153 = 4568

\ ?=
4568
= 91.36
5
= 883.75

14 57 20 2 3 20 2 1 2 4 En 50

SPEED TEST 34
17. (b) ?= ´ ´ = ´ ´ = ´ ´ =
19 70 21 1 10 21 1 1 7 7
500 ´ 32 50 ´ 162 gin
1. (a) Assume the third number = x
18. (e)
19.
?=
100
+
100
(d) 45316 + 52131 – 65229
= 160 + 81 = 241

eer
According to question
2 × 280 + x + 178.5 × 2 = 281 × 5
or, 560 + x + 357 = 1405
= ? + 15151
Þ 32218 = ? + 15151
\ ? = 32218 – 15151 = 17067 ing
or, x + 917 = 1405
or, x = 1405 - 917 = 488

20. (c) ?=
184 ´ 4 184 ´ 4
=
400 ´ 23 4 ´ 23
=8
2.
.ne
(b) Age of the fourth friend = 31 × 4 - 32 × 3
= 124 - 96 = 28 years
100
21. (a) ? 4 /3 ´ ?5 / 3 = 23 ´ 128
Þ ?3 = 25 × 27 = 2 12
\ ? = (212)1/3 = 24 = 16
3. (a) Required average

=
t
965 + 362 + 189 + 248 + 461 + 825 + 524 + 234
8
22. (b) 16% of 450 ¸ ?% of 250 = 4.8 3808
= = 476
16 ? 8
Þ 450 ´ ¸ 250 ´ = 4.8 4. (b) 21a + 21b = 1134
100 100
or, 21 (a + b) = 1134
Þ 72 ¸ 2.5 ´ ? = 4.8
1134
72 a+b= = 54
Þ 2.5 ´ ? = 21
4.8 a + b 54
\ Required average = = = 27
72 2 2
\ ?= =6
4.8 ´ 2.5 5. (e) Let the first number be = 6x
23. (e) \ Second number = 3x
? - 11 = 1521
and the third number = 2x
Þ ? - 11 = 39
According to the question,
Þ ? = 39 + 11 = 50 6x + 3x + 2x = 154 × 3
or, 11x = 154 × 3
\ ? = (50)2 = 2500

Downloaded From : www.EasyEngineering.net


y
Downloaded From : www.EasyEngineering.net

o
u
rs
m
a
h
40 101 SPEED TEST

b
o
o
16. (c) Total of 10 innnings = 21.5 × 10 = 215

b
154 ´ 3 Suppose he needs a score of x in 11th innings; then average in

.w
\x= = 42
11 215 + x

o
11 innings = = 24 or, x = 264 – 215 = 49

rd
\ Required difference = 6x – 2x = 4x = 4 × 42 = 168 11

p
6. (d)

re
x
7. (c) Average score 17. (a) Let the total journey be x km. Then km at the speed of 25

s
3

s
.c
1
= [221 + 231 + 441 + 359 + 665 + 525] x

o
6 km/hr and km at 30 km/hr and the rest distance

m
4
1
= [2442] = 407 æ x xö 5
6 çè x - - ÷ø = x at the speed of 50 km/hr..
8. (e) Let A = x 3 4 12
According to the question Total time taken during the journey of x km
x + x +1 + x + 2 + x + 3 + x +4 x x 5x 18 x 3x
= 5 × 48 = hrs + hrs + hrs = hrs = hrs
3 ´ 25 4 ´ 30 12 ´ 50 600 100
Þ 5x + 10 = 240
Þ 5x = 230 x 100 1
\ average speed = = = 33 km/hr

ww
\ x = 46 18 x 3 3
\ E = 46 + 4 = 50 600
\ A × E = 46 × 50 = 2300 18. (a) Let the total no. of workers be x.

w.E
9. (b) 16a + 16b = 672 Now, 8000 x = 7 × 12000 + (x – 7) × 6000
or, 16 (a + b) = 672 42000
Þ x= = 21
672 2000
\a+b= = 42
16

Required average =
a+b
=
42
= 21
asy 19. (d) Let the average score of 19 innings be x.

Then,
18x + 98
19
= x + 4 Þ x = 22

10. (e) Third number = 290


2 2
En The average score after 19th innings
= x + 4 = 22 + 4 = 26

11. (b)
– (48.5 × 2) – (53.5 × 2)
= 290 – 97 – 107 = 86
Total age of the family of five members = 24 × 5 = 120 gin
20. (b) Total weight of 45 students
= 45 × 52 = 2340 kg
Total weight of 5 students who leave
Total age of the family of five members before 8 years
= 120 – 5 × 8 = 120 – 40 = 80
80 eer
= 5 × 48 = 240 kg
Total weight of 5 students who join

12. (e)
So, Required average age =
5
= 16yr

x + x + 2 + x + 4 + x + 6 = 4 × 36 ing
= 5 × 54 = 270 kg
Therefore, new total weight of 45 students
= 2340 – 240 + 270 = 2370
Þ 4x + 12 = 144 Þ 4x = 144 – 12

Þ 4x = 132 Þ x=
132
= 33
Þ New average weight =
45.ne
2370 2
= 52 kg
3

t
4
13. (b) Let the score of Ajay = x 21. (c) Total score of 40 innings = 40 × 50 = 2000
Rahul = x – 15 Total score of 38 innings = 38 × 48 = 1824
Manish = x – 25 Let the highest score be x and the lowest score be y.
According to question, x = 63 + 30 Sum of the highest and the lowest score
\ x = 93 = x + y = 2000 – 1824
\ Score of Ajay = 93 Þ x + y = 176 ...(i)
then Rahul = 93 – 15 = 78
then Manish = 93 – 25 = 68
and by question, x – y = 172 ...(ii)
Total marks of Rahul, Manish and Suresh Solving (i) and (ii), we get x = 174
= 3 × 63 = 189 22. (c) Let average cost of petrol per litre be Rs x
\ Suresh = 189 – (78 + 68) = 43
12000
\ Manish + Suresh = 68 + 43 = 111 \ x=
14. (a) Suppose marks got in physics, chemistry and mathematics 4000 4000 4000
+ +
are P, C and M. 7.5 8 8.5
P + C + M = C + 120
\ P + M = 120 12000 3 3
= = =
P+M æ 1 1 1 ö 10 + 1 + 10 2 1 2
= 60 4000 ç + + ÷ + +
2 è 7.5 8 8.5 ø 75 8 85 15 8 17
15. (c) By Direct Formula :
2 ´ 60 ´ 30 2 ´ 60 ´ 30 6120
Average = = = 40 km/hr = = Rs 7.98 per litre
60 + 30 90 767

Downloaded From : www.EasyEngineering.net


y
Downloaded From : www.EasyEngineering.net

o
u
rs
m
a
h
SOLUTIONS 41

b
o
64.5% of 800 + 36.4% of 1500 = (?)2 + 38

o
23. (d) Let the three numbers are x1, x2, x3 and x3 be the largest 3. (a)

b
number. 516 + 546 = (?)2 + 38

.w
Given, average of three numbers = 135 1062 – 38 = (?)2

o
1024 = (?)2
\ x1 + x2 + x3 = 3 × 135 = 405

rd
(32)2 = (?)2
Since, x3 = 180

p
? = 32

re
x1 + x2 = 225 .......... (i) 4. (c) 41% of 601 – 250.17 = ? – 77% of 910

s
Given x1 – x2 = 25 .......... (ii) » 246 – 250 = ? – 701

s
.c
Solving (i) and (ii), x1 = 125, x2 = 100 » 701 + 246 – 250 = ?

o
Hence, the smallest number = 125 – 25 = 100 » 700 = ?

m
24. (d) Total age of 10 students =12.5 × 10 = 125 years 5. (a) 40.005% of 439.998 + ?% of 655.011 = 228.5
Total age of 20 students = 13.1 × 20 = 262 years 40 x
» ´ 440 + ´ 655 = 229
100 100
\ Average age of 30 students = 125 + 262 = 12.9 years
30 655x 655x
» 176 + = 229 » = 229 - 176
25. (d) Replacing the two numbers and arranging them in 100 100
ascending order, we get 53 ´ 100
31, 35, 35, 36, 38, 45, 46, 52, 55, 60 x= »8
655
Since no. of observation is even

ww
6. (d) 25% of 84 × 24% of 85 = ?
éN æN ö ù 21 × 204 = ?
ê 2 th + çè 2 + 1÷ø th ú 428.4 = ?
ë û 7. (b) 20.06%of 599 + 10.01% of 901 = ?
\ Median = observation

w.E =
2
( 5th + 6th ) obs.
2
» 600 ´
20
100
+ 900 ´

» 120 + 90 = ?
10
100
=?

38 + 45 83
=
2
=
2
= 41.5 asy 8. (a)
\ ? » 210
14.2% of 5500 + 15.6% of ? = 1795
15.6
26. (b) Total of x and y = 40 × 2 = 80
x+y+z 80 + 10 90 En 781 +

15.6
100
´ ? = 1795

27.
\ Average of x, y and z =
3
=

(a) Sum of 11 numbers = 11 × 10.9 = 119.9


3
=
3
= 30

gin 100

?=
´ ? = 1014

1014 ´ 100
= 6500
Sum of first 6 numbers = 6 × 10.5 = 63
Sum of last 6 numbers = 6 × 11.4 = 68.4
The middle number = Sum of the first six + Sum of the
9. (e)
eer
15.6
36% of 245 – 40% of 210 = 10 – ?
88.2 – 84 = 10 – ?
the last six – Sum of all the 11
= 63 + 68.4 – 119.9
= 11.5
ing
4.2 = 10 – ?
? = 10 – 4.2 = 5.8
1
28. (a) Sum of 10 numbers = 402
Corrected sum of 10 numbers = 402 – 13 + 31 – 18 = 402
10. (a)
2
of 3842 + 15% of ? = 2449

15 .ne
Hence, correct average = 402 = 40.2
10
29. (d) Required average number of microwave
42 ´ 12 + 20 ´ 10 504 + 200 704
1921 + x +


15
100
100
= 2449

= 2449 - 1921 t
528 ´100
= = = = 32 x= = 3520
12 + 10 22 22 15
30. (d) Temperature on the fourth day 11. (c) 57% of 394 – 2.5% of 996
= 40.2 × 4 + 41.3 × 4 – 40.6 × 7 57 2.5
= 160.8 + 165.2 – 284.2 = 41.8°C = 394 ´ - 996 ´
100 100
= 225 – 25 = 200
SPEED TEST 35 12. (a) 40% of 265 + 35% of 180 = 50% of ?
Þ 265 × 0.4 + 180 × 0.35 = ? × 0.5
1. (b) 76% of 1285 = 35% of 1256 + ? 169
Þ 976.6 = 439.6 + ? Þ 976.6 – 439.6 = ? Þ 106 + 63 = ? × 0.5 Þ ? = = 338
0.5
\ ? = 537
2. (b) (21.5% of 999)1/3 + (43% of 601)1/2 = ? 1 1 21 10
13. (b) 4 ´ 3 + ? = 20% of 120 Þ ´ + ? = 120 ´ 0.2
(1000 × 21.5%)1/3 + (600 × 43%)1/2 = ? 5 3 5 3
(215)1/3 + (258)1/2 = ? Þ 7 × 2 + ? = 24 Þ ? = 10
(216)1/3 + (256)1/2 = ? 14. (c) 14% of 250 × ? % of 150 = 840
(6)3 ×1/3 + (16)2 × 1/2 = ?
6 + 16 = ? 14 150 ´ ?
250 ´ ´ = 840
? = 22 100 100

Downloaded From : www.EasyEngineering.net


y
Downloaded From : www.EasyEngineering.net

o
u
rs
m
a
h
42 101 SPEED TEST

b
o
o
35 × 1.5 × ? = 840 23. (e) Let the maximum aggregate marks = x

b
840

.w
?=
According to the question,
35 ´ 15

o
40% of x – 4% of x = 261

rd
? = 16
(40 - 4)

p
18 27.5 or x × = 261

re
15. (b) ? = 609 ´ + 450 ´
100
100 100

s
s
= 109.62 + 123.75 = 233.37 » 233 261

.c
125 85 \x= × 100 = 725

o
16. (c) 3060 ´ -?´ = 408 36

m
100 100
24. (b) Population of the town after 2 years
85
3825 – 408 = ? × æ 7 öæ 5 ö
100
= 198000 ç 1 + ÷ç1 - ÷
3417 ´ 100 è 100 øè 100 ø
=?
85 198000 ´ 107 ´ 95
4020 = ? = = 201267
100 ´ 100
x y
17. (d) 500 × = 300 ´ 25. (b) Total expenditure = 44668 + 56732 = 101400
100 100 Total percentage expenditure

ww
Þ 5x = 3y

Þ y=
5x
3
.... (i)
\
= 100 – 22 = 78 %

Total amount =
101400 ´ 100
= ` 130000
Þ
w.E
xy ´ 200
100 ´100
5x
= 60 Þ xy = 3000

é 5x ù
êQ y = 3 ú
26. (b)
\
75 % of 16 = 12
78

12% of monthly salary = 6567


6567 ´ 100
Þ x´
3
= 3000

Þ 5x2 = 3000 × 3 Þ x2 =
ë
3000 ´ 3 asy
û \
27. (d)
100% =
12
= ` 54725
Total valid votes = 85% of 15200 = 12920

Þ x 2 = 1800
5

En 28. (c)
\ Number of valid votes to other candidate
= 45% of 12920 = 5814
If the radius is diminised by r%, then
Þ x = 1800 = 2 ´ 3 ´ 3 ´10 ´10
x = 30 2
gin æ
Area is diminished by çç 2r -
r2 ö
÷÷ % = 2 ´ 10 -
102
100
= 19%
18. (d) 400 ´
185
100
+ 240 ´
35
100
= 1648 ´

?
?
100
? eer è
29. (b) Let B’s salary be ` 100,
then A’s salary = ` 125
100 ø

Þ 740 + 84 = 1648 ×

Þ ?=
824 ´ 100
100
Þ 824 = 1648 ×
100
% lesser =
ing
125 - 100
125
´ 100 =
25
125
´100

19. (b)
1648
2 50 25
? = 630 ´ ´ ´
or ? = 50
1
= ´100 = 20%
5 .ne
Þ ?=
3 100 100
210 ´ 50 ´ 25 ´ 2
100 ´ 100
Þ ?=
20. (b) Number of transferred employees
210
4
= 52.5
30. (b) Let the original price of apple be ` x /dozen

New price ` =
4x
5
54 54 10
/dozen = 4 x -
5
=
x 12
t
1225 ´ 40
= 40% of 1225 = = 490 æ 5 1ö 5
100 Þ 54 ç - ÷ = Þ 54 çæ 1 ÷ö = 5
x è 4x x ø 6 è 4x ø 6
21. (b) Let the original fraction be .
y 54 ´ 6 4x
Þ 4x = Þ = 12.96
5 5
x + 5x 4 6x 18 x 72 12
Then, =2 Þ = Þ = =
y + 3y 7 4y 7 y 42 7 SPEED TEST 36
x
22. (c) Let the original fraction be = y 1. (d) Total number of students in the school = 819
Number of girls = 364
According to the question,
\ Number of boys = 819 - 364 = 455
350 \ Required ratio = 435 : 364 = 5 : 4

100 = 7 7x 7 x 7 8 8
400 9 Þ 8 y = 9 Þ y = 9 ´ 7 = 9 æ2 ö
y´ 2. (d) Share of Urmila in dividend = ç ´ 57834 ÷ = ` 19278
100 è6 ø

Downloaded From : www.EasyEngineering.net


y
Downloaded From : www.EasyEngineering.net

o
u
rs
m
a
h
SOLUTIONS 43

b
o
o
3. (b) Let the present ages of Richa and Shelly be 13. (a) Iron Copper

b
5x and 8x years. Alloy I 8 : 6 14 kg.

.w
According to the question, Alloy II 36 : 6 42 kg.

o
44 : 12 56 kg

rd
5 x + 10 7
=

p
After 10 years, 1 1 1
8 x + 10 10 : : = 6:4:3

re
14. (b) Ratio of sides =
2 3 4

s
or, 56x + 70 = 50x + 100 or, 56x – 50x = 100 – 70

s
æ 6ö

.c
or, 6x = 30 Largest side = ç104 ´ ÷ cm = 48 cm

o
è 13 ø

m
30 15. (d) Let the numbers be x and y.
\x= =5
6
x 4
\ Shelly's present age = 8x = 8 × 5 = 40 years \ y=7
4. (d) The sum of money is not known.
\ 7x = 4y ... (a)
5. (a) Let the age of woman be 2x years and that of her
daughter be x years. x + 30 5
=
According to the question, y + 30 8
\ 8x – 5y = – 90 ...(b)
84

ww
2x + x = 2 × 42 or, 3x = 84 or, x =
\ Daughter's age = 28 years
3
= 28 From eqn (b), 32x – 20y = – 360
From eqn (a), 35x = 20y
\ 32x – 35x = – 360
6.

w.E
(e) Let Arun’s present age be x years.
Then, Deepak’s present age = (x + 14) years

Then,
x-7
=
5
\ x=
y = 210
360
3
= 120

x + 14 - 7 7

Þ 7x - 5x = 35 + 49 Þ x =
84
= 42asy \ Avg =
330
2
= 165

1 2 3

7.
2
\ Deepak’s present age = 42 + 14 = 56 years
(d) Data is given in ratio. So age can’t be determined. En 16. (d) Given the ratio = : : = 6 : 8 : 9.
2 3 4

8. (a) Let number of students in Arts and Commerce were 4x


and 5x respectively. Then, gin
17.
æ
è

\ 1st part = ` ç 782 ´ ÷ = ` 204.
23 ø
(b) Let actual distance be x km. Then,
4x
=
5x + 65 11
8
Þ 44x – 40x = 520 Þ x =
520
4
= 130
3
4 eer
:1 :: 60 : x

9.
\ Number of students in Arts = 4 × 130 = 520
(c) Let the original number of boys and girls be 6x and 5x
respectively. 18.
3
4 ing
Þ x = 60 Þ x =
240
3
= 80 km
(d) Let A = 2x, B = 3x and C = 4x. Then,

Then,
6x + 8 11
=
5x - 2 7
A 2x 2 B 3x 3
= = , = = and
B 3x 3 C 4x 4
C 4x 2
= =
A 2x 1 .ne
Þ 55x - 42 x = 56 + 22 Þ x =
78
13
\ Number of boys = 6 × 6 + 8 = 44
=6

19.
Þ
A B C 2 3 2
: : = : : = 8 : 9 : 24.
B C A 3 4 1
(c) Total age of 3 boys = (25 × 3) years = 75 years
t
10. (a) Present age of Meena Ratio of their ages = 3 : 5 : 7.
8 ´ 8 ´ (10 - 3) 8´ 8´ 7
Age of the youngest boy = æç 75 ´ ö÷ years = 15 years
= = = 32 years 3
24 - 10 14
è 15 ø
11. (b) Let the present age of father and son be 17x and 7x 20. (b) Let income of A = ` 3x, income of B = ` 2x
respectively. and expenditure of A = ` 5y,
17x – 6 3 expenditure of B = ` 3y
Then, =
7x – 6 1 Now, saving = income – expenditure
Þ 21x – 17x = 18 – 6 Þ x = 12 ÷ 4 =3 \ 3x – 5y = 2x – 3y = 200
Þ x = 2y and y = 200
\ Father’s present age = 17 × 3 = 51 years.
12. (c) Rita : Sita : Kavita \ x = 400
7 : 15 \ A’s income = ` 1200
7 : 16 21. (c) Since, A : B = 2 : 3 and B : C = 6 : 5
49 : 105 : 240 \A: B: C = 4: 6 : 5
The ratio of money with Rita, Sita and Kavita is
Then, A’s share = 4
49 : 105 : 240 ´ 750 =` 200
We see that 49 º ` 490 \ 240 º ` 2400 4+ 6+5

Downloaded From : www.EasyEngineering.net


y
Downloaded From : www.EasyEngineering.net

o
u
rs
m
a
h
44 101 SPEED TEST

b
o
o
22. (c) Let the numbers be 5x and 4x. 5

b
Now, difference of numbers = 10 30. (b) In the mixture, milk = ´ 28 = 20 litres, and

.w
i.e. 5x – 4x = 10 Þ x = 10 7

o
\ Larger number = 10 × 5 = 50

rd
2
Water = ´ 28 = 8 litres
23. (b) Give, Ratio of distribution = 3 : 1 : 5

p
7

re
Let the share of P, Q and R be 3x, x and 5x, respectively 2 ltrs of water is added, Hence the new ratio of

s
Since, Difference between Q’s and R’s share is Rs. 3600.

s
.c
\ 5x – x = 3600 Milk 20 20
= = = 2 :1

o
Þ x = 900 Water 8 + 2 10

m
\ P’s share = 3 × 900 = 2700
and Q’s share = 900 SPEED TEST 37
Hence, Total of P’s and Q’s share
= Rs 3600 1. (a) New % of sugar in (3 + 1) litre solution
24. (b) Mrs. X spends = Rs 535 0.04 ´ 3
\ Total cost = 43 shirt + 21 ties = 535 = = 0.03 = 3%
By hit and trial, S = 10, T = 5 (3 + 1)
Þ Total cost = 43 × 10 + 21 × 5 = 535 80 ´ 15 + 20 ´ 20
2. (c) C.P. of mixture = = ` 16

ww
Hence, Ratio of shirts to ties = 10 : 5 = 2 : 1

25. (a) Given


a 2
=
b 5 \ S.P. =
(100 + 25)
80 + 20

´ 16 =` 20

w.E
Consider
2a + 3b
2a
= b
7a + 5b 7a
=
2
+3 2´ + 3
5
2
3.

Rs 3.50
100
(b) By the rule of alligation, we have
Cost of Alcohol Cost of Kerosene Oil
` 2.50
b
+5 7´ + 5
5
asy a cost of

19 En
(By putting value of
b
) mixture
` 2.75

= 5 =
19
39 39 gin
5
26. (c) Let the numbers be 6x and 13x.
eer
2.75 – 2.50 = 0.25

\ Required ratio =
0.25 1
3.50 – 2.75 = 0.75
= i.e. 1: 3
LCM (6x, 13x) = 13 × 6 × x = 312 (given)

Þ x=
312
13 ´ 6
Þ x=4
4.
ing
(b) In mixture,
0.75 3

Quantity of pure milk 3- 0 3 5

\
So, the numbers are 24 and 52.
Sum of the numbers = 24 + 52 = 76
Quantity of water
= =
.ne
3.6 - 3 0.6 1
=

Since in every 5 litres of milk, he adds 1 litre of water.


27. (d) We have,
5a + 3b 23
2a - 3b
=
5
Þ 25a + 15 b = 46a – 69b
5. (b) Let C.P. of milk be Re. 1 per litre.
Then, S.P. of 1 litre of mixture = Re. 1.
Gain = 20%
t
\ In every 25 litres of milk, he adds 5 litres of water.

a 84 4
Þ 21a = 84 b Þ = = æ 100 ö 5
b 21 1 \ C.P. of 1 litre of mixture = ` ç ´1 =` .
è 120 ÷ø 6
28. (b) Let the no. of one rupee, 50 paise and 25 paise coins be
2x, 3x and 4x respectively. By the rule of alligation, we have :
According to question, C.P. of 1 litre C.P. of 1 litre
of water of milk
æ 3x 4x ö
Rs. ç 2x + + ÷ = Rs. 216 0 Re. 1
è 2 4ø Mean
price
8x + 6x + 4x Re. 5
Þ = 216
4 6
\ x = 48
\ Number of 50 paise coins = 48 × 3 = 144 5 1 5 5
29. (c) Let the ages of two persons be X and Y respectively 1– = –0 =
6 6 6 6
X 5 X – 16 3
\ = and =
Y 7 Y – 16 5 1 5
\ Ratio of water and milk = : = 1: 5.
On solving we get X = 40, Y = 56. 6 6

Downloaded From : www.EasyEngineering.net


y
Downloaded From : www.EasyEngineering.net

o
u
rs
m
a
h
SOLUTIONS 45

b
o
6. (a) Out of 10 litre of solution, there is 1 litres of nitric acid

o
12. (d) By the rule of alligation,

b
and 9 litres of water. Alcohol concentration :

.w
Let x litres of water be added to the solution so that the

o
diluted solution is of 4% strength.

rd
Original mixture Pure alcohol
\ 4% of (10 + x) = 1 Þ x = 15.

p
15% 100%
7. (a) Let C.P. of 1 litre milk be Re. 1.

re
s
50

s
S.P. of 1 litre of mixture = Re. 1, Gain = %.

.c
3

o
m
6 25%
\ C.P. of 1 litre of mixture = Re. .
7
By the rule of alligation, we have :
C.P. of 1 litre C.P. of 1 litre
of water of milk
0 Mean Re. 1 75% 10%
price
\ Alcohol must be added in the ratio of 10 : 75 or 2 : 15
Re. 6
\ Quantity of alcohol to be added in 10 litres

ww 1
7
7
6
7
=
2
15
´ 10 =
4
3

8.
w.E
\ Ratio of water and milk =
1 6
: = 1 : 6.
7 7
(b) The first alloy does not have tin. Therefore, quantity of
13. (b) C. P. of mixture =
100 ´ 60
(100 + 25)
= ` 48

asy
tin in 2 units of the resulting alloy =
5
13
Let x kg be mixed. Then,

48 =
40 ´ x + 55 ´ 16
Þ Quantity of tin in one unit of the resulting alloy
5 En 16 + x
Þ 8x = 16[55 - 48] Þ x = 14 kg

9.
= 13 =
2
5
26
(b) By the rule of alligation,
gin
14. (c) The existing solution has 40% sugar. And sugar is to be mixed;
so the other solution has 100% sugar. So, by alligation method:
40% 100%
water concentration,
Original solution Water eer 50%
50%
10%
4
11
1

ing
\ The two mixtures should be added in the ratio 5 : 1.

Therefore, required sugar =


300
´ 1 = 60 gm

6 (mixture)
13
15. (b) Gold
19
Copper
9
5

.ne
7
13
14
143
6
15
4
\ Gold : Copper = 6 : 4 = 3 : 2
t
\ water must be added to the mixture in the ratio 16. (c) Apply the alligation on fracfion of milk in each mixture.
14 7 Mixture Water
: i.e. 2 : 11
143 13 2 0
3
2
Quantity of water to be added = ´ 55 = 10 litres 1
11 3
10. (c) Gold Copper 1 1
Type A 14 4 3 3
Type B 7 11 Ratio of mixture to water = 1 : 1
Type C 21 15 Therefore, if there is 60 liture of solution, 60 litres of water
\ The ratio of the Gold and Copper in the type C should be added.
alloy = 7 : 5. 17. (e) Initially water (weight) = 45 gm & milk 15 gm. After added
11. (c) C. P. of mixture of 18 kg = 10 × 45 + 8 × 50 = ` 850 15 gm
\ S. P. = C. P. + Profit = 850 + 32 = ` 882 water the percentage of water
\ S. P. = ` 882 for 18 kg
weight of water 60
882 = = ´ 100 = 80%
\ S. P. for 1 kg. = Rs =` 49 total weight of mixture 75
18

Downloaded From : www.EasyEngineering.net


y
Downloaded From : www.EasyEngineering.net

o
u
rs
m
a
h
46 101 SPEED TEST

b
o
o
18. (a) By the rule of alligation: 23. (c) Let the quantity of pure milk be x litres.

b
Cost of 1 kg tea of 1st kind If 5 litres of water is added to it, then the cost of (5 + x)

.w
litres = Rs. 3x

o
rd
S.P. of ( 5 + x) litres = Rs. (3x + 15)

p
\ Profit = Rs. 15

re
Given 20% of 3x = 15

s
s
.c
3x
Þ = 15 Þ x = 25

o
5

m
\ The amount of pure milk in the mixture is 25 litre
\ Required ratio = 750 : 250 = 3 : 1. 24. (c) Let the weight of tea worth Rs 25 per kg = x kg.
19. (d) Let the C.P. of spirit be Re. 1 per litre. According to question
5 5
Spirit in 1 litre mix. of A = litre; C.p. of a litre mix. in A = Re. . é x ´ 25 + 30 ´ 30 ù
7 7 \ 110% of ê x + 30 ú = 30
7 ë û
Spirit in 1 litre mix. of B = litre;
13 110 é 25x + 900 ù
5 Þ = 30
100 êë x + 30 úû
ww
C.P. of 1 litre mix. in B = Re. .
8
13
Spirit in 1 litre mix. of C= litre; Mean price = Re.
8
.
Þ 11 (25x + 900) = 300 (x+30)
Þ 275x + 9900 = 300 x + 9000 or 25x = 900

w.E
13 13
By the rule of alligation, we have : \ x = 36 kg.
C.P. of 1 litre mixture in A C.P. of 1 litre mixture in B 25. (b) Let, weight of sugar costing Rs 5.75 per kg = x kg
æ5ö æ7ö x ´ 5.75 + 75 ´ 4.50 = 5.50 ´ (x + 75)
ç ÷
è7ø Mean price
æ8ö
ç ÷
è 13 ø
asy ç ÷
è 13 ø Þ 5.75x + 337.50 = 5.50x + 412.50
Þ 0.25x = 75
\ x = 300 kg
æ1ö
ç ÷
è 13 ø
9
91
En SPEED TEST 38

\ Required ratio =
1 9
: = 7 : 9.
13 13
20. (c) Let the quantity of milk and water be 40 litres and 60
gin
1. (b) Let S.P. = ` 100. Then, C.P. = ` 96; Profit = ` 4.

litres, respectively.
(Q Ratio of milk to water = 2 : 3)
2. eer æ 4
è 96
ö
ø
25
\ Profit % = ç ´100 ÷ % = % = 4.17%. » 4.2%
6
(a) Let the cost of production of the table be ` x.
After removing 50% of solution
Quantity of milk = 20 litres and
Quantity of water = 30 litres
Þ
ing
Then, 125% of 115% of 110% of x = 1265
125 115 110
´ ´ ´ x = 1265
Therefore, the concentration of the solution is reduced
from 40 to 20 i.e. a reduction of 50%.
100 100 100
253 .ne
æ 1265 ´ 160 ö
3
21. (b) Quantity of milk = ´ 45 = 27 litres
5
2
Quantity of water = ´ 45 = 18 litres
5
3.
Þ
160
x = 1265 Þ x = ç
è 253 ø
(b) Let C. P. = ` x then profit = S.P. – C. P.

Þ
1
´ x = 891 – x Þ
11x
= 891
t
÷ = ` 800

10 10
Let x litres of water be added to make the ratio 9 : 11.
891´10
18 + x 11 Þ Þx= =` 810
\ = 18 + x = 33 Þ x =15l 11
27 9
4. (d) C. P. for 50 pencils = ` 100
22. (c) Ratio of milk in the containers are,
100
1 3 5 5 3 25 \ C. P. for 45 pencils = ´ 45 = ` 90
5´ : 4´ : 5´ = : : 50
6 8 12 6 2 12 = S.P. of 45 pencils
and the ratio of water in the containers are, \ No gain , no loss
5 5 7 25 5 35 5. (b) (100 – loss) : S1 : : (100 + gain) : S2
5´ : 4´ :5´ = : : \ (100 – 12.5) : 420 : : (100 + 12.5) : S2
6 8 12 6 2 12
Ratio of mixture of milk and water in the containers 87.5 : 420 : : 112.5 : S2
Þ 87.5 × S2 = 420 × 112.5
æ1 3 5 ö æ5 5 7 ö
= ç ´5+ ´4 + ´ 5 ÷: ç ´ 5 + ´ 4 + ´ 5÷ 420 ´1125
è6 8 12 ø è 6 8 12 ø Þ S2 = = 540
= 106 : 230 = 53 : 115 875

Downloaded From : www.EasyEngineering.net


y
Downloaded From : www.EasyEngineering.net

o
u
rs
m
a
h
SOLUTIONS 47

b
o
o
5 1 14. (d) Let C.P. = ` x. Then,

b
6. (b) S.P. for 1 egg = ` = Rs

.w
10 2 (100 - 7) 93
S.P. = ´x = x

o
100 1 5 100 100

rd
\ C. P. for 1 egg = ´ =`
(100 + 20) 2 æ 93 ö 100

p
12
Also, ç 100 x + 48 ÷ (100 5) = x

re
Þ He bought 12 eggs for 5 rupees. è ø +

s
s
150 3 Þ 93x + 4800 = 105x

.c
7. (b) C.P. for one coconut = Rs = Rs Þ 12x = 4800 Þ x = ` 400

o
100 2

m
S.P. for one coconut = ` 2 15. (e) CP of DVD player = ` x
According to the question,
3 1
Profit on one coconut = 2 - = ` æ 22 ö
2 2 Q x ç1 + ÷ = ` 10980
1 è 100 ø
\ Profit on 2000 coconut = ´ 2000 = ` 1000
2 50
8. (a) Let C.P. = Rs 100, then M. P. = ` 150 \ x = 10980 × = ` 9000
61
S.P. = 70% of 150 = ` 105 16. (c) 24 bats + 32 sticks = ` 5600
105 - 100 \ 8(3 bats + 4 sticks) = ` 5600

9. ww
\ % profit =
100
(b) C.P. of one litre = ` 6
´ 100 = 5%
Þ 3 bats + 4 sticks =
5600
8
= ` 700

w.E
After adding water to it

One has to pay Rs 7.20 for


2
3
litre of milk.
17. (a) Cost price =
1754 + 1492
2
= ` 1623

10500 ´ 100 ´ 100


So S.P. of
2
3 asy
litre of milk = Rs 7.20
18. (c) Required amount =
19. (e) 15 pendants + 24 chains
120 ´ 140
= ` 6250

Þ S.P. of 1 litre of milk = Rs


Q S.P. > C.P.
7.20 ´ 3
2
=` 10.80
En = 3 (5 pendants + 8 chains)
= 3 × 145785 = ` 437355
20. (c) CP = Rs 153, desired gain 20%

Hence gain =
10.80 - 6
´ 100 =
4.80
´ 100 gin Þ SP = 153 × 1.2 = Rs 183.60
Let the marked price be Rs x

= 0.80 × 100 = 80%


6 6

10. (a) He gives 800 grams but charges the price of 1000 grams Þ x=eer
Then, x ´ 0.85 = 183.60
183.60
= Rs. 216
(1 kg)
Þ on every 800 grams, he gains (1000 – 800) grams i.e.
200 grams.
21.
ing
0.85
(d) Applying successive discounts of 10%, 12% and 15%
on 100, we get 100 ´ 0.9 ´ 0.88 ´ 0.85 = 67.32
\ His gain % =
200
800
´ 100% = 25%
.ne
Þ Single discount = 100 – 67.32=32.68
Hence, none of the given options is correct.

Short cut : Gain % =


error
true weight–error

=
200
´ 100 = 25%.
22. (b) Let the cost price of the machine be Rs x.

Then, selling price at a profit of 10% = Rs t


11x
10

1000 - 200 9x
And the selling price at a loss of 10% = Rs
11. (d) Here, SP1 = SP2 10
CP1 6 3 Consequently, we find that
Þ 140 CP1 = 60CP2 Þ = =
CP2 14 7 æ 11x 9x ö
çè - ÷=
3 10 10 ø
\ CP1 = ´ 8000 = ` 2400
(3 + 7) x
and CP2 = 8000 – 2400 = ` 5600 Þ = 80 Þ x = Rs 400
5
12. (b) Let the C.P. of the goods be ` 100 23. (b) Let CP be Rs x and SP be Rs y.
Þ Marked price of the goods = ` 120 Then, profit = Rs (y – x)
Discount = 10% Þ S.P. is 90% of Rs 120 = ` 108 If SP = 2y, then profit = 3(y – x)
\ Gain% = (108 – 100) = 8%. Now, 2 y - x = 3( y - x )
13. (d) Let his loss = ` x. Then,
C.P. = 5000 + x = 5600 – 2x Þ y = 2x
Þ 3x = 600 Þ x = 200 y-x 2x - x
\ C.P. =5000 + 200 = Rs 5200 Now, profit = ´ 100 = ´ 100 = 100%
x x

Downloaded From : www.EasyEngineering.net


y
Downloaded From : www.EasyEngineering.net

o
u
rs
m
a
h
48 101 SPEED TEST

b
o
As, we know, profit = selling price – cost price

o
24. (c) Let CP of 12 pencils = SP of 10 pencils = Re 1

b
\ Cost price of 12 note book = 12x – 2x = 10 x

.w
1
Therefore, CP of 1 pencil = Rs and 2x

o
12 \ Profit ( percentage) = ´ 100 = 20%

rd
10x
1

p
SP of 1 pencil = Rs

re
10 SPEED TEST 39

s
s
.c
1 1 2 1
Profit on one pencil = - = = Rs P× R × T

o
10 12 120 60

m
1. (b) Simple Interest =
100
% profit = 1/ 60 ´ 100 = 20% 31400 ´ 8 ´12
1/12 =` 30144
25. (d) If any two transactions of SP is the same and also gain 100
% and loss % are same then there is always a loss \ Required amount = ` (31400 + 30144)
2 2
= ` 61544
æ Common gain or loss% ö æ 10 ö (26350 - 21250) ´ 100
\ loss % = ç ÷ = ç ÷ = 1% 2. (e) Rate =
è 10 ø è 10 ø 21250 ´ 6
26. (a) Let the cost price of geyser be Rs x. Then

ww
x × 1.1 × 1.15 × 1.25 = Rs 1265

Þ x=
1265
= Rs.800 3.
510000
=
127500
= 4%
(c) Let the required time = t years
27.
w.E 1.58125
(a) Money paid to buy watch = Rs. 1950.
Money paid as interest of 10% on Rs. 1950 = Rs. 195
\ Total money paid = Rs. 2145
Simple interest = (11442 – 9535) = `1907

Simple =
P×T×R
100

28. (c) Let the cost price be Rs 100.


\ Marked price is Rs 135.
asy
Since S.P. = Rs. 2200, therefore the man gained Rs. 55.
1907 =
9535 ´ 4 ´ t
100
At 10% discount, the customer has to pay
= Marked price – discount = 135 – 13.5 = 121.5. En \t=
1907 ´ 100
9535 ´ 4
= 5 years

\ % profit = 21.5% = 21 %.
1
2 gin
4. (b) Let the principal be = `100
\ Simple interest
29. (a) S.P. of the 1st chair = Rs. 500
Gain = 20% =
eer
100 ´ 8 ´ 6
100
= `48
\ Amount (100 + 48) = `148
\ C.P. of the 1st chair =
500 ´ 100 500 ´ 100
100 + 20
=
120
ing
\ When the amount is = `148, the principal = `100
\ When amount = `28046, the principal
=
1250
3
S.P. of the 2nd chair = Rs. 500
100
=
48
× 28046 = `18950
.ne
Loss = 12%

\ C.P. of the 2nd chair =


500 ´ 100 500 ´ 100
100 - 12
=
88
5.
\ Simple interest = (`28046 – 18950) = `9096

(c) Simple interest =


P×R×T
100
56500 × 3 × 12
t
6250 = ` = 20340
= 100
11
\ Required amount = ` (56500 + 20340) = `76840
Now S.P. of both the chairs = Rs. 1000
C.P. of both the chairs 8376 ´ 100
6. (d) Amount invested =
1250 6250 13750 + 18750 32500 8´ 6
= + = = = ` 17450
3 11 33 33 P´ 4´ 5
7. (d) 2000 =
\ Net gain = 1000 – 32500 = 500 100
\ P = 10000
33 33
éæ 4 ö
2 ù
500 33 500 Now, CI = 10000 êç1 + ÷ - 1ú = 10000 × 0.0816 = 816
Þ Gain % = ´ 100 = ´ 100
ëêè
100 ø
32500 33 32500 ûú
1120 ´ 8.5 ´ 3
100 20 8. (a) Amount = 11200 + = 11200 + 2856 = 14056
= = = 1.5% (To one place of decimal) 100
65 13 9. (b) Ratio of two parts = r2 t2 : r1 t1 = 54 : 50 = 27 : 25
30. (b) Let x be the selling price of 1 notebook. 2600
\ Selling price of 2 note book = 2x = profit \ Sum lent out at 10% = ´ 27 = ` 1350
52

Downloaded From : www.EasyEngineering.net


y
Downloaded From : www.EasyEngineering.net

o
u
rs
m
a
h
SOLUTIONS 49

b
o
o
10. (d) It dobles in 10 yrs.
(c) Difference in S.I. = P ´ T (R1 - R 2 )

b
Then trebles in 20 yrs. 17.

.w
11. (b) Suppose the rate of interest = r% and the sum = ` A 100

o
A+ r´4 P ´ 4´ 2

rd
Now, A + = 600 ; Þ 56 = (Q R1 – R2 = 2)

p
100 100

re
Ar 56 ´ 100

s
or, A + = 600 Þ P= = ` 700

s
.c
25 4´ 2

o
é r ù 18. (a) Q Rate = 5 paise per rupee = 5%

m
or, A ê1 + ú = 600 ...(1)
ë 25 û 200 ´ 5 ´ 7
\ S.I. = = ` 70
A´ r ´ 6 100
And, A + = 650 ;
100 19. (c) Let one gets = ` x
é 3r ù
then, second gets = ` (68,000 – x)
or, A ê1 + ú = 650 ...(2) Given : A1 = A2
ë 50 û
Dividing (1) by (2), we have x ´10 ´ 8 (68000 - x) ´10 ´ 6
x+ = (68, 000 - x) +
r 100 100
1+

ww
1+
25 = 600 ; or,
3r 650
50
(25 + r ) ´ 2 12
50 + 3r
=
13
Þ x[100 + 80] = (68, 000 - x)[100 + 60]

Þ
180x
= 68, 000 – x

w.E
or,
(25 + r ) ´ 2 12
50 ´ 3r
=
13
or, (50 + 2r) × 13 = (50 + 3r) × 12
160
Þ 34x = 68000 ´16 Þ x = Rs32,000
\ second gets = ` 36,000

12. (a)
\ r = 5%
Let the sum be ` x. asy
or, 650 + 26r = 600 + 36r; or, 10r = 50
20. (b) Interest for one year = ` 212.50 ´
3
100
´1 = `
51
8

\ Interest =
x ´ 8 ´ 4 32 x
100
=
100
En 21.
Thus in 8 years, the interest is ` 51.
(c) Shortcut method :
If borrowed amount be ` M and it is to be paid in equal
x-
32 x 68 x
=
100 100
gin instalments, then

M = na +
ra
´
n(n - 1)
When interest is
68 x
100
less, the sum is ` x.
eer 100 ´ Y 2
where Y = no. of instalments per annum
a = annual instalment
\ when interest is 340 less, the sum is

13. (b) S. I. = 81 – 72 = ` 9
x
68x
´100 ´ 340 = ` 500
ing
Here, M = 4200, y = 1, r = 10, n = 5, a = ?

4200 = 5a +
10a 5(5 - 1)
´
\T=
9 ´100 ´ 4
72 ´ 25
= 2 years
100 2
Þ 4200 = a [5 + 1] Þ 6a = 4200 .ne
14. (a) S.I. for 1½ years = ` (1164 – 1008) = ` 156
æ 156 ´ 2 ´ 2 ö
S. I. for 2 years = ` ç
è 3 ÷ = Rs 208
ø
22.
Þ a = ` 700
t
(d) We need to know the S.I., principal and time to find the
rate. Since the principal is not given, so data is
inadequate.
\ Principal = ` (1008 – 208) = ` 800 23. (c) Let the principal be P and rate of interest be R%.
Now , P = 800, T= 2, S.I. = 208
éæ P´ R ´ 6 öù
æ 100 ´ 208 ö ê ç 100 ÷ ú 6PR 6
\ Rate = ç ÷ % = 13% è øú =
è 800 ´ 2 ø \ Required ratio = ê = = 2 : 3.
ê æ P ´ R ´ 9 ö ú 9PR 9
15. (b) S.I. for 5 years = ` (1020 –720) = ` 300 ê çè 100 ÷ø ú
ë û
300
SI. for 2 years = Rs ´ 2 = Rs 120 24. (a) Let the sum be ` x. Then,
5
\ Principal = ` (720 – 120) = ` 600 æ x ´ 6 ´ 3 ö æ x ´ 9 ´ 5 ö æ x ´ 13 ´ 3 ö
ç 100 ÷ + ç 100 ÷ + ç 100 ÷ = 8160
Now, P = 600, T = 2, S.I. = 120 è ø è ø è ø
120 ´100 Þ 18 x + 45x + 39x = (8160 × 100) Þ102x = 816000
\R= = 10% Þx = 8000.
600 ´ 2 25. (b) Let the sum be ` 100. Then,
5
1.53 ´10 ´ 20 æ 100 ´ 10 ´ 1ö
16. (b) Let S.I. = ` x = = `2500. S.I. for first 6 months = ` ç = Rs. `
100 è 100 ´ 2 ÷ø

Downloaded From : www.EasyEngineering.net


y
Downloaded From : www.EasyEngineering.net

o
u
rs
m
a
h
50 101 SPEED TEST

b
o
o
æ 105 ´ 10 ´ 1ö 4000 ´ 5 ´ 2

b
S.I. for last 6 months = ` ç =` 5.25. 5. (a) Simple interest = = ` 400
è 100 ´ 2 ÷ø

.w
100

o
So, amount at the end of 1 year = ` (100 + 5 + 5.25) Compound interest

rd
= ` 110.25. 5 ö
2
= 4000 æç 1 +

p
\ Effective rate = ` (110.25 – 100) = 10.25%. - 4000

re
è ÷
26. (d) Let the rate be R% p.a. Then, 100 ø

s
s
4000 ´ 105 ´ 105

.c
æ 5000 ´ R ´ 2 ö æ 3000 ´ R ´ 4 ö = - 4000 = 4410 – 4000 = ` 410
÷+ç ÷ = 2200

o
ç 100 ´ 100

m
è 100 ø è 100 ø
\ Difference = 410 – 400 = ` 10
æ 2200 ö 2
Þ 100R + 120R = 2200 Þ R = ç ÷ = 10. 5 ö
è 220 ø 6. (a) Compound interest = 35000 æç 1 + ÷ – 35000
è 100 ø
27. (b) Let the savings be X and Y and the rates of simple
= 38587.50 – 35000 = ` 3587.50
interest be 5x and 4x respectively.
8730 ´ 100
1 1 1 1 X 4 7. (d) Principle = = 48500
Then, X ´ 5x ´ ´ = Y ´ 4x ´ ´ or = , 6´ 3
2 100 2 100 Y 5
i.e., X : Y = 4 : 5. éæ 6 ö
2 ù
28.
ww
(a) Let the principal be P, then amount after 12 years = 2P
Þ SI = (2 P - P ) = P
Compound Interest = 48500 êç1 +
ëêè
= 48500 × 0.1236 = ` 5994.60
÷ - 1ú
100 ø
ûú

w.E P´r ´t P ´ r ´ 12 éæ ù 594.5 æ


2 2
r ö r ö
Now, I = ÞP= 8. (a) 594.5 = 5800 êç1 + ÷ - 1ú = = ç1 + ÷ -1
100 100 êëè 100 ø úû 5800 è 100 ø
100 25 1 2
or r = = =8 %
29.
12 3
(b) Let the sum be Rs x
3

x ´ 8 ´ (3 - 2)
asy æ
0.1025 + 1 = ç1 +
r ö
è 100 ø
÷

(100 + r )2
Now, 56 =
100
Þ x = Rs 700
En 1.1025 =
10000
1.1025 × 10000 = (100 + r)2.
30. (a) SI = 10, 000 ´ 6 ´ 9 = Rs 450
12 ´ 100
gin 11025 = (100 + r)2.
105 = 100 + r
r = 5%
SPEED TEST 40
9. (d)
eer éæ
Compound interest = 7400 êç 1 +
27 ö
êëè 200 ø
2 ù
÷ - 1ú
úû

ing
2
æ 5 ö
1. (a) Required Amount = 15000 ç 1 + ÷ = ` 16537.50 éæ 227 ö 2 ù é 227 ´ 227 - 200 ´ 200 ù
è 100 ø = 7400 êç ÷ - 1ú = 7400 ê ú
êëè 200 ø úû ë 200 ´ 200 û
2.
æ Rate ö
(b) Amount = Principal ç1 +
è ÷
100 ø
Time

é 51529 - 40000 ù
= 7400 ê
11529
ú = 7400 ´ 40000 .ne
éæ
=1250 êç1 +
8 ö
ëêè 100 ø
2
ù
÷ - 1ú
úû
æ 108 ´108 -100 ´100 ö 1250 ´1664
10. (c)
ë

éæ
40000
= 2132.865 = ` 2132.87

C. I. êç 1 +
r ö
êëè 100 ø
T ù
÷ - 1ú
úû
û

t
=1250 ´ ç ÷ = = ` 208
è 100 ´100 ø 10000 éæ r ö
2 ù
4676.25 = 14500 ê ç 1 + ÷ - 1ú
Time êëè 100 ø úû
æ Rate ö
3. (e) Amount = Principal = çè1 + ÷
100 ø 4676.25 æ r ö
2
4676.25 æ r ö
2
Þ = ç1 + ÷ -1 Þ + 1 = ç1 + ÷
3 14500 è 100 ø 14500 è 100 ø
æ 5 ö 21 21 21
= 5690 ç 1 + ÷ = 5690 × 20 ´ 20 ´ 20 = `6586.90 2
è 100 ø 4676.25 ´ 14500 æ r ö
Þ = ç1 + ÷
\ Compound interest = `(6586.9 – 5690) = `896.9 » 897 14500 è 100 ø
30240 ´ 100 19176.25 r r
4. (e) Rate = = 12% Þ = 1+ Þ 1.3225 = 1 +
84000 ´ 3 14500 100 100
Compound interest
13225 r 115 r r 115
3 Þ = 1+ Þ = 1+ Þ = -1
æ 12 ö 10000 100 100 100 100 100
= 84000 ç 1 + ÷ – 84000
è 100 ø r
=
115 - 100 r
=
115
Þ Þ Þ r = 15%
= 118013.95 – 84000 = ` 34013.95 100 100 100 100

Downloaded From : www.EasyEngineering.net


y
Downloaded From : www.EasyEngineering.net

o
u
rs
m
a
h
SOLUTIONS 51

b
o
o
11. (c) Compound interest accrued half-yearly. 16. (b) Let the sum of money be Rs x.

b
R = 20% yearly = 10% half-yearly

.w
3
n = 2 years = 4 half-yearly æ r ö
Now, 8x = x ç1 +

o
è 100 ÷ø

rd
éæ r ö
n ù
CI = P êç 1 + ÷ - 1ú

p
êëè 100 ø úû

re
3
r
or, æç1 + r ö÷ = (2)3

s
or 1 + =2

s
éæ ù 4 éæ 11 ö4 ù è 100 ø 100

.c
10 ö
= 10000 êç1 + ÷ - 1ú = 10000 êç ÷ - 1ú

o
êëè 100 ø úû êëè 10 ø úû Again, let the sum becomes 16 times in n years.

m
n
é11´ 11 ´ 11 ´ 11 - 10 ´ 10 ´10 ´10 ù æ r ö
= 10000 ê Then, 16 x = x ç1 +
è 100 ÷ø
10 ´ 10 ´ 10 ´ 10 ú
ë û
é14641 - 1000 ù é 4641 ù
= 10000 ê ú = 10000 ê10000 ú = ` 4641 Þ 16 = 2 n or 2 4 = 2 n or n = 4
ë 10000 û ë û 17. (d) Required difference
2 2
æ R ö æ 14 ö é 2 ù 5000 ´ 12 ´ 2
12. (e) Required difference = P ç ÷ = 985 çè 100 ÷ø æ 12 ö
è 100 ø = ê5000ç1 + ÷ - 5000ú -

ww
= 985 ´
196
10000
= 19.306
êë è 100 ø

æ 28 28 ö
= 5000ç ´ - 1÷ - 1200
úû 100

13. (d)

w.E
Difference in amounts = 2977.54 – 2809 = ` 168.54
Now, we see that ` 168.54 is the interest on ` 2809 in one
year (it is either simple or compound interest because both
are the same for a year).
è 25 25 ø

= 5000 æç
784 - 625 ö
÷ - 1200 = Rs.72

Hence, rate of interest =


168.54 ´ 100
2809
= 6%
asy 18.
è 625 ø
(d) Let the money borrowed be Rs x and the rate of interest
charged = r%
Now, for the original sum,

æ 6 ö
2
æ 53 ö
2
En Time = 2 years
x´r´2
2809 = x ç 1 +

\ x=
è 100 ø
÷

2809 ´ 50 ´ 50
or,

= ` 2500
2809 = x ç ÷
è 50 ø
gin Now, 4000 =
100
Þ r x = 200000 ............. (i)
53 ´ 53

æ 2 ö
t eer æ
Again, x ç1 +
è
r ö
100
2
÷ = x + 4200
ø
14. (b) 6632.55 = 6250 ç1 +
è

6632.55 æ 51 ö
÷
100 ø
t
663255 æ 51 ö
t Þ ing
xr 2
+
2 xr
= 4200
or, =ç ÷
625000 è 50 ø
t
or, =ç ÷
625000 è 50 ø
t
10000 100
or 20r + 4000 = 4200
or r = 10%
[from (i)]
.ne
or,
132651 æ 51 ö æ 51 ö
=ç ÷ =ç ÷
125000 è 50 ø è 50 ø

Hence, the time is


t 3
=
2 2
\ t=3
19.
æ
(c) A = Pç1 +
r ö
è 100 ø
n
÷ ; A = Amount

P = Principal ; r = rate of interest ; n = time


t
15. (a) Whenever the relationship between CI and SI is asked for 3
years of time, we use the formula: 3
æ 105 ö
Required amount = 1000ç ÷
rt
´ CI è 100 ø
SI =
éæ r ö
r ù
100 êç 1 + ÷ - 1ú 21 21 21
ëè 100 ø û = 1000 ´ ´ ´
20 20 20
5´ 3 = Rs 1157.625
150 = ´ CI 20. (b) Let the rate of interest be r %.
éæ r ö
3 ù
100 êç1 + ÷ - 1ú
ë è 100 ø û 2520 (1 + r /100) 4
Therefore, =
2400 3
é 9261 - 8000 ù æ r ö
150 ´100 ê çè1 + ÷ø
CI = ë 8000 úû 100
5´ 3
r 21
150 ´ 100 ´ 1261 1261 Þ 1+ = or r = 5%
= = = ` 157.62 100 20
5 ´ 3 ´ 8000 8

Downloaded From : www.EasyEngineering.net


y
Downloaded From : www.EasyEngineering.net

o
u
rs
m
a
h
52 101 SPEED TEST

b
o
\ let the time in which the principal amount becomes

o
21. (d) Cash Price (C.P.)= Rs. 39,000

b
Cash Down Payment (D.P.) = Rs. 17000 16 times be n

.w
Balance Due (B.D) = Rs. 22000 n

o
P = Value of Instalment = Rs. 4800 æ 100 ö

rd
Then 16P = P ç1 + ÷
n = No. of Instalments = 5 è 100 ø

p
re
R = Rate of interest
Þ 16 = 2n Þ n = 4yrs.

s
s
æ nR ö ì (n - 1)R ü

.c
Q ç1 + ÷ BD = í1 + ý nP 24. (a) Amount = Rs. 176040, Principal = Rs. 16000

o
è 1200 ø î 2400 þ Time = 2 yrs, Rate = R

m
æ 5R ö ì 4R ü 2
æ R ö
\ çè 1 + ÷ø 22000 = í1 + 2400 ý5 ´ 4800 17640 = 16000 ç 1 + ÷
1200 î þ è 100 ø
æ 5R ö ì 4R ü 17640 æ R ö
2
æ R ö
2
Þ ç1 + ÷11 = í1 + 2400 ý12 Þ = ç1 +
è 1200 ø î þ ÷ Þ 1.1025 = ç1 + ÷
16000 è 100 ø è 100 ø
55R 24R
Þ 11 + = 12 + R R
1200 1200 Þ 1+ = 1.05 Þ = 1.05 – 1 = 0.05 Þ R = 5%

ww
Þ
55R 24R
-
1200 1200
= 1Þ
31R
1200
=1 25.
100 100
(a) Let A lent Rs. x and B lent Rs. y
Since, A and B together lent out Rs. 81600
\ x + y = 81,600

22.
w.E
Þ R=
1200
31
= 38.71%
(c) Let the amount of each instalment be Rs. x.
Now, given (r) Rate = 4%

\ 1+ r = 1 +
4
=
26

asy
Amount of Rs. 100 for 3 years = 100 æç1 + 5 ö÷
è 100 ø
3 100 25
According to the question, we have
3–2

= 100 ´
21 21 21
´ ´
20 20 20
= Rs.
9261
80 En x æ 26 ö
=ç ÷
y è 25 ø
=
26
25

Present value of Rs.


9261
80
due after 3 years = Rs. 100 gin \ Investment made by B = 81600 ´
25
51
= 40, 000

Present value of Rs. x due after 3 years


100 ´ 80
´ x = Rs.
8000x eer SPEED TEST 41
=
9261 9261
æ 5 ö
2
1. (b)
ing
Relative speed = 42 – 30 = 12 km/hr = 12 ´
5 10
=
18 3
m/s

Amount of Rs. 100 for 2 years = 100 ç1 +


è 100 ÷ø
21 21 441
Time =
Relative speed
.ne
Total length of both the trains 84 + 60
=
10
= 100 ´ ´
20 20

Present value of Rs. x due after 2 years = Rs.


=
4
400
441
x
=
144 ´ 3
10
= 43.2 seconds

120
3

t
Similarly, present value of Rs. x due after 1 year 2. (a) Speed of the train = = 24 m/s
5
21 \ time taken by the train to pass the platform
= Rs. x
20 120 + 180
= = 12.5 seconds
8000 400 20 24
\ x+ x+ x = 126100 3. (c) Length of the train = Relative speed × time
9261 441 21
Þ 8000x + 8400x + 8820x = 126100 × 9261 æ5ö 15 ´ 5 ´ 48
= (40 - 25) ç ÷ ´ 48 = = 200 m
Þ 25220x = 126100 × 9261 è ø
18 18
4. (c) Let the length of the train = x m
126100 ´ 9261
Þ x= = Rs 46305 x x + 100
25220 Then, speed of the train ==
15 25
3 or, 25x = 15x + 1500 or, 10x = 1500
æ r ö
23. (b) Given 8P = P ç 1 + ÷ \ x = 150 m
è 100 ø
Where P = Principal amount, Distance covered 2924
5. (c) Speed of bus = = = 68 kmph.
r = Compound interest rate Time taken 43
Þ r = 100% 6. (a) Distance covered = Speed × Time = 49 × 7 = 343km

Downloaded From : www.EasyEngineering.net


y
Downloaded From : www.EasyEngineering.net

o
u
rs
m
a
h
SOLUTIONS 53

b
o
o
7. (e) Let the distance between the village and the school be x x 20

b
xkm. and = + ...(ii)

.w
According to the question, y - 2 y 60

o
On solving (i) and (ii), we get x = 3y

rd
x x x + 2x Put x = 3y in (i) we get
+ = 6 or,, = 6 or,, 3x=6×4

p
re
4 2 4
3y 1

s
6´4 = 3- Þ y = 20

s
\x= = 8 km y+4 2

.c
3

o
Hence, distance = 20 × 3 = 60 km.

m
8. (a) Speed of train = (200 + 400) ´ 18 = 60 km/hr.. æ 16 ö
17. (b) Rate downstream = ç ÷ kmph = 8 kmph;
36 5 è2ø
9. (c) Distance covered in 18 seconds
æ 16 ö
5 Rate upstream = ç ÷ kmph = 4 kmph.
= 90 ´ ´ 18 = 450 m è 4 ø
18 1
\ length of platform = 450 – 160 = 290 m \ Speed in still water = (8 + 4) = 6 km/h.
2
(64 - 48) ´ 60 18. (c) After 5 minutes (before meeting), the top runner covers
10. (b) Stoppage minutes per hour =

ww
= 15 minutes.
480
64 2 rounds i.e., 400 m and the last runner covers 1 round
i.e., 200 m.
\ Top runner covers 800 m race in 10 minutes.

w.E
11. (c) Speed of bus =
12
= 40 km/hr

Speed of train = 40 ´ 9 = 72 km/hr


19. (a) Average speed =
Total distance
Total time
400 ´ 4 ´ 9 400 ´ 4 ´ 9

Speed of car =
72
18
5

´ 13 = 52 km/hr asy =
88 + 96 + 89 + 87
= 40 metres /minutes
=
360

Distance covered by car = 52 × 5 = 260 km


En 20.
æ 750 ö
(a) Speed = ç ÷ m / sec = 5 m / sec
è 150 ø
12. (a) Length of platform = 126 ´

540
5
18
× 24 – 300 = 540 meter
gin æ 18 ö
= ç 5 ´ ÷ km / hr = 18 km / hr.
è 5ø
\ Speed of man =
5 ´ 60
= 1.8 meter/second

13. (b) Distance = 64 × 8 = 512 km


21.
eer
(a) Let each side of the square be x km and let the average
speed of the plane around the field be y km/h. Then,

\ Speed =
512
6
= 85 km/hr (approx.)
x
+
x
+
ing
x
+
x
=
200 400 600 800 y
4x

14. (d) Net distance gained by car over the bus


= 40 + 60 = 100m, in 20 sec.
Þ
25x 4x
2400 y
= Þ y=ç
è 25 ø
.ne
æ 2400 ´ 4 ö
÷ = 384.

Time =
Distance
Relative speed
Þ 20 =
æ
è
100

ç 36 ´ ÷ - S2
18 ø
22.

23.
\ Average speed = 384 km/h.
(d) Let after t hours they meet then,
3t + 4t = 17.5 Þ t = 2.5
\ Time = 10 am + 2.5 h = 12 : 30pm
(a) Let original speed = S km/h
t
Þ S2 = 5 m/s = 18 kmph.
15. (b) Relative speed of the trains Here, distance to be covered is constant
= (72 – 54) km/h = 18 km/h æ 20 ö
\ S ´ 8 = (S + 5) ç ÷
æ 5ö è 3 ø
= ç 18 ´ ÷ m/sec = 5 m/sec.
è 18 ø 20 100 100
Þ 8S - S= ÞS= = 25 km / h
Time taken by the trains to cross each other 3 3 4
= Time taken to cover (100 + 120) m at 5 m/sec 24. (c) Let the speed of the bus be x km / h.
æ 220 ö then speed of the car = (x + 25) km / h
=ç ÷ sec = 44 sec. 500 500
è 5 ø \ = + 10
16. (a) Let the distance be x km. Let speed of train be y km/h. x x + 25
Then by question, we have Þ x2 + 25x – 1250 = 0 Þ x = 25
Thus speed of the bus = 25 km/h
x x 30 Speed of the car = 50 km/h
= - ...(i)
y + 4 y 60 Alternative:
Difference in speeds 25 km / hr is in only option (c).

Downloaded From : www.EasyEngineering.net


y
Downloaded From : www.EasyEngineering.net

o
u
rs
m
a
h
54 101 SPEED TEST

b
o
o
25. (a) Distance to be covered by the thief and by the owner is 1 1

b
same. 3. (a) A’s 1 day’s work = and B’s 1 day’s work = .

.w
Let after time 't', owner catches the thief. 18 9

o
æ 1 1ö 1

rd
æ 1ö \ (A + B)’s 1 day’s work = ç + ÷ = .
\ 40 ´ t = 50 ç t – ÷

p
è 2ø è 18 9 ø 6

re
s
s
5 1 4. (c) Let C completes the work in x days.

.c
Þ 10t = 25 Þ t = hr = 2 hr

o
2 2 1

m
26. (a) Let the duration of the flight be x hours. Then, Work done by (A + B) in 1 day =
10
600 600 600 1200 1
- = 200 Þ - = 200
x 1 x 2x + 1 Work done by (B +C) in 1 day =
x+ 18
2 A’s 5 days’ work + B’s 10 days’ work
Þ x (2x + 1) = 3 Þ 2x2 + x – 3 = 0 Þ (2x + 3) (x – 1) = 0 + C’s 15 days’ work = 1
Þ x = 1 hr. [neglecting the –ve value of x]. or (A + B)’s 5 days’ work + (B + C)’s 5 days’ work
180 180 + C’s 10 days’ work = 1
27. (d) Required difference = - = 15 km

28.
ww 3 4
(c) Let the husband and the wife meet after x minutes. 4500
metres are covered by Pradeep in 60 minutes.
or
5 5 10
+ + =1
10 18 x
\ x = 45 days

w.E
In x minutes, he will cover
Similarily,
4500
60
x metres. 5. (b) In one min, (A + B) fill the cistern =
1 1 1
+ = th
10 15 6

In 3 mins, (A + B) fill the cistern = 3 = 1 th


In x minutes, his wife will cover
asy
3750
60
x m.
1 1
6 2

Now,
4500
60
x+
3750
60
x = 726
En Remaining part = 1 - =
2 2

Þ x=
726 ´ 60
8250
= 5.28 min
gin Q

1
1
10
th part is filled by A in one min.

29. (b) Downstream speed = 15 + 5 = 20 km/h.


\ Required distance = 20 ´ 24 = 8km.
\
2
eer
\ Total time = 3 + 5 = 8 min.
1
nd part is filled by A in 10 ´ = 5 min .
2

30.
60
(c) Let the distance travelled during both upward and
downward journey be x km.
6.
ing
(b) Ratio of times taken by A and B = 100 : 130 = 10 : 13.
Suppose B takes x days to do the work.
Then, 10 : 13 : : 23 : x

Average speed =
Total distance covered
Total time taken
æ 23 ´ 13 ö
Þx =ç
è 10 ø
÷ Þx=
299
10
.
.ne
=
x
x+x
+
x
=
2
28 + 16
16 28 28 ´16
=
2 ´ 28 ´16
44
= 20.36 km / h A’s 1 day’s work =
1
23
; B’s 1 day's work =

æ 1 10 ö 23 1
10
299t.

(A + B)’s 1 day’s work = ç + ÷ = = .


SPEED TEST 42 è 23 299 ø 299 13
\ A and B together can complete the job in 13 days.
7. (a) Sunil takes 5 days and Pradeep takes 15 days to do the
12 ´ 8 work.
1. (c) Number of days = = 24 days
12 - 8
1 1 4 th
2. (a) In an hour, George and Sonia together can copy In a day they would complete + i.e., work.
5 15 15
1 1 7 11
+ = of a 50-page manuscript. The remaining th work would be completed by
6 8 24 15
7 11
i.e. In an hour they together can copy of the Pradeep in ´ 15 i.e. 11 days.
48 15
100-page manuscript. 8. (a) Let 1 man’s 1 day’s work = x and
i.e. They together can copy a 100-page manuscript in 1 boy’s 1 day’s work = y.
1 1
48 6 Then, 6x + 8y = and 26x + 48y = .
hours, i.e. 6 hours. 10 2
7 7

Downloaded From : www.EasyEngineering.net


y
Downloaded From : www.EasyEngineering.net

o
u
rs
m
a
h
SOLUTIONS 55

b
o
Solving these two equations, we get :

o
Part filled by (A + B + C) in 1 min.

b
.w
1 1 æ 1 1 1 ö 1
x= and y = . =ç + - ÷= .

o
100 200 è 36 45 30 ø 60

rd
\ (15 men + 20 boys)’s 1 day’s work 1

p
13. (d) (Man + Son)’s one day’s work =

re
æ 15 20 ö 1 8
=ç + ÷= .

s
è 100 200 ø 4 1

s
.c
Man’s one day’s work =
\ 15 men and 20 boys can do the work in 4 days. 10

o
m
1 1 1 1
9. (a) 1 man’s 1 day’s work = . Þ Son’s one day’s work = - =
108 8 10 40
\ Son can do it in 40 days.
æ1 ö 2 3
12 men’s 6 day’s work = ç ´ 6 ÷ = . 14. (d) Q A can do of the work in 12 days
è9 ø 3 4
æ 2ö 1 1 4 1
Remaining work = ç 1 - ÷ = . \ A can do of the work in 12 ´ ´ days = 2 days
è 3ø 3 8 13 8
15. (b) (A + B)’s 5 days’ work
æ 1 ö 1

1ww
18 men’s 1 day’s work = ç
è 108
´18 ÷ = .

work is done by them in 1 day..


ø 6
æ 1 1 ö 45
=5 ç + ÷ = =
9
è 25 20 ø 100 20

6
\ w.E
1
3
work is done by them in 6 ´ 1 = 2 days
3 11
æ
Remaining work = ç1 - ÷ =
9 ö 11
è 20 ø 20

10. (c) A’s one day’s work =


1
16
th work
asy 20
of the work would be finished by B in

11

B’s one day’s work =


1
12
th work
En 20 = 11days.
1
Let the number of days B has worked alone = x days.
Then,
A’s amount of work + B’s amount of work = 1 gin
16.
20
(a) 50 men complete 0.4 work in 25 days.
Applying the work rule, m1 ´ d1 ´ w 2 = m 2 ´ d 2 ´ w1
æ1 ö æ 1ö
Þ 4 ç ÷ + (x + 4) ç ÷ = 1
16
è ø è 12 ø eer
we have,
50 ´ 25 ´ 0.6 = m 2 ´ 25 ´ 0.4

Þ
1 x+4
4
+
12
3
= 1 Þ x = ´ 12 - 4 Þ x = 5 days
4
or m2 =
ing
50 ´ 25 ´ 0.6
25 ´ 0.4
= 75 men

æ 1 1ö 9
11. (c) (A + B)'s 1 hour's work = ç + ÷ =
è 12 15 ø 60
=
3
20
17.
.ne
Number of additional men required = (75 – 50) = 25
(c) Let C completes the work in x days.

æ 3
æ 1
(A + C)'s 1 hour's work = ç + ÷ =
1 ö 8

2 ö 17
=
è 12 20 ø 60 15
2 Work done by (A + B) in 1 day =

Work done by (B +C) in 1 day =


1
1
10
t
Part filled in 2 hrs = ç + ÷ = 18
è 20 15 ø 60 A’s 5 days’ work + B’s 10 days’ work + C’s 15 days’
æ 17 ö 17 work = 1
Part filled in 6 hrs = ç 3 ´ ÷ = or (A + B)’s 5 days’ work + (B + C)’s 5 days’ work
è 60 ø 20
+ C’s 10 days’ work = 1
æ 17 ö 3
Remaining part = ç 1 - ÷ = 5 5 10
è 20 ø 20 or + + = 1 or x = 45 days
3 10 18 x
Now, it is the turn of A and B and part is filled by 1
20 18. (d) In 1 day, work done by 12 men =
A and B in 1 hour. 18
\ Total time taken to fill the tank = (6 + 1) hrs = 7 hrs. 6 1
In 6 days, work done by 12 men = =
æ 1 1 ö 7 18 3
12. (a) Part filled in 7 min. = 7 × ç + ÷ = 2
è 36 45 ø 20 Remaining work =
3
æ 7 ö 13
Remaining part = ç1 - ÷ = Now, m1 ´ d1 ´ w 2 = m 2 ´ d 2 ´ w1
è 20 ø 20

Downloaded From : www.EasyEngineering.net


y
Downloaded From : www.EasyEngineering.net

o
u
rs
m
a
h
56 101 SPEED TEST

b
o
24. (a) Let 1 man’s 1 days’ work= x & 1 boy’s 1 day’s work = y

o
2

b
or 12 ´ 18 ´ = 16 ´ d 2 ´ 1

.w
3 1 1
Then, 2x + 3y = and 3x + 2y =

o
4 ´ 18 ´ 2 10 8

rd
or d2 = = 9 days
16 7 1

p
Solving, we get : x = and y =

re
1 200 100

s
19. (c) 10 men’s 1 day’s work = ; \ (2 men + 1 boy)’s 1 day’s work

s
15

.c
æ 7 1 ö 16 2

o
1 = çè 2 ´ + 1´ ÷= =

m
15 women’s 1 day’s work = . 200 100 ø 200 25
12
(10 men + 15 women)’s 1 day’s work So, 2 men and 1 boy together can finish the work in
æ 1 1ö 9 3 1
= ç + ÷= = . 12 days.
è 15 12 ø 60 20 2
\ 10 men and 15 women will complete the work in 25. (a) Ratio of time taken by A and B = 160 : 100
=8:5
20 2 Suppose, B alone takes x days to do the job
= 6 days.
3 3 then, 8 : 5 : : 12 : x
20.
ww 5 1
æ 1
(a) Work done by A and B in 5 days = ç + ÷ ´ 5 =
è 10

15 ø
5
6
8x = 5 × 12

x=
5 ´ 12
8
1
= 7 days.
2

w.E
Work remaining = 1 - =
6 6
\ C alone can do the work in 6 × 2 = 12 days
5 5 2
æ 1 1 ö 15
26. (c) Son’s 1 day’s work = ç - ÷ =
è 3 5ø 2
1
= 7 days
2
Ratio of their share work = : :
10 15 12
Share of wages = ` 225, ` 150, ` 75. asy
= 3 : 2 :1 27. (a) If x complete a work in x days. y will do the same task in
3x days.
3x – x = 40
21. (d) 1 man’s 1 day’s work =
1
100
.
En Þ x = 20
y will finish the task in 60 days
(x + y)’s 1 days work
(10 men + 15 women)’s 1 day’s work = .
15 women’s 1 day’s work
1
6
gin =
1
+
1
20 60 15
=
1

æ 1 10 ö æ 1 1 ö 1
=ç - ÷=ç - ÷= .
è 6 100 ø è 6 10 ø 15 eer
Both of them will complete the work in 15 days.
æ1 1 1 ö 1

\ 1 woman’s 1 day’s work =


1
225
. ing
28. (c) Part filled by (A + B + C) in 1 hour = ç + + ÷ = .
è 5 10 30 ø 3
\ All the three pipes together will fill the tank in

22.
\ 1 woman alone can complete the work in 225 days.
(a) Let the number of men originally employed be x.
9x = 15(x – 6)
3 hours.

29. (d) Part filled by first tap in one min = .ne 1


th

23.
or x = 15
1
(c) In 8 days, Anil does = rd work .
3
Part filled by second tap in one min =
12
1
18
th t
Now, 2 éê + ùú + unfilled part = 1
1 1
1
\ in 1 day, he does = th work. ë12 18 û
24
1 1 13
\ Rakesh’s one day’s work = 60% of = th work . Þ unfilled part = th
24 40 18
1 2 Q 1 th part of tank is filled by second tap in 1min.
Remaining work = 1 - = 18
3 3
(Anil and Rakesh)’s one day’s work
\ 13 th part of tank is filled by second tap in 1 min.
1 1 1 18
= + = th work
24 40 15 13
= 18 ´ min = 13 min.
1 18
Now, th work is done by them in one day..
15 30. (c) Hint : Let the time be t hours after 1 a.m.
2 2 t ( t - 1) ( t - 2 )
\ rd work is done by them in 15 ´ = 10 days \ + - =1
3 3 4 5 2

Downloaded From : www.EasyEngineering.net


y
Downloaded From : www.EasyEngineering.net

o
u
rs
m
a
h
SOLUTIONS 57

b
o
o
11. (c) Let the ratio of proportionality be x, then

b
SPEED TEST 43 4x × x = 196 or, 4x2 = 196 or, x = 7

.w
Thus, Father’s age = 28 yrs, Son’s age = 7 yrs

o
1. (c) Let the age of father and son be 15x years and x years After 5 yrs, Father’s age = 33 yrs.

rd
respectively. Son’s age = 12 yrs

p
15x + x \ Ratio = 33 : 12 = 11 : 4

re
Now, according to the question, = 16 12. (b) Difference in ratios = 8
2

s
Then 8 º 24 \ 1 º 3

s
16 ´ 2

.c
or, x = = 2years i.e., value of 1 in ratio is equivalent to 3 yrs
16

o
Thus, Rita’s age = 3 × 3 = 9 yrs

m
Hence age of the son = 2 years
Mother’s age = 11 × 3 = 33 yrs.
2. (b) Average age = 28.5
After 3 years, the ratio = 12 : 36 = 1 : 3
\ Total age = 28.5 × 2 = 57
13. (c) Let the present age be x yrs. Then
5
\ Daughter’s age = ´ 57 = 15 years 1
19 125% of (x – 10) = x; and 83 % of (x + 10) = x
3
3. (a) Let Sudha’s and Neeta’s present ages be 6x and 7x years
1
respectively. \ 125% of (x – 10) = 83 % of (x + 10)
According to the question. 3
6x - 5 5 5 5
= (x = 10) = (x + 10)
7x - 5 6 4 6

ww
Þ 36x – 30 = 35x – 25
Þx=5
\ Sudha’s present age = 6 × 5 = 30 years
5
or, x - x =
4
5x 250
5
6
50 50
6
+
4

4. (a)
w.E
Required average age = ç

= ç
æ 540 + 192 ö
÷ years = ç
æ 15 ´ 36 + 12 ´ 16 ö
è
æ 732 ö
36 + 12
÷ years
ø

÷ years = 15.25 years.


14. (b)
or,

Now,
Also,
=
12 12

x
\ x = 50 yrs.
Let the father’s present age be x and son’s age be x1 and x2.
= 3(x1 + x2)
x + 5 = 2(x1 + 5 + x2 + 5)
.....(i)

5. (a)
è 48 ø è 48 ø

asy
Let the ages of Swati and Khyati two years ago be 5x and 7x
years respectively.
x + 5 = 2(x1 + x2 + 10)
Putting value of (x1 + x2) =
x
3
.....(ii)

from (i) in equation (ii)


According to the question,
5x + 4 7
=
7x + 4 9 En æx
x + 5 = 2 ç + 10 ÷ = 45
è3
ö
ø
Þ 49x + 28 = 45x + 36
Þ 4x = 8 Þ x = 2
\ Khyati’s present age = 7x + 2 = 7 × 2 + 2 = 16 years gin
15. (a) Let the present ages of P and Q be 3x and 4x respectively.
After 4 years
4x – 3x = 5
6. (b) Shortcut method :

Son’s age =
5(9 - 1)
= 8 yrs 16. (c)
\x=5

eer
\ P’s present age = 3x = 3 × 5 = 15 years
Let the present ages of Rama and Shyama be 4x and 5x years

7. (b)
(9 - 4)
\ Father’s age = 4 × 8 = 32 yrs
Shortcut method : \ ing
respectively,
4x + 5 5
5x + 5 6
=

Son’s age =
5(7 - 1) + 5(3 - 1)
7 -3
= 10 yrs
From the first relationship of ages, if F is the age of the father
Þ 25x + 25 = 24x + 30
Þ x = 30 – 25 = 5
\ Rama’s present age = 4 × 5 = 20 years.ne
8. (c)
then F + 5 = 3 (10 + 5)
\ F = 40 yrs
Shortcut method :
Daughter’s age =
10(4 - 1) + 10(2 - 1)
= 20 yrs
17. (d) Let the mother’s age be y years.
\ The age of father = (y + 9) years
The age of son = years
y
2
t
4-2 æy ö
1 The age of daughter = ç - 7 ÷ years
9. (e) 10 yrs ago, A was of B’s ago. è2 ø
2 Now according to the given condition,
3 æy ö
AT present, A is
4
of B’s age. (y + 9) = 3ç - 7 ÷
è2 ø
æ1 ö 3 y - 42
10 ç - 1÷ Þ y+9=
è2 ø 2
\ B’s age = = 20 yrs
1 3 Þ 2y + 18 = 3y – 42
- Þ y = 60 years
2 4
18. (d) Suppose age of Ram = R
3
A’s age = of 20 = 15 yrs his son’s age = S
4 and his father’s age = F
10. (b) Let the age of the daugher be x yrs.
R 2
Then, the age of the mother is (50 – x) yrs. According to question, S = and R = F ´
5 yrs ago, 7(x – 5) = 50 – x – 5 3 5
or, 8x = 50 – 5 + 35 = 80 5R R +S+ F
\ F= and = 46
\ x = 10 2 3
Therefore, daughter’s age = 10 yrs R + S + F = 46 × 3
and mother’s age = 40 yrs

Downloaded From : www.EasyEngineering.net


y
Downloaded From : www.EasyEngineering.net

o
u
rs
m
a
h
58 101 SPEED TEST

b
o
o
R 5R 30. (a) Let the present age of the father be ‘x’ and that of the son be
R+ + = 138

b
3 2

.w
x 8
R = 36 ‘y’. Then =
y 3

o
36

rd
S= = 12 \ 3x = 8y ...(i)
3

p
5 ´ 36 x + 12 2

re
F= = 90 Further, y + 12 = 1 \ x + 12 = 2y + 24

s
2

s
19. (c) Let the ages of Abhay and his father 10 years ago be x and 5x

.c
\ x – 2y = 12 ...(ii)
years respectively. Then,

o
From eqn (i) and (ii), x = 48, y = 18

m
Abhay’s age after 6 years = (x + 10) + 6 = (x + 16) years. \ sum = 66 yrs.
Father’s age after 6 years = (5x + 10) + 6 = (5x + 16) years.
3 SPEED TEST 44
\ (x + 16) = ( 5x + 16 ) Û 7 (x + 16) = 3 (5x + 16)
7
Û 7x + 112 = 15x + 48 1. (c) Required number of ways
Û 8x = 64 Û x = 8. = ways of selecting 4 objects out of 6 given objects
Hence, Abhay’s father’s present age = (5x + 10) = 50 years. 6´5
= 6C4 = = 15
20. (b) Let their present ages be 4x, 7x and 9x years respectively. 2
Then, (4x – 8) + (7x – 8) + (9x – 8) = 56 Û 20x = 80 Û x = 4. 2. (c) Total no. of unrestricted arrangements = (7 – 1) ! = 6 !

21. (d)
ww
\ Their present ages are 16 yrs, 28 yrs. and 36 yrs. respectively.
16 years ago, let T = x years and G = 8x years
After 8 years from now, T = (x + 16 + 8) years and
When two particular person always sit together, the total no.
of arrangements = 6! – 2 × 5!
Required no. of arrangements = 6! – 2 × 5!
= 5! (6 – 2) = 5 × 4 × 3 × 2 × 4 = 480.

w.E
G = (8x + 16 + 8) years.
\ 8x + 24 = 3(x + 24) Û 5x = 48.

T x +8
= 5
48
+8 88 11
=
3. (c) In MATHEMATICS, the consonants M and T are repeated
two times each.
Also the vowel A is repeated two times.
Since there are four vowels, A, A, E and I; A being repeated,

22. (d)
8 years ago, =
48
G 8x + 8 8 ´ + 8 424 53
5
R – Q = R – T Þ Q = T. Also, R + T = 50asy
=
4
therefore vowels can be arranged in
2
= 12 ways.
Now remaining 7 consonants, with M, T being repeated, can

23. (e)
Þ R + Q = 50
So, (R – Q) cannot be determined.
Let the ages of father and son be x and (45 – x) years En be written in
7
2´2
= 7 × 6 × 5 × 3 × 2 = 1260 ways.
respectively.
Then, (x – 5) (45 – x – 5) = 34
Û (x – 5) (40 – x) = 34 Û x2 – 45x + 234 = 0
gin Now four vowels together can take any of the 8 places as
shown below:
VC VC VC VC VC VC VC V
Û (x – 39) (x – 6) = 0 Û x = 39 or x = 6.
\ Father’s age = 39 years and son’s age = 6 years eer
\ Total number of ways in which the letters of the word
MATHEMATICS can be arranged such that vowels always
come together = 1260 × 8 × 12 = 120960.
24. (a) Let the ages of children be x, (x + 3), (x + 6)
and (x + 12) years.
Then, x + (x + 3) + (x + 6) + (x + 9) + (x + 12) = 50
4. (c)

ing
There are 8 different letters in the word MATHEMATICS;
three letters M, A and T being repeated.
The number of ways in which four letters of the word

25. (b)
Û 5x = 20 Û x = 4.
\ Age of the youngest child = x = 4 years.
Anup’s age = (5 – 2) years = 3 years. Let Gagan’s age be x
MATHEMATICS can be arranged = 8 P4

6! .ne= 8 × 7 ×6 × 5 = 1680

26. (c)
years.
Then,
x -6
18
= 3 Û x – 6 = 54 Û x = 60.
Let the school ages of Neelam and Shaan be 5x and 6x
respectively. Then,
5.

6.
(c)

(e)
Number of ways =

=
2!
6 ´ 5 ´ 4 ´ 3 ´ 2 ´1
2 ´1
= 360
CYCLE whereas C comes two times.
t
(Q T letter comes in two time)

1 5! 5 ´ 4 ´ 3 ´ 2
´ 5x æ1 ö æ5 ö So, arrangements are = = = 60 ways
3 5 2! 2
= Û ç ´ 9 ´ 5x ÷ = ç ´ 6x ÷ Û 15 = 15.
1 9 è3 ø è2 ø 5!
´ 6x 7. (e) Required no. of ways = = 60 is
2 2!
Thus, Shaan’s age cannot be determined. Total no. of letters in the word is 5; T is repeated twice.
27. (d) Let C’s age be x years. Then, B’s age = 2x years. A’s age = (2x 8. (c) The committee of 4 persons is to be so formed that it has at
+ 2) years. least 1 woman.
\ (2x + 2) = 2x + x = 27 Û 5x = 25 Û x = 5. The different ways that we can choose to form such a
Hence, B’s age = 2x = 10 years. committee are:
28. (d) Let the present ages of the father and son be 2x and x years 6´ 5´ 4
(i) 1w. 3 m in 4C1 × 6C3 = 4 ´ = 80
respectively. 3 ´ 2 ´1
Then, (2x – 18) = 3 (x – 18) Û x = 36. 4´ 3 6´ 5
(ii) 2w. 2 m in 4C2 × 6C2 = ´ = 90
\ Required sum = (2x + x) = 3x = 108 years. 2 ´1 2 ´1
29. (d) Let the ages of Preeti and Sonal 1 year ago be 4x and 4 6
(iii) 3w. 1 m in C3 × C1 = 4 × 6 = 24
x years respectively. (iv) 4w in 4C4 = 1
Then, [(4x + 1) + 6] – [(x + 1) + 6] = 9 Û 3x = 9 Û x = 3. \ Total no. of different ways in which a committee of 4
\ Required ratio = (4x + 1) : (x + 1) = 13 : 4. persons can be formed so that it has at least one woman.
= 80 + 90 + 24 + 1 = 195

Downloaded From : www.EasyEngineering.net


y
Downloaded From : www.EasyEngineering.net

o
u
rs
m
a
h
SOLUTIONS 59

b
o
o
9. (d) The committee of 4 persons is to be so formed that it has at
\ Reqd probability = 7 = 1

b
least 2 men. The different ways that we can choose to form

.w
such a committee are: 21 3

o
6´5 3´3 22. (c) Taking all vowels (IEO) as a single letter (since they

rd
(i) 2m. 2w in 6C2 × 4C2 = ´ = 90
2 ´1 2 ´1 come together) there are six letters

p
re
6´ 5´ 4 6!
(ii) 3m. 1w in 6C3 × 4C1 ´ 4 = 80 Hence no. of arrangements = ´ 3! = 2160

s
3 ´ 2 ´1

s
2!

.c
6´5 [Three vowels can be arranged 3! ways among

o
(iii) 4m in 6C4 = = 15
themselves, hence multiplied with 3!.]

m
2 ´1
\ Total no. of different ways in which a committee of 4 23. (b) When 0 is the repeated digit like
persons can be formed so that it has at least 2 men. 100, 200, ...., 9 in number
= 90 + 18 + 15 = 185 When 0 occurs only once like
10. (e) One girl can be chosen in 4C1 = 4 ways 110, 220, ....., 9 in number
and 4 boys can be chosen in 6C4 = 15 ways When 0 does not occur like
\ Total number of ways = 4 × 15 = 60 ways
11. (a) CORPORATION= 11 letters
112, 211, ....., 2 × (8 × 9) = 144 in number.
‘O’ comes thrice, ‘R’ twice. Hence, total = 9 + 9 + 144 = 162.
11! 24. (b) Required number of possible outcomes
\ total no. of ways = = 3326400 = Total number of possible outcomes –
12. (b)
ww 3!2!
There are seven letters in the word “COUNTRY” and two
vowels O and U. Considering two vowels as one unit, total
Number of possible outcomes in which all vowels are
together
= 6 ! – 4 ! × 3 != 576

w.E
number of letters will be 5 + 1 = 6. So, number of arrangements
= 6! 25. (d) The required number of ways
Now, the two vowels can be arranged in 21 ways among = (10 + 1)(9 + 1)(7 + 1) - 1 = 879 .
themselves.
26. (c) Three digit number less then 600 will have first element
13. (c)
asy
\ Total number of ways = 6! × 2! = 1440
The word PROBLEM consists of 7 distinct letters.
\ Number of arrangements = 7!
= 70 × 6 × 5 × 4 × 3 × 2 × 1 = 5040
100, and last element 599. First place will not have digit
more than 6, hence, 7 and 9 can not be taken : So, first
digit can be selected in 4 ways. Second digit can be
14. (e)
En
There are 6 letter inthe word ‘ATTEND’ whereas, T comes
two times.
selected in 6 ways and since repetition of digits are
allowed, third digit can also be selected in 6 ways :

15. (a)
So, required number of ways =
6! 720
2!
=
2
= 360
In word ‘offices’, there are 7 letters and F comes two times. gin
27. (b)
So, number of ways are 4 × 6 × 6 = 144.
Selection of 2 members out of 11 has 11C2 number of
ways

eer
11C = 55
7! 7 ´ 6 ´ 5 ´ 4 ´ 3 ´ 2! 2
Required number of ways = = = 2520 28. (b) From each railway station, there are 19 different tickets
2! 2!
16. (e) ARMOUR = 6 letter whereas R repeated twice to be issued. There are 20 railway station

\
6! 6 ´ 5 ´ 4 ´ 3 ´ 2 ´ 1
2!
=
2 ´1
= 360 29. (c)
ing
So, total number of tickets = 20 × 19 = 380.
To make a 5 digit number, 0 can not come in the bagining.
So, it can be filled in 4 ways. Rest of the places can be

.ne
17. (c) Total number of ways to stand boys and girls together
= 4! × 3! × 2! = 4 × 3 × 2 × 3 × 2 × 2 = 288 filled in 4! ways. So total number of digit formed = 4 × 4!
18. (e) = 2 × 24 = 96
30. (b) Total no. of digits = 6
O, A, E S F T W R
When the vowels are always together, then treat all
the vowels as a single letter and then all the letters
can be arranged in 6! ways and also all three vowels
t
To form a odd numbers we have only 3 choice for the
unit digits.
Now, Extreme left place can be filled in 6 ways the
middle place can be filled in 6 ways.
can be arranged in 3! ways. Hence, required no. of \ Required number of numbers = 6 × 6 × 3 = 108
arrangements = 6! × 3! = 4320.
5 SPEED TEST 45
5 ´ 4 10
C2
19. (b) Reqd probability = 7 = =
C2 7 ´ 6 21 1. (a) Here, S = {1, 2, 3, 4, ...., 19, 20}.
Let E = event of getting a multiple of 3
20. (b) Treat B and T as a single letter. Then the remaining
= {3, 6, 9, 12, 15, 18}
letters (5 + 1 = 6) can be arranged in 6! ways. Since, O is
repeated twice, we have to divide by 2 and the B and T n(E) 6 3
\ P(E) = = = .
letters can be arranged in 2! ways. n(S) 20 10
6! ´ 2! 10 10 2
Total no. of ways = = 720
2 2. (c) P (getting a prize) = = = .
(10 + 25) 35 7
21. (e) If the drawn ball is neither red nor green, then it must be
3. (b) Clearly, there are 52 cards, out of which there are 16
blue, which can be picked in 7C1 = 7 ways. One ball can
face cards.
be picked from the total
(8 + 7 + 6 = 21) in 21C1 = 21 ways. 16 4
\ P (getting a face card) = = .
52 13

Downloaded From : www.EasyEngineering.net


y
Downloaded From : www.EasyEngineering.net

o
u
rs
m
a
h
60 101 SPEED TEST

b
o
o
4. (c) Here, n(S) = 52. n(E) 6 1

b
There are 26 red cards (including 2 kings) and there are \ P(E) = = = .

.w
2 more kings. n(S) 36 6

o
Let E = event of getting a red card or a king. 14. (d) In a simultaneous throw of two dice, we have

rd
Then, n(E) = 28. n(S) = (6 × 6) = 36.

p
re
Let E = event of getting a total of 10 or 11
n(E) 28 7

s
\ P(E) = = = . = {(4, 6), (5, 5), (6, 4), (5, 6), (6, 5)}

s
.c
n(S) 52 13
n(E) 5

o
5. (b) Total number of balls = (6 + 8) = 14 \ P(E) = = .

m
Number of white balls = 8 n(S) 36
15. (a) Total possible outcomes, S ={HHH, HHT, HTH, THT,
8 4 TTH, THH, TTT, HTT} and desired outcomes E ={HTH,
P (drawing a white ball) = = .
14 7 THT}
6. (d) Total number of balls = (8 + 7 + 6) = 21. Þ n(E) = 2 and n(S) = 8
Let E = event that the ball drawn is neither red nor green. n(E) 2 1
= event that the ball drawn is red. Hence, required probability = P(E) = = =
n(S) 8 4
\ n(E) = 8 16. (c) 16 tickets are sold and 4 prizes are awarded. A person
8

7. ww
\ P(E) =
21
.
(b) Clearly, n(S) = (6 × 6) = 36.
buys 4 tickets, then required probability =
17. (c) Total number of letters = 4
4 1
=
16 4

w.E
Let E = Event that the sum is a prime number.
Then, E = {(1, 1), (1, 2), (1, 4), (1, 6), (2, 1), (2, 3), (2, 5), (3,
2), (3, 4), (4, 1), (4, 3), (5, 2), (5, 6), (6, 1), (6, 5)}
\ n(E) = 15
Total number of vowels = 2 (O and E)

Required Probability =
2 1
=
4 2

\ P(E) =
n(E) 15 5
=
n(S) 36 12
= . asy 18. (d) Total no. of case = 63 = 216
Favourable cases = {(1, 1, 1), (2, 2, 2), (3, 3, 3), (4, 4, 4),
(5, 5, 5), (6, 6, 6)}.
8. (d) Here S = {HH, HT, TH, TT}.
Let E = event of getting at least one head En Probability =
6
=
216 36
1

= {HT, TH, HH}

\ P(E) =
n(E) 3
= .
gin
19. (b) No. of days in leap year = 366
No. of complete week = 52

9.
n(S) 4
(b) Here S = {TTT, TTH, THT, HTT, THH, HTH, HHT, HHH} \ eer (Q 366 ¸ 7 gives 2 as remainder )
No. of days left = 2
Let E = event of getting at least two heads
= {THH, HTH, HHT, HHH}
n(E) 4 1
ing
Required probability =
2
7

10.
\ P(E) = = = .
n(S) 8 2
(b) When a die is thrown, we have S = {1, 2, 3, 4, 5, 6}
20. (a) Total case = 6 × 6 = 36

.ne
Favourable = (1, 6), (2, 5), (3, 4), (4, 3), (5, 2), (6, 1)
6 1
Let E = event of getting a number greater than 4
= {5, 6}.

\ P(E) =
n(E) 2 1
= = .
21.
Probability =
36 6
=
(a) Total number of balls = 5 + 7 + 8 = 20
Probability that the first ball drawn is white
t
n(S) 6 3 5
C 1
11. (a) We know that in a simultaneous throw of two dice, = 20 1 =
C1 4
n(S) = 6 × 6 = 36.
Let E = event of getting a total f 7 If balls are drawn with replacement, all the four events
= {(1, 6), (2, 5), (3, 4), (4, 3), (5, 2), (6, 1)} will have equal probability.
Therefore, required probability
n(E) 6 1 1 1 1 1 1
\ P(E) = = = . = ´ ´ ´ =
n(S) 36 6 4 4 4 4 256
12. (c) In two throws of a die, n(S) = (6 × 6) = 36. 22. (b) Total no. of outcomes when two dices are thrown = n
Let E = event of getting a sum 9 (S) = 36 and the possible cases for the event that the
= {(3, 6), (4, 5), (5, 4), (6, 3)} sum of numbers on two dice is a prime number, are
(1, 1), (1, 2), (1, 4), (1, 6), (2, 1), (2, 3), (2, 5), (3, 2), (3, 4),
n(E) 4 1
\ P(E) = = = . (4, 1), (4, 3), (5, 2), (5, 6), (6, 1), (6, 5)
n(S) 36 9 Number of outcomes favouring the event = n (A) = 15
13. (a) In a simultaneous throw of two dice, n(S) = 6 × 6 = 36.
n ( A) 15 5
Let E = event of getting a doublet Required probability = = =
= {(1, 2), (2, 2), (3, 3), (4, 4), (5, 5), (6, 6)} n(S ) 36 12

Downloaded From : www.EasyEngineering.net


y
Downloaded From : www.EasyEngineering.net

o
u
rs
m
a
h
SOLUTIONS 61

b
o
o
1 Area of the room = Area of the carpet

b
23. (d) The probability of selecting any bag =

.w
2 æ 75 ö 2 2
= ç 180 ´ ÷ m = 135 m .

o
Now, probability of getting a white ball from the first è 100 ø

rd
1 3 3

p
bag = ´ = æ Area ö æ 135 ö

re
2 5 10 \ Breadth of the room = ç ÷ =ç ÷ m = 7.5 m.
è Length ø è 18 ø

s
and probability of getting a white ball from the second

s
.c
5. (a) In a rectangle,
bag = 1 ´ 2 = 1

o
m
2 6 6 (perimeter) 2
Required Probability = The probability that a white ball = (diagonal) 2 + 2 ´ area
4
is drawn either from the first or the second bag
(14)2
=
3 1 7
+ = Þ = 52 + 2 ´ area
10 6 15 4
24. (b) Out of 20 consecutive numbers there are 10 even and 49 = 25 + 2 × area
10 odd 49 - 25 24
We have to choose 2 numbers out of 20 \ Area = = = 12cm 2
2 2

ww
\ Total outcomes =
20
C2 =
20 ´ 9
2
= 190
We can odd sum only when one even and one odd is
6.

7.
(a) In an isoscele right angled triangle,
Area = 23.3 × perimeter2
= 23.3 × 202 = 9320 m2
(b) Required area covered in 5 revolutions

w.E
added.
\ No. of ways of choosing 1 even and 1 odd
Out of 10 even and 10 odd = 10C1 × 10 C1 = 10 × 10 = 100 = 5 × 2prh = 5 × 2 ×
22
7
× 0.7 × 2 = 44 m2

\ Required probability =
25. (b) Here S = {1, 2, 3, 4, 5}
100 10
=
190 19
asy 8. (c) In a triangle,
1
Area = ´ length of perpendicular × base
2
Let E be the event of getting a multiple of 3.
E = {3, 6}
En or 615 =
1
´ length of perpendicular × 123
\ P(E) =
2 1
= .
6 3 gin 2

\ Length of perpendicular =
615 ´ 2
= 10 m.
SPEED TEST 46
9. (a) D eer 14 m
C
123

1. (a) In a circle, circumference = 2pr

Hence, 44 = 2pr \ r=
44
24 ming 14 m

2p

Now, area of circle = p r2 = p ´


44 44
´ = 154 m 2 .ne
2.
2p 2p
(a) Let the length and breadth of a rectangle are 9 xm and
5 xm respectively.
In a rectangle, area = length × breadth
A
40 m

Area of the shaded portion


1
= ´ p(14 )2
B
t
\ 720 = 9x × 5x 4
or x2 = 16 Þ x = 4 = 154 m2
Thus, length = 9 × 4 = 36 m 10. (a) Circumference of circular bed = 30 cm
and breadth = 5 × 4 = 20 m (30)2
Therefore, perimeter of rectangle = 2(36 + 20) = 112 m Area of circular bed =
4p
3. (d) Perimeter of the circle = 2pr = 2(18 + 26) Space for each plant = 4 cm2
22 \ Required number of plants
Þ 2´ ´ r = 88 Þ r = 14
7 (30)2
= ¸ 4 = 17.89 = 18 (Approx)
\ Area of the circle 4p
2 22 11. (c)
= pr = ´ 14 ´14 = 616 cm 2 .
7
4. (b) Length of the carpet 10 W
æ Total cost ö = æ 8100 ö m = 180 m. 15
=ç ÷ ç ÷
è Rate / m ø è 45 ø

Downloaded From : www.EasyEngineering.net


y
Downloaded From : www.EasyEngineering.net

o
u
rs
m
a
h
62 101 SPEED TEST

b
o
o
Let the width of the path = W m
A

b
then, length of plot with path = (15 + 2W) m

.w
and breadth of plot with path = (10 + 2 W) m

o
rd
Therefore, Area of rectangular plot (wihout path)
= 15 × 10 = 150 m2

p
x x

re
and Area of rectangular plot (with path)

s
= 150 + 54 = 204 m2

s
.c
Hence, (15 + 2W) × (10 + 2W) = 204

o
B D C

m
Þ 4W2 + 50 W – 54 = 0
Þ 2W2 + 25 W – 27 = 0 Since, in an isosceles triangle, the altitude bisects the
Þ (W – 2) (W + 27) = 0 base. So, BD = DC = (16 – x).
Thus W = 2 or –27 In DADC, AC2 = AD2 + DC2
\ with of the path = 2 m Þ x2 = (8)2 + (16 – x)2
12. (a) If area of a circle decreased by x % then the radius of a Þ 32x = 320 Þ x = 10.
circle decreases by \ BC = (32 – 2x) = (32 – 20) cm = 12 cm.

(100 - 10 100 - x )% = (100 - 10 100 - 36)% æ1 ö


Hence, required area = ç ´ BC ´ AD ÷
è 2 ø

ww = (100 - 10 64)%
= 100 - 80 = 20%
æ1 ö
= ç ´12 ´10 ÷ cm 2 = 60 cm 2 .
è2 ø
13.

w.E
(a) Area of the outer rectangle = 19 × 16 = 304 m2

2m
17. (c) Area of field = 576 km2. Then,
each side of field = 576 = 24 km

asy Distance covered by the horse


= Perimeter of square field
= 24 × 4 = 96 km
2m
12
2m
En \ Time taken by horse =
distance 96
speed
=
12
=8h
15
2m gin
18. (c) Let the length and breadth of the original rectangular
field be x m and y m respectively.

Area of the inner rectangle = 15 × 12 = 180 m2 eer


Area of the original field = x × y = 144 m2

\x=
144
… (i)
14.
Required area = (304 – 180) = 124 m2
(a) Radius of a circular grass lawn (without path) = 35 m
\ Area = pr2 = p (35)2 ing y
If the length had been 6 m more, then area will be
(x + 6) y = 144 + 54
Radius of a circular grass lawn ( with path)
= 35 + 7 = 42 m
\ Area = pr2 = p(42)2
Þ (x + 6) y = 198 … (ii)
.ne
Putting the value of x from eq (i) in eq (ii), we get

\ Area of path = p(42)2 – p(35)2


= p(422 – 352)
= p( 42 + 35) (42 –35)
æ 144
ç
è y
ö
+ 6 ÷ y = 198
ø
Þ 144 + 6y = 198
t
22 2 Þ 6y = 54 Þ y = 9 m
= p × 77 × 7 = ´ 77 ´ 7 = 1694 m Putting the value of y in eq (i) we get x = 16 m
7
15. (b) Radius of the wheel of bus = 70 cm. Then, 19. (b) Perimeter = Distance covered in 8 min.
circumference of wheel = 2pr = 140 p = 440 cm æ 12000 ö
Distance covered by bus in 1 minute =ç ´ 8 ÷ m = 1600 m.
è 60 ø
66 Let length = 3x metres and breadth = 2x metres.
= ´ 1000 ´100 cms
60 Then, 2 (3x + 2x) = 1600 or x = 160.
Distance covered by one revolution of wheel \ Length = 480 m and Breadth = 320 m.
\ Area = (480 × 320) m2 = 153600 m2.
= circumference of wheel
= 440 cm æ 22 ö
20. (c) Length of wire = 2p´ R = ç 2 ´ ´ 56 ÷ cm = 352 cm.
6600000 è 7 ø
\ Revolutions per minute = = 250
60 ´ 440 352
16. (b) Let ABC be the isosceles triangle and AD be the altitude. Side of the square = cm = 88 cm.
4
Let AB = AC = x. Then, BC = (32 – 2x). Area of the square = (88 × 88) cm2 = 7744 cm2.

Downloaded From : www.EasyEngineering.net


y
Downloaded From : www.EasyEngineering.net

o
u
rs
m
a
h
SOLUTIONS 63

b
o
21. (a) Let the length of the room be l m

o
5

b
Then its, breadth = l /2 27. (a) 79.2 km/hr = 79.2 ´ = 22 m/s

.w
18

o
l 5000 2 min 40 sec = 2 ´ 60 + 40 = 120 + 40 = 160 sec.

rd
Therefore, l ´ = Circumference of circular field = speed × time
2 25

p
re
l 2 = 400 = 22 ´ 160 = 3520m
or

s
s
or l = 20 m circumference

.c
Radius of circular field (r) =

o
2p
Also, 2lh + 2 ´ l ´ h = 64800

m
2 240 3520 ´ 7
= = 560m
Þ 3 l h = 270 2 ´ 22
22
270 270 Area of circular field = pr 2 = ´ (560) 2
or h= = = 4.5 m 7
3 ´ 20 60
22. (a) In a cube, 22
= ´ 560 ´ 560
Area = 6 (side)2 7
or 150 = 6 (side)2 = 985600 m2

ww
\ side = 25 = 5 m
Length of diagonal = 3 ´ side = 5 3 m
28. (d)

29. (c)
Area of rectangle = l × b = 240
Either length or breadth should be clear, then answer can be
determined.
Area of the circle = 39424 sq cm

w.E
23. (b) Given, playground is rectangular.
Length = 36 m, Breadth = 21 m
Now, perimeter of playground = 2( 21 + 36) = 114
pr2 = 39424 Þ
22 2
7
39424 ´ 7
´ r = 39424

asy
Now, poles are fixed along the boundary at a distance
3m.
2
Þ r =
22
Þ r2 = 1792 × 7

Þ r = 12544 r = 112 cm 4a = r 4a = 112 a = 28

\ Required no. of poles =


114
3
= 38 .
En 30. (b)
\ Area of the square = a2 = (28)2 = 784 sq cm
From the figure, it is required to find the length CD.
We have CA = LB = 15m

24. (d) Area of square =


1
2
´ (diagonal)2
gin \ LD = BD – LB = 15m
D
1
2
1
= ´ ( 28) = ´ 28 ´ 28
2
2
eer C
15 m
36 m
15 m
L

25. (c) One side of square =


= 392 cm2
circumference 44
= = 11 cm
ing A
36 m
B

4 4
Circumference of rectangle = 4 × perimeter of square
31. (d)
.ne
\ CD = CL2 + DL2 = 362 + 152 = 1521 = 39 cm
Total area = Area of square + 4 (Area of a semi-circle)
æ 1 2ö é 2 ù

t
2
= 4 × 44 = 176 cm = 2 + 4 ç pr ÷ = (4 + 2p) m2 ê radius = = 1ú
è2 ø ë 2 û
width of rectangle
32. (b) In any quadrilateral,
circumference of rectangle Area of the quadrilateral
= – length
2 1
= × any diagonal × (sum of perpendiculars drawn on
2
176
= - 51 = 88 - 51 = 37cm. 1
2 diagonal from two vertices) = ´ D ´ (P1 + P2 )
2
\ Required difference = width – side = 37 – 11 = 26 cm.
1
= ´ 23 ´ (17 + 7) = 12 × 23 = 276 sq cm
circumference 220× 7 2
26. (c) Radius of circle (r) = = = 35 m. 33. (b) Length of the wire = circumference of the circle
2p 2× 22
2 ´ 22 ´ 42
22 22 = 2p × 42 = = 264 cm
area of circle = pr 2 = ´ (35)2 = ´ 35 ´ 35 7
7 7 Now, perimeter of the rectangle = 264 cm.
= 3850 m2 = area of rectangle Since, perimeter includes double the length and breadth, while
finding the sides we divide by double the sum of ratio.
\ Length of rectangle = area of rectangle 264
´ 6 = 72 cm
Therefore, length =
width 2(6 + 5)
3850 264
= = 77 m. and breadth = ´ 5 = 60 cm
50 2(6 + 5)

Downloaded From : www.EasyEngineering.net


y
Downloaded From : www.EasyEngineering.net

o
u
rs
m
a
h
64 101 SPEED TEST

b
o
o
34. (a) Let the final length of the side of the smaller square be a.
Surface area of bigger cube = 6 A2

b
Now, a + 10 + 7 = 19

.w
a = 19 – 17 = 2 or 384 = 6A2
\ A = 8 cm.

o
\ Area of the smaller square = (2)2 = 4

rd
\ Decrease in the area of the smaller square = 16 – 4 Surface area of smaller cube = 6 a 2

p
= 12 sq. units. 96 = 6a2

re
35. (c) Let the breadth be b. \ a = 4 mm = 0.4 cm

s
s
Then, length = b + 13 Perimeter = 50 = 2(l + b)

.c
Volume of bigger cube
2(b + 13 + b) = 50 b = 6 m l = 6 + 13 = 19 m So, Number of small cube =

o
\ Area = length × breadth = 19 (6) = 114 m2 Volume of smaller cube

m
SPEED TEST 47 (8)3 512
= 3
= = 8, 000
(0.4) 0.064
1. (a) Volume of the bucket = volume of the sand emptied
9. (c) Volume of the liquid in the cylindrical vessel
Volume of sand = p (21)2 × 36 = Volume of the conical vessel
Let r be the radius of the conical heap.
æ 1 22 ö 3
Then,
1 2
pr ´ 12 = p(21)2 ´ 36 = ç ´ ´12 ´12 ´ 50 ÷ cm
3 è3 7 ø

2.
ww
or r2 = (21)2 × 9 or r = 21 × 3 = 63
(a) Let the edge of the third cube be x cm.
Then, x3 + 63 + 83 = 123

æ 22 ´ 4 ´ 12 ´ 50 ö 3
è 7 ÷ cm .
ø
Let the height of the liquid in the vessel be h.

3.
w.E
Þ x3 + 216 + 512 = 1728
Þ x3 = 1000 Þ x = 10.
Thus the edge of third cube = 10 cm.
(b) Area of the inner curved surface of the well dug
Then,
22
7
´ 10 ´ 10 ´ h =
22 ´ 4 ´12 ´ 50
7

22
= [2p × 3.5 × 22.5] = 2 ´ ´ 3.5 ´ 22.5
7 asy æ 4 ´ 12 ´ 50 ö
or h = ç
è 10 ´10 ø
÷ = 24 cm.
10. (b) Volume of material in the sphere
= 44 × 0.5 × 22.5 = 495 sq. m.
\ Total cost = 495 × 3 = ` 1485. En é4
{ 3
}
3 ù 3 æ4 ö 3
= ê p´ (4) - (2) ú cm = ç p´ 56 ÷ cm .
4. (d) Let the length of the wire be h cm.
and radius of sphere and wire are R and r respectively.
Then, volume of sphere = volume of wire (cylinder) gin ë 3 û è 3
Let the height of the cone be h cm.
æ4 ö
ø

4 4
or p R 3 = p r 2h or R 3 = r 2h
3 3 eer
Then,
1
3
p´ 4 ´ 4 ´ h = ç p´ 56 ÷
è 3 ø

or
4 3
3
(3) = (0.1)2 h
Þ h =ç
ing
æ 4 ´ 56 ö
è 4´ 4 ø
÷ = 14 cm.

\ h=
4 ´ (3)3
2
=
108
= 3600 cm = 36 m
11.
æ4
è 3 ø
.ne
ö 3
(d) Volume of sphere = ç p´ 9 ´ 9 ´ 9 ÷ cm .

t
3 ´ (0.1) 0.03
æ1 ö 3
Volume of cone = ç p´ 9 ´ 9 ´ 9 ÷ cm .
5. (c) In a sphere, volume = 4 p r3 è 3 ø
3 Volume of wood wasted
and surface area = 4p r 2 éæ 4 ö æ1 öù 3
= êç p ´ 9 ´ 9 ´ 9 ÷ - ç p ´ 9 ´ 9 ´ 9 ÷ ú cm .
4 3 ëè 3 ø è 3 øû
According to question, p r ¸ 4p r 2 = 27 = (p × 9 × 9 × 9) cm3
3
or r = 27 × 3 = 81 cms æ ö
6. (a) Area of the wet surface = [2(lb + bh + lh) – lb] ç p´ 9 ´ 9 ´ 9 ÷
= 2(bh + lh) + lb \ Required percentage = ç ´ 100 ÷ %
çç 4 ÷÷
= [2(4 × 1.25 + 6 × 1.25) +6 × 4] m2 = 49 m2. ´ p´ 9 ´ 9 ´ 9
è 3 ø
7. (b) Let l be the length and b be the breadth of cold
storage. æ3 ö
L = 2B, H = 3 metres = ç ´100 ÷ % = 75%.
è4 ø
Area of four walls = 2[L × H + B × H] = 108 12. (b) Curved surface area of cylinder = 2prh
Þ 6BH = 108 Þ B = 6 \ Surface area of 50 cylindrical pillars = 50 × 2prh
\ L = 12, B = 6, H = 3 Now, Diameter of each cylindrical pillar = 50 cm
Volume = 12 × 6 × 3 = 216 m3
8. (c) Let 'A' be the side of bigger cube and 'a' be the side of 50
\ Radius = = 25 cm » .25 m
smaller cube 2

Downloaded From : www.EasyEngineering.net


y
Downloaded From : www.EasyEngineering.net

o
u
rs
m
a
h
SOLUTIONS 65

b
o
Also, height = 4m 1 cm3 weighs 10 g. Þ 2618 cm3 will weigh

o
b
\ Surface area = 50 × 2 × 3.14 ×. 25 × 4

.w
2618 ´10
= 314 × 1 sq m. = kg = 26.18 kg

o
= 314 sq. m. 1000

rd
Now, labour charges at the rate of 50 paise 22. (b) Volume of the steel used in the hemispherical bowl

p
per sq. m = 314 ×.5 = 157.0 º ` 157
[ ]

re
2 2 22
13. (c) Let the kerosene level of cylindrical jar be h. = p (4.5)3 – 43 = ´ ´ 27.125 = 56.83 cm3

s
3 3 7

s
.c
1 2 23. (d) Volume of the parallelopiped = 5 × 4 × 3 = 60 cm3
Now, Volume of conical vessel = pr h

o
Volume of the cube = 4 × 4 × 4 = 64 cm3

m
3
Since, radius (r) = 2 cm and height(h) = 3cm of conical Volume of the cylinder = p × 3 × 3 × 3 = 27p
vessel. = 84.8 cm3
1 4
\ Volume = p ´ 4 ´ 3 = 4p Volume of the sphere = p(3)3 = 36p = 113 cm3
3 3
Now, Volume of cylinderical jar = pr2h The required decreasing order is D, C, B and A.
= p (2)2h 24. (d) Let R be the radius of bigger sphere. Volume of one
= 4ph spheres + Volume of other sphere = Volume of bigger
Now, Volume of conical vessel = Volume of cylindrical sphere

ww
Þ 4 p = 4 ph
h = 1cm
Jar 4 3 4 3
Then, p (1) + p ( 2 ) = pR
3 3
4 3
3

14. (a)
w.E
Hence, kerosene level in Jar is 1 cm.
Let the rise in water level = x m
Now, volume of pool = 40 × 90 × x = 3600 x
When 150 men take a dip, then displacement of
4 3
3
Þ R3 = 9
4
Þ p(1 + 23 ) = pR 3
3

asy
1
water = 8m3
\ R = 93
3600 x 900 25. (b) Given, Diameter of building = 54 cm
\ =8 Þ x = 2 Þ x = .33m

15. (d)
150
Þ x = 33.33 cm
150

Let edge of the new cube = x cm. En \ Radius = 27 cm.


Also, given area of the base of the wall = 352 cm2
Let Dx be the thickness of the wall
Volume of the newly formed figure (cube)
= sum of volume of smaller cubes.
i.e. (x)3 = (3)3 + (4)3 + (5)3 = 27 + 64 + 125 = 216
gin \ Area of the Base = 2 p r× D x
Þ 352 = 2 × p × 27 ×D x
352 ´ 7 8 ´ 7 56

16. (b)
Þ x = 6 cm
Volume of the spherical ball = volume of the water
ÞDx=
eer2 ´ 22 ´ 27
=
27
=
27
»2
Hence, thickness of the wall is 2 cm.
displaced.
4
Þ pr 3 = p (12)2 × 6.75 ing SPEED TEST 48

Þ r3 =
3
144 ´ 6.75 ´ 3
4
= 729 Þ r = 9 cm, 1.
.ne
(c) In a right angled D, the length of the median is
1
2
the

17. (d)

18. (b)
Let x be the length of a side (edge of the cube)
Now, x3 = 12 x Þ x2 = 12
Total surface area = 6x2 = 6 × 12 = 72 square units
Let an edge of the cube be ‘a’cm.
2. (a)
length of the hypotenuse . Hence BD =

ÐD = 180 - ÐB = 180 - 70 = 110°


1
2 t
AC = 3cm.

Now, 6a2 = 726 Þ a2 = 121 Þ a = 11m \ ÐACD = 180 - ÐD - ÐCAD


Volume of the cube = (11)3 = 1331 m3. 180 - 110 - 30 = 40°
19. (c) 1 litre = 1 (dm)3 3. (b) The sum of the interior angles of a polygon of n sides
Length of the cistern = 3m = 30 dm ( Q 1 m = 10 dm) p
Breadth of the cistern = 2m = 20 dm is given by the expression (2n – 4)
Height of the cistern = 1 m = 10 dm 2
\ Volume of the cistern = 30 × 20 × 10 = 6000 (dm)3 p p
Þ ( 2n – 4) ´ = 1620 ´
20. (c) Here r = 8 cm, h = 15 cm Þ l = 8 + 15 = 17
2 2 2 180
Curved surface area of the cone 1620 ´ 2
( 2n – 4) = 180 = 18
= pr l = p ´ 8 ´ 17 = 136 p cm 2
or 2n = 22
21. (c) Volume of the cone = 1 pr 2 h or n = 11
3 4. (c) Tangent at any point of a circle is ^ to the radius
In DOPT, OP2=PT2+OT2
1 22 (13)2 = (12)2 + OT2 Þ 169 – 144 = OT2
= ´ ´ 49 ´ 51 = 2618cm 3
3 7 Þ 25 = OT2 Þ 5 = OT

Downloaded From : www.EasyEngineering.net


y
Downloaded From : www.EasyEngineering.net

o
u
rs
m
a
h
66 101 SPEED TEST

b
o
o
5. (c) Let the angles of the triangle be 5x, 3x and 2x.

b
Now, 5x + 3x + 2x = 180° AB = 32 - 12 = 2 2 cm

.w
or 10x = 180 or x = 18

o
\ AC = 4 2 cm

rd
or Angles are 36, 54 and 90°
Given D is right angled. 13. (d) Let the angles be x and 3x

p
re
6. (c) m Ð ABM = 180º –120º = 60º We know, sum of the supplementry angles = 180°

s
\ D AMB is a 30º – 60º – 90º triangle. \ x + 3x = 180°

s
.c
Þ 4x = 180° Þ x = 45°

o
3 3 14. (d)
\ AM

m
AB = ×8=4 3
2 2 15. (c) Supplementary angles are pairs of angles whose
measures upto 180 degrees.
1 1
MB = AB = x 8 = 4 Hence, let one angle be x. Since they are equal, \ the
2 2 other angle is also equal to x.
(AC)2 = (AM)2 + (MC)2 = (4 3 )2 + (4 +7)2 So, x + x = 180 Þ 2x = 180
\ x = 90
= 48 + 121 = 169 ; AC = 169 = 13. 16. (b)
7. (d) A P 1
17. (b) Four right angle + rd right angle

ww 10 cm 1
3

= 4 × 90 + × 90 = 360 + 30 = 390
3

w.E B
C Q
R
18. (b) Two right angles = 2 × 90° = 180°

Half right angles =


1
2
´ 90° = 45°
D ABC and D PQR are similar..
AB Perimeter of DABC
= Þ
AB 36
= asy Total two right angles and one half right angle
= 180°
PQ Perimeter of DPQR
36
PQ 24

En + 45°
225°

8.
or AB =
24
´ 10 = 15
(a) For the two similar triangles, we have gin
19. (a) Five and two-third of a right angle
2
h12
h 22
=
Area of 1st D
=
9
Area of IInd D 16 eer
= 5 × 90° + ´ 90° = 450° + 60° = 510°
3
20. (c) Let the angle be q.

9.
Þ h1 : h2 = 3 : 4
(c) Circumcentre of a triangle is the point of the intersection
q=ing
According to question
2
( 90 - q ) ...(1)
10.
of the perpendicular bisectors of its sides.
(a) We have, x + y + (y + 20) = 180
or x + 2y = 160 ...(i) q=
1
3

(180 - q ) .ne
and 4x – y = 10
From (i) and (ii), y = 70, x = 20
...(ii)

Angles of the triangles are 20°, 70°, 90°. Hence the


triangle is a right angled.
4
From eq. (1)
2
q = ( 90 - q )
3
...(2)

t
11. (a) Clearly (x + 1) will be the hypotenuse of the right
triangle [ x ³ 0] 3q = 180 – 2q
5q = 180 Þ q = 36º
Now, (x + 1)2 = x2 + (x – 1)2 21. (a) Let the angles are x, 2x, 3x and 4x
or x2 + 2x + 1 = x2 + x2 – 2x + 1 x + 2x + 3x + 4x = 360º
or x2 – 4x = 0 or x (x – 4) = 0 or x = 4
10x = 360º
The side of a triangle cannot be equal to zero.
x = 36º
There fore, x = 4
4th angle = 4 × x = 4 × 36 = 144º
\ Hypotenuse = (4 + 1) = 5
22. (b)
12. (d)
2 cm
O¢ O
O 5
1 cm 3 cm 3
C B 2 2cm A A B
M

Downloaded From : www.EasyEngineering.net


y
Downloaded From : www.EasyEngineering.net

o
u
rs
m
a
h
SOLUTIONS 67

b
o
In DOAM

o
11. (d) Pattern of the series would be as follows

b
52 = 32 + AM2 7 9 12 16 21

.w
25 – 9 = AM2

o
16 = AM2

rd
+2 +3 +4 +5
AM = 4

p
12. (c) Pattern of the series would be as follows

re
\ Length of chord AB
384 192 96 48 24

s
= 2 × AM = 2 × 4 = 8 cm

s
.c
23. (a) 2x + 3x = 180º ÷2 ÷2 ÷2 ÷2

o
5x = 180º 13. (e) Pattern of the series would be as follows

m
x = 36º 5× 1+1=6
Hence, angles are 72º and 108º respectively. As opposite 6 × 2 + 2 = 14
angle of parallelogram are equal. Therefore, measure of
14 × 3 + 3 = 45
all angles are 72º, 108º, 72º, 108º.
24. (d) Let the angles be 4x, 3x and 2 x. \ 45 × 4 + 4 = 184
4x + 3x + 2x = 180° x = 20° 14. (e) Pattern of the series would be as follows
\ angles are 80°, 60° and 40°. 8 9 13 22 38
25. (c) Second angle of parallelogram
= 180° – 45° = 135° +(1)2 +(2)2 +(3)2 +(4)2

ww
\ Required value
= 135 + 2 × 45
= 135 + 90 = 225°
15. (a) Pattern of the series would be as follows
6 11 21 41 81

w.E
121 117
SPEED TEST 49

108 92 67 31
16.
+5 +10

13
+20 +40
(b) The series pattern would be as follows :
7 25 49 97
1. (a)
–2
2
–3
2
–4
2
asy
–5
2
–6
2
+6 +12 +24 +48

2. (b)
50 26 14 8 5 3.5
En 17.
So, the above series is progressing of double
differences.
(d) The series pattern would be as follows :

3
÷2+1
23
÷2+1
43
÷2+1
63
÷2+1
83
÷2+1
103 gin 5 6 10 19 35

3. (c)
+20 +20 +20 +20 +20 18.
eer
(1)
2 2
(2) (3)
(a) The series pattern would be as follows :
8× 1+1=9
2
(4)
2

4. (e)
748

–11
737

–22
715

–33
682

–44
638

–55
583

ing
9 × 2 + 2 = 20
63 × 4 + 4 = 256
1 9 25 49 81 121 169
19. (e) The series pattern would be as follows :
11 13 16
.ne
20 25
5.

6.
(d)

(b)
12

36 20
E55555F E55555
32

12 8
52

6
F E555F E5555F E55555
F
5
72 92 112 132
20.
608
+2

304
+3
(c) The series pattern would be as follows :
152
+4

76
+5

38
t
¸ 2+ 2 ¸2 + 2 ¸ 2+ 2 ¸2 + 2 ¸2 + 2

7. (c) 668 656 632 584 448 296


E55555F E55555 F E55555F E55555F E55555 F ÷2 ÷2 ÷2 ÷2
-12 -24 -48 -96 -192
E55555F E55555 F E55555F E55555F 21. (d) The series is based on the following pattern:
´2 ´2 ´2 ´2 2 × 3 + 5 = 11
11 × 4 – 6 = 38
8. (e) 1 121 441 961 1681 2601 38 × 5 + 7 = 197
­ ­ ­ ­ ­ ­
12 112 212 312 412 512 197 × 6 – 8 = 1174 ; not 1172
1174 × 7 + 9 = 8227
9. (d) 9 49 201 1009 4041 20209 80841 8227 × 8 – 10 = 65806
E55F E555F E555F E555F E55555F E555F Clearly, 1172 is the wrong number and it should be
´5 + 4 ´4 +5 ´5 + 4 ´4 + 5 ´5+ 4 ´4 + 5
replaced by 1174.
10. (a) 31 35 44 60 85 121 22. (a) The series is based on the following pattern :
E55555F E55555
F E5555F E5555F E55555F
+ 22 +32 +42 +52 +62 16 + 12 = 17; not 19
17 + 22 = 21

Downloaded From : www.EasyEngineering.net


y
Downloaded From : www.EasyEngineering.net

o
u
rs
m
a
h
68 101 SPEED TEST

b
o
21 + 32 = 30

o
3. (e) The pattern of the number series is :

b
30 + 42 = 46

.w
45 + 1 × 12 = 45 + 12 = 57
46 + 52 = 71 57 + 2 × 12 = 57 + 24 = 81

o
71 + 62 = 107

rd
81 + 3 × 12 = 81 + 36 = 117
Clearly, 19 should replaced by 17.

p
117 + 4 × 12 = 117 + 48 = 165

re
23. (d) The series is based on the following pattern :
165 + 5 × 12 = 165 + 60 = 225

s
7 + 9 = 16

s
.c
9 + 16 = 25 4. (e) The pattern of the number series is :

o
16 + 25 = 41 17 + 13 = 17 + 1 = 18

m
25 + 41 = 66; 68 18 + 23 = 18 + 8 = 26
26 + 33 = 26 + 27 = 53
41 + 66 = 107 53 + 43 = 53 + 64 = 117
66 + 107 = 173
Clearly, 68 should be replaced by 66 117 + 53 = 117 + 125 = 242
24. (c) The series is based on the following pattern 5. (a) The pattern of the number series is :
4 × .5 = 2
1 1 1
2 × 1.5 = 3 ; not 3.5 + =
4 4 2

ww
3 × 2.5 = 7.5
7.5 × 3.5 = 26.25
26.25 × 4.5 = 118.125
Clearly, 3.5 should be replaced by 3.
1 1 3
+ =
2 4 4

w.E
25. (b) The series is based on the following pattern:
16 × 0.25 = 4
4 × 0.50 = 2
3 1
+ =1
4 4

1+
1
=1
1
2 × 0.75 = 1.5
1.5 × 1.00 = 1.5 ; not 1.75 asy 4
3 1
4

\? = 1 + = 2

26. (c)
1.5 × 1.25 = 1.875
Clearly, 1.75 should be replaced by 1.5.
The series is En6.
4 4
(c) The pattern of the number series is :
× 1 + 2, × 2 + 3, × 3 + 4, × 4 + 5, × 5 + 6
The wrong number is 18.
It should be 6 × 2 + 3 = 15 gin 4 + 1 × 15 = 19
19 + 2 × 15 = 49
49 + 3 × 15 = 94
27. (e) The series is × 1.5
The wrong number is 366 eer
94 + 4 × 15 = 154
154 + 5 × 15 = 229
28. (a)
It should be 243 × 1.5 = 364.5
The series is
× 6 + 42, × 5 + 30, × 4 + 20, × 3 + 12, × 2 + 6,
7.
ing
(a) The pattern of the number series is :
1 1

29. (b)
The wrong number is 3674
It should be 1220 × 3 + 12 = 3672
The series is (2)3, (3)3, (4)3, (5)3, (6)3, (7)3,
+ =1
2 2
1 1 .ne
30. (d)
The wrong number is 218
It should be (6)3 = 216
The series is + (7)2, + (6)2, + (5)2, + (4)2, + (3)2
The wrong number is 102.
1+

1 1
2

1 + =2
2 2
=1
2
t
It should be 68 + (6)2 = 104
1 1
\ ?=3+ = 3
SPEED TEST 50 2 2
8. (d) The pattern of the number series is :
1. (b) The pattern of the number series is : 101 + 2 = 103
353 + 1 = 354
103 – 4 = 99
354 – 3 = 351
99 + 6 = 105
351 + 5 = 356
105 – 8 = 97
356 – 7 = 349
97 + 10 = 107
346 + 9 = 358
2. (c) The pattern of the number series is : 9. (b) The pattern of the number series is :
219 – 3 = 216 = 63
1 + 22 = 1 + 4 = 5 344 – 219 = 125 = 53
5 + 23 = 5 + 8 = 13 408 – 344 = 64 = 43
13 + 24 = 13 + 16 = 29 \ ? = 408 + 33
29 + 25 = 29 + 32 = 61 = 408 + 27 435
61 + 26 = 61 + 64 = 125

Downloaded From : www.EasyEngineering.net


y
Downloaded From : www.EasyEngineering.net

o
u
rs
m
a
h
SOLUTIONS 69

b
o
o
22. (a)
10. (a) The pattern of the number series is :

b
45

.w
7 + 3 = 10
10 + 12 (= 2 × 3) = 16 4 12 42 196 1005 6066 42511

o
rd
16 + 12 (= 2 × 6) = 28

p
28 + 24 (= 2 × 12) = 52

re
42 is written in place of 45.

s
52 + 48 (= 2 × 24 ) = 100

s
23. (a)

.c
11. (a) 1 × 1 = 1

o
10
1×2=2

m
2 8 12 20 30 42 56
2×3=6
6 × 4 = 24 +4 +6 +8 + 10 + 12 + 14
24 × 5 = 120 8 is written in place of 6.
12. (b) 7 – 8 = 1 = 13 24. (e)
16 – 8 = 8 = 23 60
43 – 16 = 27 = 33 32 16 24 65 210 945 5197.5
? – 43 = 107 = 43 × 0.5 × 1.5 × 2.5 × 3.5 × 4.5 × 5.5

ww
232 – 107 = 125 = 53
13. (b) 4 × 3 + 1 = 13
4 × 4 + 1 = 17
25. (d)
65 is written in place of 60.

w.E
4 × 5 + 1 = 21
14. (c) 982 – 977 = 5
977 – 952 = 25
7
+6
13
+ 12
25

+ 24
49
+ 48
97

+ 96
193
194
+ 192
385

952 – 827 = 125


827 – 822 = 5
822 – ? = 25
asy 194 is written in place of 193
26. (a) The series is based on the following pattern :
16 + 12 = 17; not 19
? = 822 – 25
= 797 En 17 + 22 = 21
21 + 32 = 30
15. (c) 1 ×8=8
8 × 9 = 72 gin 30 + 42 = 46
46 + 52 = 71
71 + 62 = 107
72 × 8 = 576
576 × 9 = 5184
5184 × 8 = 41472 eer
Clearly, 19 should replaced by 17.
27. (d) The series is based on the following pattern :
16. (e) The series is as follows:
64 + 5 = 69;
7 + 9 = 16
9 + 16 = 25
16 + 25 = 41 ing
69 + 5 = 74;
74 + 5 = 79
25 + 41 = 66; 68
41 + 66 = 107 .ne
17. (b)
54 – 5 = 49;
49 – 5 = 44
The series is as follows:
÷2 +8 28. (e)
66 + 107 = 173
Clearly, 68 should be replaced by 66
32

×0.5
16

×1.5
24

×2.5
65
×3.5
210

×4.5
945 t 5197.5

×5.5
Hence, ? = 1012 ÷ 2 + 8 = 514 29. (a) The series is
18. (c) The series is as follows: – 200, –100, –50, –25, – 12.5, –6.25, ......
× 1, × 3, × 5, × 7, × 9, × 11 30. (c) The series is
×1.5, × 2, ×1.5, ×2, ×1.5, ×2, .....
Hence, ? = 75 × 7 = 525
19. (a) The series is as follows: SPEED TEST 51
1 1 1
× , × 1, × 1 , × 2, × 2 , × 3 1. (d) Total value of the quantity sold for item D
2 2 2
1 40 ´ 150 12.5 ´ 90
Hence, ? = 78 × 2 = 195 = ´ ´ 100 = 60 × 11.25 × 100 = ` 67500
2 100 100
20. (a) This is a series of prime numbers : 2. (e) Average price per kg. of items A, B and C
21. (d)
(17.5 + 10 + 7.5 ) 35
155 = = = ` 11.667 » 10.50 (approx)
3 3
3601 3602 1803 604 154 36 12 3. (a) Total value of quantity sold for item E = 15 × 25 × 100 = 37500
÷1+1 ÷2+2 ÷3+3 ÷4+4 ÷5+5 ÷6+6 Total value of quantity sold for item F
154 is written in place of 155. = 10 × 35 × 100 = 35000
Required ratio = 37500 : 35000 = 15 : 14

Downloaded From : www.EasyEngineering.net


y
Downloaded From : www.EasyEngineering.net

o
u
rs
m
a
h
70 101 SPEED TEST

b
o
o
4. (e) Total value of the quantity sold for item C
12 ´ 2000

b
= 45 × 100 × 7.50 = 33750

.w
Total value of the quantity sold for item E 100 12
15. (d) Required ratio = = = 4 :5
15 ´ 2000 15

o
= 22.5 × 100 × 15 = 33750

rd
33750 100
´ 100 = 100%

p
\ Required percentage = 16. (e) Required difference = 680 – 258 = 422

re
33750
5. (d) Required price

s
550 - 430

s
120 17.5 ´ 120 17. (b) Required percentage = ´100 » 27%

.c
= 20 ´ 100 ´ ´ = 2400 × 21 = ` 50400 430

o
100 100

m
6. (d) Total number of students studying in college H 160 + 708 + 550 + 586
18. (b) Required average = = 501
= 51.2 + 40 + 36.5 = 127.7 thousand 4
Total number of students studying in college K. 19. (a) Number of flights cancelled by airline R in 2010 due to
= 30 + 56 + 25 = 111 thousand
Required difference = (127.7 – 111) thousand 880 ´ 60
technical fault = = 528
= 16.7 thousand = 16700 100
7. (b) Total number of students studying in all the colleges together
( 600 + 546)
= [(51.2 + 40 + 36.5) + ( 65 + 50 + 33) 20. (e) Required percentage = ´ 100
+ (44 + 30 + 60) + 30 + 56 + 25)] thousand 365
(127.7 + 148 + 134 + 111) = 520.7 thousand = 520700 1146
8. (c)

ww
Number of students from the faculty of science from college
H and I together = 40 + 50 = 90
Number of students from the faculty of science from college
=
365
´ 100 = 314 (approx.)

w.E
J and K together = 30 + 56 = 86 SPEED TEST 52
90 45
Required ratio = = = 45 : 43 1. (a) 48 % of 525 + ? % of 350 = 399
86 43
9. (a) Number of students from the faculty of science from college 48 ?
I = 50

asy
Total number of students from college I = 65 + 50 + 33 = 148
50
Þ
Þ
100
´ 525 +
100
´ 350 = 399

25200 + ? × 350 = 399 × 100

10. (e)
Required percentage =
148
En
´ 100 = 33.78% » 34% (approx.)

Average number of students from the faculty of commerce


Þ ? × 350 = 39900 – 25200 = 14700
14700
from all the colleges together.

=
(36.5 + 33 + 60 + 25 )
thousand gin Þ ?=
350
3 4 5
= 42

eer
4 2. (b) ?= of of of 490
154.4 7 5 8
= thousand = 38.625 thousand = 38625
4 3 4 5
11. (a) Required percentage
æ 5 ö 14
2000 ´ ç 1 - ÷ ´ 2 14
2000 ´ ´
Þ
Þ ing
? = × × × 490
7 5 8
? = 35 × 3 = 105
è

.ne
7 ø 100 7 100 ´ 100
= ´ 100 =
2000 ´
7 140 3. (d) ? + 17 2 = 335
100
Þ ? + 289 = 335

12. (d)
=
80
140
´ 100 » 57%

Total number of teachers who teach English and History


( 7 + 27 ) 4. (a)
Þ
Þ
? = 335 - 289 = 46
? = 46 × 46 = 2116
? = 125% of 560 +22% of 450
t
together = 2000 ´ = 680
100 125 22
Þ ?= ´ 560 + ´ 450
Total number of teachers who teach Mathematics and Biology 100 100
(14 + 12 ) 70000 9900
together = 2000 ´ = 520 Þ ?= +
100 100 100
Required difference = 680 – 520 = 160 Þ ? = 700 + 99 = 799
13. (e) Total number of Biology and History teachers
12 100 + 40 ö æ 27 100 - 20 ö 28 ´ 5 - 15 ´ 6
æ 5. (c) ?=
= ç 2000 ´ ´ ÷ + ç 2000 ´ ´ ÷ 7 + 256 + (13) 2
2
è 100 100 ø è 100 100 ø
æ 12 140 ö æ 27 80 ö 140 - 90
= ç 2000 ´ ´ ÷ + ç 2000 ´ ´ ÷ Þ ?=
è 100 100 ø è 100 100 ø 49 + 16 + 169
= 336 + 432 = 768
14. (b) Required average 50 25
Þ ?= =
2000 ´ ( 25 + 27 + 12 )
234 117
1280 6. (b) ? = 18.76 + 222.24 + 3242.15
100 » 420
= = Þ ? = 3483.15
3 3

Downloaded From : www.EasyEngineering.net


y
Downloaded From : www.EasyEngineering.net

o
u
rs
m
a
h
SOLUTIONS 71

b
o
? = 784 ¸ 16 ¸ 7

o
7. (d) 15. (b) Suppose 16 men can complete the same work in x days

b
Then, Men days

.w
784
Þ ?= ¸7 10 8

o
16

rd
Þ ? = 49 ¸ 7 = 7 16 x

p
16 : 10 : : 8 : x

re
3 5 Þ 16 × x = 10 × 8

s
8. (c) ?= of 455 + of 456

s
.c
7 8 10 ´ 8
Þ

o
x= = 5 days

m
3 5 16
Þ ? = × 455 + × 456
7 8 16. (c)
Þ ? = 195 + 285
x
Þ ? = 480 17. (a) Let the original fraction be = .
9. (b) ? = 1.05% of 2500 + 2.5% of 440 y
1.05 2.5 x ´ 200 4 x 4 3 2
Þ ?= ´ 2500 + ´ 440 \ = Þ = ´ =
100 100 y ´ 300 21 y 21 2 7
2625 1100 18. (e) The word SIMPLE consists of 6 distinct letters

ww
Þ ?=

3725
?=
100
+
100

= 37.25
\ Number of arrangements = 6!
= 6 × 5 × 4 × 3 × 2 × 1 = 720

10. (b)
Þ
Þ

w.E 100
? = 4900 ¸ 28 × 444 ¸ 12
? = 175 × 37
19. (d) Let the present age of A = x and B = y years
According to first condition
x -7 3
= Þ 4 x - 28 = 3 y - 21Þ 4 x - 3 y = 7 ........ (i)
Þ ? = 6475
asy
11. (b) Q Cost price of (12 belts + 30 wallers) = ` 8940
Q Cost price of 3 × (4 belts + 10 wallets) = ` 8940
y -7 4
According to second condition

Q Cost price of 4 belts + 10 wallets =


8940
En
= ` 2980
x+9 7
y+9 8
= Þ 8x + 72 = 7 y + 63

12. (c) Q Cost price of an article = ` 1850


3

For 30% profit, selling price of this article gin Þ 7y – 8x = 9 ......... (ii)
8 x - 6 y =14

130
= 1850 ´
100
= ` 2405
eer
7 y - 8x =9
y = 23 years.

13. (e) Compound Interest after two years

æ 10 ö
2
20.

ing
(e) x + (x + 1) + (x + 2) = 1383
Þ 3x + 3 = 1383 Þ 3x = 1380
1380
= 8500 ç1 +
è 100 ÷ø
- 8500 Þ x=
3
= 460
Largest number = x + 2 = 462 .ne
11 11
= 8500 ´ ´ - 8500
10 10
= 10285 – 8500 = ` 1785
14. (a) Let length of the train be x m
21. (b) 8 × 6.5 = 52
52 × 5.5 = 286
286 × 4.5 = 1287.
t
22. (d) 3 × 14 = 42
5 50 42 × 12 = 504
Speed of the train be 60 km/h = 60 ´ = m/s
18 3 504 × 10 = 5040
x + 200 5040 × 8 = 40320.
Then, = 27
50 23. (c) 403 – 3 = 400
3 400 – 6 = 394
394 – 12 = 382
3( x + 200) 382 – 24 = 358
Þ = 27
50 358 – 48 = 310
Þ 3x + 600 = 1350 310 – 96 = 214 .
Þ 3x = 1350 – 600
Þ 3x = 750 +5 –9

750 24. (d) 7 8 4 13 –3 22 –14


Þ x= = 250 m
3 –3 –7 –11

Downloaded From : www.EasyEngineering.net


y
Downloaded From : www.EasyEngineering.net

o
u
rs
m
a
h
72 101 SPEED TEST

b
o
o
25. (a) 250000 ¸ 4 = 62500 148

b
62500 ¸ 5 = 12500 or, x = = 37

.w
12500 ¸ 4 = 3125 4

o
\ Numbers are A = 37, B = 39, C = 41, D = 43

rd
3125 ¸ 5 = 625
\ Product of B and D = 39 × 43 = 1677.

p
625 ¸ 4 = 156.25

re
33. (a) 2.31 km = 2.31 × 1000 = 2310 m

s
156.25 ¸ 5 = 31.25. Total number of days = 3 × 7 = 21

s
.c
7 2310

o
26. (b) Largest fraction = \ Distance covered by Anu each day = = 110

m
8 21
m.
1
Smallest fraction = 5
2 34. (b) 43.2 m/hr = 43.2 ´ = 12 m/s
18
7 1 7-4 3
Difference = - = = . Total distance covered = 12 × 80 = 960 m.
8 2 8 8 Perimeter of the square = 960 m.
27. (e) (?)0.6 × (?)1.4 = 26 × 104 Side of the square = 240 m.
or, ?2 = 2704 Area = (240)2 = 57600 sqm.

28.
ww
or, ? = ±52.
(a) Perimeter of the square = 84 cm
Perimeter of the rectangle = 28 cm
35. (b) Let the number of children be x.
Now, according to the question
æ 4800 ö

w.E
Perimeter of the rectangle = 2(1 + b)
or, 2(8 + b) = 28 cm
or, b = 14 – 8 = 6 cm.
\ Breadth of the rectangle = 6 cm
çè
x
- 100÷ (x + 4) = 4800

æ 48 ö
ø

Side of the square =


84
4
= 21 cmasy or, çè - 1÷ø (x + 4) = 48
x
or, (48 – x)(x + 4) = 48x

29.
Difference = 21 – 6 =15 cm.
(e) Perimeter of the rectangle = 42 m En or, x2 + 4x – 192 = 0
or, (x + 16)(x – 12) = 0
\ x = 12 sweets
2(l + b) = 42 m
or, l + 8.5 = 21m
or, l = 12.5 m. gin Number of students =
4800
12
= 400 .
Area of the rectangle = 12.5 × 8.5 = 106.25 sq.m.
\ Area of the circle. = 106.25 sq.m. eer
36. (c) Sneha's monthly income =
342000
= 28500
30. (d) Let the positive number be x.

Then,
5x 3x
´ = 504.6 ing
\ Akruti's monthly income =
28500
12

´ 116 = 34800

\x ´
100 100
5
´x´
3
= 504.6
95
Akruti's annual income = 417600.
.ne
t
100 100 256
37. (b) Time taken by the truck = = 8 hr.
32
2 504.6 ´ 100 ´ 100 Distance covered by the car = (256 + 160) = 416 km.
or, x =
15 Time = 8 hr.
\ x = 580.
416
31. (b) Two women alone can complete a piece of work in 16 \ Speed of the car = = 52 km / hr.
days. 8
\ Four women can complete the same work in 8 days. 663 - 612
Since 12 children can complete the work in 38. (a) Required percentage = ´ 100 = 5% .
1020
4 ´8 4 ´8 39. (c) Sum of the heights of all the girls = 148 × 21 = 3108 cm
= = 8 days.
8-4 4 Sum of the heights of the teacher and all the girls = 149
× 22 = 3278 cm.
12 ´ 8 Teacher's height = 3278 – 3108 = 170 cm.
\ Four children can complete the work in = 24
4 40. (a) Radius = 17.5 cm.
days.
22
32. (d) Let the four odd consecutive numbers be x, x + 2, x + 4 Area of the circle = ´ 17.5 ´ 17.5 = 962.5 sq cm.
and x + 6. 7
Also, A = x, B = x + 2, C = x + 4 and D = x + 6.
\ 4x + 12 = 4 × 40
or, 4x = 160 – 12 = 148

Downloaded From : www.EasyEngineering.net


y
Downloaded From : www.EasyEngineering.net

o
u
rs
m
a
h
SOLUTIONS 73

b
o
o
SPEED TEST 53 9. (e) 10. (c) 11. (b) 12. (b)

b
.w
13. (a) 14. (d) 15. (d)
1. (a)

o
16. (c) The author wants us to stop debating and implement

rd
2. (c) They are responsible for national disintegration. policies.

p
3. (d) They are harmful to national integrity. 17. (c) Stated in the first paragraph.

re
4. (b) India was forged into a nation on account of a common 18. (d) Uncertainty about payment is mentioned, hence option

s
s
culture evolved over the centuries. (d).

.c
5. (a) The author wants India to remain as an ideal nation

o
19. (b) Refer to the second paragraph. “State governments

m
and the passage has certainly a message behind it. have not implemented agr eed plans to ensure
6. (e) 7. (b) 8. (a) repayment when due...”
9. (e) India’s insurgence stood for gaining freedom by 20. (a) All the factors are mentioned in the passage.
adopting the path of non-violent struggle. 21. (a) Refer to the third paragraph. “The Delhi model has
10. (c) 11. (d) 12. (e) worked. But it receives no public support.”
13. (a) 14. (e) 22. (d) Clearly, populist measures would go against financial
15. (b) wellbeing.
16. (a) Option (c) can be rejected as it is out of the context. 23. (d) It is stated in the passage that the enforcement of the
Option (d) is also not true as they like to invest abroad reforms was inadequate.

17.
ww
than in India. Option (e) is true in parts. Only option
which is in sync with major portion of passage is (a).
(a) Other options are not mentioned in the passage.
24. (c) Eminent British economists and political scientists have
strongly attacked the tradition of budget secrecy.
25. (e) It leads to the control of public expenditure in order to
18.
w.E
(c) Last part of the passage chiefly describes it. Other
options show these sectors in positive light which is
against passage content.
set realistic taxation implications.
26. (b) He has presented the example of both the open budget
system and the secret budget system, practised by
19.

asy
(b) Option (B) is true in its completeness, encompassing
the whole of the passage. But other options are true in
parts only. Option (E) is false in parts.
27. (d)
various countries and has looked into all their aspects.
28. (e)
29. (a) Sir Richard Clarke was the originating genius of nearly
20.
Statement 2 is just not true.
En
(c) Both (1) and (3) are directly mentioned in passage. every important development in the British budgeting
techniques during the last two decades.
21.
22.
(c) Meaning of Clout is force.
(e) Marked means distinguished and different.
Imperceptible is that cannot be distinguished or gin
30. (b)

SPEED TEST 55
23.
percepted.
(d) Option (D) can be related to the experts as mentioned
in the passage.
1. (b)
eer 2. (e) 3. (c) 4. (c)
24. (d) Option (D) has plenty of illustrations, references and
structure to support it in the passage. Other options
5.
9.
13.
(b)
(d)
(c) ing
6.
10.
14.
(a)
(d)
(a)
7.
11.
15.
(b)
(e)
(b)
8.
12.
16.
(d)
(c)
(c)

25.
like E and B are true but not the central theme of the
passage. Option (A) and (C) are not true.
(d) PROMOTES means to advocate a particular cause.
17.
21.
25.
(d)
(d)
(a)
18.
22.
26.
(b)
(e)
(b)
19.
23.
27. .ne
(e)
(e)
(d)
20.
24.
28.
(a)
(b)
(b)
26.

27.
(e) A rise from the 4% level to 7% says that there is rise in
education in Egypt.
(a) All of the options are mentioned in one or other part of
the passage.
29. (b) 30. (a)

SPEED TEST 56
t
28. (c) First line of the passage is self explanatory. 1. (b) The word ' vicarious' means 'Endured or done by one
29. (b) Only option which can be linked with the passage is 1.
person substituting for another' which is nearest in
Their people will agitate for greater political freedom,
meaning to phrase 'Not experienced personally' which
culminating in a shift to a more democratic form of
is option (b), therefore, (b) is the correct answer. The
government.
other 3 words have following meanings: Ambitious -
30. (c) Option A is wrong as this is not intended from the use
strongly desirous; Nostalgic - homesickness; Vindictive
of this phrase. Last part of passage has this phrase
- vengeful.
and poor-rich divide has been discussed their in voting
2. (b) The word 'craven' means 'cowardly, contemptibly timid
pattern or why rich people even educated do not vote.
pusillaninuous which is nearest in meaning to cowardly'
Option (D) is out of the context. Option E can also be
which is option (b), therefore, (b) is the correct answers.
rejected as corruption is not the issue here.
3. (d) The word ' Tepid' means moderately warm; Luke warm
which is nearest in meaning to 'Lukewarm' which is option
SPEED TEST 54
(d), therefore, (d) is the correct answer. The other 3 words
have following meanings: Irreversible - uncapable of
1. (b) 2. (b) 3. (d) 4. (d) being changed; Causing fatigue - something that leads
5. (c) 6. (c) 7. (e) 8. (d) to mental exertion; Fast moving - speedy.

Downloaded From : www.EasyEngineering.net


y
Downloaded From : www.EasyEngineering.net

o
u
rs
m
a
h
74 101 SPEED TEST

b
o
o
4. (c) The word 'Tenuous' means thin or slender in form which 20. (b) Thw word ‘exaggerate’ means amplify, inflate, elaborate

b
is nearest in meaning to 'slender' which is option (c), etc. which is similar in meaning to ‘overstate’.

.w
therefore, (c) is the correct answer. The other three words 21. (d) The word ‘evident’ means obvious, clear, tangible,

o
rd
have followings meanings: Contentious - quarrelsome; distinct etc. which is nearest in meaning to ‘quite clear’.
Dark - little as no light; Malfunctioning - to fail to

p
22. (b) The word ‘penalize’ means inflict a penalty on, which

re
function.

s
5. (a) The word 'probity' means integrity or uprignt ness, is quite similar in meaning to ‘punish’.

s
.c
honesty' which is nearest in meaning to integrity which 23. (c) The word ‘remedial’ means aimed at solving a problem

o
is option (a), therefore, (a) is the correct answer. The

m
which is similar in meaning to ‘corrective’.
other 3 words have following meanings: Impudence - 24. (a) The word ‘trivial’ means not worth considering,
in solence; Profane - irreligious; Preface - an
introductory past. insignificant etc., which is similar in meaning to
6. (b) The word 'Musty' means absolute, out dated or stale ‘unimportant’.
food' which is nearest in meaning to 'stale' which is 25. (a) The word ‘ Incredible’ means beyond belief,
option (b), therefore, (b) is the correct answer. The other unbelievable, unimaginable etc. which is similar in
3 words have following meanings: Certainty - something
meaning to ‘hard to believe’.
certain; Modern - not ancient or remote; Mysterious -

7.
ww
implying or suggesting a mystery.
(b) The word 'Alleviate' means 'to diminish or lessen ' which
is nearest in meaning to 'lessen' which is option (b),
26. (d) The word ‘frenetic’ means ‘wildly excited or active’,
energetic, hectic, fast and furious etc.
27. (c) The word ‘elicit’ means draw out, extract, obtain,

w.E
therefore, (b) is the correct answer. The other three
words have following meanings: To release - to free
from confinement; To deprive - to remove from the
possession; To deceive - delude, to be unfaithful to.
provoke etc.
28. (b) The word ‘Lucrative’ means gainful, remunerative, well-
paid which is most similar in meaning to ‘profitable’.
8.
asy
(c) Morose means ‘depressed and pessimistic’. Some more
synonyms are : Cantankerous, gloomy, in a bad mood,
29. (c) Vivid means presented in very clear/distinct way. From
the options detailed is the other option which can
9.
mournful, moody, splenetic.

En
(c) Protagonist ‘means person who takes the lead or the
central figure of the narrative’. Some more synonyms
replace but only if 'clear' as option is not available.
Also 'detailed' can be confusing as well. So it is not the
are: Central character, Prime mover, Exponent, Hero,
Exemplar, Mainstay. gin best choice. Ambiguous is opposite for the Vivid.
'Categorical' is without any condition.
10. (c) Factitous means ‘unnatural, false or artificial’ so in the
given option artificial would be the right synonym of
factitious. Other synonyms : false, artificial, sham. eer
30. (d) Deplore means to disapprove /to condemn /to regret.
Deprive means lack of something. Implored is to beg
11. (d) The word ‘hospitable’ means genial, welcoming,
congenial, friendly, convivial, but welcoming is the most
suitable word. ing
impatiently. Deny is to refuse. So clear choice is
'regretted' as this is direct meaning of the word.
31. (c) Vindictive is revengeful. Other options like violent and
12. (a) The word ‘scarcely’ means hardly, barely, almost not,
only just. .ne
cruel can be actions of revenge but not the right answer
as synonym. Irritable is not correct clearly.
13.

14.
(c) The word ‘disdain’ means contempt, scorn, disrespect,
dislike. From the given options ‘hate’ is the most suitable
word.
(a) The word ‘absurd’ means stupid not logical and
sensible which is nearest in meaning to senseless.
t
32. (a) Pragmatic is practical. Pragmatic is replacing 'practical'
in English usage for common use.
33. (d) Soporific means sleep inducing. It can not be replaced
with other options. There is not much use of context in
these types of words which have direct meaning with
15. (a) The word ‘philanthropy’ means the practice of helping so clear options.
the poor and those in need especially by giving money. 34. (b) Evoke means to 'call for' or 'to rise by a reason'. From
Hence generosity (willing to give somebody money, the options, (b) is the direct meaning of the word.
gifts etc) is nearest in meaning to it. 35. (a) Abate is to reduce but not to completely remove. So
16. (a) The word ‘mutual’ is used to describe feelings that two Vanished as answer is rejected. Increased is just the
or more people have for each other equally. Hence, opposite in sense. Stabilized is to remain at same status
reciprocal is similar word in meaning to it. or as it is.
17. (d) The word ‘weird’ means very strange or unusual and 36. (c) Insulting behavior is insolent behavior. Violent behavior
difficult to explain which is similar in meaning to can not be counted as disrespectful it is extreme
unnatural. one.Yes; disrespect can be a reason for violent behavior.
18. (d) The word ‘pessimistic’ means bleak, distrustful, 37. (d) Mendacious is intentionally untrue or false. Options
hopeless, depressed etc. which is nearest in meaning are not confusing either.
to ‘not hopeful’. 38. (c) Induces means persuades, insist and provoke.
19. (c) The word ‘analogous’ means like, equivalent, related 39. (d) Authentic means genuine, real, valid and dependable.
etc. which is nearest in meaning to ‘similar’. 40. (c) Scarcity means insufficiency, shortage, lack, dearth or
paucity.

Downloaded From : www.EasyEngineering.net


y
Downloaded From : www.EasyEngineering.net

o
u
rs
m
a
h
SOLUTIONS 75

b
o
o
SPEED TEST 57 28. (a) Enrich is making richer /more valuable/more

b
meaningful/more nourished. The opposite of this word

.w
1. (a) 2. (e) 3. (d) 4. (e) is to make poor - impoverished.

o
rd
5. (c) 29. (a) Credible is loyal or believable or something in which
faith can be reposed. Incredible is just opposite of the

p
6. (b) The word ‘obligatory’ means compulsory, mandatory,

re
required whose opposite is optional or voluntary. word. Although from the options Believable is close

s
but not the best one to choose. If credible is not in the

s
7. (d) The word ‘obscure’ means unclear whose opposite is

.c
clear. options then this is the best among the rest.

o
30. (a) Inert is without motion. From the options active is clear

m
8. (a) The word ‘repulsive’ means repellent, nasty whose
opposite is attractive. choice. As Lazy is similar in meaning.While Resolute
9. (a) The word ‘vital’ means essential, urgent while trivial (Firm) is irrelevant in context. Strong does not mean
means insignificant. activeness or motion. He has very strong emotions for
10. (d) The word ‘inhibit’ means hinder, hold back whose her.
opposite word is encourage or promote. 31. (a) Affluent is extra rich. If richness is a scale then Affluent
11. (c) The word ‘exploit’ means treat unfairly, utilize, misuse is on one end and poor is on the other. Ordinary lies in
or take advantage of which is opposite in meaning to between the two. Backwardness is just the result of
support. poorness. So 'poor' is the best option.
12.

13.
ww
(b) The word ‘sharp’ means razor-edged, fierce, shrill etc.
which is opposite in meaning to blunt.
(d) The word ‘condemn’ means censure, criticize, blame
32.

33.
(d) Energetic is full of activity and lethargic means
inactivity/slowness/ laziness/dull. Gloomy is sadness.
(b) Here secure means to make safe and precarious is having

14.
w.E
etc. which is opposite in meaning to praise.
(d) The word ‘reluctant’ means timid, resistant, opposed
etc. which is opposite in meaning to eager. 34.
a sense of insecurity. So it can be a proper antonym for
the word from the available options.
(d) 'Various' word derives from the variety. Variety is range
of choices and similarity is opposite of it.
15.

asy
(a) The word ‘scarcity’ means shortage, lack, paucity,
meagreness, dearth etc. Its opposite word is ‘plenty’ in
the given options.
35. (b) Recession means depression, slump, downturn,
collapse, decline so its antonym can be inflation, boom,
16.
En
(a) The word ‘bleak’ means bare, exposed, denuded, dim
etc which is quite opposite in meaning to ‘bright’. 36.
rise increase.
(d) Denied means deprive, starve, unused, shorn of so its
antonym can be affirmed, avowed, confirmed, declared,
17.

18.
(d) The word ‘stern’ means serious, austere, unforgiving,
harsh etc. Its opposite word will be ‘forgiving’.
(a) The word ‘superficial’ means shallow, casual, hasty, gin
37.
stated.
(c) Collective means together, mutually, group, joint or

19.
trivial, silly, inane etc. which is opposite in meaning to
‘profound’.
(a) The word ‘elegance’ means grace, stylishness, charm, 38.
alone.
eer
cooperative so its antonym can be individual, single or

(b) Abducted means kidnap, snatch or seize so its antonym

20.
cleverness which is opposite in meaning to ‘balance’.
(d) The word ‘coarse’ means ugly, rough, ill-mannered, 39.
ing
can be set free, release or freedom.
(c) Enhanced means increase, augment, boost or amplify
so its antonym can be decreased, reduce or lessen.

21.
rude, crude etc. which is opposite in meaning to ‘soft’
from the given options.
(c) The word ‘extravagant’ means prodigal, lavish, costly
40.
.ne
(b) Condemn. Adore means esteem, respect and admire so
its antonym can be disapprove, criticize, revile, attack

22.
etc. which is opposite in meaning to the word
‘economical’.
(b) The word ‘diligent’ means industrious, hard-working,
rigorous etc. which is opposite in meaning to the word
1.
or condemn.

(c) shown
SPEED TEST 58 t
‘lazy’.
23. (c) The word ‘fictitious’ means fake, spurious etc. Its 2. (d) drawn to
opposite word will be genuine or real. Certain Verbs, Nouns, Adjectives, and Participles are
24. (b) The word ‘generous’ means liberal, charitable, always followed by certain Prepositions.
benevolent etc. Hence, uncharitable is the opposite 3. (d) were
word. The Past Subjunctive ‘were ‘is used after tile verb wish,
25. (b) Ignored is to overlook /to avoid /not considered/set to indicate a situation, which is contrary to fact or unreal;
aside. If something is ignored then it can be removed. as in, I wish I were a millionaire.
So 'remove' gives the same sense as 'ignored'. 4. (c) had seen me
26. (c) Lethargic is slow/lazy/inactive/dull. Active is antonym. 5. (c) concur with
Aggressive is second best option. Hungry is to make a 6. (b) swum
guessing student go on sense of the sentence and The Past Participle “swum” is to be used and not the
deviate from the right answer. past tense “swam”.
27. (d) Hazardous is harmful/perilous/marked by danger/risky; The Past Participle represents a completed action or state
opposite for it is 'safe'. Other options are not confusing of the thing spoken of.
so it is easy to answer. 7. (b) The sentence gives a condition, so the principle clause
will use ‘would’ not ‘will’.

Downloaded From : www.EasyEngineering.net


y
Downloaded From : www.EasyEngineering.net

o
u
rs
m
a
h
76 101 SPEED TEST

b
o
o
8. (b) Since there is a comparison between two, a comparative 28. (b) would not have been misunderstood.

b
degree verb must be used. 29. (d) reformed.

.w
9. (a) Gravity is the singular subject which will have the present 30. (c) When he ran across the road

o
The past tense is needed here as the sentence indicates.

rd
indefinite verb i.e., verb + s/es form.
10. (b) Since a comparison is being made there must be a subject

p
re
in both parts of sentence devided by than. There must SPEED TEST 59

s
be ‘that’ in the second part therefore, (d) is incorrect

s
.c
because of incorrect usage of article ‘the’. 1. (c) 2. (a) 3. (e) 4. (e) 5. (c)

o
11. (b) There is no need for ‘who’ or ‘which’ (both of which are 6. (b) 7. (c) 8. (e) 9. (d) 10. (e)

m
incorrect ‘whom’ will be the right pronoun) the clause 11. (c) 12. (b) 13. (b) 14. (d) 15. (b)
follows the subject ‘person’ directly and need not be 16. (c) 17. (b) 18. (c) 19. (e) 20. (c)
connected through a pronoun. 21. (b) 22. (b) 23. (e) 24. (c) 25. (c)
12. (d) It is a passive voice sentence and the phrase before 26. (c) 27. (d) 28. (e) 29. (c) 30. (e)
‘that’ should be a complete passive form of verb and not
31. (d) 32. (c) 33. (e) 34. (b) 35. (d)
a gerund.
13. (d) ‘of’ is the preposition used with frightened.
14. (c) Capitalist society is the singular subject and the statement SPEED TEST 60

15.
ww
made is a dictum, so pesent infinitive tense will be used.
(a) The use of the word here is as a ‘noun’ not as a verb thus
damage will be the right answer.
1.
5.
9.
(d)
(a)
(a)
2.
6.
10.
(c) 3.
(c) 7.
(a) 11.
(d)
(c)
(b)
4.
8.
12.
(a)
(e)
(e)

16. w.E
Tip : A sentence of form this can cause, will always be
followed by a noun.
(d) The right expression is ‘conditions necessary for’, since
the gap is followed by, this. ‘complete’ should be used in
13.
17.
21.
(d)
(a)
(b)
14.
18.
(d) 15.
(c) 19.
(e)
(b)
16.
20.
(e)
(e)
Engaged is the word which fits in both the sentences.

of this’. asy
the noun form and not adjective so it will be ‘completion
In the first sentence it means that couple has made on
agreement or a pledge to get married and in statement
II it means occupy the attention or efforts of (a person
17.
18.
(b) The right use is as + adjective + as

En
(a) There cannot be a subject in the first part of the sentence
as the same subject is given on the second part so (c)
22. (b)
or persons).
Option (b) is correct. Application in the first statement
and (d) are eliminated. To study is the infinitive verb
which will not be used here, because it suggests the gin means a written request for employment. In the II
statement application means the act of putting to a
special use or means the act of putting to a special use
action which is the affect and not the cause. e.g. ‘To gain
something you have to lose something else’.
By studying is the right answer because this gives the
23. (e)
eer
or purpose.
hearing
Statement I - an instance or a session in which testimony
19.
cause for the verb in the latter part of the sentence.
(b) In the given sentence there is no subject or verb, so the
added phrase should be of the form subject + infinitive ing
and arguments are presented, especially before an
official, as a judge in a lawsuit.
Statement II - the act of perceiving sound.

20.
verb.
(c) Modals such as must or should cannot be used with
24. (a) resort
.ne
Statement I - to have recourse for use help, or

21.
‘hope’.
(a) There must be the preposition ‘to’ to connect the verb
‘tend’ with the noun ‘result’, (b) is not correct because
first form of verb should be used and not continuous
since its a simple statement made suggesting no action
option or resource.
t
accomplishing something of ten as a final available

Statement II - a place to which people frequently or


generally go for relaxation or pleasure.
25. (d) Statement I - salary
taking place at the current moment. Statement II - to carry on (a bottle, war, conflict,
22. (b) The subject of the sentence is candidate, but in the first argument, etc).
part there is no subject, it should be passive or have an
active subject. (b) is the only option with passive verb. SPEED TEST 61
23. (d) ‘because’ cannot be used as the conjunction in this
sentence because for the sentence to be complete it 1. (b) up
should be followed by an ‘of’, i.e., because of their, since 2. (d) up with
this is not the option ‘of their’ is the most appropriate 3. (b) ‘with
use. 4. (b) of
24. (c) had better see 5. (c) on
25. (c) had known 6. (c) in———with
26. (c) than one’s speed. 7. (c) on for
Here ‘than’, used as a preposition, as; 8. (b) within——to
I need more than fifty rupees for this magazine. 9. (c) for
27. (b) which 10. (a) from
11. (c) for

Downloaded From : www.EasyEngineering.net


y
Downloaded From : www.EasyEngineering.net

o
u
rs
m
a
h
SOLUTIONS 77

b
o
o
12. (b) in ——— around 18. (a) Replace leisure by the adverbial word leisurely

b
13. (a) to 19. (b) The phrase ‘gift of the gab’ means ‘to have a talent for

.w
14. (c) off speaking’.

o
rd
15. (d) Though —— in 20. (c) The phrase ‘the order of the day’ means common,
16. (d) to

p
popular or suitable at a particular time. For example :

re
17. (c) Although — from Pessimism seems to be the order of the day.

s
18. (d) among

s
21. (d) started carrying out their plan.

.c
19. (b) in 22. (a) started the discussion.

o
20. (d) in

m
23. (c) A succession of unexpected events.
21. (b) towards 24. (c) be implemented next month
22. (c) to 25. (b) Idiom put your foot down means : to be very strict in
23. (c) concur with
opposing what somebody wishes to do.
24. (d) of
26. (d) In most probability
25. (b) on
26. (c) with 27. (a) ignorant about
27. (d) to 28. (a) in communication with
28. (c) upon 29. (d) discovered by chance
29.
30.
(d)
(a)
ww
of
at——in——at
In is used with names of countries and large towns, at is
30. (b) made a record.

SPEED TEST 63

31. (b) w.E


more often used when speaking of small towns and
villages.
In the perfect continuous tense only ‘for’ and ‘since’
are used. ‘since’ is used to suggest a particular time
1.
2.
3.
(b) ‘Sent’ is the third form of verb ‘send’ in passive voice.
(e)
(a) Replace ‘though’ by ‘As’ to express cause and effect

asy
in past and ‘for’ is used to suggest a time period, ‘a
long time’ suggest a time period so ‘for’ will be the 4.
in the sentence.
(d) Use ‘and I answered’ properly to express cause and

32. (b)
33. (b)
answer.
‘With’ is used with overwhelmed
Preposition ‘about’ is used before the word En 5.
6.
effect in the sentence.
(c) Apply the adverb ‘highly’ before the adjective ‘paid’.
(a) Replace adverb ‘necessarily’ by adjective ‘necessary’.

34. (b)
‘prohibition’.
This will be the right preposition. gin
7.
8.
(e)
(c) Use conjunction ‘and/therefore’ in place of ‘yet’.
35. (b) The right usage is ‘comes to mind’.

SPEED TEST 62
9.

10. eer
(b) ‘Any’ shows uncertainty of selection. So, use ‘one’
instead of ‘any’.
(d) Use third form of verb ‘signed’ in passive voice.
1.
2.
(a)
(b)
11.
used. ing
(a) Here, Nuclear waste will still remain/be .... should be

3.
4.
(a)
(b)
12.

.ne
(c) Here, whose attic had been should be used. Whose is
used to say which person or thing you mean.
Look at the sentence :
5.
6.
7.
8.
(d)
(d)
(c)
(a)
13.
It is the house whose door is painted red.
t
(c) Here, subject i.e. A public safety advertising campaign
is singular. Hence, hopes to draw attention ... should
be used here.
9. (e)
10. (c) 14. (b) Look at the structure of the sentence in Passive Voice
11. (e) Here, too is used as emphatic word. Lata was so scared of Past Simple.
that she could not go home alone. Hence, no correction Subject + was/were + V3 (Past Participle)
is required. Hence, awarded to the most .... should be used.
12. (b) The structure of sentence is subject + was/were + third 15. (b) Here, a time of reawakening .... should be used.
form of verb + object. Thus, Riya was dressed to kill. 16. (c) Here, and guidance to the mind tortured by doubt
13. (c) The given sentence is the statement of simple past should be used.
tense. Hence it should be ‘worried’ instead of worries. Look at the sentence:
14. (a) The phrase ‘let off’ means to give them only a light All activities take place under the guidance of an
punishment. experienced tutor.
15. (d) It should be ‘took’ instead of ‘take’. 17. (d) Here, Past Simple i.e. the educated class did not support
16. (d) Replace to by ‘too’ to make a correct phrase ‘a bit too him .... should be used as the sentence shows past
fast’ which means slightly or to a small extent. time.
17. (e) The word think about refers to consider. Hence no 18. (a) Here, Depletion (Noun) of the Ozone layer .... should
correction is required. be used.

Downloaded From : www.EasyEngineering.net


y
Downloaded From : www.EasyEngineering.net

o
u
rs
m
a
h
78 101 SPEED TEST

b
o
o
19. (a) Here, Most of the people who should be used. Who is 19. (d) : Replace 'for' by 'of

b
used to show which person or people you mean. 20. (e) : no error

.w
Look at the sentence: 21. (c) : Replace 'enough' by 'any'

o
rd
The people who called yesterday want to buy the 22. (c) : Remove 'not'

p
house. 23. (a) : Add 'having' after 'after'

re
20. (b) Here, in reducing human suffering .... should be used. 24. (c) : Replace 'was able to ' by 'could'

s
s
21. (d) Since the sentence begins in past tense. It should end 25. (d) : Replace 'for' by 'to'

.c
in past tense also since it the subject is singular the 26. (a) : Replace 'had' by 'would have'

o
m
verb will also be singular. Hence “them is missing” 27. (c) : Replace 'pumping' by 'pump'
should be “them was missing”. 28. (a) : Replace 'ential' by 'entails'
22. (a) The teacher that is the subject is singular so “were” 29. (b) : Replace 'sang' by 'sung'
will be replaced with “was”. 30. (b) : Replace 'at' by 'with'
23. (b) “Son for help her” should be “son to help her”. 31. (c) : Replace 'would have' by 'had'
24. (b) The verb “ask” will be in the past tense it will become 32. (c) : Replace 'those' by 'which'
“asked”. 33. (a) : Replace 'for' by 'two'
25. (c) “Saving” will be replaced with past tense of the verb 34. (d) : Add 'they' before 'assume'
“Save” that is saved because it is preceded by had.
26.
27.
28.
ww
(a) Change ‘decline of’ to ‘decline in’.
(b) Delete ‘for’ after order.
(c) Place on before ‘land’.
35.
36.
37.
(b) : Add 'which are' before 'available'
(a) : Replace 'being intelligent' by 'intelligence'
(a) : The correct form is 'The judges not onlyacquitted
29.
30.
31.
w.E
(c) Delete ‘to’ before Chennai.
(d) Replace ‘for’ by ‘to’.
(c) Change ‘look for’ by ‘look after’.
38.
39.
40.
(e) : No error
(b) : The correct form is ' different people diferentjobs .'
(d) : Replace 'promising' by 'promissed'. 366 (c): Replace
'done' by 'made'
32.
33.
34.
(a) Delete for after await.
(c) debarred from sending is correct.
asy
(a) Do not use of with despite. Despite means inspite of. SPEED TEST 65
35.
36.
(a) Change it to ‘yielded to’
(b) dying of hunger is correct.
En 1. (a) 2. (d) 3. (b) 4. (c) 5. (d)
37.
38.
39.
(c) Change than by ‘to’.
(d) Change ‘by’ to ‘for’.
(b) Change ‘to stay in’ by ‘rather than stay in’. She prefers gin
6.
11.
16.
(c)
(d)
(a)
7.
12.
17.
(d)
(c)
(d)
8.
13.
18.
(a)
(c)
(b)
9.
14.
19.
(d)
(d)
(c)
10.
15.
20.
(b)
(b)
(c)
40.
to write rather than to speak on telephone.
(c) Change ‘startled by’ to ‘startled at’.
21.
22.
(e)
(d) eer
Use ‘divided’ instead of ‘dividend’.
41. (b) Change ‘with’ to ‘from’.

SPEED TEST 64
23.
24.
(a)
(b) ing
Use ‘agreed’ properly for ‘consented’.
Use ‘file’ instead of ‘fill’.

1. (d) Replace ‘their’ by ‘its’. The banker’s association –


collective noun – so pronoun ‘its’ singular.
25.
26.
27.
(c)
(e)
(b)
Apply ‘funds’ for ‘investment’.

Use ‘able’ instead of ‘enable’. .ne


2.

3.
4.
(b) Replace ‘are’ by ‘is’. ‘five quintals’ refers a definite
quantity (as collective noun) so verb will be singular.
(a) Replace ‘have’ by ‘has’. Dickens is the name of a person.
(d) Insert ‘are’ after ‘hopes’. Hopes is plural, so verb will
28.
29.
30.
(d)
(b)
(a)
Use ‘overcome’ instead of ‘resort’.
Use ‘spread’ instead of ‘spend’.
Use ‘spent’ properly for ‘initiated’.
t
be plural. 31. (c) The correct spelling is souvenirs which mean memento,
5. (a) Place be after may. reminder, memorial etc.
6. (d) : Remove' a matter' 32. (a) It should be wasted instead of vested.
7. (c) : Remove' for preparing' 33. (d) It should be griping which means a sudden strong pain
8. (b) : Either' must' or' 'have to' alone should be used in stomach.
9. (c) : Raplace' were' by 'are' . 34. (e) No correction is required.
10. (c) : Replace' indefinite' by , indefinitely' . 35. (b) The correct spelling is charming.
11. (d) : Replace' days' by daily' . 36. (d) It should be ‘here was her son’s future’.
12. (d) : Replace' needs' by' need'
37. (c) The correct spelling should be ‘triumphant’.
13. (d) : Replace' of' by in'
38. (a) The word suppressed should be replaced by revealed
14. (c) : Replace' appreciating' by' appreciated
15. (a) : Remove' being' or leaked in the sentence.
16. (b) : Replace 'are' by 'is' 39. (b) It should be ‘handed over’ which means the act of
17. (d) : Remove 'independent' moving power or responsibility from one person to
18. (d) : Replace 'does' by 'did' another.
40. (a) The correct spelling is difficult.

Downloaded From : www.EasyEngineering.net


y
Downloaded From : www.EasyEngineering.net

o
u
rs
m
a
h
SOLUTIONS 79

b
o
o
SPEED TEST 66 32. (e) Piece of cake means something easy to do. Therefore,

b
option (e) is the correct choice.

.w
1. (b) 2. (d) 3. (d) 4. (b) 33. (b) Took to one’s heels means to run away. Therefore,

o
rd
5. (c) 6. (d) 7. (a) 8. (a) option (b) is the correct choice.

p
9. (a) 10. (c) 11. (d) 12. (d) 34. (e) To pledged means to make a promise. Therefore, option

re
13. (c) 14. (d) 15. (c) 16. (d) (e) is the correct choice.

s
s
17. (a) 18. (c) 19. (c) 20. (a) 35. (d) Crying need means a definite or desparate need for

.c
21. (a) 22. (b) 23. (b) 24. (a)

o
someone or something.

m
25. (d) 26. (b) 27. (d) 28. (c) Therefore, option (d) is the correct choice.
29. (c) 30. (b)
36. (d) Light upon means to arrive at something by chance.
Therefore, option (d) is the correct choice.
SPEED TEST 67

Sol: (Qs. 1-5): Clearly C must be followed by D, which must be SPEED TEST 69
further followed by the E as E reitrates the housing shortage and
says that the real deficit will be even higher. D and E provide the 1. (d) 2. (c) 3. (a) 4. (a)
statistical proof of the staggering task mentioned in C. So this 5. (b) 6. (c) 7. (b) 8. (b)

ww
leads us to two options (b) and (d). Among them (b) seems to be
more appropriate as B again emphasises on but is being said is A
and also that B cannot be the concluding statement of the
9.
13.
17.
(e)
(b)
(e)
10.
14.
18.
(a)
(a)
(b)
11.
15.
19.
(b)
(c)
(a)
12.
16.
20.
(e)
(b)
(e)

1.
5.
(a)
(e) w.E
paragraph. Hence, ABCDE gives the correct arrangement.
2. (b) 3. (c) 4. (d)
21.
25.
29.
(c)
(e)
(c)
22.
26.
30.
(b)
(a)
(a)
23.
27.
31.
(d)
(d)
(c)
24.
28.
32.
(a)
(b)
(d)

asy
Sol. For (Qs.6-10) : The arrangement EABDC is correct.
The paragraph is clearly taking about Goa state and hence E has
to be opening sentence is the paragraph. This is followed by A
33. (e) 34. (b)

SPEED TEST 70
where the phrase, ‘is an impressive case in point’, which is an
En
example of what is being said in E. A is followed by E as ‘a similar
agitation’ mentioned in B refers to the public activism mentioned
1. (b) 2. (d) 3. (e) 4. (b)
in A. B is further followed by D and C.
6. (e) 7. (a) 8. (b) 9. (d) gin
5.
9.
13.
(c)
(c)
(e)
6.
10.
14.
(c)
(b)
(b)
7.
11.
15.
(a)
(d)
(a)
8. (d)
12. (a)
16. (b)
10. (c)
Sol For. (Q.11-15): The required arrangement is ABDCE.
A is the opening sentence as is clear from the given options. A is
17.
21.
(d)
(d) intoeer 18. (e) 19.
22.
(c)
(a)
20. (c)
around
followed by B as ‘the proposal’ mentioned in B is reffering to
whatever has been talked in A. B is followed by D as D continues
23.
25.
27.
(b)
(c)
(a)
chance
other ing
translating 24.
26.
28.
(c)
(d)
(e)
practice
provided
off
to talk about the response mentioned in B. Also note that B
mentions a 2 : 1 response against the proposal which is also clear
by the 68 : 31 mentioned in D. D is followed by C and C is followed
29.
31.
(e)
(a)
hard
provides
30.
32.
(b)
(e) .ne equilibrium
form
by E. The ‘other immigration organizations’ mentioned in E clearly
states that the previous sentence must have a statement from
some other organization, which is the immigration lawyers as
mentioned in C.
33.
35.
(e)
(d)
symbol
emphasis
34.

SPEED TEST 71
(b) made
t
11. (a) 12. (b) 13. (d) 14. (c)
15. (e) 16. (e) 17. (b) 18. (e)
1. (b) 2. (a) 3. (c) 4. (b)
19. (a) 20. (c) 21. (a) 22. (b)
5. (d) 6. (c) 7. (b) 8. (c)
23. (c) 24. (d) 25. (d) 26. (d)
27. (a) 28. (a) 29. (a) 30. (d) 9. (b) 10. (d) 11. (c) 12. (c)
13. (a) 14. (e) 15. (b) 16. (e)
SPEED TEST 68 17. (b) 18. (c) 19. (e) 20. (d)
21. (e) 22. (d) 23. (e) 24. (b)
1. (b) 2. (b) 3. (a) 4. (b) 25. (b) 26. (d) 27. (d) 28. (a)
5. (c) 6. (a) 7. (d) 8. (d) 29. (c) 30. (c) 31. (e) 32. (a)
9. (d) 10. (c) 11. (c) 12. (b) 33. (c) 34. (a) 35. (d)
13. (b) 14. (a) 15. (b) 16. (a) 36. (b) substitute has for are.
17. (b) 18. (d) 19. (c) 20. (b) 37. (b) replace should by will.
21. (b) 22. (b) 23. (a) 24. (d)
38. (a) sale should come in place of selling
25. (b) 26. (b) 27. (a) 28. (d)
29. (a) 30. (d) 31. (b) 39. (c) earned.
40. (d) replace since with from.

Downloaded From : www.EasyEngineering.net


y
Downloaded From : www.EasyEngineering.net

o
u
rs
m
a
h
80 101 SPEED TEST

b
o
o
SPEED TEST 72 21. (a) 22. (a) 23. (a) 24. (b)

b
25. (c) 26. (a) 27. (e) 28. (e)

.w
29. (c) 30. (d)

o
1. (a) 2. (c) 3. (a) 4. (b)

rd
5. (e) 6. (b) 7. (b) 8. (d)

p
SPEED TEST 77

re
9. (a) 10. (e) 11. (e) 12. (b)

s
13. (a) 14. (b) 15. (a) 16. (e)

s
1. (b) 2. (b) 3. (b) 4. (c)

.c
17. (d) 18. (a) 19. (e) 20. (b)

o
21. (b) 22. (e) 23. (d) 24. (a) 5. (c) 6. (a) 7. (a) 8. (d)

m
25. (d) 26. (b) 27. (a) 28. (c) 9. (a) 10. (d) 11. (b) 12. (d)
29. (b) 30. (c) 13. (d) 14. (d) 15. (b) 16. (e)
17. (b) 18. (e) 19. (e) 20. (e)
SPEED TEST 73 21. (b) 22. (b) 23. (e) 24. (c)
25. (d) 26. (c) 27. (e) 28. (c)
1. (e) 2. (a) 3. (d) 4. (a) 29. (d) 30. (a)
5. (b) 6. (d) 7. (e) 8. (e)
9. (a) 10. (e) 11. (c) 12. (c)

ww
SPEED TEST 78
13. (b) 14. (a) 15. (b) 16. (d)
17. (d) 18. (c) 19. (c) 20. (d) 1. (b) 2. (e) 3. (e) 4. (e)
21.
25.
29.
(b)
(b)
(c) w.E
22.
26.
30.
(c)
(e)
(c)
23.
27.
(c)
(e)
24.
28.
(d)
(c)
5.
9.
13.
17.
(a)
(a)
(e)
(e)
6.
10.
14.
18.
(b)
(d)
(a)
(c)
7.
11
15.
19.
(a)
(a)
(b)
(d)
8.
12.
16.
19.
(c)
(c)
(d)
(c)
SPEED TEST 74
asy 21.
25.
(d)
(b)
22.
26.
(d)
(a)
23.
27.
(e)
(d)
24.
28.
(e)
(a)
1.
5.
(e)
(b)
2.
6.
(a)
(c)
3.
7.
(d)
(b)
4.
8.
En
(b)
(a)
29. (e) 30. (a)

9.
13.
17.
(a)
(d)
(c)
10.
14.
18.
(c)
(a)
(c)
11.
15.
19.
(a)
(b)
(c)
12.
16.
20.
(d)
(d)
(d) gin
1. (e) 2.
SPEED TEST 79

(b) 3. (c) 4. (a)


21.
25.
(a)
(a)
22.
26.
(d)
(c)
23.
27.
(a)
(b)
24.
28.
(a)
(a)
5.
9.
13.
(b)
(b)
(d) eer
6.
10.
14.
(a)
(c)
(e)
7.
11.
15.
(d)
(c)
(e)
8.
12.
16.
(e)
(c)
(c)
29. (b) 30. (c)

SPEED TEST 75
17.
21.
(d)
(c) ing
18.
22.
(e)
(d)
19.
23.
(c)
(d)
20.
24.
(e)
(e)

1.
5.
(c)
(a)
2.
6.
(a)
(b)
3.
7.
(c)
(c)
4.
8.
(c)
(b)
25.
29.
(a)
(c)
26.
30.
(a)
(b)
27. (c)

.ne 28. (d)

9.
13.
17.
(a)
(a)
(e)
10.
14.
18.
(c)
(c)
(a)
11.
15.
19.
(a)
(d)
(b)
12.
16.
20.
(b)
(d)
(a)
1.
5.
(e)
(c)
2.
6.
SPEED TEST 80

(b)
(c)
3.
7.
(d)
(a)
4.
8.
t
(a)
(c)
21. (b) 22. (c) 23. (b) 24. (a)
25. (a) 26. (a) 27. (e) 28. (a) 9. (c) 10. (a) 11. (d) 12. (b)
29. (a) 30. (c) 13. (c) 14. (a) 15. (c) 16. (e)
17. (c) 18. (d) 19. (a) 20. (d)
SPEED TEST 76 21. (e) 22. (c) 23. (a) 24. (a)
25. (a) 26. (a) 27. (e) 28. (e)
1. (e) 2. (d) 3. (c) 4. (e) 29. (d) 30. (a) 31. (b) 32. (c)
5. (d) 6. (e) 7. (d) 8. (c) 33. (e) 34. (e) 35. (c) 36. (e)
9. (c) 10. (b) 11. (c) 12. (b)
37. (a) 38. (d) 39. (c) 40. (d)
13. (d) 14. (e) 15. (b) 16. (d)
17. (b) 18. (a) 19. (d) 20. (a)

Downloaded From : www.EasyEngineering.net


y
Downloaded From : www.EasyEngineering.net

o
u
rs
m
a
h
SOLUTIONS 81

b
o
o
SPEED TEST 81 20. (b) As per RBI, growth of card acceptance infrastructure like

b
.w
automated teller machines (ATMs) and point-of-sale (PoS)
terminals is not on a par with that of card issuance.

o
1. (b) 2. (c) 3. (a) 4. (b) 5. (b) 6. (c)

rd
7. (d) 8. (c) 9. (c) 10. (c) 11. (d) 12. (b) Debit cards vastly outnumber the volume of credit cards

p
13. (d) 14. (d) 15. (b) 16. (d) 17. (a) 18. (c) issued in the country. Further, a high number of debit

re
19. (b) 20. (a) 21. (c) 22. (d) 23. (d) 24. (d) cards have been issued in recent times under the Prime

s
s
25. (b) 26. (c) 27. (b) 28. (a) 29. (d) 30. (b) Minister's Jan Dhan Yogana, especially to customers

.c
in rural areas and smaller towns.

o
m
SPEED TEST 82 Cash continues to be the predominant mode of payment
as it appears to be "costless" in comparison to the
1. (c) The accounting year of RBI starts the month between visible costs associated with card/electronic payments.
July-June. Some of the factors that have inhibited growth in the
2. (a) In Article-30, RBI permitted to the co-operative Bank acceptance infrastructure are lack of adequate and low-
for special account supervision. cost telecom infrastructure and lack of incentive for
3. (c) Open market operations of RBI refers to trading in merchants for acceptance of cards, among others.
securities. 21. (b) Reserve Bank of India said the government will buy
4. (b) Monetary policy in India is formulated and implemented back inflation indexed bonds/IIBs maturing in 2023 via

5.
ww
by RBI.
(a) Reserve Bank of India follows minimum reserve system
for the issuing of currency.
reverse auction in February 11th. Not so effective bonds
have received a below par response due to lack of
marketing and associated tax issues. GoI has also
6.

7. w.E
(b) RBI controls credit creation by the Commercial Bank in
India.
(b) Note issuing department of RBI should always possess
notified repurchase of 1.44 percent inflation indexed
government stock 2023 via a reverse auction for total
amount of INR 6500 crore. Bonds were launched as
alternative to gold as an investment in high current
8.
asy
the minimum gold stock worth ` 115 crore.
(b) RBI and CSO in India is entrusted with the collection
of data of capital formation.
account deficit. Repurchase will be prematurely held to
redeem government stock through utilization of surplus

En
9. (c) The Bank rate is the rate at which RBI gives credit to cash balance and it was an ad-hoc move.
the commercial Banks. 22. (a) RBI has four Deputy Governors. Two are from outside,
a commercial banker and an economist and two others

gin
10. (b) An increase in CRR by the RBI results in reduction in
liquidity in the economy. are promoted from within its ranks.
11. (c) Commercial Banks provide the largest-credit to - Deputy Governor of RBI can be appointed for a period
of five years or till the age of 62, whichever is earlier.
12.
agriculture and allied sectors.
(a) RBI publishes the financial report on currency and
finance. eer
Current other three Deputy Governors of RBI are:R.
Khan, S.S. Mundra and R. Gandhi.
13.
14.
15.
(a) RBI is the custodian of India’s foreign exchange funds.
(c) RBI sanctions foreign exchange for the import of goods.
(d) There are 4 posts of deputy Governor in Reserve Bank ing
- The Union Government has reappointed Dr Urjit Patel
as Deputy Governor of Reserve Bank of India (RBI).
- Dr Urjit Patel headed committee to review the monetary

16.
of India.
(d) RBI regulates the external commercial borrowings. 23.
policy framework.
.ne
(b) Urijit Patel, Deputy Governor of RBI in charge of the

t
17. (d) All the statements given above are correct except that, monetary policy department, has been appointed for a
RBI was established in 1949. term of three years for the second time. Patel, 52, will
18. (b) RBI has tweaked upcoming new rules for lending rates. finish his 3 year term on January 10, and has become
The central bank said on 29th March 2016 the fixed rate the longest serving deputy governor in 2015-2016 if he
loans of close to three years offered by lenders will be serves the full three year term. Many of the recent deputy
linked to the marginal cost of funding. Loans above governors served a maximum period of only 5 years. A
that tenor should be exempt. Earlier, all fixed rate loans deputy governor can serve for the period of three years
have been exempted from being set based on marginal w.e.f taking on the charge on or after January 11,2016
cost of funding. The change will apply to new rules or till further orders, whichever is earlier. A PhD from
implemented from April 1, 2016.New rules will force Yale University in Economics and graduate of the
lenders to adjust lending rates in relation to market University of London and Oxford, he has served as the
rates removing the sector's discretion in making a head of the U. Patel Committee proposing inflation
decision as to how much to charge for loans. targeting as the prime objective of the central bank. As
19. (d) Reserve Bank has provided relief to banks holding per the agreement, RBI will target 4 percent inflation at
bonds of State Electricity Boards that were debt ridden. the close of the financial year 2016-2017 and for
Banks have been allowed to keep Ujwal Discom subsequent years, with a band of +/- 2 percent. Other
Assurance Yojana scheme bonds under held to deputy governors of the RBI are H. R. Khan, R. Gandhi
maturity category. This will lessen pressure on bond and S. S. Mundra.
yields and debt market. HTM is part of the debt holdings 24. (c)
of the bank not subjected to daily price movement 25. (c) Since March, 1934 India is a member of the International
which can be held by banks till maturity. Monetary Fund. It has to, therefore, maintain its rate of

Downloaded From : www.EasyEngineering.net


y
Downloaded From : www.EasyEngineering.net

o
u
rs
m
a
h
82 101 SPEED TEST

b
o
o
exchange at the level which it has declared to the IMF. 10. (c) VAT is imposed on all stages between production and

b
.w
The Reserve Bank takes suitable measures to maintain final sale.
the value of the rupee at this declared level. 11. (c) Balance of payment is used in terms of Exports and

o
rd
Imports.

p
SPEED TEST 83 12. (b) The Indian Economy can be described as a developing

re
economy.

s
s
1. (a) 2. (c) 3. (d) 4. (b) 13. (b) 14. (d) 15. (d) 16. (d)

.c
5. (a) 6. (d) 17. (a) 18. (a)

o
m
7. (c) To provide basic banking services to bankless villages 19 (b) A company which has net owned funds of at least
8. (b) 9. (b) 10. (d) 11. (a) ` 300 Crore and has deployed 75% of its total assets in
12. (d) 13. (a) 14. (a) Infrastructure loans is called IFC provided it has credit
15. (a) RBI has given nod to Muthoot Finance to set up White rating of A or above and has a CRAR of 15%.
Label ATMs .ATMs set up and run by non-banking 20. (a) If the IDF is set up as a trust, it would be a mutual fund,
entities are called White Label ATMs (WLAs). regulated by SEBI.
16. (c) It is a special type of credit card which is sponsored by 21. (a) NBFC-MFI is a non-deposit taking NBFC which has at
both the credit card issuing company and the least 85% of its assets in the form of m microfinance.
participating retail company or vendor. Co-branded Such microfinance should be in the form of loan given
to those who have annual income of `60,000 in rural

17. ww
credit card carries special deals and savings from the
participating merchants.
(a) Credit cards linked to special organizations like sports
areas and `120,000 in urban areas. Such loans should
not exceed `50000 and its tenure should not be less
than 24 months. Further, the loan has to be given without

w.E
clubs, exclusive clubs and charities. Affinity credit cards
can also help raise funds, when a part of income from collateral. Loan repayment is done on weekly,
every transaction goes toward the benefit of relevant fortnightly or monthly instalments at the choice of the
organization. borrower.
18.
19.
(b)
asy
(a) Visa card In 1977, Visa was adopted internationally and
became the first credit card facility to be recognised
22. (d) Factoring refers to the process of managing the sales
register of a client by a financial services company.
Basically, there are three parties involved in a factoring

En
worldwide. It is a conditional authorization given by a
competent authority of a country for a person who is
transaction: 1. The buyer of the goods 2. The seller of
the goods 3. The factor, i.e. The financial institution.

gin
not a citizen of that country to enter its territory and to 23. (b) All NBFCs are not allowed to take deposits. Only those
remain there for limited duration. NBFCs which have specific authorization from RBI are
20. (c) A hot card is a lost or stolen card. A hot list is the list of allowed to accept/hold public deposits. NBFCs cannot

21. (d)
caution against the use of a credit card by a defaulter
holder.
eer
take demand deposits. They can accept only term
deposits with a tenure of minimum 12 months.
24. (b) Liquid assets are those assets that can be exchanged

SPEED TEST 84
ing
most readily with minimum number of obstacles and
minimum time.
25. (b) Chanakya
1. (e)
7. (c)
2. (a)
8. (a)
3. (b)
9. (c)
4. (e)
10. (a)
5. (a)
11. (e)
6. (b)
12. (b)
26. (d) Both (b) and (c)
.ne
27. (a) Ratio of money held by the public in currency to that of
13. (a)
19. (c)
25. (c)
14. (a)
20. (c)
26. (b)
15. (a)
21. (c)
27. (b)
16. (b)
22. (d)
28. (d)

SPEED TEST 85
17. (b)
23. (d)
29. (c)
18. (a)
24. (d)
30. (d)
money held in bank deposits.

as reserves. t
28. (b) The proportion of total deposits commercial banks keep

29. (c) The fraction of the deposits that commercial banks must
keep with RBI
30. (b) This includes the currency (notes and coins in
1. (c) Tertiary sector of Indian economy contributes largest circulation and vault cash of commercial banks) along
to the Gross National Product. with the deposits held by the Government of India ad
2. (a) Service sector is the main source of National Income in commercial banks with RBI.
India.
3. (d) Toll tax is not a tax levied by the government of India. SPEED TEST 86
4. (d) The most appropriate measure of a country’s economic
growth is it’s per capita product. 1. (c) ‘Eco Mark’ is given to the Indian products for
5. (c) Foreign Exchange Management Act (FEMA) was finally Environment Friendly purpose.
implemented in the year 2002. 2. (b) The Earlier name of WTO was GATT before 1995.
6. (b) The most common measure of estimating inflation in 3. (b) World Development Report is an annual publication of
India is WPI. IBRD.
7. (d) The national income of India is estimated by CSO. 4. (a) India has the maximum volume of foreign trade with
8. (c) Finance Ministry formulates the fiscal policy in India. USA.
9. (b) The devaluation of rupee in India took place twice in 5. (b) Participatory notes (PNs) are associated with Foreign
the financial year 1991-92. Institutional Investors (FII’s).

Downloaded From : www.EasyEngineering.net


y
Downloaded From : www.EasyEngineering.net

o
u
rs
m
a
h
SOLUTIONS 83

b
o
o
6. (b) The purpose of India Brand Equity Fund to make ‘Made

b
incorporating state of the art weaponry and sensors

.w
in India’ a label of quality. including the extended range Barak 8 surface-to- air
7. (a) A trade policy consists of Export-Import policy.

o
missiles.

rd
8. (b) FERA in India has been replaced by FEMA. 20. (a) Indian railway has launched the E-Samiksha an online

p
9. (b) TRIPS and TRIMS are the terms associated with WTO. project monitoring system with an aim of monitoring

re
10. (c) In the year 2006, SEZ act was passed by the parliament. implementation of various ongoing projects including

s
s
11. (d) There are total 162 members recently in WTO. Rail Budget proposals. Apart from budget-related

.c
12. (d) Capital account is classified into 3 parts in India-

o
projects, the E-Samiksha can also be used for

m
private, banking and official capital. monitoring the infrastructure target and board meeting
13. (a) The investment in productive assets and participation follow-up.
in management as stake holders in business enterprises 21. (c) The Government of India, through its Ministry of
is foreign direct investment. Environment & Forests, is implementing a project titled
14. (b) The portfolio investment by foreign institutional "Capacity Building for Industrial Pollution Management"
investors is called foreign institutional investment. under the assistance of World Bank. The project
15. (c) 16. (c) 17. (b) 18. (b) objectives are strengthening the environmental
19. (a) 20. (c) 21. (d) 22. (d) management capacity of central and state level
23. (b) 24. (a) 25. (c) 26. (d) regulatory authorities with emphasis on rehabilitation
27. (c)

ww 28. (c)

SPEED TEST 87
29. (c) 30. (b) of polluted sites and for undertaking area-based
demonstration projects on remediation of contaminated
sites. The project also aims at developing a "National

1.

2.
w.E
(c) Swabhiman scheme is associated with Rural Banking
in India.
(c) The unorganised workers social security Act was 22. (a)
Program for the Rehabilitation of Polluted Sites" to
reduce or eliminate the environmental and health risks
associated with legacy pollution.
The union government has set up a panel headed by B

3.
passed in 2008.
asy
(c) Swadhar scheme launched for the women in difficult
N Navalawala to look into various contentious issues
relating to inter-linking of rivers. The panel will works
towards speedy implementation of the inter-linking of

En
circumstances.
4. (a) The main objective of Pradhan Mantri Gramodaya river projects. The panel comprising of water experts
Yojana is meeting rural needs like housing, drinking, and senior officials of different ministries will facilitate

5.
water, healthcare, etc.
(b) Twenty point Economic programme was first launched
in 1975. gin
23. (a)
interlinking of intra-state and intra- basin rivers.
Dr Nasim Zaidi has been appointed a next Chief Election
Commissioner of India. He will assume the charge of
6. (a) The disguished unemployment is 9 prominent feature
mainly of primary sector. eer
the office from April 19, after the incumbent H S Brahma
will retire on April 18.

ing
7. (a) Golden Quadrangle project is associated with Highways 24. (d) This scheme was announced to enable minority youth
development. to obtain school leaving certificate and gain better
8. (d) All the above given statements is the objective of employment.

9.
National Food security Mission.
(c) The period 2012-17 is related to 12th Five year plan in
25. (c)

.ne
Nawaj Shaikh is from National AIDS Research Institute
(NARI) and belongs to Pune. The said competition was
held through the MyGov Platform for suggesting the

t
India.
10. (b) Liberalization of Economy is not a measure of reducing Logo, Slogan and Tagline for the New Education Policy
inequalities. of the Government of India. More than 3000 entries
11. (a) Valmiki Awas Yojana subsumed Integrated Housing were received. It was for the first time ever that this
and slum development programme. particular logo was designed by a common man of the
12. (c) Poverty level in India is established on the basis of country and not by any advertising agency or a
house hold consumer expenditure. corporate house. A cash prize of `10,000 will be
13. (b) Nirmal Bharat Abhiyan Yojana is associated with awarded to Shaikh.
community toilets in slum areas.
SPEED TEST 88
14. (c) Crop Insurance scheme in India was started in 1985.
15. (d) 'SJSRY' Scheme is not related to the rural development.
1. (a) Ruchir Kamboj, currently the Chief of Protocol, has
16. (d)
been appointed as the next Permanent Representative
17. (c) ICDS programme focused on the age group of children
of India to UNESCO, Paris, with the rank of ambassador.
upto 6 years.
The 1987-batch IFS officer, Kamboj will succeed V S
18. (d) The age group of 40-79 years old women are eligible for Oberoi.
India Gandhi Widow Pension Scheme. 2. (a) Government of India has appointed Justice Vangala
19. (b) The Project 15B of Indian navy aims to develop stealth Eswaraih, former acting Chief Justice of Andhra
guided missile destroyers. Under the project Mazagon Pradesh as the chairperson of the National Commission
Dock Limited will construct four stealth guided missile for Backward Classes. The National Commission of
destroyers. Project 15B destroyers will feature Backward Classes has been set up under the National
enhanced stealth characteristics as well as Commission for Backward Classes Act 1993.

Downloaded From : www.EasyEngineering.net


y
Downloaded From : www.EasyEngineering.net

o
u
rs
m
a
h
84 101 SPEED TEST

b
o
o
3. (e) 24. (a) Nuclear Security Summit 2016 took place in Washington

b
.w
4. (c) Justice Swatanter Kumar is the present Chairperson of DC on April 1, 2016 and leaders comprised PM Narendra
National Green Tribunal of India. Modi, UK PM David Cameron, Canadian PM Justin

o
rd
5. (b) Prime Minister of India chairs the National Ganga River Trudeau, French President Francois Hollande and other

p
Basin Authority. world leaders. World leaders have stressed the

re
6. (b) importance of Convention on Physical Protection of

s
s
7. (b) Shri Ratan Thiyam has been appointed as the Nuclear Material and its amendment in 2005 as well as

.c
chairperson of the National School of Drama Society. the International Convention on the suppression of

o
m
The President of India has appointed Shri Thiyam for a acts of nuclear terrorism.
period of four years as per the relevant rules and 25. (a) The 13th edition of the World Spice Congress began in
regulations of the Society. Ratan is a playwright and Ahmedabad, Gujarat and the theme of the two yearly
theatre director. three day event is 'Target 2020: Clean, Safe and
8. (e) 9. (e) 10. (c) 11. (c) Sustainable Supply Chain'. The World Spice Congress
12. (e) 13. (d) 14. (c) 15. (a) has been organised jointly by Spices Board, Cochin
16. (c) 17. (b) 18. (b) 19. (a) Hill Produce Merchant's Association and IndiaPepper
20. (a) 21. (a) 22. (b) 23. (c) and Spice Trade Association. It is given support by All
24. (c) 25. (a) 26. (d) 27. (b) India Spices Importers & Distributors Association and
28. (c)
ww 29. (b) 30. (c)

SPEED TEST 89
Indian Spices & Foodstuff Exporters' Association.
26. (a) The Union Health and Family Welfare Minister Shri JP
Nadda chaired the 12th Conference of the Central

1.
2. w.E
(d) Brazil is not a member of ASEAN.
(b) ‘Sanklap’ project is associated with the eradication of
Council of Health and Family Welfare which discussed
the Draft of the National Health Policy 2016. Some of
the resolutions adopted towards the close of the
meeting were as followed:
3.
HIV/AIDS.

asy
(b) 19th SAARC Summit will held in Islamabad, Pakistan in
2016.
(i) Health needs of country have changed over time
since the last National Health policy

En
(ii) Draft National Policy formulated by the Ministry
4. (d) The 29th summit of ASEAN will be organised in Laos.
of Health and Family Welfare, GoI has been
5. (d) The 2016 BRICS Summit will be held in Panaji, Goa in
formulated keeping this in mind
6.
India.
(a) The 2016 annual meeting of WEF organised in
Switzerland. gin (iii) Endorsement of the Draft National Health Policy
by the Central Council for Health and Family
Welfare.
7.
8.
(b) There are 21 members in the APEC nations organisation.
(d) The 17th NAM summit proposes to be held in
Venezuela. eer
27. (d) One day Annual Conference of the State Minorities
Commission commenced with the focus on inclusive
9.
10.
(c) The G-20 2016 Summit will be hosted in China.
(a) The 42nd summit of G-7 will be held in Japan. ing
growth with the motto "Sabka Saath, Sabka Vikas." The
one day Conference focusing on Development of
Minorities had two Technical sessions besides the
11.
12.
13.
(a) Azerbaijan and Fiji became the new members of NAM.
(b) The 2016 NATO summit organised in the Poland.
(a) A lady president elected first time in South Korea. .ne
inaugural. Valedictory. Mr. Praveen Davar addressed
the Technical I session on "Minority Welfare Schemes

t
of Government of India-An Overview".
14. (a) George Mario Bergoglio elected as a new pope of Roman Technical II session on "Functioning of State Minorities
Catholic Church. Commissions-Problems and Challenges" was
15. (a) The ‘UN Women’ came into existence on 1 July 2010. addressed by Ms. Mabel Rebello. Last year, the themes
16. (d) There are 10 members associated with BIMSTEC. of the conference were "Capacity building of Minority
17. (a) The Ranking of India in Global Hunger Index list is 63rd Educational Institutions in India" and "Corporate Social
last year. Responsibility and Development of Minorities in India".
18. (d) There are 8 members associated with MERCOSUR. 28. (a) This is the flagship initiative of the Shipping Ministry.
19. (a) The 9th World Hindi Conference held in South Africa. It provides an excellent platform for participants to fund
20. (c) There 8 point mentioned in MDG-2015 of United Nation. business opportunities. The Maritime India Summit will
21. (a) The headquarter of FAO (Food and Agriculture showcase investment opportunities in the maritime
organisation) is situated in Italy. sector for different activities such as port modernisation,
22. (b) SAMPRITI-III, 9 a special security forces exercise shipbuilding, ship repair and cycling. The aim is to
organised between India and Bangladesh. provide a platform for interacting closely with
23. (a) Shanghai Cooperation Organisation has set to pioneering international maritime organisations and
showcase alignment between China and Russia create awareness about trends and potential for
through integration of Beijing marshalled Silk Road maritime sector in India.
Economic Belt and Moscow driven Eurasian Economic 29. (d) Mr. Nitin Gadkari, Minister of Shopping, Road
Union. SCO is part of the emerging Eurasia-centred Transport and Highways launched a website entitled
Silk Road geopolitical architecture pillared by China www.maritimeinvest.in to commemorate the Maritime
and Russia, along with the Central Asian Republics. India Summit within the Make in India Summit. User

Downloaded From : www.EasyEngineering.net


y
Downloaded From : www.EasyEngineering.net

o
u
rs
m
a
h
SOLUTIONS 85

b
o
o
friendly website will facilitate access to detailed Rajasthani by a Rajasthani writer. It carries a citation, a

b
.w
information about the summit including registrations. plaque and prize money of ‘ 1 lakh.
Maritime India Summit 2016 is a global maiden summit 8. (a) Tansen Samman is conferred in the field of music which

o
rd
being carried out by the Shipping Ministry in April. For carries a cash prize of ‘ 2 lakh and a citation.

p
MIS 2016, India's partner country is the Republic of 9. (a)

re
Korea. More than 50 maritime nations have also been 10. (b) The Shanti Swarup Bhatnagar award for Science and

s
s
invited to attend the summit. Technology (SSB) is an award in India given annually

.c
30. (a) 5 day long 10th WTO Ministerial Conference was by the CSIR. It is named after the founder Director of

o
m
concluded on 19th December 2015 in Nairobi, Kenya the CSIR and carries an award money of ‘ 5 lakh each.
aimed at adoption of the Nairobi package for benefitting 11. (a) In 2006, Kejriwal was awarded the Ramon Magsaysay
the poorest members of the organisation. The Award for Emergent Leadership recognising his in-
conference was attended by trade ministers of 162 volvement in a grassroots movement Parivartan using
member countries of the WTO. India was represented right-to-information legislation in a campaign against
by Minister of State (Independent Charge) for corruption. The same year, after resigning from the IRS,
Commerce & Industry Nirmala Sitharaman. This marks he donated his Magsaysay award money as a corpus
the first time a meeting has been held in Africa by WTO fund to found the Public Cause Research Foundation,
in this capacity. a non-governmental organisation (NGO).

1.
ww SPEED TEST 90

(d) The Jnanpith award is a literary award which along


12. (a) Bidhan Chandra Roy Award was instituted in 1976 in
memory of B. C. Roy by Medical Council of India. The
Award is given annually in the categories of States-

w.E
with the Sahitya Akademi Fellowship is one of the two
most prestigious literary honours in the country. The
award was instituted in 1961. Any Indian citizen who
manship of the Highest Order in India, Medical man-
cum-Statesman, Eminent Medical Person, Eminent per-
son in Philosophy and Eminent person in Arts. It is
presented by President of India in New Delhi on July 1,

2.
gible for the honour.
asy
writes in any of the official languages of India is eli-

(a) Bharat Ratna is India’s highest civilian award. The offi-


13.
the National Doctors’ Day.
(a) Pampa Prashasti is the highest literary honour con-

En
ferred by the Karnataka government on a litterateur for
cial criteria for awarding the Bharat Ratna stipulated it
his/her lifetime contribution to Kannada literature. The
is to be conferred “for the highest degrees of national award is named after Adikavi Pampa of 10th Century.
service which includes artistic, literary, and scientific
achievements, as well as “recognition of public service
of the highest order”. The last recipient of the award is gin
14.
The award carries award money of ‘ 3 lakh and a cita-
tion.
(a) Madhya Pradesh has been awarded 10th National
3.
the cricketer Sachin Tendulkar for the year 2014.
(c) The National Film awards, one of the most prominent
film awards in India, were established in 1954. Every eer
Award for Excellence work in Mahatma Gandhi National
Rural Employment Guarantee Act (MGNREGA). The
year, a national panel appointed by the government
selects the winning entry, and the award ceremony is ing
award was given to Madhya Pradesh for convergence
of MGNREGA and other schemes for construction of
permanent assets and for excellent work in providing

4.
held in New Delhi where the President of India pre-
sents the awards.
(c) The Vyas Samman is a literary award which was first
15. (b)
.ne
opportunities for earning permanent income.

t
16. (d) The Nobel prize is a set of an international awards be-
awarded in 1991. It is awarded annually by the K.K. stowed in a number of categories which is given annu-
Birla Foundation and includes a cash of ally to the winners by Swedish and Norwegian Com-
‘ 250,000 (as of 2005). To be eligible for the award, the mittees in recognition of cultural and/or scientific ad-
literary work must be in the Hindi language and has vances. It was the will of the Swedish inventor Alfred
been published in the past 10 years. Nobel that established the Nobel prizes in 1895 in Swe-
5. (c) The Saraswati Samman is an annual award for outstand- den.
ing prose or poetry literary works in any Indian lan- 17. (c) The Nobel prize in Economics or Economic sciences
guage. It was instituted in 1991 by the K. K. Birla Foun- was established in 1968 and endowed by Sweden’s
dation. The award contains ‘ 10 lakh, a citation and a central bank, the Sveriges Riksbank, on the occasion
plaque. Candidates are selected from literary works of the bank’s 300th anniversary. While the Nobel Prize
published in the previous ten years by a panel that in particular was established in 1895.
includes scholars and former award winners. 18. (a) The Academy award is also known as the Oscar award
6. (a) The Maha Vir Chakra is the second military decoration in which is presented for various categories in the Film
India and is awarded for acts of conspicuous gallantry in industry. It was first given in 1929.
the presence of the enemy, whether on land, at sea or in the 19. (c) The Confucius Peace prize is a prize established in 2010
air. The medal may be awarded posthumously. in the People’s Republic of China (PRC). The Confucius
7. (d) Bihari Puraskar conferred by Rajasthan is a literary Peace Prize’s first winner was former Vice President of
award instituted by K. K. Birla Foundation. The award the Republic of China and Kuomintang Chairman Lien
is named after the famous Hindi poet Bihari and is Chan, for his contribution to developing positive ties
awarded to an outstanding work published in Hindi or between Taiwan and mainland China.

Downloaded From : www.EasyEngineering.net


y
Downloaded From : www.EasyEngineering.net

o
u
rs
m
a
h
86 101 SPEED TEST

b
o
o
20. (a) The Pulitzer Prize is a U.S. award for achievements in 14. (b) Anand Math is a 1952 Hindi patriotic-historical film

b
.w
newspaper and online journalism, literature, and musi- directed by Hemen Gupta, based on ‘Anandamath’,
cal composition. It was established in 1917 and admin- the famous Bengali novel written by Bankim Chandra

o
rd
istered by Columbia University in New York City by Chattopadhyay in 1882. The novel and film are set in

p
provisions in the will of American publisher Joseph the events of the Sannyasi Rebellion, which took place

re
Pulitzer. in the late 18th century in eastern India, especially

s
s
21. (c) The Nobel awards in literature, medicine, physics, chem- Bengal.

.c
istry, peace, and economics are given in Stockholm, 15. (b) 16. (b) 17. (c) 18. (c)

o
m
Sweden. The Peace prize is awarded in Oslo, Norway. 19. (b) 20. (b)
22. (a) The British Academy Film awards are presented in an 21. (a) The Vice President of India Shri M. Hamid Ansari
annual award show hosted by the British Academy of released a book titled “India and Malaysia: Interwined
Film and Television Arts (BAFTA). It is given by UK Strands” authored by former diplomat Smt Veena Sikri.
and is considered to be the counter awards for Oscars. The book offers a panoramic yet in-depth historical
23. (c) Golden Globe award is given in the field of film and analysis of the inter-linkages between India and
television by Hollywood Foreign Press Association in Malaysia, which are a microcosm of the much larger
United States of America. relationship between South Asia and South East Asia,
24. (a) The Palme d’Or is the highest prize awarded at the as these have evolved more than two millennia.

ww
Cannes Film Festival and is presented to the director of
the best feature film of the official competition. It is
presented by Festival International du film de, France.
22. (d) The Global Competitiveness Report is a yearly report
published by the World Economic Forum. The report
assesses the ability of countries to provide high levels

w.E
25. (a) The Kalinga Prize for popularization of Science is an
international distinction instituted by UNESCO. It was
started in 1951 by donation from Mr Bijoyanand Patnaik,
founder and president of the Kalinga Foundation Trust
of prosperity to their citizens. This in turn depends on
how productively a country uses available resources.
Since 2010, Switzerland has led the ranking as the most
competitive economy in the world India is at 60th
in India.
asy
26. (a) International Gandhi Peace prize is given annually by
position.
23. (b) Ashtadhyayi is written by Panini. Panini a sage who is

En
Government of India to those individuals and organi- believed to have lived around the fifth century B.C.,
zations which contribute towards changes in the po- although other claims trace him to the 4th, 6th and 7th
litical, social or economic reforms via non-violence. It centuries, is credited with having created the
was instituted in 1995.
27. (c) The Royal Institute of British Architects Stirling Prize
is a British prize for excellence in architecture. It is gin Ashtadhyayi, or Ashtak, which is a grammar defining
the structure and syntax of the Sanskrit language.
24. (c) Mitakshara is written by Vigyaneshwar. It was consid-
organised and awarded annually by the Royal Insti-
tute of British Architects (RIBA).It is named after the eer
ered one of the main authorities on Hindu Law from the
time the British began administering laws in India.
25. (c) Kautilya’s Arthashastra is an excellent treatise on state-
architect James Stirling.
28. (a) The World Economic Forum gives Crystal award to
those artists who have improved the state of the world ing
craft, economic policy and military strategy. it is said to
have been written by Kautilya, also known by the name
Chanakya or Vishnugupta, the prime minister of India’s
through their art.
29. (d) Ramon Magsaysay award is given annually to those
.ne
first great emperor, Chandragupta Maurya.
26. (d) The Post Office (Bengali: Dak Ghar) is a 1912 play by

t
Asian people who have contributed extraordinary ser- Rabindranath Tagore. It concerns Amal, a child confined to
vice in their respective fields. This award is given by his adopted uncle’s home by an incurable disease.
Philippine in the memoir of Philippine President Ramon 27. (c)
Magsaysay. He is considered to be one of the great 28. (b) English author Rudyard Kipling wrote the Jungle Book
examples of integrity, courage, and idealistic democrat. in 1894. It is a collection of stories.
30. (a) 29. (a) War and Peace is a novel by the Russian author Leo
Tolstoy, first published in 1869. The work is epic in
SPEED TEST 91 scale and is regarded as one of the most important
works of world literature. It is considered as Tolstoy’s
1. (c) The book presents the booming maintenance of the finest literary achievement, along with his other major
world’s largest democracy and achievements of India prose work, Anna Karenina (1873–1877).
since independence. 30. (b) The Prince is a 16th-century political treatise by the
2. (d) Ambedkar Speaks (Trilogy) is authored by Dr. Narendra Italian diplomat and political theorist Niccolò
Machiavelli. The descriptions within The Prince have
Jadhav.It is an attempt to develop a comprehensive
the general theme of accepting that the aims of princes—
bibliography on the speeches delivered by Ambedkar.
such as glory and survival—can justify the use of im-
3. (c) 4. (a) 5. (b) moral means to achieve those ends.
6. (d) A pictorial coffee table book by Alam Srinivas was
launched on 24 June 2013 in Mumbai by former Bombay SPEED TEST 92
High Court Chief Justice C S Dharmadhikari.
7. (b) 8. (a) 9. (b) 10. (a) 1. (c) The Olympic Museum is located in Lausanne, Switzer-
11. (d) 12. (d) 13. (a) land. The museum was founded on 23 June 1993, on

Downloaded From : www.EasyEngineering.net


y
Downloaded From : www.EasyEngineering.net

o
u
rs
m
a
h
SOLUTIONS 87

b
o
o
the initiative of Juan Antonio Samaranch. The Olympic 25. (a) Founded in 1961, Netaji Subhas National Institute of

b
.w
Museum was opened again on 21 December 2013 after Sports commonly known as National Institute of Sports
23 months of renovation. (NIS), is the Academic Wing of the Sports Authority of

o
rd
2. (b) In golf, a caddy or caddie is the person who handles a India (SAI) and Asia’s largest Sports Institute located

p
golf player’s bag and clubs, and gives also some in- in city of Patiala.

re
sightful advice and moral support to him. 26. (b) Karnam Malleshwari is an Indian weightlifter. She is

s
3. (c) The ICC Champions Trophy is a One Day International the first Indian to win an individual medal in Olympics.

s
.c
(ODI) cricket tournament organised by the International 27. (a)

o
Cricket Council (ICC). It is second in importance only 28. (b) The first world cup Hockey was played in Barcelona in

m
to the Cricket World Cup. 1971 whose final winner was Pakistan.
4. (c) 29. (b) The average length of the football field is
5. (a) The four Grand Slam tournaments, also called Majors, 100 – 110m (110 – 120 yards) with width is in the range
are the most important annual tennis events. The Grand of 64 to 75 m (70–80 yd).
Slam itinerary consists of the Australian Open in mid 30. (a) The India national field hockey team had won its first
January, the French Open in May/June, Wimbledon in Gold in 1928 at Amsterdam, Nederlands in which India
June/July, and the US Open in August/September. defeated the Nederlands by 3-0. India also won Gold in
6. (d) The first Youth Olympic Games (YOG), was held in 1932, 1936, 1948, 1952, 1956, 1964, and 1980.
Singapore from 14 to 26 August 2010. The age limit for

7. ww
the games is 14-18. The next Summer Youth Olympics
will be held in Nanjing in 2014.
(d) Subroto Cup Football Tournament is an inter-school 1. (b) 2. (b)
SPEED TEST 93

w.E
football tournament in India, named after the Indian Air
Force Air Marshal Subroto Mukerjee. Subroto Cup is
conducted by the Indian Air Force, with support from
India’s Ministry of Youth Affairs & Sports.
3. (b) With an aim of avoiding a repeat of the 2004 catastrophe,
India is building Tsunami warning device in the South
China Sea, which is likely to operate in the next 10
months.
8.
asy
(a) Wankhede stadium is in Mumbai. It is in this stadium
that India had won the World cup cricket in 2011 against
Sri Lanka.
4. (a) British drug maker GlaxoSmithKline is seeking
regulatory approval for the world’s first malaria vaccine

En
after trial data showed that it had reduced the number
9. (a) A Gambit is a term associated with the game of Chess.
of cases in African children. The vaccine known as
Gambit is a chess opening in which a player sacrifices
RTS, S was found to have almost halved the number of

gin
material, usually a pawn, with the hope of achieving a
resulting advantageous position. malaria cases in young children in the trial and to have
10. (a) The term ‘ashes’ is associated with cricket. reduced by about 25% the number of malaria cases in
infants. GSK is developing RTS,S with non-profit Path
11.

12.
(a) National Sports Day is celebrated on 29th August in the
memory of the great hockey player Major Dhyan Chand.
(c) A bogey is a score of 1-over par on any individual hole eer
Malaria Vaccine Initiative supported by funding from
the Bill & Melinda Gates Foundation.

13.
on a golf course. Golf holes are typically rated as par 3,
par 4 or par 5.
(c) FINA or Fédération Internationale de Natation or Inter-
5.

ing
(b) Kepler’s laws of planetary motion are three scientific
laws describing the motion of planets around the Sun -
The orbit of a planet is an ellipse with the Sun at one of
national Swimming Federation is the International Fed-
eration (IF) recognized by the International Olympic
Committee (IOC) for administering international com- .ne
the two foci, a line segment joining a planet and the Sun
sweeps out equal areas during equal intervals of time and

t
the square of the orbital period of a planet is proportional to
petition in Aquatics. the cube of the semi-major axis of its orbit.
14. (b) 6. (b)
15. (a) The Dronacharya award is presented by Indian Gov- 7. (b) A plant that produces both tomatoes and potatoes,
ernment to people showing excellence in sports coach- called the TomTato, has been developed for the UK
ing. B.I. Fernandez is the first foreign Coach who was market. Ipswich-based horticulture firm Thompson and
awarded by Dronacharya Award in 2012.
Morgan said the hybrid plants were not genetically
16. (b) 17. (b)
modified
18. (d) The Duleep Trophy is a domestic first-class cricket
competition played in India between teams represent- 8. (b) Australia unveiled its most powerful computer Raijin
ing geographical zones of India. The competition is named after the Japanese God of thunder and rain. Raijin
named after Kumar Shri Duleepsinhji. is considered the 27th most powerful computer in the
19. (b) The Marquess of Queensberry rules is a code of gen- world.
erally accepted rules in the sport of boxing. 9. (a)
20. (a) 21. (a) 22. (a) 10. (b) The Vikram Sarabhai Space Centre (VSSC) is a major
23. (a) The Olympic Games were a series of athletic competi- space research centre of the Indian Space Research
tions among representatives of city-states and one of Organisation (ISRO), which focuses on rocket and space
the Panhellenic Games of Ancient Greece. They were vehicles for India's satellite programme. It is located in
held in honor of Zeus, and the Greeks gave them a Thiruvananthapuram, Kerala.
mythological origin. The first Olympics is traditionally 11. (c) The first fertilizers’ plant was established in Sindri which
dated to 776 BC. is an industrial township within the Dhanbad municipal
24. (d) limits of the Dhanbad District of Jharkhand state.

Downloaded From : www.EasyEngineering.net


y
Downloaded From : www.EasyEngineering.net

o
u
rs
m
a
h
88 101 SPEED TEST

b
o
o
12. (a) PARAM is a series of supercomputers designed and 5. (b) The two day meet of 13th Broadband Commission for

b
.w
assembled by the Centre for Development of Advanced Sustainable Development was held in Dubai (UAE). It
Computing (C-DAC) in Pune, India. The latest machine

o
aims to promote the power of broadband networks &

rd
in the series is the PARAM Yuva II. services for the 2030 Agenda for sustainable

p
13. (b) Cinnabar is the common ore of mercury.Generally it Development.

re
occurs as a vein-filling mineral associated with recent 6. (c) World Consumer day is celebrated across the world on

s
s
volcanic activity and alkaline hot springs. 15th March 2016. The theme for 2016 is- antibiotics off

.c
14. (a) 15. (c) the menu.

o
m
16. (a) ETWS installed in Rangachang in Andaman and 7. (d) President Pranab Mukherjee inaugurated the 5th
Nicobar Islands to predict Tsunami within three minutes edition of India Aviation 2016 in Hyderabad which was
of being triggered. jointly organised by the Union Civil Aviation Ministry
17. (c) & the federation of Indian Chambers of Commerce &
18. (c) Hindustan Aeronautics Limited (HAL): Tejas is India’s Industry (FICCI).
indigenously built light combat aircraft. It has come 8. (c) The fourth Nuclear Security Summit (NSS) is being held
from Light Combat Aircraft (LCA) programme. in Washington, D.C. on March 31-April 1, 2016.
19. (a) Telefonica, Spanish broadband & telecommunications 9. (d) Divine speaker and spiritualist Chaganti Koteswara Rao
provider has launched the world's first smartphone with has been appointed Adviser to Andhra Pradesh

20. ww
the Firefox Operating System (OS) to compete with
devices running Google's Android and Apple's IOS.
(b) Indian Space Research Organization (ISRO) has
10.
Government.
(b) Hashim Thaci was sworn-in as the President of Kosovo
in April 2016.

w.E
successfully launched IRNSS-1A on PSLV C 22 from 11. (a) Argentina topped the April 2016 edition of FIFA or
Satish Dhawan Space Centre, Sriharikota in Andhra Coca-Cola World Rankings of Soccer.
Pradesh. 12. (a) Shardul Amarchand Mangaldas advised Future Group
21. (a) on its acquisition of 100% stake in FabFurnish.com.
22.
asy
(a) Global biotechnology company 'Life Technologies' has
launched India's first private DNA forensics laboratory
13. (a) NASA astronaut Scott Kelly will write a memoir titled
Endurance: My Year in Space and Our Journey to Mars,
as announced on 6 April 2016. The book will be

En
in Gurgaon which is expected to accelerate sampling
process thereby reducing the burden on existing published by Alfred A. Knopf.
forensic laboratories. 14. (b)
23. (c) IBM have made the world's smallest movie "A Boy and
His Atom" using atoms. The movie has been verified
by the Guinness World Records gin
15. (b) The World Health Day (WHD) 2016 theme is "Beat
Diabetes" to scale up diabetes prevention, strengthen
care and enhance surveillance.
24. (b) "Nirbhay", First cruise missile of India has a Long-
range (1,000-2,000 km) & strikes targets more than 700
16.

17. eer
(b) The Australian grand prix was won by Nico Rosberg
overtaking Sebastian Vettel & Lewis Hamilton.
(c) Agasthyamala Biosphere Reserve shared by Kerala &
25.
km away carrying nuclear warheads.
(c) Sqn Ldr Rakesh Sharma was a test pilot in the IAF. In
1984 he became the first citizen of India to go into space 18. ing
Tamil Nadu has been placed in the new 20 Biosphere
Reserve list released by UNESCO.
(a) Union Urban Development, Housing & Urban Poverty
when he flew aboard the Soviet rocket Soyuz T-11.
.ne
Alleviation & Parliamentary Affairs Minister M.
Venkaiah Naidu was bestowed with Skoch Lifetime

t
SPEED TEST 94 Achievement Award at 43rd SKOCH Summit held in
New Delhi for his contributions to inclusive growth &
1. (c) 2. (b) 3. (a) 4. (d) towards poverty alleviation in India.
5. (c) 6. (b) 7. (d) 8. (a) 19. (b) Union Health Ministry launched Bedaquiline new anti-
9. (b) 10. (b) 11. (b) 12. (a) TB drug for Drug Resistant TB as part of the Revised
13. (d) 14. (a) 15. (d) 16. (b) National Tuberculosis Control Program on the eve of
17. (c) 18. (d) 19. (b) 20. (a) World TB Day.
21. (b) 22. (b) 23. (c) 20. (a) Veteran Singer Stevie Wonder was honoured with
Global Green Hero Award at the 13th Annual Global
SPEED TEST 95 Green US ceremony.
21. (b) The International Women's Day is celebrated on 8th
1. (c) The Pradhan Mantri Ujjwal Yojana is a scheme to provide March globally. 2016 Theme is "Planet 50-50 by 2030:
LPG to women of all households below poverty line. Step It up for Gender Equality."
2. (b) 22. (c) Luxembourg 23. (a) Bangladesh
3. (b) The British mathematician Andrew J.Wiles won the 2016 24. (a) Justice Permod Kohli was appointed as the new
Abel prize for solving a centuries-old hypothesis- chairman of the Central Administrative Tribunal (CAT).
Fermat's Last theorem. 25. (a) Rajasthan has become the first state to pass land title
4. (a) The World Happiness Report, 2016 released by bill.
sustainable Development Solutions Network, a global 26. (b) Italian super-car brand Lamborghini appointed former
initiative of the United Nations placed India in 118th Audi India head of field forces, Sharad Agarwal, as its
rank in terms of Happiness Index. new Head of India operations.

Downloaded From : www.EasyEngineering.net


y
Downloaded From : www.EasyEngineering.net

o
u
rs
m
a
h
SOLUTIONS 89

b
o
o
27. (b) Khadija Ismayilova became the 2016 recipient of the 27. (b) RM Lodha committee The Lodha committee, which was

b
.w
UNESCO/Guillermo Cano World Press Freedom Prize. primarily assigned with the task of determining the
28. (a) Historian Srinath Raghavan wrote the book titled "India's quantum of punishment for players and others involved

o
rd
War: The Making of Modern South Asia 1939-1945" which in spot fixing in IPL, also gave recommendation on

p
details India's contribution to World War II. change in the structure and ecosystem in the Indian

re
29. (c) Danny Willett has won the 80th edition of US Masters cricket board saying no to politicians' involvement in

s
s
golf tournament 2016 at Augusta National in Georgia, BCCI.

.c
United States. 28. (a) The Shuklaphanta Wildlife Reserve is a protected area

o
m
30. (a) UN commemorated the 125th birth anniversary of B. R. in the Terai of the Far-Western Region, Nepal, covering
Ambedkar on April 13 at the UN headquarters. 305 km2 (118 sq mi) of open grassland, forests, riverbeds
and tropical wetlands at an altitude of 174 to 1,386
SPEED TEST 96 metres. It was gazetted in 1976 as Royal Shuklaphanta
Wildlife Reserve.
1. (b) 2. (d) 3. (d) 4. (e) 29. (d) Diarrhoea caused by Rotavirus is one of the leading
5. (e) 6. (b) 7. (e) 8. (e) causes of severe diarrhoea and death among children
9. (c) 10. (e) 11. (b) 12. (a) less than five years of age in India. Rotavac is the new
13. (c) 14. (d) vacccine launched by Health Ministry to co
15.

ww
(a) Narmada is a west flowing river. It flows westward over
a length of 1,312 km before draining through Gulf of
Cambay into the Arabian Sea.
30. (c) The International Day of Remembrance of the Victims
of Slavery and the Transatlantic Slave Trade is observed
every year on March 25 to offer the opportunity to
16.

w.E
(d) Floating rate bonds have variable interest rate and
protect investors against a rise in interest rates (which
have an inverse relationship with bond prices) . They
honour and remember those who suffered and died at
the hands of the brutal slavery system. The 2016 theme
is "Remember Slavery: Celebrating the Heritage and
Culture of the African Diaspora and its Roots".

asy
also carry lower yields than fixed notes of the same
maturity. 31. (a) Swayam Shikshan Prayog (SSP) aims to promote
17. (c) As per rules, government has to get all money bills empowerment of women as leaders and entrepreneurs

En
related to the union budget passed within 75 days of through self help groups, social enterprises and
the presentation of the budget. community led initiatives.
18. (d) Commodity markets in India are regulated by Forward 32. (a) Bangladesh Export Processing Zones Authority BEPZA

19. (d)
Markets Commission (FMC) headquartered at Mumbai.
20. (a) gin stands for "Bangladesh Export Processing Zones
Authority", which is a government organization, that
includes information of investment opportunities and
21. (c) The National Maritime Day of India is celebrated every
year on April 5. On this day, in 1919 navigation history
was created when SS Loyalty, the first ship of the eer
for investors, proposals, reports, export promotion
zones (EPZ) in Bangladesh.
Scindia Steam Navigation Company, journeyed to the
United Kingdom.
33.

ing
(c) Assam rifles is India's oldest paramilitary force, raised
originally in 1835 as Cachar Levy. They perform many
roles including the provision of internal security under

.ne
22. (b) The Mullaiperiyar Dam is a masonry gravity dam on
thePeriyar River in the Indian state of Kerala. It is located the control of the army through the conduct of counter
insurgency and border security operations. Assam
881 mabove mean sea level, on the Cardamom Hills of

t
Rifles is known as "Sentinels of the Northeast". Its
the Western Ghats in Thekkady, Idukki District of Kerala.
headquarters are located at Laitkor (Shillong). The motto
It has been asubject of controversy between Tamil
of Assam Rifles is "Friends of the Hill People". While
Nadu and Kerala.
Assam Rifles functions under the Ministry of Defence,
23. (c) The Confederation of Indian Industry (CII) is an
its administrative control is under the Ministry of Home
association of Indian businesses which works to create
Affairs.
an environment conducive to the growth of industry in
34. (a) Gurmeet Singh has become the first Indian athlete to
the country. CII is located at New Delhi. clinch a gold medal at the 2016 Asian 20km Race Walk
24. (b) The Maharashtra government has recently tied up with Championships in Nomi, Japan. It is the first time in 34
the Tata Trusts to link all government colleges and years that an Indian has won a gold in either an Asian
medical colleges with he latter's national cancer grid. Championships or Asian Games in this event. Before
With this, Maharashtra has become the India's first him, Hakam Singh and Chand Ram won a gold each in
state to join the initiative. The national cancer grid links 20km road walk in 1978 and 1982 Asian Games
all existing and proposed cancer care centres to create respectively.
a uniform line of treatment for cancer patients across 35. (b) Bank The Indian Overseas Bank (IOB) has become the
India. first bank to commence the sale of Indian Gold Coin
25. (b) Kachhabali village in Bhim tehsil of Rajsamand district (IGC) in the domestic market. The bank has tied up with
has become the first liquor-free village in Rajasthan. Metals and Minerals Trading Corporation (MMTC) for
26. (a) Kakrapara Atomic Power Station (KAPS) in Gujarat was the sale of IGC of 24 carat purity. IGC is the first ever
recently making news, when its one unit was shut down national gold offering by the Centre. The IGC has the
after leakage of heavy water from its coolant system. national emblem, the Ashok Chakra, engraved on one

Downloaded From : www.EasyEngineering.net


y
Downloaded From : www.EasyEngineering.net

o
u
rs
m
a
h
90 101 SPEED TEST

b
o
o
side and the face of Mahatma Gandhi on the other. The is a land mass towards the north of the Black Sea and

b
.w
coin carries advanced anti-counterfeit features and has Ukrainian region of Kherson to the south and
comes in tamper-proof packaging. The current Russian region of Kuban to the west. Crimea and

o
rd
denominations available are 5, 10 and 20 gm. adjacent territories were united in the Crimean Khanate

p
36. (a) India has recently signed a loan agreement of $35 before becoming part of Russia during 1783. Its

re
Million with the World Bank (WB) for "Madhya economy primarily depends on tourism and agriculture,

s
s
Pradesh Citizen Access to Responsive Services but since Russian annexation of Crimea in 2014, there

.c
Project". The purpose of the project is to improve access

o
has been a considerable drop in the number of people

m
and quality of public services in Madhya Pradesh visiting it. With an area of 27,000 square kilometres,
through implementation of the 2010 Public Service Crimea has a population of 2.2 million people. .
Delivery Guarantee Act. The duration of project is 5 45. (a) Infrasound has a frequency lower than 50 hertz
years. Infrasound has a frequency lower than 20 hertz . Though
37. (d) Mossad is the national intelligence agency of Israel. It it cannot be heard by humans, animals such as
is one of the most powerful secret service agencies in elephants, rhinos, alligators and whales produce
the world. It's headquarters is in Tel Aviv, Israel. The
infrasound. One interesting feature of these sound
agency has active agents spread across the world and
waves is that they can travel longer distances than
are involved in intelligence gathering, covert operations
high frequency waves. Avalanches and earthquakes
38. (a)
ww
and protecting Jews and Jewish interests.
The United Nations Educational, Scientific and Cultural
Organization (UNESCO) Institute of Physics has 46. (c)
also produce infrasound.
The ICICI Bank has launched "iWork@home", a first-
of-its-kind programme that allows women employees

w.E
recently released the e-Atlas of Gender Inequality in
Education. The e-Atlas shows gender gaps from primary
to tertiary education and educational pathways of girls
and boys in more than 200 countries and territories.
to work from home for up to a year. The company has
deployed face recognition technology to facilitate
women to work from home and provide access to the
39. (b)
asy
The Songkran festival, also known as the World's
biggest water festival, is celebrated every year in 47. (c)
bank's core banking servers.
The Karnataka government has launched
Mukhyamantri Santwana Harish Yojana for free medical

En
Thailand from 13 to 15 April. Recently, it is in news
because Thailand government has decided to impose treatment to accident victims. As per the scheme, the
a curfew during 2016 Songkran festival to show accident victims will be given free medical treatment

40. (d)
solidarity with farmers hit by drought.
India's first contact-less mobile payment solution "iTap"
has been launched by ICICI Bank. Through "iTap" its gin for the first 48 hours at any hospital and financial aid
upto Rs. 25,000 will also be provided. The scheme has

eer
been named after Harish Nanjappa who had donated
customers will be able to make payments by waving his eyes moments before his death. .
their cell phones near an NFC- enabled merchant 48. (b) The Jalandhar-based Capital Local Area Bank Ltd
terminal. In it, ICICI bank has used the Host Card
Emulation (HCE) technology which creates 'virtual'
cards for 'physical' credit or debit cards (Visa/ ing
(CLABL) will become the India's first small area finance
bank (SFB) by flagging-off its operations on April 13,
2016. The small bank will launch operations under a

41. (b)
MasterCard) of the bank, as selected by the customer.
The Uttar Pradesh has launched awareness campaign
.ne
new name "Capital Small Finance Bank Limited". Apart
from CLABL, the Reserve Bank of India (RBI) has

t
at school levels to create awareness about granted 9 entities in-principle licences to open small
conservation of birds on March 14, 2016. The awareness finance banks to expand access to financial services in
campaign would also be spread among the students to rural and semi-urban areas. These are Ujjivan Financial
make the state clean and green under "CleanUP - Green Services Pvt. Ltd, Janalakshmi Financial Services Pvt.
UP".
Ltd, Au Financiers (India) Ltd, etc.
42. (b) The Union Power Ministry has recently named the LED
49. (b) India's first 75-seater solar powered ferry will come up
based Domestic Efficient Lighting Programme (DELP)
in the favourite tourism spot of Kerala "Allepey (the
as "UJALA". Presently, DELP is running successfully
Venice of the East)".These boats will be commissioned
in over 120 cities across India. UJALA, an acronym for
Unnat Jyoti by Affordable LEDs for All, is being within the next three months and will run on the 2.5 km-
implemented by Energy Efficiency Services Limited long Vaikkom-Thavanakkadavu route. The boat is 20 m
(EESL). The National LED programme was launched long, 7 m wide with a maximum cruising speed of 7.5
by the Prime Minister on January 2015 with a target of knots. The boat is built by a Kochi-based NavAlt, in
replacing 77 crore incandescent lamps with LED bulbs. collaboration with a French firm AltEn, which makes
43. (d) In 1966, MS Subbulakshmi was invited by Secretary- solar powered ferries across the world.
General of the United Nations, U Thant to give a special 50. (a) India's first self cleaning smart toilets have been
concert at the United Nations. This was the first installed in Chennai, Tamil Nadu. Around 180 self-
performance by any Indian classical musician at the cleaning public e-Toilets have been installed and are
UN. . free for public use. Each toilet automatically cleans itself
44. (a) Black Sea and Sea of Azov Crimean Peninsula is before entry and after use; and sensors in it enable
surrounded by two seas - Black Sea and Sea of Azov. It consumption of less water and power with each use.

Downloaded From : www.EasyEngineering.net


y
Downloaded From : www.EasyEngineering.net

o
u
rs
m
a
h
SOLUTIONS 91

b
o
o
SPEED TEST 97 : PRELIM TEST - 1

b
.w
o
Answer Key

rd
p
1 (d) 16 (b) 31 (c) 46 (c) 61 (d) 76 (b) 91 (a)

re
2 (a) 17 (c) 32 (b) 47 (b) 62 (a) 77 (c) 92 (e)

s
s
.c
3 (c) 18 (d) 33 (e) 48 (e) 63 (c) 78 (a) 93 (b)

o
4 (a) 19 (e) 34 (d) 49 (a) 64 (e) 79 (a) 94 (e)

m
5 (b) 20 (e) 35 (a) 50 (d) 65 (b) 80 (e) 95 (c)
6 (e) 21 (a) 36 (c) 51 (a) 66 (c) 81 (a) 96 (b)
7 (d) 22 (b) 37 (c) 52 (d) 67 (b) 82 (e) 97 (d)
8 (c) 23 (c) 38 (a) 53 (e) 68 (c) 83 (b) 98 (c)
9 (a) 24 (c) 39 (d) 54 (c) 69 (a) 84 (c) 99 (d)
10 (e) 25 (b) 40 (b) 55 (b) 70 (c) 85 (d) 100 (a)
11 (b) 26 (b) 41 (e) 56 (a) 71 (d) 86 (e)

ww 12
13
14
(d)
(e)
(b)
27
28
29
(b)
(a)
(e)
42
43
44
(d)
(b)
(d)
57
58
59
(d)
(c)
(a)
72
73
74
(d)
(c)
(e)
87
88
89
(a)
(c)
(b)

w.E
15 (c) 30 (d) 45 (e) 60 (b) 75 (b) 90 (d)

Hints & Solutions


asy
En 9. (a) ? = 78.45 + 128.85 + 1122.25 = 1392 . 55
1. (d) ? 848
11
16
4 5
5 11
212
gin
10. (e) 5598 = ? + 2785
? = 5598 – 2785 = 2813

2. (a) ?
150 1.4
100
480 2.2
100 11. (b)
87
5 eer 37
8
?
375
8
= 10.50 + 10.56 = 21.06
116 3 87 2 ? ing3219
40
375
8

.ne
3. (c) ?
4 3
= 87 – 58 = 29 3219 1875 1344

4. (a) ?
6.96 18.24
1.2 7.6
= 5.8 – 2.4 = 3.4
168
5
40

33
3
5
40

t
5. (b) ? = 32.25 × 2.4 × 1.6 = 123.84 5616
12. (d) ? 39
250 136 550 ? 18 8
6. (e) 670
100 100 420 288
13. (e) ? 135
340 + 5.5 × ? = 670 28 32
5.5 × ? = 670 – 340 = 330 14. (b) 484 ? 516
330
? 60 ? 516 484 32
5.5
? 32 32 1024
448
7. (d) ? 35 980
16 660 45 450 28
15. (c) ?
100 100
14 25 125 225 75 11
8. (c) ? = 1 = 297 + 126 = 423
120 72 192 64 64
16. (b) The pattern of the number series is:

Downloaded From : www.EasyEngineering.net


y
Downloaded From : www.EasyEngineering.net

o
u
rs
m
a
h
92 101 SPEED TEST

b
o
o
12 + 22 = 16

b
11

.w
16 + 23 = 24 0.85
13

o
24 + 24 = 40

rd
40 + 25 = 72 7 4 9 5 11

p
Clearly,

re
17. (c) The patern of the number series is : 9 5 11 6 13

s
25. (b) Ratio of the profit of Srikant and Vividh

s
9 + 10 = 19

.c
= 185000 : 225000 = 37 : 45

o
19 + 20 = 39

m
39 + 40 = 79 Sum of the ratios = 37 + 45 = 82
Total profit earned
79 + 80 = 159
18. (d) The pattern of the number series is: 82
= 9000 = ` 16400
8 + 32 = 17 45
17 + 52 = 42 26. (b) Father’s present age = 6x years
42 + 72 = 91 Son's present age = x years
After four years
91 + 92 = 172
19.
ww
(e) The pattern of the number series is:
7× 1+1=8
8 × 2 + 2 = 18
6x 4 4
x 4 1
6x + 4 = 4x + 16

w.E
18 × 3 + 3 = 57
57 × 4 + 4 = 232 2x = 12
12
x
2
6
20. (e) The pattern of the number series is:
3840 4 = 960
960 4 = 240 asy 27.
Son’s present age = 6 years
(b) Let the number be x.

240 4 = 60
En x × 65
100
2x
5
140
13 x
20
2x
5
140

gin
60 4 = 15
13 x 8 x x
x 75 3 3x 3 140 140
21. (a) y y 20 4
100

x 3
7

4 4
4 7

eer
x = 4 × 140 = 560
560 30

22.
y 7
(b) Speed of the train
3 7
28. ing
30% of 560
100
168

(a) Let the original number be 10x + y where y > x.


Length of train and platform
= Time taken to cross each other
10y + x – 10x – y = 27
9(y – x) = 27
.ne
275 275
33
m/sec.
550 18
33 5
kmph = 60 kmph
y–x=3
and x + y = 13
....(i)
....(ii)
From equations (i) and (ii),
y = 8 and x = 5
t
Time
Rate Original number = 58
23. (c) CI P 1
100
1
29. (e) M1D1 = M2D2
22 × 16 = 32 × D2
2
9 22 16
45000 1 1 = 45000 × [(1.09)2 –1] D2 = = 11 days
100 32
30. (d) Let the smallest odd number A be x
= 45000 × 0.1881 = ` 8464.5
x + x + 4 = 2 × 59
9 7 2x = 118 – 4 = 114
24. (c) 0.82; 0.78
11 9 114
x 57
5 4 2
0.83; 0.8 31. (c) 60% of 150 = 90. It means those who obtained either
6 5
90 or more than 90 marks in the average of five
subjects will be declared as passed. Therefore,
the required number = 31 + 17 = 48

Downloaded From : www.EasyEngineering.net


y
Downloaded From : www.EasyEngineering.net

o
u
rs
m
a
h
SOLUTIONS 93

b
o
o
32. (b) Reqd no. = 52 + 41 + 13 = 106 Conclusions I and II form Complementary Pair.

b
.w
35. (a) No. of students who obtained more than or equal to Therefore, either I or II follows.

o
40% marks in Science 47. (b) All curtains are rods.

rd
= 67 + 22 + 14 = 103

p
re
No. of students who obtained less than 60% marks in

s
Hindi = 19 + 59 + 47 = 125

s
.c
o
125 103

m
Reqd % 100 17.60% Some rods are sheets.
125
(A + I No Conclusion)
36. (c) Meaningful words : ARE, EAR, ERA
48. (e) Some plugs are bulbs
37. (c) A D J E C T I V E
1 1 1 1 1 1 1
B C I F B S J U F
38. (a) na pa ka so birds fly very high All bulbs are sockets.

ww
ri so la pa
ti me ka bo
birds are very beautiful
the parrots could fly
(I + A I-type)
“Some plugs are sockets”.
Conclusion I is Converse of thi3s Conclusion.

w.E
Thus high is coded as na.
39. (d) 1 2 3 4 5 6 7 8 9 Conclusion II is Converse of the first Premise.
Difference = 8 – 4 = 4 49. (a) All fishes are birds. (conversion)
40. (b) 1 2 3 4 5 6 7 8 9 10 11 12 13
CO M PA T I B I L I T Y
asy
Meaningful word

2
LI M B

I
2
G En All birds are rats.

gin
41. (e) As F H (A + A A-type)
2 2 “All fishes are rats”.
N P E C
All birds are rats. (conversion)
Similarly,

S
2
U L
2
J eer
I
2
K T
2
R
ing
All rats are cows.
42. (d) As L
A
8
&
and H
I
7
*
(A + A A-type)
“All birds are cows”. .ne
Similarly,
T
E
4
$
R
E
3
$ 50.
This is Conclusion I.
(d) Some windows are doors. t
H 7 A &
I * L 8 All doors are roofs.
43. (b) Others relate to ‘parts of tree’. (I + A I-type)
44. (d) Since ‘potato’ is called Banana. Thus, ‘Banana’ grows “Some windows are roots”.
underground. 51. (a) P + S P is daughter of S.
45. (e) EI, EG, GI and NL. S – T S is father of T.
46. (c) Some suns are planets. Therefore, P is sister of T.
52. (d) P × Q P is wife of Q.
Q – T Q is father of T.
T is child of P and Q.
All planets are satellites. The sex of T is not known.
(I + A I-type) T is either son or daughter of P.
“Some suns are satellites”.

Downloaded From : www.EasyEngineering.net


y
Downloaded From : www.EasyEngineering.net

o
u
rs
m
a
h
94 101 SPEED TEST

b
o
o
53. (e) P × S P is wife of S. 60. (b) E £; G $; A 2; K 1; R *; L 3

b
.w
S ÷ T S is son of T. Condition (ii) is applied.

o
T is either father-in-law or mother-in-law of P. For (Qs. 61-65): Given information can be tabulated as follows

rd
P ÷ S P is son of S.

p
re
S × T S is daughter of T

s
Person Sex Company Specialisation

s
Therefore, T is father of P.

.c
A Male X Finance

o
P – S P is father of T. B Male Z Marketing

m
P + T P is daughter of T C Male Y Engineer
T ÷ S T is son of S. D Female X HR
Therefore, T is father of P. E Male Y Doctor
54. (c) P + Q P is daughter of Q. F Male Y Marketing
Q – S Q is father of S. G Female Z Finance
H Male Z HR
Therefore, P is sister of S.
P ÷ Q P is son of Q.

ww
Q × S Q is wife of S. Thus, ‘G’ is a sister of ‘C’.
Therefore, P is son of S. 66. (c) The number 49 is a perfect square of a natural number.
P ÷ Q P is son of Q.

w.E
Q + S Q is daughter of S.
Therefore, P is grandson of S.
67. (b) 1
P
3
H
3 4 5 6 7 8
Y S 1 C A L

55. (b) P + Q P is daughter of Q.


Q × T Q is wife of P.
Therefore, T is father of P.
asy 68.
Meaningful word HAIL
(c) Obviously option (c) may be the conclusion of the
passage. Because the passage also talks about the
56. (a) B 9; A 2; R *;N %; I #; S 4
En 69.
literal the actual sense of culture.
(a) Option (a) have the same summary hense it strengthens

gin
57. (d) D 2; M @; B 9; N %; I #; A 6
Condition (i) is applied. the conclusion of the passage.
58. (c) I $; J 8; B 9; R *; L £; G # 70. (c) Option (c) has the opposite sense of the passage. Thus

59.
Condition (ii) is applied.
(a) B 9; K ©; G $; Q 7; J ©; N % eer
it weakens the conclusion of the passage.

Condition (iii) is applied.


ing
.ne
t

Downloaded From : www.EasyEngineering.net


y
Downloaded From : www.EasyEngineering.net

o
u
rs
m
a
h
SOLUTIONS 95

b
o
o
SPEED TEST 98 : PRELIM TEST - 2

b
.w
o
Answe r Key

rd
p
1 (d) 16 (b) 31 (b) 46 (b) 61 (c) 76 (b) 91 (d)

re
2 (c) 17 (b) 32 (e) 47 (e) 62 (b) 77 (c) 92 (c)

s
s
.c
3 (a) 18 (c) 33 (a) 48 (c) 63 (a) 78 (d) 93 (a)

o
4 (a) 19 (b) 34 49 (d) 64 (d) 79 (a) 94 (a)

m
(c)
5 (e) 20 (b) 35 (b) 50 (b) 65 (e) 80 (d) 95 (b)
6 (b) 21 (c) 36 (a) 51 (a) 66 (b) 81 (e) 96 (c)
7 (d) 22 (a) 37 (c) 52 (e) 67 (c) 82 (a) 97 (b)
8 (c) 23 (b) 38 (b) 53 (a) 68 (b) 83 (c) 98 (b)
9 (b) 24 (a) 39 (b) 54 (b) 69 (c) 84 (e) 99 (a)
10 (e) 25 (d) 40 (e) 55 (a) 70 (e) 85 (d) 100 (d)

ww
11 (d) 26 (d) 41 (d) 56 (d) 71 (b) 86 (b)
12 (b) 27 (a) 42 (e) 57 (b) 72 (c) 87 (e)
13 (a) 28 (a) 43 (a) 58 (c) 73 (d) 88 (a)
14
15 w.E
(d)
(a)
29
30
(e)
(c)
44
45
(c)
(d)
59
60
(a)
(e)
74
75
(e)
(e)
89
90
(d)
(c)

asy
Hints & Solutions
En Thus 1001 is the largest number which divides the

gin
3 4 5 3 4 5 1015
1. (d) of of of 1015 = 1015 145 numbers of the type 25025, 67067 etc.
5 7 12 5 7 12 7
12. (b) A three digit number to be exactly divisible by 5 must

eer
have either 0 or 5 at its units place.
2. (c) 1.5 0.025 (?) 2 0.1 (?) 2 0.1 1.5 0.025
Such numbers will be 100, 105, 110, ..........., 995.
(?) 2 0.1 0.0375 ? .0625 0.25 First term = 100, last term = 995

3. (a) 1.5 2 0.0375 2.25 0.15 0.3375 ing


Let the required number be n.
To find the value of n, we may use the following formula

.ne
5. (e) 125% of 260 + ?% of 700 = 500 of arithmetic progression,
Tn = a + (n – 1) d.......... (1)
?% of 700 = 500 – 125% of 260
Where d = common difference = 5
?% of 700 = 175

?
175 100
700
25
Tn = 995
a = 100
Hence from (1)
995 = 100 + (n – 1) 5 5n = 900
t
6. (b) 45% of 750 – 25% of 480 n = 180
45 750 25 480 Digits to be used = 0, 1, 2, 3, 4, 5, 6, 7, 8, 9.
= 337.5 120 217.5 13. (a) Let the principal be P, then amount after 12 years = 2P
100 100
SI (2P P ) P
7. (d) 758.5 753.8 75(8.5 3.8) 754.7
P r t P r 12
8. (c) 39798 + 3798 + 378 = 43974 Now, I P=
100 100
7 3 1 40 80 3 160 6 100 25 1
9. (b) 3 7 1 = 14 or r 8 %
11 11 2 11 11 2 11 11 12 3 3
14. (d) Applying successive discounts of 10%, 12% and 15%
1080
10. (e) 1080 12 10 9 on 100, we get 100 0.9 0.88 0.85 67.32
12 10
Single discount = 100 – 67.32=32.68
11. (d) First start with the option (d). 15. (a) Let the prices of two houses A and B be Rs 4x and
1001× 25 = 25025 Rs 5x, respectively for the last year.
1001× 67 = 67067 etc.

Downloaded From : www.EasyEngineering.net


y
Downloaded From : www.EasyEngineering.net

o
u
rs
m
a
h
96 101 SPEED TEST

b
o
o
Then, the prices of A this year = `(1.25 × 4x) and that of

b
103

.w
B = ` (5x + 50,000) 10 10609

o
This year, Ratio of their prices = 9 : 10 100

rd
203 609
1.25 4x 9

p
609

re
5x 50,000 10 0

s
s
5x = 4,50,000

.c
50x 45x 450000 Also, 7938.81 89.1 , by long division method, as

o
x = 90,000 below:

m
Hence, the price of A last year was 89.1
4x = `3,60,000 8 79388.81
Totaldistance 64
16. (b) Average speed = 169 1538
Total time 1521
1781 1781
80 80 1781
= = = 32 km/h
60 20 2.5 0
40 20 Hence 103 × 89.1 = 9177.3 9200
17.
ww
(b) Let the usual speed of the aeroplane be x km/h.

Then, 1500 1 1500


23. (b) 18.4% of 656 + 12.7% of 864 = 0.184 × 656 + 0.127 × 864
= 120.704 + 109.728 = 230.432 231
(a) (98.4)2 + (33.6)2 = 9682.56 + 1128.96 = 10811.52 10812

w.E
24.
x 2 (x 250) 25. (d) Let there be x in place of question mark
Solving, we get x = 750 km/h
18. (c) Sum of 40 instalments = S40 = 3600 = 20 (2a + 39d) 8959 8959
So,

asy
26.35 x 17
or 2a + 39d = 180 ...(i) x 4 5 26.35 20
Sum of 30 instalments = S30 = 2400 = 15 (2a + 29d) 26. (d) ? = 3739 + 164 × 27 = 3739 + 4428 8200

En
or 2a + 29d = 160 ...(ii) 27. (a) Required distance covered = 2(6 × 415 + 500) metre
From (i) and (ii), we get a = 51 and d = 2 = 2(2490 + 500) metre = 5980 metre = 5.98 km
The value of first instalment = ` 51 28. (a) Total marks obtained by Amit
19. (b) Volume of the earth taken out = 30 × 20 × 12 = 7200 m3
gin = 44 + 55 + 77 + 79 + 76 = 331
331
470 m
30 m
Field
29. eer
Required percentage =
500
× 100 = 66.2
(e) The pattern of the number series is:

30 m ing
7 × 2 – 3 = 11
11 × 2 – 3 = 19

20 m
19 × 2 – 3 = 35
35 × 2 – 3 = 67
.ne
470 m
Tank
30 m

Area of the remaining portion (leaving the area of dug


30.
67 × 2 – 3 = 134 – 3 = 131
(c) The pattern of the number series is:
5 + 12 = 6
6 + 22 = 10
t
out portion) 10 + 32 = 19
= 470 × 30 + 30 × 10 = 14100 + 300 = 14400 m3 19 + 42 = 35
Let h be the height to which the field is raised when the 35 + 52 = 35 + 25 = 60
earth dug out is spread. 31. (b) No. of students (of JNU) listening to Radio City
Then, 14400 × h = 7200
h = 0.5 m 20 65
120000 15600
100 100
198 194 185 169 144
32. (e) The no. of Indraprastha students listening to FM
20. (b)
2 2 2 2 Rainbow = 120000 × 13% × 48%
–2 –3 –4 –5
The no. of Jamia students listening to
21. (c) The first, third, fifth .... and second, fourth .... terms are
FM Gold = 120000 × 18% × 52%
groups of consecutive natural numbers.
Reqd. percentage
22. (a) 10609 7938.81 120000 13% 48%
100 66.66%
10609 103, by long division method, as below: 120000 18% 52%

Downloaded From : www.EasyEngineering.net


y
Downloaded From : www.EasyEngineering.net

o
u
rs
m
a
h
SOLUTIONS 97

b
o
o
33. (a) Indraprastha University 43. (a) G ©

b
.w
34. (c) The no. of Indraprastha students listening to
Q *

o
13 46

rd
Red FM 120000 12 598 R 1

p
100 100

re
D 7
The no. of Jamia students listening to Red FM

s
s
B 3

.c
18 36
120000 12 648

o
100 100 N ©

m
Condition III is applied.
Total students = (12 × 598) + (12 × 648)
= 12 × 1246 = 14952 44. (c) I £
35. (b) Radio City P 5
36. (a) Since Badminton is not played by using a ball and also
E $
called Volleyball. So, Volleyball is required answer.
B 3
37. (c) T E C H N O L O G Y Q *
1 2 3 4 5 6 7 8 9 10 S £

38. ww Word formed CE N T


(b) Arranging the number in ascending order, we get
45.
Condition I is applied.
(d) R #

w.E
M @
1 23 45 78 9
4th number (Right) = 5 A 9
3rd number (left) = 3 Difference = 5 – 3 = 2 P 5
39. (b) Ja

mo la
Ki mo pe

Ki so
at a frog's
asy
leap

take a leap ahead


S
I
£
1
Condition II is applied.

re bo Ja na insects are frog's diet


En 46. (b) A 8

40.
Thus, at is coded as pe.
(e) As W 9 and S # gin R
E
1
$

E % K
I
$
7
M
Q eer *
@

ing
A 2
K $ T @ 8
N
Condition I is applied.
Thus, W
A
9
2
47. (e) K ©
.ne
41.
I
T
(d) DG, IG and SN
7
@
J
B
P
2
3
5
t
42. (e) 1
G F R 1
1 D ©
L K
1 Condition III is applied.
A B 48. (c) P × R P is son of R.
1 R – S R is daughter of S.
D C
Therefore, P is grandson of S.
1 49. (d) P + R P is wife of R.
I J
R × S R is son of S.
1
O P Therefore, P is daughter-in-law of S.
1 P R P is father of R.
L K R × S R is son of S.
1 Therefore, S is wife of P.
U V
P + R P is wife of R.
1 R S R is father of S.
S R
Therefore, S is child of P.

Downloaded From : www.EasyEngineering.net


y
Downloaded From : www.EasyEngineering.net

o
u
rs
m
a
h
98 101 SPEED TEST

b
o
o
P R P is father of R. 60. (e) All leaves are roots. (conversion)

b
.w
R S R is wife of S.

o
Therefore, S is son-in-law of P.

rd
50. (b) P – Q P is daughter of Q.

p
Q S Q is wife of S.

re
All roots are flowers.

s
Therefore, S is father of P.

s
A+A A-tpye of conclusion

.c
51. (a) P × Q P is son of Q.

o
Q S Q is father of S. “All leaves are flowers”.

m
Therefore, P is brother of S. Conclusion II is Converse of this Conclusion.
52. (e) P R P is wife of R. All stems are roots. (conversion)
R S R is father of S.
The sex of S is not clear.
S is either son or daughter of P.
53. (a) Lowest number = 189 + 1 = 190
All roots are flowers.
Highest number = 972 + 2 = 974
A+A A-type of conclusion
difference = 9 – 4 = 5
54. (b)
ww
Interchanging the digits of given number we get
279, 286, 981, 892 and 157.
3rd highest number = 286 or 682
“All stems are flowers”.
Conclusion I is Converse of this Conclusion.
For (Qs. 61-67) : The given information can be tabulated as

55. (a)
w.E
Arranging all three digits of given number in
descending order, we get
972, 862, 981, 982 and 751.
follows:

Pers on S ex Relations hip Profes sion

56. (d)
3rd highest number = 972
asy
No pen is a mobile. (conversion)
L Female W ife of M
Father of P.
A rchitect
Business man

En M M ale Husband of L.
Son of Q and O.
Some mobiles are bottles.
E+I O-type of conclusion gin N
Female Daughter-in-law of
O and Q.
Hous ewife
“Some bottles are not pens”.
Some mobiles are bottles. (conversion)
O
P eer
Female W ife of Q
M ale Son of M and L.
Pilot
Journalis t

ing Grandfather of P. Doctor

.ne
All bottles are papers. Q M ale Husband of Q.
I+A I-tpye of conclusion Father of M .

57.
“Some mobiles are papers”.
(b) All computers are radios. (conversion)

66.
R – –

Two married couples : LM and QO.


(b) According to question
9146756083
A dvocate

t
All radios are televisions. 67. (c) ab, abc, abcd, abcde, abcdef, abcd e
A+A A-type of conclusion
68. (b) Clearly option (b) is best support by the passage.
“All computers are televisions”. 69. (c) The passage provides the facts about the preparation
Conclusion II is Converse of this Conclusion. of history. Hence option (c) is the assumption which
58. (c) Conclusion I and II from Complementary Pair. Therefore, can be implicit in the facts of the passage.
either Conclusion I or II follows. 70. (e) Obviously. None of the given option is conclusion
59. (a) Some fishes are moons. (conversion)
which can be drawn from the facts of the passage.
71. (b) Refer to the Ist sentence of para 1 ‘Govind’s father
........................ left large tract of land to Govind’.
All moons are birds. 72. (c) Refer to the 3rd sentence of para 2 “To his
I+A I-type of conclusion surprise.................into gold”.
“Some fishes are birds”. 73. (d) Refer to the last two sentences of para 1 of the passage.
Conclusion I is Converse of this Conclusion. 74 (e)

Downloaded From : www.EasyEngineering.net


y
Downloaded From : www.EasyEngineering.net

o
u
rs
m
a
h
SOLUTIONS 99

b
o
o
75. (e) Refer to the sixth sentence of para 2 of the passage 79. (a) The word dismay (noun) means feeling of fear and

b
.w
‘You have..................during winter’. discouragement. For instance,

o
76. (b) Refer to the third last sentence of para 3 of the passage. They were struck with dismay at the news.

rd
77. (c) The word spend (verb) means to pay out, to employ While, the word joy (noun) means feeling of happiness.

p
(labour, time etc.) on some objects. For instance, Thus, both are antonyms.

re
s
How do you spend your leisure-time? 80. (d) The word tend (verb) means look after. For instance,

s
.c
Similarly, the word devote (verb) means to give time and The nurse tended the injured persons.

o
energy to something. For instance, While, the word ignore (verb) means take no notice of.

m
He is devoted to his studies. For instance,
Thus, spend and devote are synonymous. Raman ignored the advice of his elders.
78. (d) The word lie (verb) means to be at rest. For instance, Thus, both are antonyms.
She is lying on the bed. 86. (b) ‘Sent’ is the third form of verb ‘send’ in passive voice.
Similarly, the word remain (verb) means continue to 88. (a) Replace ‘though’ by ‘As’ to express cause and effect
stay. For instance, in the sentence.
Twenty pens were distributed and the remaining were 89. (d) It should be ‘and I answered all of them’ to properly
returned. express cause and effect in the sentence.

ww
Thus, lying and remaining are synonymous. 90. (c) Apply the adverb ‘highly’ before the adjective ‘paid’.

w.E
asy
En
gin
eer
ing
.ne
t

Downloaded From : www.EasyEngineering.net


y
Downloaded From : www.EasyEngineering.net

o
u
rs
m
a
h
100 101 SPEED TEST

b
o
o
SPEED TEST 99 : PRELIM TEST - 3

b
.w
o
Answer Ke y

rd
p
1 (e) 16 (b) 31 (c) 46 (c) 61 (a) 76 (a) 91 (e)

re
2 (d) 17 (b) 32 (a) 47 (e) 62 (b) 77 (e) 92 (b)

s
s
.c
3 (c) 18 (d) 33 (e) 48 (a) 63 (b) 78 (b) 93 (a)

o
4 (d) 19 (d) 34 (e) 49 (b) 64 (a) 79 (d) 94 (b)

m
5 (a) 20 (e) 35 (a) 50 (c) 65 (b) 80 (c) 95 (d)
6 (d) 21 (c) 36 (c) 51 (d) 66 (d) 81 (c) 96 (e)
7 (b) 22 (e) 37 (c) 52 (c) 67 (b) 82 (e) 97 (c)
8 (a) 23 (b) 38 (b) 53 (a) 68 (c) 83 (a) 98 (c)
9 (e) 24 (d) 39 (b) 54 (d) 69 (c) 84 (c) 99 (c)
10 (d) 25 (a) 40 (e) 55 (b) 70 (d) 85 (e) 100 (d)

ww
11 (d) 26 (c) 41 (b) 56 (a) 71 (e) 86 (d)
12 (b) 27 (b) 42 (c) 57 (b) 72 (d) 87 (a)
13 (b) 28 (b) 43 (b) 58 (c) 73 (c) 88 (b)
14
15 w.E
(d)
(b)
29
30
(b)
(d)
44
45
(d)
(a)
59
60
(b)
(a)
74
75
(a)
(e)
89
90
(d)
(a)

asy
Hints & Solutions
En
1. (e) ? 75
gin (d) Clearly, 1 1 7

eer
Squaring on both the sides, we get 11. 7
? = 75 × 75 = 5625 7 7 7

2. (d)
21 7 1
8 72 171
? 1
7 ing
is the smallest number..

or ? =
21 7 1 3
8 72 171 19
12. (b) Let the sum be Rs x

x 8 (3 2) .ne
3. (c) ? 4
1
2
6
2
3
5
1
3

3 4 2 9 1
13.
Now, 56
100
x = `700

(b) Let the cost price of the machine be `x. t


11x
(4 6 5) 15 16 Then, selling price at a profit of 10% = `
6 6 2 10
4. (d) ? = 792.02 + 101.32 – 306.76 = 586.58 9x
5. (a) 300% of 150 = ? % of 600 And the selling price at a loss of 10% = `
10
or , ? of 600 = 45000 or, ? = 75
6. (d) 34.95 + 240.016 + 23.9800 = 298.946 Consequently, we find that
7. (b) 48.95 – 32.006 = ? 11x 9x
or, ? = 48.95 – 32.006 = 16.944 80
10 10
8. (a) 3889 + 12.952 – ? = 3854.002
or ? = 3889 + 12.952 – 3854.002 = 47.95 x
80 x `400
9. (e) ? + 72.64 = 74.64 or ? = 74.64 – 72.64 = 2.00 5
14. (d) Let the capacity of the jar be of x bottles.
10. (d) 6.25 0.0025 = ? or ? 6.25 2500 since 6 bottles were taken out from jar and 4 bottles of
0.0025
oil poured into it 2 bottles were taken out

Downloaded From : www.EasyEngineering.net


y
Downloaded From : www.EasyEngineering.net

o
u
rs
m
a
h
SOLUTIONS 101

b
o
o
Therefore, we have

b
22. (e) 963.7 950.9989 95?

.w
4 3

o
x 2 x or 95? 953.7 0.9989

rd
5 4

p
or 95? 9527 or , ? = 2.70

re
4 3

s
x x 2 x = 40 3.001

s
5 4

.c
23. (b) 1000 of 1891.992 = ?
4.987

o
m
Total distance 3
15. (b) Average speed = or ? = 100 of 1900 = 100 + 1140 = 1230
Total time 5
24. (d) 12.25 × ? × 21.6 = 3545.64
80 80
= = 32 km/h 3545.64
60 20 2.5 ?= 13.4 13
264.6
40 20
3
16. (b) Let the usual speed of the aeroplane be x km/h. 25. (a) ? = 4096 = 3 16 16 16 16

ww
26. (c) M1D1 = M2D2
1500 1 1500
Then, 6 × 20 = 8 × D2
x 2 (x 250)
6 20
17.
w.E
Solving, we get x = 750 km/h
(b) Using Venn Diagram
363
D2 =

?
8
= 15 days

35-15 15 42-15
=20 =27
asy 27. (b)
? 3
?2 = 363 × 3 = 121 × 32 = 112 × 32

En 28.
? = 11 × 3 = 33
(b) Raju's age at the time of marriage
Failed in only the Failed
first subject in both
the subjects
Failed in only the
second subject
gin
29.
= 17 + 3 + 2 + 1 + 3 = 26 years
(b) Required number

Thus, percentage of students who passed in both 30.


eer
= 2040 + 2300 + 2400 + 2200 + 2090 + 2120 = 13150
(d) Required percentage growth

ing
subjects
2250 2180
= 100 – [(35 – 15) + (42 – 15) + 15] = 100 – (35 + 42 – 15) = 100 3.21%
2180
= 100 – (62) = 38%
and percentage of students who failed in both subject
= 15%
31. (c) Required %
2540 .ne
Therefore, the percentage of students who passed in
either subject = 100 – (38 + 15) = 100 – 53 = 47%
Hence, required no. of students who passed in either
=

=
2500 2040 2100 2280 2540 2320

254000
13780
% 18.43% 18%
100

t
47
subject but not in both = 2500 × = 1175 32. (a) Required ratio = (2250 + 2480) : (2260 + 2440)
100
18. (d) Let the length and breadth of the rectangle be x and = 4730 : 4700 = 473 : 470
y cm, respectively. 33. (e) Required average number
Then, (x – 4) (y + 3) = xy 3x – 4y = 12 ... (i)
Also, (x – 4) = (y + 3) [sides of square] 2500 2250 2450 2150 2020 2300
=
x–y=7 ... (ii) 6
From (i) and (ii),
13670
x = 16 and y = 9 = 2278.33 2278
Perimeter of the original rectangle = 2(x + y) = 50 cm 6
19. (d) The series is × 1 + 1 × 7, × 2 + 2 × 6, × 3 + 3 × 5... 34. (e) 5x + 6x + 7x = 180°
20. (e) The series is × 1 + 12, × 2 + 22, × 3 + 32.., ... 18x = 180°
21. (c) The series is × 1 + 2, × 2 + 3, × 3 + 4, ...

Downloaded From : www.EasyEngineering.net


y
Downloaded From : www.EasyEngineering.net

o
u
rs
m
a
h
102 101 SPEED TEST

b
o
o
II. False. From both figures it is clear that either I or III

b
180

.w
x= 10 follows.
18

o
rd
Sum of the smallest and the largest angles

p
re
= 12x = 12 × 10 = 120°
er

s
mm

s
35. (a) 232 = 529 Books

.c
45. (a) Chair Ha

o
242 = 576 Flowers

m
Required number = 536 – 529 = 7

Fl
ow
Mats er
s
or Books
r
Chair
mme
43.

ww
(b) Desks

Tables Chair
Ha

w.E I. False II. False III. False

asy 46. (c)


Cameras Photo

En
Stores
Chair
Desks
Roofs
or

Mats
Tab
les
gin
eer eras
I. False II. True III. False
ing
Ca m Pho
to

or
Roofs

.ne
44. (d)
Sweets
Pencil
I. False
Store

II. False
t
III. True

Fruits Glasses Crows


Tablets

47. (e) Nails Horses

Sweets
or sses
Gla Pencil I. True II. True III. True
63. (b) Since the passage talks about the defense system of
Fruits
our country and role of the government option (b) may
be a conclusion of the passage.

Downloaded From : www.EasyEngineering.net


y
Downloaded From : www.EasyEngineering.net

o
u
rs
m
a
h
SOLUTIONS 103

b
o
o
64. (a) Obviously option (a) strengthens the conclusion of

b
5 8 6 9 7 2

.w
the passage. 69. (c)

o
65. (b) Option (b) has the opposite sense. Thus it weakens 9 8 7 6 5 2

rd
the conclusion of the passage.

p
70. (d) 9 8 7 6 5 4 3 2 1

re
66. (d) Except sugar, all others are liquid.

s
8 7 6 5 4 3 2 1

s
67. (b) Meaningful Word DINE

.c
7 6 5 4 3 2 1

o
68. (c) Except 75, all other numbers are perfect cubes.

m
6 5 4 3 2 1
343 = 7 × 7 × 7
64 = 4 × 4 × 4
27 = 3 × 3 × 3
216 = 6 × 6 × 6

ww
w.E
asy
En
gin
eer
ing
.ne
t

Downloaded From : www.EasyEngineering.net


y
Downloaded From : www.EasyEngineering.net

o
u
rs
m
a
h
104 101 SPEED TEST

b
o
o
SPEED TEST 100 : FULL MAIN TEST - 4

b
.w
Answer Key

o
rd
1 (d) 26 (b) 51 (e) 76 (e) 101 (b) 126 (c) 151 (d) 176 (e)

p
2 (b) 27 (a) 52 (a) 77 (d) 102 (a) 127 (a) 152 (a) 177 (b)

re
s
3 (d) 28 (b) 53 (e) 78 (a) 103 (a) 128 (d) 153 (c) 178 (a)

s
.c
4 (e) 29 (b) 54 (d) 79 (c) 104 (e) 129 (d) 154 (e) 179 (e)

o
m
5 (c) 30 (c) 55 (d) 80 (b) 105 (a) 130 (c) 155 (a) 180 (c)
6 (d) 31 (d) 56 (e) 81 (c) 106 (d) 131 (c) 156 (e) 181 (b)
7 (e) 32 (b) 57 (b) 82 (a) 107 (a) 132 (a) 157 (b) 182 (d)
8 (c) 33 (a) 58 (d) 83 (b) 108 (e) 133 (e) 158 (d) 183 (a)
9 (a) 34 (d) 59 (e) 84 (d) 109 (e) 134 (c) 159 (c) 184 (d)
10 (e) 35 (b) 60 (a) 85 (e) 110 (c) 135 (b) 160 (e) 185 (c)
11 (d) 36 (c) 61 (e) 86 (d) 111 (c) 136 (b) 161 (c) 186 (a)
12 (b) 37 (a) 62 (d) 87 (a) 112 (a) 137 (c) 162 (a) 187 (a)
13
14 ww (a)
(e)
38
39
(b)
(c)
63
64
(a)
(d)
88
89
(b)
(e)
113
114
(b)
(d)
138
139
(e)
(b)
163
164
(e)
(b)
188
189
(a)
(d)

w.E
15 (c) 40 (c) 65 (a) 90 (c) 115 (c) 140 (c) 165 (d) 190 (b)
16 (c) 41 (d) 66 (b) 91 (a) 116 (b) 141 (d) 166 (d)
17 (b) 42 (a) 67 (c) 92 (c) 117 (b) 142 (c) 167 (a)
18
19
20
(a)
(a)
(a)
43
44
45
(b)
(d)
(b)
68
69
70 asy (d)
(c)
(a)
93
94
95
(a)
(c)
(c)
118
119
120
(e)
(a)
(a)
143
144
145
(c)
(b)
(c)
168
169
170
(c)
(e)
(d)
21
22
(d)
(d)
46
47
(a)
(a)
71
72
(c)
(d)
96
97 En
(e)
(d)
121
122
(c)
(d)
146
147
(b)
(b)
171
172
(e)
(b)
23
24
(d)
(c)
48
49
(d)
(a)
73
74
(d)
(b)
98
99
(d)
(e) gin
123
124
(b)
(e)
148
149
(a)
(e)
173
174
(c)
(a)
25 (b) 50 (c) 75 (d) 100

Hints & Solutions


(d) 125 (e)

eer
150 (c) 175 (d)

ing
51. (e) 3 × ? + 30 = 0
30
105 2
ab
c
a bc 1010
.ne
52.

53. (e) ? = 3
?
3

1
10

(a) ? = 40.83 × 1.02 × 1.2 = 49.97592


3 1 22
6 1
57.

58.
(b) ? = 7589 – 3534 = 4055

(d) ? =
1225 35
5 5
7
t
3 7 2 7 59. (e) ? = 300 + (100 × 2) = 300 + 200 = 500
10 45 3 22 5 1.6 2 1.4 8 2.8 5.2
2.44 60. (a) ? 4
3 7 2 7 1.3 1.3 1.3
54. (d) 1.5625 1.25 2 1 1 2 1 1
61. (e) 3 7 5 = (3 + 7 – 5) + =
55. (d) 3978 + 112 × 2 = ? 2 5 5 4 5 5 4
? = (3978 +224) × 2 = 8404
8 4 5 7
5 5
3.7 1.3 2 ? 20 20
56. (e) 10 10 10
62. (d) Let the number be x.
1.3 2 x x x
103.7 10? [ a b ac ab c
] Now 3 or 3 or x = 36
3 4 12

10? 105
2 63. (a) 25 92 32 81 2592 .

Downloaded From : www.EasyEngineering.net


y
Downloaded From : www.EasyEngineering.net

o
u
rs
m
a
h
SOLUTIONS 105

b
o
o
Difference = 25 × 92 – 2592 74. (b) Add 3 after doubling the previous number.

b
(d) The series exhibits the pattern of n2 + 1, n2 – 1,

.w
= 2592 – 2592 = 0 75.
Hence, the numerical difference is 0. alternatively, n taking values 1, 2.............

o
rd
64. (d) Let the third number be 100. Then, the first and second 76. (e) Let x be there in the question mark.

p
numbers will be 20 and 50, respectively.

re
So, 3
860000 x x3 860000

s
20

s
Required % = 100 40 Taking log10 on boths the sides 3log10x = 5.9345

.c
50 log x = 1.9782, Taking antilog we get x 95

o
m
65. (a) Let the cost price of an article be `100 [log x is nearly 2 so, x will be near to but less than 100]
then, S.P. = 100 + 10 = `110
5 1 2 13 16 12
If S.P. = 2 × 110 = `220 77. (d) 1 5 2
8 3 5 8 3 5
(220 100)
then, profit % = 100 = 120% 15 13 40 16 12 24 195 640 288
100 =
66. (b) Ratio of investment 120 120
= 3500 : 4500 : 5500 = 35 : 45 : 55 = 7 : 9 : 11 1123
Since, Ratio of investment is same as ratio of profit. = 9.35 9.
120

ww
Ratio of profit = 7 : 9 : 11
Now, profit = ` 405
78. (a) 8769 82 4
8769 8769
4 82 328
= 26.73 27

w.E
7
A’s share = 405 = `Rs 105
105 79. (c) Let x be there in place of question mark so, x% of
27
45.999 × 16% of 83.006 = 116.073.
1 1 1 x 16

asy
67. (c) Part of the tank filled in one hour = We take 46 83 116
8 16 16 100 100
Hence, the tank will be filled in 16 hours. x × 0.46 × 13.28 = 116

En
68. (d) Part of the tank filled by the three pipes working or x × 6.11 = 116
1 1 1 17 x = 18.98 19.
simultaneously in one hour is =

gin
5 6 12 60 80. (b) 12.998 × 27.059 × 17.999
Can be taken as near to 13 × 27 × 18 6318
60 81. (c) r2 = 2 × 11088
i.e. it takes hours to fill up the tank completely..

1
17
eer 22
7
r 2 = 2 × 11088
Now,
2
of the tank is filled with all the pipes open,

60 1 13 ing
r2 =
2 11088 7
22
7056

69. (c)
simultaneously together in
17 2
= 1 hours
17
Any even number is given by 2n for all n Z, where Z
r= 7056 = 84 metre
.ne
is a set of integers. This is divisible by 9 if it form 9 × 2n
= 18n, which is divisible by 18.
For example, number 36 is even and sum of digits
(3 + 6) is 9, which is divisible by 9. Hence, the number
82. (a) x + y = 8
x y 8
Circumference = 2 r = 2 ×
22
7
t
× 84 = 528 metre

36 is divisible by 18.
2 x 16
70. (a) Volume of water left in the tank = × b × h
= 20 × 7 × (10 – 2) = 1120 m3 x=8
71. (c) This is a series of prime number y=0
72. (d) Let x = 8 Two digit number = 80
then 15 = 2x – 1 = y 83. (b) Required ratio
28 = 2y – 2 = z
= (4220 – 2420) : 2420
53 = 2z – 3 = m
Next term in the pattern should be 2m – 4 = 2 x 53 – 4 = 1800 : 2420 = 90 : 121
= 102 84. (d) The word SCENIC consists of 6 letters in which C comes
73. (d) It is a combination of two series, namely twice.
24, 49, – 94; and 15, 31, 59, 58 6!
The two series correspond to Number of arrangements =
2!
x, (2x + 1), (4x – 1), (4x – 1), (4x – 2)
Hence the missing term is 6 5 4 3 2 1
4 × 24 – 1 = 95 = 360
2

Downloaded From : www.EasyEngineering.net


y
Downloaded From : www.EasyEngineering.net

o
u
rs
m
a
h
106 101 SPEED TEST

b
o
o
85. (e) 4x + 12y = 1520

b
10

.w
Multiplying both sides by 2.5, Gain per cent 100 25
40

o
10x + 30y = 1520 × 2.5

rd
94. (c) If difference between the compound interest and simple
= ` 3800

p
interest on a certain sum of money for 2 yr at r% rate is

re
86. (d) Difference of production of C in 2010 and A in 2015 = `x.

s
5,00,000 tonnes

s
2

.c
87. (a) Percentage increase of A from 2011 to 2012 100

o
Sum x

m
r
55 40
100 37.5% 2 2
40 100 25
128 128
88. (b) Percentage rise/fall in production for B 8 2
128 25 25
` 20000
2011 2012 2013 2014 2015 2 2
95. (c) Let the original number be 10x + y.
9% –16.6% 10% – 9% 10% Number obtained by interchanging the digits

ww
Here, the maximum difference is from 2011 to 2012, which
is 10. And the second nearest to it is fall or rise of 5. So,
= 10y + x
10x + y – 10y – x = 18
9x – 9y = 18

w.E
undoubtedly the answer is 2012.
x–y=2
120 Again, x+y=6
89. (e) Percentage production = 100 133.3%
90 x = 4 and y = 2
90. (c) Average production of A = 50
Average production of B = 54.17 asy Original number
= 10 × 4 + 2 = 42
96. (e) Let Samar’s monthly salary be `x.
Average production of C = 50
Difference of production = 54.17 – 50 = 4.17 En According to the question,
x – (52 + 23)% of x = 4500
91. (a) Let the number be x.
x 63 gin x – 75% of x = 4500
25% of x = 4500

eer
2583 4500 100
100 x `18000
25
2583 100

ing
x 4100 97. (d) 55% of total students = 44
63
44 100
45 Total number of student 80

.ne
45% of this number 4100 1845 55
100 Number of boys = 80 – 44 = 36
92. (c) Decimal equivalents of the fractions

t
45 75 40 78
4 6 98. (d) Required average marks
0.8, 0.857 45 40
5 7
3375 3120 6495
2 9 76.41
0.22, 0.82 85 85
9 11 99. (e) Perimeter = 2(Length + Breadth)
3 48 = 2(5x + 3x)
0.375
8 48
x 3
Clearly, 16
0.857 > 0.82 > 0.8 > 0.375 > 0.22 Area = (5 × 3) × (3 × 3) = 15 × 9 = 135 m2
6 9 4 3 2 100. (b) Let the children in row = x
The children in a column = (x + 2)
7 11 5 8 9
According to the question,
93. (a) Purchasing price of articles = `(245 × 30) = `7350 x(x + 2) = 63
Total cost = `(7350 + 980 + 1470) = `9800 x2 + 2x – 63 = 0
9800 x2 + 9x – 7x – 63 = 0
CP per piece ` 40 x(x + 9) – 7(x + 9) = 0
245
SP = `50 per piece (x + 9) (x – 7) = 0
Hence, there are 7 children in each row.

Downloaded From : www.EasyEngineering.net


y
Downloaded From : www.EasyEngineering.net

o
u
rs
m
a
h
SOLUTIONS 107

b
o
o
101. (b) Clearly option (b) is best supported by the passage. For Q.No.114-119

b
.w
102. (a) The given passage talks about the cause of corruption P$Q P<Q

o
and option (a) also has the same sense. Hence, it may P©Q P>Q

rd
be the inference.

p
P Q P=Q
103. (a) Obviously, option (a) is the most effective statement in

re
P@Q P Q

s
the context of the given passage.

s
.c
104. (e) According to the question, after interchanging the P*Q P Q

o
symbols, equations becomes 114. (d) Given statements, M@D M K,

m
26 + 15 ÷ 5 – 4 × 2 = 26 + 3 – 8 = 29 – 8 = 21 D K D = K, K© R K >R
+4 +2 +2 +2
Conclusions :
+2
I. R $ M R < M (True)
105. (a) R P N W S U H D F L H J Q M O
II. K M K= M (True)
–2 +2 +2 +2 +2 III. K $ M K < M (True)
So, RPN is different from other four. Hence, only either II or III and I is true from the given
Left Right statements.
10 students 5th
D R 115. (c) Given statements F*T F< T,,

ww
106. (d)
40 students T $ N T < N, N@ R N R
So, D’s position from the left end of the row Conclusions :
I. R $ T R < T (False)

w.E
= 40 – (10 + 1 + 5 ) + 1 = 41 – 16 = 25 th
II. N ©F N>F (True)
R I R III. F $ R F < R (False)
107. (a) O B T L
Hence, only conclusion II is true from the given
+1
C
+1
+1
P
–1
asy M
+1
+1
+1
statements.
116. (b) Given statements, B © N
N@ R N R, F * R,
B> N,
F R

En
S H Conclusions :
Similarly, I. B © R B > R (True)

C
H
A G
E
R
gin II. F * N
III. R $ B
F N (False)
R < B (True)
Hence, only I and III conclusion are true from the given
+1
+1
+1
–1
+1
+1
+1

eer
statements.
117. (b) Given statements, D $ M D< M,
B I D Q S F H
108. (e) According to the question, Scored order among the
M*B

ing
M < B, B J B =J
Conclusions :
I. J © D J > D (True)

.ne
students = A > D or D > A > B > C > E
II. B @ D B D (False)
So, E student is the lowest marked student.
III. J @ M J M (True)
109. (e) O V E R W H E L M

So, such number of pairs are OR, EH, LM, MR.


110. (c) B R O W N and M E A N
statements.
118. (e) Given statements, W K
K©F K >F, F $ M F<M
W =K, t
Hence, only I and III conclusion are true from the given

Conclusions :
I. M © K M > K (False)
5 3 1 @ % 2 6 © %
II. W @ F W F (False)
R O B E
III. F @ W F W (False)
3 1 5 6 Hence, any conclusion is not true from the given
111. (c) According to the question, in the above arrangements statements.
seventh to the left of the nineteenth from the left is ©. 119. (a) Given statements, F @ T F T,
112. (a) According to the question, in the above arrangements T K T =K, K * D K<D
following series is progressing (+ 2) and ( + l)step in his Conclusions :
group and next group comes (+ 3) step, I. D @ F D F (False)
So, next series comes in the group is U I *. II. F @ K F K (True)
113. (b) According to the question, in the above arrangements III. D @ T D T (True)
one vowel is immediately preceded by a symbol and Hence, conclusion II and III are true from the given
immediately followed by a number, Vowel is © A6 statements.

Downloaded From : www.EasyEngineering.net


y
Downloaded From : www.EasyEngineering.net

o
u
rs
m
a
h
108 101 SPEED TEST

b
o
o
For Q. 120-125. Conclusions :

b
.w
Sitting arrangements of 8 persons would be as follows : I. Some buses are cars. (False)
II. Some trains are bikes. (False)

o
D

rd
M Q III. Some buses are bikes. (False)

p
So, none conclusion follows given statements.

re
128. (d) According to the question, Venn-Diagram is

s
s
P

.c
B

o
m
T A Rats
Dogs Rats Dogs
R or ts
Ma
120. (a) From the above arrangements, M is to the immediate Cats Ma
ts
Cats
right of D.
121. (c) From the above arrangements, T is second to the right Conclusions :
of M. I. Some mats are cats. (True)
122. (d) From the above arrangements, P is second to the left of D. II. Some mats are dogs. (False)
123. (b) From the above arrangements, M is third to the right of P.

ww
III. Some rats are cats. (True)
124. (e) From the above arrangements, P and R are the immediate So, conclusion I and III follow given statements.
neighbours of A. 129. (d) According to the question, Venn-Diagram is
125. (e) From the above arrangements, in the following pairs, P

w.E
is immediate to the right of A.
For Q. No. 126-131.
126. (c) According to the question, Venn-Diagram is

asy Pins
Dr
um

En
cups Drums
s

Sticks
or cups
tes

Rings
Ki
s

Benches

gin
ite

Benches Rods
K

Conclusions :
I. Some kites are cups. (False) Conclusions :
II. Some kites are benches. (True)
III. Some drums are·cups. (False) eer
I. Some rings are pens. (True)
II. Some roads are sticks. (True)

ing
So, only conclusion II follows, given statements. III. Some roads are pens. (True)
127. (a) According to the question, Venn·Diagram is So, all I, II, III conclusions follow, given statements.
130. (c) According to the question, Venn-Diagram is

.ne
Bikes Cars Trains Buses
Tables
t
es

Chairs
Hous

ts
Ten

Cars

Bikes
or
Conclusions :
Buses I. All houses are chairs. (False)
Trains II. Some tents are chairs. (True)
III. Some houses are tables. (True)
So, conclusion II and III follow given statements.
Cars
131. (c) According to the question, Venn-Diagram is

Bikes
Boxes Walls Roads

or
Trains Rivers
Buses
or

Downloaded From : www.EasyEngineering.net


y
Downloaded From : www.EasyEngineering.net

o
u
rs
m
a
h
SOLUTIONS 109

b
o
o
b
140. (c) green grass everywhere dik pa sok

.w
o
cow eats grass nok ta pa

rd
Boxes Walls Roads

p
The code for 'cow' is 'nok' or 'ta'

re
Rivers 151. (d) The reason behind the question is not mentioned in

s
s
.c
the passage.
Conclusions :

o
152. (a) Refer to the second sentence of the first para of the

m
I. Some rivers are walls. (False) passage that the king had to agree to a contract.......
II. Some roads are boxes. (False) being a king.
III. No wall is river. (True) 153. (c) Refer to the sentence that the island was covered.......
So, only conclusion III follows given statements. discovered dead bodies.......past kings of the second
132. (a) M A I N D E A R last sentence of the third para of the passage.
154. (e) Refer to the fourth sentence that in the first
9 3 6 4 8 5 3 2 month.......trees were cut down.......of the fourth para
Therefore, of the passage.

9 ww
M E

5
N

4
D

8
155. (a) Refer to the last sentence “I turned the deadly
island..............a beautiful abode.......peacefully” of the
sixth para of the passage.

w.E
133. (e) D R E A M I N G 156. (e) From reading the passage thoroughly we come to the
conclusion that the king was intelligent, foresight and
cunning as he made deadly island a beautiful place to
live in with all luxuries of the kingdom.

asy
+1 –1
157. (b) Refer to the sentence that ‘I know.......complete the
year’ of the statement made by king in the fourth para
B F S E F M H L of the passage.
Similarly,
T R E A T I S E En 158. (d) Refer to the second last sentence of the fourth para
that he sent all the.......for storage.......of the passage.

+1 –1 gin
159. (c) Refer to the third sentence that however, the king.......leave
the kingdom.......of the last para of the passage.

eer
160. (e) The moral of the story is ‘Always think and plan
ahead’.
161. (c) The word contract means agreement. Compact, bond,

ing
B F S U D R H S
deal, bargain etc. are the most similar word in meaning.
162. (a) The word bidding means order, command, wish, desire,
request, direction etc. Hence wishing is most similar in
134. (c) 5 3 1 4 6 9 7
1 3 4 5 6 7 9
meaning of the given word.
.ne
163. (e) The word abode means house, dwelling, residence and
135. (b) C > E, A > B > D

136. (b)
Now, A > B > D > C > E

N
habitation etc.
164. (b) The word survived means remained alive, lived,
t
endured etc. Hence died is the most opposite word in
meaning.
30 m 165. (d) The word vicious means brutal, savage, dangerous,
cruel etc. Its opposite word will be gentle, kindly,
E harmless etc.
W
20 m 166-170. Sentences are re-arranged in order as DACFBE. A mother
duck is the clue and qualifier of a sentence which is
S followed by A, C, F and finally B and E.
171. (e) Here, too is used as emphatic word. Lata was so scared
137. (c) Meaningful words that she could not go home alone. Hence, no correction
TOUR, ROUT is required.
138. (e) 48 Q 12 R 10 P 8 W 4 = ? 172. (b) The structure of sentence is subject + was/were + third
? = 48 ÷ 12 × 10 – 8 + 4 form of verb + object. Thus, Riya was dressed to kill.
? = 4 × 10 – 8 + 4 173. (c) The given sentence is the statement of simple past
tense. Hence it should be ‘worried’ instead of worries.
? = 40 – 8 + 4 = 36
174. (a) The phrase ‘let off’ means to give them only a light
139. (b) Second Highest Number punishment.
7 39 175. (d) It should be ‘took’ instead of ‘take’.

Downloaded From : www.EasyEngineering.net


y
Downloaded From : www.EasyEngineering.net

o
u
rs
m
a
h
110 101 SPEED TEST

b
o
o
b
SPEED TEST 101 : MAIN TEST - 5

.w
Answer Key

o
rd
1 (d) 26 (a) 51 (d) 76 (a) 101 (d) 126 (a) 151 (a) 176 (a)

p
re
2 (c) 27 (b) 52 (b) 77 (d) 102 (b) 127 (b) 152 (e) 177 (e)

s
3 (c) 28 (d) 53 (a) 78 (b) 103 (b) 128 (a 153 © 178 (b)

s
.c
4 (c) 29 (b) 54 (c) 79 (a) 104 (c) 129 (e) 154 (a) 179 (d)

o
m
5 (a) 30 (a) 55 (e) 80 (e) 105 (a) 130 (a) 155 (d) 180 (d)
6 (c) 31 (c) 56 (d) 81 (c) 106 (e) 131 (d 156 (e) 181 (e)
7 (a) 32 (c) 57 (b) 82 (a) 107 (d) 132 (c) 157 (d) 182 (a)
8 (d) 33 (a) 58 (c) 83 (d) 108 (c) 133 (c) 158 (c) 183 (c)
9 (d) 34 (c) 59 (e) 84 (e) 109 (c) 134 (a) 159 (b) 184 (a)
10 (d) 35 (c) 60 (e) 85 (b) 110 (b) 135 (e) 160 (b) 185 (c)
11 (b) 36 (a) 61 (b) 86 (c) 111 (a) 136 (d) 161 (b) 186 (c)
12 (a) 37 (b) 62 (c) 87 (b) 112 (a) 137 (d) 162 (c) 187 (c)
13
14ww (d)
(b)
38
39
(c)
(d)
63
64
(b)
(b)
88
89
(a)
(d)
113
114
(c)
(c)
138
139
(b)
(a)
163
164
(d)
(e)
188
189
(a)
(d)
15
16
17 w.E (a)
(d)
(c)
40
41
42
(c)
(a)
(c)
65
66
67
(a)
(c)
(b)
90
91
92
(e)
(b)
(c)
115
116
117
(d)
(d)
(e)
140
141
142
(c)
(a)
(c)
165
166
167
(a)
(d)
(e)
190 (a)

18
19
20
(e)
(c)
43
44
45
(b)
(a)
(a)
68
69
70
asy
(a)
(a)
(c)
93
94
95
(a)
(e)
(d)
118
119
120
(c)
(a)
143
144
145
(d)
(c)
168
169
170
(a)
(b)

En
(a) (c) (a) (c)
21 (a) 46 (c) 71 (c) 96 (c) 121 (b) 146 (a) 171 (d)
22 47 (b) 72 97 (d) 122 147 172

gin
(a) (d) (c) (b) (c)
23 (a) 48 (b) 73 (b) 98 (b) 123 (b) 148 (c) 173 (a)
24 (a) 49 (b) 74 (c) 99 (b) 124 (b) 149 (d) 174 (b)
25 (c) 50 (a) 75 (c) 100

Hints & Solutions


(a) 125 (d)
eer
150 (a) 175 (a)

ing
21. (a) Iran Recently, India has offered to invest up to 20 billion
US dollars in Iran's energy industry, subject to provision .ne
Railway station to Agra Cantt. Station. The train
completed its 200 km journey to Agra within 100

22.
of concessional rights.The two nations, also discussed
developments regarding Farzad-B gas field. .
(a) Telangana Mission Bhagiratha is the water grid project
in Telangana State of India, which aims at reaching out
27.
t
minutes. It is capable of running at a maximum speed of
160 kmph.
(b) The world's first 'White Tiger Safari' has recently opened
for public at Mukundpur in Satna district, Madhya
drinking water supply to even remotest place in the Pradesh. The first of its kind safari has cost Rs. 50 crore
State. and is spread over an area of 25 hectares. It is home to
23. (a) simplification of income tax laws three white tigers including one male, two females and
24. (a) Subhash Palekar, popularly known as Krishi ka Rishi two Royal Bengal tigers.
(the "sage of agriculture"), is a famous exponent of 28. (d) Unakoti hill is an ancient Shaivite place of worship with
natural farming. huge rock reliefs celebrating Shiva. It is the prime tourist
25. (c) The Gujarat Government has launched the state-wide spot of Unakoti Tripura District in the Kailashahar
"Maa Annapurna Yojna (Scheme)" at Sanand, Subdivision in the North-eastern Indian state of Tripura.
Ahmedabad to provide cereals at concessional rates .
to poor and middle class families under the National 29. (b) The Central Industrial Security Force (CISF) Raising
Food Security Act (NFSA). This scheme will cover BPL, Day is celebrated every year on March 10. On this day,
Antyodaya and APL families. in 1969, the CISF was set up under an act of the
26. (a) India's fastest train Gatimaan Express "Gatimaan Parliament of India with a strength of 2,800. It is directly
Express" has been flagged off from Hazrat Nizamuddin under the Union Ministry of Home Affairs and its
headquarters are at New Delhi.

Downloaded From : www.EasyEngineering.net


y
Downloaded From : www.EasyEngineering.net

o
u
rs
m
a
h
SOLUTIONS 111

b
o
o
30. (a) A 108-feet tall idol of Lord Rishabhdeva, the first 37. (b) Gujarat Nargol port is located in Gujarat. Indian Railways

b
.w
Teerthankar of Jains, carved out of a single rock, has proposes to undertake implementation of rail connectivity

o
entered the "Guinness World Records" as the world's for the ports of Nargol and Hazira under PPP

rd
tallest Jain statue. The impressive statue is located atop 38. (c) Madhya Pradesh The 42nd edition of world fame

p
re
Mangi Tungi mountain near Teharabad village of Khajuraho Dance Festival has started at Khajuraho in

s
Baglan tehsil in Nashik district of Maharashtra. Before Chhatrapur District, Madhya Pradesh. This cultural

s
.c
this statue, the 57 feet idol of Lord Bahubali in festival highlights the richness of the various Indian

o
Shravanabelagola in Karnataka was considered the

m
classical dance styles like Kathak, Bharathanatyam,
world's tallest Jain statue. Odissi, Kuchipudi, Manipuri and Kathakali with
31. (c) The President of India Pranab Mukherjee has declared performances of some of the best exponents in the field.
Kerala as India's first digital state during the launch of 39. (d) The Maharashtra State Economic Development Council
digital empowerment campaign at Kozhikode, Kerala. and Naik Environmental Research Institute Ltd in
The campaign aims to develop better digital Nashik have formed Deepak Naik committee to work
communication across the state by 2020. Kerala is the towards reviving the river Godavari. Naik is the
first Indian state to successfully link its villages with chairman of Maharashtra state Economic development
broadband connectivity under the National Optical

32. ww
Fibre Network (NOFN) programme.
(c) February 28 The National Science Day is celebrated in
40.
council. He is an expert in river issues.
(c) India's first ever Gender Park has been inaugurated by
President Pranab Mukherjee at Kozhikode, Kerala. The

w.E
India on February 28 every year to mark the discovery
park is set up to promote research and other initiatives
of the Raman effect by Indian physicist C V Raman.
to enable total gender equality. The Gender Institute at
The 2016 theme is "Scientific Issues for Development
the park would focus on learning, research and capacity
of the Nation" that aims at raising public appreciation.

33. asy
(a) The NDDB foundation for Nutrition (NFN) has
development, as part of supporting the efforts of the
State and Central governments in ensuring an inclusive,
discrimination free society.

En
launched its novel 'Gift Milk' initiative to provide
nutritional support for school children. NDDB's wholly 41. (a) The World Day of Social Justice is observed on
February 20 to recognize the need to promote efforts to
owned subsidiary, Indian Immunologicals Limited (IIL)
partnered with NFN and launched the initiative in the
Z.P. School, Laxmapur village, Ranga Reddy District, gin tackle issues such as poverty, exclusion and
unemployment. The 2016 theme is "A Just Transition -
Telangana. IIL has adopted this school as part of its
CSR activity. . 42.
eer
environmentally sustainable economies and societies".
(c) The Odisha Government has launched the Pucca Ghar
34. (c) India The World's first fast-acting anti-rabies drug
"Rabies Human Monoclonal Antibody (RMAb)" will
be launched in India. The drug has been developed by ing
Yojana for construction workers. The pucca houses
will be built for 50,000 construction workers during the
current financial year with a cost of Rs. 500 crore. The
Pune-based Serum Institute of India (SII) and US-based
MassBiologics of the University of Massachusetts .ne
state government has decided to provide an assistance
of Rs.1 lakh to each registered construction worker of

35.
Medical School.
(c) Prof. Goverdhan Mehta has been conferred with the
Federal Republic of Germany's prestigious
"Bundesverdienstkreuz" (the Cross of the Order of
t
the state for construction of his/her house under the
Yojana. A construction worker registered with the
Odisha Buildings and Construction Workers' Welfare
Board for the three years can avail the benefit.
Merit). He is a well-known researcher in Chemical 43. (b) Arunachal Pradesh is also known as the "land of the
Sciences & former Vice-Chancellor of the University of dawn-lit mountains". Arunachal Pradesh was granted
Hyderabad (UoH). He is a Fellow of the Royal Society statehood on 20th February, 1987. . 7. Utkarsha Bangla
(FRS), a Foreign Member of the Russian Academy of scheme has been launched by the West Bengal
Sciences and has been conferred with 'Chevalier de la government, for what purpose? [A]To train school
Legion d'Honneur' by the President of France. dropouts [B]To stop human trafficking [C]To promote
36. (a) GVK Biosciences The Hyderabad-based GVK entrepreneurship among women [D]To promote organic
Biosciences has received the Global CSR Excellence & farming Hide Answer To train school dropouts The
Leadersh ip Awards in the category of Best West Bengal government has recently launched
Environment-Friendly Project for its "Go Green, Grow Utkarsh Bangla Scheme, a welfare scheme for school-
Green" plantation drive. The global CSR awards are dropouts. The scheme is aimed at giving vocational
constituted by the World CSR Congress every year on training to school dropouts who will be trained free of
the World CSR Day and are presented to companies charge in tailoring, driving, repairing television and
who believe in the value of sustainable corporate social others. Each youth will get training ranging from 400 to
responsibility. . 1200 hours.

Downloaded From : www.EasyEngineering.net


y
Downloaded From : www.EasyEngineering.net

o
u
rs
m
a
h
112 101 SPEED TEST

b
o
o
44. (a) The first train connecting China to Iran through the ancient 63. (b) Let the sum of money be ` x.

b
Silk Road has arrived in the Iranian capital after traveling

.w
3
over 10,000 kilometers. The train had started its journey r
Now, 8x

o
x 1

rd
from China's eastern Zhejiang province and it took 14 days 100

p
to reach final destination. It had passed through two Central 3

re
Asian countries Kazakhstan and Turkmenistan r r
or, 1 (2)3 or 1 2

s
100

s
45. (a) The Indian Institute of Petroleum and Energy (IIPE) 100

.c
has been established at Visakhapatnam by the Ministry Again, let the sum becomes 16 times in n years. Then,

o
of Petroleum and Natural Gas (MoPNG), Government

m
n
of India. This institute has the mandate to lead the r
16 x x 1
nation forward in education, research and training 100
related to all areas of Petroleum and Energy.
46. (c) 47. (b) 48. (b) 49. (b) 50. (a) 16 2 n or 2 4 2 n or n 4
64. (b) Let the cost price of machine be ` 100
5 4 3 SP of machine at a profit of 10% = ` 110
51. (d) ? 222 37
8 9 5 SP of machine at a loss of 10% = ` 90
52. (b) Let the number be x If SP is (110 – 90) = ` 20 less then CP = ` 100
Therefore, if SP is ` 40 less, then

ww 56
100
450 x

(27)1.5 (21)3.5
300 or x = 300 – 252 = 48

(27) ?
CP =
100
20
40 ` 200

w.E
53. (a) 65. (a) Let us work with the options.
For (a), total cost = 7 480 2 130 3620
?=5 ax ay a(x y)
For (b), total cost = 7 480 3 130 3750
54. (c) Let the number be x.

asy
For (c), total cost = 2 480 7 130 1870
27.06 × 25 – x = 600 Hence, option (a) is correct.
or, x = 676.5 – 600 = 76.5 66. (c) Relative speed of the trains = (40 + 20) = 60 m/s

En
39 30 45 1 Distance = (120 + 120) = 240 m
55. (e) ? 11 Time taken by trains to cross each other completely
8 13 4 4
240
56. (d) 8 4
83
1
8
82
5
8
1
8 4 3 5 2
?=4 4

gin =
60
4s
Larger the no. of cogs (tooth of wheel) of wheel,
57. (b) –(a – b) . x = b – a
Put x replacing ‘?’ (question mark)
or [ (a b) x] [b a] or (a b) x a b
67.
eer
lesser will be that no. of revolution made by it.
(b) Let Sunil finishes the job in x hours.
x

or x
a b
1 ing
Then, Ramesh will finish the job in

x
2
hours.

.ne
a b We have, x 3 x 6
2
58. (c) a b ? ( a b) Therefore, Sunil finishes the job in 6 hours and Ramesh

t
or a b x.( a b) in 3 hours.
[Put x replacing ‘?’ (question mark)] 1 1 1
or a + = x. (a + b) Work done by both of them in 1 hour =
6 3 2
or x = – 1 They together finish the piece of work in 2 hours.
59. (e) |? + 14| = 11 or ? + 14 = 11 or – 11 68. (a) Let the present age of the man = x years
? = – 25 or – 3 Now, (x + 15) = 4 (x – 15)
60. (e) 16 + 26 × 2 = 16 + 52 = 68 or 3x = 75 or x = 25 years
12 1 69. (a) Area of the outer rectangle = 19 × 16 = 304 m 2
61. (b) 0.1008, 0.1
119 10 2m
Verandah
4 7
0.102 and 0.101
39 69
1 12
Thus, is the least. 2m
10 2m
62. (c) Let there be n points marked on the plane.
15 Floor of room
Total number of line segments = n C 2 =10
2m
n(n 1)
10 or n 2 n 20 0 Area of the inner rectangle = 15 × 12 = 180 m2
2
Required area = (304 – 180) = 124 m2
or (n 5)(n 4) 0 or n = 5 [n = – 4 is rejected]

Downloaded From : www.EasyEngineering.net


y
Downloaded From : www.EasyEngineering.net

o
u
rs
m
a
h
SOLUTIONS 113

b
o
o
70. (c) The terms exhibit the pattern 21, 22, 23 and so on.

b
1.08

.w
71. (c) Try the pattern n 3 – 1, n = 1, 2, ....... 87. (b) Required percentage = 100 34
3.14
Pattern is 22, 42 – 1,42 , 62 – 1, 62 and so on.

o
72. (d)

rd
73. (b) Can you see that the pattern is 88. (a) Total number of candidates appearing from all the cities

p
together
12 , 2 3 , 3 2 , 4 3 , 5 2 , 6 3 , 7 2

re
= (1.25 + 3.14 + 1.08 + 2.27 + 1.85 + 2.73) lakhs

s
74. (c) Note that

s
= 12.32 lakhs

.c
0 = 13 – 1 6 = 2 3 – 2

o
Number of candidates passing from City F

m
24 = 33 – 3
7
75. (c) ? 1223.9975 34 = 2.73 1.5925 lakh
12
76. (a) ? = 503 × 201 = 101103 101100
77. (d) ? = 1205 2.5 = 482 480 1.5925
Required percentage = 100 12.93
78. (b) ? = 22020 0.011 = 2001818 2002000 12.32
89. (d) Number of failures:
79. (a) ? 20800 144 12
3
80.

81.
(e) ?
ww
8 15 22
5 7 3
176
7
(c) Required average height
25.14 25
City A
10

3
× 1025 lakhs = 0.375 lakh

w.E
City B × 3.14 lakhs = 1.1775 lakh
8
142 156 162 178 638
159.5 cm 5
4 4 City C × 1.08 lakhs = 0.6 lakh
82. (a) Manoj's monthly income

= 2.16 100000 = ` 18000 asy City D


9

3
× 2.27 lakh = 1.7025 lakh
12
Pratul's monthly income
En 4
2

= 18000 ×
1
4
= ` 4500
gin City E

5
5
× 1.85 lakhs = 0.74 lakh

Pratul's annual income


= 12 × 4500 = ` 54000 90. eer
City F
12
× 2.73 lakh = 0.455 lakh
(e) Number of passed students from City E
83. (d) The pattern of the number series is:
17 + 92 = 17 + 81 = 98
98 + 72 = 98 + 49 = 147
=
3
5 ing
1.85 lakhs = 111000.

147 + 52 = 147 + 25 = 172


172 + 32 = 172 + 9 = 181
91. (b) Interest = `8791 – 7450 = `1341
I 100 1341 100 .ne
2235

84.
181 + 12 = 181 + 1 = 182
(e) The pattern of the number series is:
11 + 8 = 19
92.
Time
P R 7450 6
(c) 36% of x – 12% of x = 82.82

x
36
–x
12
82.32
745
3 yr

t
or
19 + 12 (= 8 + 4) = 31 100 100
31 + 16 (= 12 + 4) = 47
9 x 3x
47 + 20 (= 16 + 4) = 67 or – 82.32
25 25
67 + 24 (= 20 + 4) = 91
6x
85. (b) The pattern of the number series is: or 82.32
25
748 – 460 = 288
or 6x = 82.32 × 25
460 – 316 = 144
316 – 244 = 72 82.32 25
or x 343
6
244 – 208 = 36 ? = 208 – 18 = 190
93. (a) 16 watches + 21 calculators = `30485
3 3 32 watches + 42 calculators = 2 × `30485 = `60970
86 (c) Required ratio = 2.27 : 1.25
4 10 94. (e) x2 + (66)2 = 4840
= 1.7025 : 0.375 = 227 : 50 or x2 + 4356 = 4840
or x2 = 4840 – 4356

Downloaded From : www.EasyEngineering.net


y
Downloaded From : www.EasyEngineering.net

o
u
rs
m
a
h
114 101 SPEED TEST

b
o
o
or x2 = 484

b
102. (b) G I V E F A I L

.w
or x = 22
(d) Let the money received by A, B and C be `3x, 4x and

o
95.

rd
5x, respectively and money received by P and Q be `2y 5 1 @ © % 2 1 9

p
and y, respectively.

re
4x – y = 1050 …(i) Therefore, L E A F

s
s
.c
Since, we cannot from another equation here. So, we cannot

o
solve it. 9 © 2 %

m
96. (c) x2 + y2 = (x + y)2 – 2xy
= (18)2 – 2 × 72 103. (b) 3 8 6 7 4 5 1 0 9 2
= 324 – 144
3rd from right
= 180
104. (c) All others are parts of a Car.
484
97. (d) 100 64.533 105. (a) S U B S T A N C E
750
= 65% about
–1 +1 +1

98. (b)
ww
CI = p 1
r
100
t
–1
R A T R U F D O B

`5000 w.E 1
3
100
3
–1
Similarly,
T E N T H O U S E

`5000
103 103 103
–1 asy –1 +1 +1
100 100 100

1092727 –1000000 En S M D S I F T V P
`5000
1000000
gin
106. (e) 519
287
915; 364
782; 158
463;
851;

eer
92727 835 538
`5000
1000000 Second lowest number
= `5000 × 0.092727

ing
538 83 5
= `463.635
107. (d) Second highest number
= `464 (about)

.ne
99. (b) In 1 day, Girish attends 7 customer's call 581 158
In 60 days, Girish attends 60 × 7 = 420 calls 108. (c) Highest number 835
100. (a) Suppose that numbers are x and y Lowest number 158
From question
2x + 3y = 126
3x + 2y = 144
…(i)
…(ii)
Eq. (i) multiple by 3 and Eq. (ii) multiple by 2 and then
Required difference
(109–113) : Sitting arrangement

M
D
5–3=2
t
subtract Eq. (ii) from Eq. (i), we get W
5y = 90
or y = 18 R Q
Put the value of y in Eq. (i)
2x + 3y = 126 P B
or 2x + 3 × 18 = 126 A
or 2x + 54 = 126 109. (c) Q is second to the left of D.
or 2x = 126 – 54 = 72 110. (b) D is third to the left of P.
or x = 36 111. (a) W is to the immediate right of W.
x = 36, y = 18 112. (a) D and M are the first and second respectively to the
right of W.
Smaller number = 18
113. (c) B is sitting to the immediate immediate right of A.
101. (d) treeis very beautiful ka na da ta = $ @
(114– 118) : #
this is strong tree na pa sa ka
The code for ‘beautiful’ is either ‘da’ or ‘ta’.

Downloaded From : www.EasyEngineering.net


y
Downloaded From : www.EasyEngineering.net

o
u
rs
m
a
h
SOLUTIONS 115

b
o
o
114. (c) W @ T W T

b
121. (b) Number Symbol Number

.w
T©M T M

o
There is only one such combination: Q $ 6

rd
M$D M < D Therefore, W T M < D

p
Conclusions

re
122. (c) Number Consonant Symbol
I. W # D W D : Not True

s
s
Such combinations are:

.c
II. W @ M W M : Not True

o
1H % ; 2 Q $

m
III. D # T D T : True
1 3
115. (d) F R F R 123. (b) 8 1 D
R M R M 1 2
7 5
M$D M D
1 3
Therefore, F= R M D P 2 7
Conclusions E
1
J
3
R
I. D # R D > R : True
II. D # F D > F : True 1 3

ww
T 4 J
III. M @ F M F : True (124– 130).
116. (d) V © M V M (i) All petals are trees Universal Affirmative (A-type).

w.E
M B M B
B$ F B F
Therefore, V M B F
(ii) Some days are nights Particular Affirmative (I-type).
(iii) No lock is toy Universal Negative (E-type).
(iv) Some locks are not toys Particular Negative (O-type)

asy
124. (b) All keys are locks.
Conclusions
I. F # M F > M : True
II.
III.
B @V
F #V
B V : True
F V : True En No lock is toy.
A + E E-type of Conclusion

117. (e) D # N
N@B
D>N
N B gin “No key is toy.”
All bags are toys.

B F B F
eer
No toy is lock.

ing
Therefore, D> N B = F A + E E-type of Conclusion
Conclusions “No bag is lock.”
I. F$ D F D : True All bags are toys.
II. N# F N > F : Not True
III. N F N = F : Not True .ne
N is either greater than or equal to F. Therefore, either I
or II is true.
118. (c) R $ T R T
No toy is key.
A + E E-type of Conclusion
“No bag is key.
This is Conclusion I.
t
125. (d) All the three Premises are Particular Affirmative (I-type).
T# K T>K
No Conclusion follows from the two Particular Premises.
K@M K M Conclusions I and III from complementary pair.
Therefore, R < T > K M Therefore, either I or III follows.
Conclusions 126. (a) Some tyres are wheels.
I. R $ M R M : Not True
II. T# M T > M : True
All wheels are buses.
III. R$ K R < K : Not True I + A I-type of Conclusion
119. (a) 21st from the right end is B and sixth to the right of B “Some tyres are buses.”
is 8. Conclusion I is Converse of it.
Trick : Required answer = 21 – 6 = 15th from the right, Conclusion II is Converse of the second premise.
i.e., 8. 127. (b) Some cats are horses.
120. (c) Symbol Vowel Consonant
Such combinations are:
@EJ ; %AV All horses are tigers.

Downloaded From : www.EasyEngineering.net


y
Downloaded From : www.EasyEngineering.net

o
u
rs
m
a
h
116 101 SPEED TEST

b
o
o
I + A I-type of Conclusion

b
“Some cats are tigers.” 139. (a) 1 8 4 3 5 7 2 9

.w
Conclusion I is Converse of it.

o
5th of the left of 2

rd
128. (a) All ropes are sticks.
140. (c) Z Y X W V U T

p
re
Z Y X W V U

s
Z Y X W V

s
Some sticks are hammers.

.c
A + I No Conclusion Z Y X W

o
m
129. (e) The following changes occur in the subsequent figures.
Z Y X
(1) to (2) (1) to (3)
151. (a) Refer to the sentence ‘He could not find.........in search
of work’.
152. (e) Refer to the sentence “Please do not help him”..........of
the second para of the passage.
153. (c) Refer to the sentence “You are...........bite me?”........of
These two steps are continued in the subsequent the second para of the passage.
figures alternately. 154. (a) Refer to the sentence “He would be.......Ghanshyamdas”

ww
130. (a) The following changes occur in the subsequent figures:
(1) to (2) (2) to (3)
.......of the third para of the passage.
155. (d) Refer to the sentence, “He gave him........delicious
fruits”........of the third para of the passage.

w.E 156. (e) Refer to the sentence “I shall creep........bite her........


hand on her forehead” of the fourth para of the passage.
157. (d) Refer to the sentence, “A man brought .......prince who
is missing”........of the third para of the passage.

These two steps are continued in the subsequent


figures alternately. asy 158. (c) Refer to the sentence “He at once........pieces of gold”
of the second last sentence of fourth para of the
passage.
131. (d) The given passage talks about the disposal of election
petitions. It also gives the resolution of the disposal of En 159. (b) Refer to the sentence “Finally, the king
declared........handsomely rewarded” of the fourth para
the election petition. While option (d) also have the
same sanse, Hence it may be a conclusion.
gin of the passage.
160. (b) The moral of the story can possibly be “A good deed
never goes in vain”.

eer
132. (c) Obviously, option (c) strengthens the conclusion of
the passage. 161. (b) The word ‘live’ is most similar in meaning to the word
133. (c) The summary of option (c) opposes the conclusion of the ‘go’ in the context of the passage.

134. (a)
passes. Thus it weakens the conclusion of the passage.
The code has been generated by writing the letters in
reverse order OMNIBUS SUBINMO ing
162. (c) The word ‘modest’ is a synonym of humble, while other
words are plain, simple, ordinary and unpretentious.
163. (d) The most similar word is ‘ask for’ in the context of the
Therefore, TROUBLE ELBUORT
passage.

.ne
164. (e) The word ‘stopped’ is most opposite in meaning like
break off.
135. (e) 3 15 18 16 15 18
C O R P O R A
1 20 5
T E
165. (a) The word ‘meagerly’ is the antonym of the word
handsomely in the context of the passage. t
166. (d) Replace to by ‘too’ to make a correct phrase ‘a bit too
fast’ which means slightly or to a small extent.
167. (e) The word think about refers to consider. Hence no
136. (d) 1 2 3 4 5 6 7 8 9 correction is required.
E D U C A T I 0 N 168. (a) Replace leisure by the adverbial word leisurely.
169. (b) The phrase ‘gift of the gab’ means ‘to have a talent for
Meaningful word DATE speaking’.
170. (c) The phrase ‘the order of the day’ means common,
137. (d) C 20 m B popular or suitable at a particular time. For example :
Pessimism seems to be the order of the day.
10 m 10 m 171. (d) It should be ‘here was her son’s future’.
172. (c) The correct spelling should be ‘triumphant’.
E 173. (a) The word suppressed should be replaced by revealed
5 km D A or leaked in the sentence.
Required distance = AE = AD + DE = 20 km + 5 km = 25 km 174. (b) It should be ‘handed over’ which means the act of
138. (b) 12 B 20 A 4 Q 10 F 30 = ? moving power or responsibility from one person to
? = 12 × 20 ÷ 4 – 10 + 30 another.
? = 12 × 5 – 10 + 30 ? = 90 – 10 = 80 175. (a) The correct spelling is difficult.

Downloaded From : www.EasyEngineering.net

You might also like